Sunteți pe pagina 1din 2195

The Brigham Renal

Board Review Course


Provided by:
The Renal Division
Brigham and Womens Hospital and
The Department of Continuing Education
Harvard Medical School
Release Date
October 1, 2010
1-800-284-8433 www.cmeinfo.com
738
Oakstone

Medical Publishing, LLC


The Brigham Renal Board Review Course
Harvard Medical School
October 1, 2010
Topic/Speaker
B
o
o
k

P
a
g
e

#
T
o
t
a
l

D
u
r
a
t
i
o
n
D
V
D

D
i
s
c

#
C
D

D
i
s
c

#
C
D

T
r
a
c
k

#
ELECTROLYTES AND ACID BASE
Basic Physiology for Tackling Electrolyte/Acid Base Cases
Alan S.L. Yu, MD
1 1:17:37 1 1 1-17
Sodium Disorders
Mark L. Zeidel, MD
27 0:41:59 1 2 1-9
Potassium Disorders
Julian L. Seifter, MD
68 0:42:39 2 3 1-9
Disorders of Divalent Metabolism
David B. Mount, MD
98 0:38:20 2 4 1-8
Electrolytes and Acid Base Practice for the Boards I
Alan S.L. Yu, MD
133 1:28:52 2
5
6
1-14
1-5
Electrolytes and Acid Base Practice for the Boards II
Alan S.L. Yu, MD
218 0:41:04 3 6 6-14
Take-Home Messages
David B. Mount, MD
271 0:31:51 3 7 1-7
Pathology Refresher
Helmut G. Rennke, MD
309 1:39:27 4
8
9
1-15
1-6
Lupus Nephritis
Gerald B. Appel, MD
402 0:46:36 5 9 7-16
Nephrology Board Review 1 (Glomerulonephritis)
Gerald B. Appel, MD
429 0:56:11 5 10 1-12
Workshop: Challenging GN Cases for the Boards
Gerald B. Appel, MD
486 0:57:44 6 11 1-12
RPGN
David J. Salant, MD
523 0:43:00 6 12 1-9
Genetic Renal Disease: Overview
Martin A. Pollak, MD
549 0:42:26 6 13 1-9
Pediatric Kidney Disease: A Board Primer
Julie R. Ingelfinger, MD
584 0:43:35 7 14 1-9
Membranous and FSGS
Johannes S. Schlondorff, MD, PhD
620 0:50:07 7 15 1-11
Polycystic Kidney Disease
Theodore I. Steinman, MD
650 0:45:37 7 16 1-10
IgA Nephropathy
Gerald B. Appel, MD
685 0:39:23 7 17 1-8
Light Chain Deposition Disease and Amyloidosis
Laura M. Dember, MD
708 0:36:37 8 17 9-16
ELECTROLYTES AND ACID BASE
GLOMERULONEPHRITIS
Harvard Medical School
The Brigham Renal Board Review Course
October 1, 2010
Topic/Speaker
B
o
o
k

P
a
g
e

#
T
o
t
a
l

D
u
r
a
t
i
o
n
D
V
D

D
i
s
c

#
C
D

D
i
s
c

#
C
D

T
r
a
c
k

#
Pregnancy and Renal Disease
Robert A. Cohen, MD
744 0:35:17 8 18 1-7
Treating GN
Ajay K. Singh, MD
769 0:43:52 8 18 8-16
GN Board Review Cases
Dana Miskulin, MD
791 0:23:27 8 19 1-5
Take Home Messages
Ajay K. Singh, MD
812 0:40:52 8 19 6-14
Nephrology Board Review 2
Ajay K. Singh, MD
847 1:05:20 9 20 1-14
Kidney Stone Cases
Gary C. Curhan, MD, ScD
910 0:57:16 9 21 1-12
Hypertension: An Overview
John Forman, MD
939 0:33:31 9 22 1-7
Cardiovascular Disease and Renal Disease
David M. Charytan, MD, MSc
974 0:44:49 10 22 8-16
Renovascular Disease
Joseph M. Garasic, MD
998 0:44:09 10 23 1-9
AKI Epidemiology
Sushrut S. Waikar, MD
1038 0:28:38 10 23 10-15
Pathophysiology of AKI
Joseph V. Bonventre, MD, PhD
1055 0:42:20 11 24 1-9
CVVH: An Overview
Kenneth B. Christopher, MD
1082 0:35:48 11 24 10-17
AKI Syndromes
Sushrut S. Waikar, MD
1109 0:47:56 11 25 1-10
AKI and Cancer
Benjamin Humphreys, MD, PhD
1139 0:43:14 12 26 1-9
Nephrology Board Review 3
Bradley M. Denker, MD
1171 1:09:08 12 27 1-14
Take Home Messages
Sushrut S. Waikar, MD
1236 0:10:51 12 27 15-17
Nephrology Board Review 4
Daniel W. Coyne, MD
1247 1:16:51 13 28 1-16
CKD AND GENERAL NEPHROLOGY
ACUTE KIDNEY INJURY
Harvard Medical School
The Brigham Renal Board Review Course
October 1, 2010
Topic/Speaker
B
o
o
k

P
a
g
e

#
T
o
t
a
l

D
u
r
a
t
i
o
n
D
V
D

D
i
s
c

#
C
D

D
i
s
c

#
C
D

T
r
a
c
k

#
CKD (Cont'd) AND DIALYSIS
Dialysis: An Overview for the Boards
J. Kevin Tucker, MD
1311 0:44:34 13 29 1-10
Vascular Calcifications: Clinical Implications
Eduardo Slatopolsky, MD
1362 0:50:33 13 30 1-11
Iron in Anemia Management
Daniel Coyne, MD
1408 0:39:35 14 31 1-8
Dialysis Vascular Access
Dirk M. Hentschel, MD, PhD
1436 0:46:42 14 32 1-10
Peritoneal Dialysis: An Overview
Joanne M. Bargman, MD
1478 0:50:19 14 33 1-11
Dialysis Pearls for the Boards
J. Kevin Tucker, MD
1521 0:24:47 15 33 12-17
PD Complications
Joanne M. Bargman, MD
1552 0:52:06 15 34 1-11
Poisonings and Intoxications
J. Kevin Tucker, MD
1605 0:23:22 15 34 12-17
Epo in Anemia Management
Ajay K. Singh, MD
1644 0:48:17 15 35 1-10
Dialysis Complications
Bertrand Jaber, MD
1673 0:31:35 16 35 11-17
Dialysis Dose
J. Kevin Tucker, MD
1702 0:27:44 16 36 1-6
Take-Home Messages
J. Kevin Tucker, MD
1723 0:18:54 16 36 7-12
Nephrology Board Review 5
Steven M. Brunelli, MD
1755 1:18:08 16 37 1-16
Nephrology Board Review 6
David M. Charytan, MD, MSc
1805 1:02:27 17 38 1-13
Key Transplant Studies - What You Should Know for the
Boards
Anil Chandraker, MD
1850 0:43:02 17 39 1-10
Transplantation Immunobiology for the Boards
Nader Najafian, MD
1883 0:47:25 18 40 1-10
Transplant Immunosuppression for the Boards
Steven Gabardi, PharmD & Anil Chandraker, MD
1914 0:50:35 18 41 1-11
TRANSPLANTATION
CKD (Cont'd) AND DIALYSIS
Harvard Medical School
The Brigham Renal Board Review Course
October 1, 2010
Topic/Speaker
B
o
o
k

P
a
g
e

#
T
o
t
a
l

D
u
r
a
t
i
o
n
D
V
D

D
i
s
c

#
C
D

D
i
s
c

#
C
D

T
r
a
c
k

#
Tissue Typing a Boards Primer
Edgar L. Milford, MD
1955 0:45:37 18 42 1-10
Pre-Transplant Evaluation of Recipients
M. Javeed Ansari, MD
1975 0:26:06 19 42 11-16
Donor Evaluation: Living & Deceased
Didier Mandelbrot, MD
2006 0:28:11 19 43 1-6
Early Post-Transplant Medical & Surgical Complications:
Cases for the Boards
Nidyanandh Vadivel, MD & Sayeed Malek, MD
2025 0:42:23 19 43 7-15
Late Post-Transplant Medical Complications
John Vella, MD
2053 0:31:05 19 44 1-7
Infections in Immunocompromised Hosts
Sarah P. Hammond, MD
2080 0:43:21 19 44 8-17
Take-Home Messages in Transplantation
Anil Chandraker, MD
2113 0:23:41 19 45 1-5
Transplant Board Review
Martha Pavlakis, MD
2137 0:56:38 20 45 6-17
Basic Physiology for Tackling
Electrolyte/Acid Base Cases
Alan S. L. Yu, M.B., B. Chir.
University of Southern California Keck School of
Medicine
Financial disclosures
No conflict of interest to disclose.
Alan S. L. Yu, M.B., B. Chir.
Copyright Harvard Medical School, 2010. All Rights Reserved.
1
Outline
1. Volume regulation - Na
+
handling
2. Osmoregulation - Water handling
3. K
+
handling
4. Renal acid-base homeostasis
Outline
1. Volume regulation - Na
+
handling
2. Osmoregulation - Water handling
3. K
+
handling
4. Renal acid-base homeostasis
Copyright Harvard Medical School, 2010. All Rights Reserved.
2
Extracellular volume regulation
Extracellular volume is maintained by regulating the
rate of Na
+
and water excretion (primarily by regulating
Na
+
)
Na
+
is freely filtered at the glomerulus(25,000
mmol/day)
Na
+
excretion is regulated at the level of tubule
reabsorption
Na
+
reabsorption along the nephron
Proxi mal tubul e
60-70%
Thi ck ascendi ng l i mb
20-30%
Di stal convol uted tubul e
5-10%
Col l ecti ng duct
1-3%
Uri ne
1-2% ( =F
E
Na)
Copyright Harvard Medical School, 2010. All Rights Reserved.
3
Sites of action of natriuretics
Proxi mal tubul e
Osmotic diuretics
Carbonic
anhydrase
inhibitors
Thi ck ascendi ng l i mb
Loop diuretics
Di stal convol uted tubul e
Thiazide diuretics
Col l ecti ng duct
K
+
-sparing diuretics
Proximal tubule Na
+
absorption
Cl
-
H
2
O
Lumen Bl ood
AngII
+
Copyright Harvard Medical School, 2010. All Rights Reserved.
4
Carbonic anhydraseinhibitors
Cl
-
H
2
O
Thick ascending limb Na
+
absorption
NKCC2
Lumen Bl ood
Copyright Harvard Medical School, 2010. All Rights Reserved.
5
Loop diuretics: Mechanism of action
Bartters
syndrome
DCT Na
+
absorption
Na
+
Na
+
K
+
Cl
-
NCC
Lumen Bl ood
WNK4
-
Gitelmans
syndrome
PHAII
Gordons syndrome
Thiazide
diuretics
PHA =pseudohypoaldosteronism
Copyright Harvard Medical School, 2010. All Rights Reserved.
6
CCD Na
+
absorption
Arachi doni c
aci d
Angi otensi nogen JGA
PGE
2
Reni n
COX2
AngI AngII
ACE
AT1R
Corti sol
Corti sone
b-HSD
Tubule
flow
CCD Na
+
absorption
PHA I
PHA I
+
Liddles syndrome
Copyright Harvard Medical School, 2010. All Rights Reserved.
7
K
+
-sparing diuretics
Epl erenone
Regulation of ECF volume
ECV
AngII, aldosterone
ANP
Tubule Na
+
reabsorption
Copyright Harvard Medical School, 2010. All Rights Reserved.
8
Outline
1. Volume regulation - Na
+
handling
2. Osmoregulation - Water handling
3. K
+
handling
4. Renal acid-base homeostasis
Serum [Na
+
] is simply a surrogate marker for
serum osmolality
Regulation of osmolality
The body senses and regulates serum osmolality
(not serum Na
+
)
Copyright Harvard Medical School, 2010. All Rights Reserved.
9
Na
+
concentration =
Amount of Na
+
Volume of water
Serum osmolality is regulated by regulating
water balance (not Na
+
balance)
Regulation of osmolality
1400
Urinary concentration and dilution
290
290
290
140
Antidiuresis: 1150
Water diuresis: 65
NaCl
Water
N
a
C
l
N
a
C
l
Water Water
NaCl NaCl
ADH
Interstitial
osmotic gradient
290
Copyright Harvard Medical School, 2010. All Rights Reserved.
10
Mechanism of action of ADH
V2 receptor
cAMP
CD principal cells
Apical membrane
aquaporin-2
Osmotic water
reabsorption(equilibrate
with interstitium)
U
Osm
PGE
2
Aquaretic
NSAIDs
ADH
Regulation of serum osmolality
P
OSM
ADH
U
OSM
Thirst
Copyright Harvard Medical School, 2010. All Rights Reserved.
11
Expected U
Osm
(mOsm/kg)
Normal Hyponatremia Hypernatremia
50-1200 Low (<100) High (>800)
Non-osmolar stimuli to ADH secretion
U
OSM
Thirst
P
OSM
ADH
ECV
Copyright Harvard Medical School, 2010. All Rights Reserved.
12
Regulation of ADH secretion
Plasma osmolality
280
P
l
a
s
m
a

A
D
H
Blood volume depletion (%)
0
P
l
a
s
m
a

A
D
H
10 5 15 20
Outline
1. Volume regulation - Na
+
handling
2. Osmoregulation - Water handling
3. K
+
handling
4. Renal acid-base homeostasis
Copyright Harvard Medical School, 2010. All Rights Reserved.
13
Intracellular pool of K
+
acts as buffer
Na
+
K
+
K
+
[K
+
] 120 mEq/L
H
+
Acidosis
Hemolysis
Rhabdomyolysis
Tumor lysis
Digitalis
Insulin
Catechols (b2)
Diabetic ketoacidosis
bblocker
K
+
handling along the nephron
Proxi mal tubul e
100% K
+
reabsorbed
Corti cal
col l ecti ng duct
K
+
secretion
Copyright Harvard Medical School, 2010. All Rights Reserved.
14
CCD K
+
secretion
4 mmol/L 32 mmol/L [K
+
]:
Transtubular K gradient (TTKG) =32/4 =8
CCD K
+
secretion
Arachidonic
acid
Angiotensinogen JGA
PGE
2
Renin
COX2
AngI AngII
ACE
AT1R
Cortisol
Cortisone
b-HSD
Tubule
flow
K
+
Copyright Harvard Medical School, 2010. All Rights Reserved.
15
Regulation of CD K
+
secretion: summary
Distal tubule flow and NaCl delivery
(dependent on upstream Na reabsorption)
Mineralocorticoid receptor
Aldosterone
Renin-angiotensin system
Serum K
+
Glucocorticoids
Regulation of serum K
+
K+
CCD K
+
secretion
Aldosterone
Na-K-ATPase, ROMK
Copyright Harvard Medical School, 2010. All Rights Reserved.
16
Expected urine K
+
Hyperkalemia Hypokalemia
24 hr urine K
+
High (>40 mEq) Low (<25 mEq)
TTKG >6 <3
Outline
1. Volume regulation - Na
+
handling
2. Osmoregulation - Water handling
3. K
+
handling
4. Renal acid-base homeostasis
Copyright Harvard Medical School, 2010. All Rights Reserved.
17
Acid-Base homeostasis
1. Extracellular buffering by HCO
3
-
(immediate)
2. Respiratory regulation of pCO
2
(minutes)
3. Intracellular and bone buffering (hours)
4. Renal regulation of acid excretion (1-5 days)
Extracellular buffering of acid by the
CO
2
/HCO
3
buffer system
pH =6.10 +log
[HCO
3
-
]
0.03 P
CO
2
HCO
3
-
+ H
+
H
2
CO
3
CO
2
+ H
2
O
Copyright Harvard Medical School, 2010. All Rights Reserved.
18
Arterial pH P
CO2
and HCO
3
-
Primary disturbance
Acidemia HCO
3
-
Metabolic acidosis
P
CO2
Respiratory acidosis
Alkalemia HCO
3
-
Metabolic alkalosis
P
CO2
Respiratory alkalosis
Respiratory and renal compensation
Primary disorder Expected compensation
Metabolic acidosis Each 1 mEq/L HCO
3
-
1.2 mm Hg P
CO2
Metabolic alkalosis Each 1 mEq/L HCO
3
-
0.7 mm Hg P
CO2
Respiratory acidosis
Acute Each 1 mm Hg P
CO2
0.1 mEq/L HCO
3
-
Chronic Each 1 mm Hg P
CO2
0.3 mEq/L HCO
3
-
Respiratory alkalosis
Acute Each 1 mm Hg P
CO2
0.2 mEq/L HCO
3
-
Chronic Each 1 mm Hg P
CO2
0.4 mEq/L HCO
3
-
Copyright Harvard Medical School, 2010. All Rights Reserved.
19
Daily Acid Generation
(mostly dietary)
~1 mmol/kg
Net acid excretion ~1 mmol/kg
Glomerulus 4200 mmol HCO
3
-
filtered
Proximal tubule (& TALH) Reabsorb almost all
HCO
3
-
Collecting duct Secrete H
+
Titratable acidity (phosphate) 30%
NH
4
+
70% (can be )
Free H
+
10
-5
mol/L (@ pH 5)
Renal acid handling: Overview
Acid excretion mechanisms in renal tubule
Type 2 RTA
Type 1 RTA
Type 4 RTA
Copyright Harvard Medical School, 2010. All Rights Reserved.
20
Proximal tubule HCO
3
-
absorption
Cl
-
Lumen
Bl ood
AngII
+
ProxRTA
Carbonic anhydraseinhibitors
Cl
-
Copyright Harvard Medical School, 2010. All Rights Reserved.
21
CCD H
+
secretion
NH
3
NH
4
+
Distal
(Type I)
RTA
CCD HCO
3
-
secretion
NH
3
NH
4
+
H
+
HCO
3
-
Cl
-
Pendrin
H-ATPase
b-IC cell
Copyright Harvard Medical School, 2010. All Rights Reserved.
22
NH
3
NH
4
+
Arachidoni
c acid
Angiotensinogen JGA
PGE
2
Renin
COX2
AngI AngII
ACE
AT1R
Cortisol
Cortisone
b-
HSD
Tubule
flow
Regulation of extracellular pH
Blood pH
NH
4
+
/Urine pH
Net acid excretion
H-ATPase
NH
3
generation
Copyright Harvard Medical School, 2010. All Rights Reserved.
23
Urine anion gap is a surrogate marker of
urine ammonium
Measured cations measured anions =unmeasured anions unmeasured cations
Na
+
+K
+
- Cl
- e.g. NH
4
+
Urine anion gap
U
r
i
n
e

N
H
4
+
-40 0 -20 +20 +40
Expected urine acid excretion
Acidemia
Urine pH <5.5
Urine NH
4
+
>20 mEq/L
Urine anion gap <0
Copyright Harvard Medical School, 2010. All Rights Reserved.
24
Diarrhea
Proximal
RTA
Distal RTA
Type I Type 4
Serum K
+

Urine AG Negative Variable Positive
Urine pH Variable >5.5 <5.5
Other
Fanconi
syndrome
Nephro-
calcinosis
Clinical features of RTA
Regulation of acid-base handling
H
+
secretion
Extracellular pH
Distal tubule Na
+
delivery
AngII, aldosterone
ECV depletion (AngII, aldo)
Hypokalemia ( Intracellular pH in tubule cells
H
+
secretion)
HCO
3
-
secretion
Cl
-
depletion (inactvates pendrin)
Copyright Harvard Medical School, 2010. All Rights Reserved.
25
Suggested reading
Rennke, H.G., Denker, B.M., Renal Pathophysiology The
Essentials, 2nd Edition, Lippincott Williams & Wilkins, 2010
Eaton, D., Pooler, J ., Vander, A. Vander's Renal Physiology, 9th
Edition, McGraw-Hill Medical, 2009
Koeppen, B.M. and Stanton, B.A., Renal Physiology, 4th Edition,
Mosby, 2006
Financial disclosures
No conflict of interest to disclose.
Alan S. L. Yu, M.B., B. Chir.
Copyright Harvard Medical School, 2010. All Rights Reserved.
26
Disorders of Osmolality: From
Molecular Mechanisms to Clinical
Medicine
Mark L. Zeidel, M.D.
Herrman L. Blumgart Professor of
Medicine
Harvard Medical School
Chair, Department of Medicine
Physician-in-Chief
Beth Israel Deaconess Medical Center
Disorders of Osmolality: From
Molecular Mechanisms to Clinical
Medicine
Mark L. Zeidel, M.D.
Disclosures: None
Copyright Harvard Medical School, 2010. All Rights Reserved.
27
Each of these
creatures
survives in the
Galapagos by
highly effective
t f management of
salt and water
Disorders of Osmolality: From
Molecular Mechanisms to Clinical
Medicine Medicine
Scope of the problem.
Control of body fluid tonicity: Regulation of ADH
release.
Control of body fluid tonicity: Renal water excretion.
Disturbances of body fluid tonicity: Neural effects Disturbances of body fluid tonicity: Neural effects
Disturbances of body fluid tonicity: Hyponatremia.
Disturbances of body fluid tonicity: Hypernatremia.
Copyright Harvard Medical School, 2010. All Rights Reserved.
28
Case1
A 45 year old woman with a history of depression
presents wtih disorientation and inappropriate presents wtih disorientation and inappropriate
behavior. Her husband notes that she has had
nausea recently and has eaten little, but has
continued to take her amitryptaline.
PE reveals a BP of 125/70 and lethargy. She is oriented
to person only, exhibiting intact cranial nerves and
slurred speech slurred speech.
Na
+
108; Urine Na
+
is 28. Serum Osmolality is 221;
Urine osmolality is 685.
Case 2
A 67 year old man suffers a major MI, from which he
recovers. One month later he notes increased
edema d spnea on e ertion and 2 pillo edema, dyspnea on exertion, and 2 pillow
orthopnea.
PE reveals P = 120, BP = 100/65, RR = 22, afebrile.
Exam notable for distended neck veins, rales 1/3 up,
a palpable S3, and 2+ pitting edema to the mid calf.
Serum Na
+
125
Urine Na
+
is 2. Urine osmolality is 685; Serum
Osmolality is 262.
Copyright Harvard Medical School, 2010. All Rights Reserved.
29
Case #3
A 66 year old man with no significant prior illnesses
presents to the Emergency Ward complaining of
feeling out of sorts. History is notable for 50 pack
years of smoking. He was seen two weeks
previously in the outpatient clinic where a routine
chest X-ray was obtained. Physical exam is entirely
benign. reveals: BP = 105/70; P = 100; R = 18;
Afebrile Neck flat Chest clear Cardiac e am Afebrile. Neck vv flat, Chest clear, Cardiac exam
benign, Extr. No edema.
Na
+
188; K
+
4.2; Cl
-
153; HCO
3
-
25; S
osm
385; U
OSM
88.
CXR from 2 weeks ago: Solitary 2cm lung nodule.
Major Premise: A fundamental
understanding of transport understanding of transport
processes and their regulation
permits the physician to manage
fluid and electrolyte
disturbances in patients on the disturbances in patients on the
basis of mechanisms rather than
rote memorization.
Copyright Harvard Medical School, 2010. All Rights Reserved.
30
Disorders of Osmolality: From
Molecular Mechanisms to Clinical
Medicine Medicine
Scope of the problem.
Control of body fluid tonicity: Regulation of ADH
release.
Control of body fluid tonicity: Renal water excretion.
Disturbances of body fluid tonicity: Neural effects Disturbances of body fluid tonicity: Neural effects
Disturbances of body fluid tonicity: Hyponatremia.
Disturbances of body fluid tonicity: Hypernatremia.
Water Balance
P
Osm
= 285
P
Na
= 140
Copyright Harvard Medical School, 2010. All Rights Reserved.
31
Free water excretion: First morning
void
Compare urine osmolality with that of plasma (approx. Compare urine osmolality with that of plasma (approx.
300 mOsm/kg).
If you void 600 ml of urine with an osmolality of 800
mOsm/kg, you excreted 480 mOsm of solute (800
mOsm/kg X 0.6 kg). If you had excreted a urine of
300 mOsm/kg, you would have required 1.6 liters of
urine (480 mOsm/300 mOsm/kg) to excrete this urine (480 mOsm/300 mOsm/kg) to excrete this
solute load.
0.6 liters 1.6 liters = -1.0 liters
You therefore absorbed 1.0 liter of free water.
Free water excretion: Drinking during
the day
Compare urine osmolality with that of plasma (approx. Compare urine osmolality with that of plasma (approx.
300 mOsm/kg).
Assume you void 200 mOsm of solute in 2 liters of
urine. If you had excreted a urine of 300 mOsm/kg,
you would have required 0.67 liters of urine (200
mOsm/300 mOsm/kg) to excrete this solute load.
2 0 liters 0 67 liters = 1 33 liters 2.0 liters 0.67 liters = 1.33 liters
You therefore excreted 1.33 liters of free water.
Copyright Harvard Medical School, 2010. All Rights Reserved.
32
Copyright Harvard Medical School, 2010. All Rights Reserved.
33
Control of ADH Release
Control of ADH Release
Copyright Harvard Medical School, 2010. All Rights Reserved.
34
Hyperosmolality & ADH release
thirst
adapted from
Robertson et al, Robertson et al,
Am J Med 1982
Volume depletion & ADH release
adapted from Dunn et al. J Clin Invest, 1973
Copyright Harvard Medical School, 2010. All Rights Reserved.
35
Control of ADH Release
Regulation of ADH
Release
Hypertonicity
Volume Depletion
Beta-adrenergic stimulation
CNS dysfunction CNS dysfunction
Drugs/stress
Pulmonary diseases
Copyright Harvard Medical School, 2010. All Rights Reserved.
36
Actions of ADH
Renal water
Reabsorption of
water in water in
distal neprhon
Disorders of Osmolality: From
Molecular Mechanisms to Clinical
Medicine Medicine
Scope of the problem.
Control of body fluid tonicity: Regulation of ADH
release.
Control of body fluid tonicity: Renal water excretion.
Disturbances of body fluid tonicity: Neural effects Disturbances of body fluid tonicity: Neural effects
Disturbances of body fluid tonicity: Hyponatremia.
Disturbances of body fluid tonicity: Hypernatremia.
Copyright Harvard Medical School, 2010. All Rights Reserved.
37
Control of Body Fluid Tonicity:
Renal Water Excretion
Proximal tubule
Thin Descending Limb of Henle
Thin Ascending Limb of Henle
Thi k A di i b f H l Thick Ascending Limb of Henle
Cortical and Medullary Collecting Ducts
Nephron Structure
Copyright Harvard Medical School, 2010. All Rights Reserved.
38
No ADH, Excretion of a Dilute
Urine
High ADH, Excretion of a
Concentrated Urine
Copyright Harvard Medical School, 2010. All Rights Reserved.
39
Diffusion of water
through the lipid bilayer
Copyright Harvard Medical School, 2010. All Rights Reserved.
40
Water-tight barrier apical
membrane
Nephron Structure
Copyright Harvard Medical School, 2010. All Rights Reserved.
41
mTAL: Transport Function and
regulation by Ca
2+
mTAL: Cation-selective
paracellular pathway
Copyright Harvard Medical School, 2010. All Rights Reserved.
42
Anatomy of urinary concentrating
mechanism
Nephron Structure
Copyright Harvard Medical School, 2010. All Rights Reserved.
43
Cortical Collecting Duct:
Microanatomy
Cortical Collecting Duct:
Transport Function
Copyright Harvard Medical School, 2010. All Rights Reserved.
44
Transporters: Aquaporin I
Copyright Harvard Medical School, 2010. All Rights Reserved.
45
How did we figure out how
vasopressin works?
????
Copyright Harvard Medical School, 2010. All Rights Reserved.
46
+ AVP
No AVP
AQP2 localization
and Water Permeability
Copyright Harvard Medical School, 2010. All Rights Reserved.
47
Arrows
P i t t AQP2 Point to AQP2
ATP
AC
AVP
cAMP
Copyright Harvard Medical School, 2010. All Rights Reserved.
48
ATP
AC
AVP
cAMP
Nephron Structure
Copyright Harvard Medical School, 2010. All Rights Reserved.
49
Disorders of Osmolality: From
Molecular Mechanisms to Clinical
Medicine Medicine
Scope of the problem.
Control of body fluid tonicity: Regulation of ADH
release.
Control of body fluid tonicity: Renal water excretion.
Disturbances of body fluid tonicity: Neural effects Disturbances of body fluid tonicity: Neural effects
Disturbances of body fluid tonicity: Hyponatremia.
Disturbances of body fluid tonicity: Hypernatremia.
Neurological effects of altered
serum osmolality and its correction
Brain edema with increased intracranial
pressure.
Losses of volume in the brain with traction
on fixed structures.
Direct neuronal damage due to the g
disturbance.
Osmotic demyelination syndrome following
overly rapid correction of the disturbance.
Copyright Harvard Medical School, 2010. All Rights Reserved.
50
Why are neurons
so sensitive to
osmotic damage?
Volume Regulation in Neurons
Copyright Harvard Medical School, 2010. All Rights Reserved.
51
Copyright Harvard Medical School, 2010. All Rights Reserved.
52
Osmotic Demyelinating Syndrome
AKA central pontine myelinosis
Follows overly rapid and excessive Follows overly rapid and excessive
correction of severe hyponatremia.
May involve the pons, subcortical
regions, corpus callosum, and basal
ganglia. g g
May be diagnosed on MRI.
Susceptability of individual patients to
ODS may vary widely.
Osmotic Demylination
Copyright Harvard Medical School, 2010. All Rights Reserved.
53
Disorders of Osmolality: From
Molecular Mechanisms to Clinical
Medicine Medicine
Scope of the problem.
Control of body fluid tonicity: Regulation of ADH
release.
Control of body fluid tonicity: Renal water excretion.
Disturbances of body fluid tonicity: Neural effects Disturbances of body fluid tonicity: Neural effects
Disturbances of body fluid tonicity: Hyponatremia.
Disturbances of body fluid tonicity: Hypernatremia.
Case1
A 45 year old woman with a history of depression
presents wtih disorientation and inappropriate presents wtih disorientation and inappropriate
behavior. Her husband notes that she has had
nausea recently and has eaten little, but has
continued to take her amitryptaline.
PE reveals a BP of 125/70 and lethargy. She is oriented
to person only, exhibiting intact cranial nerves and
slurred speech slurred speech.
Na
+
108; Urine Na
+
is 28. Serum Osmolality is 221;
Urine osmolality is 685.
Copyright Harvard Medical School, 2010. All Rights Reserved.
54
Factors determining the rate of
correction of hyponatremia
What are the risks of uncorrected
hyponatremia?
Wh t th i k f th ti What are the risks of the correction
itself?
Copyright Harvard Medical School, 2010. All Rights Reserved.
55
Rapid Correction of Hyponatremia
For patients with acute hyponatremia
ith l i fi di with neurologic findings.
For volume depletion: normal saline
For SIADH: Use hypertonic saline (3%
rather than 0.9%).
Optimal rate of correction is 0 5 1 Optimal rate of correction is 0.5 - 1
mEq/h, < 12 mEq/D, or until a safe
level is achieved and/or neurological
symptoms improve.
Case 2
A 67 year old man suffers a major MI, from which he
recovers. One month later he notes increased
edema, dyspnea on exertion, and 2 pillow orthopnea.
PE reveals P = 120, BP = 100/65, RR = 22, afebrile.
Exam notable for distended neck veins, rales 1/3 up,
a palpable S3, and 2+ pitting edema to the mid calf.
Serum Na
+
125
Urine Na
+
is 2. Urine osmolality is 685; Serum U e a s U e os o a ty s 685; Se u
Osmolality is 262.
Copyright Harvard Medical School, 2010. All Rights Reserved.
56
Therapy of Chronic Hyponatremia
Congestive Heart Failure Congestive Heart Failure
Cirrhosis
Spinal Cord Injured Patients
SIADH
Studies show that low serum sodium
predicts poor prognosis.
Pharmacologic Treatment of SIADH:
Lithium and Demeclocycline
Pharmacologic Treatment of SIADH:
Lithium and Demeclocycline
ATP
AVP
Demeclocycline
Lithium
AC
Copyright Harvard Medical School, 2010. All Rights Reserved.
57
Pharmacologic Treatment of SIADH:
Vasopressin Receptor Antagonists
Pharmacologic Treatment of SIADH:
Vasopressin Receptor Antagonists
AC
AVP
Tolvaptan Trials: SALT1 and
SALT2, Schrier et al., NEJ M
2006
Euvolemic or Hypervolemic
Hyponatremia, Tolvaptan
dose 15 mg to 30 mg to 60
mg depending on serum
sodium.
Copyright Harvard Medical School, 2010. All Rights Reserved.
58
Copyright Harvard Medical School, 2010. All Rights Reserved.
59
Disorders of Osmolality: From
Molecular Mechanisms to Clinical
Medicine Medicine
Scope of the problem.
Control of body fluid tonicity: Regulation of ADH
release.
Control of body fluid tonicity: Renal water excretion.
Disturbances of body fluid tonicity: Neural effects Disturbances of body fluid tonicity: Neural effects
Disturbances of body fluid tonicity: Hyponatremia.
Disturbances of body fluid tonicity: Hypernatremia.
Copyright Harvard Medical School, 2010. All Rights Reserved.
60
Case #3
A 66 year old man with no significant prior illnesses y g p
presents to the Emergency Ward complaining of
feeling out of sorts. History is notable for 50 pack
years of smoking. He was seen two weeks
previously in the outpatient clinic where a routine
chest X-ray was obtained. Physical exam is entirely
benign. reveals: BP = 105/70; P = 100; R = 18;
Afebrile. Neck vv flat, Chest clear, Cardiac exam
benign, Extr. No edema.
Na
+
188; K
+
4.2; Cl
-
153; HCO
3
-
25; S
osm
385; U
OSM
88.
CXR from 2 weeks ago: Solitary 2cm lung nodule.
Hypernatremia: Case 2
Causes for hypernatremia in this patient:
1. Loss of secretion of ADH (despite a
strong osmotic stimulus) indicating
central diabetes insipidus.
2. Lack of thirst in response to a strong
osmotic stimulus.
Copyright Harvard Medical School, 2010. All Rights Reserved.
61
Hypernatremia: Symptoms
Muscle weakness Muscle weakness
Fasciculations
Seizures
Lethargy
Altered mental status Altered mental status
Coma
Control of ADH Release
Copyright Harvard Medical School, 2010. All Rights Reserved.
62
No ADH, Excretion of a Dilute Urine
Hypernatremia: CNS Adaptation Hypernatremia: CNS Adaptation
H
2
O
Hypertonicity
H
2
O
O l t
Rapid Adaptation
El t l t
Chronic Adaptation
Osmolytes
Electrolytes
Electrolytes
H
2
O
Copyright Harvard Medical School, 2010. All Rights Reserved.
63
Hypernatremia: CNS Pathology Hypernatremia: CNS Pathology
Vascular injury due to brain shrinkage Vascular injury due to brain shrinkage
Subdural hematoma
Petechial hemorrhages in cortex and
subcortical white matter
Osmotic cell injury
Myelinolysis
Treatment of Hypernatremia Treatment of Hypernatremia
Restore intravascular volume
Replace free-water deficits
Replace 50% of deficit over 24 hours
Replace remaining 50% of deficit over the
next 24 to 48 hours
Replace ongoing renal and extra-renal Replace ongoing renal and extra-renal
fluid losses
Treat underlying causes of fluid losses
Copyright Harvard Medical School, 2010. All Rights Reserved.
64
Hypernatremia in Hospitalized Patients:
Outcome
Hypernatremia in Hospitalized Patients:
Outcome
All Patients Appropriate
Therapy
Delayed
Therapy
(n=103)
py
(n=50)
py
(n=53)
Overall Mortality 41 % 32 % 49 %
Mortality related to
hypernatremia
*
16 % 8 % 25 %
Decreased functional status
related to hypernatremia
14 % 12 % 15 %
*
p <0.05; Appropriate Therapy v Delayed Therapy
Palevsky PM, et al, Ann Intern Med 1996; 124:197-203
Disorders of Osmolality: From
Molecular Mechanisms to Clinical
Medicine Medicine
Scope of the problem.
Control of body fluid tonicity: Regulation of ADH
release.
Control of body fluid tonicity: Renal water excretion.
Disturbances of body fluid tonicity: Neural effects Disturbances of body fluid tonicity: Neural effects
Disturbances of body fluid tonicity: Hyponatremia.
Disturbances of body fluid tonicity: Hypernatremia.
Copyright Harvard Medical School, 2010. All Rights Reserved.
65
A 47 year old high thoracic cord quadraplegic man
presents with 3 days of increasing confusion and
obtundation. Serum chemistries reveal: Na
+
= 118,
K
+
= 3.6, Cl
-
= 85, HCO
3
-
= 24, BUN = 7, S
Osm
= 232,
U 546 H h ld th h t i b t t d? U
Osm
= 546. How should the hyponatremia be treated?
A. Normal saline with recheck of electrolytes in the morning.
B. Hypertonic saline dosed according to Adrogue and Madias
formula, with recheck of electrolytes in the morning.
C. Normal saline with furosemide with check of serum
electrolytes and urine osmolality every 2 3 hours. y y y
D. Normal saline with furosemide or hypertonic saline with
check of serum electrolytes and urine osmolality every 2 3
hours. Limit correction rate to 0.5 - 1 MEq/kg/hr; < 12 mEq/D.
A 67 year old reformed alcoholic with chronic severe
cirrhosis has a serum sodium of 122 125 over several
months of care. He has chronic ascites, and edema,
partially controlled with spironolactone and
furosemide. He complains of chronic thirst. Which of
the following statements is not true?
A. Low serum sodium predicts adverse outcome in chronic
cirrhosis.
B. Chronic administration of tolvaptan has been shown to raise
serum sodium in patients with chronic cirrhosis, with an
acceptable profile of side effects.
C. Chronic administration of tolvaptan has been shown to raise
di i ti t ith h i i h i ith serum sodium in patients with chronic cirrhosis, with an
acceptable profile of side effects, and has been shown to
reduce mortality.
D. Chronic administration of tolvaptan raises serum sodium to
normal or near normal levels equally effectively in patients
with severe or mild hyponatremia.
Copyright Harvard Medical School, 2010. All Rights Reserved.
66
Disorders of Osmolality: From
Molecular Mechanisms to Clinical
Medicine
Mark L. Zeidel, M.D.
Disclosures: None
Copyright Harvard Medical School, 2010. All Rights Reserved.
67
A Physiological Approach to
Potassium Disorders
Julian L. Seifter, MD
Overview of the relevant physiology
New developments in regulated K
+
secretion by
the distal nephron
Renal and adrenal RAS and K
+
homeostasis
Hyper/hypokalemia
Urinary indices and other diagnostic tests
Clinical consequences
Treatment of both disorders
DD
x
of both disorders
Disclosures
None
Copyright Harvard Medical School, 2010. All Rights Reserved.
68
Giebisch, AJP-Renal, 274, 1998
Factors Affecting K
+
Shift
Factor Transmembrane K
+

Shift
Insulin uptake
-catecholamine uptake
-catecholamine uptake
Acidosis uptake
Alkalosis uptake
Hyperosmolarity efflux

Copyright Harvard Medical School, 2010. All Rights Reserved.
69
Giebisch, AJP-Renal, 274, 1998
Na
+
, K
+
and H
2
O
Transport in
Principal Cells
ENaC epithelial Na channel
ROMK secretory K channel
Maxi-K flow-activated K
channel
Copyright Harvard Medical School, 2010. All Rights Reserved.
70
K
+
Secretion is Proportional to Distal Flow
Animals
on
different
K
+
diets
Giebisch, AJP-Renal, 274, 1998
K
+
Excretion is Dependent on Na
+
Intake
Young et al, AJP-Renal, 246, 1984
Copyright Harvard Medical School, 2010. All Rights Reserved.
71
K
+
Excretion as a Function of Plasma K
+
and Circulating Aldosterone
Adrenalectomized
with different
levels of aldo
replacement
Young et al, AJP-Renal, 255, 1988
Aldo-Dependent and AldoIndependent
Regulation of K
+
Excretion
Wang and Giebisch, Eur J Physiol, 2009
TAKE-HOME:
K
+
channels are
mostly regulated
by K
+
intake,
whereas
aldosterone
mostly regulates
ENaC, and thus
the driving force
for K
+
excretion.
Copyright Harvard Medical School, 2010. All Rights Reserved.
72
Transtubular K
+
Gradient (TTKG)
Useful to assess the renal response to or
serum K+
TTKG =[urine K+(urine Osm/ Plasma Osm)]
Plasma K+
Should be >7 during hyperkalemia
Should be <3 during hypokalemia
Urine Na
+
needs to be >20-25 mEqu/L; if <20-
25, distal Na
+
delivery and distal flow rate may
be limiting
Causes of Hyperkalemia
Increased intake
K
+
supplements, diet, transfusions, iatrogenic
Decreased renal excretion
Renal disease, particularly with type IV RTA
DRUGS
Adrenal insufficiency not universal
Intra extracellular shifts
Hyperosmolarity
Insulinopenia
Metabolic acidemia
DRUGS
Artifactual
in vitro hemolysis, leukocytosis, thrombocytosis
pseudohyperkalemia
Copyright Harvard Medical School, 2010. All Rights Reserved.
73
Take A Dietary History!
Highest content (>25 mmol/100 g)
Dried figs, molasses, seaweed
Very high (>12.5 mmol/100 g)
Dried fruits, nuts, avocados, bran cereals,
wheat germ, lima beans
High content (>6.2 mmol/100 g)
Vegetables: spinach, tomatoes, broccoli,
beets, carrots, potatoes
Fruits: bananas, cantaloupe, oranges,
mangos, kiwis
Meats: ground beef, steak, pork, veal, lamb
Gennari, NEJM, 1998
Hyporeninemic Hypoaldosteronism
Hyperchloremic acidosis in ~50%, with
urine pH classically < 5.5
Hyperkalemia
Plasma renin activity (PRA) and aldo
Response of PRA to stimuli such as
furosemideand captopril
Commonly with age and GFR,
classically in diabetics
Often hypertensive, with clinical ECFV
Copyright Harvard Medical School, 2010. All Rights Reserved.
74
Causes of Hyporeninemic
Hypoaldosteronism
Diabetic nephropathy
Acute GN, i.e. nephritic syndrome
Tubulointerstitial nephropathies, eg. Sickle
cell disease
Drugs, e.g. NSAIDS, cyclosporin, FK-506
Hereditary causes, e.g.
pseudohypoaldosteronismtype II
The Juxtaglomerular Apparatus,
Intra-Renal Source of Renin
MD, macula densa
AA, afferent arteriole
EA, efferent arteriole
GC: granular cell
SMC: smooth muscle cell
Schnermann & Briggs, JCI, 1999
Copyright Harvard Medical School, 2010. All Rights Reserved.
75
ANP Blunts the
Aldo from K
+
Healthy young
subjects, infused
With K +/- ANP
Clark et al, J Am Soc Nephrol. 1995 Nov;6(5):1459-62
Drugs and the RAS
Copyright Harvard Medical School, 2010. All Rights Reserved.
76
TAKE HOME MESSAGES:
The Renin-Angiotensin-Aldosterone Axis
ANP, systemic and local RAS, and prostaglandins all
affect renal renin release AND adrenal aldosterone
release, i.e. remember the adrenal effect
The role in hyporeninemic hypoaldosteronismof
volume expansion and ANP/BNP
renal renin and adrenal aldosterone release
The evolving risk of hyperkalemia from combining
multiple RAS inhibitors, e.g. MLR antagonist and
ACE-I in CHF
Question #1
You are referred a 17 year-old high school student for
management of high blood pressure. He has not seen a
physician since childhood, is on no medications.
He denies drug abuse, including cocaine.
FH: His 50 year-old father is also hypertensive, with a
history of renal stones.
Since you have access to a clinical research center, you
admit the father and son for biochemical profiling while
ingesting a diet with rigorously controlled salt content.
Copyright Harvard Medical School, 2010. All Rights Reserved.
77
Parameter Son

Father
Dietary Na
+

(mmol/day)
200 10 200 10
BP 150/90 110/64 142/90 110/70
K
+
6.0 4.5 5.6 4.6
Cl
-
119 102 114 102
HCO
3
-
18 25 21 27
pH 7.33 7.41 7.36 7.38
PRA 0.2 6.3 0.4 2.6
Aldo 15 61 13 41
ANP 48 9 32 14
FE
K
(%)
basal
7.8 10.3 8.5 7.8
FE
K
(%)
saline
8.1 33.8 8.2 15.0

A. Aggressive K restriction
B. NaCl restriction to 10 mEqu/day
C. Nifedipine
D. Amiloride
E. Hydrochlorothiazide
Which of the following is the most
appropriate therapy for this patient?
Copyright Harvard Medical School, 2010. All Rights Reserved.
78
Pseudohypoaldosteronism Type II (PHA-II)
Also known as Gordons syndrome or the
chloride-shunt disorder
The mirror image of Gitelmans syndrome:
- hypertension
- hyperkalemic acidosis
- suppression of plasma renin, aldosterone
- hypercalciuria, nephrolithiasis
Responsive to thiazides
Autosomal dominant transmission; three
different genes, two characterized (WNK1/4)
An evolving BOARDS favorite.
Copyright Harvard Medical School, 2010. All Rights Reserved.
79
WNK1 and WNK4 are Homologous
Serine/Threonine Kinases
WNK1 is Expressed in the DCT and CCD
B) WNK1 (red) and Aqp-2 (green)
D) WNK1 (red) and NCC (green)
Copyright Harvard Medical School, 2010. All Rights Reserved.
80
WNK4 is Expressed in the DCT and CCD
B) Co-expression of WNK4 (red) and Aqp-2 (green)
D) Co-expression of WNK1 (red) and NCC (green)
PHA-II Mutations in the WNK4 Kinase
Abrogate Its Inhibition of the Thiazide-
Sensitive Na-Cl Cotransporter
NCCT Na-Cl cotransporter
Choate et al, PNAS, 100, 2003
Copyright Harvard Medical School, 2010. All Rights Reserved.
81
WNK-Dependent Signaling;
A molecular switch controlling NaCl and K
excretion
Welling et al, Kidney Int, 2010 green arrows activating
red blunt end inhibiting
Hypokalemia - Causes
Pseudohypokalemia leukocytosis, with uptake
of K
+
by WBCs, e.g. in AML
Redistribution
InsulinopeniaDKA
Sympathomimetics

2
-agonists, dopamine, theophyline
Hypokalemic periodic paralysis, incl. thyrotoxic
Acute anabolic state pernicious anemia
Non-renal loss skin, stomach (suctioning),
intestine (diarrhea, laxatives)
Copyright Harvard Medical School, 2010. All Rights Reserved.
82
Renal Loss and Hypokalemia
Drugs
Diuretics
Antibiotics
Non-reabsorbable anions,
e.g. Penicillin
Aldosterone excess
Bicarbonaturia
Magnesium deficiency
inhibition of muscle Na/K-
ATPaseand Mg
2+
-
dependent block of ROMK
distal K
+
excretion
Tubular damage
ATN
Cisplatin,
aminoglycosides,
amphotericin
Intrinsic renal
transport defects
Liddles syndrome
Bartters syndrome
Gitelmans syndrome
Hereditary dRTA
Consequences of Hyperkalemia
Excitable tissue change in resting
membrane potential
Cardiac, decreased myocardial conduction
velocity, PR and QRS and increased rate of
repolarization (T wave changes)
Skeletal muscle weakness, fatigue,
paralysis
Kidney decreased ability to secrete NH
4
+
acidosis
Copyright Harvard Medical School, 2010. All Rights Reserved.
83
Typical Electrocardiographic Features of
Hyperkalemia
Serum K+ Major change
5.5-6.5 Tall peaked T waves
6.5-7.5 Loss of P waves
7.0-8.0 Widening of QRS
8.0-10 Sine wave,
ventricular arrhythmia,
asystole

Caveats: ECGs and Hyperkalemia
Remember the first symptom of
hyperkalemiais death..
ECG changes are not sensitive, particularly in
ESRD
Peaked Ts in other disorders
Atypical ECGs
Complete heart block
Intraventricular conduction delays
QRS axis shift
Brugada sign - pseudo-RBBB and coved ST
Copyright Harvard Medical School, 2010. All Rights Reserved.
84
Treatment of Hyperkalemia
Mechanism Therapy Dose Onset Duration
Stabilize membrane
potential
Calcium 10% Ca-gluconate,
10 ml over 10 min.
1-3 min. 30-60
min
Cellular K
+
uptake Insulin


2-agonist

10 U R with 50 ml
of D50, if BS<250

nebulized albuterol,
10 mg
30 min.


30 min.
4-6 h


2-4 h

K
+
removal Kayexalate


Hemodialysis
30-60 g PO

?


Immediate
?

Bicarb
Doesnt
Work!
Blumberg et al, Am J Med, 85, 1988
Copyright Harvard Medical School, 2010. All Rights Reserved.
85
Insulin and Glucose
Recommended dose is 10 units of regular
insulin followed by 50 g of 50% glucose
Followed by 10% dextrose infusion at a rate of
50-75 ml/hour (to prevent hypoglycemia)
In hyperglycemic patients (glucose >200-250
mg/dl) insulin alone is enough
D50W alone should be avoided
hyperosmolality can increase K
+
, primarily in
predisposed patients (e.g. DM with type IV RTA)

2
-Adrenergic Agonists (Inhaled)
10-20 mg of nebulized albuterol in 4 ml of
normal saline, inhaled over 10 minutes
Hypokalemic effect starts in 30 minutes,
peaks at 90 minutes and lasts for 2-6
hours
Reduces K
+
level by 0.5-1.0 mmol/L
Synergistic with insulin, but ineffective
as the sole agent in ESRD
Copyright Harvard Medical School, 2010. All Rights Reserved.
86
Kayexalate/SPS Complications
Ischemic colitis and colonic necrosis
- risk in enema form
- often fatal
- risk with sorbitol - but can occur without sorbitol
and is associated with intestinal SPS crystals
- post-transplant and post-op patients at risk
Volume overload
Reduction in serum calcium
Iatrogenic hypokalemia
Caveats/Concerns re
Kayexalate/SPS
Slow onset of effect ? makes SPS unnecessary
and inappropriate in most patients with acute
hyperkalemia.
Intestinal necrosis due to SPS in sorbitol is often a
fatal complication, NOT restricted to post-op setting.
New FDA advisory September, 2009 do NOT
administer SPS with sorbitol.
Yet SPS with sorbitol remains a very popular reflex
mechanism of therapy for hyperkalemia, often the
only formulation of SPS available.
Copyright Harvard Medical School, 2010. All Rights Reserved.
87
Hemodialysis
Serum K
+
level reaches a nadir at ~3
hours, but potassium removal continues to
the end of the session
The amount of K
+
removed depends on:
- type and surface area of the dialyzer
- blood flow rate
- dialysate flow rate
- dialysis duration
- serum:dialysate K
+
gradient
The Serum - Dialysate Gradient
Dialysates with lower K
+
concentration are more
effective, but may lead to rebound hypertension
Dialysates with very low K
+
concentration (0 or 1
mmol/L) should be used cautiously, given the risk
of arrhythmia
Graded reduction in K
+
concentration is effective,
with less arrhythmia, and is the standard of care
at BWH
Continuous cardiac monitoring is recommended
when using very low K
+
concentration dialysates
Copyright Harvard Medical School, 2010. All Rights Reserved.
88
Consequences of Hypokalemia
Arrhythmias
Muscles weakness,
paralysis, myopathy
Metabolic alkalosis
Insulin resistance
HYPERTENSION
Polydipsia, polyuria,
nephrogenic DI
Structural renal
disease AKI, ESRD
Predisposition to
Rhabdomyolysis
Hepatic encephalopathy
Treatment of Hypokalemia
First replete magnesium
Usually oral therapy, preferably K-Cl
IV can be given safely at 10 mEqu/hour,
but up to 40-60 mEqu/hr in a monitored
setting need central line, preferably
femoral
DO NOT USE DEXTROSE SOLUTIONS
acute in K
+
, due to the induced insulin
release
Copyright Harvard Medical School, 2010. All Rights Reserved.
89
Question #2
32 yo Hispanic man admitted to Brigham and
Women's Hospital with weakness and a K of 2.0
HPI: The patient was very healthy until 2 months
PTA, when he developed leg weakness. This
weakness fluctuated, and was more severe at
night-time. He denies drug abuse, laxative
abuse, is on no medications.
ROS: no nausea, no vomiting or diarrhea.
SH: Taxi driver, married with one child
FH: 10 siblings, mother has DM, one sister has
thyroid disease.
Physical Exam
Temp 97.2 bp 176/96 HR 102, RR 16
HEENT normal
J VP visible and not elevated, good peripheral
pulses, no edema
S1, S2 normal, no murmurs
Abdomen soft, non-tender, no organomegaly
Neuro decreased DTRs, otherwise normal
Copyright Harvard Medical School, 2010. All Rights Reserved.
90
Admission: 5 months PTA:
Na 139 143
K 2.0 3.8
Cl 105 107
HCO
3
-
26 29
BUN 11 16
Creat 0.6 1.0
Glu 145 136
PO4 1.2
Ca 8.8 8.8
Mg 1.3 1.9
Alb 3.8
Posm 290 TTKG =2.0
UOsm 590
UK 10
A. TSH
B. Genetic sequence of the gene encoding
the Na/K-ATPasealpha-1 subunit
C. Genetic sequence of the gene encoding
a muscle-specific K
+
channel
D. A & C
E. A, B & C
Which of the following is likely to be
abnormal in this patient?
Copyright Harvard Medical School, 2010. All Rights Reserved.
91
Question #3
67 year-old man with peri-operative hyperkalemia
PMH inclusive of hypertension, hyperlipidemia, and chronic
renal failure (baseline creatinine3.8 mg/dL).
Patient had been admitted for elective 3-vessel CABG.
Admission medications included:
Lasix60 mg bid calcitriol 0.5 g three times per week
Lisinopril 10 mg OD calcium carbonate 500 mg PO tid
Isordil 30 mg tid EPO 2000 units per week
Amlodipine 10 mg OD lovostatin 20 mg PO OD
The patient underwent CABG without any
intraoperative events (other than hyperkalemia).
Bypass time: 1 hour 39 minutes
Cross-clamp time: 65 minutes
The patient received:
two units of packed RBCs
1200 ml of D5W/NS
three 10 gm boluses of epsilon-aminocaproicacid
Urine output during surgery was 376 ml, the
patient urinated 475 ml in the six hour
postoperative period.
Copyright Harvard Medical School, 2010. All Rights Reserved.
92
Development of Hyperkalemia
Na
+
K
+

Pre-op 134 4.9

Intra-op
30 min 131 5.4
60 min
90 min 6.2
140 min 6.7
(insulin/D50)
180 min 132 6.6



Post-op
0 min 6.2
90 min 5.4
4 hours 5.1
Day 1 135 4.6
Day 2 4.2

A. Transfusion with older-than-ideal pRBCs
B. Lisinopril
C. Epsilon-aminocaproic acid
D. Transient hemolysis due to cardiac
bypass
E. Efflux of K
+
due to cardiac bypass
What was the likely cause of this
patients hyperkalemia?
Copyright Harvard Medical School, 2010. All Rights Reserved.
93
A 66 year-old man was admitted to hospital with a plasma K
+
concentration of 1.7 mEqu/L and profound weakness. The patient
had noted progressive weakness over several days, to the point that
he was unable to rise from bed. The past medical history was
notable for small cell lung cancer with metastases to brain, liver, and
adrenals. The patient had been treated with one cycle of
cisplatin/etoposideone year prior this admission, complicated by
acute kidney injury (peak creatinineof 5, with residual chronic
kidney disease) and three subsequent cycles of
cyclophosphamide/doxorubicin/vincristine, in addition to fifteen
treatments with whole brain radiation.
On physical examination the patient was jaundiced. BP was 130/70,
increasing to 160/98 after a liter of saline, with a J VP at 8 cm. There
was generalized muscle weakness.
Question #4
Laboratory Data: 2 Months PTA: Admission: HD2: Units
Sodium 143 149 144 mEq/L
Potassium 3.7 1.7 3.5 mEq/L
Chloride 103 84 96 mEq/L
Bicarbonate 26 44 34 mEq/L
Venous pH 7.47 pH
Venous pCO2 62 mm Hg
BUN 21 41 40 mg/dL
Creatinine 2.8 2.9 2.3 mg/dL
Magnesium 1.3 1.6 2.4 mg/dL
ALT 8 75 Units/L
Albumin 3.4 2.8 2.3
Adjusted anion gap 15 24 18
Tot bilirubin 0.65 5.19 mg/dL
Urine sodium 35 28 mEq/L
Urine Potassium 25 49 mEq/L
Urine Chloride 48 51 mEq/L
Urine Osmolality 391 mOsm/kg
Plasma Osmolality 312 mOsm/kg
TTKG 12
Copyright Harvard Medical School, 2010. All Rights Reserved.
94
A. ACTH
B. Circulating aldosterone level
C. Circulating cortisol level
D. A & C
E. A, B & C
Which of the following is likely to be
elevated in this patient?
Why is he hypokalemic?
Why is he hypertensive?
What is the underlying
pathophysiology?
How should he be treated?
Explain his acid-base picture.
Copyright Harvard Medical School, 2010. All Rights Reserved.
95
Syndrome of Apparent Mineralocorticoid
Excess (SAME)
Low-reninhypertension with hypokalemic alkalosis,
subnormal aldosterone
DDxis Liddles syndrome; Liddles R
x
by amiloride, but
not spironolactone, which treats SAME
Exacerbated by ACTH, suppressed by dexamethasone
Also associated with nephrocalcinosis
Decrease in peripheral conversion of cortisol to inactive
cortisonedefect in 11-hydroxysteroid
dehydrogenase-2 (11-HSD2)
Mimicked by licorice and other inhibitors of 11-HSD2
K
K
Cl
Na
NCC
+
+
+
+
+
+
Aldo
ML Receptor
Cortisol
Cortisone
11BHSD2
ENaC
ROMK
Cl
Aldosterone, Cortisol, and Distal Ion Transport
Copyright Harvard Medical School, 2010. All Rights Reserved.
96
Disclosures
None
Copyright Harvard Medical School, 2010. All Rights Reserved.
97
David B. Mount, MD
Renal Division, BWH
Renal Division,
VA Boston Healthcare System
Calcium and Magnesium
Disorders
Disclosures
None relevant to calcium and
magnesium disorders
Copyright Harvard Medical School, 2010. All Rights Reserved.
98
Calcium and Magnesium Homeostasis
The role of the calcium receptor in calcium
homeostasis and renal pathophysiology
Molecular physiology and pathophysiology
of vitamin D
Calcium and magnesium transport in the
kidney and intestine
The role of FGF23 in controlling serum
phosphate and calcium
Disorders of serum magnesium and
calcium
Calcium and Phosphate Balance
Regulated absorption (intestine), resorption or
incorporation (bone), and reabsorption or
excretion (kidney)
Bone a major reservoir
hydroxyapatite: Ca
10
(PO4)
6
(OH)
2
Serum calcium: 8.5-10.2 mg/dL (women), 9.0-
10.9 mg/dL (men)
~40% albumin-bound, 15% citrate/PO
4
/SO
4
-bound,
~45% ionized (free) Ca
2+
Ionized calcium: 1.12-1.23 mM
Serum phosphorus: 3.0-4.5 mg/dL
Copyright Harvard Medical School, 2010. All Rights Reserved.
99
Key Hormonal Mediators
Ca
2+
itself Calcium-sensing receptor
Parathyroid hormone, PTH secreted by
the parathyroid gland
Vitamin D, specifically 1,25-(OH)
2
D
3
(calcitriol)
Fibroblast growth factor-23 (FGF23)
Complex inter-relationships
e.g., FGF23 calcitriol,
calcitriol FGF23
The Role of PTH in Defending Serum Ca
2+
Copyright Harvard Medical School, 2010. All Rights Reserved.
100
Regulation of PTH Secretion by the
Parathyroid Calcium-Sensing Receptor
(CaSR)
Human Disorders Due to Loss-of-Function
in the Calcium Sensing Receptor (CaSR)
Familial benign (hypocalciuric) hypercalcemia (FHH)
Lifelong hypercalcemia, autosomal dominant inheritance
Normal or PTH, inappropriately urinary calcium excretion
Hypocalciuriapersists after parathyroidectomy
Can have chondrocalcinosis, pancreatitis, but generally
asymptomatic
Rightward shift in set point of PTH response to Ca
2+
Neonatal severe hyperparathyroidism
Severe neonatal hypercalcemia, failure to thrive,
hypophosphatemia, bony undermineralization, multiple fractures
and rib cage deformity
Autosomal recessive, i.e. bi-allelic loss-of-function in CaSR
Copyright Harvard Medical School, 2010. All Rights Reserved.
101
Structure of the Calcium Receptor
Gain-of-Function Mutations in the CaSR
Autosomal dominant hypocalcemia with
hypercalciuria
Hypocalcemia with associated hypomagnesemia,
PTH inappropriately in the low to normal range
Asymptomatic, although some have tetany, muscle
spasms, febrile seizures
Inappropriately high or normal urinary calcium
excretion
Treatment with 1,25-(OH)
2
D
3
leads to marked
hypercalciuria, can cause nephrocalcinosis and
CKD
Copyright Harvard Medical School, 2010. All Rights Reserved.
102
TAKE HOME MESSAGES
Loss-of-function mutations in CaSR cause
familial benign (hypocalciuric) hypercalcemia
(FHH) and neonatal severe
hyperparathyroidism, both with
inappropriately increased/normal PTH for
degree of hypercalcemia.
Gain-of-function mutations cause autosomal
dominant hypocalcemia with hypercalciuria
Calcium and Urinary
Concentrating Ability
Hypercalcemia is associated with
nephrogenic DI, in part due to induction of
medullaryCOX-2 (as in lithium R
x
)
In addition, activation of the calcium-
sensing receptor (CaSR) in the thick
ascending limb and collecting duct directly
inhibits urinary concentrating ability
Copyright Harvard Medical School, 2010. All Rights Reserved.
103
The CaSR Inhibits Transport in the
Thick Ascending Limb
Apical Expression of the CaSR in the
Renal Collecting Duct
A)Aquaporin-2 C) Aquaporin-4
B) Aquaporin-3 D) CaSR
E) CaSR immunoab
Sands, et al, JCI, 1997
Copyright Harvard Medical School, 2010. All Rights Reserved.
104
Activation of the Luminal CaSR
Inhibits Vasopressin-Stimulated Water
Transport in the Collecting Duct
Copyright Harvard Medical School, 2010. All Rights Reserved.
105
Low Calcium Diet in Hypercal ci uric Enuretic
Children Restores Aquaporin-2 Excretion (AVP
Response) and Reduces Enuresis
Valenti et al, AJP-Renal, 2002
Nocturnal Aqp-2 excretion indicates AVP-resistance,
reversed by calcium restriction
TAKE HOME MESSAGES
The calcium-sensing receptor directly
modulates renal function.
Specifically, activation of CaSR reduces
salt transport by the TAL and water
transport by the collecting duct.
These effects in large part underlie the
polyuria and the renal concentrating
defect that is seen with hypercalcemiaand
hypercalciuria.
Copyright Harvard Medical School, 2010. All Rights Reserved.
106
Vitamin 1,25-(OH)
2
D
3
and Calcium Homeostasis
Jones et al, Physiol Rev, 1998
Vitamin D
Is
Activated
by UVB Light
and
Hydroxylation in
Liver and Kidney
Copyright Harvard Medical School, 2010. All Rights Reserved.
107
Regulation of 1-Hydroxyl ase in Proximal
Tubule:
Activi ty by 1,25(OH)D
3
, by PTH
con control F forskolin (PKA)
D3 1,25(OH)D
3
PTH
Bland et al, Endocrinology, 1999
Vitamin D Hydroxylation by the
Proximal Tubule
Copyright Harvard Medical School, 2010. All Rights Reserved.
108
1 Hydroxylase in Lymphoid Tissue
E) Normal lymph node
G) Granuloma (sarcoid)
H) Sarcoidosis skin
Granolomas are associated with hypercalcemia,
due to UNREGULATED generation of calcitriol
Take Home Messages
Generation of the active form of vitamin D (1,25-
(OH)
2
D
3
) requires UVB light and two P450 hydroxylase
enzymes
Deficiency in vitamin D, its receptor, the 1-hydroxylase,
or of megalin (the endocytic receptor for vitamin D
binding protein in the proximal tubule) causes
hypocalcemia and rickets
1,25-(OH)
2
D
3
and Ca
2+
inhibit 1-hydroxylase, whereas
PTH and low PO
4
activate the enzyme
1,25-(OH)
2
D
3
increases renal and intestinal
absorption/reabsorptionof Ca
2+
Copyright Harvard Medical School, 2010. All Rights Reserved.
109
Renal Tubular Re-Absorption of Cations
Ion Transport in the TAL (Thick Ascending Limb)
Ca
2+
/Mg
2+
are
reabsorbed via
the
paracellular path
in the TAL,
driven by the
lumen-positi ve
potential
Copyright Harvard Medical School, 2010. All Rights Reserved.
110
Familial Hypomagnesemia/Hypercalciuria;
Loss of Paracellular Transport in the TAL
Invariably accompanied by hypercalciuria
Often have renal colic, UTIs,
nephrocalcinosis renal failure
Marked increase in fractional excretion of
magnesium, cured by renal transplantation
Associated polyuria and isosthenuria
(inability to concentrate the urine)
Science, 285, 1999
Paracellin-1 (Claudin-16)
is the disease gene for
familial hypomagnesemia
with hypercalciuria
Copyright Harvard Medical School, 2010. All Rights Reserved.
111
Paracellin/Claudin-16 is Expressed in the
Thick Ascending Limb and DCT
Ion Transport in the TAL (Thick Ascending
Limb)
Paracellin
affects
cation
permeability
of the
paracellular
pathway
Copyright Harvard Medical School, 2010. All Rights Reserved.
112
The DCT (Distal Convoluted Tubule)
Na
+
2K
+
ATP
K
+
Mg
2+
Ca
2+
Na
+
K
+
ROMK
ECaC-1
TRPM-6
NCC
KCC4
CLC-K1
Lumen
Ca /Mg
2+ 2+
3Na
+
Cl
-
Cl
-
Cl
-
Ca
2+
/Mg
2+
Transport in the DCT
Calbindins function
as intracellul ar
calcium buffers .
What are the
DCT pathways for
magnesium
Transport ?
Copyright Harvard Medical School, 2010. All Rights Reserved.
113
The DCT (Distal Convoluted Tubule)
Na
+
2K
+
ATP
K
+
Mg
2+
Ca
2+
Na
+
K
+
ROMK
ECaC-1
TRPM-6
NCC
KCC4
CLC-K1
Lumen
Ca /Mg
2+ 2+
3Na
+
Cl
-
Cl
-
Cl
-
Copyright Harvard Medical School, 2010. All Rights Reserved.
114
Copyright Harvard Medical School, 2010. All Rights Reserved.
115
The EGFR Inhibitor CetuximabCauses
Magnesium Wasting
B:

Normagnesemic control pts


Cetuximab-treated pts
P1070L EGF pts
Copyright Harvard Medical School, 2010. All Rights Reserved.
116
TAKE-HOME MESSAGES
Ca
2+
/Mg
2+
absorption by the thick
ascending limb is paracellular, requiring
paracellin/claudin-16
Vitamin D responsive epithelial calcium
channels (ECaC) have been identified.
Ca
2+
/Mg
2+
absorption in the DCT is a
transcellular process, involving ECaC1
and TRPM6
ParacrineEGF regulates DCT magnesium
transport
Renal Phosphate Transport
Copyright Harvard Medical School, 2010. All Rights Reserved.
117
Phosphate Transport in the
Proximal Tubule
Expression pattern of FGF23
Renal phosphate wasting, hypophosphatemia,
low or inappropriately normal calcitriol concentrations,
and osteomalacia
Copyright Harvard Medical School, 2010. All Rights Reserved.
118
FGF-23 Inhibits Phosphate Transport
in Proximal Tubular Cells
FGF23 and Oncogenic
Hypophosphatemic Osteomalacia (OHO)
Hypophosphatemia,
low calcitriol, and osteomalacia
Over-expression of FGF23
in sarcomas associated
with OHO
White et al, J Clin Endo Met, 2001
Copyright Harvard Medical School, 2010. All Rights Reserved.
119
Hypervitaminosis D and Suppressed PTH
in FGF23 Knockout Mice
FGF23
inhibits
generation
of calcitriol,
by inhibiting
1-hydroxylase
Copyright Harvard Medical School, 2010. All Rights Reserved.
120
Loss-of-Function in FGF23 and
Familial Hyperphosphatemic
Tumoral Calcinosis
Copyright Harvard Medical School, 2010. All Rights Reserved.
121
TAKE-HOME MESSAGES
FGF23 is a novel factor that increases
renal phosphate excretion by inhibiting
NPT2a; FGF23 is associated with both
hereditary and paraneoplastic
hypophosphatemicbone disease.
FGF23 also inhibits generation of calcitriol
Loss of function in FGF23 causes
hypercalcemia, due to in 1,25-(OH)D
3
Evolving role for FGF23 in CKD
Common Causes of
Hypercalcemia
Primary Hyperparathyroidism
Adenoma
Carcinoma
Hyperplasia
Malignancy
Humoral hypercalcemia PTH-RP, TNF/LT
(myeloma OAFs)
Lytic bone disease, incl. myeloma
Ectopic/lymphoid calcitriol production
Copyright Harvard Medical School, 2010. All Rights Reserved.
122
Less Common Causes of
Hypercalcemia
Inherited disease MENI/II, FHH
Granulomatous disease
Drugs lithium, thiazides, vitamin D,
estrogens
Milk alkali syndrome
Immobilization
Neonatal/childhood hypercalcemia
Milk-Alkali Syndrome
HYPERCALCEMIA
RENAL FAILURE ALKALOSIS
Copyright Harvard Medical School, 2010. All Rights Reserved.
123
PTH Reduces Apical Expression of
NPT2a
The Vicious Cycle of
Milk Alkali Syndrome
Increased
calcium
Decreased
GFR +
volume
depletion
+ emesis
Worsening
Alkalosis
Ca
excretion
Copyright Harvard Medical School, 2010. All Rights Reserved.
124
Initial Workup of Hypercalcemia
PTH should be suppressed, if WNL or
increased hyperparathyroidism
PTH-RP
1,25-OHD
3
and 25-OHD
2
levels
SPEP, urine IEP, and serum light chains
CXR +/- chest CT
Bone scan if biochemistry unrevealing
Management of Hypercalcemia
Saline hydration
Loop diuretcs, once euvolemic
Calcitonin
Bisphosphonates
zolendrate ATN,
pamidronate collapsing FSGS
Ibandronatemay be safer, ? less effective
Dialysis
Specific therapy
Sensipar or PTHx for PTH
Steroids for 1-hydroxylase
Copyright Harvard Medical School, 2010. All Rights Reserved.
125
Causes of Hypocalcemia
Hypoparathyroidism
Hereditary CaSR, PTH, etc.
Post-surgical, post irradiation
PTH resistance, e.g. with hypomagnesemia
Vitamin D abnormality
Genetic VDR, 1-hydroxylase
Nutritional, hypovitaminosis D / rickets
GI disease, hepatobiliarydisease
Renal insufficiency
Drugs foscarnet, kayexalate, citrate, dilantin,
phenobarbital
Hypomagnesemia
Renal FeMgof >3% in someone with
normal GFR is consistent with renal
magnesium wasting
Extra-renal
Nutritional alcoholics (also renal), TPN
Intestinal malabsorption celiac, proton-pump
inhibitors, etc.
Diarrhea
Redistribution to bone, e.g. in hungry bone
syndrome
Copyright Harvard Medical School, 2010. All Rights Reserved.
126
Renal Hypomagnesemia
Polyuria
ECFV expansion
Diuretics, particularly loop diuretics
EGF receptor inhibition - cetuximab
Hypercalcemia
Tubule toxins / acute tubular injury
Cisplatin - cyclosporine
Aminoglycosides - Foscarnet
Amphotericin
Pentamidine
Manifestations of Hypomagnesemia
Cardiac
ECG changes
Arrhythmias ectopy, torsades, VF
Neuromuscular
Weakness
Tremor, twitching, +Troussea/Chvosteks
Seizures, nystagmus
Metabolic
HYPOKALEMIA
Hypocalcemia
Copyright Harvard Medical School, 2010. All Rights Reserved.
127
Question #1
A 70 yo man with a history of DM, hyporeninemic
hypoaldosteronism, CKD, and recently treated
hypovitaminosis D is admitted to the hospital
with recurrent hypocalcemia.
Daily medications include lasix, calcitriol,
amlodipine, kayexalate, amlodipine,
omeprazole, bicitra, and hydralazine.
Laboratory Studies:
Ca: 6.0, I Ca: 2.7, Mg: 1.7, Phos: 6.2, iPTH: 906,
Albumin 3.4, 25-Vitamin D: 67 ng/ml (20 100)
1,25-Vitamin D
3
: 74 pg/ml (6-62)
A. Therapy with furosemide
B. Delayed intestinal response to
calcitriol
C. Therapy with omeprazole
D. Therapy with kayexalate
E. Therapy with Bicitra
Which of the following is the most likely
cause of his hypocalcemia?
Copyright Harvard Medical School, 2010. All Rights Reserved.
128
Question #2
A 70 yo man with a history of
hypercalemiaand biopsy-proven
sarcoidosis presents after a New Years
Eve party with a fractured leg. He was lost
to follow up just after his sarcoidosis
diagnosis 5 months prior to admission,
and has never received prednisone. Other
medical issues include mild CKD.
His peak calcium was 13.0 mg/dL five
months ago, but now it is 9.0 mg/dL with
an albumin of 3.0
A. His corrected calcium is in fact in the
hypercalcemic range
B. His sarcoidosis has undergone spontaneous
remission
C. He is taking another immunosuppressive
medication
D. He had previously been abusing Rolaids
E. His serum levels of 25-hydroxyvi tami n D
have decreased
Why is he now normocalcemic, without
having received prednisone?
Copyright Harvard Medical School, 2010. All Rights Reserved.
129
Seasonal Hypercalcemia in
Sarcoidosis
Tayl or et al , Am J Med, 1963
Response to UV Light Treatment in a Patient
With Sarcoidosis and Seasonal Hypercal cemi a
Smi th et al , Postgrad Med J, 1976
Copyright Harvard Medical School, 2010. All Rights Reserved.
130
Question #3
A 60 yo man presents with hypomagnesemia
despite oral replacement. He has type II DM,
with minimal proteinuria and a creatinine of 1.4.
PMH is notable for CAD, HTN, peptic ulcer
disease, and alcoholism. He says he has been
abstinent for several years. ROS is negative for
diarrhea, polyuria, or other Sx.
Medications include metoprolol, glipizide,
lisinopril, magnesium oxide 420 mg tid, and
omeprazole. Physical exam is unremarkable.
Labs are notable for a serum magnesium of 1.4,
a 24 hour urine contains ZERO magnesium and
150 mg of calcium.
A. Poorly controlled diabetes
B. Alcoholism
C. Glipizide
D. Celiac disease
E. Omeprazole
What is the most likely cause of his
hypomagnesemia?
Copyright Harvard Medical School, 2010. All Rights Reserved.
131
Disclosures
None relevant to calcium and
magnesium disorders
Copyright Harvard Medical School, 2010. All Rights Reserved.
132
Electrolytes and Acid-Base
Practice for the Boards I
2:15-3:45 pm
Alan S. L. Yu, M.B., B.Chir.
University of Southern California Keck School
of Medicine
30 cases
Financial disclosures
No conflict of interest to disclose.
Alan S. L. Yu, M.B., B. Chir.
Copyright Harvard Medical School, 2010. All Rights Reserved.
133
A 55 year-old man with colonic carcinoma undergoes
resection of his tumor and diverting colostomy. Post-
operatively he has prolonged ileus and is therefore started
on intravenous total parenteral nutrition. Four days later, a
renal consult is called for hypernatremia. On examination,
the patient is clinicallyeuvolemic.
Serum sodium 151 mEq/L
Blood glucose 138 mg/dL
Blood urea nitrogen 33 mg/dL
Serum creatinine 0.9 mg/dL
Urine osmolality 480 mOsm/kg
Urine sodium 30 mEq/L
Urine volume (24 h) 3700 mL
Case 1
Case 1
Which of the following is the MOST LIKELY cause
of his hypernatremia?
(A) Central diabetes insipidus
(B) Nephrogenic diabetes insipidus
(C) Osmotic diarrhea
(D) Osmotic diuresis
(E) Excess intravenous saline
Copyright Harvard Medical School, 2010. All Rights Reserved.
134
Hypernatremia
Insensible H
2
O loss
GI H
2
O loss
Na
+
intake
Renal H
2
O loss
U
Osm
<800 mOsm/kg >800 mOsm/kg
+
Water intake
Glucose, urea,
mannitol
DI Osmotic
diuresis
CDI NDI
Polyuria (UO > 3L/d)
Osmolar excretion rate (UO x U
Osm
)
>1000 mOsm/d <1000 mOsm/d
Osmotic diuresis
NaCl, glucose,
urea, mannitol
Water diuresis
Serum Na
>140 mEq/L <140 mEq/L
Diabetes insipidus Polydipsia
Copyright Harvard Medical School, 2010. All Rights Reserved.
135
Case 1
Which of the following is the MOST LIKELY cause
of his hypernatremia?
(A) Central diabetes insipidus
(B) Nephrogenic diabetes insipidus
(C) Osmotic diarrhea
(D) Osmotic diuresis
(E) Excess intravenous saline
Copyright Harvard Medical School, 2010. All Rights Reserved.
136
A 35-year-old man with bipolar disorder, maintained on
lithiumfor 10 years, is referred to you for chronic polyuria
and polydipsia. He complains that he has to void once
everyhour.
Serumsodium 144mEq/L
Bloodurea nitrogen 35mg/dL
Serumcreatinine 1.9mg/dL
Serumosmolality 292mOsm/kg
24-hr urine volume 5L
Urine sodium 28mEq/L
Urine osmolality 190mOsm/kg
Case 2
Case 2
All of the following might be appropriate in the
management of this patient EXCEPT:
(A) Thiazide diuretic
(B) Dietary salt reduction
(C) Discontinuation of lithium
(D) Fluid restriction
(E) Amiloride
Copyright Harvard Medical School, 2010. All Rights Reserved.
137
Polyuria (UO > 3L/d)
Osmolar excretion rate (UO x U
Osm
)
>1000 mOsm/d <1000 mOsm/d
Osmotic diuresis
NaCl, glucose,
urea, mannitol
Water diuresis
Serum Na
>140 mEq/L <140 mEq/L
Diabetes insipidus Polydipsia
Case 2
All of the following might be appropriate in the
management of this patient EXCEPT:
(A) Thiazide diuretic
(B) Dietary salt reduction
(C) Discontinuation of lithium
(D) Fluid restriction
(E) Amiloride
Copyright Harvard Medical School, 2010. All Rights Reserved.
138
A 17-year-old woman who is otherwise fit and well, and on
no medications, presents with a history of polyuria since
earlychildhood.
On presentation:
Serum sodium 141 mEq/L
Serum osmolality 285 mOsm/kg
Urine osmolality 210 mOsm/kg
Urine volume 4 L
Case 3
Copyright Harvard Medical School, 2010. All Rights Reserved.
139
After 4 hrs of water deprivation:
Serum sodium 146 mEq/L
Serum osmolality 298 mOsm/kg
Urine osmolality 250 mOsm/kg
1 hr after administration of 10 g intranasal DDAVP:
Serum sodium 145 mEq/L
Serum osmolality 296 mOsm/kg
Urine osmolality 425 mOsm/kg
Case 3
Case 3
Which of the following statements could be TRUE?
(A) She is entirely normal since her baseline serum
sodium is normal
(B) Her condition may have been caused by head trauma
(C) She may have an inherited mutation in a vasopressin
receptor gene
(D) She has an osmotic diuresis with washout of her
medullaryinterstitiumconcentrating gradient
(E) The water deprivation test is non-diagnostic because
water deprivation was not continued for long enough
Copyright Harvard Medical School, 2010. All Rights Reserved.
140
Water deprivation test
P
osm
295 mOsm/kg
U
Osm
>800 300-800 <300
Not renal
water loss
Complete DI Partial DI
DDAVP
D+50% No D
CDI NDI
DDX of renal water loss
Hypernatremia
Case 3
Which of the following statements could be TRUE?
(A) She is entirely normal since her baseline serum
sodium is normal
(B) Her condition may have been caused by head trauma
(C) She may have an inherited mutation in a vasopressin
receptor gene
(D) She has an osmotic diuresis with washout of her
medullaryinterstitiumconcentrating gradient
(E) The water deprivation test is non-diagnostic because
water deprivation was not continued for long enough
Copyright Harvard Medical School, 2010. All Rights Reserved.
141
An 87-year-old woman trippedinher bedroomat home and
was unable to get up. She was found by her neighbor 3
days later, and brought into the emergency room. On
examination she has a broken left hip, appears severely
dehydrated, and is making small volumes of very
concentratedurine.
Laboratory studies:
Serum sodium 157 mEq/L
Urine sodium <5 mEq/L
Urine osmolality 850 mOsm/kg
Case 4
Copyright Harvard Medical School, 2010. All Rights Reserved.
142
Case 4
Which of the following statements is TRUE?
(A) Her hypernatremia is most likely to be due to osmotic
diuresis
(B) She likely has underlying diabetes insipidus
(C) Treatment carries a potential risk of cerebral edema
(D) Non-steroidal anti-inflammatory drug would be helpful
(E) Her intracellular fluid volume is increased
Hypernatremia
Insensible H
2
O loss
GI H
2
O loss
Na
+
intake
Renal H
2
O loss
U
Osm
<800 mOsm/kg >800 mOsm/kg
+
Water intake
Glucose, urea,
mannitol
DI Osmotic
diuresis
CDI NDI
Copyright Harvard Medical School, 2010. All Rights Reserved.
143
Case 4
Which of the following statements is TRUE?
(A) Her hypernatremia is most likely to be due to osmotic
diuresis
(B) She likely has underlying diabetes insipidus
(C) Treatment carries a potential risk of cerebral edema
(D) Non-steroidal anti-inflammatory drug would be helpful
(E) Her intracellular fluid volume is increased
Copyright Harvard Medical School, 2010. All Rights Reserved.
144
A 45-year-old female underwent transsphenoidal resection
of a pituitary adenoma 1 week ago. Immediately
postoperatively she developed severe polyuria and was
started on DDAVP. She returns with change in mental
status, has a seizure in the emergency room, and is found
to have a serumsodiumof 114 mEq/L.
Case 5
Case 5
Which of the following statements is TRUE?
(A) Her postoperative condition was almost certainly
misdiagnosed and she should never have been
treated with DDAVP
(B) She currently has diabetes insipidus
(C) Her hyponatremia now is most likely due to central
(pituitary) hypothyroidism
(D) Her hyponatremia is primarily due to psychogenic
polydipsia
(E) She is at risk of developing diabetes insipidus
Copyright Harvard Medical School, 2010. All Rights Reserved.
145
Hensenet al. Clin Endocrinol. 1999;50, 431
Pathogenesis of disordered water metabolism
after transsphenoidal surgery
Day 1 - Day 4-5
Day 6-11
AVP
secretion
normal
Before
surgery
Day 1 Day 7 Day 14
Secretiona
l arrest
Degeneration
& AVP release
Residual
AVP
secretion
Monophasic (40%)
Biphasic (3%)
Triphasic (1%)
H
y
p
o
n
a
t
r
e
m
i
a
H
y
p
e
r
n
a
t
r
e
m
i
a
/
p
o
l
y
u
r
i
a
Case 5
Which of the following statements is TRUE?
(A) Her postoperative condition was almost certainly
misdiagnosed and she should never have been
treated with DDAVP
(B) She currently has diabetes insipidus
(C) Her hyponatremia now is most likely due to central
(pituitary) hypothyroidism
(D) Her hyponatremia is primarily due to psychogenic
polydipsia
(E) She is at risk of developing diabetes insipidus
Copyright Harvard Medical School, 2010. All Rights Reserved.
146
A 64-yr-old woman with coronary artery disease, multiple prior
myocardial infarctions and ischemic cardiomyopathy, with a left
ventricular ejection fraction of 15%, is admitted with pulmonary edema.
Her medications include aspirin, metoprolol, furosemide,
spironolactone, digoxin, isosorbide dinitrate, and lisinopril. On
examination, the blood pressure is 97/54 mm Hg, pulse rate 85 per
minute, jugular venous pressure 9 cm, moist mucous membranes,
lungs with diffuse inspiratorycrackles, heart with an S3 gallop, and
cool, clammy extremities with 1+peripheral edema.
Serum sodium 128 mEq/L
Serum potassium 3.6 mEq/L
Serum chloride 87 mEq/L
Serum bicarbonate 34 mEq/L
Blood urea nitrogen 46 mg/dL
Serum creatinine 1.2 mg/dL
Serum osmolality 264 mOsm/L
Case 6
Copyright Harvard Medical School, 2010. All Rights Reserved.
147
Urine electrolytes (6 hrs after last diuretic dose):
Urine sodium 15 mEq/L
Urine chloride <5 mEq/L
Urine osmolality 220 mOsm/kg
Case 6
Case 6
All of the following might be appropriate in the
management of this patient EXCEPT:
(A) Intravenous 0.9% saline
(B) Restriction of free water intake
(C) Dietary sodium restriction
(D) Dobutamine
(E) Acetazolamide
Copyright Harvard Medical School, 2010. All Rights Reserved.
148
Hyponatremia
P
osm
>290 mOsm/kg <275 mOsm/kg Normal
Glucose
Mannitol
Hypoosmolal
hyponatremia
Lipid
Protein
Volume status
Hypovolemic Edematous Euvolemic
Dehydration*
Addisons
Diuretics
CHF*
Nephrosis*
Liver failure*
Renal failure
U
Osm
> 100
<100
SIADH
Hypothyroid
Polydipsia
* U
Na
< 20
Hypoosmolal hyponatremia
Copyright Harvard Medical School, 2010. All Rights Reserved.
149
Case 6
All of the following might be appropriate in the
management of this patient EXCEPT:
(A) Intravenous 0.9% saline
(B) Restriction of free water intake
(C) Dietary sodium restriction
(D) Dobutamine
(E) Acetazolamide
Copyright Harvard Medical School, 2010. All Rights Reserved.
150
A 45-year-old male smoker presents with confusion and
drowsiness. His only medications are bronchodilator and
steroid inhalers. On examination, his BP is 125/86, HR 78,
moist mucous membranes, good skin turgor, jugular venous
pressure 4 cm, lung fields clear to auscultation, no
peripheral edema. Chest radiograph shows
emphysematous changes but is otherwise normal.
Serum sodium 116 mEq/L
Serum osmolality 256 mOsm/kg
Urine sodium 96 mEq/L
Urine potassium 87 mEq/L
Urine osmolality 670 mOsm/kg
Case 7
The appropriate management of this patient should include:
(A) Thyroid radioiodine scan
(B) Computed tomography scan of the chest
(C) Thiazidediuretic
(D) Haloperidol
(E) Intravenous 0.9% sodium chloride to correct the
serum sodium
Case 7
Copyright Harvard Medical School, 2010. All Rights Reserved.
151
Volume status
Hypovolemic Edematous Euvolemic
Dehydration*
Addisons
Diuretics
CHF*
Nephrosis*
Liver failure*
Renal failure
U
Osm
> 100
<100
SIADH
Hypothyroid
Polydipsia
* U
Na
< 20
Hypoosmolal hyponatremia
What is the effect of this IV fluid on patients serum
Na
+
/osmolality?
Urine:
Na 96 mEq/L
K 87 mEq/L
Urea 900 mg/dL
Osm 670 mOsm/L
Ineffective osmoles
0.9% saline IV
Na 154 mEq/L
Effective osmoles:
2(Na +K) =366 mOsm/L
Urine tonicity >isotonic saline
Electrolyte-free water
clearance is negative
Infusion of isotonic saline
would lower serum Na
+
!
Serum Na
+
116 mEq/L
Copyright Harvard Medical School, 2010. All Rights Reserved.
152
Rx of hyponatremia
Hypovolemia Isotonic saline
Polydipsia Water restriction
SIADH Water restriction
Isotonic (U
Na
+U
K
<150) saline
Hypertonic saline / Na tablets
Furosemide
Demeclocycline/Conivaptan
Case 7
The appropriate management of this patient should include:
(A) Thyroid radioiodine scan
(B) Computed tomography scan of the chest
(C) Thiazidediuretic
(D) Haloperidol
(E) Intravenous 0.9% sodium chloride to correct the
serum sodium
Copyright Harvard Medical School, 2010. All Rights Reserved.
153
An 18-year-old woman with schizophrenia, maintained on
haloperidol, presents with a generalized seizure.
Serum sodium 120 mEq/L
Serum osmolality 252 mOsm/kg
Urine sodium 20 mEq/L
Urine osmolality 90 mOsm/kg
Case 8
Copyright Harvard Medical School, 2010. All Rights Reserved.
154
Case 8
Which of the following statements about this patient is
TRUE:
(A) The serum lipid and total protein must be measured to
exclude pseudohyponatremia
(B) Her condition has been caused by haloperidol, which
should be discontinued
(C) Correction of the hyponatremia may be complicated
by cerebral edema
(D) The serum sodium can be corrected at 1-2 mEq/L per
hour
(E) Fluid restriction will not correct the hyponatremia
Hyponatremia
P
osm
>290 mOsm/kg <275 mOsm/kg Normal
Glucose
Mannitol
Hypoosmolal
hyponatremia
Lipid
Protein
Copyright Harvard Medical School, 2010. All Rights Reserved.
155
Volume status
Hypovolemic Edematous Euvolemic
Dehydration*
Addisons
Diuretics
CHF*
Nephrosis*
Liver failure*
Renal failure
U
Osm
> 100
<100
SIADH
Hypothyroid
Polydipsia
* U
Na
< 20
Hypoosmolal hyponatremia
Case 8
Which of the following statements about this patient is
TRUE:
(A) The serum lipid and total protein must be measured to
exclude pseudohyponatremia
(B) Her condition has been caused by haloperidol, which
should be discontinued
(C) Correction of the hyponatremia may be complicated
by cerebral edema
(D) The serum sodium can be corrected at 1-2 mEq/L per
hour
(E) Fluid restriction will not correct the hyponatremia
Copyright Harvard Medical School, 2010. All Rights Reserved.
156
A 19-year-old male who recently returned from trekking in
the Himalayas has had watery diarrhea for one week. On
examination he appears tired and mucous membranes are
dry. BP is 105/70 and HR 80 sitting, BP 95/60 and HR 102
standing.
Serum sodium 128 mEq/L
Serum potassium 2.8 mEq/L
Serum chloride 102 mEq/L
Serum bicarbonate 18 mEq/L
Blood urea nitrogen 48 mg/dL
Serum creatinine 1.1 mg/dL
Urine sodium <5 mEq/L
Urine osmolality 350 mOsm/kg
Case 9
Copyright Harvard Medical School, 2010. All Rights Reserved.
157
Case 9
Which of the following would be the most appropriate
management of this patient:
(A) Hypertonic saline
(B) Conivaptan
(C) 0.9% NaCl and hydrocortisone
(D) 0.9% NaCl and furosemide
(E) 0.9% NaCl and KCl
Volume status
Hypovolemic Edematous Euvolemic
Dehydration*
Addisons
Diuretics
CHF*
Nephrosis*
Liver failure*
Renal failure
U
Osm
> 100
<100
SIADH
Hypothyroid
Polydipsia
* U
Na
< 20
Hypoosmolal hyponatremia
Copyright Harvard Medical School, 2010. All Rights Reserved.
158
Rx of hyponatremia
Hypovolemia Isotonic saline
Polydipsia Water restriction
SIADH Water restriction
Isotonic (U
Na
+U
K
<150) saline
Hypertonic saline / Na tablets
Furosemide
Demeclocycline/Conivaptan
Case 9
Which of the following would be the most appropriate
management of this patient:
(A) Hypertonic saline
(B) Conivaptan
(C) 0.9% NaCl and hydrocortisone
(D) 0.9% NaCl and furosemide
(E) 0.9% NaCl and KCl
Copyright Harvard Medical School, 2010. All Rights Reserved.
159
An 18 year-old female college student runs the Boston
marathon. It is her first marathon and so she is careful
to keep well hydrated before and during the race. At
the finish line, she feels severely nauseated, dyspneic
and has generalized headache. BP is 102/64, HR 95,
otherwise normal exam.
Her serum sodium concentration is 112 mEq/L.
Case 10
Copyright Harvard Medical School, 2010. All Rights Reserved.
160
Case 10
Which of the following would be the best initial treatment
for this patient:
(A) 1 liter of intravenous 0.9% NaCl
(B) Fluid restriction
(C) Salt tablets orally
(D) 100 mL intravenous 3% NaCl
(E) 1 liter of sports drink
Exercise-associated hyponatremia (EAH)
Pathogenesis of EAH
EAH is dilutional due to
excessive water or sports
drink consumption, not due
tosalt loss/dehydration
In addition, ADH levels are
inappropriately elevated and
U
osm
is high
Almond et al.(2005) NEJ M 352:1550
Copyright Harvard Medical School, 2010. All Rights Reserved.
161
Drinkonlyaccordingto thirst
Use the USATF guidelines, or analogous methods, to estimate
hourly sweat losses during exercise and avoid consuming amounts
greater than this duringendurance exerciseevents
Any athlete with EAH and respiratory insufficiency, confusion,
obtundation, N/V can be treated on-site with 100 mL of 3% NaCl
over 10 min (expectedD[Na
+
] ~2-3mmol/L)
If symptomatic EAH persists, repeat 100 mL 3% NaCl hourly at a
rateof 100 mL/hand monitor serumNa hourly
Osmotic demyelination in association with the rapid correction of an
acute hyponatremia has not been reported and should never be an
impedimentto rapidlycorrectinghyponatremiainsymptomaticEAH
Hew Butler et al. (2005) Clin J Sport Med 15:208
Consensus Statement of the 1st International Exercise-
Associated Hyponatremia Consensus Development
Conference, Cape Town 2005:
Case 10
Which of the following would be the best initial treatment
for this patient:
(A) 1 liter of intravenous 0.9% NaCl
(B) Fluid restriction
(C) Salt tablets orally
(D) 100 mL intravenous 3% NaCl
(E) 1 liter of sports drink
Copyright Harvard Medical School, 2010. All Rights Reserved.
162
An 30-year-old female with anorexia nervosa is found on
routine laboratory examination to be hypokalemic. BP
95/45, HR 66.
Serum sodium 133 mEq/L
Serum potassium 3.2 mEq/L
Serum chloride 93 mEq/L
Serum bicarbonate 32 mEq/L
Blood urea nitrogen 6 mg/dL
Serum creatinine 0.6 mg/dL
24 hour urine
Volume 1 L
Potassium 80 mEq/L
Chloride 44 mEq/L
Calcium 300 mg/day (normal <150)
Case 11
Copyright Harvard Medical School, 2010. All Rights Reserved.
163
Which of the following is the most likely cause of this
patients laboratory findings:
(A) Loop diuretic abuse
(B) Laxative abuse
(C) Gitelman syndrome
(D) Thiazide diuretic abuse
(E) Surreptitious vomiting
Case 11
DDX of hypokalemia
Cellular shift GI loss Urinary K wasting
Alkalemia
Insulin
-agonist
HypoKPP
Vomiting
Diarrhea
24 hr U
K
>25 mEq
Copyright Harvard Medical School, 2010. All Rights Reserved.
164
BP/volume
Aldo
Bartter
Loop diuretic
Renin
Cushing
Liddle
Licorice
AME
RAS
Reninoma
Malig HTN
1 aldo
GRA
Urine Cl
Consider:
Diarrhea/laxatives
RTA
Toluene
Ampho
Low HCO
3
-
Vomiting
Gitelman
Thiazide
Urine Ca
High Low
High Low
Low High
High Low
High Low
Consider:
1 HypoMg
V. low Mg
2+
Renal K wasting with CCD [K
+
]
Which of the following is the most likely cause of this
patients laboratory findings:
(A) Loop diuretic abuse
(B) Laxative abuse
(C) Gitelman syndrome
(D) Thiazide diuretic abuse
(E) Surreptitious vomiting
Case 11
Copyright Harvard Medical School, 2010. All Rights Reserved.
165
A 23-year-old male is referred for work-up of newly
diagnosed hypertension. He is otherwise asymptomatic.
Family history is non-contributory. BP 145/97. He is mildly
obese.
Serum sodium 143 mEq/L
Serum potassium 3.0 mEq/L
Serum chloride 106 mEq/L
Serum bicarbonate 29 mEq/L
Blood urea nitrogen 12 mg/dL
Serum creatinine 1.2 mg/dL
Plasma reninactivity 0.7 ng/mL/hr (NR 1-6)
Plasma aldosterone 3.9 ng/dl (NR 5-20)
24 hour urine potassium 65 mEq
Case 12
Copyright Harvard Medical School, 2010. All Rights Reserved.
166
Which of the following tests would be most likely to be
helpful in the diagnosis:
(A) Serum metanephrines
(B) Doppler ultrasound of the renal arteries
(C) 24 hr urine aldosterone
(D) 24 hr urine 18-hydroxycortisol and 18-oxocortisol
(E) 24 hr urine cortisol
Case 12
BP/volume
Aldo
Bartter
Loop diuretic
Renin
Cushing
Liddle
Licorice
AME
RAS
Reninoma
Malig HTN
1 aldo
GRA
Urine Cl
Consider:
RTA
Toluene
Ampho
Low HCO
3
-
Vomiting
Gitelman
Thiazide
Urine Ca
High Low
High Low
Low High
High Low
High Low
Consider:
1 HypoMg
V. low Mg
2+
Renal K wasting with CCD [K
+
]
Copyright Harvard Medical School, 2010. All Rights Reserved.
167
Case 12
Which of the following tests would be the most likely to be
helpful in the diagnosis:
(A) Serum metanephrines
(B) Doppler ultrasound of the renal arteries
(C) 24 hr urine aldosterone
(D) 24 hr urine 18-hydroxycortisol and 18-oxocortisol
(E) 24 hr urine cortisol
Copyright Harvard Medical School, 2010. All Rights Reserved.
168
A 19-year-old male student visiting from Taiwan presents to
the emergency room with acute onset of flaccid muscle
weakness. He denies diarrhea or vomiting and is using no
medications. BP 125/88, HR 95.
Serum sodium 139 mEq/L
Serum potassium 1.7 mEq/L
Serum chloride 105 mEq/L
Serum bicarbonate 25 mEq/L
Serum glucose 100 mg/dL
Blood urea nitrogen 11 mg/dL
Urine potassium <5 mEq/L
Case 13
Which of the following would be appropriate in the
management of this patient:
(A) Repletion with KCl, 160 mEq daily
(B) Loperamide
(C) Indomethacin
(D) Check serum thyroid-stimulating hormone level
(E) None of the above
Case 13
Copyright Harvard Medical School, 2010. All Rights Reserved.
169
DDX of hypokalemia
Cellular shift GI loss Urinary K wasting
Alkalemia
Insulin
-agonist
Hypokalemic periodic paralysis
Vomiting
Diarrhea
24 hr U
K
>25 mEq
Thyrotoxic hypokalemic periodic
paralysis
Presents age 20-40 yr
Predominantly Asians
Mostly male (20:1 male:female ratio)
Only with thyrotoxicosis (~2% of Asians with
thyrotoxicosis) which may be asymptomatic
Clinical features identical to familial form
Copyright Harvard Medical School, 2010. All Rights Reserved.
170
Which of the following would be appropriate in the
management of this patient:
(A) Repletion with KCl, 160 mEq daily
(B) Loperamide
(C) Indomethacin
(D) Check serum thyroid-stimulating hormone level
(E) None of the above
Case 13
Copyright Harvard Medical School, 2010. All Rights Reserved.
171
A 67 year-old male with hypertension for 30 years has
been having worsening blood pressure control over the
past year. He had a myocardial infarction 2 years ago. His
current medications are aspirin, atenolol, amlodipine,
lisinopril and spironolactone. On a clinic visit, his BP is
163/95.
Serum sodium 141 mEq/L
Serum potassium 3.4 mEq/L
Serum chloride 108 mEq/L
Serum bicarbonate 27 mEq/L
Blood urea nitrogen 15 mg/dL
Serum creatinine 1.6 mg/dL
Urinalysis: pH 5, 1+protein, no cells or casts
Case 14
Which of the following test would be most likely to identify
the cause of this patient's hypertension:
(A) Magnetic resonance angiography of the renal
arteries
(B) CT scan of the adrenal glands
(C) Serum metanephrines
(D) Urine tetrahydrocortisol/tetrahydrocortisoneratio
(E) None of the above
Case 14
Copyright Harvard Medical School, 2010. All Rights Reserved.
172
Renal K-wasting with hypertension
Aldo
Renin
Cushing
Liddle
Licorice
Syndrome apparent
mineralocorticoid excess
Renal artery stenosis
Reninoma
Malignant HTN
Primary aldosteronism
Glucocorticoid-remediable
aldosteronism
High Low
High Low
Captopril renal scan
Doppler US
MRA
PAC/PRA
Serum/urine aldosterone
Urine 18-oxo- & -hydroxycortisol
Serum/urine cortisol
ENaC mutation
analysis
Urine THC/THE
ratio
Which of the following test would be most likely to identify
the cause of this patient's hypertension:
(A) Magnetic resonance angiography of the renal
arteries
(B) CT scan of the adrenal glands
(C) Serum metanephrines
(D) Urine tetrahydrocortisol/tetrahydrocortisoneratio
(E) None of the above
Case 14
Copyright Harvard Medical School, 2010. All Rights Reserved.
173
A 41-year-old female with lupus nephritis and chronic renal
insufficiency maintained on prednisone and candesartan
presents for a routine clinic visit. On exam, BP is 146/95
and there is moderate dependent edema.
Laboratory studies:
Serum sodium 136 mEq/L
Serum potassium 6.3 mEq/L
Serum chloride 109 mEq/L
Serum bicarbonate 18 mEq/L
Blood urea nitrogen 25 mg/dL
Serum creatinine 1.6 mg/dL
Case 15
Copyright Harvard Medical School, 2010. All Rights Reserved.
174
Which of the following would NOT be appropriate in the
management of this patient:
(A) Hydrochlorothiazide
(B) Discontinue candesartan
(C) Fludrocortisone
(D) Low potassium diet
(E) Sodium polystyrene sulfonate
Case 15
Hyperkalemia
Intake
Cell shift
Metabolic acidosis
Hyperglycemia
b-blocker
Digitalis
Hyperkalemic
periodic paralysis
Cell lysis
Decreased urinary
K
+
excretion
24 hr urine K
+
<40 mEq
Copyright Harvard Medical School, 2010. All Rights Reserved.
175
ACTH test Hyporenin
hypoaldo
Meds
Addisons
1 hypoaldo
NSAIDs
ACEI/ARB
Heparin
Spironolactone
Amiloride
Trimethoprim
Pentamidine
Cyclosporine
Decreased urinary K
+
excretion
Renal
failure
CCD [K
+
] Tubular flow
ECV
Which of the following would NOT be appropriate in the
management of this patient:
(A) Hydrochlorothiazide
(B) Discontinue candesartan
(C) Fludrocortisone
(D) Low potassium diet
(E) Sodium polystyrene sulfonate
Case 15
Copyright Harvard Medical School, 2010. All Rights Reserved.
176
A 26-year-old male with AIDS is admitted with
pneumocystis pneumonia and treated with prednisone and
intravenous trimethoprim-sulfamethoxazole. On
examination, BP is 115/65, HR 90, RR 24. He appears
tachpneic with diffuse rales on chest auscultation. EKG
appears normal.
Serum sodium 134 mEq/L
Serum potassium 5.9 mEq/L
Serum chloride 105 mEq/L
Serum bicarbonate 22 mEq/L
Blood urea nitrogen 8 mg/dL
Serum creatinine 0.7 mg/dL
Case 16
Copyright Harvard Medical School, 2010. All Rights Reserved.
177
Which of the following would be the best treatment for the
hyperkalemia in this patient:
(A) Fludrocortisone
(B) Sodium bicarbonate
(C) Discontinue trimethoprim-sulfamethoxazole and
start pentamidine
(D) Discontinue trimethoprim-sulfamethoxazole and
start atovaquone
(E) None of the above
Case 16
Arachidonic
acid
Angiotensinogen
Adrenal
glomerulosa
JGA
PGE
2
Renin
COX2
AngI AngII
ACE
AT1R
Aldosterone
MCR
ENaC block:
Amiloride
Trimethoprim
Pentamidine
Copyright Harvard Medical School, 2010. All Rights Reserved.
178
Which of the following would be the best treatment for the
hyperkalemia in this patient:
(A) Fludrocortisone
(B) Sodium bicarbonate
(C) Discontinue trimethoprim-sulfamethoxazole and
start pentamidine
(D) Discontinue trimethoprim-sulfamethoxazole and
start atovaquone
(E) None of the above
Case 16
Copyright Harvard Medical School, 2010. All Rights Reserved.
179
A 57-year-old female with end-stage renal disease
secondary to diabetic nephropathy maintained on chronic
hemodialysis is seen on a non-dialysis day.
Serum sodium 139 mEq/L
Serum potassium 7.1 mEq/L
Serum chloride 108 mEq/L
Serum bicarbonate 20 mEq/L
Blood urea nitrogen 26 mg/dL
Serum creatinine 4.5 mg/dL
Case 17
All of the following would lower the serum potassium
substantially EXCEPT:
(A) Insulin and glucose
(B) Sodium bicarbonate
(C) Albuterol
(D) Sodium polystyrene sulfonate
(E) Hemodialysis
Case 17
Copyright Harvard Medical School, 2010. All Rights Reserved.
180
Treatment of hyperkalemia
Stabilize membrane excitability
Calcium chloride or gluconate, 1 g IV
Increase K
+
entry into cells
Glucose 25 g and insulin 10 U
b
2
-adrenergic agonist (albuterol 10-20 mg inh)
NaHCO
3
(poor efficacy in ESRD patients)
Removal of excess K
+
Cationexchange resin (Kayexalate)
Diuretics
Dialysis
Dietary K
+
restriction
All of the following would lower the serum potassium
substantially EXCEPT:
(A) Insulin and glucose
(B) Sodium bicarbonate
(C) Albuterol
(D) Sodium polystyrene sulfonate
(E) Hemodialysis
Case 17
Copyright Harvard Medical School, 2010. All Rights Reserved.
181
A 23 year-old male with diabetes mellitus presents with 5
days of polyuria, polydipsiaandabdominal pain.
Serum sodium 134 mEq/L
Serum potassium 6.5 mEq/L
Serum chloride 99 mEq/L
Serum bicarbonate 15 mEq/L
Serum glucose 296 mg/dL
Blood urea nitrogen 38 mg/dL
Serum creatinine 1.6 mg/dL
Urinalysis:
Specific gravity 1.028, pH 4.5, 1+protein, 2+ketones
Case 18
Copyright Harvard Medical School, 2010. All Rights Reserved.
182
Which of the following statements about the total body
stores in this patient is most likely to be correct:
(A) Total body potassium depleted
(B) Total body potassium overloaded
(C) Normal total body potassium stores, total body
sodium depleted
(D) Normal total body potassium stores, total body
sodium overloaded
(E) None of the above
Case 18
Osmotic diuresis:
Free water
Na
+
K
+
Electrolyte imbalance in diabetic ketoacidosis
ICF => ECF
K
+
Copyright Harvard Medical School, 2010. All Rights Reserved.
183
Which of the following statements about the total body
stores in this patient is most likely to be correct:
(A) Total body potassium depleted
(B) Total body potassium overloaded
(C) Normal total body potassium stores, total body
sodium depleted
(D) Normal total body potassium stores, total body
sodium overloaded
(E) None of the above
Case 18
Copyright Harvard Medical School, 2010. All Rights Reserved.
184
A 27-year-old African-American female presents with dyspnea,
fatigue and weight loss.
Blood urea nitrogen 32 mg/dL
Serum creatinine 2.1 mg/dL
Serum albumin 4 g/dL
Serum calcium 11.5 mg/dL
Serum phosphorus 5.5 mg/dL
Serum immunoreactiveparathyroid hormone (intact)
3.1 pg/mL (Normal range 10-65 pg/mL)
Serum 25-hydroxycholecalciferol
8.3 pg/mL (Normal range 9-43 pg/mL)
Serum 1,25-dihydroxycholecalciferol
95 pg/mL (Normal range 15-60 pg/mL)
Case 19
The most likely cause of hypercalcemia in this patient is:
(A) Primary hyperparathyroidism
(B) Familial hypocalciuric hypercalcemia
(C) Sarcoidosis
(D) Munchausens syndrome
(E) Occult malignancy
Case 19
Copyright Harvard Medical School, 2010. All Rights Reserved.
185
Hypercalcemia: Work-up
iPTH
High Low
Urine Ca
FHH 1 PTH
1,25-D
HypervitD
Sarcoidosis
TFTs
T4 Malignancy
Skeletal survey
Bone scan
SPEP
UPEP
PTHrP
High Low High Low
High Low
The most likely cause of hypercalcemia in this patient is:
(A) Primary hyperparathyroidism
(B) Familial hypocalciuric hypercalcemia
(C) Sarcoidosis
(D) Munchausens syndrome
(E) Occult malignancy
Case 19
Copyright Harvard Medical School, 2010. All Rights Reserved.
186
An 18-year-old female presents with severe muscle weakness. She has
no past medical history, denies taking any medications, and the family
history is non-contributory. BP 90/67.
Serum sodium 136 mEq/L
Serum potassium 3.4 mEq/L
Serum chloride 106 mEq/L
Serum bicarbonate 29 mEq/L
Blood urea nitrogen 15 mg/dL
Serum creatinine 1.0 mg/dL
Serum calcium 8.8 mg/dL
Serum magnesium 1.2 mg/dL
Serum phosphorus 3.1 mg/dL
24-hour urine magnesium 151 mg
24-hour urine calcium 58 mg
Case 20
Copyright Harvard Medical School, 2010. All Rights Reserved.
187
Which of the following is the most likely cause of her
hypomagnesemia:
(A) Gitelmans syndrome
(B) Conns syndrome
(C) Surreptitious loop diuretic use
(D) Surreptitious vomiting
(E) Laxative abuse
Case 20
Differential diagnosis of hypomagnesemia
Copyright Harvard Medical School, 2010. All Rights Reserved.
188
Diagnosis of renal Mg wasting
F
E
Mg > 1%
24 hr urine Mg > 24 mg
Renal Mg
2+
wasting
Polyuria
Tubule
reabsorption
DKA
ATN recovery
Postobstructive
PCT reabs TAL reabs DCT reabs
Tubule toxins
Aminoglycosides
Cisplatin
Amphotericin
Cyclosporine
Pentamidine
Loop diuretics
Bartters
Familial
hypercalciuric
hypomagnesemia
Thiazides
Gitelmans
Autosomal recessive
hypomagnesemia
Volume expansion
Hyperaldosteronism
Copyright Harvard Medical School, 2010. All Rights Reserved.
189
Which of the following is the most likely cause of his
hypomagnesemia:
(A) Gitelmans syndrome
(B) Conns syndrome
(C) Surreptitious loop diuretic use
(D) Surreptitious vomiting
(E) Laxative abuse
Case 20
Copyright Harvard Medical School, 2010. All Rights Reserved.
190
A 47 yr old man with Type II diabetes mellitus is seen for
routine follow up for chronic kidney disease, stage III,
thought to be due to diabetic nephropathy. Medications
include glipizide, hydrochlorothiazide, losartan, ferrous
sulfate, darbepoietin, calcium acetate, and vitamin D.
Serum sodium 156 mEq/L
Serum potassium 5.1 mEq/L
Serum chloride 117 mEq/L
Serum bicarbonate 24 mEq/L
Blood urea nitrogen 21 mg/dL
Serum creatinine 1.9 mg/dL
Serum glucose 135 mg/dL
Serum calcium 11.6 mg/dL
Serum phosphate 4.1 mg/dL
S1
Which of the following is the most likely cause of the
hypernatremia in this patient?
(A) Hydrochlorothiazide
(B) Losartan
(C) Vitamin D
(D) Osmotic diuresis from glucose
(E) Surreptitious laxative abuse
S1
Copyright Harvard Medical School, 2010. All Rights Reserved.
191
Nephrogenic diabetes insipidus
Hypokalemia
Hypercalcemia
Tubulointerstitial nephropathies
Sickle cell disease
Myeloma
Obstructive uropathy
Recovery from ATN
Lithium
Familial (rare)
X-linked recessive (mutations in V2 receptor)
Autosomal recessive (mutations in AQP2)
S1
Which of the following is the most likely cause of the
hypernatremia in this patient?
(A) Hydrochlorothiazide
(B) Losartan
(C) Vitamin D
(D) Osmotic diuresis from glucose
(E) Surreptitious laxative abuse
Copyright Harvard Medical School, 2010. All Rights Reserved.
192
A 34 yr old male with alcoholic cirrhosis is admitted with
esophageal variceal bleeding and hepatic encephalopathy.
He underwent variceal banding, was maintained NPO and
was treated with propanolol, octreotide, lactulose and
maintenance fluids with D5-1/2 NS.
Serum sodium 151 mEq/L
Serum potassium 3.0 mEq/L
Serum chloride 124 mEq/L
Serum bicarbonate 19 mEq/L
Blood urea nitrogen 25 mg/dL
Serum creatinine 1.2 mg/dL
Urine osmolality 855 mOsm/kg
S2
Copyright Harvard Medical School, 2010. All Rights Reserved.
193
Which of the following would be the BEST treatment to
prevent worsening hypernatremia in this patient:
(A) Reduce the dose of lactulose
(B) Reduce the dose of octreotide
(C) Change fluids to 5% dextrose
(D) Start hydrochlorothiazide
(E) Start democlocycline
S2
Hypernatremia
Insensible H
2
O loss
GI H
2
O loss
Na
+
intake
Renal H
2
O loss
U
Osm
<800 mOsm/kg >800 mOsm/kg
+
Water intake
Glucose, urea,
mannitol
DI Osmotic
diuresis
CDI NDI
Copyright Harvard Medical School, 2010. All Rights Reserved.
194
Osmotic diarrhea
S
t
o
o
l

o
s
m
o
l
a
l
i
t
y
Which of the following would be the BEST treatment to
prevent worsening hypernatremia in this patient:
(A) Reduce the dose of lactulose
(B) Reduce the dose of octreotide
(C) Change fluids to 5% dextrose
(D) Start hydrochlorothiazide
(E) Start democlocycline
S2
Copyright Harvard Medical School, 2010. All Rights Reserved.
195
A 66 yr old female with NIDDM presents with several days
of malaise,lethargy, polyuriaand decreased oral intake.
Physical examination reveals tachycardia, orthostatic
hypotension and dry mucous membranes.
Serum sodium 134 mEq/L
Serum potassium 5.4 mEq/L
Serum chloride 94 mEq/L
Serum bicarbonate 25 mEq/L
Blood urea nitrogen 42 mg/dL
Serum creatinine 1.8 mg/dL
Serum glucose 975 mg/dL
S3
Copyright Harvard Medical School, 2010. All Rights Reserved.
196
After initial resuscitation with insulin and isotonic saline,
which of the following treatments would be appropriate in
this patient:
(A) Hypertonic saline
(B) Oral fluid restriction
(C) Hypotonic saline
(D) Tolvaptan
(E) Furosemide
S3
Hyponatremia
P
osm
>290 mOsm/kg <275 mOsm/kg Normal
Glucose
Mannitol
Hypoosmolal
hyponatremia
Lipid
Protein
Correction for hyperglycemia:
For every 100 mg/dL increase in glucose, add 1.6 mEq/L to Na
+
Copyright Harvard Medical School, 2010. All Rights Reserved.
197
S3
After initial resuscitation with insulin and isotonic saline,
which of the following treatments would be appropriate in
this patient:
(A) Hypertonic saline
(B) Oral fluid restriction
(C) Hypotonic saline
(D) Tolvaptan
(E) Furosemide
Copyright Harvard Medical School, 2010. All Rights Reserved.
198
A 74 yr old female with a history of hypertension and
hyperlipidemiais admitted with unsteady gait and a fall at home.
Medications: Hydrochlorothiazide, lisinopril, simvastatin, aspirin.
On exam she appears euvolemic.
Serum sodium 117 mEq/L
Serum potassium 4.6 mEq/L
Serum chloride 92 mEq/L
Blood urea nitrogen 7 mg/dL
Serum creatinine 0.5 mg/dL
Serum triglycerides 345 mg/dL
Serum cortisol 15 g/dL (NR 5-15 g/dL)
Serum free T4 0.5 ng/dL (NR 0.7-2.0 ng/dL)
Urine sodium 38 mEq/L
Urine osmolality 230 mOsm/kg
S4
What is the most likely cause of her hyponatremia
(A) Pseudohyponatremia
(B) Adrenal insufficiency
(C) Hypothyroidism
(D) SIADH
(E) Adverse effect of medications
S4
Copyright Harvard Medical School, 2010. All Rights Reserved.
199
Volume status
Hypovolemic Edematous Euvolemic
Dehydration*
Addisons
Diuretics
CHF*
Nephrosis*
Liver failure*
Renal failure
U
Osm
> 100
<100
SIADH
Hypothyroid
Thiazides
Polydipsia
* U
Na
< 20
Hypoosmolal hyponatremia
S4
What is the most likely cause of her hyponatremia
(A) Pseudohyponatremia
(B) Adrenal insufficiency
(C) Hypothyroidism
(D) SIADH
(E) Adverse effect of medications
Copyright Harvard Medical School, 2010. All Rights Reserved.
200
A 48 yr old female with small cell lung cancer has been
noted to have a low serum Na
+
for 3 months. She presents
to the ER with nausea and inability to tolerate oral fluids.
On exam, her weight is 55 kg. She appears mildly
hypovolemic but alert and oriented.
Admission:
Serum sodium 108 mEq/L
Urine osmolality 800 mg/dL
She is initiated on 3% NaCl at 200 mL/hr. Five hours later
her serum sodium is 121 mEq/L. Her mental status is
unchanged.
S5
Copyright Harvard Medical School, 2010. All Rights Reserved.
201
Which of the following would be the most appropriate next
step in the management of this patient:
(A) 3% NaCl at 80 mL/hr
(B) Discontinue fluids and initiate tolvaptan
(C) Discontinue fluids and initiate 1 L daily oral fluid
restriction
(D) 5% dextrose at 150 mL/hr for 2 hr
(E) None of the above
S5
Rate of correction of hyponatremia
Acute (<48 hr) and symptomatic
1-2 mEq/l per hour
Chronic (>48 hr) including SIADH and
asymptomatic
0.5 mEq/l per hour
Do not exceed 12 mEq/L increase, or correct to
>120-125 mEq/L in the 1st day
Copyright Harvard Medical School, 2010. All Rights Reserved.
202
Osmotic demyelination syndrome
Central and extrapontine myelinolysis
Risk factors :
Excessive rate or amount of correction of serum Na
+
Malnutrition and alcoholism
Severe liver disease
Hypoxia
Classic CPM presents with dysphagia, quadriparesis,
locked-in syndrome
Can be permanent or fatal
S5
Which of the following would be the most appropriate
next step in the management of this patient:
(A) 3% NaCl at 80 mL/hr
(B) Discontinue fluids and initiate tolvaptan
(C) Discontinue fluids and initiate 1 L daily oral fluid
restriction
(D) 5% dextrose at 150 mL/hr for 2 hr
(E) None of the above
Copyright Harvard Medical School, 2010. All Rights Reserved.
203
A 25 yr-old male is evaluated because of elevated blood
pressure found on routine exam. In clinic, blood pressure is
160/110 mm Hg. Exam is otherwise normal.
Serum sodium 145 mEq/L
Serum potassium 3.0 mEq/L
Serum chloride 106 mEq/L
Serum bicarbonate 29 mEq/L
Blood urea nitrogen 14 mg/dL
Serum creatinine 0.9 mg/dL
Plasma reninactivity 0.2 ng/mL/hr (NR 1-6)
Plasma aldosterone 2.5 ng/dl (NR 5-20)
24 hr urine cortisol 24 g (NR 10-100)
Urine free cortisol/cortisone ratio 0.4 (NR 0.10.7)
S6
Copyright Harvard Medical School, 2010. All Rights Reserved.
204
Which of the following would be MOST effective at
lowering the blood pressure in this patient:
(A) Amiloride and spironolactone
(B) Amiloride but not spironolactone
(C) Phenoxybenzamine
(D) Renal artery angioplasty
(E) Adrenalectomy
S6
Aldo
Renin
Cushing
Liddle
Licorice
Syndrome apparent
mineralocorticoid excess
Renal artery stenosis
Reninoma
Malignant HTN
Primary aldosteronism
Glucocorticoid-
remediable aldosteronism
High Low
High Low
Hyperaldosteronism
Hypokalemic metabolic alkalosis with hypertension
Copyright Harvard Medical School, 2010. All Rights Reserved.
205
Hyperaldosteronism
Hypokalemic metabolic alkalosis with hypertension
Aldo
Renin
Cushing
Liddle
Licorice
Syndrome apparent
mineralocorticoid excess
Renal artery stenosis
Reninoma
Malignant HTN
Primary aldosteronism
Glucocorticoid-remediable
aldosteronism
High Low
High Low
Urine
cortisol/cortisone ratio
Serum &
urine cortisol
Reni n
Angi otensi nogen
AngI AngII
ACE
Al dosterone
MCR
Corti sol
Corti sone
b-
HSD
1 hyperal do
GRA
RAS
Reni noma
Cushi ngs
Li cori ce
AME
Li ddl es
Hyperal dosteroni sm
Copyright Harvard Medical School, 2010. All Rights Reserved.
206
S6
Which of the following would be MOST effective at
lowering the blood pressure in this patient:
(A) Amiloride and spironolactone
(B) Amiloride but not spironolactone
(C) Phenoxybenzamine
(D) Renal artery angioplasty
(E) Adrenalectomy
Copyright Harvard Medical School, 2010. All Rights Reserved.
207
A 17 yr-old male presents with symmetrical thigh
weakness. He has otherwise been in good health, and
denies using any medications. Blood pressure is 105/70
mm Hg.
Serum sodium 136 mEq/L
Serum potassium 2.8 mEq/L
Serum chloride 101 mEq/L
Serum bicarbonate 27 mEq/L
24 hour urine volume 3 L
Urine sodium 35 mEq/L
Urine potassium 40 mEq/L
Urine chloride 55 mEq/L
S7
What is the most likely diagnosis in this patient:
(A) Gitelmans syndrome
(B) Familial hypokalemic periodic paralysis
(C) Hyperthyroidism
(D) Bulimia
(E) Adrenal adenoma
S7
Copyright Harvard Medical School, 2010. All Rights Reserved.
208
DDX of hypokalemia
Cellular shift GI loss Urinary K wasting
Alkalemia
Insulin
-agonist
Hypokalemic periodic paralysis
Vomiting
Diarrhea
24 hr U
K
>25 mEq
Cryptogenic hypokalemic metabolic
alkalosis
Volume
status/BP
Urine Cl
-
Urine
diuretics
Hyperaldosteronis
m
>40 mEq/L -
Surreptitous
vomiting
Nl or <25 mEq/L -
Diuretic abuse Nl or >40 mEq/L +
Bartter/Gitelman
syndrome
Nl or >40 mEq/L -
Copyright Harvard Medical School, 2010. All Rights Reserved.
209
What is the most likely diagnosis in this patient:
(A) Gitelmans syndrome
(B) Familial hypokalemic periodic paralysis
(C) Hyperthyroidism
(D) Bulimia
(E) Adrenal adenoma
S7
Copyright Harvard Medical School, 2010. All Rights Reserved.
210
A 48 yr-old female was evaluated for several weeks of
fatigue and nausea. Exam revealed a thin, tanned female
with blood pressure of 90/45 mm Hg.
Serum sodium 131 mEq/L
Serum potassium 6.1 mEq/L
Serum chloride 105 mEq/L
Serum bicarbonate 20 mEq/L
Blood urea nitrogen 11 mg/dL
Serum creatinine 0.5 mg/dL
Serum glucose 40 mg/dL
S8
Which of the following tests would be most likely to
establish the diagnosis in this patient:
(A) Plasma reninand aldosterone
(B) Serum cortisol at baseline and after ACTH
(C) Serum C-peptide
(D) Iothalamate GFR determination
(E) Genetic testing of the sodium channel gene,
SCN4A
S8
Copyright Harvard Medical School, 2010. All Rights Reserved.
211
S8
Which of the following tests would be most likely to
establish the diagnosis in this patient:
(A) Plasma renin and aldosterone
(B) Serum cortisol at baseline and after ACTH
(C) Serum C-peptide
(D) Iothalamate GFR determination
(E) Genetic testing of the sodium channel gene,
SCN4A
Copyright Harvard Medical School, 2010. All Rights Reserved.
212
A 65 yr old female with a history of hypertension, diabetes
mellitus, ischemic cardiomyopathyand diabetic nephropathy
presents with a 1 week history of nausea, confusion, blurry vision
and hallucinations. Medications were insulin, carvedilol,
amlodipine, furosemide, and digoxin. EKG showed bradycardia,
peaked T waves and widened QRS complexes.
Serum sodium 134 mEq/L
Serum potassium 5.9 mEq/L
Serum chloride 103 mEq/L
Serum bicarbonate 21 mEq/L
Blood urea nitrogen 18 mg/dL
Serum creatinine 1.5 mg/dL
Serum glucose 135 mg/dL
S9
Which of the following would be the most appropriate
INITIAL treatment for this patient:
(A) Calcium chloride or gluconate
(B) Hydrocortisone
(C) Hemodialysis
(D) Digoxin-specific Fab fragments (Digibind)
(E) None of the above
S9
Copyright Harvard Medical School, 2010. All Rights Reserved.
213
Which of the following would be the most appropriate
INITIAL treatment for this patient:
(A) Calcium chloride or gluconate
(B) Hydrocortisone
(C) Hemodialysis
(D) Digoxin-specific Fab fragments (Digibind)
(E) None of the above
S9
Copyright Harvard Medical School, 2010. All Rights Reserved.
214
A 48 yr old male with hypertension and end-stage renal
disease who had been on hemodialysis for 4 yrs underwent
cadaveric renal transplantation. The operation was
uneventful and the patient was initiated on routine
immunosuppression with anti-thymocyte globulin,
mycophenolate and prednisone.
Post-operatively, the patient remained oliguric and serum
K
+
rose to 6.5 mEq/L. This was treated with 60 g of sodium
polystyrene sulfonate (Kayexalate) and sorbitol as an
enema. 12 hr later, the patient complained of diffuse
abdominal pain. On exam, the abdomen was distended
and tympanitic with hypoactive bowel sounds.
S10
What is the most likely cause of abdominal pain in this
patient?
(A) Ileus due to hypokalemia
(B) Stress-induced gastric ulceration
(C) Mycophenolate side-effect
(D) Adrenal insufficiency
(E) Colonic necrosis
S10
Copyright Harvard Medical School, 2010. All Rights Reserved.
215
S10
What is the most likely cause of abdominal pain in this
patient?
(A) Ileus due to hypokalemia
(B) Stress-induced gastric ulceration
(C) Mycophenolate side-effect
(D) Adrenal insufficiency
(E) Colonic necrosis
Copyright Harvard Medical School, 2010. All Rights Reserved.
216
Intestinal necrosis due to sodium
polystyrene sulfonate (SPS) in sorbitol
Present with abdominal pain, distention or GI bleed
Onset 3 hr-11 days after treatment with SPS (both oral
and enema) for hyperkalemia
Risk factors: age (mean 70 yr), ESRD, post-operative,
critically ill
Colon affected most commonly, but also small intestine
and stomach
Pathology shows SPS crystals and intestinal wall
necrosis resembling ischemia, but with preserved
vessels
Rat studies suggest sorbitol is the culprit
Financial disclosures
No conflict of interest to disclose.
Alan S. L. Yu, M.B., B. Chir.
Copyright Harvard Medical School, 2010. All Rights Reserved.
217
Electrolytes and Acid-Base
Practice for the Boards II
4:00-4:45 pm
Alan S. L. Yu, M.B., B.Chir.
University of Southern California Keck School
of Medicine
19 cases
Financial disclosures
No conflict of interest to disclose.
Alan S. L. Yu, M.B., B. Chir.
Copyright Harvard Medical School, 2010. All Rights Reserved.
218
An 18-year-old female is brought in after a suspected suicidal attempt.
Unidentified pills were found in her pocket. On exam she is obtunded.
BP 101/64 mm Hg, HR 112/min, RR 30/min, T 101.8 C.
Serum sodium 140 mEq/L
Serum potassium 3.8 mEq/L
Serum chloride 99 mEq/L
Serum bicarbonate 15 mEq/L
Blood urea nitrogen 9 mg/dL
Serum creatinine 0.8 mg/dL
Serum glucose 81 mg/dL
Arterial pH 7.35
Arterial PCO
2
26 mm Hg
Case A1
Which of the following is most likely to be helpful in this
patient:
(A) Forced alkaline diuresis
(B) Intravenous ethanol
(C) Insulin drip
(D) Glucagon
(E) Potassium citrate
Case A1
Copyright Harvard Medical School, 2010. All Rights Reserved.
219
pH 7.35, HCO
3
15
Actual PCO
2
=26 (expected: 1.5(15)+8 ~30)
Primary metabolic acidosis and primary respiratory alkalosis
AG =140 - 99 - 15 =27
DAG/DHCO
3
=(27-10)/(24-15) ~2
Anion gap acidosis and metabolic alkalosis (i.e. triple acid-base disorder)
Pt is febrile, tachycardic, tachypneic and had probable drug overdose
Suspect salicylate poisoning
Rx is alkaline diuresis +/- hemodialysis
Case A1
COO
OCOCH
3
COOH
OCOCH
3
-
Blood
pH 7.4
Urine
pH 4.5
Renal tubule
Active secretion by
organic anion transporter
Reabsorption by nonionic
diffusion
-
COO
OCOCH
3
Urine
pH 6.5
Ionized form trapped in
tubule lumen
Alkaline diuresis increases urinary salicylate
excretion by ion trapping
Copyright Harvard Medical School, 2010. All Rights Reserved.
220
Which of the following is most likely to be helpful in this
patient:
(A) Forced alkaline diuresis
(B) Intravenous ethanol
(C) Insulin drip
(D) Glucagon
(E) Potassium citrate
Case A1
Copyright Harvard Medical School, 2010. All Rights Reserved.
221
A 29-year-old female is found unconscious and brought into the
emergency room. BP 105/67 mm Hg. She is comatose and has an
alcoholic fetor.
Serum sodium 129 mEq/L
Serum potassium 3.4 mEq/L
Serum chloride 94 mEq/L
Serum bicarbonate 11 mEq/L
Blood urea nitrogen 11 mg/dL
Serum creatinine 1.6 mg/dL
Serum glucose 72 mg/dL
Serum lactate 1 mmol/L
Serum creatine kinase 20 mU/mL
Serum osmolality 300 mOsm/kg
Arterial pH 7.22
Arterial PCO
2
24 mm Hg
Serum levels of ethanol, ketones by the nitroprusside test, -
hydroxybutyrate, and salicylate were all negative.
Case A2
The most appropriate first step in the management of this
patient is:
(A) Forced alkaline diuresis
(B) Dopamine
(C) Fomepizole
(D) Thiamine
(E) Hemodialysis
Case A2
Copyright Harvard Medical School, 2010. All Rights Reserved.
222
pH 7.22, HCO
3
11
Actual PCO
2
=24 (expected: 1.5(11)+8 =24)
Pure metabolic acidosis
AG =129 - 94 - 11 =24
DAG/DHCO
3
=(24-10)/(24-11) =1.1
Pure anion gap acidosis
Case A2
Serum osmolal gap
Osmolal gap = Measured S
osm
- Calc S
osm
Calculated S
osm
:
2 [Na
+
] +[glucose]/18 +[BUN]/2.8
Calculated serum osmolality =2(129) +72/18 +11/2.8
=266
Osmolar gap =300-266 =34(normal <10)
Copyright Harvard Medical School, 2010. All Rights Reserved.
223
Anion gap
acidosis
Osmolal gap
+ Normal
High
-
Salicylates
Ethanol
Ethylene glycol
Propylene glycol
Methanol
Isopropanol
+
High
Anion and osmolar gap in diagnosis of intoxications
Alcohol
dehydrogenase
Ethylene glycol
Glycolic acid
Glyoxylic acid
Oxalic acid
Ethanol
Fomepizole
Glycine
-hydroxy--
ketoadipate
Pyridoxine
Thiamine
Copyright Harvard Medical School, 2010. All Rights Reserved.
224
GI decontamination
Sodium bicarbonate
Inhibit alcohol dehydrogenase
Hemodialysis (level >50 mg/dL, renal failure or
severe acidosis)
Thiamine
Pyridoxine
Management of (suspected) ethylene
glycol or methanol poisoning
The most appropriate first step in the management of this
patient is:
(A) Forced alkaline diuresis
(B) Dopamine
(C) Fomepizole
(D) Thiamine
(E) Hemodialysis
Case A2
Copyright Harvard Medical School, 2010. All Rights Reserved.
225
A 16-year-old male has a witnessed grand mal seizure and is brought
immediately to the emergency room. He is post-ictal and has vomitus
on his shirt. BP 120/83.
Serum sodium 139 mEq/L
Serum potassium 4.2 mEq/L
Serum chloride 95 mEq/L
Serum bicarbonate 14 mEq/L
Blood urea nitrogen 12 mg/dL
Serum creatinine 1.1 mg/dL
Serum glucose 75 mg/dL
Serum creatine kinase 35 mU/mL
Serum osmolality 289 mOsm/kg
Serum and urine toxicology screen Negative
Urine ketones Negative
Arterial pH 7.30
Arterial PCO
2
33 mm Hg
Case A3
Copyright Harvard Medical School, 2010. All Rights Reserved.
226
The most appropriate first step in the management of this
patient is:
(A) Alkaline diuresis
(B) Ethanol infusion
(C) Fomepizole
(D) Hemodialysis
(E) None of the above
Case A3
pH 7.30, HCO
3
14
Actual PCO
2
=33 (expected: 1.5(14)+8 =29)
Mixed metabolic and respiratory acidosis
AG =139 - 95 - 14 =30
DAG/DHCO
3
=(30-10)/(24-14) =2
Mixed anion gap acidosis and metabolic alkalosis
Calculated serum osmolality =2(139) +75/18 +12/2.8 =286
Osmolar gap =289-286 =3
No osmolar gap
Most likely cause is lactic acidosis from the seizure
Case A3
Copyright Harvard Medical School, 2010. All Rights Reserved.
227
The most appropriate first step in the management of this
patient is:
(A) Alkaline diuresis
(B) Ethanol infusion
(C) Fomepizole
(D) Hemodialysis
(E) None of the above
Case A3
Copyright Harvard Medical School, 2010. All Rights Reserved.
228
A 48-year-old male in the ICU is intubated for respiratory failure due to
severe pneumonia and is paralysed with vecuronium and sedated with
lorazepam. A renal consult is called because of the laboratory finding of
metabolic acidosis.
Serum sodium 143 mEq/L
Serum potassium 4.9 mEq/L
Serum chloride 109 mEq/L
Serum bicarbonate 16 mEq/L
Blood urea nitrogen 12 mg/dL
Serum creatinine 1.1 mg/dL
Serum glucose 194 mg/dL
Serum lactate 8 mmol/L
Serum osmolality 312 mOsm/kg
Arterial pH 7.40
Arterial PCO
2
24 mm Hg
Case A4
The most appropriate next step in the management of
this patient is:
(A) Dopamine
(B) Dobutamine
(C) Abdominal computed tomography
(D) Discontinue vecuronium
(E) Discontinue lorazepam
Case A4
Copyright Harvard Medical School, 2010. All Rights Reserved.
229
pH 7.40, HCO
3
16, PCO
2
24
Mixed metabolic acidosis and respiratory alkalosis
AG =143 - 109 - 16 =18
DAG/DHCO
3
=(18-10)/(24-16) =1
Pure anion gap acidosis
Calculated serum osmolality =2(143) +194/18 +12/2.8 =301
Osmolar gap =312 - 301 =11
Osmolar gap
DDX is AKA, ethylene glycol, methanol, propylene glycol
Case A4
Common IV drugs containing
propylene glycol
Arroliga 2004 Crit Care Med, 32(8):1709
Drug Propylene glycol (% v/v)
Lorazepam, 2 mg/mL 80
Phenobarbital, 30-130 mg/mL 68-75
Diazepam, 5 mg/mL 40
Pentobarbital, 50 mg/mL 20-40
Phenytoin, 50 mg/mL 40
TMP-SMX, 16:80 mg/mL 40
Nitroglycerin, 5 mg/mL 35
Esmolol, 250 mg/mL 25
Copyright Harvard Medical School, 2010. All Rights Reserved.
230
The most appropriate next step in the management of
this patient is:
(A) Dopamine
(B) Dobutamine
(C) Abdominal computed tomography
(D) Discontinue vecuronium
(E) Discontinue lorazepam
Case A4
Copyright Harvard Medical School, 2010. All Rights Reserved.
231
A 78 year-old female has been in the ICU for 2 weeks because of
cholecystitis, gram negative sepsis, hypotension, and ARDS requiring
mechanical ventilation. A renal consult is called because of a
persistently low serum bicarbonate. Her medications include imipenem,
acetaminophen, dopamine, and noradrenaline.
Serum sodium 134 mEq/L
Serum potassium 3.8 mEq/L
Serum chloride 99 mEq/L
Serum bicarbonate 18 mEq/L
Blood urea nitrogen 14 mg/dL
Serum creatinine 1.4 mg/dL
Serum glucose 187 mg/dL
Serum lactate 1.6 mmol/L
Arterial pH 7.27
Arterial PCO
2
41 mm Hg
Case A5
Which of the following are appropriate in the management
of this patients acid-base disturbance:
(A) CT scan to rule out ischemic bowel
(B) Discontinue acetaminophen
(C) Increase minute ventilation
(D) A and C
(E) B and C
Case A5
Copyright Harvard Medical School, 2010. All Rights Reserved.
232
pH 7.27, HCO
3
18, PCO
2
41
Actual PCO
2
=41 (expected: 1.5(18)+8 =35)
Mixed metabolic and respiratory acidosis
AG =134 - 99 - 18 =17
DAG/DHCO
3
=(17-10)/(24-18) ~1
Pure anion gap acidosis
Case A5
Pyroglutamic acidemia (5-oxoprolinemia)
Glutathione
Cysteine Glutamate
Pyroglutamate
g-glutamyl
cysteine
g-glutamyl cysteine
synthetase
Glutathione
synthetase
Acetaminophen
Sepsis
5-oxoprolinase
Flucloxacillin
Copyright Harvard Medical School, 2010. All Rights Reserved.
233
Pyroglutamic acidemia (5-oxoprolinemia)
18 cases described so far
16 female
9 sepsis, 5 SIRS
14 liver dysfunction, 3 renal failure
12 treated with acetaminophen, 2 with
flucloxacillin
All have anion gap acidosis with elevated blood
and urine pyroglutamic acid
Discontinue acetaminophen, flucloxacillin,
supportive care
Case A5
Which of the following are appropriate in the
management of this patients acid-base disturbance:
(A) CT scan to rule out ischemic bowel
(B) Discontinue acetaminophen
(C) Increase minute ventilation
(D) A and C
(E) B and C
Copyright Harvard Medical School, 2010. All Rights Reserved.
234
A 39 year-old African-American female with AIDS and recently
diagnosed liver failure was admitted for vague abdominal discomfort
and nausea. Her only medications are stavudine, didanosine, tenofovir
and ritonavir. Exam: BP 98/56 mm Hg, HR 83/min, afebrile, cachectic.
Serum sodium 132 mEq/L
Serum potassium 4.9 mEq/L
Serum chloride 97 mEq/L
Serum bicarbonate 9 mEq/L
Blood urea nitrogen 18 mg/dL
Serum creatinine 1.7 mg/dL
Serum glucose 74 mg/dL
Serum lactate 12.8 mmol/L
AST 59 U/L (NR 10-40)
ALT 64 U/L (NR 10-40)
Prothrombin time 18 s, INR =1.5
Arterial pH 7.22
Arterial PCO
2
19 mm Hg
Case A6
Copyright Harvard Medical School, 2010. All Rights Reserved.
235
What is the most likely cause of the acid-base
disturbance:
(A) Seizure
(B) Drug-induced lactic acidosis
(C) Ischemic bowel
(D) Occult sepsis
(E) D-lactic acidosis
Case A6
Type B lactic acidosis
Lactic acidosis that is not associated with
impaired tissue oxygenation
Seizures
Malignancy (acute leukemia, lymphoma, solid
tumor with liver metastases)
Liver failure
Vitamin deficiency (thiamine, riboflavin)
Drugs/toxins (ethanol, methanol, ethylene glycol,
propylene glycol, salicylates, metformin, NRTI,
isoniazid)
Copyright Harvard Medical School, 2010. All Rights Reserved.
236
Inhibition of mitochondrial DNA polymerase-g
DDI and stavudine most common cause
Often associated with hepatotoxicity
2 mth - 2 yr after start of Rx
Risk factors: Female gender, low GFR, low CD4
N/V, abdo pain, fatigue, wt loss
Can be induced by drug interaction (tenofovir increases
AUC of DDI by 50%)
30-60% mortality
Type B lactic acidosis 2 to nucleoside
reverse transcriptase inhibitors (NRTI)
What is the most likely cause of the acid-base disturbance:
(A) Seizure
(B) Drug-induced lactic acidosis
(C) Ischemic bowel
(D) Occult sepsis
(E) D-lactic acidosis
Case A6
Copyright Harvard Medical School, 2010. All Rights Reserved.
237
A 28-year-old Asian female with an erythematous malar rash is referred
to renal clinic because of abnormal laboratory electrolyte values.
Laboratory studies:
Serum sodium 138 mEq/L
Serum potassium 3.2 mEq/L
Serum chloride 114 mEq/L
Serum bicarbonate 15 mEq/L
Blood urea nitrogen 7 mg/dL
Serum creatinine 0.5 mg/dL
Urine pH 6.5
Urine sodium 25 mEq/L
Urine potassium 35 mEq/L
Urine chloride 34 mEq/L
Case A7
Copyright Harvard Medical School, 2010. All Rights Reserved.
238
The most appropriate management for this patient is:
(A) Loperamide
(B) Hydrochlorothiazide
(C) Cyclophosphamide
(D) Potassium citrate, 1-3 mEq/kg/day
(E) Sodium bicarbonate, 5-15 mEq/kg/day
Case A7
Non-gap (hyperchloremic)
metabolic acidosis
Lower GI bicarbonate loss
Renal tubular acidosis
Dilutional acidosis
Copyright Harvard Medical School, 2010. All Rights Reserved.
239
Diagnosis of RTA
Determination of the urine anion gap
Urine anion gap = [Na
+
] + [K
+
] - [Cl
-
]
Normal < 0
Urine anion gap
= Measured cations - Measured anions
= (Na
+
+K
+
) - (Cl
-
+HCO
3
-
)
= Unmeasured anions - unmeasured cations
Sulfate
Phosphate
Bicarbonate
Organic anions
Calcium
Magnesium
NH
4
+
Copyright Harvard Medical School, 2010. All Rights Reserved.
240
Urine anion gap
In the setting of acidemia, urine HCO
3
-
should be negligible, and urine NH
4
+
should be HIGH
=> Urine AG should be LOW
A
m
m
o
n
i
u
m

e
x
c
r
e
t
i
o
n
Urine anion gap
-50 0 +50 +100
90
60
30
Diarrhea
Distal RTA
-100
Batlle, N Engl J Med. 1988; 318: 594
Copyright Harvard Medical School, 2010. All Rights Reserved.
241
Acid excretion mechanisms in renal tubule
Type 2
Type 1
Type 4
Hyperkalemic
distal
Diarrhea
Proximal
RTA
Distal RTA
Type I Type 4 Hyperkalemic
distal
Serum K
+

Urine AG Negative Variable Positive
Urine pH Variable >5.5 <5.5
Other
Fanconi
syndrome
Nephro-
calcinosis
Clinical features of RTA
Copyright Harvard Medical School, 2010. All Rights Reserved.
242
The most appropriate management for this patient is:
(A) Loperamide
(B) Hydrochlorothiazide
(C) Cyclophosphamide
(D) Potassium citrate, 1-3 mEq/kg/day
(E) Sodium bicarbonate, 5-15 mEq/kg/day
Case A7
Copyright Harvard Medical School, 2010. All Rights Reserved.
243
A 54-year-old male with a 12 year history of Type 2 diabetes mellitus
maintained on insulin, and proliferative diabetic retinopathy, is referred
because of proteinuria.
Laboratory studies:
Serum sodium 140 mEq/L
Serum potassium 6.0 mEq/L
Serum chloride 112 mEq/L
Serum bicarbonate 19 mEq/L
Blood urea nitrogen 27 mg/dL
Serum creatinine 1.6 mg/dL
Serum glucose 206 mg/dL
Urinalysis shows 3+proteinuria.
Case A8
The most likely cause of this patients metabolic acidosis is:
(A) Diarrhea due to diabetic autonomic neuropathy
(B) Diabetic ketoacidosis
(C) Chronic renal failure
(D) Type I renal tubular acidosis
(E) Type IV renal tubular acidosis
Case A8
Copyright Harvard Medical School, 2010. All Rights Reserved.
244
Diarrhea
Proximal
RTA
Distal RTA
Type I Type 4 Hyperkalemic
distal
Serum K
+

Urine AG Negative Variable Positive
Urine pH Variable >5.5 <5.5
Other
Fanconi
syndrome
Nephro-
calcinosis
Clinical features of RTA
Type IV RTA (hyporeninemic
hypoaldosteronism)
Hyperkalemia (disproportionate to level of GFR)
Non-gap metabolic acidosis with normal urine acidifying
ability
Mild CRF
Often underlying tubulointerstitial disease:
- Diabetes mellitus
- SLE, obstruction, myeloma/amyloid, HIV etc.
- NSAIDs
Copyright Harvard Medical School, 2010. All Rights Reserved.
245
The most likely cause of this patients metabolic acidosis is:
(A) Diarrhea due to diabetic autonomic neuropathy
(B) Diabetic ketoacidosis
(C) Chronic renal failure
(D) Type I renal tubular acidosis
(E) Type IV renal tubular acidosis
Case A8
Copyright Harvard Medical School, 2010. All Rights Reserved.
246
A 22-year-old male with HIV infection was started on HAART three
months ago. His medications are didanosine, tenofovirand lopinavir.
Serum sodium 141 mEq/L
Serum potassium 3.4 mEq/L
Serum chloride 115 mEq/L
Serum bicarbonate 18 mEq/L
Serum glucose 76 mg/dL
Blood urea nitrogen 7 mg/dL
Serum creatinine 0.4 mg/dL
Serum calcium 8.6 mg/dL
Serum phosphorus 0.9 mg/dL
Serum uric acid 1.7 mg/dL
Urinalysis: pH 6.0, specific gravity 1.015, 3+glucose, trace protein
Urine phosphorus 25 mg/dL
Urine creatinine 38 mg/dL
Case A9
The most likely cause of this patient acid-base
disturbance is:
(A) HIV-related diarrhea
(B) D-lactic acidosis
(C) Didanosine-induced lactic acidosis
(D) Hepatic failure-associated renal tubular acidosis
(E) Tenofovir toxicity
Case A9
Copyright Harvard Medical School, 2010. All Rights Reserved.
247
Serum bicarbonate is 18 mEq/L
Anion gap =141 - 115 - 18 =8
Probably non-gap metabolic acidosis
Urine pH 6, serum K 3.4
Inappropriately alkaline urine and hypokalemiasuggests Type I
or II renal tubular acidosis
Case A9
Diarrhea
Proximal
RTA
Distal RTA
Type I Type 4 Hyperkalemic
distal
Serum K
+

Urine AG Negative Variable Positive
Urine pH Variable >5.5 <5.5
Other
Fanconi
syndrome
Nephro-
calcinosis
Clinical features of RTA
Copyright Harvard Medical School, 2010. All Rights Reserved.
248
Diagnosis of renal phosphate
wasting
F
E
PO
4
=
UPO
4
x SCr
SPO
4
x UCr
F
E
PO
4
=(25 x 0.4)/(0.9 x 38) =29% (normal 5-15%)
Renal phosphate wasting
Glycosuriadespite normoglycemia, hypouricemia
Fanconi syndrome
Cause:
Tenofovir
Case A9
Copyright Harvard Medical School, 2010. All Rights Reserved.
249
Acyclic nucleoside phosphonates: adefovir (22-50%),
cidofovir (12% ARF, 1% Fanconi), tenofovir (2-4%)
Transported into prox. tubule by organic anion
transporter (OAT1), causing mitochondrial toxicity
Both Fanconi syndrome and ARF (due to ATN)
Can be induced by drug interaction (most pts with
tenofovir toxicity were on ritonavir)
Can be induced by renal insufficiency (all renally
excreted)
Can occur 3 wk to 18 mth after start of Rx
Fanconi syndrome & ARF 2 to nucleotide-
analog reverse transcriptase inhibitors
The most likely cause of this patient acid-base
disturbance is:
(A) HIV-related diarrhea
(B) D-lactic acidosis
(C) Didanosine-induced lactic acidosis
(D) Hepatic failure-associated renal tubular acidosis
(E) Tenofovir toxicity
Case A9
Copyright Harvard Medical School, 2010. All Rights Reserved.
250
A10-14
For each patient with hyperchloremic metabolic acidosis
described below, select the most likely cause of the
acidosis (A-E).
(A) Proximal (Type II) renal tubular acidosis
(B) Classic distal (Type I) renal tubular acidosis
(C) Hyporeninemic hypoaldosteronism(Type IV renal
tubular acidosis)
(D) Gastrointestinal bicarbonate loss
(E) Toluene intoxication
Copyright Harvard Medical School, 2010. All Rights Reserved.
251
38-year-old woman with dry eyes and polyarthritis. The
serum potassium is 3.4 mEq/L, the urine pH is 6.5 and the
urine sodium is 28 mEq/L, potassium 50 mEq/L and
chloride 57 mEq/L.
Case A10
(A) Proximal (Type II) renal tubular acidosis
(B) Classic distal (Type I) renal tubular acidosis
(C) Hyporeninemic hypoaldosteronism(Type IV renal
tubular acidosis)
(D) Gastrointestinal bicarbonate loss
(E) Toluene intoxication
Copyright Harvard Medical School, 2010. All Rights Reserved.
252
56-year-old Hispanic male with Type II diabetes mellitus,
diabetic nephropathy, and chronic renal insufficencywith a
serum Cr of 2.3 mg/dL. He is unable to tolerate
angiotensin-converting enzyme inhibitors due to
hyperkalemia. The urine pH is 5.0 and the urine sodium is
43 mEq/L, potassium 13 mEq/L and chloride 41 mEq/L.
Case A11
(A) Proximal (Type II) renal tubular acidosis
(B) Classic distal (Type I) renal tubular acidosis
(C) Hyporeninemic hypoaldosteronism(Type IV renal
tubular acidosis)
(D) Gastrointestinal bicarbonate loss
(E) Toluene intoxication
Copyright Harvard Medical School, 2010. All Rights Reserved.
253
21-year-old college senior with a history of substance
abuse. The serum potassium is 1.9 mEq/L and the urine
sodium is 42 mEq/L, potassium 48 mEq/L and chloride 12
mEq/L. The urine sediment shows numerous needle-
shaped crystals.
Case A12
(A) Proximal (Type II) renal tubular acidosis
(B) Classic distal (Type I) renal tubular acidosis
(C) Hyporeninemic hypoaldosteronism (Type IV renal
tubular acidosis)
(D) Gastrointestinal bicarbonate loss
(E) Toluene intoxication
Copyright Harvard Medical School, 2010. All Rights Reserved.
254
Toluene intoxication
Due to inhalation of toluene which is used as an
organic solvent in glues, paint thinners etc.
1. Non-anion gap metabolic acidosis
2. Hypokalemia
3. Severe hypophosphatemia
Hippuric acid
Distal RTA (esp.
chronic use)
26-year-old woman with osteosarcoma of the humerus,
undergoing chemotherapy with ifosfamide. The serum
potassium is 2.3 mEq/L, glucose 97 mg/dL, phosphate 1.5
mg/dL. The urinalysis shows pH 7.0, 1+protein, 2+glucose.
The urine sodium is 27 mEq/L, potassium 48 mEq/L and
chloride 56 mEq/L.
Case A13
(A) Proximal (Type II) renal tubular acidosis
(B) Classic distal (Type I) renal tubular acidosis
(C) Hyporeninemic hypoaldosteronism(Type IV renal
tubular acidosis)
(D) Gastrointestinal bicarbonate loss
(E) Toluene intoxication
Copyright Harvard Medical School, 2010. All Rights Reserved.
255
24-year-old man with Type I diabetes mellitus complicated
by diabetic nephropathy, chronic renal failure with a serum
Cr of 1.8 mg/dL, potassium 3.0 mg/dL, autonomic
neuropathy and chronic diarrhea. The urine sodium is 10
mEq/L, potassium 14 mEq/L and chloride 36 mEq/L.
Case A14
(A) Proximal (Type II) renal tubular acidosis
(B) Classic distal (Type I) renal tubular acidosis
(C) Hyporeninemic hypoaldosteronism(Type IV renal
tubular acidosis)
(D) Gastrointestinal bicarbonate loss
(E) Toluene intoxication
Copyright Harvard Medical School, 2010. All Rights Reserved.
256
A 22-year-old female presents with polyuria, constipation and fatigue.
She denies taking any prescription or over-the-counter medication. BP
96/54. Exam shows only mild bilateral parotid enlargement.
Serum sodium 133 mEq/L
Serum potassium 2.9 mEq/L
Serum chloride 82 mEq/L
Serum bicarbonate 38 mEq/L
Blood urea nitrogen 5 mg/dL
Serum creatinine 0.6 mg/dL
Urine sodium 15 mEq/L
Urine potassium 17 mEq/L
Urine chloride <5 mEq/L
Urinalysis: pH 4.5, 1+ketones
Case A15
Copyright Harvard Medical School, 2010. All Rights Reserved.
257
What is the most likely cause of this patients hypokalemia
and alkalosis:
(A) Liddle's syndrome
(B) Gitelmansyndrome
(C) Conn's syndrome
(D) Diuretic abuse
(E) Bulimia
Case A15
Cryptogenic metabolic alkalosis
Volume
status
Urine Cl
-
Urine
diuretics
Hyperaldo > 40 mEq/L -
Surreptitous
vomiting
Nl or < 25 mEq/L -
Diuretic abuse Nl or > 40 mEq/L +
Bartters
syndrome
Nl or > 40 mEq/L -
Copyright Harvard Medical School, 2010. All Rights Reserved.
258
Time-dependent change in urine lytes
in vomiting
Na
+
K
+
Cl
-
HCO
3
-
pH
Early > 6. 5
Late < 5. 5
What is the most likely cause of this patients
hypokalemia and alkalosis:
(A) Liddle's syndrome
(B) Gitelmansyndrome
(C) Conn's syndrome
(D) Diuretic abuse
(E) Bulimia
Case A15
Copyright Harvard Medical School, 2010. All Rights Reserved.
259
A 48 year-old male with congestive cardiac failure is seen in clinic for
dyspnea and orthopnea. He has known systolic dysfunction with an
ejection fraction of 20% and is maintained on aspirin, carvedilol,
furosemide, metolazone, candesartan and digoxin. BP 109/74. Exam
shows bilateral diffuse inspiratorycrackles in the lung fields, and 1+
dependent pitting edema.
Serum sodium 128 mEq/L
Serum potassium 3.5 mEq/L
Serum chloride 87 mEq/L
Serum bicarbonate 31 mEq/L
Blood urea nitrogen 64 mg/dL
Serum creatinine 2.6 mg/dL
Arterial pH 7.51
Arterial PCO
2
42 mm Hg
Arterial PO
2
68 mm Hg
Case A16
Copyright Harvard Medical School, 2010. All Rights Reserved.
260
All of the following are contributing to this patients
alkalemia EXCEPT:
(A) Metolazone
(B) Candesartan
(C) Hypokalemia
(D) Secondary hyperaldosteronism
(E) Renal failure
Case A16
Reni n
Angi otensi nogen
AngI AngII
ACE
Al dosterone
MCR
Na
+
del i very:
Loop & thiazide
diuretics
Secondary
hyperal dosteroni sm
K
+
K
+
H
+
Copyright Harvard Medical School, 2010. All Rights Reserved.
261
All of the following are contributing to this patients
alkalemia EXCEPT:
(A) Metolazone
(B) Candesartan
(C) Hypokalemia
(D) Secondary hyperaldosteronism
(E) Renal failure
Case A16
Copyright Harvard Medical School, 2010. All Rights Reserved.
262
A 16-year-old male presents with muscle weakness. He has had
several prior episodes that have resolved spontaneously. He denies
vomiting and is taking no medications. Physical exam is normal.
Serum sodium 136 mEq/L
Serum potassium 2.8 mEq/L
Serum chloride 90 mEq/L
Serum bicarbonate 33 mEq/L
Blood urea nitrogen 13 mg/dL
Serum creatinine 0.9 mg/dL
Urine sodium 33 mEq/L
Urine potassium 48 mEq/L
Urine chloride 72 mEq/L
Case A17
All of these disorders could be the cause of this patients
alkalosis EXCEPT:
(A) Bartter syndrome
(B) Gitelmansyndrome
(C) Loop diuretic abuse
(D) Thiazidediuretic abuse
(E) Surreptitious vomiting
Case A17
Copyright Harvard Medical School, 2010. All Rights Reserved.
263
Cryptogenic metabolic alkalosis
Volume
status
Urine Cl
-
Urine
diuretics
Hyperaldo > 40 mEq/L -
Surreptitous
vomiting
Nl or < 25 mEq/L -
Diuretic abuse Nl or > 40 mEq/L +
Bartters
syndrome
Nl or > 40 mEq/L -
All of these disorders could be the cause of this patients alkalosis
EXCEPT:
(A) Bartter syndrome
(B) Gitelman syndrome
(C) Loop diuretic abuse
(D) Thiazide diuretic abuse
(E) Surreptitious vomiting
Case A17
Copyright Harvard Medical School, 2010. All Rights Reserved.
264
A 31 year-old stockbroker with a several month history of acid reflux
and heartburn was admitted with nausea, vomiting, abdominal pain,
and confusion. He avoids seeing doctors and takes only over-the-
counter medications including ibuprofen, omeprazole, calcium
carbonate and sodium bicarbonate. On exam he appears dehydrated
and his abdomen is mildly tender.
Serum sodium 147 mEq/L
Serum potassium 5.1 mEq/L
Serum chloride 102 mEq/L
Serum bicarbonate 38 mEq/L
Blood urea nitrogen 68 mg/dL
Serum creatinine 4.1 mg/dL
Serum calcium 12.1 mg/dL
Serum albumin 4.5 g/dL
Case A18
Copyright Harvard Medical School, 2010. All Rights Reserved.
265
The most likely diagnosis in this patient is:
(A) Vomiting-induced metabolic alkalosis
(B) Hyporeninemic hypoaldosteronism
(C) Primary hyperparathyroidism
(D) Milk-alkali syndrome
(E) Acute pancreatitis
Case A18
Metabolic alkalosis +hypercalcemia +ARF
Most commonly due to CaCO
3
ingestion
Synergistic effects:
Alkalosis inhibits renal Ca
2+
excretion
Hypercalcemia inhibits renal bicarbonate
excretion
Hypercalcemia causes ARF
Renal failure inhibits both Ca
2+
and bicarbonate
excretion
Milk-alkali syndrome
Copyright Harvard Medical School, 2010. All Rights Reserved.
266
The most likely diagnosis in this patient is:
(A) Vomiting-induced metabolic alkalosis
(B) Hyporeninemic hypoaldosteronism
(C) Primary hyperparathyroidism
(D) Milk-alkali syndrome
(E) Acute pancreatitis
Case A18
Copyright Harvard Medical School, 2010. All Rights Reserved.
267
A 54-year-old smoker with chronic obstructive pulmonary disease,
diabetes and chronic renal failure is admitted with pneumonia.
Overnight he becomes progressively more obtunded; arterial blood gas
shows a PCO
2
of 78 mm Hg and he is intubatedand mechanically
ventilated. A renal consult is called 3 days later because of persistent
alkalemiaand failure to wean from the ventilator.
Serum sodium 135 mEq/L
Serum potassium 4.6 mEq/L
Serum chloride 82 mEq/L
Serum bicarbonate 43 mEq/L
Blood urea nitrogen 38 mg/dL
Serum creatinine 2.4 mg/dL
Arterial pH 7.64
Arterial PCO
2
41 mm Hg
Case A19
All of the following would correct the metabolic alkalosis in
this patient EXCEPT:
(A) Acetazolamide
(B) Intravenous saline
(C) Decreasing the tidal volume
(D) Hemodialysis
(E) Argininehydrochloride
Case A19
Copyright Harvard Medical School, 2010. All Rights Reserved.
268
Post-hypercapnic alkalosis
Chronic CO
2
retention
Compensatory renal HCO
3
-
retention
Mechanical ventilation
Normalization of PCO
2
Excretion of excess HCO
3
-
if.....
Sufficient time
Adequate renal function
Euvolemia
Case A19
All of the following would correct the metabolic alkalosis
in this patient EXCEPT:
(A) Acetazolamide
(B) Intravenous saline
(C) Decreasing the tidal volume
(D) Hemodialysis
(E) Argininehydrochloride
Copyright Harvard Medical School, 2010. All Rights Reserved.
269
Financial disclosures
No conflict of interest to disclose.
Alan S. L. Yu, M.B., B. Chir.
Copyright Harvard Medical School, 2010. All Rights Reserved.
270
Take Home Messages
David B. Mount, MD
Renal Division, BWH
Renal Division,
VA Boston Healthcare System
Disclosures
Consultant (hyponatremia) Salix, Otsuka
Speaker bureau, 2005-2008 Astellas
(conivaptan)
Copyright Harvard Medical School, 2010. All Rights Reserved.
271
OVERVIEW
Key concepts in acid-base and electrolyte
disorders will not cover alkalosis and
hypernatremia
Key recent papers
Newly described disorders
The weird and wonderful hereditary
disorders that appear on the boards
Hyponatremia
Copyright Harvard Medical School, 2010. All Rights Reserved.
272
4
8
12
280 290 300 310 270 260
0
Hypovolemic
Euvolemic
Hypervolemic
Plasma osmolality (mOsm/Kg)
P
l
a
s
m
a

A
V
P

(
p
g
/
m
L
)
Volume Status and Vasopressin Release
What About Thirst??
Thirst is stimulated
over same range of
osmolalityas AVP,
via activation of
central osmoreceptors
Typically need intake
of H
2
O to generate
hyponatremia
4
8
12
280 290 300 310 270 260
0
Plasma osmolality (mOsm/Kg)
P
l
a
s
m
a

A
V
P

(
p
g
/
m
L
)
Increasing Thirst
Copyright Harvard Medical School, 2010. All Rights Reserved.
273
Leftward Shift of the Thirst Response
in SIADH, i.e. Thirst Also Abnormal
A-H: SIADH pts
Grey: Controls
Smith et al, AJP Endocrinol, 2004
Diagnostic Algorithm
Kumar and Berl, Atlas of Diseases of the Kidney, 1999
Copyright Harvard Medical School, 2010. All Rights Reserved.
274
Spot Urinary Na
+
and the D
x
of
Hypovolemic Hyponatremia
Urine Na
+
in non-
edematous patients with
hyponatremia, who do or
do not respond to saline
infusion with serum Na
+
.
Na
+
-avid patients have
vasopressin due to
hypovolemia
suppressed by normal
saline infusion.
Am J Med 83: 905-908, 1987
Malignancy
SIADH
Carcinoma
Lung - ,
mesothelioma
Oropharynx
GI tract - stomach,
duodenum, pancreas
GU tract - ureter,
bladder, prostate
Endocrine thymoma
Lymphomas
Sarcomas
small cell CA
Central Nervous System
Infections
Encephalitis
Meningitis
Abscess
Bleeding, masses
Subdural hematoma
Subarachnoid bleed
Traumatic brain injury
Cavernous sinus thromb
Other
Multiple sclerosis
Guillain-Barre syndrome
Drugs
Ifosmamide
Cyclophosphamide
[NSAIDS]
[Chlorpopramide]
AVP analogs
DDAVP
Vasopressin
Ocytocin
SSRIs
Tricyclic antidepressants
Clofibrate
Carbamazepine
Vincristine
Nicotine
Narcotics
Anti-psychotics
Pulmonary Disorders
Infections
Bacterial pneumonia
Viral pneumonia
Pulmonary abscess
Tuberculosis
Aspergillosis
Asthma
Cystic fibrosis
Respiratory failure
with positive pressure ventilation
Miscellaneous
Pain
Postoperative state
Nausea
Endurance excercise
Hereditary (NSIAD)
Idiopathic
Copyright Harvard Medical School, 2010. All Rights Reserved.
275
Patterns of AVP Release in SIADH
A) Unregulated, erratic
B) Baseline increase,
normal osmotic response
C) Reset osmostat
D) Suppressed AVP
Robertson, Am J Med. 2006;119(7 Suppl 1):S36-42
Question #1 Two Boys with
Hyponatremia
Characteristic Patient 1 Patient 2 Controls
Age 3.0 2.5 -
BP 80-120/34-92 93-118/37-57
MRI brain Normal Normal -
Serum: Na 123 118 134-43
K 4.6 4.7 3.4-4.9
Osm 252 247 285-293
BUN/creat 5/0.3 3/0.3
Aldosterone 10 24 6-68
Vasopressin <1 <1 1-13.3
TSH 0.77 4.58 1.7-9.1
Free thyroxine 1.09 0.97 0.8-1.8
Cortisol 13.3 10.3 4-20
Urine: Na 35 75
Osm 284 390

Feldman et al, NEJM, 2005
Copyright Harvard Medical School, 2010. All Rights Reserved.
276
How Would You Characterize
Their Clinical Syndrome?
A) SIADH with increased circulating oxytocin
B) Mineralocorticoid resistance and hypovolemic
hyponatremia
C) Co-existent central DI and renal salt-wasting
D) Normotonic hyponatremia vasopressin levels
are appropriately suppressed
E) Increased renal sensitivity to vasopressin
vasopressin levels are appropriately
suppressed
How Would You Characterize
Their Clinical Syndrome?
E) Increased renal sensitivity to vasopressin
vasopressin levels are appropriately
suppressed
Copyright Harvard Medical School, 2010. All Rights Reserved.
277
The Patients Have Different Mutations
in the V2 Vasopressin Receptor (V2R)
Fel dman et al , N Engl J Med. 2005 May 5;352(18):1884-90
R137 C137 R137L
Gain of Function in the V2R in Nephrogeni c
Syndrome of Inappropriate Antidiuresi s
Copyright Harvard Medical School, 2010. All Rights Reserved.
278
NSAID Mutations Target a Critical Residue in
the V2 Receptor, Also Invol ved in
Hereditary Nephrogenic DI
Patterns of AVP Release in SIADH
A) Unregulated, erratic
B) Baseline increase,
normal osmotic response
C) Reset osmostat
D) Suppressed AVP
Robertson, Am J Med. 2006;119(7 Suppl 1):S36-42
Copyright Harvard Medical School, 2010. All Rights Reserved.
279
Question #2
A 56 year-old male veteran is seen in Renal consultation for recurrent
hyponatremia, during multiple admissions to the local VA hospital.
PMH includes atrial flutter, chronic alcoholism (beer), cocaine abuse,
depression, and bipolar disorder. Current medications include
bupropion, lamotrigine, and quetiapine
ROS is negative. He lives alone.
BP is 100/60, HR 72 without orthostatic change.
Physical exam is unremarkable.
Laboratory Studies:
Na
+
124 BUN 6 mg/dL
K
+
4.0 creat 0.9 mg/dL
S Osm 264 mOsm/kg Uosm 161 mOsm/Kg
Urine Na <20 TSH 1.7 iU/ml (.3-5.5)
Cortisol 18.50 g/dL
A. Bupropion
B. Primary adrenal failure
C. Secondary adrenal failure
D. Hypovolemic hyponatremia
E. Low dietary solute intake
Which of the following is the most likely
cause of his hyponatremia?
Copyright Harvard Medical School, 2010. All Rights Reserved.
280
E. Low dietary solute intake
Beer potomania 6-8 beers/day
Which of the following is the most likely
cause of his hyponatremia?
Beer Potomania
Due to marked in urinary solutes, limiting
urinary free water excretion.
Beer, typically the sole nutrient in these pts, is
very low in protein and salt content, containing
only 1-2 millimole per liter of Na
+
.
Thought to have very high risk of overcorrection
and osmotic demyelination.
Urine Na <20, urine Osms usually 100-200
mOsm/kg, rather than <100
Copyright Harvard Medical School, 2010. All Rights Reserved.
281
Free Water Clearance and Solute Excretion
Thaler et al, Am J Kidney Dis 1998;31(6):1028-1031
Causes of Acute Hyponatremia
Iatrogenic
Postoperative premenopausal women
Hypotonic fluids with cause of vasopressin
Glycine irrigant TURP, uterine surgery
Colonoscopy preparation
Recent institution of thiazides
Polydipsia
Ecstasy ingestion
Exercise induced
Multifactorial, e.g. thiazide and polydipsia
Copyright Harvard Medical School, 2010. All Rights Reserved.
282
Pulmonary Edema After Ecstasy Ingestion
Nguyen et al, Nature ClinPrac Nephrology, 2006
20 year-old female
college student
[Na
+
] 117 mM
Is Chronic Hyponatremia
Really That Asymptomatic?
Case-control series of 122 consecutive
asymptomatic hyponatremia patients.
Na ranged from 115-135
Prevalence of falls was 21.3% versus 5.4
% in case controls (p < 0.001).
Fall was often reason for admission.
Subtle gait and attention defects in a
separate cohort of hypoNa patients.
Renneboog, et al, Am J Med, 2006
Copyright Harvard Medical School, 2010. All Rights Reserved.
283
Falls and the Magnitude of Hyponatremia:
Occurrence at All Levels
Renneboog, et al, Am J Med, 2006
Are Falls and Other Symptoms
Common in Chronic Hyponatremia?
223 cases of thiazide-associated
hyponatremia, 1996-2002
Symptoms included malaise and lethargy
(49%), dizzy spells (47%), vomiting (35%),
confusion or obtundation(17%), and falls
(17%).
Confusion or vomiting much more likely at
Na </= 115.
Chow et al, J Natl Med Assoc, 2004
Copyright Harvard Medical School, 2010. All Rights Reserved.
284
J Bone Miner Res. 2010 Mar;25(3):554-63.
Treatment of Hyponatremia
Management of acute, symptomatic
hyponatremia
Management of chronic hyponatremia
Fluid restriction
Lasix and salt tabs - countercurrent
mechanism and repletion of excreted NaCl
Democlocycline - V
2
R response
Vasopressin antagonists
Copyright Harvard Medical School, 2010. All Rights Reserved.
285
We Hopefully Agree
Treatment of acute, symptomatic
hyponatremiacan be life-saving
Management should include:
hypertonic saline
ABG, CXR, and CNS imaging (if available
but do NOT delay therapy)
supplemental O
2
prn
loop diuretic (R
x
of pulmonary edema and
countercurrent mechanism)
Rate of Correction
Acute (<48 hr) symptomatic hyponatremia
1-2 mEq/l per hour, correcting by 4-6 mEqu/L and/or
until correction of severe symptoms, then chronic
criteria
hypertonic saline, plus furosemide (particularly if in
pulmonary edema)
Close attention to oxygenation
Chronic hyponatremia (>48 hr)
0.5 mEq/l per hour, 10 mEqu/first 24 hours, and
18/first 48 hours
Copyright Harvard Medical School, 2010. All Rights Reserved.
286
David Ellison and Tom Berl, NEJM, 356, 2064-2072, 2007
Formulas For Calculating
Initial Saline Infusion Rates
****MAJOR CAVEAT*****
No matter how precise a given formula for
estimating Na
+
after treatment, it cannot
predict changes in the underlying
physiology
risk of over-shooting R
x
goal
Copyright Harvard Medical School, 2010. All Rights Reserved.
287
The Adrogue/Madi as Formula Underestimates
Na
+
After Hypertoni c Saline
Mohmand et al, CJASN, 2007
What If You Over-Correct :
Treatment of Osmotic Demyelination
DDAVP and D5W to re-induce
hyponatremia animal and human data
Myo-inositol supplementation during
correction animal data
Dexamethasoneto restore blood brain
barrier function animal data
Copyright Harvard Medical School, 2010. All Rights Reserved.
288
Re-Induction of Hyponatremia
With DDAVP + Free H
2
O
Sterns et al, CJASN, 2008
Course of
one case
Hyponatremic Causes Associated
With Overcorrection
Hypovolemic hyponatremia, after volume
resuscitation with saline and AVP (half-life of
AVP is 10-20 minutes)
Hypopituitarismwith 2 adrenal failure, after
treatment with cortisol and and AVP
Thiazide-associated hyponatremia, after
discontinuation and saline hydration
Rapid spontaneous resolution of SIADH or of a
non-osmotic increase in AVP, e.g. post-
operative nausea/pain
Copyright Harvard Medical School, 2010. All Rights Reserved.
289
Tolvaptan in Hyponatremia:
The SALT Trials
Schrier et al, NEJM, 2006
Rapid Correction with IV Conivaptan
Zeltser et al, Am J Nephrol, 2007
Copyright Harvard Medical School, 2010. All Rights Reserved.
290
Pros and Cons of the Vaptans
What are the indications for these drugs in clinical
practice?
Expensive but so is democlocycline
Risk of overcorrection minimize by frequent
monitoring of serum Na during the first 48 hours
Tolvaptan can be used in liver disease
(conivaptan should NOT)
Conivaptan is IV but not PO; tolvaptan is PO but
not IV..
Potassium
Copyright Harvard Medical School, 2010. All Rights Reserved.
291
Na
+
, K
+
and H
2
O
Transport in
Principal Cells
ENaC epithelial Na channel
ROMK secretory K channel
Maxi-K flow-activated K
channel
Question #3
You are asked to evaluate moderate, variable
hyperkalemia in a 26 yo woman with Type I
diabetes since age 10. She has hypertension
and proteinuria, with Uprot:creat of 1.
Medications include lisinopril 40mg daily, insulin,
prn NSAIDs (for H/A), oral contraceptive
Laboratory Studies:
Na
+
134 BUN 30
K
+
5.2 creatinine 1.5
Cl
-
110 TTKG 5
HCO
3
-
20
Copyright Harvard Medical School, 2010. All Rights Reserved.
292
A. Underlying renal artery stenosis
B. Excessive dietary intake of potassium
C. Subclinical adrenal insufficiency
D. Hyporeninemic hypoaldosteronism
E. Her oral contraceptive
Which of the following factors is most
likelyto have contributed to her risk of
hyperkalemia?
E. Her oral contraceptive
Which of the following factors is most
likelyto have contributed to her risk of
hyperkalemia?
Copyright Harvard Medical School, 2010. All Rights Reserved.
293
Drospirenone and Hyperkalemia
Drospirenone3 mg is the progestin component of
several oral contraceptives (Ocs), including Yaz,
Yasmin, and Yasminelle.
Drospirenonealso inhibits the mineralocorticoidreceptor
3 mg is equivalent to ~25 mg of spironolactone.
OCs containing 3 mg drospirenonecan reduce bp, but
do not cause hyperkalemia in healthy women.
Risk of hyperkalemia in patients with underlying
predispositions, +/- other RAS-active meds.
Direct and Indirect Inhibition of the
Amiloride-Sensi ti ve Sodium Channel (ENaC)
MLR mineralocorticoidreceptor
CAP channel-activating protease
ENaC epithelial sodium channel
+
-
-
+
+
CAP
ENaC
MLR
Aldosterone
Nafamostat
Ami loride
Tri methopri m
Tri amterene
Copyright Harvard Medical School, 2010. All Rights Reserved.
294
Question #4
You are asked to evaluate a female patient with
intestinal pseudo-obstruction (Ogilvies syndrome),
with diarrhea and profound hypokalemia.
Meds include metoprolol, risperidone, insulin
Exam and imaging notable for signs of colonic
distension.
Laboratory Studies:
Na
+
151 BUN 30
K
+
2.5 creatinine 1.5
Cl
-
115 TTKG 3
HCO
3
-
15 stool K
+
100 mEqu/kg
stool Na
+
10 mEqu/kg
Which of the following is the most likely
cause of this patients hypokalemia?
A. Ischemic bowel
B. C diff colitis
C. Activation of intestinal K
+
secretion
D. Osmotic diarrhea
E. Sympathetic activation with
redistributive hypokalemia
Copyright Harvard Medical School, 2010. All Rights Reserved.
295
Which of the following is the most likely
cause of this patients hypokalemia?
C. Activation of intestinal K
+
secretion
Ogilvies Syndrome and Hypokalemia
Hypokalemiadue to massive upregulation
of colonic BK channels, leading to K
+
-rich
diarrhea
BK channels are the major secretoryK
+
channel in colon, responsible for increased
K
+
secretion in ESRD and in response to
sympathetic activation; ? link between
sympathetic activation after intestinal
pseudo-obstruction
Copyright Harvard Medical School, 2010. All Rights Reserved.
296
Colonic BK Channels in Ogilvies
Syndrome
Patient ESRD control patient
Nephrology Dialysis Transplantation 2008 23(10):3350-3352
Treatment of Hyperkalemia
Mechanism Therapy Dose Onset Duration
Stabilize membrane
potential
Calcium 10% Ca-gluconate,
10 ml over 10 min.
1-3 min. 30-60
min
Cellular K
+
uptake Insulin

2
-agonist

10 U R with 50 ml
of D50, if BS<250

nebulized albuterol,
10 mg
30 min.


30 min.
4-6 h


2-4 h

K
+
removal Kayexalate


Hemodialysis
30-60 g PO

?


Immediate
?

Copyright Harvard Medical School, 2010. All Rights Reserved.
297
Caveats/Concerns re Kayexalate/SPS
Slow onset of effect ? makes SPS
unnecessary and inappropriate in most patients
with acute hyperkalemia.
Intestinal necrosis due to SPS in sorbitol is often
a fatal complication, NOT restricted to post-op
setting.
New FDA advisory September, 2009 do NOT
administer SPS with sorbitol.
Yet SPS with sorbitol remains a very popular
reflex mechanism of therapy for hyperkalemia,
often the only formulation of SPS available.
Question #5
40 yo man with a h/o depression, back pain, HTN, and asthma,
presents with dyspnea, abdominal pain, and vomiting.
Meds include paracetamol (hydrocodone/acetaminophen) metoprolol,
candesartan, and rabeprazole.
He denies alcohol or drug use, antifreeze ingestion, etc.
Physical exam remarkable only for tachypnea (36 bpm)
Laboratory Studies:
ABG 6.98/6/145/3 acetone negative
Na
+
144 ethanol <5
K
+
4 tylenol 7.5(10-30)
Cl
-
109 lactate 4.3(0.5-2.2)
Glucose 209 AG 32
BUN 6
Osmolality 312(gap ~10)
Copyright Harvard Medical School, 2010. All Rights Reserved.
298
Which of the following is the most
likely cause of this acidosis?
A. Alcoholic ketoacidosis
B. Ethylene glycol toxicity
C. D-Lactic acidosis
D. Pyroglutamic aciduria
E. Toluene ingestion
Which of the following is the most
likely cause of this acidosis?
D. Pyroglutamic aciduria,
also known as 5-oxoprolinuria
Acetaminophen therapy, AG-acidosis
Urinary 5-oxoproline
20.7 mmol/mmol creatinine(<.06)
Copyright Harvard Medical School, 2010. All Rights Reserved.
299
5-Oxoprolinuria and Acidosis
Classically due to errors of metabolism in
the -glutamyl cycle.
Acquired cases increasingly reported,
almost always in association with
acetaminophen use.
Variable presentations, but all have
metabolic acidosis with increased anion
gap.
Acetaminophen and the Anion Gap in a
Patient with 5-Oxoprolinuria
Humphreys et al, AJ KD; 2006, 46, 143-146
Copyright Harvard Medical School, 2010. All Rights Reserved.
300
The -Glutamyl Cycle and 5-Oxoprolinuria
* Involved in hereditary forms
Glutathione
reduction by
acetaminophen
reduces
feedback
inhibition
5-oxoproline
Anion Gap Acidoses
Keto-acidosis
Diabetic (DKA) Type I and Type II DM
Alcoholic (AKA)
Lactic Acidosis, including D-Lactic Acidosis
Methanol poisoning
Ethylene glycol poisoning
Propylene glycol IV drug infusions
Salicylate toxicity
Congenital organic acidoses
Pyroglutamic acidemia (5-oxoprolinuria)
Genetic
Acetaminophen, flucloxacillin, vigabratin
Uremia
Copyright Harvard Medical School, 2010. All Rights Reserved.
301
Utility of the AG/HCO
3
-
Ratio
AG 8
24 HCO
3
-
AG/HCO
3
-
=
AG/HCO
3
-
Diagnosi s
1 Pure AG acidosis
<1 Concomitant non-AG
acidosis
>1 Concomitant metabolic
alkalosis

Classically:
Caveats for the AG/HCO
3
-
Ratio
Assumes sensitivity/accuracy of the anion gap;
albumin etc. can measured anion gap
Assumes an equivalent volume of distribution of
the offending anions and H
+
not always the
case, e.g. in lactic acidosis
Many anions are rapidly excreted AG
Examples:
D-lactate not reabsorbed by proximal tubule
Ketoacids in DKA variable excretion, dependent on
volume status, etc.
Rastegar, JASN, 18, 2007
Copyright Harvard Medical School, 2010. All Rights Reserved.
302
Classification of Renal Tubular
Acidosis (RTA)
Type I, classic hypokalemic distal RTA
K+, unable to decrease urine pH <5.5 despite
acidemia
Type II, proximal RTA
typically K+, threshold for HCO
3
-
reabsorptionby
proximal tubule urine pH can to 5.5 only once
serum HCO
3
-
<15
Type IV, hyperkalemic distal RTA
K+, urine pH usually <5.5
either generalized in aldosterone or renal
resistance
NHE
NBC
Na
+
V-ATP
H
+
Blood
Tubular
fluid
filtered
HCO
3
-
CA-4
CO
2
+
H
2
O
CO
2
+H
2
O
CA-2
Na
+
nHCO
3
-
H
+
HCO
3
-
80-90%
Proximal Tubule HCO
3
-
Absorption
Dr. M. F. Romero, CWRU, 2002
Copyright Harvard Medical School, 2010. All Rights Reserved.
303
Cause of Proximal RTA (pRTA)
Isolated pRTA
Primary hereditary (Na-HCO
3
transporter), sporadic
(often transient)
Carbonic anhydrase deficit hereditary, CA inhibitors
Generalized proximal tubular dysfunction
associated glycosuria, phosphaturia, etc.
Primary hereditary Fanconis syndrome
Genetic disease cystinosis, Wilsons disease
Acquired Sjoegrens, paraproteinemias, vitamin D
deficiency, etc.
Drugs/toxins ifosfamide, FK-506/tacrolimus,
cyclosporine, toluene, outdated tetracycline, lead,
mercury, etc.
Pathophysiology of Proximal RTA
Reduced proximal HCO
3
-
reabsorption
overwhelms distal nephron
Cl- reabsorption hyperchloremic acidosis
filtered load of HCO
3
-
distal reabsorption
urine pH
At steady state, [HCO
3
-
] = 15-18 mEq/L,
with urine pH <5.5
Copyright Harvard Medical School, 2010. All Rights Reserved.
304
Diagnosis of Proximal RTA
Typically need to distinguish from
hypokalemic distal RTA
Plasma bicarbonate concentration
toward normal (18 to 20 meq/L) with IV
bicarbat 0.5 to 1.0 meq/kg per hour
In proximal RTA, urine pH should to >7.5
with FeHCO
3
of 15-20%; in hypokalemic
distal RTA will see minimal change in
urine pH, FeHCO3 should be <3%
2K
+
NH /K
4
+ +
NH
3
NH
3
NH
4
+
NH
4
+
NH + H
3
+
2K
+
ATP
ATP
ATP
ATP
K
+
H
+
H
+
K
+
K
+
ROMK
kAE1
KCC4
ENaC
Aquapori n-2
Lumen
3Na
+
3Na
+
Na
+
Cl
-
Cl
-
Cl
-
Cl
-
HC0
3
-
(-) (+)
NH
4
+
Excretion,
Renal Collecting
Duct
Take-home point:
Principal
cells generate a
lumen-negative
potential difference,
which drives K
+
and
H
+
excretion
Copyright Harvard Medical School, 2010. All Rights Reserved.
305
Functional Classification of Distal RTA
Defective H-ATPase secretory
defect
Unfavorable electrical gradient for H
+
secretion voltage defect
Back diffusion of H
+
permeability
defect
Insufficient delivery of NH
3
to the distal
nephronNH
3
defect, e.g. in
hyperkalemia
Causes of Hypokalemic dRTA
Primary hereditary
H
+
-ATPase subunit genes
type II carbonic anhydrase (proximal and distal RTA)
AE1 Cl-HCO
3
exchanger (SLC4A1)
Genetic causes sickle-cell, Fabrys, Wilsons,
elliptocytosis, MCDK
Auto-immune disorders SLE, etc.
Nephrocalcinosis/hypercalciuria
Amyloidosis
Drugs/toxins amphotericin
Tubulo-interstitial disease reflux nephropathy,
obstructive uropathy
Copyright Harvard Medical School, 2010. All Rights Reserved.
306
Mutations in the B-subunit of H-ATPase
Cause Autosomal Recessive Distal RTA With
Deafness
Karet et al, Nature Genetics, 21, 1999
Index
case
Age
at Dx
pH HCO
3
-
K
+
Urine
pH
Urine Ca
2+

(mg/kg/d)
RTA18 3 mo 7.26 13 2.2 6.9 8.5 (high)
RTA59 2 mo 7.10 8.7 2.9 7.5 6.0
RTA35 3 yrs 7.19 8.9 2.8 6.0 5.0
RTA29 6 mo 7.12 11 3.2 7.0 8.5

H
+
, NH
3
, and HCO
3
-
Transport in
Intercalated Cells
2K
+
NH /K
4
+ +
ATP
ATP
ATP
K
+
H
+
H
+
H
+
AE1
CA-II
KCC4
H -ATPase
+
H /K -ATPase
+ +
Lumen
Interstitium
3Na
+
Cl
-
Cl
-
Cl
-
HC0
3
-
C0
2
H 0
2
NH
3
NH
4
+
NH
3
RhCG
RhBG NH
3
(-) (+)
Copyright Harvard Medical School, 2010. All Rights Reserved.
307
Hyperkalemic Distal RTA
Type IV RTA (hyporeninemic hypoaldo)
Diabetic nephropathy, SLE, acute post-Strep GN
Type II Pseudohypoaldosteronism (Gordons
syndrome)
Adrenal insufficiency
Addisons, AIDS, isolated hypoaldo, CAH, etc.
Mineralocorticoid resistance
Type I Pseudohypoaldosteronism
Interstitial disease, e.g. AIN, obstructive uropathy
Drug-induced
Various mechanisms; K
+
redistribution, ENaC
inhibition, RAS inhibition, etc.
Disclosures
Consultant (hyponatremia) Salix Otsuka Consultant (hyponatremia) Salix, Otsuka
Speaker bureau, 2005-2008 Astellas
(conivaptan)
Copyright Harvard Medical School, 2010. All Rights Reserved.
308
PATHOLOGY AND
MECHANISMS OF
GLOMERULAR DISEASES
Helmut G. Rennke, M.D.
Disclosures
NONE NONE
Copyright Harvard Medical School, 2010. All Rights Reserved.
309
PT
DT
a
PT
DT
e
MD PT
DT
BC
CL
US
Ep
End
Ep
CL
Copyright Harvard Medical School, 2010. All Rights Reserved.
310
Ep
FP
US
CL
End
THE CLINICAL SYNDROMES
1. The Nephrotic Syndrome
2. The Acute Nephritic Syndrome
3. Rapidly Progressive Glomerulonephritis
4. Asymptomatic Hematuria / Proteinuria
5. The Chronic Nephritic Syndrome
(Chronic Renal Failure)
Copyright Harvard Medical School, 2010. All Rights Reserved.
311
1.- Epithelial Cell Disease (Minimal Change
Disease)
THE BASIC STRUCTURAL PATTERNS
OF GLOMERULAR INJURY
Disease)
2.- Focal Segmental Glomerulosclerosis
3.- Membranous Nephropathy
4.- Diffuse Proliferative Glomerulonephritis
5.- Membranoproliferative Glomerulonephritis
6.- Crescentic Glomerulonephritis
7.- Focal Proliferative and Necrotizing 7. Focal Proliferative and Necrotizing
Glomerulonephritis
8.- Mesangial Proliferative Glomerulonephritis
9.- Basement Membrane Abnormalities
10.- Focal Global Glomerulosclerosis
MECHANISMS OF KIDNEY INJURY
Genetic and congenital
abnormalities
M t b li b liti
Paraproteins
Thrombosis and
b liti f th Metabolic abnormalities
Immune mechanisms
Types II, III, and IV
Cytokines
Abnormalities in
complement-regulatory
proteins
abnormalities of the
coagulation/fibrinolysis
system
Physical factors: radiation
Obstruction of the
excretory system
Infections: bacteria fungi proteins
Ischemic injury
Toxic damage:
environmental, dietary,
iatrogenic toxins
Infections: bacteria, fungi,
viruses, parasites
Functional and structural
adaptations
Copyright Harvard Medical School, 2010. All Rights Reserved.
312
IMMUNE-MEDIATED DISORDERS
TYPE MECHANISM OF TISSUE INJURY
I. Immediate
Hypersensitivity
IgE antibodies immediate release of vasoactive
amines, lipid mediators, cytokines from mast cells -
it t f i fl t ll
II. Antibody-
mediated
III Immune
recruitment of inflammatory cells.
IgG, IgM antibodies; binding to target molecule, cell, or
matrix component:
A. loss or enhancement of cell/protein function
B. lysis of target cell via complement activation
C. antibody-dependent cell cytotoxicity
Antigen-antibody complexes in circulation Complement III. Immune
Complex-
mediated
IV. T Cell-mediated
Antigen antibody complexes in circulation Complement
and Fc receptor-mediated recruitment of neutrophils -
release of lysosomal enzymes and other toxic mediators
CD4
+
T cells macrophage activation cytokines -
inflammation
CD8
+
T cells - direct cytotoxicity cytokines - inflammation
Proteinuria (>3.5g/day)
THE NEPHROTIC SYNDROME
Hypoalbuminemia
Hyperlipidemia
Lipiduria
Edema
Bland urine sediment
Copyright Harvard Medical School, 2010. All Rights Reserved.
313
1.- Epithelial Cell Disease (Minimal Change Disease)
2 F l S l Gl l l i
THE BASIC STRUCTURAL PATTERNS
OF GLOMERULAR INJURY
2.- Focal Segmental Glomerulosclerosis
3.- Membranous Nephropathy
4.- Diffuse Proliferative Glomerulonephritis
5.- Membranoproliferative Glomerulonephritis
6.- Crescentic Glomerulonephritis
7.- Focal Proliferative and Necrotizing g
Glomerulonephritis
8.- Mesangial Proliferative Glomerulonephritis
9.- Basement Membrane Abnormalities
10.- Focal Global Glomerulosclerosis
THE NEPHROTIC SYNDROME
The structural abnormality y
shared by all nephrotic
conditions or diseases with
heavy proteinuria is diffuse
i lifi ti f i f th simplification or fusion of the
foot processes of the glomerular
visceral epithelial cells.
Copyright Harvard Medical School, 2010. All Rights Reserved.
314
MOLECULAR ORGANIZATION OF THE
FOOT PROCESS OF THE GLOMERULAR
VISCERAL EPITHELIAL CELL
C t t i N=NHERF2 Cat=catenins
CD-CD2AP
Cas=p130Cas
Ez=ezrin
FAK=focal adhesion kinase
TPV=talin-paxilin-vinculin
ILK-integrin-linked kinase
M=myosin
N=NHERF2
NSCC=non-selective Ca
chanel
PC=podocalyxin
S=synaptopodin
U=utrophin
Z=ZO-1
Endlich. 2001 Curr Opin
Nephrol Hypertens
10:331,
Copyright Harvard Medical School, 2010. All Rights Reserved.
315
THE PHENOTYPIC EXPRESSION
OF GENETIC DEFECTS OF THE
PODOCYTE - THE FAMILIAL
PODOCYTOPATHIES
Diffuse Simplification ( fusion )
of Foot Processes
Diffuse Mesangial Sclerosis
Focal and Segmental
Glomerulosclerosis
Diffuse Simplification ( fusion ) of
Foot Processes
Diffuse Mesangial Sclerosis
Focal and Segmental
Glomerulosclerosis
Genetic Abnormalities of the Podocyte
The Diffuse Familial Podocytopathies
Nephrin; NPHS1; 19q13.1; the congenital
nephrotic syndrome (CNS); diffuse lesion, AR nephrotic syndrome (CNS); diffuse lesion, AR
Podocin ; NPHS2; 1q25-q31; familial and
sporadic SRNS and FSGS; diffuse lesion, AR
CD2-associated protein, p130
CAS
associated;
NS; Chr 6; AR; truncated protein; diffuse
mesangial sclerosis and collapsing lesions; AR
Wilms Tumor gene; WT-1; 11p13; Diffuse
Mesangial Sclerosis, AD
- WAGR (Wilms tumor, Aniridia, Gonadal, Renal abn)
- Denys-Drash syndrome (Renal dis, mPsH, WT)
- Frasier syndrome (FSGS, mPsH, Gblast)
- WT-1 splice mutation in 46XX female
Copyright Harvard Medical School, 2010. All Rights Reserved.
316
Genetic Abnormalities of the Podocyte
The Focal Familial Podocytopathies
-actinin-4; ACTN4; 19q13; progressive
proteinuria and familial FSGS; age-related
penetrance; AD
Canonical Transient Receptor Potential 6;
TRPC6; 11q; progressive proteinuria and
familial FSGS; age-related penetrance; AD
Member of the formin family of actin-
regulating proteins; IFN2; 14q; progressive
proteinuria and familial FSGS; age-related
penetrance; AD
-galactosidase deficiency (Fabry; Xq24-24)
Sialidase deficiency (4q21 23)
OTHER GENETIC DISEASES WITH FSGS,
PROTEINURIA, OR RENAL FAILURE
Sialidase deficiency (4q21-23)
Familial form of unilateral renal agenesis
Charcot-Marie-Tooth Disease (FSGS)
Mitochondrial disease, mDNA-A3243G; Myopathies and FSGS
b4-Integrin mutation: epidermolysis bullosa and congenital
FSGS
Inherited basement membrane defects:
- X-linked Alport syndrome (COL4A5)
- autosomal recessive hereditary nephritis
(COL4A3; COL4A4)
- Thin basement membrane disease (COL4A4)
Nail-patella syndrome (LMX1B; 9q34)
Copyright Harvard Medical School, 2010. All Rights Reserved.
317
DIFFUSE EPITHELIAL CELL
DISEASE
MINIMAL CHANGE DISEASE
DIFFUSE EPITHELIAL CELL
DISEASE
Hereditary Podocytopathies Hereditary Podocytopathies
Congenital nephrotic syndrome (Finish type)
Defect in Nephrin; NPHS1; 19q13.1; AR
Denys-Drash syndrome (mesagial sclerosis)
Defect in WT1;
Steroid-resistant nephrotic syndrome
Defect in Podocin; NPHS2; Defect in Podocin; NPHS2;
Acquired Epithelial Cell Disease
Minimal Change Disease and Associated Lesions
Idiopathic Focal and Segmental Glomerulosclerosis
Copyright Harvard Medical School, 2010. All Rights Reserved.
318
Copyright Harvard Medical School, 2010. All Rights Reserved.
319
IgG
Copyright Harvard Medical School, 2010. All Rights Reserved.
320
CLINICAL CONDITIONS ASSOCIATED
WITH MINIMAL CHANGE DISEASE
Idiopathic minimal change disease
(lipoid nephrosis, nil disease) ( p p )
subgroup with acute renal failure
Secondary forms of diffuse epithelial cell disease:
- associated with drug use, with or without AIN
- early stage of HIV-associated nephropathy,
collapsing glomerulopathy, and heroin
nephropathy
- associated with Hodgkin's disease and other
lymphoproliferative disorders
- associated with insect bites, immunizations, etc.
- associated with IgA nephropathy, diabetes
mellitus, SLE, and other glomerulopathies
ACUTE RENAL FAILURE IN ADULT
MINIMAL CHANGE DISEASE
MCD + ARF MCD
Serum Creat. (mg/dl) 2.2-15.2 0.6 1.4
Age (years) 59.6 40.3
Systolic BP (mmHg) 158 138
Proteinuria (g/day) 13 5 7 9 Proteinuria (g/day) 13.5 7.9
Arteriosclerosis Index (0-4) 1.7 0.7
Evidence of ATN 71% 0%
J C J ennette, RJ Falk. Am J Kidney Dis 16:432-437, 1990.
Copyright Harvard Medical School, 2010. All Rights Reserved.
321
NSAID-ASSOCIATED NEPHROTIC SYNDROME
24 cases of the NS associated with use of NSAID
17 biopsies: all showed minimal disease
Average 24-hour urine protein was 11.2g
Peak plasma creatinine 0.8 to 14.8 mg/dl
The NS remitted or improved in 23/24
Course of remission: Course of remission:
19/24 < 2 months
4/24 > 2 months
Feinfeld, Olesniky, Pirani, Appel. Nephron 37:174, 1984.
Copyright Harvard Medical School, 2010. All Rights Reserved.
322
FOCAL AND SEGMENTAL
GLOMERULOSCLEROSIS (FSGS)
1. IDIOPATHIC OR PRIMARY FSGS
2. SECONDARY (ADAPTIVE) FSGS
3. SEGMENTAL GLOMERULAR SCARRING 3. SEGMENTAL GLOMERULAR SCARRING
4. FAMILIAL FSGS
Copyright Harvard Medical School, 2010. All Rights Reserved.
323
Copyright Harvard Medical School, 2010. All Rights Reserved.
324
IgM
Copyright Harvard Medical School, 2010. All Rights Reserved.
325
Copyright Harvard Medical School, 2010. All Rights Reserved.
326
IMMUNE-MEDIATED DISORDERS
TYPE MECHANISM OF TISSUE INJURY
I. Immediate
Hypersensitivity
IgE antibodies immediate release of vasoactive
amines, lipid mediators, cytokines from mast cells -
it t f i fl t ll
II. Antibody-
mediated
III Immune
recruitment of inflammatory cells.
IgG, IgM antibodies; binding to target molecule, cell, or
matrix component:
A. loss or enhancement of cell/protein function
B. lysis of target cell via complement activation
C. antibody-dependent cell cytotoxicity
Antigen-antibody complexes in circulation Complement III. Immune
Complex-
mediated
IV. T Cell-mediated
Antigen antibody complexes in circulation Complement
and Fc receptor-mediated recruitment of neutrophils -
release of lysosomal enzymes and other toxic mediators
CD4
+
T cells macrophage activation cytokines -
inflammation
CD8
+
T cells - direct cytotoxicity cytokines - inflammation
FOCAL AND SEGMENTAL
GLOMERULOSCLEROSIS (FSGS)
1. IDIOPATHIC OR PRIMARY FSGS
2. FAMILIAL FSGS
3. SECONDARY OR ADAPTIVE FSGS
4. SEGMENTAL GLOMERULAR SCARRING
Copyright Harvard Medical School, 2010. All Rights Reserved.
327
Copyright Harvard Medical School, 2010. All Rights Reserved.
328
FOCAL AND SEGMENTAL
GLOMERULOSCLEROSIS (FSGS)
1. IDIOPATHIC OR PRIMARY FSGS
2. FAMILIAL FSGS
3. SECONDARY OR ADAPTIVE FSGS
4. SEGMENTAL GLOMERULAR SCARRING
Unilateral renal agenesis and compensatory hypertrophy
Copyright Harvard Medical School, 2010. All Rights Reserved.
329
Copyright Harvard Medical School, 2010. All Rights Reserved.
330
METABOLIC SYNDROME
Copyright Harvard Medical School, 2010. All Rights Reserved.
331
IDIOPATHIC vs. ADAPTIVE FSGS
Sudden onset
nephrotic syndrome
(edema)
Slowly progressive
proteinuria without edema
(edema)
No history of prior
kidney disease
Normal size glomeruli
Diffuse effacement of
History of prior kidney
disease (pregnancy) or
vascular conditions
Glomerular hypertrophy
Foot processes largely
foot processes
No significant
parenchymal atrophy
preserved
Pre-existing kidney injury
COLLAPSING
GLOMERULOPATHY
Copyright Harvard Medical School, 2010. All Rights Reserved.
332
Copyright Harvard Medical School, 2010. All Rights Reserved.
333
Albumin
Copyright Harvard Medical School, 2010. All Rights Reserved.
334
MEMBRANOUS GLOMERULOPATHY
Copyright Harvard Medical School, 2010. All Rights Reserved.
335
Copyright Harvard Medical School, 2010. All Rights Reserved.
336
IgG
Copyright Harvard Medical School, 2010. All Rights Reserved.
337
IMMUNE-MEDIATED DISORDERS
TYPE MECHANISM OF TISSUE INJURY
I. Immediate
Hypersensitivity
IgE antibodies immediate release of vasoactive
amines, lipid mediators, cytokines from mast cells -
it t f i fl t ll
II. Antibody-
mediated
III Immune
recruitment of inflammatory cells.
IgG, IgM antibodies; binding to target molecule, cell, or
matrix component:
A. loss or enhancement of cell/protein function
B. lysis of target cell via complement activation
C. antibody-dependent cell cytotoxicity
Antigen-antibody complexes in circulation Complement III. Immune
Complex-
mediated
IV. T Cell-mediated
Antigen antibody complexes in circulation Complement
and Fc receptor-mediated recruitment of neutrophils -
release of lysosomal enzymes and other toxic mediators
CD4
+
T cells macrophage activation cytokines -
inflammation
CD8
+
T cells - direct cytotoxicity cytokines - inflammation
Copyright Harvard Medical School, 2010. All Rights Reserved.
338
TYPE II, ANTIBODY-MEDIATED
A. Antibody-dependent and complement-mediated:
Autoimmune hemolytic anemia
Agranulocytosis
Autoimmune thrombocytopenia
Erythroblastosis fetalis (Rh)
Idiopathic Membranous Glomerulopathy (M type
phospholipase A2 receptor, enolase)
Congenital Membranous GN (neutral endopeptidase) g ( p p )
Bullous pemphigoid ? (hemidesmosomes; PA 1 and 2)
Rheumatic fever ? (cross-reactive antibodies to M proteins)
Antibody-mediated allograft rejection, vasculitis (HLA)
PATHOGENESIS OF MEMBRANOUS
NEPHROPATHY
In situ formation of immune complexes: endogenous
epithelial cell antigen and circulating
t tib di autoantibodies
Complement activation
Epithelial cell injury (MAC)
Permeability defect due cell damage, retraction of foot
processes, and loss of filtration slit diaphragms
Proteinuria
Active inflammation does not occur; a gradient of
chemotactic agents is not established against the
ultrafiltration flux; lack of endothelial cell injury
Copyright Harvard Medical School, 2010. All Rights Reserved.
339
CONDITIONS ASSOCIATED WITH
MEMBRANOUS NEPHROPATHY
1.- Idiopathic Membranous Nephropathy
Congenital Membranous Nephropathy
2.- Autoimmune Diseases
SLE and RA
Hashimoto's disease, Grave's disease,
MCTD Sjgren's syndrome MCTD, Sjgren s syndrome,
primary biliary cirrhosis,
bullous pemphigoid, dermatitis
herpetiformis, ankylosing spondylitis,
small bowel enteropathies
Copyright Harvard Medical School, 2010. All Rights Reserved.
340
CONDITIONS ASSOCIATED WITH
MEMBRANOUS NEPHROPATHY
3.- Infectious or Parasitic Diseases
H titi B d C Hepatitis B and C
Syphilis
Filariasis, hydatidic cyst, schistosomiasis,
malaria, leprosy, enterococcal endocarditis
4 D 4.- Drugs
Au, Hg, Penicillamine, Captopril,
NSAIDs, Hydrochlorothiazide, Hydralazine,
Trimethadione, Chlormethiazole
WHY DO PATIENTS WITH DIFFUSE
EPITHELIAL CELL DISEASE TEND TO
HAVE EDEMA (NEPHROTIC SYNDROME),
WHILE PATIENTS WITH ONLY FOCAL
DAMAGE TO THE PODOCYTES USUALLY
LACK EDEMA?
The theory of the low serum albumin
concentration as the driving force for salt
retention (the under-fill theory)
The notion of the proteinuric nephron as the
immediate cause of sodium retention; the
reset volume-sensing mechanism in the
proteinuric tubule
Copyright Harvard Medical School, 2010. All Rights Reserved.
341
STARLING FORCES OPERATING IN
THE PERIPHERAL CAPILLARIES
Na
+
DELIVERY AND REABSORPTION IN
PROTEINURIC AND NON-PROTEINURIC
NEPHRONS IN UNILATERAL PAN-INDUCED
NEPHROTIC SYNDROME NEPHROTIC SYNDROME
J Clin Invest 71:91-103, 1983
Copyright Harvard Medical School, 2010. All Rights Reserved.
342
Klisic J , Zhang J , Nief V, Reyes L,
Moe OW, Ambuhl PM. 2003.
Albumin regulates the Na+/H+
exchanger 3 in OKP cells. J Am
Soc Nephrol 14:3008-3016
THE ACUTE NEPHRITIC
SYNDROME
Hematuria
(red cell casts, dysmorphic RBCs)
Oliguria
Azotemia Azotemia
Hypertension
Edema
Copyright Harvard Medical School, 2010. All Rights Reserved.
343
THE ACUTE NEPHRITIC
SYNDROME
Di h t i d b th t Diseases characterized by the acute
nephritic syndrome are associated
invariably with deposition of immune
complexes in the more proximal layers
of the glomerular capillary wall in close of the glomerular capillary wall, in close
proximity to the endothelial cell surfaces
or in the lamina rara interna.
MECHANISMS OF IMMUNE COMPLEX
FORMATION WITHIN THE PERIPHERAL
CAPILLARY WALL
Subepithelial IC Subendothelial IC
Endogenous epithelial
cell antigens
(membranous GN,
membranous lupus
Entrapment of circulating
immune complexes
(lupus nephritis, post-
infectious GN)
Subepithelial IC Subendothelial IC
membranous lupus
nephritis)
Planted exogenous
antigens
(post-infectious GN)
)
Planted exogenous or
endogenous antigens
(DNA in lupus nephritis)
Copyright Harvard Medical School, 2010. All Rights Reserved.
344
24 hours
2 days
Copyright Harvard Medical School, 2010. All Rights Reserved.
345
2 weeks
14 days
4 weeks
Copyright Harvard Medical School, 2010. All Rights Reserved.
346
5 minutes
Copyright Harvard Medical School, 2010. All Rights Reserved.
347
30 minutes
4 hours
Copyright Harvard Medical School, 2010. All Rights Reserved.
348
24 hours
Copyright Harvard Medical School, 2010. All Rights Reserved.
349
MODELS OF DISEASES WITH IMMUNE
COMPLEXES PLANTED WITHIN THE
CAPILLARY WALL
SUBEPITHELIAL IC
SUBENDOTHELIAL IC
SUBEPITHELIAL IC
Epithelial cell injury
Proteinuria
No endothelial injury
SUBENDOTHELIAL IC
No epithelial cell injury
No proteinuria
Endothelial injury
No inflammation
Evolves over month
Acute inflammation
Evolves over hours or
few days
1.- Epithelial Cell Disease (Minimal Change Disease)
2 F l S l Gl l l i
THE BASIC STRUCTURAL PATTERNS
OF GLOMERULAR INJURY
2.- Focal Segmental Glomerulosclerosis
3.- Membranous Nephropathy
4.- Diffuse Proliferative Glomerulonephritis
5.- Membranoproliferative Glomerulonephritis
6.- Crescentic Glomerulonephritis
7.- Focal Proliferative and Necrotizing g
Glomerulonephritis
8.- Mesangial Proliferative Glomerulonephritis
9.- Basement Membrane Abnormalities
10.- Focal Global Glomerulosclerosis
Copyright Harvard Medical School, 2010. All Rights Reserved.
350
HepC-associated acute GN
IgM
Copyright Harvard Medical School, 2010. All Rights Reserved.
351
Copyright Harvard Medical School, 2010. All Rights Reserved.
352
IgG
Copyright Harvard Medical School, 2010. All Rights Reserved.
353
RB
US
RB
C
US
RBC
Plt
CL
Copyright Harvard Medical School, 2010. All Rights Reserved.
354
CONDITIONS ASSOCIATED WITH DIFFUSE
PROLIFERATIVE GLOMERULONEPHRITIS
Acute Postinfectious Glomerulonephritis:
bacterial: streptococcal, staphylococcal,
pneumococcal, and typhoid fever
viral: varicella, mumps, ECHO and Coxsackie,
infectious mononucleosis, hepatitis B,
hepatitis C with cryoglobulins, measles
Other: syphilis, leptospirosis, toxoplasmosis,
falciparummalaria parasitic diseases falciparum malaria, parasitic diseases
Diffuse proliferative lupus nephritis
Dense Deposit Diseases (MPGN type II), early phase
Essential mixed cryoglobulinemia, early phase
THE MEMBRANOPROLIFERATIVE
PATTERN OF INJURY
STRUCTURAL CHANGES:
HYPERCELLULARITY HYPERCELLULARITY
CAPILLARY WALL THICKENING (double contours)
CONDITIONS ASSOCIATED WITH THE
MPGN PATTERN:
IMMUNE COMPLEX DISEASES IMMUNE COMPLEX DISEASES
ABNORMALITIES OF COMPLEMENT-
REGULATORY PROTEINS
THROMBOTIC ANGIOPATHIES
DEPOSITION DISEASE
Copyright Harvard Medical School, 2010. All Rights Reserved.
355
THE MEMBRANOPROLIFERATIVE
PATTERN OF INJURY
STRUCTURAL CHANGES:
HYPERCELLULARITY
CAPILLARY WALL THICKENING (double contours)
CONDITIONS ASSOCIATED WITH THE
MPGN PATTERN:
IMMUNE COMPLEX DISEASES IMMUNE COMPLEX DISEASES
ABNORMALITIES OF COMPLEMENT-
REGULATORY PROTEINS
THROMBOTIC ANGIOPATHIES
DEPOSITION DISEASE
Copyright Harvard Medical School, 2010. All Rights Reserved.
356
Copyright Harvard Medical School, 2010. All Rights Reserved.
357
CONDITIONS ASSOCIATED WITH A
MEMBRANOPROLIFERATIVE PATTERN OF INJURY
1. IMMUNE COMPLEX-MEDIATED DISEASES
Chronic infections:
Vi l h titi B h titi C d ti l i d Viral: hepatitis B, hepatitis C and essential mixed
cryoglobulinemia
Bacterial: endocarditis, infected ventriculo-atrial (or
jugular) shunt, multiple visceral abscesses,
leprosy, meningococcal meningitis
Protozoa: malaria, schistosomiasis
Other infections: mycoplasma ?Borreliosis Other infections: mycoplasma, ?Borreliosis,
Leishmaniasis
Miscellaneous:
Chronic liver disease (cirrhosis and alpha1-antitrypsin
deficiency)
Copyright Harvard Medical School, 2010. All Rights Reserved.
358
1. IMMUNE COMPLEX-MEDIATED DISEASES:
Autoimmune diseases:
SLE
CONDITIONS ASSOCIATED WITH A
MEMBRANOPROLIFERATIVE PATTERN OF INJURY
SLE
Sjgren syndrome
Rheumatoid arthritis
Inherited complement deficiencies, in
particular C2 deficiency
Idiopathic forms of MPGN (unknown association):
MPGN type I yp
MPGN type II or dense deposit disease and
partial lipodystrophy
MPGN type III (Strife and Anders variant,
Burkholder variant)
IMMUNE-MEDIATED DISORDERS
TYPE MECHANISM OF TISSUE INJURY
I. Immediate
Hypersensitivity
IgE antibodies immediate release of vasoactive
amines, lipid mediators, cytokines from mast cells -
it t f i fl t ll
II. Antibody-
mediated
III Immune
recruitment of inflammatory cells.
IgG, IgM antibodies; binding to target molecule, cell, or
matrix component:
A. loss or enhancement of cell/protein function
B. lysis of target cell via complement activation
C. antibody-dependent cell cytotoxicity
Antigen-antibody complexes in circulation Complement III. Immune
Complex-
mediated
IV. T Cell-mediated
Antigen antibody complexes in circulation Complement
and Fc receptor-mediated recruitment of neutrophils -
release of lysosomal enzymes and other toxic mediators
CD4
+
T cells macrophage activation cytokines -
inflammation
CD8
+
T cells - direct cytotoxicity cytokines - inflammation
Copyright Harvard Medical School, 2010. All Rights Reserved.
359
2 DEFECTS IN COMPLEMENT REGULATORY
CONDITIONS ASSOCIATED WITH A
MEMBRANOPROLIFERATIVE PATTERN OF
INJURY
2. DEFECTS IN COMPLEMENT-REGULATORY
PROTEINS:
Dense Deposit Disease (MPGN type II),
- C3 nephritic factor
- genetic defects in factor H, factor I,
d MCP CD46 and MCP or CD46
- partial lipodystrophy
Glomerulonephritis C3
Atypical HUS (d-HUS)
DENSE DEPOSIT DISEASE
MEMBRANOPROLIFERATIVE
GLOMERULONEPHRITIS TYPE II
Affects predominantly children or young Affects predominantly children or young
adults
Nephritic syndrome, often acute in onset
Chronic or subacute clinical course;
progressive proteinuria
P i h l i Persistent hypocomplementemia
Associated with familial lipodystrophy
Nephritic factor; autoantibody against C3
convertase of the alternative pathway
Copyright Harvard Medical School, 2010. All Rights Reserved.
360
US
Cap
C3
Copyright Harvard Medical School, 2010. All Rights Reserved.
361
PATHOGENESIS OF DENSE DEPOSIT
DISEASE
Mechanisms of complement
dysregulation that result in DDD:
Homozygous factor H deficiency (factor
H dissociates the C3bBb complex)
Deletion of a single AA in regulatory
domain 4 of factor H
dysregulation that result in DDD:
Inactivation of factor H by a circulating
inhibitor (monoclonal paraproteins)
The autoantibody C3 nephritic factor
(stabilizes the C3bBb complex)
ACTIVATION OF THE COMPLEMENT SYSTEM
CLASSICAL LECTIN ALTERNATIVE
Ag-Ab SUGARS Agg. Ig; Endotoxin
Activated C1 Mg; D C3b; B g; ;
C4a
C4b2b
C3 CONVERTASE
C3
C3bBb
C3b
feedback
pathway
Properdin
C3 CONVERTASE
C3bBbP
C3b C3a
C4b2b
C5b67 C5b
C5
C5a
C5b-9
Copyright Harvard Medical School, 2010. All Rights Reserved.
362
GLOMERULONEPHRITIS C3
OR
Primary glomerulonephritis with
isolated C3 deposits
Servais A, Fremeaux-Bacchi V, Lequintrec M, Salomon R, Blouin J,
Knebelmann B, Grunfeld JP, Lesavre P, Noel LH, Fakhouri F. Primary
glomerulonephritis with isolated C3 deposits: a new entity which shares
common genetic risk factors with haemolytic uraemic syndrome. J Med
Genet 2007 44:193-9. Epub 2006 Oct 3.
Copyright Harvard Medical School, 2010. All Rights Reserved.
363
C
3
Copyright Harvard Medical School, 2010. All Rights Reserved.
364
Copyright Harvard Medical School, 2010. All Rights Reserved.
365
Copyright Harvard Medical School, 2010. All Rights Reserved.
366
C3
Copyright Harvard Medical School, 2010. All Rights Reserved.
367
MEMBRANOPROLIFERATIVE
GLOMERULONEPHRITIS TYPE III
Strife and Anders variant; with defects of
the lamina densa
unknown pathogenesis; ? thrombotic
angiopathy
B kh ld i t ith b ith li l Burkholder variant; with subepithelial
deposits
? post-infectious GN
Copyright Harvard Medical School, 2010. All Rights Reserved.
368
CONDITIONS ASSOCIATED WITH A
MEMBRANOPROLIFERATIVE PATTERN OF INJURY
3. CHRONIC AND HEALED THROMBOTIC
MICROANGIOPATHIES:
H li h f HUS/TTP Healing phase of HUS/TTP
The syndrome of circulating anti-phospholipid
(anti-cardiolipin) antibodies
POEMS syndrome
Radiation nephritis
Nephropathy associated with bone marrow
transplantation
Drug-associated thrombotic angiopathies
Sickle cell anemia and polycythemia
Dysfibrinogenemia and other pro-thrombotic states
Transplant glomerulopathy
Copyright Harvard Medical School, 2010. All Rights Reserved.
369
Fibrin
US
End
*
CL
Mes
Copyright Harvard Medical School, 2010. All Rights Reserved.
370
Mes Mes
End
*
Copyright Harvard Medical School, 2010. All Rights Reserved.
371
CONDITIONS ASSOCIATED WITH A
MEMBRANOPROLIFERATIVE PATTERN OF
INJURY
4. PARAPROTEIN DEPOSITION DISEASES:
Gl l thi i t d ith Glomerulopathies associated with
cryoglobulinemia type I
Waldenstrm macroglobulinemia
Immunotactoid glomerulopathy
Immunoglobulin light chain or
heavy chain deposition diseases
POEMS syndrome (polyneuropathy,
organomegaly, endocrinopathy,
m-protein, skin change)
Fibrillary glomerulonephritis
Copyright Harvard Medical School, 2010. All Rights Reserved.
372
Fibrillary GN
IgG
Copyright Harvard Medical School, 2010. All Rights Reserved.
373
Copyright Harvard Medical School, 2010. All Rights Reserved.
374
Fibrillary GN Amyloidosis
IgG/kappa Paraprotein deposition disease
Copyright Harvard Medical School, 2010. All Rights Reserved.
375
IgG Kappa Lambda
Copyright Harvard Medical School, 2010. All Rights Reserved.
376
CONDITIONS ASSOCIATED WITH A
MEMBRANOPROLIFERATIVE PATTERN OF
INJURY
3. PARAPROTEIN DEPOSITION DISEASES:
Gl l thi i t d ith Glomerulopathies associated with
cryoglobulinemia type I
Waldenstrm macroglobulinemia
Immunotactoid glomerulopathy
Immunoglobulin light chain or
heavy chain deposition diseases
POEMS syndrome (polyneuropathy,
organomegaly, endocrinopathy,
m-protein, skin change)
Fibrillary glomerulonephritis
Copyright Harvard Medical School, 2010. All Rights Reserved.
377
RAPIDLY PROGRESSIVE
GLOMERULONEPHRITIS
Rapid decline in renal function
(over several days or few weeks)
Active urine sediment
Usually no edema and no hypertension
1.- Epithelial Cell Disease (Minimal Change Dis.)
2 F l S l Gl l l i
THE BASIC STRUCTURAL PATTERNS
OF GLOMERULAR INJURY
2.- Focal Segmental Glomerulosclerosis
3.- Membranous Nephropathy
4.- Diffuse Proliferative Glomerulonephritis
5.- Membranoproliferative Glomerulonephritis
6.- Focal Proliferative and Necrotizing
Glomerulonephritis p
7.- Crescentic Glomerulonephritis
8.- Mesangial Proliferative Glomerulonephritis
9.- Basement Membrane Abnormalities
10.- Focal Global Glomerulosclerosis
Copyright Harvard Medical School, 2010. All Rights Reserved.
378
CLINICAL CONDITIONS ASSOCIATED WITH
CRESCENTIC GLOMERULONEPHRITIS
Type I, anti-GBM disease
Type II, immune complex-mediated
Other primary glomerulonephritides:
post-infectious GN, IgA nephropathy, MPG
Systemic diseases (SLE, RA, H-S purpura)
Type III, pauci immune (ANCA-associated):
Idiopathic pauci-immune crescentic GN
Polyangiitis or SVV (ANCA-associated):
microscopic form of polyarteritis nodosa,
Wegener's granulomatosis
Churg-Strauss syndrome
Drug-induced vasculitides
Copyright Harvard Medical School, 2010. All Rights Reserved.
379
Fibrin
Copyright Harvard Medical School, 2010. All Rights Reserved.
380
Copyright Harvard Medical School, 2010. All Rights Reserved.
381
IgG
3-IV COLLAGEN AS TARGET
OF IMMUNE INJURY OF IMMUNE INJURY
Copyright Harvard Medical School, 2010. All Rights Reserved.
382
IMMUNE MECHANISMS OF FOCAL
NECROTIZING AND CRESCENT FORMATION
Linear staining for IgG Granular staining for IgG No staining for IgG
Anti-GBM disease Immune-complex pauci-immune
Goodpasture syndr. disease (SLE) disease
(ANCA)
Copyright Harvard Medical School, 2010. All Rights Reserved.
383
IgG
Copyright Harvard Medical School, 2010. All Rights Reserved.
384
IMMUNE MECHANISMS OF FOCAL
NECROTIZING AND CRESCENT FORMATION
Linear staining for IgG Granular staining for IgG No staining for IgG
Anti-GBM disease Immune-complex pauci-immune
Goodpasture syndr. disease (SLE) disease
(ANCA)
Copyright Harvard Medical School, 2010. All Rights Reserved.
385
Copyright Harvard Medical School, 2010. All Rights Reserved.
386
c-ANCA p-ANCA
Copyright Harvard Medical School, 2010. All Rights Reserved.
387
IMMUNE-MEDIATED DISORDERS
TYPE MECHANISM OF TISSUE INJURY
I. Immediate
Hypersensitivity
IgE antibodies immediate release of vasoactive
amines, lipid mediators, cytokines from mast cells -
it t f i fl t ll
II. Antibody-
mediated
III Immune
recruitment of inflammatory cells.
IgG, IgM antibodies; binding to target molecule, cell, or
matrix component:
A. loss or enhancement of cell/protein function
B. lysis of target cell via complement activation
C. antibody-dependent cell cytotoxicity
Antigen-antibody complexes in circulation Complement III. Immune
Complex-
mediated
IV. T Cell-mediated
Antigen antibody complexes in circulation Complement
and Fc receptor-mediated recruitment of neutrophils -
release of lysosomal enzymes and other toxic mediators
CD4
+
T cells macrophage activation cytokines -
inflammation
CD8
+
T cells - direct cytotoxicity cytokines - inflammation
TYPE II, ANTIBODY-MEDIATED
B. Antibody-dependent and cell-mediated (NK and
ADCC)
Copyright Harvard Medical School, 2010. All Rights Reserved.
388
ASYMPTOMATIC
HEMATURIA/PROTEINURIA
These conditions are characterized
morphologically either by focal
necrotizing and/or inflammatory lesions
of the glomeruli or by basement g y
membrane anomalies that result in
greater capillary fragility.
1.- Epithelial Cell Disease (Minimal Change Dis)
THE BASIC STRUCTURAL PATTERNS
OF GLOMERULAR INJURY
2.- Focal Segmental Glomerulosclerosis
3.- Membranous Nephropathy
4.- Diffuse Proliferative Glomerulonephritis
5.- Membranoproliferative Glomerulonephritis
6.- Focal Proliferative and Necrotizing
Glomerulonephritis Glomerulonephritis
7.- Crescentic Glomerulonephritis
8.- Mesangial Proliferative Glomerulonephritis
9.- Basement Membrane Abnormalities
10.- Focal Global Glomerulosclerosis
Copyright Harvard Medical School, 2010. All Rights Reserved.
389
MESANGIOPROLIFERATIVE
GLOMERULONEPHRITIS
IgA Nephropathy IgA Nephropathy
Recovery phase of a postinfectious
glomerulonephritis
SLE WHO Class II, mesangial form, and
other IC mediated diseases other IC-mediated diseases
Idiopathic Mesangioproliferative GN
Some of the deposition diseases
Copyright Harvard Medical School, 2010. All Rights Reserved.
390
IgA
Copyright Harvard Medical School, 2010. All Rights Reserved.
391
Asymptomatic
Hematuria and/or
Proteinuria
Acute
Nephritis
Rapidly
Progressive
Nephritis
Chronic
Nephritis
or ESKD
CLINICAL AND MORPHOLOGICAL EXPRESSION
OF IgA NEPHROPATHY AND HSP
Copyright Harvard Medical School, 2010. All Rights Reserved.
392
Unusual Presentations in IgA
Nephropathy
IgA Nephropathy with gross hematuria and
Acute Renal Failure Acute Renal Failure
IgA Nephropathy with superimposed
Nephrotic Syndrome and Minimal
Change-like lesions
IgA Nephropathy with Proteinuria and g y
Membranous Nephropathy
IgA Nephropathy with ANCA serology and
Necrotizing and Crescentic
Glomerulonephritis
Copyright Harvard Medical School, 2010. All Rights Reserved.
393
PATHOGENESIS OF IgA NEPHROPATHY
AND HSP
A glycosylation defect of the O-linked glycans in the
hinge region of IgA1 molecules is believed to: hinge region of IgA1 molecules is believed to:
reduced clearance from the circulation because of
lack of receptor engagement by the abnormal
IgA
increased aggregation of IgA in the circulation
resulting in mesangial trapping
development of immune complex-forming development of immune complex-forming
autoantibodies directed against the abnormal
IgA
increased affinity of the abnormal IgA for mesangial
matrix
CLINICAL CONDITIONS ASSOCIATED WITH
GLOMERULAR BASEMENT MEMBRANE
ABNORMALITIES
Hereditary nephritis (Collagen type IV): Hereditary nephritis (Collagen type IV):
Classical Alport syndrome
Autosomal recessive hereditary nephritis
Autosomal dominant hereditary nephritis
Thin basement membrane disease (TBMD)
Epstein and Fechtner syndromes (myosin HC IIA)
Pierson Syndrome (laminin 2) (CNS+eye abn.)
Nail-patella syndrome or hereditary osteo- p y y
onychodysplasia (LMX1B, LIM homeodomain
transcription factor)
Collagen III glomerulopathy, Fibronectin glomerulopathy
Lecithin-cholesterol acyltranferase deficiency
(Resolving immune complex mediated GN)
Copyright Harvard Medical School, 2010. All Rights Reserved.
394
Copyright Harvard Medical School, 2010. All Rights Reserved.
395
BASEMENT MEMBRANE COLLAGENS
COLLAGEN TYPE IV
GENE/CHR CHAIN EXPRESSION DISEASE
COL4A1/c13 1(IV) All BM, Mes Porencephaly and
COL4A2/ 13 2(IV) All BM M HANAC COL4A2/c13 2(IV) All BM, Mes HANAC
COL4A3/c2 3(IV) GBM, BC, dTBM arAS; adAS;TBM
COL4A4/c2 4(IV) GBM, BC, dTBM arAS; adAS;TBM
COL4A5/cX 5(IV) GBM, BC, dTBM X-AS
COL4A6/cX 6(IV) BC, dTBM X-AS +
Kashtan CE: Alport syndrome and thin glomerular basement membrane disease. J Am Soc Nephrol
9:1736-1750, 1998.
Gubler MC: Inherited disease of the glomerular basement membrane. Nature Clin Prac Nephrol 4:24-
37, 2008.
leiomyomatosis
Copyright Harvard Medical School, 2010. All Rights Reserved.
396
GENETICS OF HEREDITARY NEPHRITIS
DISEASE MUTATIONS
X linked HN COL4A5 (only 50%of cases): X-linked HN COL4A5 (only 50% of cases):
large deletions, missense mutations,
splice-site mutations, small deletions
Sporadic AS de novo mutation of COL4A5 or germ
line mutation of COL4A5 in mother
ar HN COL4A3 COL4A4 ar HN COL4A3, COL4A4
ad HN COL4A3, COL4A4
TBM COL4A4, COL4A3
1.- Epithelial Cell Disease (Minimal Change Disease)
2 Focal Segmental Glomerulosclerosis
THE BASIC STRUCTURAL PATTERNS
OF GLOMERULAR INJURY
2.- Focal Segmental Glomerulosclerosis
3.- Membranous Nephropathy
4.- Diffuse Proliferative Glomerulonephritis
5.- Membranoproliferative Glomerulonephritis
6.- Crescentic Glomerulonephritis
7.- Focal Proliferative and Necrotizing
Glomerulonephritis
8.- Mesangial Proliferative Glomerulonephritis
9.- Basement Membrane Abnormalities
10.- Focal Global Glomerulosclerosis
Copyright Harvard Medical School, 2010. All Rights Reserved.
397
CHRONIC RENAL FAILURE
CHRONIC NEPHRITIC SYNDROME
Th t t l i l t f thi d i The structural equivalent of this syndrome is
end-stage renal disease, with widespread
global glomerular obsolescence (sclerosis),
tubular atrophy, interstitial fibrosis, and
variable degree of arterial and arteriolar variable degree of arterial and arteriolar
sclerosis. A more precise diagnosis can often
be established by immunohistochemical and
ultrastructural studies.
Fabry
Copyright Harvard Medical School, 2010. All Rights Reserved.
398
Copyright Harvard Medical School, 2010. All Rights Reserved.
399
1.- Epithelial Cell Disease (Minimal Change
THE BASIC STRUCTURAL PATTERNS
OF GLOMERULAR INJURY
Disease)
2.- Focal Segmental Glomerulosclerosis
3.- Membranous Nephropathy
4.- Diffuse Proliferative Glomerulonephritis
5.- Membranoproliferative Glomerulonephritis
6.- Crescentic Glomerulonephritis
7 Focal Proliferati e and Necroti ing 7.- Focal Proliferative and Necrotizing
Glomerulonephritis
8.- Mesangial Proliferative Glomerulonephritis
9.- Basement Membrane Abnormalities
10.- Focal Global Glomerulosclerosis
Copyright Harvard Medical School, 2010. All Rights Reserved.
400
Disclosures
NONE NONE
Copyright Harvard Medical School, 2010. All Rights Reserved.
401
Cur r ent and Fut ur e
Ther apy of Lupus
Nephr i t i s
GERALD APPEL, MD
Professor of Clinical Medicine
Columbia University College of
Physicians and Surgeons Physicians and Surgeons
NY-Presbyterian Hospital
New York, New York
Financial Disclosures Financial Disclosures
Dr. Appel has research grants, consultanships
and served on speakers bureaus, adjudication
committees and scientific advisory boards y
of the following companies: Merck, Pfizer,
Bristol-Myers Squibb, Takeda, Roche,
Aspreva, Genentech, Amgen, OrthoBiotech
and QuestCor.
Copyright Harvard Medical School, 2010. All Rights Reserved.
402
Prognostic Features in LN Prognostic Features in LN
Histological Predictors Histological Predictors
Histologic Histologic Class IV (diffuse proliferative LN) Class IV (diffuse proliferative LN)
High Activity and High Activity and Chronicity Chronicity on Biopsy on Biopsy
Crescents and Interstitial fibrosis Crescents and Interstitial fibrosis Crescents and Interstitial fibrosis Crescents and Interstitial fibrosis
Segmental necrotizing lesions Segmental necrotizing lesions
Clinical Predictors Clinical Predictors
Hypertension Hypertension
Anemia Anemia
High baseline serum High baseline serum creatinine creatinine
Higher baseline Higher baseline proteinuria proteinuria
Delay in therapy Delay in therapy
Epidemiologic Predictors Epidemiologic Predictors
African American Race African American Race
Low socioeconomic status. Low socioeconomic status.
Appel G, Cameron JS in Comprehensive Clinical Nephrology 2007. Comprehensive Clinical Nephrology 2007.
Ancestry and Lupus Nephritis:
Poverty and Ancestry prediction of
progression
( Barr RG, et al. NDT 18:2039- 2046, 2003)
1
2
3
4
5
6
7
RR of
Progressio
n
0
Poverty
unadjusted for
Ancestry
Poverty
Adjusted for
Ancestry
Ancestry
unadjusted for
Poverty
Ancestry
Adjusted for
Poverty
Poverty unadjusted for Ancestry Poverty Adjusted for Ancestry
Ancestry unadjusted for Poverty Ancestry Adjusted for Poverty
Copyright Harvard Medical School, 2010. All Rights Reserved.
403
Outcomes in African Americans
and Hispanics with LN
213 LN patients
93 African American 100 Hispanic 20 93 African American, 100 Hispanic, 20
Caucasian
WHO II ( 9%) III(13%) IV ( 52%) V(26%)
AA had higher BP and Screat , and lower
household income.
Screat X2, ESRD higher in AA and Hispanic Screat X2, ESRD higher in AA and Hispanic
than Caucasians.
Both biologic and low income a factor.
Contreras et al KI 69:1846, 2006
Outcomes in African Americans and
Hispanics with LN
Contreras et al KI 69:1846, 2006
Free of doubling Screat, ESRD, death
Copyright Harvard Medical School, 2010. All Rights Reserved.
404
Long-term Follow-up of Protocol
Completers in WHO Class IV LN
50
Doubling SCr
Dialysis
P
a
t
i
e
n
t
s

(
%
)
40
30
20
10
50% Rise SCr
Doubling SCr
MP + CY
(n = 20)
MP alone
(n = 24*)
CY alone
(n = 21)
*14 of 24 patients received CY after study completion
Illei GG, et al. Ann Intern Med. 2001;135:248-257.
0
Copyright Harvard Medical School, 2010. All Rights Reserved.
405
Event Cy Therapy
(n =21)
Combination Therapy
(n =20)
n/n n/n
Hypertension 10/20 10/20 yp
Ischemic heart disease 1/19 4/19
Hyperlipidemia 7/20 8/19
Valvular heart disease 9/19 7/21
Avascular necrosis 6/21 6/20
Osteoporosis 4/18 3/19
Premat re menopa se 9/16 10/18 Premature menopause 9/16 10/18
Major infections 7/21 9/20
Herpes zoster infection 6/21 5/20
The Euro-Lupus Nephritis Trial
Multicenter prospecitive trial of 90 LN pts
with Proliferative LN ( WHO III IV Vc d ) with Proliferative LN ( WHO III,IV,Vc-d )
High dose IVCYT ( 6 mo IVP + 2 quarterly
pulses ) vs Low dose IV CYT ( IVP q 2
wks x 6 followed by AZA )
Follow 41 months
Houssiau et al. Arthritis & Rheumatisms 46: 2121-2131,
2002
Copyright Harvard Medical School, 2010. All Rights Reserved.
406
Euro Lupus Trial Euro Lupus Trial Treatment failure
)
LD
90
100
F
r
e
e

o
f

F
a
i
l
u
r
e

(
%
)
50
60
70
80
HD
HD
0 12 24 36 48 60
Follow-up (months)
50
0
LD
Houssiau Houssiau et al et al., Arthritis Rheum, 2002 ., Arthritis Rheum, 2002
Euro Lupus Trial Euro Lupus Trial - - Remission Remission
0.8
1
s
s
i
o
n
LD
HD
0.2
0.4
0.6
LD
P
r
o
b
a
b
i
l
i
t
y

o
f

r
e
m
i
s
HD
HD
Remission: < 10 RBC/hpf, 24-h proteinuria < 1g, no DSC
0
0 12 24 36 48 60
Follow-up (months)
Houssiau Houssiau et al et al., ., Arthritis Arthritis Rheum Rheum, 2002 , 2002
UCL UCL
Copyright Harvard Medical School, 2010. All Rights Reserved.
407
Euro Lupus Trial Renal Findings
3 0
3.5
4.0
r
i
a

(
g
)
LD
HD
1.3
1.4
(
m
g
/
d
l
)
HD
LD
0.5
1.0
1.5
2.0
2.5
3.0
2
4
-
h
o
u
r

P
r
o
t
e
i
n
u
r
0.8
0.9
1.0
1.1
1.2
S
e
r
u
m

C
r
e
a
t
i
n
i
n
e

p < 0.005 for repeated measures analyses (ANOVA)
p > 0.05 for between groups comparisons
0 3 12 6 0 3 6 12
Months Months
0 0
Houssiau Houssiau et al et al., ., Arthritis Arthritis Rheum Rheum, 2002 , 2002 UCL UCL
Euro Lupus Trial Euro Lupus Trial - - Renal Renal flares flares
100 LD
HD
LD
HD
80
60
40
20
F
r
e
e

o
f

r
e
n
a
l
f
l
a
r
e

(
%
)
Houssiau Houssiau et al et al., ., Arthritis Arthritis Rheum Rheum, 2002 , 2002
0 12 24 36 48 60
0
Follow-up (months)
Copyright Harvard Medical School, 2010. All Rights Reserved.
408
Euro Lupus Trial Euro Lupus Trial - Severe infection
o
n

(
%
)
LD
90
100
r
e
e

o
f

s
e
v
e
r
e

i
n
f
e
c
t
i
o
HD
50
60
70
80
HD
F
r
0 12 24 36 48 60
Follow-up (months)
50
0
LD
HD
Houssiau et al., Arthritis Rheum, 2002
UCL UCL
ELNT ELNT - - Baseline data Baseline data
Parameter Parameter All All
(n = 90) (n = 90)
HD HD
IV CPM IV CPM
(n = 46) (n = 46)
LD LD
IV CPM IV CPM
(n = 44) (n = 44)
Age (years) Age (years) 31 31 11 11 30 30 11 11 33 33 12 12
Females/Males (n) Females/Males (n) 84/6 84/6 43/3 43/3 41/3 41/3
Race Race
Caucasians (n) Caucasians (n)
Asians (n) Asians (n)
Afro Afro--Car./Blacks (n) Car./Blacks (n)
76 76
66
88
37 37
44
55
39 39
22
33
Past history Past history
Houssiau Houssiau et al et al., ., Arthritis Arthritis Rheum Rheum, 2002 , 2002
UCL UCL
yy
Past renal disease (n) Past renal disease (n)
Past GC treatment (n) Past GC treatment (n)
Past IS treatment (n) Past IS treatment (n)
21 21
55 55
77
11 11
27 27
44
10 10
28 28
33
Copyright Harvard Medical School, 2010. All Rights Reserved.
409
ELNT - 10 year FU - ESRD
HoussiauFA et al. Ann Rheum Dis 2009,
ELNT - 10 year FU
HoussiauFA et al. Ann Rheum Dis 2009, J an 20 (Epubaheadof print)
Copyright Harvard Medical School, 2010. All Rights Reserved.
410
Euro-Lupus Regimen achieves good
ELNT - 10 year FU - Conclusions
Euro Lupus Regimen achieves good
clinical results in the very long-term
Death and ESRD rates are low
mainly Caucasians
moderately severe LN moderately severe LN
longterm IS (GC and other IS)
anti-proteinuric therapy
referral centers
Efficacy of MMF vs sequential POCY-AZA
in 42 patients with DPLN
76%
81%
Compl ete
remi ssi on
Group 1: MMF
(2 g x 6 mo,
then 1 g x 6 mo)
33%
11%
14%
19%
15%
14%
Infecti on
Rel apse
Parti al
remi ssi on
then 1 g x 6 mo)
+ prednisone
(0.8 mg/kg)
Group 2: POCY
(2.5 mg/kg/d
x 6 mo), then
AZA (1.5-2.0
mg/kg/d) +
prednisone
10%
0%
0 20 40 60 80 100
Death
Chan TM et al. New Engl J Med 2000; 343:1156-62.
Pts (%)
p
Copyright Harvard Medical School, 2010. All Rights Reserved.
411
Long-Term Study of MMF as Continuous Induction and
Maintenance (n=32) Treatment for DPLN compared to
Sequential POCY-AZA (n=30)
10
13
Chroni c renal
fai l ure
Group 1: MMF
induction
(2 g x 6 mo, 1 g or
1 5 g x 6 mo then
36%
40%
30%
4%
13%
34%
Amenorrhea
Infecti ons
Rel apse
P=0.013
P=0.004
1.5 g x 6 mo, then
1 g x 12 mo or
followed by AZA
(1-1.5 mg/kg/d)
Group 2: POCY
(2.5 mg/kg/d
x 6 mo), then
AZA (1.5-2
/k /d 6
Chan TM et al. JASN 2005; April
7%
0%
0 10 20 30 40 50
Mortal i ty
Patients (%)
mg/kg/d x 6 mo,
then 1-1.5
mg/kg/d). Both
groups received
corticosteroids
Sequential Therapy for Proliferative LN IV Cy
Induction IV Cy vs. AZA vs. MMF
Maintenance
N = 59 93% F 33 yo 46% AA N 59 93% F 33 yo 46% AA
WHO III = 12 WHO IV = 46 Vb = 1
AI > 8 / 24 CI 1.9 - 3.6
HBP > 95% Active Serology
Neph Synd 64% Palb 2 7 g/dl Up/Ucr > 5 Neph Synd 64% Palb 2.7 g/dl Up/Ucr > 5
Pcreat 1.6 g/dl
U. Miami G. Contreras et al. NEJM 2004
Copyright Harvard Medical School, 2010. All Rights Reserved.
412
t
y
Patient survival
1.00
u
m
u
l
a
t
i
v
e

p
r
o
b
a
b
i
l
i
t
p = 0.11, MMF vs IVCY
p = 0.02, AZA vs IVCY
p = 0.33, MMF vs AZA
0.50
0.75
19 19 15 10 9 4 2 AZA
20 19 12 6 3 2 1 IVCY
20 20 14 11 6 2 2 MMF
Time (months)
C
u
0.00
0.25
0 12 24 36 48 60 72
Free of clinical event (death or CRF)
t
y
1.00
p = 0.049, MMF vs IVCY
p = 0.009, AZA vs IVCY
p = 0.503, MMF vs AZA
u
m
u
l
a
t
i
v
e

p
r
o
b
a
b
i
l
i
t
0.50
0.75
19 19 15 10 9 4 2 AZA
20 19 12 6 3 2 1 IVCY
20 20 14 11 6 2 2 MMF
C
u
Time (months)
0.00
0.25
0 12 24 36 48 60 72
Copyright Harvard Medical School, 2010. All Rights Reserved.
413
Relapse during maintenance phase
Free of Relapse
p = 0.021, MMF vs IVCY
p = 0.124, AZA vs IVCY
p = 0.222, MMF vs AZA
y
1.00
m
u
l
a
t
i
v
e

p
r
o
b
a
b
i
l
i
t
y
0 25
0.50
0.75
C
u
m
19 15 10 6 4 3 1 AZA
17 10 4 2 2 1 1 IVCY
19 17 12 8 3 2 1 MMF
0.00
0.25
0 12 24 36 48 60 72
Time (months)
Sequential LN Rx: IV CY vs AZA vs
MMF Maintenance Therapy
Side Effects of Therapy
Hospital Days
Per Patient
Year Amenorrhea Infection Major
(%) (%) (%)
IV CY 13 32 68 12
AZA 1* 7* 28* 3
MMF 1* 6* 21* 3 MMF 1 6 21 3
Contreras G, et al. NEJ M 2004
Copyright Harvard Medical School, 2010. All Rights Reserved.
414
Multicenter Trial of MMF vs IVCyc for
Induction Therapy of Severe LN
MMF (n=71) IVC (n=69)
Ginzler E. Appel G N Eng J Med Nov. 2005
( ) ( )
Age ( yrs) 32.5 10.0 31.0 9.0
Female 61 (86%) 65 (94%)
Black 43 (61%) 36 (52%)
Duration of SLE, mo. 43.72 66.88 58.70 80.64
Screatinine, mg/dL 1.06 0.52 1.08 0.49
Urine protein g/24 hr 4 06 3 14 4 41 3 51 Urine protein, g/24 hr 4.06 3.14 4.41 3.51
Urine sediment
RBC/hpf
WBC/hpf
24.1 50.3
12.6 23.5
33.2 115.5
10.3 17.3
Salbumin, g/L 2.81 0.95 2.69 0.56
Remission Rates: MMF vs. IVC
Intent to treat analysis
60
P=0.009
22.5
29.6
52.1
4
24.5
30.4
10
20
30
40
50
60
P=NS P=0.005
e
r
c
e
n
t

R
e
s
p
o
n
d
i
n
g
0
Complete Remission Partial Remission Complete +Partial
Remission
MMF IVC
P
e
Ginzler E. Appel G N Eng J Med Nov. 2005
Copyright Harvard Medical School, 2010. All Rights Reserved.
415
Change in Serum Creatinine and
Urine Protein Excretion
Serum
Creatinine
Urine
Protein
1
2
3
4
5
0.9
1
1.1
1.2
Creatinine
Protein
u
m

C
r
e
a
t
i
n
i
n
e

(
m
g
/
d
L
)
U
r
i
n
e

P
r
o
t
e
i
n

(
m
g
/
d
L
)
0
1
0 4 8 12 16 20 24
Weeks
0.8
0 4 8 12 16 20 24
Weeks
MMF IV CYC
S
e
r
u
NEJM 11/05 Study Last Outcomes
MMF IVC
Renal Flare 8 8
Renal Failure 4 7
Death 4 8
All p = NS at Mean follow-up 36 and 37
months
Ginzler E. Appel G N Eng J Med Nov. 2005
Copyright Harvard Medical School, 2010. All Rights Reserved.
416
Randomized (n = 370)
Open-label treatment
MMF
IVC
ALMS TRIAL RCT MMF vs IVC in Severe LN
Appel , Contreras, Dooley et al JASN 2009
Allocated to MMF
(n = 185)
Received MMF (n = 184)
Withdrawals (n = 35)
Due to adverse event (n = 24)
Consent withdrawn (n = 6)
Other reason (n = 5)
Allocated to IVC
(n = 185)
Received IVC (n = 180)
Withdrawals (n = 29)
Due to adverse event (n = 13)
Consent withdrawn (n = 5)
Other reason (n = 11)
Maintenance phase
Double-blind re-randomization to corticosteroids plus MMF or azathioprine for up to 3 years
Primary endpoint: responders in randomized population (n = 370)
Responders
ALMS TRIAL Primary Endpoint:
Responders at Month 6
100
Response judged by
blinded Clinical Endpoint
56.2%
53.0%
20
40
60
80
100
p
o
r
t
i
o
n

o
f

p
a
t
i
e
n
t
s

r
e
p
o
n
d
i
n
g

(
%
)
p
Committee:
Decrease in proteinuria
to <3g if baseline
nephrotic (3g/d) ,
or by 50% in patients ith
subnephrotic (<3g/d)
proteinuria
d
0
20
P
r
o
pr
and
Stabilization of serum
creatinine level (24-week
level 25% of baseline),
or improvement
MMF was not superior to IVC
(p = 0.575)
MMF IVC
Appel , Contreras, Dooley et al JASN 2009
Copyright Harvard Medical School, 2010. All Rights Reserved.
417
100
120
140
160

(

m
o
l
/
L
,

S
D
)
IVC
ALMS Trial - Renal Variables
0
20
40
60
80
S
e
r
u
m

c
r
e
a
t
i
n
i
n
e

MMF
7
8
9
g
/
d
a
y
,

S
D
)
Serum creatinine
and urine protein
0
1
2
3
4
5
6
Baseline 4 8 12 16 20 24 Endpoint
Week
2
4

h
o
u
r

u
r
i
n
e

p
r
o
t
e
i
n

(
g
and urine protein
levels improved
in both the MMF
and IVC groups
ALMS - Key Non-Renal Variables
MMF
MMF
IVC
Appel , Contreras, Dooley et al JASN 2009
40
60
80
100
120
a

c
o
n
c
e
n
t
r
a
t
i
o
n

(
U
/
m
L
,

S
D
)
MMF
IVC
20
30
40
50
u
m

a
l
b
u
m
i
n

(
g
/
L
,

S
D
)
IVC
0
20
Baseline Endpoint Baseline Endpoint Baseline Endpoint
Anti-dsDNA Complement C3 Complement C4
M
e
a
n

p
l
a
s
m
a
0
10
Baseline Endpoint
Albumin
S
e
r
u
Copyright Harvard Medical School, 2010. All Rights Reserved.
418
ALMS Trial - Adverse Events (AEs)
Occurring in 10% of Patients
MMF (n = 184)
AE n (%)
Diarrhea 52 (28)
IVC (n = 180)
AE n (%)
Nausea 82 (45)
Diarrhea 52 (28)
Headache 38 (20)
Peripheral edema 35 (19)
Arthralgia 29 (15)
Nausea 27 (14)
Hypertension 26 (14)
Nasopharyngitis 25 (13)
Vomiting 25 (13)
( )
Vomiting 68 (37)
Alopecia 64 (35)
Headache 47 (26)
Arthralgia 43 (23)
Leukopenia 38 (21)
Pyrexia 30 (16)
Edema peripheral 30 (16)
o t g 5 ( 3)
Cough 24 (13)
Anemia 23 (12)
Alopecia 20 (10)
Deaths 9(4.9)
Edema, peripheral 30 (16)
Nasopharyngitis 29 (16)
URI 28 (15)
Hypertension 25 (13)
Diarrhea 23 (12)
Deaths 5(3)
ALMS Trial - Summary
Study did not show that MMF was
superior to IVC
Overall response rates similar with MMF Overall response rates similar with MMF
and IVC in all renal and non-renal
parameters
Adverse Events for MMF and IVC were
broadly similar over 24 weeks, and
consistent with previous reports
Ongoing maintenance phase compares
MMF with azathioprine for up to 3 years
Appel , Contreras, Dooley et al JASN 2009
Copyright Harvard Medical School, 2010. All Rights Reserved.
419
MMF in LN with Poor Renal Function:
Analysis of the ALMS Data
29 Pts (18 MMF, 11 IVC).
Baseline: MMF vs IVC similar in age proteinuria ( 5 1 Baseline: MMF vs IVC similar in age, proteinuria ( 5.1
and 4.6 g/day ), chronicity on bx, and GFR ( 21 vs
24 ml/min).
No difference in composite outcome of response to
proteinuria and Scr;
GFR increased 20 ml/min more with MMF ;
Proteinuria decreased (0.8g/d) more with MMF.
25% of both groups had treatment limiting adverse 25% of both groups had treatment limiting adverse
events.
Conclusion: No evidence that IVC is more effective
than MMF in pts with severe LN.
Walsh M, Solomons N, Jayne D, for the ALMS group JASN 19: 780A, 2008.
Lupus Nephritis Class V
Copyright Harvard Medical School, 2010. All Rights Reserved.
420
Lupus Membranous Nephropathy:
I VC vs. MMF
84 pure Class V LN
in Ginzler Trial +
ALMS Trial
MMF group
N=42
IVC group
N=42
MMF for 24 wks
N=33
IVC for 24wks
N=32
Discontinued
N=9
Discontinued
N=10
CR: N=6
PR: N=17
NR: N=10
N 32
CR: N=2
PR: N=19
NR: N=11
Radhakrishnan J, Moutzouris D, Ginzler G, and Appel G J
Kidney Int. 77:152-160, 2009.
Absolute partial and Complete Remission
Rates
Membranous LN
21
23
25
19
21
6
17
10
15
20
N
u
m
b
e
r

o
f

p
a
t
i
e
n
t
s
IVC
MMF
Radhakrishnan J, Moutzouris D, Ginzler E, and Appel G
Kidney Int 77:152-160, 2009.
2
0
5
CR PR CR+PR
Copyright Harvard Medical School, 2010. All Rights Reserved.
421
ALMS Maintenance Trial: One of the Largest
Trials Ever Conducted in Lupus Nephritis
Vancouver 2010
24-wk induction phase 36-mo maintenance phase
MMF
1.5 g BID
Response
or
Remission Yes
Re-
randomization
IVC 0.51
g/m
2
Monthly
p 36 mo maintenance phase
MMF
1 g BID
AZA 2
mg/kg/d
*Oral corticosteroids administered in induction and maintenance phases
AZA, azathioprine; IVC, intravenous cyclophosphamide; MMF, mycophenolate mofetil
No further treatment
(exit study)
No
ALMS Maintenance Trial 2010
Kaplan-Meier Curve
Time to Treatment Failure ITT, N=227
1
e
e
Time to treatment failure MMF Time to treatment failure AZA MMF AZA
0.4
0.6
0.8
1
t
y

o
f

b
e
i
n
g
e
v
e
n
t

f
r
e
p = 0.003
0
0.2
month 3 month 6 month 9 month 12month 15month 18month 21month 24month 27month 30month 33month 36
P
r
o
b
a
b
i
l
i
t
3 6 9 12 15 18 21 24 27 30 33 36
Month
Copyright Harvard Medical School, 2010. All Rights Reserved.
422
ALMS Maintenance Trial
Subjects Achieving Primary and Key
Secondary Endpoints
100
%
)
MMF AZA
40
60
80

w
i
t
h

T
x

F
a
i
l
u
r
e

(
%MMF AZA
1 end-
point
2 endpoints
0
20
Tx failure ESRD Renal
flare
Creat
doubled
Rescue
meds
Broad
definition
P
a
t
i
e
n
t
s

ALMS Maintenance Trial 2010
MMF was superior to AZA in maintaining
renal response and preventing relapse in
subjects with active LN who responded to subjects with active LN who responded to
induction therapy with either MMF or IVC
Failure rate was 32% in the AZA group vs
16% in MMF group (p=0.005); completion
rate at 3 years was 63% for MMF vs 49% for
AZA
Superiority of MMF was consistent regardless
of induction treatment, race or region
Superiority of MMF was confirmed by
consistent results in key secondary endpoints
Copyright Harvard Medical School, 2010. All Rights Reserved.
423
ALMS Maintenance Trial
The pattern and frequency of AEs was
consistent with reported previously
for MMF and AZA for MMF and AZA
Serious AEs, and withdrawals due to
AEs, were more common in the AZA
group
This trial demonstrated improved
clinical benefit for MMF over AZA as
maintenance therapy for lupus
nephritis
LUNAR Rituximab Study Design
Rituximab + MMF (n=72)
Treatment Period
Screening
Follow-up Period Placebo + MMF (n=72)
Prednisone taper
Weeks
1 and 2
(Days 1 and 15)
Week
16
Weeks
24 and 26
(Days 168 and 182)
Week
52
Week
78
(Days 1 and 15) (Days 168 and 182)
=Study drug infusion.
=Corticosteroids:
1000 mg IV methlyprednisolone given at days 1 and then days 2, 3, or 4
Oral prednisone initiated at 0.75 mg/kg/day after IV steroids and then tapered to
10 mg/day by day 112
Copyright Harvard Medical School, 2010. All Rights Reserved.
424
Patient Disposition
Patients
Randomized 1:1
(N=144)
Placebo
(n=72)
Rituximab
(n=72)
9 Withdrawals Total
5 Lost to Follow-up
3 Patients Decision
5 Withdrawals Total
2 Lost to Follow-up
2 Deaths
Completed Week 52
(n=63)
88%
Completed Week 52
(n=67)
93%
1 Physicians Decision 1 Patients Decision
Primary Endpoint:
Renal Response at Week 52
54.1
60
Placebo (N=72) Rituximab (N=72)
30.6
15.3
26.4
30.6
43.0
10
20
30
40
50
P
r
o
p
o
r
t
i
o
n

o
f

P
a
t
i
e
n
t
s
P=0.55* P=0.55*
0
Complete Renal
Response (CRR)
Partial Renal
Response (PRR)
No Response (NR)
Mean MMF dose: Placebo: 2.40.62 g; Rituximab: 2.70.41 g
Furie R et al ACR 2009
Copyright Harvard Medical School, 2010. All Rights Reserved.
425
Conclusions
LUNAR is the largest randomized, placebo-controlled
trial to evaluate rituximab in LN
Although there were more responders with rituximab
(57% vs. 46%), there was no statistically significant
difference in primary or clinical secondary endpoints at
week 52
Rituximab had a statistically significant effect on levels of Rituximab had a statistically significant effect on levels of
anti-dsDNA and complement at week 52
Adverse and serious adverse events were similar in
frequency , with no new or unexpected safety signals
Remission rates in the multitarget therapy and IVCY
groups after 6 and 9 months (intention-to-treat )
Bao, H. et al. J Am Soc Nephrol 2008;19:2001-2010
Copyright Harvard Medical School, 2010. All Rights Reserved.
426
Probability of achieving complete remission for
patients treated with multitarget therapy or IVCY
Bao, H. et al. J Am Soc Nephrol 2008;19:2001-2010
The Future
More use of new drugs ( MMF, Rituximab, co-
stimulatory blockers ) stimulatory blockers ).
More use of combination therapies for
induction treatment.
Longer maintenance therapy.
Development of entirely new drugs (
t l l l bl k f t ki tolerance molecules, blockers of cytokines ,
etc )
Need for creativity not one regimen for all.
Copyright Harvard Medical School, 2010. All Rights Reserved.
427
Financial Disclosures
Dr. Appel has research grants, consultanships
and served on speakers bureaus, adjudication
committees and scientific advisory boards y
of the following companies: Merck, Pfizer,
Bristol-Myers Squibb, Takeda, Roche, Aspreva,
Genentech, Amgen, OrthoBiotech and QuestCor.
Copyright Harvard Medical School, 2010. All Rights Reserved.
428
Board Review Questions
Glomerular and
Parenchymal Disease
Brigham Hospital Boston August 2010 Brigham Hospital Boston August 2010
GERALD APPEL, MD
Professor of Clinical Medicine
Columbia University College of
Physicians and Surgeons
NY-Presbyterian Hospital
New York, New York
Financial Disclosures
Dr. Appel has research grants, consultanships and
served on speakers bureaus, adjudication
committees and scientific advisory boards y
of the following companies: Merck, Pfizer, Bristol-
Myers Squibb, Takeda, Roche, Aspreva,
Genentech, Amgen, OrthoBiotech and QuestCor.
Copyright Harvard Medical School, 2010. All Rights Reserved.
429
Question 1
Which patient is least likely to have
minimal change disease?
1) 65 year old WM with ARF-AKI and 4+
proteinuria
2) 32 year old WF with 2.1 g proteinuria
daily
3) 45 ld BM ith 9 t i i 3) 45 year old BM with 9 g proteinuria
daily
4) 28 year old WM with High BP,
microhematuria and 4+ proteinuria.
Question 1
Which patient is least likely to have
minimal change disease?
1) 65 year old WM with ARF-AKI and 4+
proteinuria
2) 32 year old WF with 2.1 g proteinuria
daily
3) 45 ld BM ith 9 t i i 3) 45 year old BM with 9 g proteinuria
daily
4) 28 year old WM with High BP,
microhematuria and 4+ proteinuria.
Copyright Harvard Medical School, 2010. All Rights Reserved.
430
Minimal Change Disease Minimal Change Disease
Copyright Harvard Medical School, 2010. All Rights Reserved.
431
Adult Minimal Change Disease:
Daily vs Alternate Day Steroids
95 Nephrotic adults with MCD
80% white, 7% black, 5% Hispanic p
Age: 45yo, Male: 61%
SCr: 1.4 mg/dl, SAlb: 2.2 g/dl, UPro: 10 g/d
Treatment:
Daily steroids (1 mg/kg/d for 26 wks): 65 pts
Alternate day steroids (2 mg/kg/qod for 26 wks):
23 pts
Remission: Remission:
CR= UPro <0.3 g/d
PR= >50% reduction in UPro from baseline
Baseline features similar among the 2 groups
Waldman M Appel G CJASN 2007
Steroids
Adult MCD:
Daily vs Alternate Day Steroids
Daily QOD p value
N 65 23
Duration (wks) 26 29 NS
Remission 76% 74% NS
CR 64% 77% NS
Relapse 75% 62% NS
Waldman M Appel G CJASN 2007
Copyright Harvard Medical School, 2010. All Rights Reserved.
432
TIME TO REMISSION
100
Adult MCD Time to Remission
Waldman Appel CJASN 2007
40
50
60
70
80
90
R
E
M
I
S
S
I
O
N

(
%
)
TIME TO REMISSION (ALL)
TIME TO REMISISION (QD)
TIME TO REMISISON (QOD) p =NS
0
10
20
30
0 2 4 6 8 10 12 14 16 18 20 22 24 26 28 30 32
TIME (WEEKS)
RELAPSE FREE SURVIVAL
100
Adult MCD Relapse Free Survival
Waldman Appel CJASN 2007
70
80
90
R
E
L
A
P
S
E

F
R
E
E

(
%
)
RELAPSE FREE SURVIVAL (ALL)
RELAPSE FREE SURVIVAL (QD)
RELAPSE FREE SURVIVAL (QOD)
p =NS
50
60
1 7 13 19 25
TIME (WEEKS)
Copyright Harvard Medical School, 2010. All Rights Reserved.
433
Question 2
A 38 year old WF has the nephrotic syndrome due to
biospy proven MCD. She has responded to a course py p p
of prednisone with a complete remission but has
relapsed x 3 whenever she discontinues the
prednisone. Which therapy will give the highest
remission rate for treatment at this time?
1) cyclosporine
2) l h h id 2) cyclophosphamide
3) tacrolimus
4) mycophenolate mofetil
5) All above are equivalent
Question 2
A 38 year old WF has the nephrotic syndrome due to
biospy proven MCD. She has responded to a course py p p
of prednisone with a complete remission but has
relapsed x 3 whenever she discontinues the
prednisone. Which therapy will give the highest
remission rate for treatment at this time?
1) cyclosporine
2) l h h id 2) cyclophosphamide
3) tacrolimus
4) mycophenolate mofetil
5) All above are equivalent
Copyright Harvard Medical School, 2010. All Rights Reserved.
434
Steroid-Dependent Adult MCD:
Response to Second Line Agents is Similar
80
10
20
30
40
50
60
70
P
e
r
c
e
n
t

P
t
s
0
10
CyT CyA Tac MMF ALL
CR PR
Waldman, Appel et al, CJ ASN 2007
Copyright Harvard Medical School, 2010. All Rights Reserved.
435
Question 3
Which clinical and laboratory features
t b h ti ti t i suggest an obese nephrotic patient is
more likely to have secondaryrather
than primary FSGS.
1) Black race and 9 g proteinuria/day
2) White race and 3 5g proteinuria/day 2) White race and 3.5g proteinuria/day
3) BMI >40 and absence of hematuria
4) BMI > 40 and plasma albumin < 2 g/dl
Question 3
Which clinical and laboratory features
t b h ti ti t i suggest an obese nephrotic patient is
more likely to have secondaryrather
than primary FSGS.
1) Black race and 9 g proteinuria/day
2) White race and 3 5g proteinuria/day 2) White race and 3.5g proteinuria/day
3) BMI >40 and absence of hematuria
4) BMI > 40 and plasma albumin < 2 g/dl
Copyright Harvard Medical School, 2010. All Rights Reserved.
436
Obesity Related Glomerulomegaly and
FSGS
Copyright Harvard Medical School, 2010. All Rights Reserved.
437
Obesity-related Glomerulopathy: An Emerging
Epidemic
Review >6800 Bxs 1986-2000
ORG 0.2% (1986-1990) 2.0% (1996-2000)
ORG (71) FSGS (50) ORG (71) FSGS (50)
Age /Race 43 yo /75% W 32 yo/52% W p<.01
NS 5.6% 54% p<.01
Salb 3.9 g/dl 2.9 g/dl p<.001
FSGS 18% 39%
Glomerulomeg 100% 10%
FPE 40% 75%
Double Pcreat 14% 50%
ESRD 3.6% 42%
Kambham, Markowitz, Valeri et al. Kidney Int 59:1498-1509. 2001.
Copyright Harvard Medical School, 2010. All Rights Reserved.
438
Kambham N, Markowitz G, Valeri A, et al. Kidney Int 59:1498-1509, 2001
Question 4
Which genetic defect of FSGS is not
i t d ith t l d i t associated with an autosomal dominant
inheritance?
1) alpha actinen 4
2) Inverted formin 2
3) Podocin 3) Podocin
4) TRPC6 channel
Copyright Harvard Medical School, 2010. All Rights Reserved.
439
Question 4
Which genetic defect of FSGS is not
i t d ith t l d i t associated with an autosomal dominant
inheritance?
1) alpha actinen 4
2) Inverted formin 2
3) Podocin 3) Podocin
4) TRPC6 channel
Copyright Harvard Medical School, 2010. All Rights Reserved.
440
Genetic Podocyte Mutations
Alpha actinin 4 familial autosomal
d i t FSGS dominant FSGS.
Podocin ( NPHS2 ) autosomal recessive
steroid resistant nephrotic syndrome
TRPC6 channel defect autosomal
dominant FSGS ( Nature Genetics 2005) dominant FSGS ( Nature Genetics 2005)
Formin INF2 gene autosomal dominant
(Nature Genetics 42:72, 2010)
Question 5
Which statement is NOT true about the
incidence of FSGS.
1) The incidence of FSGS is increasing in the
US and it is the most common pattern of
idiopathic nephrotic syndrome in Blacks.
2) The incidence of FSGS is increasing in
Caucasians in the US.
3) The incidence of FSGS in most other 3) The incidence of FSGS in most other
countries is as high as the US.
4) FSGS is the most frequent form of
idiopathic nephrotic syndrome to lead to
ESRD.
Copyright Harvard Medical School, 2010. All Rights Reserved.
441
Question 5
Which statement is NOT true about the
incidence of FSGS.
1) The incidence of FSGS is increasing in the
US and it is the most common pattern of
idiopathic nephrotic syndrome in Blacks.
2) The incidence of FSGS is increasing in
Caucasians in the US.
3) The incidence of FSGS in most other 3) The incidence of FSGS in most other
countries is as high as the US.
4) FSGS is the most frequent form of
idiopathic nephrotic syndrome to lead to
ESRD.
Copyright Harvard Medical School, 2010. All Rights Reserved.
442
Copyright Harvard Medical School, 2010. All Rights Reserved.
443
Nephrotic Syndrome: Main pathologies (%)
Span Italy
1
Dnmark
6
Japan
7
Korea
9
UA Emerits
19
MCD 17.1 12 23.5 37.7 59.4 26.2
FGS 14.1 12.3 8 9 11.1 15.4
MN 22.9 32.9 22.4 23.3 15.9 28.3
IgAN 4.5 2.4 17.5 19.2 3.3 3.2
RIVERA F, LPEZ-GMEZ JM, PREZ-GARCA R Kidney Int 66:898; 2004
Primary Glomerulonephritis in China
Type 1979-1999 2000-2008
IgAN 45% 45%
MsPGN 25% 25%
MN 10% 11%
FSGS 6% 9%
MPGN 3% 1% MPGN 3% 1%
CrGn 2% 1%
Lei-Shi LI, Zhi-Hong Liu KI 2004
Copyright Harvard Medical School, 2010. All Rights Reserved.
444
Trends in the progressive G.N*
1975 - 2005
N= 9256
75-79 80-84 85-89 90-94 95-99 00-04 Total
MGN 134 172 171 164 129 143 920
MPGN 90 67 33 46 37 31 304
FSGS 141 164 163 239 311 271 1289
IGA 129 215 227 262 309 356 1498
LUPUS 170 191 143 174 136 100 921
Vasculitis 29 66 76 93 76 68 411
* Toronto GN Registry
FSGS: Incidence in Olmstead MN
A retrospective study of 195 native renal
biopsies from 1974-2003.
All adult patients residing in Olmstead
County, MN
>90% Caucasian Pts.
1974-1983 1984-1993 1994-2003
5.7 % 17.9% 20.2%
Swaminathan S, Leung N, Lager DJ et al Clin JASN 1: 483, 22006.
Copyright Harvard Medical School, 2010. All Rights Reserved.
445
Question 6
Which therapy has been studied in a
t ll d d i d f hi i controlled randomized fashion in
adults with FSGS.
1) Mycophenolate mofetil
2) Cyclopsorine 2) Cyclopsorine
3) Rituximab
4) IV cyclophosphamide
Question 6
Which therapy has been studied in a
t ll d d i d f hi i controlled randomized fashion in
adults with FSGS.
1) Mycophenolate mofetil
2) Cyclopsorine 2) Cyclopsorine
3) Rituximab
4) IV cyclophosphamide
Copyright Harvard Medical School, 2010. All Rights Reserved.
446
Randomized Trial of Cyclosporine for
Steroid Resistant FSGS
Placebo (23) CyA (26)
Age 40 yo 38 yo Age 40 yo 38 yo
%Male 74% 65%
%W 87% 88%
BP 134/85 130/87
Salb(g/dl) 3.0 3.1
Screat(mg/dl) 1.4 1.3 Screat(mg/dl) 1.4 1.3
Urine prot (g/d) 8.7 6.9
D.Cattran, G.Appel, M.Pohl et al - N. Amer. N. Syn Study Group
Kidney Int 56:2220-2226, 1999
Copyright Harvard Medical School, 2010. All Rights Reserved.
447
Cyclosporin in FSGS: Remission
CSA: 26 pts
Pbo: 23 pts
p <0.001
Pbo: 23 pts
p <0.05
Cattran, Appel, Hebert et al, N Amer Nephrotic Synd Study Group
Kidney Int 56:2220-2226, 1999
Cyclosporine in SR-FSGS
52%
Placebo
p <0.05
25%
p <0 05
CSA
Cattran, Appel, Hebert et al, N Amer Neph Syn Study Group
Kidney Int 56:2220-2226, 1999
p <0.05
Copyright Harvard Medical School, 2010. All Rights Reserved.
448
Question 7
Which is a common feature of idiopathic
b h th ? membranous nephropathy?
1) Calcium oxalate stones
2) Uric acid stones
3) Renal vein thrombosis
4) Exudative pleural effusions
Question 7
Which is a common feature of idiopathic
b h th ? membranous nephropathy?
1) Calcium oxalate stones
2) Uric acid stones
3) Renal vein thrombosis
4) Exudative pleural effusions
Copyright Harvard Medical School, 2010. All Rights Reserved.
449
Thrombotic Abnormalities in the
Nephrotic Syndrome
Increased coagulation tendency
( plat. hyperaggregability, high fibrinogen and ( plat. hyperaggregability, high fibrinogen and
fibrinogen-fibrin transfer, decreased
fibrinolysis, low anti-thrombin III )
DVT, RVT, pulmonary emboli
Membranous NS greatest risk (up to 35% ) Membranous NS greatest risk (up to 35% )
Most RVT asymptomatic , but flank pain,
microhematuria, low GFR
Copyright Harvard Medical School, 2010. All Rights Reserved.
450
Copyright Harvard Medical School, 2010. All Rights Reserved.
451
Question 8
Which features on the biopsy of a patient
with membranous nephropathy y
suggest a secondary form of the
disease rather than primary?
1) Segmental scarring
2) Mesangial deposits 2) Mesangial deposits
3) Global glomeruloslcerosis
4) Subepithelial deposits without
spike formation
Copyright Harvard Medical School, 2010. All Rights Reserved.
452
Question 8
Which features on the biopsy of a patient
with membranous nephropathy y
suggest a secondary form of the
disease rather than primary?
1) Segmental scarring
2) Mesangial deposits 2) Mesangial deposits
3) Global glomeruloslcerosis
4) Subepithelial deposits without
spike formation
Copyright Harvard Medical School, 2010. All Rights Reserved.
453
Question 9
Recent evidence strongly suggests the
di f th ti i idi thi discovery of the antigen in idiopathic
membranous nephropathy. It is felt to
be which protein?
1) Thrombospondin 2
2) Beta glycoprotein 1 2) Beta glycoprotein 1
3) Beta catenin 4
4) Phospholipase A 2 receptor
Question 9
Recent evidence strongly suggests the
di f th ti i idi thi discovery of the antigen in idiopathic
membranous nephropathy. It is felt to
be which protein?
1) Thrombospondin 2
2) Beta glycoprotein 1 2) Beta glycoprotein 1
3) Beta catenin 4
4) Phospholipase A 2 receptor
Copyright Harvard Medical School, 2010. All Rights Reserved.
454
Phospholipase A2 Receptor
D. Salant et al NEJM 2009
Copyright Harvard Medical School, 2010. All Rights Reserved.
455
PLA2R and IgG4 Co-Localize in
Human iMGN
Modified from Beck et al, NEJM 2009
Anti-PLA
2
R: sensitivity and
specificity
Beck et al, ASN 09
Copyright Harvard Medical School, 2010. All Rights Reserved.
456
Anti-PLA2R antibodies in MN sera
parallels clinical course of disease
16
serum albumin (g/dl) urine protein:creatinine
6
8
10
12
14
16
0
2
4
J an 07 Apr 07 Oct 07 Dec 07 Mar 08 Apr 08
Modified from Beck et al, NEJM 2009
Question 10
Which therapy has NOT proven effective in
producing remissions of the nephrotic producing remissions of the nephrotic
syndrome in a randomized controlled trial
in idiopathic membranous nephropathy?
1) Tacrolimus
2) Alteranting months of corticosteroids and
cyclophosphamide cyclophosphamide
3) ACTH
4) Rituximab
Copyright Harvard Medical School, 2010. All Rights Reserved.
457
Question 10
Which therapy has NOT proven effective in
producing remissions of the nephrotic producing remissions of the nephrotic
syndrome in a randomized controlled trial
in idiopathic membranous nephropathy?
1) Tacrolimus
2) Alteranting months of corticosteroids and
cyclophosphamide cyclophosphamide
3) ACTH
4) Rituximab
Copyright Harvard Medical School, 2010. All Rights Reserved.
458
Changes in Daily Protein Excretion in 42 Treated
Pts and 39 Controls During 2 Years Follow-up
*
*
* *
P = 0.00001
Control
group
Treatment
group
7
6
5
4
3
2
1
Months
B 6 12 18 24
*P <0.01 vs basal value.
Ponticelli C et al. N Engl J Med. 1989;320:8-13.
Changes in Reciprocals of Creatinine in 30
Treated and 25 Control Pts Followed for >5 Years
*
*
P = 0.0041
Control
group
Treatment
group
0.014
0.013
0.012
0.011
0.010
0.009
0.008
0 007
*
Plasma Creatinine
(mol/L)
1
*P <0.0002 vs basal value.
Ponticelli C et al. N Engl J Med. 1989;320:8-13.
Months
B 12 24
0.007
0.006
36 48 60
Copyright Harvard Medical School, 2010. All Rights Reserved.
459
Cumulative Probability of PR or CR, or CR Alone of Neph.
Syn. in Pts w Methylprednisolone plus Chlorambucil (- - -)
or Methylprednisolone plus Cyclophosphamide ()
100
Partial or complete remission
20
40
60
80
Complete remission
Percent
0
20
0 6 12 18 24 30 36 42 48 54 60
Months
Ponticelli C et al. J Am Soc Nephrol. 1998;9:444-450.
Cyc l ospor i ne i n Pat i ent s w i t h St er oi d-
r esi st ant Membr anous Nephr opat hy: A
Randomi zed Tr i al .
n=51 RX for 26 wks
Cattran Appel Kidney I nt. 59:1484-90, 2001.
Copyright Harvard Medical School, 2010. All Rights Reserved.
460
Tacrolimus Monotherapy in Membranous
Nephropathy: A randomized controlled trial
Praga M, Barrio G, Juarez GF, et al. Kidney Int 71:924-930,2007
Rituximab in Idiopathic Memb. Nephropathy:
Time Course of Proteinuria and S Creatinine.
J Am Soc Nephrol 14:1851-1857, 2003
Copyright Harvard Medical School, 2010. All Rights Reserved.
461
Rituximab Therapy of Idiopathic
Membranous Nephropathy
Fervenza FC, Cattran D Kidney Int. 73:117-126 ,2008
Steroids + Cyclophosphamide vs ACTH in
Membranous NS
Ponticelli et al. 2006 AJKD
Copyright Harvard Medical School, 2010. All Rights Reserved.
462
Controlled Trial in MN Ponticelli et al. AJKD 2006
32 pts with idiopathic MN ( mean proteinuria 5-7 g/day)
randomized to steroids plus cyclohosphamide (A) vs ACTH (B)
MethylPred + cyclophosphamide ACTH
16000
ACTHAR: 1
st
ten American patients with
iMN treated with ACTH
2000
4000
6000
8000
10000
12000
14000
P
r
o
t
e
i
n
u
r
i
a

(
m
g
/
d
a
y
)
Pre-treatment proteinuria
Last Proteinuria
0
1 2 3 4 5 6 7 8 9 10
Pati ents wi th membranous
nephropathy
Copyright Harvard Medical School, 2010. All Rights Reserved.
463
Question 14
A 22 yo WM with a 2 yr
history of proteinuria and
hematuria has this biopsy. hematuria has this biopsy.
Which is NOT likely to be
found with this lesion?
1) Low C4 level
2) C3Neph
3) Low factor H level
4) Low C3 level
Question 14
A 22 yo WM with a 2 yr
history of proteinuria and
hematuria has this biopsy. hematuria has this biopsy.
Which is NOT likely to be
found with this lesion?
1) Low C4 level
2) C3Neph
3) Low factor H level
4) Low C3 level
Copyright Harvard Medical School, 2010. All Rights Reserved.
464
MPGN Type II DDD
Rare disease w deposition of electron dense
material w/i GBM and Bruchs memb. of eye.
M t 5 15 t d Most 5-15 yo at dx.
Within 10 yr 50% ESRD.
Uncontrolled activation of alternate pathway
of complement cascade.
Loss of complement regulation by C3
Nephritic Factor, an autoantibody directed
i t C3 t against C3 convertase.
Some patients mutation of H gene ( plasma
replacement may help ).
Recurs in most allografts associated with
renal failure.
Copyright Harvard Medical School, 2010. All Rights Reserved.
465
Concept of pathogenesis of
complement-mediated renal diseases
Endothelial
cells
Glomerular
capillary loop
CFH deficiency/defect
C3NeF
Impaired C Alt Path control
in fluid phase
CFH, IF, MCP
mutations
aHUS
Impaired C Alt Path control
on surfaces
GBM
DDD
GN C3 +MPGN
Servais et al, J Med Genet 2007
Question 15
A 32 year WM ex IV drug abuser presents with edema.
He has a normal BP and physical exam is negative
t f 2/6 t li t th d except for 2/6 systolic murmur at the apex and a non
blanching purpuric rash on his legs.
He has a serum creatinine of 1.8 mg/dl , U/A with
dysmorphic rbcs and 3.6 g proteinuria daily
The following tests are negative ANA,ANCA, ASLO,
SPEP. Serum total complement is markedly reduced
with a normal C3 and a low C4 level. with a normal C3 and a low C4 level.
USG reveal normal sized kidneys with increased
echogenicity.
Copyright Harvard Medical School, 2010. All Rights Reserved.
466
Question 15
Which diagnosis is most likely?
1) HIVAN
2) MPGN due to HCV
3) Proliferative GN due to SBE
4) AA amyloidosis
Copyright Harvard Medical School, 2010. All Rights Reserved.
467
Question 15
Which diagnosis is most likely?
1) HIVAN
2) MPGN due to HCV
3) Proliferative GN due to SBE
4) AA amyloidosis
Copyright Harvard Medical School, 2010. All Rights Reserved.
468
Question 16
27 yo F stock analyst has had 3 spontaneous abortions, and
Raynauds phenomenon. 2 years ago she developed a DVT of Raynaud s phenomenon. 2 years ago she developed a DVT of
the calf after a long auto trip. She develops arthralgias, low
temps, and malaise.
Px BP 152/92, P 84, malar flush, 2/6 SEM, swollen MCP and PIP
joints, livedo reticularis of legs and 2+ ankle edema.
Lab: WBC 3.6 K, Hct 24%, plts 89K
U/A 4+ prot, 3+ heme, 8-15 rbc, + rbc casts
BUN 43 mg/dl, Pcreat 2.6 mg/dl, 24 hr UV prot 1.2g/d , PT 14.6,
PTT 85. ANA + 1:160, anti-dsDNA negative, C3 and C4 borderline
low
Question 16
Renal biopsy is most likely to show
hi h f th f ll i ? which of the following ?
1) Diffuse proliferative GN
2) Focal proliferative GN
3) Thrombotic microangiopathy
4) Membranous lupus nephropathy
Copyright Harvard Medical School, 2010. All Rights Reserved.
469
Question 16
Renal biopsy is most likely to show
hi h f th f ll i ? which of the following ?
1) Diffuse proliferative GN
2) Focal proliferative GN
3) Thrombotic microangiopathy
4) Membranous lupus nephropathy
Copyright Harvard Medical School, 2010. All Rights Reserved.
470
Antiphospholipid Antibodies
Family of autoantibodies ( IgG, IgM,IgA ) against
negatively charged phospholipids negatively charged phospholipids
Lupus Anticoagulant prolongs lipid dependent
coagulation tests, interferes with phospholipid of
the prothrombin activator complex.
Anticardiolipin Antibodies- bind to cardiolipin -
the antigen used in VDRL syphilis assay (+VDRL) the antigen used in VDRL syphilis assay (+VDRL)
APL Antibodies have a procoagulant effect!
The Euro-phospholipid study:
Presenting Manifestation
DVT (32%)
Hemolysis
DVT (32%)
Thrombocytopenia (22%)
Livedo (20%)
Stroke
Superficial thrombophlebitis
Pulmonary embolism
Fetal loss
Hemolysis
Skin ulcers
Seizures
Vasculitic skin lesions
Myocardial infarction
Amaurosis Fugax
Fetal loss
TIA
Renal manifestations 2.2%
g
Digital Gangrene
Copyright Harvard Medical School, 2010. All Rights Reserved.
471
Copyright Harvard Medical School, 2010. All Rights Reserved.
472
Question 17
A 75 year old white male presents with a
three week lowgrade fever malaise and a three week low grade fever, malaise and a
petechial skin rash. Bx of the rash shows
leukocytoclastic vasculitis.
His creatinine is 4.8 mg/dl and he has 2.4 g
proteinuria daily with red cells and rbc
casts on U/A MPO ANCA is strongly casts on U/A. MPO ANCA is strongly
positive. Bx shows a crescentic GN.
Question 17
Which statement is true about treatment of this
patients disease? p
1) PO cyclophosphamide is superior to IV
cyclophosphamide in inducing remissions.
2) IV rituximab and IV cyclophosphamide are
equally effective at inducing remissions.
3) IV rituximab is superior to IV 3) IV rituximab is superior to IV
cyclophosphamide at inducing a remission.
4) If effectively treated he will have low
morbidity and mortaliy from therapy.
Copyright Harvard Medical School, 2010. All Rights Reserved.
473
Question 17
Which statement is true about treatment of this
patients disease? p
1) PO cyclophosphamide is superior to IV
cyclophosphamide in inducing remissions.
2) IV rituximab and IV cyclophosphamide are
equally effective at inducing remissions.
3) IV rituximab is superior to IV 3) IV rituximab is superior to IV
cyclophosphamide at inducing a remission.
4) If effectively treated he will have low
morbidity and mortaliy from therapy.
Copyright Harvard Medical School, 2010. All Rights Reserved.
474
RITUXIVAS STUDY
RITUXIVAS STUDY
1
.
0
0
o
n
RTX CYC
0
.
2
5
0
.
5
0
0
.
7
5
P
r
o
p
o
r
t
i
o
n

A
c
h
i
e
v
i
n
g

R
e
m
i
s
s
i
o
Sustained
remission
25/33
(76%)
9/11
(82%)
No
sustained
2
incomplete
1
incomplete
0
.
0
0
P
0 100 200 300 400
Time (days)
Cyclophosphamide Rituximab
remission
p
response
p
response
Copyright Harvard Medical School, 2010. All Rights Reserved.
475
Survival in Renal Vasculitis
Trial GFR (ml/min) Age(yrs) Mortality
CYCAZAREM NEJM, 2003 49 58 5%
CYCLOPS ASN, 2006 38 57 9.5%
RITUXVAS NEJM2010 17 68 18%
MEPEX JASN, 2007 <10 66 25%
Question 18
A 4 yo WF goes to a petting zoo with her parents .
She cuddles the sheep, calves, and other baby
animals She does not wash her hands immediately animals. She does not wash her hands immediately
afterwards.
3 d later she develops abdominal cramps, diarrhea,
and bloody stools. N/V, fever. She is given
antibiotics by her pediatrician.
Day 6 later there are echymoses of extremities and
lips, thrombocytopenia, oliguria, and seizures. p , y p , g ,
Stool is + for E.coli O157:H7.
Copyright Harvard Medical School, 2010. All Rights Reserved.
476
Question 18
Which therapy has proven effective for this
di ? disease?
1) IV quinolone antibiotics
2) Plasma exchange therapy
3) Synsorb shiga toxin binding agent
4) None of the above
Question 18
Which therapy has proven effective for this
di ? disease?
1) IV quinolone antibiotics
2) Plasma exchange therapy
3) Synsorb shiga toxin binding agent
4) None of the above
Copyright Harvard Medical School, 2010. All Rights Reserved.
477
DANGER! Petting Zoo!
Copyright Harvard Medical School, 2010. All Rights Reserved.
478
Glomerulus with thrombus
Copyright Harvard Medical School, 2010. All Rights Reserved.
479
Copyright Harvard Medical School, 2010. All Rights Reserved.
480
Therapy of STx-HUS D+
?Antibiotics Yes if sepsis, Yes w Shigella dysenteria ?Antibiotics Yes if sepsis, Yes w Shigella dysenteria
Type I disease, others No
Supportive care Dialysis, ACE ARBs , BP Control
Lack of efficacy
Heparin, fibrinolytic agents, platelet txs, PTE
plasma infusion, IVIG, antioxidants, Steroids, anti-plt
agents, vincristine, splnx.
SYNSORB Pk ( STx binding agent ) failed in acute
trials.
Transplant effective and safe.
Copyright Harvard Medical School, 2010. All Rights Reserved.
481
Question 19
A 36 year old previously healthy AA female develops
fever and bruising . She goes to her LMD and CBC
shows plats 15 K, Hct 28%, normal PT and PTT.
Smear microangiopathic hemolytic anemia. Over Smear microangiopathic hemolytic anemia. Over
next few days her urine output declines and she
develops sudden blindness followed by decreased
mental status.
What should be done next for this patient?
1) Send for an assay for the metalloprotease
ADAMTS 13.
2) Institute immediate plasma exchange and
corticosteroid therapy.
3) Institute immediate heparin and start coumadin
anticoagulation.
4) Start rituximab and corticosteroids.
Question 19
A 36 year old previously healthy AA female develops
fever and bruising . She goes to her LMD and CBC
shows plats 15 K, Hct 28%, normal PT and PTT.
Smear microangiopathic hemolytic anemia. Over Smear microangiopathic hemolytic anemia. Over
next few days her urine output declines and she
develops sudden blindness followed by decreased
mental status.
What should be done next for this patient?
1) Send for an assay for the metalloprotease
ADAMTS 13.
2) Institute immediate plasma exchange and
corticosteroid therapy.
3) Institute immediate heparin and start coumadin
anticoagulation.
4) Start rituximab and corticosteroids.
Copyright Harvard Medical School, 2010. All Rights Reserved.
482
Question 20
A 62 yo W M plumber with mild HBP is
l t df t i i d d evaluated for proteinuria and edema.
Px BP 142/84, Cor S1S2 S4 G, Chest clr,
Abd no LKKS, ext 2-3+ pedal edema
Lab. BUN 32, Pcreat 1.2, U/A 4+ prot, 24
hr UV prot 4 2 g/d hr UV prot 4.2 g/d,
ANA , complement , Hep B-C, neg
Spep 1.8 g IgG kappa, Palb 2.8 g/dl
Copyright Harvard Medical School, 2010. All Rights Reserved.
483
Question 20
His Biopsy might show all EXCEPT which
of the Following?
1
22
3
4
3
4
Question 20
His Biopsy might show all EXCEPT which
of the Following?
1
22
3
4
3
4
Copyright Harvard Medical School, 2010. All Rights Reserved.
484
Financial Disclosures
Dr. Appel has research grants, consultanships and
served on speakers bureaus, adjudication
committees and scientific advisory boards y
of the following companies: Merck, Pfizer, Bristol-
Myers Squibb, Takeda, Roche, Aspreva,
Genentech, Amgen, OrthoBiotech and QuestCor.
Copyright Harvard Medical School, 2010. All Rights Reserved.
485
The Challenging
Biopsy Boston
August 2010 August 2010
Jerry Appel, MD FASN
Helmut Rennke MD FACP
Financial Disclosures
Dr. Appel has research grants, consultanships
and served on speakers bureaus, adjudication
committees and scientific advisory boards y
of the following companies: Merck, Pfizer,
Bristol-Myers Squibb, Takeda, Roche, Aspreva,
Genentech, Amgen, OrthoBiotech and QuestCor.
Copyright Harvard Medical School, 2010. All Rights Reserved.
486
CASE 1
An 82 yo WM presents with ARF.
P ti t h h f HBP 8 Patient has a hx of HBP x 8 yrs, once
told high BS diet only, gout, s/p small
MI 4 yrs ago. Meds: Toprol XL,
lisinopril, allopurinol.
4 mo PTA BUN 28 mg/dl Pcreat 1.4
/dl O 2 3 t d mg/dl. Over 2-3 mo noted some
increase in normal mild ankle
swelling.
CASE 1
3 wks PTA developed >15 # weight gain and
progressive edema of feet ankles and legs progressive edema of feet, ankles, and legs.
2-3 days PTA developed Nausea/vomiting
and abdominal cramps. To cardiologist who
ordered labs. BUN 115 mg/dl Pcreat. 7.4
mg/dl.
24 hr urine protein 18 g/day 24 hr urine protein 18 g/day.
U/A 4+ prot. 5-10 rbc, 0-5 wbc, 0 casts.
ANA , CH50, HBV-HCV neg.
Copyright Harvard Medical School, 2010. All Rights Reserved.
487
Case 1
1) What other serologic tests should be
d h ? done here?
2) Should renal Biopsy be done now?
vs Wait and follow Pcreatinine?
3) Is there data on biopsies in pts > 80
years old? years old?
4) What is likely Dx?
Kidney Biopsy
Copyright Harvard Medical School, 2010. All Rights Reserved.
488
Copyright Harvard Medical School, 2010. All Rights Reserved.
489
Copyright Harvard Medical School, 2010. All Rights Reserved.
490
Renal biopsy in patients aged >80 yo
Nephropathology Associates, Little Rock, AR
Files examined from 2001- 2003 to identify biopsies in
>80
100 patients out of 3,227 native biopsies (3.1%)
Comparisons made with group aged 65 79 years
Nair et al AJ KD 2004 44;4: 618-626
Comparisons made with group aged 65-79 years
Copyright Harvard Medical School, 2010. All Rights Reserved.
491
Presentation by renal syndromes
40
45
0
15
20
25
30
35
%

o
f

p
a
t
i
e
n
t
s
>80 years
65-79 years
Nair et al AJ KD 2004 44;4: 618-626
0
5
10
NS Nephr RPGN Prot/Hem ARF CRF
Idiopathic nephrotic syndrome
45
50
15
20
25
30
35
40
%

o
f

p
a
t
i
e
n
t
s
> 80 years
65 - 79 years
Nair et al AJ KD 2004 44;4: 618-626
0
5
10
MCD FSGS MN
Copyright Harvard Medical School, 2010. All Rights Reserved.
492
Most common diagnoses
18
20
> 80 years65 - 79 ye
4
6
8
10
12
14
16
%

o
f

p
a
t
i
e
n
t
s
0
2
Crescentic GNFSGS MCD IgA Amyloid MN MPGN SLE
Nair et al AJ KD 2004 44;4: 618-626
Renal biopsy in patients > 80 years
Renal Pathology Registry at Laboratory at
Columbia University Medical Center NY Columbia University Medical Center, NY
Files examined from 2005 - 2008 to identify
biopsies in patients aged >80 years old
235 patients identified - the largest series for
Moutsakis D-A, Herlitz L, Appel GB, et al.. Renal Biopsy in the
Very Elderly. Clinical J Amer Soc Nephrol. 4: 1073-1082, 2009.
patients >80
Copyright Harvard Medical School, 2010. All Rights Reserved.
493
Clinical Presentation
46.4
45
50
23.8
13.2
9.4
5.5
5
10
15
20
25
30
35
40
%

o
f

p
a
t
i
e
n
t
s
0.4 1.3
0
5
AKI CRF NS AKI
and
NS
Prot Prot
and
Hem
Hem
Moutsakis D-A, Herlitz L, Appel GB, et al.. Renal Biopsy in the Very
Elderly. Clinical J Amer Soc Nephrol. 4: 1073-1082, 2009.
Nephrotic Syndrome in >80
20
25
5
10
15
20
%

o
f

p
a
t
i
e
n
t
s
0
MN Amyloid MCD IgA DM MPGN pi - GN
Moutsakis Moutsakis D D--A, A, Herlitz Herlitz L, L, Appel Appel GB, et al.. Renal Biopsy in the Very GB, et al.. Renal Biopsy in the Very
Elderly. Clinical J Elderly. Clinical J Amer Amer Soc Soc Nephrol Nephrol. 4: 1073 . 4: 1073- -1082, 2009. 1082, 2009.
Copyright Harvard Medical School, 2010. All Rights Reserved.
494
Clinical Presentation per diagnosis
120
ARF CRF NS ARF/NSProt Prot&He
20
40
60
80
100
%

o
f

p
a
t
i
e
n
t
s
0
pauci GN2o FSGS
HTN
HTN NS IgA MN Amyl MCD Myel Cast
Moutsakis Moutsakis D D--A, A, Herlitz Herlitz L, L, Appel Appel GB, et al.. Renal Biopsy in the Very GB, et al.. Renal Biopsy in the Very
Elderly. Clinical J Elderly. Clinical J Amer Amer Soc Soc Nephrol Nephrol. 4: 1073 . 4: 1073- -1082, 2009. 1082, 2009.
Primary Diagnosis # of Cases % of Cases
Most Common Renal Biopsy Diagnoses in 259 Adults >60 Years
Old Presenting with Acute Renal Insufficiency
AJKD 35: 433-447, 2000
Primary Diagnosis # of Cases % of Cases
Pauci-immune crescentic GN 79 31
Acute interstitial nephritis 47 19
ATN + nephrotic syndrome 19* 8
Atheroemboli 18 7
ATN 17 7
Light chain cast nephropathy 15 6 Light chain cast nephropathy 15 6
Post-infectious GN 14 6
Anti-GBM nephritis 10 4
*15 of 19 patients had minimal change disease, 3 FSGS, 1
membranous
Copyright Harvard Medical School, 2010. All Rights Reserved.
495
Etiologies of ARF with MCD
Ischemic tubular injury
Interstitial edema
Volume depletion secondary diuretics
Reduced intraglomerular pressure w/
ACEi/ARBs
NSAIDs
Serum Creatinine
<1.5 mg/dl >2.0 mg/dl
Number of patients 50 21
Adult minimal change glomerulopathy with
acute renal failure
Number of patients 50 21
Serum creatinine 1.0 + 0.2 5.5 + 3.3*
Age 40 + 16 60 + 16*
Systolic BP 138 + 27 158 + 24*
Diastolic BP 85 + 13 89 + 7
Serum albumin (g/dl) 2.7 + 1.0 2.1 + 0.8** Serum albumin (g/dl) 2.7 + 1.0 2.1 + 0.8
Proteinuria (g/24h) 7.9 + 5.6 13.5 + 9.4*
Arteriosclerosis (0-4 scale) 0.7 + 0.9 1.7 + 1.4*
**p <0.01 **p <0.05
Falk RJ, Jennette JC. Adult minimal change glomerulopathy with acute renal failure. Am J Kidney Dis 16: 432-
437, 1990
Copyright Harvard Medical School, 2010. All Rights Reserved.
496
Features,Treatment, and Outcome of
Adult MCD
95 Bx MCD at CUMC from 1990-2005
61%F 45yo 81%W 61%F 45yo 81%W
GFR 72 cc/min Pcreat 1.4 mg/dl Palb 2.2 g/dl
24hProt 9.93 g/d hematuria 20% HBP 43%
ARF 17% (eventual 24%)
88 Rx stds 65 Daily v 23 QOD 76v74% response
No Dfference QD vsQOD or in time to response ( 11 v No Dfference QD vsQOD or in time to response ( 11 v
16 wks ) or percent relapse ( 75 v 63% )
Of responders 60% relapse , most respond again
54% Rx w cyA, cytoxan, MMF, or Tacrolimus
Waldman MAppel GB Clin JASN 2:445-454 2007
Case 2
54yo WF w DM x15 yrs and Nephrotic Syndrome.
DM x 15 yrs, HBP x15yrs, obesity, GERD y y y
1 yr ago 2-3+proteinuria on dipstick; Pcreat 0.8 mg/dl
Over 4 months increasing edema, Pcreat. Increased
from 1.0 to 1.9 mg/dl and proteinuria increased from
0.8 g/day to 20 g/day.
Saw opthalmologist 1 month ago no clear -
retinopathy.
Meds: ramipril 10 mg, furosemide 40mg, glyburide 5 Meds: ramipril 10 mg, furosemide 40mg, glyburide 5
mg/d
Px BP 140/92, P64, Wt 232# Ht 55
HENT, Cor, Chest, Abd all neg. Ext 2+pedal and
pretibial edema
Copyright Harvard Medical School, 2010. All Rights Reserved.
497
Case 2
Laboratory:
BUN 28 /dl PC t 1 9 /dl BUN 28 mg/dl, PCreat. 1.9 mg/dl
U/A 4+ prot. 2+heme 5-10 rbc 0-4 wbc 0
casts
BS 132 mg/dl HgA1C 6.5
Palb 3.2 g/dl Pcholesterol 235 mg/dl Palb 3.2 g/dl Pcholesterol 235 mg/dl
USG 12-13 cm non-echogenic kidneys
ANA, serum complement, Spep-Upep,
ANCA, all negative or wnl
Case 2
Is this most likely to be isolated DM
h th ? Wh h t? nephropathy? Why or why not?
What physical exam finding might help
you?
If there is another dx what is it likely to be?
Copyright Harvard Medical School, 2010. All Rights Reserved.
498
Kidney Biopsy
Copyright Harvard Medical School, 2010. All Rights Reserved.
499
Copyright Harvard Medical School, 2010. All Rights Reserved.
500
Bx 168 TypeII DM w R.Disease
All documented DM.
All some indication for BX.
Eye fundoscopy of NO Predictive Value
Predictors another disease w or w/o DM
changes.
1) ARF
2) Very active urine sediment
3) Sudden onset nephrotic proteinuria
Bx findings in DM with other RD
DN/NDRDn=32, NDRDn=43 /168 DN/NDRD n 32, NDRD n 43 /168
Glomerular Diseases
Fsgs (14 ), MN ( 6 ), IgA ( 3), HSP ( 3 ),
MPGN ( 4 ), other (5 )
T b l i t titi l di ( 28) ( AIN 10 ) Tubulointerstitial dis ( n=28) ( AIN = 10 )
Vascular disease ( n = 15 )
Atheroemboli ( n = 8 )
Copyright Harvard Medical School, 2010. All Rights Reserved.
501
Case 3
48 yo WF interior designer w asymptomatic proteinuria
Pt in excellent health w no significant med Hx; Fam Pt in excellent health w no significant med Hx; Fam
Hx neg.; denies drug use or meds.
Routine exec. Px. Wnl BP 132/84 mm Hg
Noted proteinuria and refer for eval.
Lab: CBC wnl, CXry and EKG wnl, BUN 18 mg/dl,
Pcreat 1.0 mg/dl, U/A 4+ prot 3-5 rbc, 0 wbc 0 casts
24 hr UV prot. 1.8 g/d, Palb 3.9 g/dl, Pcholest 204 24 hr UV prot. 1.8 g/d, Palb 3.9 g/dl, Pcholest 204
mg/dl
ANA, CH50, HBSAg-Ab, HCV neg.
CASE 3
1) Should pt receive an ACEinhib.-ARB to
reduce proteinuria? reduce proteinuria?
2) Should Bx be done now or after ACEi-ARB?
3) If proteinuria decreases to < 1g/ day w ACEi
ARB should Bx be done?
4) Should other lab.-serologic tests be done
i t B ? Whi h ? prior to Bx? Which ones?
5) What are likely dxs for asympt. proteinuria
of 1.8 g/day in 48 yo WF
Copyright Harvard Medical School, 2010. All Rights Reserved.
502
Copyright Harvard Medical School, 2010. All Rights Reserved.
503
Copyright Harvard Medical School, 2010. All Rights Reserved.
504
Boston University Eight-Year Experience
Amyloid SCT
701 patients evaluated
309 underwent HDM/SCT
Median Survival 4.6 years
L. Dember. Bone Marrow Transplantation for the Treatment of Primary Amyloidosis.
Clinical Nephrology Conference: New Therapies for Old Diseases. ASN 2006
Overall Survival According to
Randomization Arm (intent to treat)
80
100
%
)
OS = 56 months
M D
0
20
40
60
80
O
v
e
r
a
l
l
S
u
r
v
i
v
a
l
(
%
OS = 56 months
OS = 22 months
M -Dex
ASCT
20
deaths
31
deaths
p <0.05
Median survival of entire cohort : 48.5 months
Median follow-up for living patients : 3 years
0
0 10 20 30 40 50 60 70 80
Months
P Ronco. Free Communications: Non-Traditional Variables and Outcomes, ASN 2006. SA-FC141
Copyright Harvard Medical School, 2010. All Rights Reserved.
505
Conclusions
This first randomized trial shows a survival
advantage of M-Dex-based oral regimen over
HDM ith ASCT i th t t t f t i AL HDM with ASCT in the treatment of systemic AL
amyloidosis
HDM with ASCT is associated with shorter
survival in patients with kidney involvement
These results confirm the efficacy and tolerance
of M-Dex in Al amyloidosis
Th l d t d M D i fi t li d They lead to recommend M-Dex in first line and
to propose ASCT only for refractory patients
P Ronco et al. ASN 2006
Case 4
A 23 yo WF student presents with ARF.
Past hx neg, No meds, Neg Fam Hx.
6 mo PTA pain in right eye and blurry
vision. Opth Dx uveitis. Steroid drops with
resolution.
Vague joint aches, no arthritis, no rash ,
no Fever/chills no visual sx took several no Fever/chills, no visual sx, took several
doses of motrin. No other meds.
Check up by LMD BP 122/74 mHg Afeb,
nl HENT, Cor, Chest, Abd, no edema.
Copyright Harvard Medical School, 2010. All Rights Reserved.
506
Case 4
U/A tr prot 2+heme 4-10 rbc 6-10 wbc U/A tr prot 2+heme 4-10 rbc 6-10 wbc
BUN 38 Pcreat 2.3 mg/dl
Urine culture no Growth
USG large echogenic kidneys
WBC 6.8K 3% eos , Hct 34%, plts 325K
Palb 3.6 g/dl, ESR 43, ANA neg, CH50 nl,
HBV neg, HCV neg, HIV neg, Chest Xray-.
BX peformed
Copyright Harvard Medical School, 2010. All Rights Reserved.
507
TINU Syndrome
Over 150 cases
Pathogenesis unclear T cell proliferation cell
mediated immunity defect mediated immunity defect.
Most young Females, median age 15, 3:1 F:M,
no genetic or familial tendency,
Uveitis ( typical bilateral ) + Interstitial Nephritis ( up
to months before or months after uveitis )
+/- fever, wt loss, fatigue, malaise, anorexia,
arthralgias, myalgias, H/A.
Renal findings: sterile pyuria flank pain hematuria Renal findings: sterile pyuria, flank pain, hematuria,
renal insufficiency, and ARF.
Lab: nonspecific, some eosinophilia, high ESR,
Therapy: spontaneous resolution, steroids.
Copyright Harvard Medical School, 2010. All Rights Reserved.
508
Sarcoid and the Kidney
Hypercalcemia yp
Hypercalciuria
Nephrolithiasis
Interstial Infiltrates with granulomas
Glomerular Lesions ( may be coincidental) Glomerular Lesions ( may be coincidental)
Sarcoid and the Kidney
Renal histologic prevalence very variable g p y
post mortem 20% , Bx series 35-50%.
Clinical disease clearly less than histologic
disease
Hypercalcemia common, hypercalciuira
2.5-20%some stones, nephrocalcinosis 2.5 20% some stones, nephrocalcinosis
and renal insuffic/
Copyright Harvard Medical School, 2010. All Rights Reserved.
509
Sjogrens Syndrome and the Kidney
Sjogrens assoc w a lymphocytic and plasmacytic
infiltrate in salivary, parotid, and lacrimal glands.
Same proces in kidney causes interstitial nephritis and p y p
tubular defects.
Renal involvement very variable ( 2-67%)
Acute interstitial infiltrates, tubular damage and with
chronicity interstitial fibrosis. Glomerular lesions are rare.
Benign U/A, mild proteinuria, occas Fanconis-Type I
distal RTA, Nephrogenic DI.
Usually responds to corticosteroids.
Pertovaara M. et al Clin Neph 56:10, 2001
Bessini N. et al NDT 16:2328, 2001
Copyright Harvard Medical School, 2010. All Rights Reserved.
510
Copyright Harvard Medical School, 2010. All Rights Reserved.
511
Case 5
A 20 y.o. BM college student presents with a
3-4 week hx of progressive ankle and leg
edema and an 18 # wt gain.
On Px BP 120/74 mm HG. Weight 274# Ht
56 Cor chest abd WNL ext 2 3+ pedal and 5 6 Cor-chest-abd WNL, ext 2-3+ pedal and
pretibial edema.
Copyright Harvard Medical School, 2010. All Rights Reserved.
512
Case 5
Laboratory evaluation:
CBC normal
BUN 18 mg/dl, Pcreatinine 1.0 mg/dl,
Palbumin 2.1 g/dl, Cholesterol 284 mg/dl
U/A 4+ protein, 0 rbc, 0-2 wbc, 0 casts
24 hour proteinuria 16.2 g/Day
ANA complement Hep B and C serology HIV ANA, complement, Hep B and C serology, HIV,
all negative or WNL.
USG large echogenic kidneys.
Case 5
What is the Bx most likely to show
(MCD, FSGS, MN, IgA Nephropathy ?)
What Dx if patient were Caucasian ?
What Dx if Asian Descent?
What if patient weighed 154 #?
Copyright Harvard Medical School, 2010. All Rights Reserved.
513
Obesity-related Glomerulopathy: An Emerging
Epidemic
Review >6800 Bxs 1986-2000
ORG 0.2% (1986-1990) 2.0% (1996-2000)
ORG (71) FSGS (50)
Age /Race 43 yo /75% W 32 yo/52% W p<.01
NS 5.6% 54% p<.01
Salb 3.9 g/dl 2.9 g/dl p<.001
FSGS 18% 39%
Glomerulomeg 100% 10% Glomerulomeg 100% 10%
FPE 40% 75%
Double Pcreat 14% 50%
ESRD 3.6% 42%
Kambham, Markowitz, Valeri et al. Kidney Int 59:1498-1509. 2001.
Copyright Harvard Medical School, 2010. All Rights Reserved.
514
Kambham N, Markowitz G, Valeri A, et al. Kidney Int 59:1498-1509, 2001
Copyright Harvard Medical School, 2010. All Rights Reserved.
515
26
25
Copyright Harvard Medical School, 2010. All Rights Reserved.
516
Focal cystic dilation of tubules and prominent
tubulointerstitial inflammation are frequent in
collapsing FSGS
Course Patient 5
Could not use cyclosporine away at
h l school
Prednisone 120 QOD x 2 months then
taper over 6 months
Returned in complete remission
Follow x 4 yrs U/A 0 tr protein normal Follow x 4 yrs U/A 0-tr protein, normal
BUN/creatinine
Copyright Harvard Medical School, 2010. All Rights Reserved.
517
Copyright Harvard Medical School, 2010. All Rights Reserved.
518
Injured podocytes may take
different pathways
Podocyte injury
FSGS CG
Proliferation
Cell hypertrophy
Cell death/Detachment
De-differentiation
Collapse
Sclerosis
Collapsing FSGS
LaureniviciusRennke Kidney Int 1999
Collapsing FSGS
LaureniviciusRennke Kidney Int 1999
Copyright Harvard Medical School, 2010. All Rights Reserved.
519
Collapsing Glomerulopathy:
Clinicopathological Findings
74 cases in 10 years period (1995-2005)
mean age 42 years 41 M(56%) 33 F (44%) mean age 42 years. 41 M (56%) 33 F (44%).
18/74 AA (24.4%)
Clinical associations: 39 idiopathic
10 viral infection
8 autoimmune disease
9 post-transplant
4 pamidronate
4 b 4 obese
Conclusion: diverse group of disorders are associated with collapsing
glomerulopathy, most have in common an altered immune status.
ASN 2005 Sandra M. Soares, Sanjeev Sethi - Mayo Clinic, Rochester, MN.
Copyright Harvard Medical School, 2010. All Rights Reserved.
520
Renal Survival Curve: Collapsing vs Control FSGS
(ValeriDAgati, Kidney Int 50: 1734, 1996)
Renal Survival Curve: Collapsing vs Control FSGS
(ValeriDAgati, Kidney Int 50: 1734, 1996)
100
90
80
e
n
a
l

S
u
r
v
i
v
a
l

(
%
)
Control FSGS
P < 0.001
Collapsing FSGS
80
70
60
50
40
30
Time (months)
R
e
p g
20
10
0
0 12 24 36 48 60 72
Glomerular Disease Collaborative Netwok
Outcomes of FSGS Variants:
UNC Chapel Hill
20
30
40
50
60
70
80
P
e
r
c
e
n
t
Remission
ESRD at 3 years
0
10
A
l
l
F
S
G
S
C
o
l
la
p
s
i
n
g
T
i
p

L
e
s
i
o
n
P
e
r
i
h
i
l
a
r
N
O
S
Kidney International (2006) 69, 920-926
Copyright Harvard Medical School, 2010. All Rights Reserved.
521
Outcomes of FSGS Variants:
Columbia University
80
REMI SSI ON ESRD
20
30
40
50
60
70
80
0
10
ALL FSGS CELL COLL TI P
LESI ON
NOS
Kidney International (2006) 70, 17831792.
Financial Disclosures
Dr. Appel has research grants, consultanships
and served on speakers bureaus, adjudication
committees and scientific advisory boards y
of the following companies: Merck, Pfizer,
Bristol-Myers Squibb, Takeda, Roche, Aspreva,
Genentech, Amgen, OrthoBiotech and QuestCor.
Copyright Harvard Medical School, 2010. All Rights Reserved.
522
David J. Salant
Boston University Medical Center
Approach to the Diagnosis
and Management of Rapidly
Progressive
Glomerulonephritis
Disclosures:
Consulting:
Questcor
Taligen
Grant support:
Questcor
Patent pending:
Diagnostics for Membranous Nephropathy
Copyright Harvard Medical School, 2010. All Rights Reserved.
523
Rapidly Progressive
Glomerulonephritis
(RPGN)
Rapidly progressi ve renal failure
Hematuria rbc casts; RBC dysmorphi a
Oliguria - variable
Hypertensi on - unusual
Proteinuria - variable
Immunofluorescence in RPGN
l i near IgG on GBM pauci-i mmune
granul ar IgG on capi l l ary wal l
granul ar IgA i n mesangi um
Copyright Harvard Medical School, 2010. All Rights Reserved.
524
Renal Biopsy
Necrotizing and/or Crescentic GN
Linear GBM deposits
11%
(anti-GBM antibody disease)
Granular immune deposits
8%
(immune complex disease)
No immune deposits
81%
(ANCA-associated disease)
With pulmonary hemorrhage
Goodpasture syndrome
Renal limited
Anti-GBM GN
Systemic symptoms
SLE
HSP
Postinfectious
Cryoglobulinemia
Renal limited
IgA nephropathy
MPGN
Systemic symptoms
Wegener granulomatosis
Microscopic PAN
Churg-Strauss syndrome
Renal limited
Pauci-immune GN
Percentages from Jennette, 1993; n = 65
A 25 year old man presents to the ER with cough and
minor hemoptysis, microscopic hematuria and Screat - 1.3 mg/dL. A
viral syndrome is diagnosed and he is sent home.
Two weeks later he represents with more severe hemoptysis, oliguria
and Screat - 6.8 mg/dL.
Treatment: Pulse methylprednisolone; ventilation; cyclophosphamide;
plasmapheresis
Chest Xray:
Renal biopsy:
Case 1
Copyright Harvard Medical School, 2010. All Rights Reserved.
525
IgG IgG
Copyright Harvard Medical School, 2010. All Rights Reserved.
526
Which of the following serological tests is most
likely to be positive?
A. Anti-proteinase 3
B. Low C3 and C4
C. Anti-dsDNA
D. Anti-GBM
What is the target antigen?
The NC1 domain of the 3 (and
5) chains of type IV collagen
Pedchenko et al. NEJM July 22, 2010
Salant NEJM July, 22, 2010
Copyright Harvard Medical School, 2010. All Rights Reserved.
527


GBM
Mesangium
NC1
Goodpasture epi tope
Chromosome 13
Chromosome 2
Chromosome X
Location of the Goodpasture Antigen
Type IV col l agen
His pulmonary symptoms resolve within 72 hours but he
remains oliguric and dialysis dependent after 10 days.
Which of the following treatment options should you
pursue?
A. Discontinue plasmapheresis and taper off
immunosuppressives
B. Continue plasmapheresis and immunosuppressives
C. Add IVIG
D. Add rituximab
Copyright Harvard Medical School, 2010. All Rights Reserved.
528
Anti-GBM Antibody Disease
Risk factors for pulmonary hemorrhage
(Goodpasture syndrome):
Young males
Smoking, volatile solvents, viral respiratory infection
Recurrence of Anti-GBM Nephritis after
Renal Transplantation
Nephritis is likely to recur if there is circulating antibody at
the time of transplantation
Delay transplantation until anti-GBM is undetectable by
ELISA for anti-GBM
A weakly positive western blot is not a contraindication to
transplantation if the ELISA is negative.
Copyright Harvard Medical School, 2010. All Rights Reserved.
529
Case 2
A 35-year old man presents with crampy
abdominal pain, painful joints, swollen ankles and
a rash on his legs.
BP is 150/90, the abdomen is diffusely tender and
stool occult blood test is positive.
Urinalysis - 3+blood, 3+albumin, numerous
dysmorphic RBCs and occasional RBC casts
Se Cr 2.2 mg/dL, Urine prot/Cr 3.3, C3 and C4
normal, serology pending
Copyright Harvard Medical School, 2010. All Rights Reserved.
530
Which of the following is the
most likely diagnosis?
A. Polyarteritis nodosa
B. Microscopic polyangiitis
C. Mixed cryoglobulinemia
D. Henoch Schonlein purpura
IgA IgA
Copyright Harvard Medical School, 2010. All Rights Reserved.
531
Case 3:
A 25-year old woman presents with malaise and pleuriticchest pain of
one weeks duration. Her BP is 145/100 and she has 2+bilateral leg
edema. There is no rash or arthritis. Urinalysis shows 3+protein and 3+
blood and there are numerous red cells, 5-10 leukocytes and occasional
red cell casts on microscopy.
Serum creatinineis newly elevated at 2.3 mg/dL and serum albumin is
3.3 g/dL. Hb 11.5, wbc 3,300, platelets 120,000. Serological tests for
ANA, ANCA and anti-GBM are pending but serum complement levels
return with C3 60 mg/dL and C4 10 mg/dL (both low).
Urine protein:Cr 4.5
A renal biopsy is obtained.
Case 3:
A 25-year old woman presents with malaise and pleuriticchest pain of
one weeks duration. Her BP is 145/100 and she has 2+bilateral leg
edema. There is no rash or arthritis. Urinalysis shows 3+protein and 3+
blood and there are numerous red cells, 5-10 leukocytes and occasional
red cell casts on microscopy.
Serum creatinineis newly elevated at 2.3 mg/dL
Serum albumin is 3.3 g/dL
Hb11.5, wbc 3,300, platelets 120,000
Serological tests for ANA, ANCA and anti-GBM are pending
Serum complement - C3 60 mg/dL and C4 10 mg/dL (both low)
Urine protein:Cr 4.5
A renal biopsy is obtained.
Copyright Harvard Medical School, 2010. All Rights Reserved.
532
Which of the following immunofluorescent patterns would
you expect in this case?
IgG
IgA
IgG
IgG
A
B
C
D
C3 C4
Poststrep GN low normal
Other postinfectious (e.g. endocarditis) low low
SLE nephritis III/IV low low
MPGN 1 (Hepatitis C-associated) low v. low
Dense deposit disease low normal
Mixed cryoglobulinemia low v. low
IgAN/HSP normal normal
Anti-GBM normal normal
ANCA-associated (pauci-immune) GN normal normal
Serum complement levels in RPGN
Copyright Harvard Medical School, 2010. All Rights Reserved.
533
Disease Pulmonary
disease
Cutaneous
vasculitis
Goodpasture syndrome present absent
Wegeners granulomatosis present present
Microscopic polyangiitis present present
Churg-Strauss allergic
granulomatosis
present present
Mixed cryoglobulinemia rare present
Henoch-Schnlein purpura rare present
Systemic lupus
erythematosus
rare unusual
Case 4
A 56-year old man presented with 6-week history
cough productive of small amounts of bloody
sputum.
Apart from fatigue he had no other complaints.
Physical exam - BP 125/80; afebrile; O
2
sat 93%
RA; no edema, rash or joint abnormalities
Urinalysis - 1+albumin, microscopic hematuria
with RBC casts
Serum Cr 2.5 mg/dL; Hb 11.1 g/dL
Copyright Harvard Medical School, 2010. All Rights Reserved.
534
Copyright Harvard Medical School, 2010. All Rights Reserved.
535
IgG
Which of the following serological
tests is most likely to be positive in
this case?
A. Anti-myeloperoxidase
B. Anti-GBM
C. RPR
D. Anti-proteinase 3
Copyright Harvard Medical School, 2010. All Rights Reserved.
536
pANCA Control
ANCA
Desi gnation: c-ANCA p-ANCA
Staining
pattern:
diffuse
cytoplasmic

perinuclear
Antigen: proteinase 3
(29 kD PMN
serine
proteinase)

myeloperoxidase;
other
Disease
association
WG > MPA MPA > WG

Copyright Harvard Medical School, 2010. All Rights Reserved.
537
Clinical presentation of
patient
Prevalence of
PI-CGN
PPV for
PI-CGN
NPV for
PI-CGN
Rapidly progressive
glomerulonephritis
47%
(106/224)
98% 80%
Hematuria, proteinuria,
and creatinine 1.5-3 mg/dl
21%
(181/862)
92% 93%

Hematuria, proteinuria,
and creatinine <1.5 mg/dl
7%
(51/685)
77% 98%


Estimated positive and negative predictive values of
ANCA tests for pauci-immune crescentic GN (PI- CGN)
Patients: adults (>18 years old) with different clinical presentations
Assay has a 72.5% sensitivity and a 98.4% specificity
From: Jennette JC, Wilkman AS, Falk RJ. Kidney Int 1998;53:796-798
Does a rise in ANCA titer predict a
relapse of vasculitis or GN?
Patients with WG who had a rise in ANCA titer
experienced fewer relapses if they were treated
expectantly - Cohen Tervaert et al. Lancet 1990.
Meta-analysis showed only 48% specificity and 52%
sensitivity - Cohen Tervaert et al. 1995.
Relapse occurs more frequently in the presence of a
positive ANCA (80% vs 20%) but a rise in titer is a poor
predictor of relapse - Kyndt et al. Am J Med 1999.
PPV: cANCA 28%; anti-PR3 12%; anti-MPO 43%
Copyright Harvard Medical School, 2010. All Rights Reserved.
538
Drug-Associated ANCA-Positive
Necrotizing Vasculitis
Hydralazine
Propylthiouracil
Minocycline - HLA-DR4 or -DR2 linked
Penicillamine
Cimetidine
Allopurinol
Silicon ??
Case 5
A 43-year old man with end-stage kidney failure from pANCA-
associated pauci-immune crescentic glomerulonephritis is being
considered for renal transplantation. He presented with RPGN and
acute renal failure 8 years ago and responded well to
immunosuppressive therapy, however over the past 18 months his renal
function has declined slowly but progressively. He has no symptoms or
signs of systemic vasculitis and has been off all immunosuppressive
agents for 6 years. His urinalysis is benign except for 2+proteinuria.
Repeat renal biopsy six months ago showed extensive
glomerulosclerosis with fibrous crescents in 20% of glomeruli, tubular
atrophy and interstitial fibrosis. ELISA for myeloperoxidaseis positive
15 units (normal <2.8).
Copyright Harvard Medical School, 2010. All Rights Reserved.
539
Which of the following statements is correct regarding this
patients candidacy for renal transplantation?
A. Renal transplantation is contraindicated because of
the high rate of recurrent disease.
B. Transplantation should be delayed until ANCA
becomes undetectable by ELISA.
C. Renal transplantation can proceed using standard
immunosuppression.
D. Renal transplantation can proceed but the
immunosuppressive regimen should include
cyclophosphamide.
Relapse rate: ~10-20% - not all renal
Relapse is not correlated with ANCA positivity at the time
of transplant, duration on dialysis, nature of the vasculitis
(WG or MPA), or type of ANCA
Most respond to cyclophosphamide
5 year overall and death-censored graft survivals were 94
and 100%, respectively (Gera et al)
No clear risk factors identified for relapse
Recurrence of ANCA-associated Renal Vasculitis
and Pauci-Immune GN after Renal Transplantation
Allan et al: J ASN 1998 - 22 cases
Nachmanet al: Kidney Int 1999 - 127 cases
Schmitt et al: Lancet 1993 - 20 cases of WG
Gera et al: Kidney Int 2007 - 35 cases (20 MPA, 15 WG)
Copyright Harvard Medical School, 2010. All Rights Reserved.
540
Conclusion: Renal transplantation is a safe and effective
treatment for end-stage ANCA-associated vasculitis with a
low recurrence rate
Case 6
A 63-year old woman presents with RPGN
and is found to have renal limited
vasculitis with pauci-immune crescentic
GN on biopsy, positive pANCA and anti-
MPO.
Serum creatinine falls from 3.8 mg/dL and
stabilizes at 1.5 mg/dL after pulse
methylprednisoloneand initiation of oral
prednisone and cyclophosphamide.
Copyright Harvard Medical School, 2010. All Rights Reserved.
541
Which of the following maintenance
therapies should you prescribe?
A. Taper prednisone and continue
cyclophosphamide until the patient has been in
clinical remission for 12 months
B. Taper prednisone and switch
cyclophosphamide to azathioprine after 6
months if the patient is in clinical remission
C. Taper prednisone and switch
cyclophosphamide to TMP/sulfa after 6 months
if the patient is in clinical remission
A Randomized Trial of Maintenance Therapy
for Vasculitis Associ ated with Antineutrophil
Cytoplasmic Autoanti bodies
Jayne et al for the European Vasculitis Study Group
N Engl J Med 349:36-44, 2003
About 30% of patients with ANCA-associated
vasculitis that enter remission will relapse!
Copyright Harvard Medical School, 2010. All Rights Reserved.
542
0 6 9 12 15 18
Months after randomization
1.0
0.9
0.8
0.7
0.6
Induction therapy
Cyclophosphamide + prednisone
3 mnths
or
3-6 mnths
Randomization
Remission
Cyclophosphamide
N = 79
Azathioprine
N = 76
12 mnths
Azathioprine
End follow up 18 mnths after entry Jayne et al N Engl J Med 349:36, 2003
Comments
1. Open label adequately powered prospective
randomized controlled study
2. The relapse rate with maintenance azathioprine
was the same as with cyclophosphamide
3. Serious adverse events over 18 months were
not substantially different in the two groups
Copyright Harvard Medical School, 2010. All Rights Reserved.
543
Case 7
A previously healthy 35-year old man presents
with malaise and is found to be oliguric and
serum creatinine is 6.8 mg/dl (1.0 on routine
exam 6 months ago).
Urinalysis shows microscopic hematuria with
mostly dysmorhic RBCs.
pANCA is strongly positive.
Renal biopsy shows cellular crescents in 75% of
glomeruli with periglomerular inflammation and
moderate interstitial fibrosis.
He remains oliguric despite prednisone
started by the referring nephrologist. In
addition to supportive care with dialysis,
which of the following is the most
appropriate treatment for this patient?
A. No further treatment is indicated
B. Pulse methylprednisolonefollowed by
oral prednisone and cyclophosphamide
C. Plasma exchange and oral prednisone
and cyclophosphamide
Copyright Harvard Medical School, 2010. All Rights Reserved.
544
Randomized Trial of Plasma Exchange or High-
Dosage Methylprednisolone as Adjunctive Therapy
for Severe Renal Vasculitis
David R.W. J ayne*, Gill Gaskin, Niels Rasmussen, Daniel Abramowicz, Franco Ferrario||, Loic
Guillevin, Eduardo Mirapeix**, Caroline O.S. Savage, Renato A. Sinico||, Coen A. Stegeman, Kerstin
W. Westman, Fokko J . van der Woude||||, Robert A.F. de Lind van Wijngaarden, Charles D. Pusey on
behalf of the European Vasculitis Study Group
J Am Soc Nephrol 18: 2180-2188, 2007
MEPEX n=151 (Courtesy David Jayne)
Entry
WG/MPA/RLV
Creatinine >500/L (5.8mg/dL)
Renal Biopsy cellular crescents
Methyl Predni sol one
IV 1g/day x 3
Pl asma exchange
60ml/kg x 7 over 2 weeks
Pri mary end poi nt
Renal recovery
at 3 months:
Patient survival
Dialysis independence
Creatinine <500/L
CYC p.o. 6/12, AZA
Predp.o. taper
Copyright Harvard Medical School, 2010. All Rights Reserved.
545
Modified from Jayne, D. R.W. et al. J Am Soc Nephrol 2007;18:2180-2188
51 evaluated for
Secondary end-points
29 dialysis independent
22 ESRD
56 evaluated for primary
end-point
33 dialysis independent
23 ESRD
59 evaluated for primary
end-point
48 dialysis independent
11 ESRD
151
Screened
137
randomized
14 excluded
67
IV methylprednisolone
70
Plasma Exchange
11 deaths
23 ESRD
8 deaths
2 ESRD
11 deaths
11 ESRD
5 deaths
2 ESRD
51 evaluated for
Secondary end-points
41 dialysis independent
10 ESRD
Entry
3 months
12 months
Enrollment, Patient Survival and Renal Outcome
Comments
1. Open label adequately powered prospective randomized
controlled study.
2. Plasma exchange significantly improved the chances of
renal recovery at 3 and 12 months.
3. Serious adverse events were common and the mortality
rate was high (23% in MP group and 27% in PE group).
4. The results apply only to patients with severe, active
renal disease without life-threatening extra-renal
manifestations or chronic kidney damage.
Copyright Harvard Medical School, 2010. All Rights Reserved.
546
NOVEL IMMUNOTHERAPIES
TNF inhibition therapies:
Encouraging small studies with Infliximab(humanized anti-TNF)
suggested efficacy.
Soluble TNF receptor:Fc fusion protein - Etanercept did not reduce
the rate of relapses in a prospective randomized study (NEJ M
2005).
Anti-CD20 (Rituximab):
RAVE was a phase II/III double-blind, placebo-controlled trial of
rituximabin WG, MPA and ANCA-associated GN.
Rituximabwas as effective as cyclophosphamide (both in
combination with glucocorticoids) in inducing remission.
The adverse event rates were similar.
(NEJ M J uly 15, 2010)
ANCA-associated GN - Summary
ANCA is an excellent marker of pauci-immune crescentic GN.
c-ANCA tends to occur more often in patients with features of
classical Wegeners granulomatosis and p-ANCA in patients with
renal limited disease but there is much overlap.
MPO (p-ANCA)- and PR3 (c-ANCA)-associated GN and vasculitis
have a similar prognosis and response to treatment.
The titer of ANCA does not correlate well with disease activity but
disappearance of ANCA is associated with remission.
The value of ANCA in predicting relapse is controversial.
Azathioprineis effective maintenance therapy.
Plasmapheresis is indicated for AAGN vasculitis with ARF.
Rituximabmay be an alternative to cyclophosphamide for induction
of remission.
Copyright Harvard Medical School, 2010. All Rights Reserved.
547
Disclosures:
Consulting:
Questcor Questcor
Taligen
Grant support:
Questcor
Patent pending:
Diagnostics for Membranous Nephropathy
Copyright Harvard Medical School, 2010. All Rights Reserved.
548
Genetics of Renal Disease Genetics of Renal Disease
Martin Pollak, M.D.
NephrologyDivision, BIDMC Nephrology Division, BIDMC
Associate Professor of Medicine, HMS
Financial Disclosures
Dr. Pollakhas no conflicts of interest to disclose.
Copyright Harvard Medical School, 2010. All Rights Reserved.
549
Topics
Common genetic variations and kidney disease
GWAS studies, admixture
Quantitatively important hereditary disorders
Cystic disease
Alport syndrome/Hereditary nephritis
Novel pathogenetic mechanisms
FSGS, nephrotic syndrome, proteinuria p y p
Novel molecular treatment
Fabry disease
Tubular disorders
Topics
Common genetic variations and kidney disease
GWAS studies, admixture ,
Quantitatively important hereditary disorders
Cystic disease
Alport syndrome/Hereditary nephritis
Novel pathogenetic mechanisms
FSGS, nephroticsyndrome, proteinuria FSGS, nephrotic syndrome, proteinuria
Novel molecular treatment
Fabry disease
Tubular disorders
Copyright Harvard Medical School, 2010. All Rights Reserved.
550
Copyright Harvard Medical School, 2010. All Rights Reserved.
551
a
r
i
a
n
t

a
l
l
e
l
e

c
o
n
f
e
r
r
e
d

b
y

v
a
Diseasefrequency
R
i
s
k

c
Copyright Harvard Medical School, 2010. All Rights Reserved.
552
rs12917707: The rare allele (T vs G)
confers 80% of normal CKD risk
(from OMIM):
FAMILIAL J UVENILE HYPERURICEMIC NEPHROPATHY ; FJHN
Phenotypes caused by UMOD mutations
UMOD: encodes uromodulin, aka Tamm-Horsfall protein
Alternative titles: HYPERURICEMIC NEPHROPATHY, FAMILIAL JUVENILE ; HNFJ
NEPHROPATHY, FAMILIAL, WITH GOUT
GOUTY NEPHROPATHY, FAMILIAL JUVENILE
AD inheritance. Low FeUrea.
GLOMERULOCYSTIC KIDNEY DISEASE WITH HYPERURICEMIA AND
ISOSTHENURIA
cyst dilatation and collapse of glomeruli with hyperuricemia and isosthenuria, Electron y p g yp
microscopy demonstrated accumulation of dense fibrillar material within the endoplasmic
reticulum. Patient urine samples consistently showed a severe reduction of excreted
uromodulin.
MEDULLARY CYSTIC KIDNEY DISEASE 2; MCKD2
Alternative titles: CYSTIC KIDNEY DISEASE 2, AUTOSOMAL DOMINANT; ADMCKD2
Copyright Harvard Medical School, 2010. All Rights Reserved.
553
Familial interstitial disease
11 11
Copyright Harvard Medical School, 2010. All Rights Reserved.
554
Admixture mapping
From Kopp, 2008
Copyright Harvard Medical School, 2010. All Rights Reserved.
555
FSGS association
Results of association of 180
African American FSGS cases and
205 African American controls were
genotyped. (A) Fisher's exact test,
(B) Logistic regression controlling
for rs73885319, (C) Logistic
regression controlling for
rs73885319 and rs71785313.
Copyright Harvard Medical School, 2010. All Rights Reserved.
556
APOL1 background
ApoL1 travels in the blood
in HDL3 complexes. p
Hungry Trypanosomes eat
HDL3 and the complex
goes in the Trypanosome
stomach (the lysosome).
ApoL1 causesthe Trypanosome to lyse. SRA (serum response
associated protein) in T.b.rhodiense inactivates ApoL1
Haplotype homozygosity in Yoruba
The 230 genotyped
chromosomes from the 167
Yoruba samples, a mix of
trios, pairs, and unrelated p
samples, were divided in three
groups according to the
presence of the core alleles
G1 (n=88), G2 (n=17), or the
absence of either (n=125).
Haplotypes were colored in
such a way that chromosomes
with the same color at a given
iti h th position have the same
haplotype from that position
up to the position of the C-
terminal part of APOL1, and
their colors are different when
this is not the case.
Copyright Harvard Medical School, 2010. All Rights Reserved.
557
19
Summary: ApoL1
CodingvariantsinAPOL1geneexplainthe Coding variants in APOL1 gene explain the
increased risk of kidney disease in African
Americans under a recessive inheritance
model
These variants make the encoded protein
able to kill Trypanosome b. rhodiense
These variants are recent (~3000 years old)
and are an example of balancing selection
Copyright Harvard Medical School, 2010. All Rights Reserved.
558
Topics
Common genetic variations and kidney disease
GWAS studies, admixture
Quantitatively important hereditary disorders
Cystic disease
Alport syndrome/Hereditary nephritis
Novel pathogenetic mechanisms
FSGS, nephrotic syndrome, proteinuria , p y , p
Novel molecular treatment
Fabry disease
Tubular disorders
Autosomal Dominant Polycystic Kidney
Disease (ADPKD)
PKD1, chromosome 16, 85% of cases
Polycystin-1 protein (PC1), 4302 amino
acids
PKD2, chromosome 4, 15% of cases
Polycystin-2 protein (PC-2), 968 amino
acids
Cystogenesis requires a second hit,
either in the remaining normal allele of the
germline inherited gene, or in the other
PKD gene (trans-heterozygous mechanism)
Copyright Harvard Medical School, 2010. All Rights Reserved.
559
ADPKD: Clinical features
Renal cysts Renal cysts
Liver cysts
Intracranial aneuryms
Cardiac valve abnormalities
Abdominal inguinal hernias Abdominal, inguinal hernias
Kidney stones
Domain Structure of Human Cystogenic
Proteins
Fibrocystin
ARPKD gene
Igarashi and Somlo, JASN, 2002
Copyright Harvard Medical School, 2010. All Rights Reserved.
560
Renal Disease in ADPKD the Role of
Genotype
Medianageof ESRDis54inPKD1 74inPKD2 Median age of ESRD is 54 in PKD1, 74 in PKD2
Less hypertension, UTIs, and hematuria in PKD2-
associated disease
Hateboer et al, Lancet, 1999
Genetic Testing in PKD
Mutational analysisversuslinkage Mutational analysis versus linkage
Clinical utility
Kidney transplant donor evaluation (does
potential living related donor have
presymptomatic PKD?)
Earlier treatment????
ACE-I, antioxidants, K+supplementation,
Copyright Harvard Medical School, 2010. All Rights Reserved.
561
Genotyping: PKD1
Genotype family members at flanking polymorphic markers and
intragenic markers (typically several)
1 2 3
Define haplotypes in family (groups of marker alleles inherited
together)
1 3 2
2 4 3
Example: Linkage analysis
Near ESRD
?
Family with ADPKD: 22 y.o. wants to donate kidney
to dad approaching ESRD. No cysts on CT scan.
Copyright Harvard Medical School, 2010. All Rights Reserved.
562
Eaxample 1: Linkage analysis
12 36
45 23 13 26 23 24
? 24
43 25
Step 1: Is disease in this family linked to PKD1 locus?
This example: Yes - Lod score ~ 2.0, or 1 in 100 odds of seeing
this cosegregation pattern by chance (disease is segregating
with the 2 haplotype).
Linkage analysis
23
24 24
Variation: Only 1 affected available
Copyright Harvard Medical School, 2010. All Rights Reserved.
563
Direct mutational analysis:
Screen for sequence variants by PCR amplification of PKD1, PKD2
exons from patients genomic DNA
Problems: Sensitivity? Decision as to whether nucleotide change is
disease causing?
Copyright Harvard Medical School, 2010. All Rights Reserved.
564
Other renal cystic diseases
ARPKD
Von Hippel Lindau
Tuberous Sclerosis
MCKD/nephronophthisis complex p p p
Multiple simple cysts
Nephronophthisis (NPHP)
Nephronophthisis (NPHP) is an autosomal recessive kidney disease leading to end-
stage renal failure
Key histological findings: tubulointerstitial fibrosis, tubular dilatation and cyst
formation and tubular atrophy.
NPHP is often a feature of a multisystem disease that may include retinal dystrophy
(SeniorLoken Syndrome) and cerebello-ocular-renal syndromes (Joubert syndrome
and related diseases (JSRD)).
NPHP may present with an early decrease in urinary concentration
End-stage renal failure (ESRF) typically occurs during early teenage years, with the
exception of the rare infantile forms, where there is ESRF before 5 years of age.
Molecular genetics allows detection of the most common mutations (involving NPHP1 o ecu a ge et cs a o s detect o o t e ost co o utat o s ( o g
and accounting for 25% of all cases).
Evidence to date implicates the primary renal cilium and basal body apparatus in the
pathogenesis of NPHP.
Patients need regular monitoring of renal and liver function, eye examinations and
preparation for renal transplantation, which is the treatment of choice for the renal
failure that invariably ensues.
Copyright Harvard Medical School, 2010. All Rights Reserved.
565
EJ HG, Simms et al 2008
EJ HG, Simms et al 2008
Copyright Harvard Medical School, 2010. All Rights Reserved.
566
Nephrocystin proteins and their protein domains. Domain structure of the nephrocystin proteins. Nephrocystin
proteins contain a diverse variety of protein domains and no common pattern can be identified.
EJ HG, Simms et al 2008
Subcellular localization of nephrocystins to primary cilia, basal bodies, the mitotic spindle, focal adhesions and
adherens junctions, and functional interaction with other proteins mutated in renal " ciliopathies
Hildebrandt, F. et al. J Am Soc Nephrol 2009;20:23-35
Copyright Harvard Medical School, 2010. All Rights Reserved.
567
Topics
Common genetic variations and kidney disease
GWAS studies, admixture
Quantitatively important hereditary disorders
Cystic disease
Alport syndrome/Hereditary nephritis
Novel pathogenetic mechanisms
FSGS, nephrotic syndrome, proteinuria , p y , p
Novel molecular treatment
Fabry disease
Tubular disorders
Alport Syndrome and Other Disorders
of Type IV Collagen
Hudson, et al, NEJM, 2003
Copyright Harvard Medical School, 2010. All Rights Reserved.
568
Alport syndrome
Most commonform: X linked Most common form: X-linked
Defects in COL4A5
Renal disease: always
Deafness: often
Ocular disease: sometimes Ocular disease: sometimes
Alport syndrome and related disorders
Hudson, NEJM, 2003
Copyright Harvard Medical School, 2010. All Rights Reserved.
569
Alport Syndrome
Atlas of Renal Pathology, AJKD, 2000
The Thin Basement Membrane Lesion
Atlas of Renal Pathology, AJKD, 2000
Copyright Harvard Medical School, 2010. All Rights Reserved.
570
Alport GBM is More Susceptible to
in vitro Proteolysis
Kalluri et al, J Clin Invest, 1996
Post-Transplant Anti-GBM Disease
Occurs in 3-4% of patients with Alports, both
autosomal and X-linked
the possibility of anti-GBM disease does not
preclude transplantation
X-linked, auto-antibodies 5 chain, but also
against 3 and 4
Clinical anti-GBM disease typically occurs within
thefirst year post transplant the first year post-transplant
Clinical disease may be more common in those
with large deletions
Copyright Harvard Medical School, 2010. All Rights Reserved.
571
Evaluation of hematuria
Consider familial basement membrane syndromes
P f f il hi d/ i i l Presence of family history and/or proteinuria strongly
suggests glomerular rather than lower tract
(urologic) problem
Definitive dx of Alport syndrome: Renal biopsy
(genetic testing is not widely available)
Consider other diseasesaswell: Consider other diseases as well:
Familial IgA nephropathy
Nail patella sydrome
Fabry disease
Topics
Common genetic variations and kidney disease
GWAS studies, admixture
Quantitatively important hereditary disorders
ADPKD
Alport syndrome/Hereditary nephritis
Novel pathogenetic mechanisms
FSGS, nephrotic syndrome, proteinuria , p y , p
Novel molecular treatment
Fabry disease
Tubular disorders
Copyright Harvard Medical School, 2010. All Rights Reserved.
572
38 y.o. man with renal insuffuciency
Molecular anatomy of the podocyte foot process cytoskeleton
MAGI-1
Podocalyxin
NHERF2
Ezrin
FAT
NEPH-1?
V
T
P
SGS Dominant FSGS:
-actinin-4
TRPC6
CD2AP(?)
Recessive FSGS/NS
NPHS1
NPHS2
LAMB2
PLCE1
Copyright Harvard Medical School, 2010. All Rights Reserved.
573
Podocyte molecules in context
From Tryggvason
NPHS2: Podocin
NH2
COOH
Schwarz, JCI
Copyright Harvard Medical School, 2010. All Rights Reserved.
574
Familial FSGS
1/3of pediatricsteroid resistant nephrotic 1/3 of pediatric steroid-resistant nephrotic
syndrome or FSGS is due to mutations in
the podocin gene NPHS2
Recessive: absence of FH does not rule out
familial form of disease
Topics
Common genetic variations and kidney disease
GWAS studies, admixture
Quantitatively important hereditary disorders
ADPKD
Alport syndrome/Hereditary nephritis
Novel pathogenetic mechanisms
FSGS, nephrotic syndrome, proteinuria , p y , p
Novel molecular treatment
Fabry disease
Tubular disorders
Copyright Harvard Medical School, 2010. All Rights Reserved.
575
Fabry Disease
X-linked deficiency of lysosomal -galactosidase
accumulation of neutral sphingolipids, mostly
globotriasylceramide (GL3) g y ( )
Classically, symptoms begin ~10 years of age
Characteristic angiokeratomas, groin, hips, umbilical region
Characteristic corneal opacity, which does not affect vision
Peripheral neuropathy with severe acroparasthesias
Mixed glomerular and tubular renal disease, typically
ESRDby3 4
th
decade ESRD by 3-4
th
decade
Cardiac involvement LVH, conduction abnormalities,
arrhythmias
Vascular disease (endotheliopathy)
Angiokeratomas in Fabry Disease
Desnick et al, Ann Int Med, 2003
Copyright Harvard Medical School, 2010. All Rights Reserved.
576
Corneal Opacity in Fabry Disease
Desnick et al, Ann Int Med, 2003
Lamellated Lysosomal Inclusions of GL3 in
Podocytes of a Patient with Fabry Disease
Atlas of Renal Pathology, AJKD, 2001
Copyright Harvard Medical School, 2010. All Rights Reserved.
577
Diagnosis
Affected males -galactosidase activity in peripheral WBC.
Atypical variants may have residual activity.
Female carriers may have normal enzyme activity DNA y y y
analysis
Female carriers may have significant disease burden, due to
random X-inactivation
Treatment
Enzyme replacement therapy, with Replagal or Fabrazyme
Reduces plasma GL3 to undetectable, in addition to reduced
tissue accumulation
Indications, dosage, endpoints, and cost-effectiveness are as yet
unclear
Reduced Glomerular Endothelial GL3 After
Enzyme Replacement
5 mos. treatment Pre-treatment
Thurberg et al, KI, 2002 GL3 deposits
Copyright Harvard Medical School, 2010. All Rights Reserved.
578
Topics
Common genetic variations and kidney disease
GWAS studies, admixture
Quantitatively important hereditary disorders
ADPKD
Alport syndrome/Hereditary nephritis
Novel pathogenetic mechanisms
FSGS, nephrotic syndrome, proteinuria , p y , p
Novel molecular treatment
Fabry disease
Tubular disorders
Copyright Harvard Medical School, 2010. All Rights Reserved.
579
NEJM, 1999
NEJM, 1999
Copyright Harvard Medical School, 2010. All Rights Reserved.
580
Chronic metabolic
alkalosis/hypokalemia
Diuretic abuse
Bulemia
Hyperaldosteronism
Bartter/Gitelman syndromes
Dx: Urinary chloride
Family history
Molecular genetics
Bartters vs Gitelmans syndromes
BARTTERs GITELMANs
Location of defect Ascending limb of Henle Distal tubule
Age of presentation Prenatal, infancy, early
childhood
Late childhood or
adulthood
Biochemical differences Serum Mg sometimes low
Increased urine Ca
excretion
Serum MG low
Decreased urine Ca
excretion
Molecular etiology Na-K-2Cl cotransporter,
apical K channel (ROMK),
basolateral Cl channel
(CLCNKB)
Henle
Na-Cl cotransporter in
distal tubule
Copyright Harvard Medical School, 2010. All Rights Reserved.
581
Familial Hypertension and the
WNK (With-no-Lysine)
Protein Kinases
Pseudohypoaldosteronism Type II
Also known as Gordons syndrome or the
chloride-shunt disorder
Themirror image of Gitelmanssyndrome: The mirror image of Gitelman s syndrome:
- hypertension
- hyperkalemic acidosis
- suppression of plasma renin, aldosterone
- hypercalciuria
Responsive to thiazides, suggesting defect in p , gg g
the DCT
Autosomal dominant transmission, linked to
chromosomes 1q32-42, 12p13, and 17p11-
q21
Copyright Harvard Medical School, 2010. All Rights Reserved.
582
Financial Disclosures
Dr. Pollakhas no conflicts of interest to disclose.
Copyright Harvard Medical School, 2010. All Rights Reserved.
583
Pediatric Kidney Disease: A Board
Primer
J ulie R. Ingelfinger, M.D.
Professor of Pediatrics, Harvard Medical School
Senior Consultant, Pediatric Nephrology,
MassGeneral Hospital for Children at MGH
Pediatric Kidney Disease: A Board
Primer
J ulie R. Ingelfinger, M.D.
DISCLOSURE: I am a deputy editor at the
New England J ournal of Medicine, a paid
position
Copyright Harvard Medical School, 2010. All Rights Reserved.
584
Pediatric Kidney Disease:
*Issues of growth and development
*Congenital diseases
*Differences in presentation, evaluation,
treatment and implication of diseases
EXAMPLES:
Hematuria
Hypertension
Nephrotic Syndrome
Renal Functional Development: GFR
m
l
/
m
i
n
/
1
.
7
3
m
2

Data from Schwartz and Furth. PediatrNephrol 22:1839, 2007
Copyright Harvard Medical School, 2010. All Rights Reserved.
585
Renal Functional Development: GFR
P
e
r
c
e
n
t

o
f

A
d
u
l
t

F
u
n
c
t
i
o
n
Data from Rhodin et al. Human renal function maturation: a quantitative description
using weight and postmenstrual age. Pediatr Nephrol 2009; 24: 67-76
Renal Functional Development: Uosm
U
r
i
n
e

O
s
m
o
l
a
l
i
t
y
Data from Polaceket al. The osmotic concentrating ability in healthy infants and
children AJDC 1965; 40: 291-295
Copyright Harvard Medical School, 2010. All Rights Reserved.
586
Renal Functional Development
*FENadrops over first weeks of life
*Proximal tubular function continues to mature
*Renal blood flow continues to mature
*Medication handling must be considered
in terms of age
QED: Think of the AGE of the CHILD
Three Examples of Nephrology Issues
Different in Children cf. Adults
*HEMATURIA
*HYPERTENSION
*NEPHROTIC SYNDROME
Copyright Harvard Medical School, 2010. All Rights Reserved.
587
#1-Hematuria in Children
*Isolated hematuriain child: malignancy RARE
*Evaluation differs from that in adults
*Gross hematuriaestimated at 1/1000 visits
Invasive radiographic modalities and
cystoscopy are not always indicated but
sometimes prove necessary and useful
Hematuria in Children
*J immy goes to the pediatrician for a well-
child check. His pediatrician decides to do
a screening urinalysis. J immy is found to
have isolated microhematuria. What should
be done?
Copyright Harvard Medical School, 2010. All Rights Reserved.
588
Hematuria in Children
*Dont miss something serious
*Avoid unnecessary and invasive testing
*Careful history and physical helpful
*Reassure family
Screening Detection
Isolated hematuria: Screening
*7 million school children screened by urinalysis.
*1044 - abnormal urinalysis
*719 (60.1%) - isolated hematuria.
*52 biopsy
-33 thin basement membrane disease
-16 definable pathology, mainly IgAN
Data from: Park YH, Choi JY, Chung HS, et al.
Hematuriaand proteinuriain a mass school urine
screening test. Pediatr Nephrol 2005; 20:11261130.
Copyright Harvard Medical School, 2010. All Rights Reserved.
589
Screening Detection
* School screening of all children in South Korea
*461 children persistently abnormal urinalysis
*289 isolated hematuria
*Renal biopsy- almost half (47.1%) had normal
findings
*The most common pathologic diagnosis
-thin basement membrane disease
-IgAN
Data from: Lee YM, BaekSY, Kim JH, et al. Analysis of renal
biopsies performed in children with abnormal findings in
urinary mass screening. ActaPaediatr 2006; 95:849853.
Hematuria in Children
Define type of hematuria:
Define hematuria as macro or microscopic
Define whether evidence of glomerular or
non-glomerular hematuria.
Radiologic Examination
Microscopic: not always needed; consider US
Macroscopic: depends on whether trauma or
not
Copyright Harvard Medical School, 2010. All Rights Reserved.
590
ACR Guidelines: Hematuria in Children
* CAREFUL HISTORY:
Urinary tract infection
Strenuous activity
Tropical exposure
Recent strep throat
Recent trauma,
Menstruation,
Bleeding tendency
Bloody diarrhea
Pain: joint pain, rash, flank pain dysuria and frequency
Search for h/o trauma, foreign body
FH: stone, sickle cell, stone
Coley BD, GundermanR, Blatt ER, Fordham L, PodbereskyDJ, Prince JS,
Expert Panel on Pediatric Imaging. Hematuria- child. Reston (VA):
American College of Radiology (ACR); 2006
ACR Guidelines: Hematuria in Children
* PHYSICAL EXAMINATION
height and weight
Fevers
arthritis
Rashes
Edema
Abdominal examination-- nephromegaly, abdominal masses
Genital exam
Rectal exam
CVA tenderness
Cranial nerves- look for obvious deafness
Coley BD, GundermanR, Blatt ER, Fordham L, PodbereskyDJ, Prince JS,
Expert Panel on Pediatric Imaging. Hematuria- child. Reston (VA):
American College of Radiology (ACR); 2006
Copyright Harvard Medical School, 2010. All Rights Reserved.
591
Gross Hematuria
Most common: glomerulopathy, urinary
infection, hypercalciuria, stones,
congenital urologic conditions, and trauma
Less common: malignancy, angiomatous
malformations of the collecting system, the
nutcracker syndrome, ADPKD, sickling
hemoglobinopathies, hyperoxaluria,
idiopathic thrombocytopenic purpura,
schistosomiasis, and factitious hematuria
Urological Series
342 patients with gross hematuria.
272 boys52 (19%) had benign
urethrorrhagia
Other findings: URI, trauma, stones
3 patients with low-grade transitional cell
carcinoma
1 patient with Wilms tumor.
Greenfield SP, Williot P, Kaplan D. Gross hematuriain children: a ten-year review. Urology 2007;
69:166169.
Copyright Harvard Medical School, 2010. All Rights Reserved.
592
#2-Hypertension in Children
*Hypertension in child: etiology is age-
dependent
*Evaluation differs from that in adults
*Treatment varies, depending on dx.
Staged evaluation, age-dependent
evaluation
4th Report Updates
Updated BP norms from NHANES 1999-
2000
Redefined terminology
BP percentiles- 50
th
, 90
th
, 95
th
, 99
th
www.nhlbi.nih.gov/guidelines/
hypertension/child_tbl.htm
Copyright Harvard Medical School, 2010. All Rights Reserved.
593
Manual Reading is Standard
Auscultatory(manual) and Oscillometric
(automated) BP measurements are not
interchangeable
Park et al, Arch Pediatr Adolesc Med, 2001;155
Podoll et al, Pediatrics, 2007;119:e538-43
Oscillometric machines are not
interchangeable
Kaufmann et al, Anesth Analg, 1996;82:377381
Underdiagnosis of HTN in Children
Review of electronic medical record from Ohio of
14,187 pts, 3-18 years
507 (3.6%) with HTN
131 (26% of those) identified as HTN
485 (3.4%) with Pre-HTN
55 (11% of those) identified as Pre-HTN
Data from: Hansen et al, JAMA, 2007 Aug;298:874-9
Copyright Harvard Medical School, 2010. All Rights Reserved.
594
Rising Rates of HTN
School based BP screening in 5102
adolescents
20% overweight (BMI 95
th
percentile)
Prevalence of HTN after 3 readings 4.5%
All ethnic variability was accounted for by
BMI
Sorof et al, Pediatrics 2004;113:475-482
Measurement of Blood
Pressure in Children
Children >3 years old should have their BP
measured
Auscultation is the preferred method of BP
measurement
Correct measurement requires a cuff that is
appropriate to the size of the childs upper arm
Elevated BP must be confirmed on repeated
measurement
BP >90th percentile obtained by oscillometric
devices should be repeated by auscultation
Copyright Harvard Medical School, 2010. All Rights Reserved.
595
Conditions Under Which Children Under 3
Years Old Should Have BP Measured
Solid organ transplant
Malignancy or bone marrow transplant
Treatment with drugs known to raise BP
Other systemic illnesses associated with
hypertension
Evidence of elevated intra-cranial pressure
Recommended Dimensions
for Blood Pressure Cuff Bladders
Maximum Arm
Age Range Width (cm) Length (cm) Circumference (cm)
*
Newborn 4 8 10
Infant 6 12 15
Child 9 18 21
Small adult 10 24 26
Adult 13 30 34
Large adult 16 38 44
Thigh 20 42 52
*Calculated so that the largest arm would still allow bladder to encircle arm by at
least 80%.
Copyright Harvard Medical School, 2010. All Rights Reserved.
596
BP Cuff Size
40%of arm
circumference
Technique
Small cuff raises BP
Converse less dramatic
Leg BPs higher than arm measure
Oscillometric measurement highly
sensitive to movement
Copyright Harvard Medical School, 2010. All Rights Reserved.
597
MARIANA
Mariana is a 12 yo girl whose BP has been high
in her pediatricians office on three occasions.
She is at the 50
th
percentile for height and
weight.
Her mom says the BP is lower at CVS, where
they measured it, because, she tells you, we
were worried. Her grandfather died of a stroke.
You take Marianas BP, and it is 128/82 mmHg.
You ask the school nurse to check the BP, and
the values jump around.
What might you do next?
Systolic BP (mmHg) Diastolic BP (mmHg)
Age BP Percentile of Height Percentile of Height
(Year) Percentile 5th 10th25th50th75th90th95th 5th 10th25th50th75th90th95th
12 50th 101 102 104 106 108 109 110 59 60 61 62 63 63 64
90th 115 116 118 120 121 123 123 74 75 75 76 77 78 79
95th 119 120 122 123 125 127 127 78 79 80 81 82 82 83
99th 126 127 129 131 133 134 135 86 87 88 89 90 90 91
Blood Pressure Levels for Girls
by Age and Height Percentile
Copyright Harvard Medical School, 2010. All Rights Reserved.
598
Classification of HTN in Children &
Adolescents, With Measurement Frequency
and Therapy Recommendations
SBP or DBP Percentile
Normal <90th
Prehypertension 90th to <95th or if BP exceeds 120/80 even
if below 90th percentile up to <95th
percentile
Stage 1 hypertension 95th percentile to the 99th percentile plus 5
mmHg
Stage 2 hypertension >99th percentile plus 5 mmHg
Classification of HTN in Children &
Adolescents, With Measurement Frequency
and Therapy Recommendations
Frequency of BP Measurement
Normal Recheck at next scheduled physical
examination
Prehypertension Recheck in 6 months
Stage 1 hypertension Recheck in 12 weeks or sooner if the
patient is symptomatic; if persistently
elevated on 2 additional occasions, evaluate
or refer to source of care within 1 month
Stage 2 hypertension Evaluate or refer to source of care within 1
week or immediately if the patient is
symptomatic
Copyright Harvard Medical School, 2010. All Rights Reserved.
599
Ambulatory Blood
Pressure Monitoring
Useful in the evaluation of
White-coat hypertension
Target organ injury risk
Apparent drug resistance
Drug-induced hypotension
Provides additional BP information in
Chronic kidney disease
Diabetes
Autonomic dysfunction
ABPM should be performed by clinicians
experienced in its use and interpretation.
Treating Mariana
You find that most of Marianas BPs are
between Prehypertensive and Stage 1.
Would you start medication?
Especially in view of her grandfathers
history?
Copyright Harvard Medical School, 2010. All Rights Reserved.
600
Classification of HTN in Children &
Adolescents, With Measurement Frequency
and Therapy Recommendations
Therapeutic Lifestyle Changes
Normal Encourage healthy diet, sleep, and physical
activity
Prehypertension Weight management counseling if
overweight, introduce physical activity and
diet management
Stage 1 hypertension Weight management counseling if
overweight, introduce physical activity and
diet management
Stage 2 hypertension Weight management counseling if
overweight, introduce physical activity and
diet management
Indications for
Antihypertensive Drug
Therapy in Children
Symptomatic hypertension
Secondary hypertension
Hypertensive target-organ damage
Diabetes (Types 1 and 2)
Persistent hypertension despite
nonpharmacologicmeasures
Copyright Harvard Medical School, 2010. All Rights Reserved.
601
MARIANA, CONTINUED
HOW MUCH WORK UP SHOULD
MARIANA UNDERGO?
SHE AND HER FAMILY TELL YOU THAT
THEY DO NOT WANT HER TO HAVE
TOO MANY TESTS.
Clinical Evaluation of
Confirmed Hypertension
Study or Procedure Purpose Target Population
Evaluation for identifiable causes
History, physical
examination, including sleep
history, family history, risk
factors, and habits such as
smoking, alcohol, eating
habits
History and physical
examination help focus
subsequent evaluation
All children with persistent
BP >95th percentile
BUN, creatinine, electrolytes,
urinalysis, urine culture
R/O renal disease and
chronic pyelonephritis
All children with persistent
BP >95th percentile
CBC R/O anemia, consistent with
chronic renal disease
All children with persistent
BP >95th percentile
Renal U/S R/O renal scar, congenital
anomaly or disparate renal
size
All children with persistent
BP >95th percentile
Copyright Harvard Medical School, 2010. All Rights Reserved.
602
Clinical Evaluation of
Confirmed Hypertension
Study or Procedure Purpose Target Population
Evaluation for comorbidity
Fasting lipid panel, fasting
glucose
Identify hyperlipidemia,
identify metabolic
abnormalities
Overweight patients with BP
at 90th94th percentile; all
patients with BP >95th
percentile. Family history of
hypertension or
cardiovascular disease.
Child with chronic renal
disease
Drug screen Identify substances that
might cause hypertension
History suggestive of
possible contribution by
substances or drugs.
Polysomnography Identify sleep disorder in
association with
hypertension
History of loud, frequent
snoring
Clinical Evaluation of
Confirmed Hypertension
Study or Procedure Purpose Target Population
Evaluation for target-organ damage
Echocardiogram Identify LVH and other
indications of cardiac
involvement
Patients with comorbid risk
factors* and BP 90th94th
percentile; all patients with
BP >95th percentile
Retinal exam Identify retinal vascular
changes
Patients with comorbid risk
factors and BP 90th94th
percentile; all patients with
BP >95th percentile
Further evaluation as indicated
Ambulatory BP monitoring Identify white-coat
hypertension, abnormal
diurnal BP pattern, BP load
Patients in whom white-coat
hypertension is suspected,
and when other information
on BP pattern is needed
Copyright Harvard Medical School, 2010. All Rights Reserved.
603
Clinical Evaluation of
Confirmed Hypertension
Study or Procedure Purpose Target Population
Plasma renin determination Identify low renin, suggesting
mineralocorticoid-related
disease
Young children with Stage 1
hypertension and any child
or adolescent with Stage 2
hypertension
Positive family history of
severe hypertension
Renovascular imaging Isotopic scintigraphy (renal
scan)
Magnetic resonance
angiography
Plasma and urine steroid
levels
Identify steroid-mediated
hypertension
Young children with Stage 1
hypertension and any child
or adolescent with Stage 2
hypertension
Plasma and urine
catecholamines
Identify catecholamine-
mediated hypertension
Young children with Stage 1
hypertension and any child
or adolescent with Stage 2
hypertension
Primary Hypertension and
Evaluation for Comorbidities
Primary hypertension is identifiable in
children and adolescents
Hypertension and prehypertension are
significant health issues in the young due
to the marked increase in the prevalence
of overweight children
The evaluation of hypertensive children
should include assessment for additional
risk factors
Copyright Harvard Medical School, 2010. All Rights Reserved.
604
Management Algorithm
Measure BP and Height and Calculate BMI
DetermineBP categoryfor sex, age, andheight*
Educateon
Heart Healthy
Lifestyle

For the family


RxSpecific
for Cause
DrugRx

Monitor
Q6Mo
>95%
Prehypertensive
DiagnosticWorkupInclude Evaluationfor Target-
OrganDamage

Secondary
Hypertension
Overweight
Normal
BMI
Overweight
90<95%
<90%
>95%
Normotensive
Therapeutic
Lifestyle
Changes

Repeat BP
In 6 months
Consider DiagnosticWorkupandEvaluation
for Target-OrganDamage

If overweight or comorbidity exists


Weight
Reduction
Primary
Hypertension
Normal
BMI
Consider Referral
To provider with expertise
in pediatric hypertension
DrugRx Weight Reduction
andDrugRx
Overweight
Stage 2 Hypertension Stage 1 Hypertension
Repeat BP
Over 3 visits
Weight
Reduction
Still >95%
90<95% or 120/80mmHg
or 120/80
mmHg
Normal
BMI
DiagnosticWorkupInclude Evaluationfor Target-
OrganDamage

Therapeutic
Lifestyle
Changes

Secondary
Hypertension
or Primary
Hypertension
Classification of HTN in Children &
Adolescents, With Measurement Frequency
and Therapy Recommendations
Pharmacologic Therapy
Normal
Prehypertension None unless compelling indications such as
CKD, diabetes mellitus, heart failure, LVH
Stage 1 hypertension Initiate therapy based on compelling
indications
Stage 2 hypertension Initiate therapy
Copyright Harvard Medical School, 2010. All Rights Reserved.
605
Ambulatory BP Monitoring
ABPM
White Coat HTN
Efficacy of therapy
Can differentiate between Primary and
Secondary HTN
Flynn, Pediatrics, 2002;110:89-93
Better relationship with markers of end
organ damage then casual BP readings
#3-Nephrotic Syndrome in Children
*Nephrotic syndrome in child: etiology
varies with age
*Evaluation differs from that in adults
*Treatment varies, depending on dx.
Copyright Harvard Medical School, 2010. All Rights Reserved.
606
Nephrotic Syndrome Definition
Massive proteinuria
urinary protein loss of >50 mg/kg per 24 hours or protein/creat ratio
>2g/g or 50 mg/kg/day, or 3-4+on dipstick
Hypoproteinemia
(<2 -2.5 g/dL [20 g/L )
Hyperlipidemia
Edema
This is a SYNDROME with multiple causes
Nephrotic Syndrome Definitions- 2
Remission
Urinary protein excretion <4 mg/m2/hour or 0-trace on dipstick
or protein/creatinine ratio <0.2 g/g for 3 consecutive days
Initial responder
Attains complete remission within initial 8 wks of steroids
Initial nonresponder/steroid resistance
Failure to achieve remission during the initial 8 wks of steroids
Relapse
protein loss of >40mg/m2/hour or protein/creat ratio >2g/g or
50 mg/kg/day, or 2+on dipstick for 3 consecutive days
Copyright Harvard Medical School, 2010. All Rights Reserved.
607
Nephrotic Syndrome Definitions-3
Infrequent relapse
1 relapse within 6 months of initial response or 1-3 relapses in
12 months.
Frequent relapse
2 or more relpases within 6 months of initial response; 4 or more
per 12 months.
Steroid dependence
Two consecutive relapses during steroid therapy or within 14
days of ceasing therapySteroid dependence
Late nonresponder
Proteinuria for >8 weeks following one or more remissions
Incidence and Prevalence
US, UK, AU and NZ:
Incidence ~1-2/ 100,000 population .
Prevalence ~ 16-19/ 100, 000 population.
China- may be higher
South Asia- appears higher
Copyright Harvard Medical School, 2010. All Rights Reserved.
608
Primary Glomerulopathies Underlying
Nephrotic Syndrome in Children and
Adults
ENTITY CHILDREN ADULT <60 Y ADULT >60 Y
Minimal lesion 76 20 20
FSGS 8 15 2
Membranous 7 40 39
Membranoproliferative 4 7 0
OTHER 5 18 39
Data from Lewis EJ. Management of the nephroticsyndrome in adults. In: Cameron
JS, GlassockRJ, eds. The nephroticsyndrome. New York: Marcel
Dekker, 1988:461-521. and from OrthS and Ritz Engl J Med 1998; 338:1202-1211
Typical Course and Response
Most children with idiopathic nephrotic
syndrome (INS) respond to corticosteroids
and have minimal change disease (MCD) .
Response to steroids is associated with a
good long-term prognosis for disease
resolution.
Copyright Harvard Medical School, 2010. All Rights Reserved.
609
ISKDC and Nephrotic Syndrome
The International Study of Kidney Disease in
Childhood [ISKDC]-- 1967 to 1974
Enrolled over 500 children aged 12 weeks to 16 years
Diagnoses % % remission
Minimal change 76.4% 93
FSGS 6.9 50
MesProlif 2.3 25
Response highly predictive of renal histology
Glomerulus in Minimal Lesion Nephrotic Syndrome
Copyright Harvard Medical School, 2010. All Rights Reserved.
610
Normal Podocytes
Podocytes in NS
X 2125 Scanning EM
Renal Biopsy/NS
X 6000 Transmission EM
Steroid Responsiveness
Steroid Responsive ~80%
Urinary remission: ~50% by ~1 week; ~80%
by 2-2.5 weeks; ~98% by 28 days
Complete remission: ~50% by 3 weeks; ~80%
by 30 days; ~90% by 5 weeks; ~100% by 8
weeks
Steroid-dependent during 1-year
follow-up, 57% of children.
Copyright Harvard Medical School, 2010. All Rights Reserved.
611
Initial Treatment
Corticosteroids since the mid-1950s.
1956 Dr. Gavin Arneil treated 4 children
with 60 mg daily.
ISKDC -- 60 mg/m2/day for 4 weeks
followed by 40 mg/m2/day 3 days of 7 for
4 weeks.
APN -- alternate days after remission.
Length of Initial Treatment
Studies comparing 2 months vs. 3 or
more:
Meta-analysis favors longer Rx
Studies comparing 3 months vs. 6 months:
Meta-analysis favors longer Rx
From: HodsonEM, Knight JF, Willis NS, Craig JC. Update of Cochrane Database Syst Rev:
CD001533, 2004; PMID: 15106158 [Review]. Cochrane Database of SystRev CD001533,
2005.
Copyright Harvard Medical School, 2010. All Rights Reserved.
612
Prednisone +/- AlkylatingAgent
LattaK, von SchnakenburgC, EhrichJH. A meta-analysis of cytotoxictreatment of
Frequently relapsing nephroticsyndrome in children. PediatrNephrol 2001; 16:
271-82.
Meta-analysis of steroids plus either placebo or alkylatingagent.
Overall: 5 trials: relative risk favored alkylatingagent: 0.32 (0.16, 0.63)
Drugs: either cyclophosphamideor chlorambucil.
BUT: Issues to consider: toxicity of these agents: In these studies:
N assessed
Death 7 (1.1%) 625
Malignancies 3 (0.6%) 534
Seizures 9 (3.4%) 266
Infections 35 (6.3%) 552
Hemorrhagic cystitis 0 552
Leukopenia 151 (33%) 456
Thrombocytopenia 24 ((5.9%) 408
Hair loss 5 (2.1%) 237
Other Therapy
At Presentation:
-Prednisone or Prednisone plus Cyclosporine. Sustained remission longer
In children receiving both. Hoyer, PF et al. J AmerSoc Nephrol 2006; 17: 1151-1157.
Frequent RelapsingNephroticSyndrome:
-Cyclophosphamide
-Chlorambucil
-Cyclosporine A
-Mycophenolate
-Levamisole
-Azathioprineno reduction in relapse rate
-Mizoribine-relapse rates fell by up to 50%
-Tacrolimus
-Vincristine
-Monoclonal antibodies
-ACEi, ARB, etc.
Copyright Harvard Medical School, 2010. All Rights Reserved.
613
Other Therapy
Cyclosporine cf. Cyclophosphamide: Study in children and adults:
cyclophosphamide(2.5 mg/kg/day) for 8 weeks or
CsA(5 mg/kg/day in adults, 6 mg/kg/day in children) for
9 months, tapered off by 25%every month until
complete discontinuation at month 12.
- Almost 90%on both meds in remission for 6 months.
- By one year, about 65%in remission.
- Cyclophosphamidepatients stayed at ~65%fpr 2 years.
-Cyclosporine patients fell to 25% in remission by early
part of second year.
Data from Ponticelli C, Cyclosporinversus cyclophosphamidefor patients
with steroid-dependent and frequently relapsing idiopathic nephrotic
syndrome: a multicentre randomized controlled trial. Nephrol Dial Transplant
1993; 8: 1326-1332.
Probability of Minimal Lesion in Early Biopsy
of Idiopathic Nephrotic Syndrome in a Child
FEATURE PROBABILITY
No adverse findings 90-95%
Random case 90
Female child 70
Child >6 years old 65
Hypertension alone 60
Hematuriaalone 50
Infant (>3 months to 1 year) 20
Postpubertal 20
Steroid resistant 20
Hematuriaplus HTN 10
Infant (<3 months) 0-5%
Tune B, Mendoza S. Treatment of the idiopathic nephroticsyndrome: regimens
and outcomes in children and adults. JASN, 1997 May;8(5):824-32.
Copyright Harvard Medical School, 2010. All Rights Reserved.
614
Focal Segmental Glomerulosclerosis
(FSGS)
In childhood, usually steroid-resistant and
difficult to treat.
There is evidence that FSGS in Children is
increasing in frequency. (Eddy and Symons.
Lancet 2003; 362: 629-39).
Slide: courtesyC. Jennette
Approach to FSGS
Current strategies for control of FSGS use
a stepwise approach
Goals
Normalization of urinary protein excretion and
Prevention of kidney failure.
Copyright Harvard Medical School, 2010. All Rights Reserved.
615
Approach to FSGS
Steroids initially declare self steroid-
resistant.
Cytotoxic medications.
Ancillary care control of nephrotic state.
Steroid-resistant FSGS
Consider genetic testing, Rule out other underlying
Syndromic FSGS? diseases leadingto FSGS
Positive for mutation in podocin, Not done, not available or negative
WT1, CD2AP, TRPC6, etc.
Careful watching, anti-
proteinuric Rx with
ACE inhibitors or
ARBs
No response
Response
Trial with CyA, and Pred.
for at least 6 months
Continue CyA as
Long-term treatment
APPROACH TO STEROID-RESISTANT FSGS
Copyright Harvard Medical School, 2010. All Rights Reserved.
616
Board Review Question 1
A 3 year old boy presents with nephroticsyndrome. He has no
hypertension, no hematuriaand no family history of renal
disease. If you were to do a renal biopsy , what is the
probability that he will have minimal lesion nephroticsyndrome?
1)20%
2)40%
3)65%
4)90%
Board Review Question 1
A 3 year old boy presents with nephroticsyndrome. He has no
hypertension, no hematuriaand no family history of renal
disease. If you were to do a renal biopsy , what is the
probability that he will have minimal lesion nephroticsyndrome?
1)20%
2)40%
3)65%
4)90%
In a male child of this age with no adverse correlates, early
biopsies have shown a 90-95% probability of minimal lesion
disease.
Copyright Harvard Medical School, 2010. All Rights Reserved.
617
Board Review Question 2
Indications for initiating pharmacotherapy in a child with stage I
Hypertension include all of the following except.
1) Hypertensive target-organ damage
2) Type 2 diabetes
3) Renal transplant
4) Obesity
Board Review Question 2
Indications for initiating pharmacotherapy in a child with stage I
Hypertension include all of the following except.
1)Hypertensive target-organ damage
2)Type 2 diabetes
3)Renal transplant
4)Obesity
In a child with stage I hypertension, non-pharmacologic therapy
is indicated, unless there are other findings, which include
symptomatic hypertension, secondary hypertension, hypertensive
target-organ damage, type 1 diabetes or type 2 diabetes, and
persistent hypertension, despite use of non-pharmacologic measures.
Copyright Harvard Medical School, 2010. All Rights Reserved.
618
Pediatric Kidney Disease: A Board
Primer
Julie R. Ingelfinger, M.D.
DISCLOSURE: I am a deputy editor at the DISCLOSURE: I am a deputy editor at the
New England Journal of Medicine, a paid
position
Copyright Harvard Medical School, 2010. All Rights Reserved.
619
Membranous Nephropathy
and FSGS
August 11, 2010
J ohannes Schlondorff
Instructor in Medicine
Division of Nephrology
Beth Israel Deaconess Medical Center
Brigham Renal Board Review Course
Disclosures
None
Copyright Harvard Medical School, 2010. All Rights Reserved.
620
Membranous Nephropathy
Clinical Presentation
Pathology
Etiology
Epidemiology
Natural History / Prognosis
Management and Therapeutic Options
Question 1:
A 54 year old woman developed nephrotic syndrome 2 months
ago. Renal biopsy at that time revealed membranous nephropathy,
~10% tubular atrophy and interstitial fibrosis. She is currently taking
Lisinopril 40mg qhs, Furosemide 40mg bid. On exam, BP 130/80, P
80. Lungs clear, 3+pitting edema. Na 133, K 4.2, Cr 0.9mg/dl;
albumin 2.9g/dl. Urine protein/Cr ratio 6.
All of the following are reasonable therapeutic interventions to
begin today, except:
A. Irbesartan 150mg qd.
B. Increase Furosemide to 80mg bid.
C. Simvastatin 10mg qhs.
D. Protein restricted diet.
E. Alternating steroid/cyclophosphamide modified Ponticelli protocol.
Copyright Harvard Medical School, 2010. All Rights Reserved.
621
60-80% with nephrotic syndrome (NS)
remainder with subnephrotic proteinuria
detected incidentally
~60% of those with subnephrotic range
proteinuria progress to NS within 1-2 years
of presentation
30-40% with microscopic hematuria
Only 10-20% with HTN
<20% with significantly impaired renal
function
Clinical Presentation
Uniform thickening of glomerular capillary
wall by PAS / light microscopy.
Craters and spikes in glomerular
basement membrane on silver stain
IF with diffuse, granular deposits of IgG
(predominantly IgG4) along basement
membrane
Subepithelial electron dense deposits on
EM (stages described by Ehrenreich&
Churg)
Pathology
Copyright Harvard Medical School, 2010. All Rights Reserved.
622
Copyright Harvard Medical School, 2010. All Rights Reserved.
623
IgG
Copyright Harvard Medical School, 2010. All Rights Reserved.
624
Stage I:
Subepithelial deposits only
located on the surface of the
GBM.
Stage II:
Deposits are partially
surrounded by new basement
membrane. Classic spike
pattern on silver stain.
Stage III:
Incorporation of deposits into
basement membrane.
Stage IV:
Electo-lucent remnants of
deposits within the basement
membrane.
Stage V:
Capillary walls diffusely
thickened. No increase in
mesangial cells or matrix.
Pathology
History on MN
1957: David Jones identifies the unique glomerularpathologic findings of
MN, distinguishing it from other causes of nephrotic
glomerulonephritis:
Striking granular aggregations of IgGand electron-dense deposits
along the subepithelial aspect of the GBM, believed to represent
immune complexes arising from the circulation
1959: Heymann et al. describe a rat model of MN, induced by active
immunization of Lewis rats with preparations of renal brush border
proteins (Heymannnephritis model).
Hypothesis: Subepithelial deposits result from glomerulartrapping of
circulating immune complexes formed by circulating BB antigens and
corresponding antibodies.
1978: Van Damme et al. and Couser et al.: Injection of anti-BB antibodies in
live rats and in ex vivo kidneys can bind glomeruli in the absence of
BB antigen.
Hypothesis: The immune complex formation occurs in situ (fixed
antigen)
Copyright Harvard Medical School, 2010. All Rights Reserved.
625
Glassock(2009) NEJM 361:81
Several candidate antigens in idiopathic
membranous nephropathy have recently
been identified:
M-type phospholipase A
2
receptor (PLA
2
R)
Beck et al (2009) NEJM
neutral endopeptidase
Debiec et al (2002) NEJM; Ronco & Debiec (in press) Arch Med Sci
a-enolase (?)
Wakui et al (1999) Clin Exp Immunol
Pathophysiology
Copyright Harvard Medical School, 2010. All Rights Reserved.
626
Pathophysiology
Reported sensitivity and specificity for PLA2R antibodies for IMN are 75%
and 100%, respectively.
Unclear if 25% of patients with MN and no PLA2R antibody have antibodies
to another antigen, have secondary MN, or represent those with a lag
between loss of PLA2R antibodies and resolution of proteinuria.
Preliminary data (Beck et al, 8
th
International Podocyte Conference, 2010)
suggest that PLA2R antibody titers and proteinuria have a linear correlation,
though residual proteinuria may remain for some time after no PLA2R
antibody is detected.
2/3 of cases are primary or idiopathic
Etiology
Ponticelli (2007) J Nephrol 20:268
Copyright Harvard Medical School, 2010. All Rights Reserved.
627
Idiopathic MN until recently the leading cause of
NS in Caucasian adults. Still remains 2
nd
or 3
rd
cause.
Peak incidence commonly quoted to be between
30-50 years of age, but more recent studies shift
peak to 60-79.
Incidence of MN equal in men and women, but
patients with ESRD due to MN have a M:F ratio
of 2-3:1.
In USA and Europe, MN is second or third
leading cause of ESRD in patients with a
primary glomerulonephritis.
Epidemiology
Rule of thirds 30% spontaneous
remission; 30% progression to ESRD;
30% variable proteinuria but stable
renal function; 10% death from non-
renal causes.
BUT, rate of progression to ESRD
within 5 years appears to correlate with
prevalence of nephrotic syndrome.
Natural History
Schieppati et al (1993) NEJM
du Buf-Vereijke et al (2005) AJKD
Copyright Harvard Medical School, 2010. All Rights Reserved.
628
Little role for pathology in predicting
prognosis independently of clinical data.
Predictive risk model developed by
Cattran et al (1997) KI
Prognostic Factors
Ponticelli (2007) J Nephrol 20:268
Risk Stratification - Cattran(2005) J ASN 16:1188
Requires a minimum of a 6 month observation
period.
Utilizes (1) lowest sustained degree of
proteinuria over the observation period, (2) initial
CrCl, and (3) DCrCl over 6 month period.
Risk Group Low Moderate High
Proteinuria <4g/day 4-8g/day >8g/day
Renal fxn Normal Normal Impaired GFR
CRI / 5yrs 5% ~50% 75%
Copyright Harvard Medical School, 2010. All Rights Reserved.
629
Value of Remission - Troyanovet al (2004)KI
348 nephrotic iMN patients in Toronto GN Registry with
>12mo follow-up
CR: <0.3g protein/d
PR: <3.5g protein/d & >50% reduction in proteinuria
Substantial relapse rates
for both CR (23%) and PR
(47%); many experienced
a 2
nd
remission.
Relapses affected rate of
decline of CrCl, but not
risk of ESRD.
Management
If secondary MN, treat the cause if possible.
Conservative therapy:
Maintain BP<125/75mm Hg
Dietary protein restriction (Giordano et al (2001) KI)
ACE-I ARB for proteinuria reduction (?) and BP
Statin for hyperlipidemia and proteinuria reduction
Prophylactic anticoagulation controversial.
Variable reported incidence of renal vein thrombosis
and thromboembolic events (20-50% in severe
nephrotics)
Thromboembolic mortality 14%/4.5yrs; 42% in high
risk patients (Bellomo et al (1993) Nephron)
Decision analysis model favors anticoagulation (Sarasin
& Schifferli (1994) KI)
Copyright Harvard Medical School, 2010. All Rights Reserved.
630
Management
Cattran(2005) JASN 16:1188
Evidence for Therapies
Alternate day prednisone ineffective
3 RCT (Collaborative (1979) NEJ M 201:1301; Cameron et al (1990)
QJ M 274:133; Cattran et al (1989) NEJ M 320:210)
Cytoxic agent (chlorambucil or cyclophosphamide) +
prednisone effective
Three 2-month cycles: Methylpred 1g IV x 3 days, then
prednisone 0.5mg/kg/d x 27 days; then d/c prednisone,
chlorambucil 0.2mg/kd/d x 1 month (Ponticelli (1995) KI)
Similar protocol using cyclophosphamide 2.5mg/kg/d rather than
chlorambucil similarly effective, but with fewer side effects
(Ponticelli (1998) J ASN)
Cyclophosphamide 1.5-2mg/kd/d x 12 months, Methylpred 1g x3
days months 1,3,5 and Prednisone 0.5mg/kd qod x6 months (du
Buf-Vereijken et al (2004) NDT; pts with impaired renal fxn)
Copyright Harvard Medical School, 2010. All Rights Reserved.
631
Chlorambucil Protocol
Ponticelli et al (1995) KI 48:1600
Survival without dialysis
Probability of Response (CR or PR)
Chlorambucil vs. Cyclophosphamide
Ponticelli et al (1998) J ASN 9:444
Probability of Response Probability of Relapse
Cyclophosphamide
Chlorambucil
Copyright Harvard Medical School, 2010. All Rights Reserved.
632
Patients with renal insufficiency
du Buf-Kereijken et al (2004) NDT
Probability of Response Probability of Relapse
Oral cyclophosphamide +steroids x12 months
Renal survival 86% @ 5yrs, 74% @ 7yrs
Historical control renal survival 32%
Cyclosporine
Cattranet al (1999) KI
Multi-center, single blind
49 pts, steroid-resistant MN, NS
CsA 3.5mg/kg/d divided bid +
Pred0.15mg/kg/d x26 weeks,
then tapered off vs. pred alone
CR/PR at 26wks: 75% vs 22%
At 78wks, 39% vs 13%
Renal fxnunchanged.
Cattranet al (1995) KI
17 pts, progressive MN
CsA 3.5mg/kd/d vs placebo
CsA reduced rate of renal
decline, reduced proteinuria;
effect persisted in 6/8pts
Alexopoulos et al (2006) NDT
51 pts, MN, NS, Cr<2mg/dl
CsA vs CsA +prednisolone x12 mo
CsA 2-3mg/kd/d divided bid; goal
trough 100-200ng/ml
Prednisolone 0.6mg/kg/d, tapered
to 10-15mg/d by 6mo
Responders at 12mo maintained on
CsA 1-1.5mg/kd/d prednisolone
0.1mg/kd/d
CR/PR at 12mo: 85% vs 83%
Relapse rate: 47% vs 15%
(associated with trough<100ng/ml)
Copyright Harvard Medical School, 2010. All Rights Reserved.
633
Tacrolimus
Praga et al (2007) KI 71:924
48 pts, MN, NS, CrCl >50ml/min/1.73m
2
Tacrolimus (0.05mg/kg/d) x12 mo then taper vs. placebo
CR/PR @ 18mo: 94% vs. 35%
50% increase sCr: 4% vs 26%
47% of Tac group relapsed by month 30
Li et al (2008) J Neph21:584
25 Chinese pts, proteinuria>6g/24hr after 3-6mo
observation
Tacrolimus vs. cyclophosphamide, all prednisone.
Short follow-up (24wk), but higher response rate
(CR/PR) in tacrolimus group (92% vs 85%)
MMF mixed results
Dussol et al (2008) AJ KD
36 pts, MN, NS, Cr<2.26
MMF 2g/d x 12 months vs
placebo
CR/PR @ 12mo: 37 vs 41%
Branten et al (2007) AJ KD
32 pts, NS, median Cr 1.8
MMF 2g/d +steroids, compared
to historical cyclophosphamide
controls.
CR/PR @ 12mo: 66 vs 72%
No response: 16% vs 0%
Relapse: 38 vs 13%
Chan et al (2007) Nephrology
20 pts, MN, NS, mean Cr 1.1
Treatment nave
MMF 2g/d +pred vs.
chlorambucil
CR/PR @ 15mo: 64 vs 67%
2 non-controlled studies
evaluated patients with MN which
had failed multiple agents or had
steroid or CsA dependence, and
demonstrated response in terms
of proteinuria reduction ability to
withdraw steroids/CsA. (Miller et
al (2000) AJ KD 36:250; Choi et al
(2002) KI 61:1098.)
Copyright Harvard Medical School, 2010. All Rights Reserved.
634
Rituximab
Remuzzi et al (mult publications)
50pts treated with rituximab as
first line therapy.
Rituximab 375mg/m
2
qweek x 4
weeks
CR 20%; PR 36-50%
Generally well tolerated (1
episode of laryngospasm)
Fervenza et al (2008) KI
15 pts, 7 failed other
immunosuppression
Rituximab 1g days 1 and 15
2
nd
course at 6mo if
proteinuria>3g/d and B cell
count >15/ul (10 of 15 pts)
CR 13%; PR 40%
Generally well tolerated (1 pt
with lung neoplasm dx 3
months post therapy)
No randomized control trials.
2 large case series; systematic review by Bomback et al
(2009) CJ ASN
Rituximab
Segarra et al (2009) CJ ASN
Single center; 13 pts, iMN, eGFR>60ml/min
CNI dependence (4+relapses of NS on taper); 8
failed cyclophosphamide; 2 failed chlorambucil;
10 IgG infusion, 11 MMF to limit CNI dose
Rituximab 375mg/m
2
qweek x4; 1 month later,
steroids/MMF discontinued, CNI weaned by 30%
qmonth
All patients weaned from CNI, MMF, pred.
At 12mo 4/13 CR, 9/13 PR.
3 relapses, all responded to single re-treatment
with PR.
At 30mo, 13/13 PR.
Copyright Harvard Medical School, 2010. All Rights Reserved.
635
ACTH
Ponticelli et al (2007) AJ KD
Single center, randomized control study
32 patients, iMN, NS, Cr<1.9
Synthetic ACTH 1mg IM, increased from qoweek
to 2x/week x1 year vs. methylprednisone +
chlorambucil (9/16) or cyclophosphamide (7/16)
CR/PR: 87% vs. 93%. 1 pt progressed to ESRD
in ACTH group
Relapse: 3/14 vs. 7/15.
Side effects mild, reversible.
Summary
Consider rituximabin patients with contraindication to cytotoxictherapy or CNI
dependence, or in a research setting.
Copyright Harvard Medical School, 2010. All Rights Reserved.
636
Question 1:
A 54 year old woman developed nephrotic syndrome 2 months
ago. Renal biopsy at that time revealed membranous nephropathy,
~10% tubular atrophy and interstitial fibrosis. She is currently taking
Lisinopril 40mg qhs, Furosemide 40mg bid. On exam, BP 130/80, P
80. Lungs clear, 3+pitting edema. Na 133, K 4.2, Cr 0.9mg/dl;
albumin 2.9g/dl. Urine protein/Cr ratio 6.
All of the following are reasonable therapeutic interventions to
begin today, except:
A. Irbesartan 150mg qd.
B. Increase Furosemide to 80mg bid.
C. Simvastatin 10mg qhs.
D. Protein restricted diet.
E. Alternating steroid/cyclophosphamide modified Ponticelli protocol.
Focal Segmental
Glomerulosclerosis
Epidemiology
Pathology
Classification
Clinical Presentation
Prognosis
Initial Treatment
Treatment of Steroid-resistant and
dependent FSGS
Copyright Harvard Medical School, 2010. All Rights Reserved.
637
Question 2:
A 46 yo man with a history of hypertension, obesity and peptic ulcer
disease developed nephrotic syndrome. Renal biopsy revealed
secondary FSGS with glomerulomegaly. He returns today for follow-up
4 months after his biopsy. Current medications include Enalapril,
Valsartan, Furosemide, Atorvastatinand Pantoprazole.
On exam: BP 124/72, P 76. Weight 272lbs. Lungs are clear. Heart
exam unremarkable. Abdomen benign. Ankles with 2+edema. Labs
are notable for BUN 16, Cr 1.1mg/dl, Glc 104mg/dl, Alb 3.1g/dl. CBC
within normal limits. UA with 1+blood, 3+protein; urine protein/Cr ratio
4.2.
Based on his presentation, you recommend the following course of
action:
A. Continued conservative therapy and observation.
B. Treatment with Prednisone 80mg qd.
C.Treatment with Cyclosporine 100 mg bid and Prednisone 15mg qd.
D.Treatment with Tacrolimus 2mg bid.
E. Referral for bariatric surgery.
Incidence of FSGS in biopsy series has
risen over the last 30 years.
FSGS most common pathology in biopsies
for idiopathic NS in US; 35% in all cases,
50% in African-Americans
FSGS most common primary glomerular
disease causing ESRD in US (USRDS)
Epidemiology
Copyright Harvard Medical School, 2010. All Rights Reserved.
638
FSGS is a descriptive term for a
pathologic lesion, not a disease entity.
Pathology
Etiological Classification of FSGS
Primary (idiopathic) FSGS
Permeability factor
mediated?
Secondary FSGS
1.Familial / genetic
ApoL-I, a-actinin-4, TRPC6,
NPHS1&2, PLCE1, COQ2
2.Virus-associated
HIV, ParvoB-19, CMV,
HTLV-1
3.Drug/toxin-mediated
Interferon-a, Sirolimus,
Bisphosphonates, Lithium,
Heroin
4.Arising from adaptive
structural-functional
responses
Reduced renal mass (e.g.
renal agenesis or dysplasia,
reflux nephropathy, cortical
necrosis, surgical ablation)
Initially normal renal mass
(e.g. hypertension, obesity,
OSA, sickle cell anemia,
atheroembolicdisease)
Adapted from DAgati et al (2004) AJ KD
Copyright Harvard Medical School, 2010. All Rights Reserved.
639
Histological Variants of FSGS
5 variants:
1. Classic
2. Perihilar
Frequently seen with secondary FSGS due to glomerular
hypertension
3. Tip lesion
Abrupt onset, high degree of proteinuria; better response to
steroids, some spontaneous remissions reported; ? less
likely to progress to ESRD
4. Cellular
5. Collapsing
More severe, poorer prognosis
Prognostic value of histological variants remains
debated.
Pathophysiology
Intact Foot Process
Effacement
Persistent Podocyte
Stress
Podocyte Loss
Apoptosis/Detachment
Decreased
Podocyte Density
Compensatory
Podocyte Hypertrophy
Glomerular
Hyperfiltration/
Hypertrophy
Pathologic
Podocyte Hypertrophy
Glomerulosclerosis
Podocyte
Injury
Sustained Injury
Second hit
TRPC6
?
ACTN4
?
Copyright Harvard Medical School, 2010. All Rights Reserved.
640
FSGS and People of African Descent
Cumulative lifetime risk for ESRD varies
by ancestry; e.g. ~2% for European
Americans, ~7.5% for African Americans
African Americans have ~4x higher risk of
sporatic FSGS, 18-50x higher risk of
HIVAN
The higher risk of FSGS in peoples of
African descent is not specific to US
FSGS and People of African Descent
Higher risk of hypertensive ESKD, HIVAN and FSGS all link to
genetic variants in ApoL1 found in high frequency in West Africa,
and which confer protection against certain trypanosomal infections.
A situation analogous to sickle cell disease/thalasemias?
Odds ratios for recessive carriers: FSGS 10.5; H-ESRD 7.3
Mechanism of disease, value of testing and therapeutic implications
are all still unknown, but stay tuned
Copyright Harvard Medical School, 2010. All Rights Reserved.
641
Mean age 40-50; male:female ~1:1
Primary FSGS
Nephrotic range proteinuria: 60-75%
Microscopic hematuria: 30-50%
Hypertension: 45-65%
Impaired renal function: 25-50%
Secondary FSGS
Slowly progressive proteinuria and renal insufficiency
Peripheral edema and hypoalbuminemia uncommon
even with nephrotic range proteinuria.
Heavy proteinuria uncommon.
Clinical Presentation
Clinical Presentation
Deegens et al (2008) Neth J Med
Copyright Harvard Medical School, 2010. All Rights Reserved.
642
Prognostic Factors
Proteinuria
Nephrotic vs non-nephrotic renal survival of 76% vs. 92% at 5yrs, 57
vs 92% at 10yrs.
Massive proteinuria (>10g/d) has renal survival of ~20% at 5 yrs.
Renal insufficiency
10-yr renal survival 27% if Cr>1.3; 100% if Cr<1.3.
Histology
Interstitial fibrosis
Collapsing variant worse; ?Better prognosis with tip lesion
Response to Therapy
Single most important prognostic factor
10 yr renal survival in one series (Chun et al (2004) J ASN):
Classic Scar Tip Lesion Collapsing
Remission 100% 100% 80%
Non-responder 49% 25% 21%
Prognostic Factor Response to
Therapy
Troyanovet al (2005) JASN 16:1061
Copyright Harvard Medical School, 2010. All Rights Reserved.
643
Treatment Options
Unlike MN, spontaneous remissions in
FSGS are rare (<6%), therefore attempting
to obtain a treatment induced remission is
of importance for primary FSGS.
Treatment of secondary FSGS is limited to
conservative management, and (if
possible) treatment of underlying cause
Treatment in Primary FSGS
Conservative therapy (ACE-I/ARB, BP
control 125/75, ?statin) for all patients.
Immunosuppressive therapy is generally
not suggested for patients with:
1. Preserved renal function and sub-nephrotic
range proteinuria (due to relatively good
prognosis)
2. Impaired renal function and sub-nephrotic
range proteinuria (due to poor prognosis even
with treatment, question of secondary FSGS)
Copyright Harvard Medical School, 2010. All Rights Reserved.
644
Treatment in Primary FSGS
No RCT data for use of
steroids.
Steroids remain first line
therapy.
Current recommendation:
Prednisone 1mg/kg/d for 8-16
wks, or 2mg/kg/qod if high risk
for complications.
Prolonged course may be
necessary! Median time to
remission reported in 3.5-5
month range; additional
responses up to 9 months into
therapy.
Korbet (1998) J ASN
Steroid Taper after Response
CR within 12 weeks:
Continue full dose 1-2 weeks, then
Taper dose (every 2-3 weeks) over 2-3 months.
Consider switching to alt day dosing during taper.
PR by 12 weeks:
Consider switch to alt day dosing.
Slow taper (every 6 weeks) over 6-9 months.
Halt taper if proteinuria rises, consider adding CsA or MMF.
Decreased proteinuria by 12-16 weeks, but no PR:
Assess slope of proteinuria decline, risks/toxicity of continued high
dose prednisone.
Consider continued therapy, alt day dosing, adding CsA or MMF.
No significant response by 12-16 weeks:
Considered steroid resistant FSGS.
Consider starting CsA, alternate day dosing, and tapering steroids.
Copyright Harvard Medical School, 2010. All Rights Reserved.
645
Treatment of Relapses
>75% of patients who relapse will respond to
retreatment.
If relapse occurs after >6-12 months off of
steroids, retreat with prednisone.
If relapse occurs <6-12 months after
discontinuation of steroids, or during steroid
taper, consider patient steroid-dependent.
If significant steroid toxicity, or repeated
relapses, treat with CsA (3.5mg/kg/d) and low
dose prednisone.
Treatment of steroid-dependent FSGS
Response to CsA usually within 1 month, but
relapse rate of ~75% upon taper, leading to
need for prolonged low dose CsA therapy.
Cytotoxic therapy (cyclophosphamide 2mg/kg/d
x2-3 months, high dose prednisone for 1 month)
Korbet (1998) J ASN
Copyright Harvard Medical School, 2010. All Rights Reserved.
646
Treatment of steroid-resistant FSGS
Cyclosporine A
Several RCT demonstrating benefit
Tacrolimus
Uncontrolled, open label trials
Cytotoxic Agents
Limit use to patients with limited response to prednisone alone,
and high risk of CsA toxicity.
No benefit in prolonged treatment (>12 weeks)
MMF
One RCT comparing to prednisone alone in nave patients
Largely observational data; limit to those with contra-indication to
CsA, renal insufficiency
Sirolimus
Single prospective trial suggesting some efficacy, but reports of
adverse effects on renal function.
Rituximab
Case reports of response in children with FSGS, FSGS in tx.
CsA for steroid-resistant FSGS
Cattran et al (1999) KI
Multicenter, RCT
49 pts with SR FSGS
CsA 3.5mg/kg/d divided
bid +Pred 0.15mg/kg/d
vs. Pred alone x 26 wks
CR/PR at 26wks: 70% vs
4%
Relapse rate: 40% at 52
wks, 60% at 78 wks
50% decline in CrCl:
25% vs. 52% at 200 wks
Ponticelli et al (1993) KI
Multicenter, adult & peds,
SR MCD & FSGS
CsA 5mg/kg/d x 6mo,
then taper vs. supportive
care
CR/PR: 57% vs 16%
Relapse rate of 70%
Heering et al (2004) AJ KD
57 pts SR FSGS
Pred 1.5mg/kg/d +CsA
5mg/kg/d vs. Pred +
Chlorambucil 0.1-
0.4mg/kg/d x 6-12 weeks,
followed by CsA
CR/PR: 60 vs 65% (NS)
Copyright Harvard Medical School, 2010. All Rights Reserved.
647
Cattran et al (2007) KI 72:1429
CsAdosing goals:
CR/PR, stable GFR, CsA trough
125-175ng/ml
MMF for FSGS
Senthil Nayagam et al (2008) NDT
Single center, RCT
33pts FSGS and NS
MMF 1g bid +Pred 0.5mg/kg/d x 8-12 wks vs. Pred
1mg/kg/d x 12-24wks
CR/PR: 70 vs 69%
Relapse: 24 vs 19% (NS)
Several observational studies have reported decreases
in proteinuria >50% in 40-50% of patients, occasional
remission, and stable Cr levels in patients with FSGS
resistant or dependent upon steroids, cyclosporine or
cytotoxic agents.
Copyright Harvard Medical School, 2010. All Rights Reserved.
648
Question 2:
A 46 yo man with a history of hypertension, obesity and peptic ulcer
disease developed nephrotic syndrome. Renal biopsy revealed
secondary FSGS with glomerulomegaly. He returns today for follow-up
4 months after his biopsy. Current medications include Enalapril,
Valsartan, Furosemide, Atorvastatin and Pantoprazole. , , p
On exam: BP 124/72, P 76. Weight 272lbs. Lungs are clear. Heart
exam unremarkable. Abdomen benign. Ankles with 2+ edema. Labs
are notable for BUN 16, Cr 1.1mg/dl, Glc 104mg/dl, Alb 3.1g/dl. CBC
within normal limits. UA with 1+ blood, 3+ protein; urine protein/Cr ratio
4.2.
Based on his presentation, you recommend the following course of
action:
A. Continued conservative therapy and observation.
B. Treatment with Prednisone 80mg qd.
C. Treatment with Cyclosporine 100 mg bid and Prednisone 15mg qd.
D. Treatment with Tacrolimus 2mg bid.
E. Referral for bariatric surgery. (?)
Disclosures
None None
Copyright Harvard Medical School, 2010. All Rights Reserved.
649
Polycystic Kidney Disease: an
Update p
Theodore I. Steinman, M.D.
Clinical Professor of Medicine
Harvard Medical School Harvard Medical School
Beth Israel Deaconess Medical Ctr.
Brigham and Womens Hospital
August 9, 2010
Disclosures
None
Copyright Harvard Medical School, 2010. All Rights Reserved.
650
Autosomal Dominant
Polycystic Kidney Disease
Mutation in ~70% located on short arm of
chromosome 16p 13.3 p13.1 (PKD1 locus)
~30% of mutation is located on chromosome
4q21-q23 milder phenotype with PKD2 locus. q q p yp
Later age of diagnosis and hypertension,
smaller kidney volume, fewer kidney cysts,
later age of ESRD system: disorder is kidney,
liver, pancreatic and vascular abnormalities.
Family History of Severity of Renal
Disease Predicts Mutated Gene in ADPK
Median Age of ESRD
PKD1 ~ 53 years PKD1 ~ 53 years
PKD2 ~ 73 years
Presence of at least one affected family member who developed
ESRD at age < 55 years was highly predictive of PKD1 mutation
(PPV 100%, sensitivity 72%)
If family member developed ESRD at age > 70 years, then highly
predictive of PKD 2 mutations (PPV 100%, Sensitivity 74%)
Barua et al JASN 20: 1833, 2009
Copyright Harvard Medical School, 2010. All Rights Reserved.
651
Sonographic Criteria for the Diagnosis of ADPKD
Age (yrs) Criteria PPV NPV
15-29
Original Ravines PKD1 diagnostic criteria
> 2 cysts unilateral or bilateral 99 2 87 7 15-29
30-39
40-59
> 60
> 2 cysts, unilateral or bilateral
> 2 cysts in each kidney
> 2 cysts in each kidney
> 4 cysts in each kidney
99.2
100
100
100
87.7
87.5
94.8
100
15-29
30-39
40-59
60
Revised unified diagnostic criteria
> 3 cysts, unilateral or bilateral
> 3 cysts, unilateral or bilateral
> 2 cysts in each kidney
2 t i h kid
100
100
100
100
85.5
96.4
94.8
100 > 60 > 2 cysts in each kidney 100 100
15-29
30-39
30-49
> 60
Revised criteria for exclusion of diagnosis
> 1 cyst
> 1 cyst
> 2 cysts
> 4 cysts in each kidney
96.6
94.0
96.7
100
90.8
98.3
100
100
Torres, KI 76:149-168, 2009
Cystogenesis
When cysts detach early from the tubule of origin, fluid
accumulates within the expanding cyst as a result of the
primary transepithelial transport of chloride through cyclic
AMP regulated channels in the apical lining cells.
CFTR (cystic fibrosis transmembrane conductance
regulator) present on apical surface of cyst epithelial cells.
Copyright Harvard Medical School, 2010. All Rights Reserved.
652
TwoTypesofCilia
Motilecilia
Beatrhythmically
Foundprimarilyinthelungsandairways
Helptomovefluidandsecretions
Primarycilia
Donotbeat
Foundonalmostallcells
Sensoryfunction
Cilia
Tiny,hair-like
structures that Cilia structuresthat
projectfromthe
surfaceofthecell
Cilia
Cell
Nucleus
Copyright Harvard Medical School, 2010. All Rights Reserved.
653
PrimaryCiliaonKidneyCells
Wheatley,1995;Pan,2005
EvidencethatPKDisaDisorderofCilia
TheproteinsthataremutatedinADPKD
andARPKDarelocatedinkidneycilia
Polycystin-1
Polycystin-2
Fibrocystin
Mice lacking kidney cilia develop PKD MicelackingkidneyciliadevelopPKD
Polycystin-1andpolycystin-2arerequired
forciliarysignaling
Copyright Harvard Medical School, 2010. All Rights Reserved.
654
Cilia in ADPKD
J Am Soc Nephrol 13:2384-2398, 2002
CiliaBendingProducesaRiseinCell
Calcium
Calcium
concentration
adaptedfromPraetorius,2001
Copyright Harvard Medical School, 2010. All Rights Reserved.
655
The calcium increases results
in K secretion
PolycystinsareRequiredfortheRisein
CellCalcium
e
Pkd1mutant
Normal
%

c
a
l
c
i
u
m

i
n
c
r
e
a
s
e
Nauli,2003
calcium
cilia
bending
polycystin-1
polycystin-2
X
X
X
Copyright Harvard Medical School, 2010. All Rights Reserved.
656
Cilia
FLOW
RegulationofCellGrowth byCilia
Cell
inversin
-catenin
Nocell
growth
Simons,2005
genes
Nucleus
LossofCiliaCausesAccumulationof-Catenin
Cell
X
Nucleus
-catenin
Nucleus
Lin,2003
Cell
growth
genes
Copyright Harvard Medical School, 2010. All Rights Reserved.
657
Hypothesis About Role of
Angiogenesis in ADPKD
Necessary for cyst cells to grow Necessary for cyst cells to grow
Responsible for increased vascular
permeability facilitating fluid secretion
into cysts
Neovascularization may result in
formation of aneurysms formation of aneurysms
? Brain PC 1 & 2 expressed in smooth
muscle cells of cerebral arteries
? Renal bleeding
Copyright Harvard Medical School, 2010. All Rights Reserved.
658
CRISP RESULTS
Total Kidney Volume (TKV) increased over the
three-year interval +204 ml+246 (n=214 P<.001)
and Total Cyst Volume (TCV) increase +218ml
+ 263 (n 210 P< 001) + 263 (n=210, P<.001).
There was a direct correlation (r= 0.95, P<.001)
between the rate of change in TKV and the rate
of change in TCV.
The change of volume in the left kidney
correlated directly with the change in the right
kidney (r=0.67, P<.001).
New England J. Of Medicine 354(20):2122-30
Copyright Harvard Medical School, 2010. All Rights Reserved.
659
Relation between Age and Total Kidney Volume in Patients with
Autosomal Dominant Polycystic Kidney Disease.
Total kidney volume (the sum of the volumes of the left and right kidneys) is shown in 232 women (blue
data points) and men (red data points) studied over a 3-year period. The total kidney volume increased
progressively in most patients but at widely differing rates. The two black lines superimposed on the data
points demarcate slow, moderate, and rapid rates of progression. The rate of disease progression in
individual adults at any age can be determined by measuring kidney length, width, and thickness on MRI,
CT, or ultrasound scans and estimating the kidney volume with the ellipsoid equation.
Grantham, JJ N Engl J Med 359:14
Copyright Harvard Medical School, 2010. All Rights Reserved.
660
Relationship between kidney
volume (US) and GFR
Fick-Brosnahan AJ KD: 112, 2002
Significant correlation between GFR and renal volume
in 229 patients with ADPKD
Schrier JASN 20:1888-1893, 2009
Copyright Harvard Medical School, 2010. All Rights Reserved.
661
Volume as an Endpoint in ADPKD
Cysts are the disease, volume equals severity
Cyst growth is distinctive and measurable
Cyst volume strongly linked to functional decline
Proliferation of promising treatments for ADPKD
Acquired Cystic Kidney Disease
ACKD becomes a confounding variable
goes on at a subliminal level for years in
patients with ADPKD preceding ESRD
(related to slow progression of CKD).
Copyright Harvard Medical School, 2010. All Rights Reserved.
662
Relationship between
hypertension and PKD
Across all age ranges
hypertension is noted when TKV
is ~ 1000 mL (~600 mL/cm height)
Progressive Loss of Kidney Function
Rate of decline of GFR (data from MDRD study, starting
at GFR <55 ml/min)
Males 5 - 6 ml/min/year
Females 4 - 5 ml/min/year
Pattern of GFR loss has recently been established by
CRISP study
GFR stable for many years, despite progressive
increase in total kidney volume increase in total kidney volume
GFR decrease not detected until total kidney
volume exceeds 1500 ml
Copyright Harvard Medical School, 2010. All Rights Reserved.
663
Loss of Kidney Function in ADPKD
80
100
%
)
20
40
60
R
e
n
a
l

f
u
n
c
t
i
o
n

(
%
Torres Mayo <aupCP1047707-9
0
0 10 20 30 40 50 60
Age (years)
Predictors of Disease Progression
(MR)
Hemodynamic
Anatomic
Hemodynamic
RBF
RVR
Anatomic
Total kidney
volume
RVR
GFR
Total cyst volume
% cyst volume
CRISP Study KI 2003; 64:2214-2221
Copyright Harvard Medical School, 2010. All Rights Reserved.
664
Principal Extrarenal Manifestations
Hepatic and pancreatic cysts
Asymptomatic in many patients, but can expand and
cause pain and infection; rarely massive PLD cause pain and infection; rarely massive PLD
Cardiac valvular abnormalities
Mitral, tricuspid and aortic valve prolapse and
regurgitation
Intracranial aneurysms
Risk of rupture; found in approximately 5% of patients s o uptu e; ou d app o ate y 5% o pat e ts
with no family history and about 22% of patients with
family history of ICA or SAH
Seminal vesicle cysts
Found in ~39-60% of men; undefined risk of infertility
Nephrolithiasis in ADPKD
Occurs in ~20-36% of patients
Uric acid (UA) (~50%); Ca Ox (~47%)
Predisposing factors include
hypocitraturia, hyperoxaluria,
hypercalciuria, hypomagnesuria, possible
distal acidification defects
S
UA
and renal urate handling normal
UA
g
Expanding cysts compress the collecting
system producing urinary stasis, which
predisposes to stone formation and
infection
Copyright Harvard Medical School, 2010. All Rights Reserved.
665
Patient Characteristics in Stone
Formers
Old Older
Higher incidence of hypertension
Hepatic cysts
Impairment in submaximal Uosm
History/Presence of UTIs
Low urine volume
LOW URINE pH (major factor)
ADDITIONAL FACTORS in Stone
Formation
Activation of RAAS occurs early in ADPKD
Type A acid secreting intercalated cells
(OMCD) express Type angiotensin
receptors (AT
1
R)
A stimulates vacuolar H
+
-ATPase activity
in intercalated cells in intercalated cells
Copyright Harvard Medical School, 2010. All Rights Reserved.
666
Hypothesis
In ADPKD upregulation of A renal
i lt i ti l ti f H
+
i t expression results in stimulation of H
+
into
CD lumen
Failure to adequately buffer increased H
+
results in urine acidification (UA stone risk
?Will Rx with ACEI &/or ARBs increase
urinary pH by downregulating H
+
-ATPase
Pech et al JASN 19:84-91, 2008
Treatment of Nephrolithiasis in
ADPKD
With high incidence of h pocitrat ria K With high incidence of hypocitraturia, K
citrate might be useful to prevent uric acid
stones and reduce calcium oxalate super
saturation.
Alkalinization of urine
Copyright Harvard Medical School, 2010. All Rights Reserved.
667
ECSWL in ADPKD
Can be performed safely, but residual
fragments in ~50% of patients (higher than
in the general population undergoing
lithotripsy).
Percutaneous Nephrolithotomy for
Large & Multiple Upper Tract
Calculi
Can be done safely when stone >2 cm
In minority of pts second stage endoscopic
ultrasonic lithotripsy procedure
Percutaneous basket extraction (?) ( )
Nishiuria et al CJASN 4:838-844, 2009
Copyright Harvard Medical School, 2010. All Rights Reserved.
668
Kidney Infection in ADPKD
30 to 50% of patients with ADPKD will have a
urinary tract infection either pyelonephritis or urinary tract infection, either pyelonephritis or
cyst infection, during their lifetime
Urinary tract infections are more common in
women with ADPKD
Fever and flank pain are the presenting
symptoms y
Urine culture may be negative in cyst infection,
as cysts frequently dont communicate with the
collecting system
Hematuria in ADPKD
Cyst hemorrhage occurs in ~60% of individuals
gross or microscopic hematuria if cyst connects to
collecting system g y
intracyst or subcapsular hemorrhage without hematuria
Correlation with increased TKV
Excessive angiogenesis results in fragile blood
vessels stretched across walls of enlarging cysts;
susceptible to minor trauma with resultant p
hemorrhage
Early onset of recurrent gross hematuria correlates
with kidney size and more rapid progression to
kidney failure
Copyright Harvard Medical School, 2010. All Rights Reserved.
669
Cyst Infection v. Intracystic
Hemorrhage in ADPKD:
CT & MR Imaging Criteria
Characteristics of infected c sts on MRI Characteristics of infected cysts on MRI:
Intensity: higher on T1WI and DWI and
lower on T2WI than uninfected cysts
Fluid-level, cyst wall thickening and
increased density (on both MR & CT) increased density (on both MR & CT)
Characteristics of hemorrhage on MRI:
Significantly higher T1WI, T2WI and DWI
Irregular higher density on CT
Evaluation of Infection
PET (positron emission tomography) scan
is most useful in detecting infected cysts
(detects degree of inflammation).
Copyright Harvard Medical School, 2010. All Rights Reserved.
670
Treatment of Hematuria in ADPKD
Appropriate diagnosis and treatment of
specific entity, such as infection or stone
Correction of coagulopathy, if present
Conservative management with hydration,
bed rest, and appropriate use of
analgesics
Rarely, massive bleeding may require
transfusion, kidney embolization (TAE) or
nephrectomy
Treatment of Kidney Cyst Infection
in ADPKD
Lipophilic antibiotics such as ciprofloxacin, Lipophilic antibiotics such as ciprofloxacin,
norfloxacin, trimethoprim, chloramphenicol
Percutaneous or operative drainage is
rarely needed; only for refractory infection
Resistant organisms
Localization of infected cyst is difficult y
Labeled WBC or gallium scan
MRI with contrast
PET scan
Copyright Harvard Medical School, 2010. All Rights Reserved.
671
Liver Cysts: Sx & Infection
With marked hepatomegaly:
-- heaviness, dull ache
-- mechanical low back pain
-- early satiety
If fever persists 1-2 wks after antibiotics in
infected cysts, drainage frequently needed
Hepatic cyst infection more serious than
renal cyst infection. Do not delay drainage.
Carbohydrate Antigen (CA) 19-9:
Marker for Hepatic Cyst Infection
CA 19-9 secreted by biliary epithelium CA 19 9 secreted by biliary epithelium
lining hepatic cysts and overproduction in
hepatic cyst infection
Immunostaining for CA 19-9 showed
strong positivity in biliary tree epithelia and
in liver cysts. in liver cysts.
Rise (active infection) / fall (improvement)
Kanaan et al AJKD March, 2010
Copyright Harvard Medical School, 2010. All Rights Reserved.
672
Screening for cerebral aneurysm
Polycystin expression in VSMC and
endothelium vascular
malformation.
A d l t t l t d t Aneurysm development not related to
hypertension in ADPKD, but uncontrolled
BP can accelerate the process
Aneurysm (ICA) Formation
in ADPKD
General Population
ADPKD
Incidence
General Population
2.3 %
16.0 % *
* 2 first degree relatives with ICA,
smoking history, hypertension
ADPKD
~ 8 %
~ 17%
With family history of ICA/SAH
90% of aneurysms in anterior circulation
10% of aneurysms in posterior circulation (greater risk of rupture)
Torres 2009
Copyright Harvard Medical School, 2010. All Rights Reserved.
673
Risk of ICA Rupture in ADPKD
%Yr
1.1
Aneurysm Size
2-6 mm
2.2
2.8
7-9 mm
10-26 mm
0.05% per year < 10 mm. No personal or family history of SAH
0.5% per year < 10 mm. With personal or family history of SAH % p y p y y
Betz KI 63: 2003
Gibbs KI 64: 2004
No Hx SAH
0
Hx SAH
1.5% per year
Risk Factors for ICA Rupture
1. Most aneurysms have a very low risk of
rupture and occurs without a family
history
2. With 2 PKD relatives with SAH the RR =
2.15
3 F > M and ICA > 8 mm 3. F > M and ICA > 8 mm
4. Pack years of smoking
5. Hypertension > 10 years
Torres 2009
Copyright Harvard Medical School, 2010. All Rights Reserved.
674
Risks of Intervention
Mortality 0.6 3.5%
Morbidity 4.1 25.4%
Follow up of ICA in ADPKD
< 7 mm Obser ation < 7 mm Observation
7 12 mm Risk assessment
> 12 mm Intervene
Copyright Harvard Medical School, 2010. All Rights Reserved.
675
Recommendations for ICA
Screening
1 Famil histor of ICA or SAH 1. Family history of ICA or SAH
2. Personal history of SAH
3. Prior to major elective surgery
(transplant)
4 High risk occupation 4. High risk occupation
5. Need for reassurance ?
(A and B) PRA (A) and aldosterone (B) are stimulated in
ADPKD as compared with patients with essential
hypertension.
Schrier JASN 20:1888-1893, 2009
Copyright Harvard Medical School, 2010. All Rights Reserved.
676
Pathogenetic role of RAAS in ADPKD. In addition to its vasoconstrictor
effect on BP, angiotensin II is known to increase cell proliferation,
angiogenesis, oxidant injury, and fibrosis, known renal components of
ADPKD P t ti l i di t ff t f i t i II i ADPKD. Potential indirect effects of angiotensin II on causing
hypertension include stimulation of the sympathetic nervous system,
endothelium, and aldosterone with sodium retention. Angiotensin II also
stimulates reactive oxygen species, which could account, at least in part,
for the observed endothelial dysfunction in ADPKD.
Schrier JASN 20:1888-1893, 2009
(A) Effect of BP control with
amlodipine versus enalapril on
LVMI in patients with ADPKD over
7 yr.
Effect of rigorous versus
standard BP control on LVMI in
patients with ADPKD over 7 yr
Schrier JASN 20:1888-1893, 2009
Copyright Harvard Medical School, 2010. All Rights Reserved.
677
Correlation between BP and LVMI in PKD
Schrier JASN 20:1888-1893, 2009
HALT-PKD
(Halt Progression of Autosomal
Dominant Polycystic Kidney Disease)
Objective: Two concurrent, randomized, double-blinded Objective: Two concurrent, randomized, double blinded
controlled trials to assess the effects of multi-level blockade
of the renin-angiotensin-aldosterone system (RAAS) and
aggressive blood pressure control on progression of early
(NKF Stage 1-2) and late (NKF Stage 3) ADPKD over a 5-
year period
H th Hypotheses:
1) Blockade of RAAS will significantly reduce renal
progression as compared to other antihypertensive therapy
2) Lower blood pressure will significantly reduce renal
progression as compared to standard BP targets
Copyright Harvard Medical School, 2010. All Rights Reserved.
678
Tolvaptan
Diagram depicting putative pathways up- or downregulated in
polycystic kidney disease and rationale for treatment with a V2
receptor antagonists, somatostatin, triptolide; tyrosine kinase,
src, MEK, TNFa, mTOR, or CKD inhibitors; metformin, and CFTR
or KCA3.1 inhibitors (green boxes)
Torres, Harris, KI 76:149-168, 2009
Copyright Harvard Medical School, 2010. All Rights Reserved.
679
Water in Treatment in PKD
Tubule cell proliferation Transepithelial fluid secretion
cAMP stimulates growth
Enlarging Renal cysts
V2 receptor
AVP (ADH)
Renal dysfunction
Torres, Bankir, Grantham CJASN: 4: 1140-1150, 2009
Blocking action of AVP dramatically ameliorates the disease process
Cysts are the Disease
Interference with primary cilia formation or its
resident PC1/PC2 complex leads to the resident PC1/PC2 complex leads to the
formation of cysts.
High cystic burden destroys normal parenchyma
and ultimately results in ESRD for majority of
patients
Goal is to target cyst growth before it
disconnects from parent nephron. Prevent initial
proliferative phase stemming from cilial defects
Copyright Harvard Medical School, 2010. All Rights Reserved.
680
Cyst Reduction with mTOR
Inhibitors
C sts de elop beca se of epithelial cell Cysts develop because of epithelial cell
proliferation, in part mediated by mTOR
In experimental animals, reduction of
kidney & liver cyst growth (and hence
increase in parenchymal volume) with p y )
mTOR inhibitors
No change in pain pattern reported in early
limited clinical studies
mTOR Inhibitor Trials
Everolimus slowed the rate of increased kidney
volume (decreased cell formation & volume (decreased cell formation &
proliferation), but did not alter the rate of decline
in eGFR.
CRITIQUE: more than half of the volunteers had
Stage 3-4 CKD (point of no return). Also
CRISP data explanation. In PKD the GFR p
decline is a late phenomenon and may be
difficult to obtain therapeutic benefit with mTOR
inhibitors at a late stage.
Walz et al NEJM, June 26, 2010
Copyright Harvard Medical School, 2010. All Rights Reserved.
681
Sirolimus
Sirolimus had no effect on changes in kidney
d t l GFR A ti t d and cyst volume or eGFR. A negative study
except for showing side effects of sirolimus
(apthous stomatitis).
CRITIQUE: Suboptimal dose of sirolimus (as
d t i l t di ) h compared to animal studies) may have
contributed to lack of effect.
Serra et al, NEJM June 26, 2010
Somatastatin in ADPKD
Octreotide: no change in kidney cyst
volume after 1 yr. of Rx. In placebo group
there was an ~8% increase in volume,
with improved perception of bodily pain
and physical activity.
Lanreotide: 3-5% decrease in liver cyst Lanreotide: 3 5% decrease in liver cyst
volume, with improved health perception.
Copyright Harvard Medical School, 2010. All Rights Reserved.
682
Future Therapeutic Drugs
Act at site of primary cilium to re-establish calcium influx thru PC2 channel
directly or target downstream mediation
Or
Associated protein ssocated p ote
Cell Cycle Inhibition
TNF inhibition
(Etanercept)
Cell cycle related
Kinase Nek 8
100
Potential Therapies
HALT PKD
Study A
Tolvaptan
Somatostatin
water
Roscovitine,
Sirolimus,
Everolimus,
Triptolide
HALT PKD
Study B
20
40
60
80
n
a
l

f
u
n
c
t
i
o
n

(
%
)
0
20
0 10 20 30 40 50 60
R
e
n
Age (years)
Torres Mayo <aupCP1047707-9
Copyright Harvard Medical School, 2010. All Rights Reserved.
683
Disclosures
None
Copyright Harvard Medical School, 2010. All Rights Reserved.
684
Updat e on
I gA Nephr opat hy
i 2010 i n 2010
GERALD APPEL, MD
Professor of Clinical Medicine Professor of Clinical Medicine
Columbia University College of
Physicians and Surgeons
NY-Presbyterian Hospital
New York, New York
Financial Disclosures Financial Disclosures
Dr. Appel has research grants, consultanships and
served on speakers bureaus, adjudication
committees and scientific advisory boards y
of the following companies: Merck, Pfizer, Bristol-
Myers Squibb, Takeda, Roche, Aspreva,
Genentech, Amgen, OrthoBiotech and QuestCor.
Copyright Harvard Medical School, 2010. All Rights Reserved.
685
I gA Nephr opat hy
Most common idiopathic GN world
Defined by IgA deposition in glomerular
mesangium
Presents- Young gross hematuria
Adults Proteinuria + hematuria
Not benign hematuria ( Bergers Dis )
ESRD in 15-20% by 10 yrs from onset and 30- ESRD in 15-20% by 10 yrs from onset and 30-
40 % by 20 yrs.
Risk Factors for Progression.
Rx Not one therapy fits all.
IgA Nephropathy IgA Nephropathy
IgA
Copyright Harvard Medical School, 2010. All Rights Reserved.
686
Pathogenesis of IgAN
Abnormal immune response to mucosal
d i d i t l ti derived environmental antigens.
Stimulation of IgA1 secreting B cells in
marrow and/or tonsils.
Defective B1-3 galactosyl-transferase activity g y y
in B cells with excessive synthesis of
galactose-deficient IgA1
Pathogenesis of IgAN
Auto-Antibody response to GD-IgA1 and Auto Antibody response to GD IgA1 and
formation of immune complexes or self
aggregating IgA ( with impaired clearance )
Deposition of GD-IgA1 in mesangium - as immune
complexes or self-aggregated GD-IgA ).
M i l C ll lif ti d t i j Mesangial Cell proliferation, podocyte injury,
capillary hypertension, altered permselectivity
Glomerulosclerosis/Tubulointerstitial fibrosis.
Copyright Harvard Medical School, 2010. All Rights Reserved.
687
IgAN Pts have Increased Serum Galactose Deficient
IgA1 Levels
Z. Moldoveanu et al Kidney Int 71: 1148-1154, 2007
Copyright Harvard Medical School, 2010. All Rights Reserved.
688
IgAN: GFR Prediction
(-ml/min/year)
14
Prediction of Progression in IgAN in 298 Pts
4
6
8
10
12
97mmHg
102mmHg
107mmHg
112mmHg
0
2
4
0.2g/d 0.5g/d 1g/d 2g/d 4g/d 6g/d
g
Bartosik, et al AJKD 38:728-735, 2001
Copyright Harvard Medical School, 2010. All Rights Reserved.
689
IgA Nephropathy:
Relative Risk of 2xScr after 5.6y
(Nishitani, et al KI 68:1078, 2005)
6
2
3
4
5
6
Male Sex
Age >60y
Scr
Upro
0
1
2
Clinical or Pathologic Features
p
HTN
IgA Nephropathy IgA Nephropathy
IgA
Copyright Harvard Medical School, 2010. All Rights Reserved.
690
IgA nephropathy is
morphologically heterogeneous
(Roberts, et al ASN CNC. 2008)
MEST-Oxford Classification System
Mesangial Hypercellularity
0= <50%; 1=>50% glomeruli involved g
Endocapillary proliferation
0= Absent; 1= Present
Segmental glomerulosclerosis
0= Absent; 1=Present
Tubulo-Interstitial Fibrosis
0= <25%; 1= 25-50%; 2= >50%
Preliminary Studies Show Good Correlation with Outcome
Copyright Harvard Medical School, 2010. All Rights Reserved.
691
Difficulties in Treatment Studies in
IgAN
Slow progression in many requires use Slow progression in many requires use
of surrogate markers of progression
Variable rate of progression
Heterogeneous population- phenotype
Only a few RTC to define outcome of RX -
Recent meta analysis many of low y y
quality and poorly reported
Everyone knows how to treat some of the
pts Nobody is certain how to treat others
IgA Nephropathy:Point of No Return
Point of no Return is the level of
renal function where no form of specfic p
treatment will improve renal function or
forestall eventual progression to ESRD
Values vary between a Scr of 2.0 to
3.0mg/dL (eGFR=30-35ml/min-Stage 4
CKD) CKD)
DAmico, 2001; Komatsu et al J Nephrol 18:690, 2005
Copyright Harvard Medical School, 2010. All Rights Reserved.
692
Therapy of IgA Nephropathy
ACE inhibitors, ARBs, Combinations
Tonsillectomy
Glucocorticoids (QD,QOD,Cyclic pulse)
Fish Oils ( n-3 PUFA )
Immunosuppressives
Azathioprine + steroids Azathioprine + steroids
Cyclophosphamide + steroids
Mycophenolate mofetil
ACE Inhibitors in IgA
Nephropathy:
A Controlled Trial
A RCT comparing ACEi or ARB to non-ACEi A RCT comparing ACEi or ARB to non ACEi
therapy in IgA N
SCr<1.2mg/dl UpV>500mg/d
Follow-up=7536mo.
A 50% increase in SCr from baseline:
3/23 (13%) in ACEi or ARB
12/21 (57%) in PBO
(Praga M, et al. JASN 14:1578, 2003)
Copyright Harvard Medical School, 2010. All Rights Reserved.
693
Survival without the combined end point of 30%
reduction of baseline CrCl and/or increase in
proteinuria up to >3.5 g/d/ 1.73 m2
Coppo, R. et al. J Am Soc Nephrol 2007;18:1880-1888
Effects of Dietary Sodium and HCTZ on
the Antiproteinuric Efficacy of Losartan
Vogt et al J ASN 19:999-1007,2008
Copyright Harvard Medical School, 2010. All Rights Reserved.
694
RAAS Inhibition in IgA Nephropathy
The goal of RAAS inhibition therapy
should be to reduce proteinuria to the
lowest value possible (preferably a
Up/Ucr <0.2g/g) without quality of life
interfering symptoms.
Blood pressure should be maintained
at 110-120mm Hg Systolic at 110-120mm Hg Systolic
Diuretics + Salt restriction should be
used to augment the anti-proteinuric
action of A-II inhibition
Copyright Harvard Medical School, 2010. All Rights Reserved.
695
The efficacy of tonsillectomy on
long-term survival in pts with IgAN
118 IgAN Bxed 1973-1980
48 / T il d 70 / T il f ll 192 48 s/p Tonsilx and 70 w/oTonsilx follow 192 mo.
No dif in age, gender, Uprot, Screat, SIgA, BP,
histology, Rx.
Renal survival 90% w Tonsx vs. 64% w/o Tonsx at
240 mo. By MVA tonsilx significant effect on
outcome.
Tonsillectomy has a favorable effect on long-term
o tcome IF performed earl in the co rse outcome IF performed early in the course.
Xie Y, Nishi S, Ueno M et al. Kidney Int 63:1861-1867,
2003.
Copyright Harvard Medical School, 2010. All Rights Reserved.
696
Copyright Harvard Medical School, 2010. All Rights Reserved.
697
IgA Nephropathy:Fish Oils (Omacor)
(Donadio J , et al. Semin Nephrol, 2004)
90%
30%
40%
50%
60%
70%
80%
Fi sh Oi l
Pl acebo
0%
10%
20%
Survi val Free of
ESRD at 8 yrs
Survi val Free of
2XScr at 8 yrs
Multicenter Controlled Trial of QOD
Pred.vs QD Omega 3 FA vs PBO in IgAN
99 IgAN <40yo GFR > 50ml/min Up/Ucr >0.5
33 Pts Randomized to Pred 60mg/m2 QOD x3m with g
taper x 2yr
32 Pts OM-3 FA 4g/d ( 1.88g EPA, 1.48 g DHA ) x 2 yrs
31 Pts PBO x 2 yrs
Primary end-point GFR < 60% baseline;l HBP Rx ACEi
Neither Rx group showed a benefit over PBO g p
14 Rx failures 2 Pred, 8 FO, 4 PBO
Major factor associated w RF was higher baseline
Up/Ucr
Hogg RJ, LeeJ, Nardelli NA, et al. Clin JASN 2006
Copyright Harvard Medical School, 2010. All Rights Reserved.
698
Therapy of IgA Nephropathy
ACE inhibitors, ARBs, Combinations
Tonsillectomy
Fish Oils ( n-3 PUFA )
Glucocorticoids ( QD,QOD,Cyclic
pulse )
Immunosuppressives Immunosuppressives
Azathioprine + steroids
Cyclophosphamide + steroids
Mycophenolate mofetil
IgAN: Controlled Trial of Steroids
Pozzi et al. JASN 15:157-163, 2004
Copyright Harvard Medical School, 2010. All Rights Reserved.
699
N = 63
Steroids plus ACEi versus ACEi alone in IgA
Nephropathy
A Prospective Randomized Controlled Trial
N 63
18 to 65 years old
Biopsy-proven IgAN within a one year period
Urine protein excretion of 1-5g/d
Estimated (eGFR) >30ml/min/1.73m
2
according to a
Modified MDRD equation for a Chinese population Modified MDRD equation for a Chinese population.
Jicheng L, Haiyan Wang. AJKD 2008
Treated with Cilazapril or Combination of cilazapril +
prednisone: 0.8-1.0 mg/Kg/day X 8 weeks tapered by
5-10mg every two weeks
Steroids plus ACEi versus ACEi alone in IgA
Nephropathy
A Prospective Randomized Controlled Trial
A B
Jicheng L, Haiyan Wang. AJKD 2008
Copyright Harvard Medical School, 2010. All Rights Reserved.
700
PROSPECTIVE RANDOMISED CONTROLLED TRIAL
OF STEROIDS PLUS RAMIPRIL IN
PROTEINURIC IgA NEPHROPATHY
n = 97
Proteinuria > 1g/24h - GFR > 50 ml/min Proteinuria 1g/24h GFR 50 ml/min
ALL PATIENTS
Ramipril
dose titrated to achieve
BP < 120/80 Proteinuria < 1g/24h
RANDOMISATION
Prednisolone 1 mg/kg/d for 2 months
Manno C et al. NDT 2009 Epub 23 July
g g
tapered by
0.2 mg/kg per month
PROSPECTIVE RANDOMISED CONTROLLED TRIAL
OF STEROIDS PLUS RAMIPRIL IN
PROTEINURIC IgA NEPHROPATHY
n = 97
Proteinuria > 1g/24h - GFR > 50 ml/min Proteinuria 1g/24h GFR 50 ml/min
STEROIDS CONTROL
1/48 ESRD 8/49
RAMIPRIL mean dose 4.5 mg/day
Manno C et al. NDT 2009 Epub 23 July
98% 8 year renal survival 70% P = 0.006
Copyright Harvard Medical School, 2010. All Rights Reserved.
701
Steroids and Cytotoxic Agents in
Progressive IgA Nephropathy
Ballardie, F & Roberts, I. JASN 13:142, 2002
Therapy of IgA Nephropathy
ACE inhibitors, ARBs, Combinations , ,
Tonsillectomy
Glucocorticoids (QD,QOD,Cyclic pulse)
Fish Oils ( n-3 PUFA )
Azathioprine + steroids Azathioprine steroids
Cyclophosphamide + steroids
Mycophenolate mofetil
Copyright Harvard Medical School, 2010. All Rights Reserved.
702
ControlledTrial of MMF in IGA
Nephropathy
33 pts - Pcreat 1.4 mg/dl UV prot 1.6 g/d
LowNa+ ACEi Low Na+, ACEi
MMF 2g/d vs. placebo x 2 yrs
MMF Placebo
Pcreat 1.48 - 1.71 1.40 1.53
UV t 1 79 1 80 1 30 0 75 UVprot 1.79 1.80 1.30 0.75
In IgA Nephrop. At mod risk no advantage to MMF
Maes BD et al. KI 65:1842-1849, 2004
MMF in IgA GN: A Controlled Trial
Maes R, et al KI 65:1842-9, 2004
2
0 6
0.8
1
1.2
1.4
1.6
1.8
2
MMF
Control
0
0.2
0.4
0.6
UpV Initial UpV 3 yr Scr initial Scr 3 yr
Copyright Harvard Medical School, 2010. All Rights Reserved.
703
MMF in IgA N: A Controlled Trial
(Tang et al, KI 68:802, 2005)
60%
70%
20%
30%
40%
50%
60%
MMF- % CR+PR
Control % CR+PR
0%
10%
20%
2 months 6 months 12 months
Mycophenolate Mofetil in IgA N:
Controlled Trials
Maes (n=34) Tang (n=40) Maes (n 34) Tang (n 40)
MMF- 2.0g/d x 3yr 1.5-2.0g/d x 6mo
eGFR- 71ml/min (Cin) 72ml/min (Ccr)
Scr- 1.43mg/dl 1.59mg/dl
UpV- 1.6g/d 1.8gm/d
SBP- 128mmHg 121mmHg
ACEi/ARB All All ACEi/ARB- All All
Histology- II-III (Churg/Sobin) 3.0 (Haas)
Ethnicity- N. European Oriental
Copyright Harvard Medical School, 2010. All Rights Reserved.
704
Mycophenolate Mofetil in IgA N:
A Controlled Trial
(Frisch G, et al NDT, 2005)
A randomized controlled trial of A randomized, controlled trial of
MMF 2.0gm/d X one year or placebo
in 40 patients* with severe, high-
risk IgA N.
Average Scr at entry= 2.4mg/dl UpV
> 1.0gm/d;
All received A-II inhibition
Follow-up 2 years
Mycophenolate Mofetil in IgA N: A
Controlled Trial
(Frisch GAppel GB et al NDT 2005)
15.00%
20.00%
25.00%
30.00%
MMF
Control
0.00%
5.00%
10.00%
50% increase i n Scr 50% decrease i n UpV
Control
Copyright Harvard Medical School, 2010. All Rights Reserved.
705
MMF in IgA Nephropathy
MMF might be more effective if used g
before a point of no return (Scr <2.0-
3.0mg/dl)
Dosage and duration of MMF required for
a beneficial effect on outcome is uncertain
Are effects of MMF additive to AII
inhibition and/or Fish Oils?
Ethnic/Racial associated differences in
pharmacokinetics/pharmacodynamics of
MMF may be important in response
Appels Therapy for IgAN in
2010
All pts ACEi or ARB or ACEi/ARB. All pts ACEi or ARB or ACEi/ARB.
All pts strongly consider Rx w statin.
All pts consider low protein diet.
All pts BP <130/80.
Tonsillectomy for pts with frequent bad
URI d t illiti URI and tonsillitis.
Fish Oils for those who want them
Should not replace other therapies.
Copyright Harvard Medical School, 2010. All Rights Reserved.
706
Appels Therapy for IgAN in 2010
Mild Disease- ( nl GFR, < 0.5g Uprot/d, good Bx)
No other Rx. Must have follow to check for
increase in Uprot. And Pcreat.
Moderate or SevereDis. ( Abnl GFR, or > 1gUprot/d, or
Bx w signif activity or risk of progression ,
Crescentic GN)
- Steroids x 6mo
- Consider Cyt +Stds or MMF if other therapy not - Consider Cyt +Stds or MMF if other therapy not
acceptable
-High Pcreat. w Bx chronic damage GS-TIF no
immunosuppressives
Financial Disclosures Financial Disclosures
Dr. Appel has research grants, consultanships and
served on speakers bureaus, adjudication
committees and scientific advisory boards y
of the following companies: Merck, Pfizer, Bristol-
Myers Squibb, Takeda, Roche, Aspreva,
Genentech, Amgen, OrthoBiotech and QuestCor.
Copyright Harvard Medical School, 2010. All Rights Reserved.
707
Amyloidosis
and
Light Chain Deposition Disease
Laura M. Dember, M.D.
Associate Professor of Medicine
Renal Section and Amyloid Treatment and
Research Program
Boston University School of Medicine
Disclosures
Research Support:
--Neurochem, Inc.
--FDA
--Boston University Amyloid Program
--NIDDK
Copyright Harvard Medical School, 2010. All Rights Reserved.
708
Amyloidosis
Group of diseases in which proteins that
are normally soluble deposit extracellularly
in tissues as insoluble fibrils with a
characteristic biochemical structure
Amyloidogenic Proteins Differ
Functionally and Structurally
Transthyretin Lysozyme Apolipoprotein A-I IgG Kappa
Beta 2M A-beta
Copyright Harvard Medical School, 2010. All Rights Reserved.
709
But Resulting Amyloid is Morphologically
Indistinguishable
Classification of the
Amyloidoses
Amyloidogenic precursor protein
Distribution of amyloid deposits
systemic
localized
Copyright Harvard Medical School, 2010. All Rights Reserved.
710
Systemic Amyloidosi s
Precursor Protein
AL (Primary) Ig Light Chain
AA (Secondary) Serum AA (SAA)
Hereditary
TTR, lysozyme,
ApoA1, ApoA2,
fibrinogen, gelsolin
Senile Systemic TTR
Dialysis-Related 2 microglobulin


Localized Forms
Precursor Protein
Localized AL Ig Light Chain
Alzheimers disease A
Creutzfeldt-J akob APrP
Type II DM Amylin

Copyright Harvard Medical School, 2010. All Rights Reserved.
711
Clinical Presentation and
Diagnosis
Patient 1
58 yo man with new lower extremity edema
On exam, hepatomegaly, orthostatic hypotension
Urinary protein excretion 7.5 g/day, creatinine 1.7 mg/dl,
albumin 3.1 g/dl, cholesterol 340 mg/dl
Alkaline phosphatase 380 U/L
Echocardiography: diastolic dysfunction, wall thickening
Kidney biopsy: Light microscopy - mesangial expansion
with early nodule formation, nodules very weakly PAS-
positive and stain orange-pink with Congo red dye
Copyright Harvard Medical School, 2010. All Rights Reserved.
712
Whats the Disease?
Does he have amyloidosis?
If so, what type?
Whats the Disease?
Does he have amyloidosis?
Copyright Harvard Medical School, 2010. All Rights Reserved.
713
Whats the Disease?
Does he have amyloidosis?
This is not enough
Whats the Disease?
Does he have amyloidosis?
Need polarized microscopy result to call this
Congo red-positive
This is not enough This clinches the diagnosis
Copyright Harvard Medical School, 2010. All Rights Reserved.
714
Whats the Disease?
Does he have amyloidosis? YES
If so, what type?
Can clinical presentation help?
Can laboratory studies help?
Can kidney biopsy help?
Clinical Manifestations
AL Amyloidosis
Can involve any organ/tissue except brain
Kidney and heart most common
Liver, ANS, PNS, GI tract, soft tissue, thyroid,
adrenal glands
Macroglossia is highly specific for AL disease
Copyright Harvard Medical School, 2010. All Rights Reserved.
715
Clinical Manifestations
AA Amyloidosis
Occurs in setting of longstanding inflammation
(RA, IBD, FMF, osteomyelitis, bronchiectasis)
Most patients have kidney involvement
Liver, autonomic nervous system, GI tract
involvement can occur
Thyroid involvement is more common than in
other types of amyloidosis
Heart involvement can occur but is unusual
Clinical Manifestations
Hereditary Amyloidoses
Amyl oidogenic Protein Organ Invol vement
Transthyretin Peripheral nervous system, heart, vitreous
opacities. Kidney involvement is not typical.
Fibrinogen A chain Kidney, liver, spleen. Hypertension is common.
Kidney involvement is predominantly glomerular.
Apolipoprotein AI Kidney (predominant medullary deposition), liver,
heart, skin, larynx.
Apolipoprotein AII Kidney
Lysozyme Kidney, liver, GI tract, spleen, lymph nodes, lung,
thyroid, salivary glands
Gelsolin Cranial nerves, lattice corneal dystrophy
Cystatin C Cerebral vessels

Copyright Harvard Medical School, 2010. All Rights Reserved.
716
Back to the Patient
58 yo man with new lower extremity edema
Hepatomegaly, orthostatic hypotension
Urinary protein excretion 7.5 g/day,
serum creatinine 1.7 mg/dl, albumin 3.1 g/dl
Alkaline phosphatase 380 U/L
Echocardiography wall thickening, diastolic
dysfunction
Back to the Patient
58 yo man with new lower extremity edema
Hepatomegaly, orthostatic hypotension
Urinary protein excretion 7.5 g/day,
serum creatinine 1.7 mg/dl, albumin 3.1 g/dl
Alkaline phosphatase 380 U/L
Echocardiography wall thickening, diastolic
dysfunction
Clinical manifestations suggest
AL disease but not sufficient to make
diagnosis
Copyright Harvard Medical School, 2010. All Rights Reserved.
717
Can Laboratory Studies Help
Determine the Type of Amyloid?
AL Disease: plasma cell disorder is usually
evident by:
Serum IFE
Urine IFE
Quantitative free light chain assay
Bone marrow biopsy
SPEP and UPEP are not sensitive enough!
AA Disease: No laboratory marker
Hereditary forms: Isoelectric focusing, DNA
sequencing
Dialysis-Related: No laboratory marker
Back to the Patient
SIFE: monoclonal IgG lambda protein
UIFE: monoclonal lambda light chain
evident by UIFE
Serum free light chain kappa / lambda
ratio 0.08 (normal: 0.26-1.65)
Bone marrow biopsy 6% plasma cells with
lambda predominance
Copyright Harvard Medical School, 2010. All Rights Reserved.
718
Does Kidney Biopsy Help Determine
the Type of Amyloid?
Determining Type of Amyloid from Kidney
Biopsy
Amyloidogenic protein can be identified by
immunofluorescence or immunohistochemistry in some,
but not all, cases.
Epitopes recognized by commercially available reagents
may be lost.
Reagents are not available for several of the hereditary
forms.
Lambda LC Kappa LC
Copyright Harvard Medical School, 2010. All Rights Reserved.
719
Does Electron Microscopy Help?
Amyloid fibrils
8 - 12 nm diameter
non-branching
random orientation
Fibrillary GN fibrils
15 - 20 nm diameter
non-branching
random orientation
Immunotactoid GN fibrils
30 - 60 nm diameter
non-branching
ordered orientation
Immunotactoid GN
Abdominal Fat Aspirate
How sensitive is abdominal fat Congo red
staining?
AL: sensitivity 80-90%
AA: sensitivity 65-75%
Hereditary: probably <65%
How specific is Congo red staining (of any
tissue)?
Operator-dependent
Over-staining is common with inexperienced labs
Copyright Harvard Medical School, 2010. All Rights Reserved.
720
Our Patient has AL Disease
Amyloid present in kidney
Plasma cell dyscrasia evident by
hematologic studies
Single light chain isotype (lambda) present
in kidney
Treatment of Amyloidosis
Copyright Harvard Medical School, 2010. All Rights Reserved.
721
Disorder of Protein Misfolding
Soluble
Precursor
Unstable
Variant
Mutation
Proteolytic event
Environmental factors
Excessive concentration
Disorder of Protein Misfolding
Precursor
Unstable
Fragment
Folding
Intermediate
Self-Aggregation
Amyloid Fibril Degradation
Soluble
Precursor
Unstable
Variant
Folding
Intermediate
Self-Aggregation
Amyloid Fibril Degradation
Mutation
Proteolytic event
Environmental factors
Excessive concentration
Copyright Harvard Medical School, 2010. All Rights Reserved.
722
Treatment Targets
Precursor Protein Production
Fibril Formation
Tissue Deposition
Degradation
Treatment of AL Amyloidosi s
Copyright Harvard Medical School, 2010. All Rights Reserved.
723
AL Disease: Survival from
Diagnosis in Untreated Patients
Overall 13 mos
Cardiac 6 mos
Renal 21 mos
Peripheral Neuropathy 26 mos
Treatment Targets
Precursor Protein Production
Fibril Formation
Tissue Deposition
Degradation
Copyright Harvard Medical School, 2010. All Rights Reserved.
724
Reducing Precursor Protein in AL Disease:
Oral Melphalan and Prednisone
1. Skinner et al, Am J Med 1996 Median Survival
--Melphalan/Prednisone/Colchicine 12 months
--Colchicine 7 months
2. Kyle et al, NEJ M 1999 Median Survival
--Melphalan/Prednisone 18 months
--Melphalan/Prednisone/Colchicine 17 months
--Colchicine 8 months
High-Dose Melphalan with Autologous
Stem Cell Transplantation
G-CSF
Stem Cell
Collection
I.V. Melphalan Stem Cell Infusion
Copyright Harvard Medical School, 2010. All Rights Reserved.
725
BU Experience with HDM/SCT
1994-2008
497 Patients
Sanchorawalaet al, ASH 2008
Compl ete Response 40%
Med. Survi val 73 months S
u
r
v
i
v
a
l
Years
5-Year Survi val 55%
Skinner et al, Ann Intern Med 140:85-93, 2004
Non-Complete Response
N=108
Median 5.2 yrs
Impact of Hematologic Response
on Survival
Complete Response
N=73
Median >5 yrs
Yes 73 73 61 49 35 24 16 6 2
No 108 108 77 50 32 17 4 1 0
Copyright Harvard Medical School, 2010. All Rights Reserved.
726
Hematologic Response is Durable
Relapse rate is low (8%)
Longest follow-up is 16 years
Proteinuria Improves with Achievement
of Hematologic CR
0
25 25
50
75
100
0
Complete
Remission
Persistent
Disease
Complete
Remission
Persistent
Disease
Urine Protein Creatinine Clearance
2
4
6
8
10
g

/

2
4

h
r
m
l

/

m
i
n
Dember et al, Ann Intern Med 134:746-53, 2001
Baseline
12 Months
Copyright Harvard Medical School, 2010. All Rights Reserved.
727
Treatment Toxicities
Arrhythmias
Heart Failure
Sepsis
Gastrointestinal bleeding
Splenic Rupture
Mucositis
Acute Kidney Injury
Toxicity
Treatment-Related Mortality at BU:
1994-2002 14%
2000-2005 9%
2004-2008 <5%
Copyright Harvard Medical School, 2010. All Rights Reserved.
728
Toxicity
Treatment-Related Mortality at BU:
1994-2002 14%
2000-2005 9%
2004-2008 <5%
Tolerability Improves with Experience
Skinner et al, Ann Intern Med 140:85-93, 2004
N=173
Median=6.8 yrs
N=136
Median=1.59 yrs
Impact of Cardiac Involvement
on Survival
Copyright Harvard Medical School, 2010. All Rights Reserved.
729
BU Eligibility Criteria for SCT
Performance status <3 by SWOG criteria
Left ventricular ejection fraction >40%
Room air oxygen saturation >95%
Supine systolic blood pressure greater
than 90 mm Hg
Criteria are evolving with accumulating
experience
BU Eligibility Criteria for SCT
Performance status <3 by SWOG criteria
Left ventricular ejection fraction >40%
Room air oxygen saturation >95%
Supine systolic blood pressure greater
than 90 mm Hg
Criteria are evolving with accumulating
experience AND with emergence of
effective alternatives
Copyright Harvard Medical School, 2010. All Rights Reserved.
730
Summary: Autologous Stem Cell
Transplant for AL Amyloidosis
Can produce a complete hematologic remission
in a substantial proportion of patients.
Complete hematologic remission is associated
with prolonged survival and with improvement in
organ function.
The hematologic response appears to be
sustained.
Treatment toxicity is prohibitive for many
patients.
Additional Approaches
Melphalan / dexamethasone
Modified-dose IV melphalanwith SCT
Tandem HDM/SCT
Thalidomide / dexamethasone
Lenalidomide / dexamethasone
Bortezomib / dexamethasone
Copyright Harvard Medical School, 2010. All Rights Reserved.
731
Melphalan / Dexamethasone?
80 of 100 Randomized Patients
Underwent Treatment
Copyright Harvard Medical School, 2010. All Rights Reserved.
732
Better Survival in Mel/Dex Group
AA Amyloidosi s
Copyright Harvard Medical School, 2010. All Rights Reserved.
733
AA Amyloidosi s Treatment
Anti-inflammatory or immunosuppressive
therapy to suppress precursor protein production
(SAA)
For FMF, colchicine can prevent disease
Eprodisate targets amyloid formation and
deposition
Small molecule developed to interfere with interactions between
AA protein and GAGs and thereby reduce amyloid fibril
formation and tissue deposition
Appears to reduce rate of decline of kidney function (NEJ M
2007)
Not approved, not available
Orthotopic Liver Transplantation
for TTR Amyloidosis
Used since 1991 and now considered the
definitive treatment
Mutant form of TTR disappears from
circulation
Issues:
optimal timing difficult to determine
wild-type TTR may deposit as amyloid at
sites with pre-existing amyloid.
Copyright Harvard Medical School, 2010. All Rights Reserved.
734
Strategy: Stabilize the Native
Conformation
Sacchettini J C & Kelly J W, Nat Rev Drug Discov 2002
NSAIDs Bind to TTR Tetramer and
Inhibit Dissociation into Monomers
Multi-center trial using diflunisal as tetramer
stabilizer is underway.
Could prevent development or progression
of disease.
Stabilization of wild-type TTR could be of
value after liver transplantation.
This strategy of conformation stabilizers
may be applicable to other amyloidogenic
precursor proteins.
Copyright Harvard Medical School, 2010. All Rights Reserved.
735
Dialysis-Related Amyloidosis
Kidney transplantation is best treatment
Symptoms improve even with persistence of
deposits
Hemofiltration?
Adsorption membranes?
Super-flux membranes?
Light Chain Deposition
Disease
Copyright Harvard Medical School, 2010. All Rights Reserved.
736
Monoclonal Immunoglobulin-Associated
Kidney Diseases
AL amyloidosis
Light (or heavy) chain deposition disease
Cast nephropathy
Waldenstroms macroglobulinemic GN
Immunotactoid glomerulopathy (?)
Type 1 cryoglobulinemia
Proliferative GN with monoclonal IgG deposits
Patient 2
58 yo man with new lower extremity edema
Hepatomegaly, orthostatic hypotension
Urinary protein excretion 7.5 g/day,
serum creatinine1.7 mg/dl, albumin 3.1 g/dl, cholesterol
340 mg/dl
Alkaline phosphatase 380 U/L
Echocardiography: diastolic dysfunction, wall thickening
Kidney biopsy: Light microscopy: mesangial expansion
with nodule formation; nodules very weakly strongly
PAS-positive; no staining with Congo red dye; marked
thickening of tubular basement membranes
Copyright Harvard Medical School, 2010. All Rights Reserved.
737
Patient 2
Thickened basement
membranes
Strongly PAS-positive
nodules
Linear kappa staining of
tubular basement
membranes and periphery
of mesangial nodules
Continuous glomerular
subendothelial electron
densities
Electron densities are
granular (not fibrillar)
RoncoP et al. ClinJ Am Soc Nephrol 1: 1342, 2006
Patient 2
Faint IgG kappa band by serum IFE
Urine IFE unremarkable
Serum free light chain kappa / lambda
ratio 287 (normal 0.26-1.65)
Bone marrow biopsy 4% plasma cells;
kappa clonalityby flow cytometry
Copyright Harvard Medical School, 2010. All Rights Reserved.
738
AL Amyloidosis vs LCDD
Both can occur with or without multiple myeloma
Concentration of light chain in serum or urine
can be very low and barely detectable
(sometimes undetectable)
Kappa or lambda light chain, but lambda more
common in AL amyloidosis and kappa more
common in LCDD
Kidney manifestations similar: nephrotic
syndrome, progressive loss of GFR
Extra-renal manifestations can occur in both
(more common in amyloidosis)
Treatment of LCDD
Until recently, if myeloma was not present
LCDD was considered a form of MGUS
Now recognized that the light chains are
pathogenic, kidney failure is typical, and
extra-renal manifestations occur
LCDD starting to be approached in same
way as AL amyloidosis (chemotherapy)
Copyright Harvard Medical School, 2010. All Rights Reserved.
739
High-Dose Melphalan with
Autologous SCT for LCDD
Royer et al, KI 2004 11 patients
Weichman et al, BMT 2006 6 patients
Lorenz et al, NDT 2008 6 patients
Question 1
A 65 year old man with recently diagnosed multiple
myeloma is found to have serum creatinine of 2 mg/dl. 24
hour protein excretion is 3.2 g per day. Urinalysis shows
3+protein, no blood, and no cells or casts in sediment.
Which of the following would you expect to find on kidney
biopsy?
A. Congo red-positivity, non-branching fibrils 10 nm in
diameter by EM
B. Nodular glomerulosclerosis, strong kappa staining of
glomeruli and tubular basement membranes by IF, and
non-fibrillary electron dense deposits by EM
C. Intra-tubular PAS-negative casts and peritubular
interstitial inflammation
D. A or B
Copyright Harvard Medical School, 2010. All Rights Reserved.
740
Question 1
A 65 year old man with recently diagnosed multiple
myeloma is found to have serum creatinineof 2 mg/dl. 24
hour protein excretion is 3.2 g per day. Urinalysis shows
3+protein, no blood, and no cells or casts in sediment.
Which of the following would you expect to find on kidney
biopsy?
A. Congo red-positivity, non-branching fibrils 10 nm in
diameter by EM
B. Nodular glomerulosclerosis, strong kappa staining of
glomeruli and tubular basement membranes by IF, and
non-fibrillaryelectron dense deposits by EM
C. Intra-tubular PAS-negative casts and peritubular
interstitial inflammation
D. A or B
Question 2
A 40 year old woman with LCDD (without multiple
myeloma) develops end-stage renal disease 6 months after
her initial presentation. She begins peritoneal dialysis. She
tells you she would like to undergo kidney transplantation.
What do you tell her?
A. Because she is clinically well without extra-renal
disease it is reasonable to undergo transplantation.
B. She should undergo aggressive treatment of the
plasma cell disorder and pursue kidney transplantation
if there is remission of the plasma cell disorder.
C. She is not a candidate for kidney transplantation
because of the plasma cell disorder, and she is not a
candidate for aggressive treatment of the plasma cell
disorder because she is dialysis-dependent.
Copyright Harvard Medical School, 2010. All Rights Reserved.
741
Question 2
A 40 year old woman with LCDD (without multiple
myeloma) develops end-stage renal disease 6 months after
her initial presentation. She begins peritoneal dialysis. She
tells you she would like to undergo kidney transplantation.
What do you tell her?
A. Because she is clinically well without extra-renal
disease it is reasonable to undergo transplantation.
B. She should undergo aggressive treatment of the
plasma cell disorder and pursue kidney transplantation
if there is remission of the plasma cell disorder.
C. She is not a candidate for kidney transplantation
because of the plasma cell disorder, and she is not a
candidate for aggressive treatment of the plasma cell
disorder because she is dialysis-dependent.
Key References
1. Merlini G and Bellotti V. Molecular mechanisms of amyloidosis.
N Engl J Med 2003; 349:583-596
2. Falk RH, Comenzo RL and Skinner M. The systemic amyloidoses.
N Engl J Med 1997; 337:898-909
3. Dember LM. Amyloidosis-associated kidney disease. J Amer Soc
Nephrol 2006; 17:3458-71
4. Skinner M, Sanchorawala V, Seldin et al. Survival and clinical
response to treatment with high-dose melphalan and autologous
stem cell transplantation in paitents with AL amyloidosis: an 8-
year study. Ann Intern Med 2004; 140:85-93
5. Ronco P, Plaisier E, Mougenot B, Aucouturier P. Immunoglobulin
light (heavy)-chain deposition: from molecular medicine to
pathophysiology-driven therapy. Clin J Am Soc Nephrol 2006;
1:1342-50
Copyright Harvard Medical School, 2010. All Rights Reserved.
742
Disclosures
Research Support:
--Neurochem, Inc.
--FDA
--Boston University Amyloid Program
--NIDDK
Copyright Harvard Medical School, 2010. All Rights Reserved.
743
Pregnancy and
Renal Disease Renal Disease
Robert A. Cohen M.D.
Beth Israel Deaconess Medical Center
Harvard Medical School
Disclosures
None
Copyright Harvard Medical School, 2010. All Rights Reserved.
744
Case One
A 28 year-old G0,P0 consults you about risks
i l d i b i t Sh h I A involved in becoming pregnant. She has IgA
nephropathy diagnosed by renal biopsy at age
14. She complains of swelling in her ankles at
the end of the day. She has proteinuria;
creatinine has gradually increased. She is taking
lisinopril 20 mg daily but uses no NSAIDS Her lisinopril 20 mg daily but uses no NSAIDS. Her
blood pressure is 152/100 and she has 1+
pretibial edema.
Case One (cont.)
The urine dipstick reveals 3+ protein and 3+
heme; microscopically she has 5-12 RBC/HPF,
mostly acanthocytes.
BUN 31 mg/dl; creatinine 2.1 mg/dl; albumin 4.2
mg/dl, Hb 12.4
Urine protein/creatinine ratio 2.4
Copyright Harvard Medical School, 2010. All Rights Reserved.
745
Question #1:
Which statement(s) is(are) true?
A) IgA nephropathy with proteinuria portends > 50% risk of
l t d i f l f ti pregnancy-related worsening of renal function.
B) An increase in proteinuria and plasma creatinine is
expected in first half of pregnancy.
C) The risk of pregnancy-related decline in renal function
is < 10%.
D) The risk of pregnancy-related decline in renal function
may exceed 30% may exceed 30%.
E) A. and B.
F) B. and D.
G) None of the above.
Question #2:
Which statement(s) most accurately
describe(s) the risk to her pregnancy?
A) Sh i t i d i k f l i A) She is at increased risk for preeclampsia.
B) Fetal survival is reduced due to underlying
IgA nephropathy.
C) The fetus will be at increased risk for
prematurity and intrauterine growth restriction.
D) A and C D) A. and C.
E) B. and C.
F) A., B. and C.
Copyright Harvard Medical School, 2010. All Rights Reserved.
746
Fundamental Issues
What is the impact of pregnancy on renal
function in patients with underlying renal disease
or impaired renal function?
What is the impact of underlying renal disease or
impaired renal function on the fetus?
Data Are Limited
No randomized clinical trials, no meta-anaylses
Difficult to identify appropriate controls Difficult to identify appropriate controls
Primarily small series
Usually only one or a few collaborative centers
Absence of uniform data points; limited # data points
Variable, often limited, post-partum follow-up
Difficulty distinguishing whether acceleration of
renal insufficiency during or right after renal insufficiency during or right after
pregnancy is due to pregnancy or natural history
of renal disease.
Copyright Harvard Medical School, 2010. All Rights Reserved.
747
Impact of Pregnancy on Renal
Disease/Renal Insufficiency
D d tl it f d l i l di Depends mostly on severity of underlying renal disease
at onset of pregnancy
Depends in part on whether hypertension controlled
during pregnancy
Whether severe preeclampsia occurs
Less dependant on type of renal disease
Some small series suggest MPGN and FSGS important
Diabetes may excacerbate proteinuria and hypertension
Lupus nephritisactivity of disease affects pregnancy
Normal Renal Function at Baseline
(With or Without Renal Disease)
Creatinine tends to decrease first half of
pregnancy (due to increased GFR)
Baseline proteinuria (if present) tends to
increase
Copyright Harvard Medical School, 2010. All Rights Reserved.
748
Mild Renal Insufficiency
(Creatinine < 1.4 mg/dl)
Data from several small series with mild
renal insufficiency at baseline: Minimal to
no impact of pregnancy in terms of
acceleration renal insufficiency.
Moderate to Severe Renal
Insufficiency Creatinine 1.4 mg/dl
Largest Series: 82 pregnancies with creatinine 1 4 at Largest Series: 82 pregnancies with creatinine 1.4 at
conception (mean creatinine = 1.9)
Mean creatinine increased to 2.5 by 3
rd
trimester
20% had worse renal function during pregnancy
23% had worse renal function by 6 wks post-partum
8% recovered; 10% declined further
8 women (10%) ESRD within year of pregnancy
Creatinine >2.0 outset: Highest likelihood decline (33%
had accelerated decline renal function)
Jones DC, Hayslett JP. N Enl J Med 1996;335:225-32.
Copyright Harvard Medical School, 2010. All Rights Reserved.
749
Impact of Renal Disease on
Pregnancy Outcomes
Fetal Survival Good (70 to 100%) Fetal Survival Good (70 to 100%)
Lower with uncontrolled hypertension
Especially with MAB > 105 start of conception
Risk of Prematurity estimated 45 -70%
Risk of Intrauterine Growth Restriction estimated
20-50%
Risk of Preeclampsia increased Risk of Preeclampsia increased
Greater risk of occurrence during late 2nd trimester
Associated with both prematurity and intrauterine
growth restriction
Question #1:
Which statement(s) is(are) true?
A) IgA nephropathy with proteinuria portends > 50% risk of
pregnancy-related worsening of renal function. pregnancy related worsening of renal function.
B) An increase in proteinuria and plasma creatinine is
expected in first half of pregnancy.
C) The risk of pregnancy-related decline in renal function
is < 10%.
D) The risk of pregnancy-related decline in renal function
may exceed 30%. may exceed 30%.
E) A. and B.
F) B. and D.
G) None of the above.
Copyright Harvard Medical School, 2010. All Rights Reserved.
750
Question #2:
Which statement(s) most accurately
describe(s) the risk to her pregnancy?
A) Sh i t i d i k f l i A) She is at increased risk for preeclampsia.
B) Fetal survival is reduced due to underlying
IgA nephropathy.
C) The fetus will be at increased risk for
prematurity and intrauterine growth restriction.
D) A and C D) A. and C.
E) B. and C.
F) A., B. and C.
Case Two
A 32 year old G0P0 has diffuse proliferative lupus
nephritis diagnosed 19 months ago on renal biopsy. She nephritis diagnosed 19 months ago on renal biopsy. She
was treated with mycophenolate mofetil. Her labs at
diagnosis: creatinine 1.5 (0.7 previously); dsDNA titer
1:1280; C3 28 (90-180); C4 6 (10-40); urinalysis 4+
protein 3+ heme with many acanthocytes, wbcs and
mixed cellular casts; urine protein/creatine ratio 9.2.
She currently feels great. Exam is unremarkable. Labs:
creatinine 0 8 C3 65 and C4 9 normal dsDNA titer and creatinine 0.8, C3 65 and C4 9, normal dsDNA titer and
urine protein/creatinine ratio 0.6. Her urine sediment is
now unremarkable. Her anti-Ro and anti-La negative;
aPL negative. She inquires about becoming pregnant.
Copyright Harvard Medical School, 2010. All Rights Reserved.
751
Question # 1
Which statement is true?
A. Worsening proteinuria is an excellent indicator
of lupus relapse during pregnancy.
B. Her risk for fetal loss or preterm delivery is now
comparable to the general population
C. Azathioprine can be used to treat a lupus
relapse during pregnancy.
D. All of the above.
E. None of the above.
Lupus Nephritis and Pregnancy
Mayo Clinic Cohort: 90 pregnancies in 58
women with SLEretrospective analysis women with SLE retrospective analysis
47 SLE without LN at conception
23 active LN at conception
Urine prot/creat ratio 0.5 and/or active urine sediement
20 quiescent LN
Urine prot/creat ration < 0.5 and inactive sediment
Identified maternal and fetal complications
Maternal: preeclampsia, eclampsia, HELLP syndrome,
t k t l d th stroke, maternal death
Fetal: preterm births, fetal loss
Wagner SJ et al. Lupus 2009;18:342-347.
Copyright Harvard Medical School, 2010. All Rights Reserved.
752
Lupus Nephritis and Pregnancy: Mayo
Clinic Cohort
Results:
Maternal complications in active LN much greater Maternal complications in active LN much greater
compared to SLE without LN
57% versus 11% (P < 0.001)
Higher preterm births active LN vs. SLE no LN
52% versus 19% (P < 0.001)
Higher rate fetal loss active LN vs. SLE no LN
35% versus 9% (P < 0.03)
Maternal and fetal outcomes better for SLE without
LN compared to quiescent LN
Wagener SJ et al. Lupus 2009; 18:342-347
Lupus Nephritis and Pregnancy
Retrospective analysis: 118 pregnancies in 81
Italian women with LN Italian women with LN
Biopsy proven prior to conception
Hypocomplementemia at conception predicted poor
fetal outcome
Low-dose aspirin appears to be protective
Renal flare during or right after pregnancy predicted
by renal status at conception by renal status at conception
Significantly higher likelihood in active LN compared with
remission or partial remission
Imbasciati E et al Nephrol Dial Transplant 2009;24:519-525
Copyright Harvard Medical School, 2010. All Rights Reserved.
753
Active lupus nephritis associated with
increased fetal loss pre term delivery and
Lupus Nephritis and Pregnancy
increased fetal loss, pre-term delivery and
maternal complications
Risk even higher if antiphospholipid Ab
positive
Maternal risk: increased thrombosis and
preeclampsia
Fetal risk: early and late miscarriage, prematurity
Andrade R et al Clin Exp Rhem 2008:28;268-274
Lupus Nephritis in Pregnancy
Lupus nephritis flare or preeclampsia ?
H l t i d i d d DNA tit i l Hypocomplementemia and increased dsDNA titers in lupus
nephritis; thrombocytopenia in either
Active urine sediment and proteinuria in lupus nephritis
Proteinuria alone seen with preeclampsia but can be seen
with lupus nephrits
Treatment lupus nephritis
Glucocorticoids and azathioprine safe Glucocorticoids and azathioprine safe
Hydroxychloroquine also safe for extra-renal lupus
Cyclophosphamide and mycophenolate mofetil not safe
Copyright Harvard Medical School, 2010. All Rights Reserved.
754
Question # 1
Which statement is true?
A Worsening proteinuria is an excellent indicator A. Worsening proteinuria is an excellent indicator
of lupus relapse during pregnancy.
B. Her risk for fetal loss or preterm delivery are
now comparable to the general population
C. Azathioprine can be used to treat a lupus
relapse during pregnancy relapse during pregnancy..
D. All of the above.
E. None of the above.
Case Three
A 41 y.o. nulliparous woman who underwent in-vitro
fertility is now pregnant with twins. She has enjoyed fertility is now pregnant with twins. She has enjoyed
good health but is obese and has a 20-year smoking
history. The first half of pregnancy has been
uneventful. At 32 weeks she is hypertensive
(145/98) and has proteinuria (urine protein/creatinine
ratio 1.7). Five days later she complains of
persistent, severe headache and right upper
quadrant abdominal pain The blood pressure is quadrant abdominal pain. The blood pressure is
165/108. Platelet count is 70, creatinine 1.3, AST 94,
and ALT 92.
Copyright Harvard Medical School, 2010. All Rights Reserved.
755
Question #1:
Which statement(s) is(are) true?
A) Risk factors for preeclampsia include A) Risk factors for preeclampsia include
primagravida status, advanced maternal age,
mutiple gestation pregnancy, cigarette smoking,
and obesity.
B) Definitive treatment is delivery
C) Preeclampsia most likely would have been
averted with low-dose aspirin and calcium averted with low dose aspirin and calcium
supplementation during pregnancy.
D) A. and B.
E) B. and C.
Preeclampsia
Systemic disorder unique to pregnancy
marked by new onset hypertension and
proteinuria after 20 weeks gestation
Eclampsia: onset of seizures in
preeclampsia
Preeclampsia may be superimposed on Preeclampsia may be superimposed on
chronic hypertension in pregnancy
Incidence is 3-14% pregnancies worldwide
Copyright Harvard Medical School, 2010. All Rights Reserved.
756
Risk Factors
First pregnancy, prior preeclampsia, family
history preeclampsia, multiple gestation,
obesity, pre-gestational diabetes, pre-
existing renal disease, pre-existing
hypertension, advanced maternal age,
antiphospholipid antibody syndrome (aPL). p p p y y ( )
Cigarette smokers have lower risk preeclampsia v.
nonsmakers
Pathogenesis
Disorder of placental hypopoxia/ischemia
Due to impaired trophoblast invasion or
diff i i differentiation (impaired implantaton)
Leads to generalized endothelial dysfunction
Manifested by: hypertension, platelet activation, CNS changes,
seizures, glomerular endotheliosis, proteinuria, renal insufficiency,
hepatic ischemia/ necrosis, edema, hemolysis
Marked by increased placental secretion/expression
ti i i f t ( Flt d l bl d li ) antiangiogenic factors (sFlt and soluble endoglin)
Bind placental growth factor (PlGF) and VEGF
Rising circulating levels sFlt1 may predict preeclampsia
Levine et al. N Engl J Med 2004;350:672.
Copyright Harvard Medical School, 2010. All Rights Reserved.
757
Severe Preeclampsia
Defined as preeclampsia accompanied
by at least one of following: by at least one of following:
cns dysfunction; liver capsule distension;
hepatocellular injury (transaminases > twice
normal); marked blood pressure elevation;
thrombocytopenia, marked proteinuria ( 5 g/24
hrs); oliguria; severe fetal growth restriction; hrs); oliguria; severe fetal growth restriction;
pulmonary edema; stroke
ACOG Practice Bulletin #33, January 2002
Preeclampsia: General
Management Principles
Definitive teatment is delivery
Prevents maternal or fetal complications
Timing of deliverydepends on preeclampsia
severity, gestational age, maternal and fetal status
Corticosteroids should be given at 24-34 weeks gestation in
l i preeclampsia
Promotes fetal lung maturation
Severe preeclampsia: delivery indicated regardless of
gestational age
Copyright Harvard Medical School, 2010. All Rights Reserved.
758
Preeclampsia:
Seizure prophylaxis
Magnesium sulfate: drug of choice for Magnesium sulfate: drug of choice for
preventing eclampsia
Evidence suggests use reduces incidence of
eclampsia
But no effect on disease progression aside from seizure
prevention
Dose for renal insufficiencyy
Observe for toxicity (loss of deep tendon reflexes,
respiratory paralysis, cardiac arrest)
Calcium gluconate---antidote for toxicity
Preeclampsia: Hypertension
Treatment
Lack of data about level of blood pressure to treat Lack of data about level of blood pressure to treat
Does not alter the course of preeclampsia
Concerns about deleterious effects of blood pressure
lowering on fetal growth need to be weighed with
concern for possible maternal vascular complications
(especially stroke)
Usually treat systolic >150-160; diastolic > 100
Medications for acute blood pressure lowering: IV
labetalol, hydralazine (less commonly diazoxide)
Avoid sodium nitroprussidepossible fetal cyanide poisoning
Copyright Harvard Medical School, 2010. All Rights Reserved.
759
Mild Preeclampsia
Management
Frequent monitoring, including labs
Deliver by 36 to 38 weeks
unless severe preeclampsia or abruption
placentae, marked fetal growth restriction, non-
reassuring fetal testing, or oligohydramnios
develop
Preeclampsia Prevention
No clearly established preventive
measures
Some data (very small studies) suggest
possible benefit of low-dose aspirin in
prevention in high risk groups p g g p
But weight of evidence not strong
No benefit from calcium supplementation
Copyright Harvard Medical School, 2010. All Rights Reserved.
760
Other Forms Hypertension in
Pregnancy
Chronic hypertension: Antedates pregnancy
(systolic 140 and diastolic 90 mmHg)
Limited data regarding degree of hypertension to treat
and target goals
Expert opinion: treat bp > 150/95-100 mmHg
Aim for lower levels if end-organ damage present g g p
Gestational hypertension: mild hypertension
latter half pregnancy (but no proteinuria or other
signs preeclampsia)
Antihypertensive Drugs in
Pregnancy
D f h i Drugs of choice:
Methyldopa: 50 years experience; excellent long-term
safety profile
No alteration of uteroplacental or fetal hemodynamics
Labetalol: both alpha and beta-adrenergic blocking
properties; may have less impact on uterine/placental
blood flow than other beta blockers blood flow than other beta blockers
Nifedipine (long-acting): appears safe
Less data on diltiazem and verapamil but likely safe
May worsen hypotension and neuromuscular blockade if
magnesium sulfate used for preeclampsis
Copyright Harvard Medical School, 2010. All Rights Reserved.
761
Antihypertensive Drugs in
Pregnancy
D t id Drugs to avoid:
ACE inhibitors and ARBs
First trimester exposure: higher risk major congenital
malformations, esp. cardiovascular and cns malformations
Later pregnancy exposure: oligohydramnios, hypocalvaria
Drugs to use with caution:
Diuretics: avoid volume depletion Diuretics: avoid volume depletion
Beta blockers: may uterine/placental blood flow;
may be linked to intrauterine growth restriction and
fetal bradycardia
Question #1:
Which statement(s) is(are) true?
A) Risk factors for preeclampsia include A) Risk factors for preeclampsia include
primagravida status, advanced maternal age,
mutiple gestation pregnancy, cigarette smoking,
and obesity.
B) Definitive treatment is delivery
C) Preeclampsia most likely would have been
averted with low-dose aspirin and calcium averted with low dose aspirin and calcium
supplementation during pregnancy.
D) A. and B.
E) B. and C.
Copyright Harvard Medical School, 2010. All Rights Reserved.
762
Case Four
A 28 year-old woman who underwent a deceased
d l t l t 26 th i 6 donor renal transplant 26 months ago is now 6
weeks pregnant (unplanned pregnancy). She
has CKD due to focal segmental
glomerulosclerosis. Her immunosuppressive
medications include prednisone, tacrolimus and
mycophenolate mofetil She is not hypertensive mycophenolate mofetil. She is not hypertensive.
The creatinine is 1.2 mg/dl and the urine
protein/creatinine ratio is 0.4.
Question #1:
Which Statement is true?
Sh h ld ti h t i i A) She should continue her current immunosuppressive
medications.
B) Preeclampsia occurs in about two thirds of pregnant
recipients of renal grafts.
C) A. and B.
D) None of the Above
Copyright Harvard Medical School, 2010. All Rights Reserved.
763
Pregnancy in Recipients of Renal
Transplants
ESKD associated with infertility ESKD associated with infertility
Altered hypothalamic function
Increased levels FSH, LH, prolactin
Most anovulatory
incidence pregnancy estimated 0.5% yearly
Transplantation restores fertility in women
Usually within months
Some woman have no restoration of fertiliy Some woman have no restoration of fertiliy
ESKD can be associated with early menopause
Average 4.5 years earler than general population
Optimal contraception in place before transplantation
Women have become pregnant in peri-transplant interval
Pregnancy in Recipients of Renal
Transplants
Timing of pregnancy individualized Timing of pregnancy individualized
Avoid in peri-transplant interval
Highest potential exposure to fetotoxic and
teratogenic medications
Highest doses of immunosuppressive
medications used
Highest risk for rejection
Copyright Harvard Medical School, 2010. All Rights Reserved.
764
Pregnancy in Recipients of Renal
Transplants
Previous advice: wait two years
New AST Consensus: proceed when
Graft function optimal (creatinine < 1.5 and proteinuria
< 500 mg/day)
No fetotoxic infections
No use of fetoxic or teratogenic medications
Immunosuppressive medication doses stable and at Immunosuppressive medication doses stable and at
maintenance levels
Usually about one year after transplant
McKay et al. Am J Transplantation;5:1-8,2005.
Pregnancy in Recipients of
Renal Transplants
Risk of pregnancy-related renal function
decline
Depends on renal function at outset of
pregnancy
Generally, creatinine < 1.3 confers little risk
Creatinine > 1.5 and proteinuria > 500 mg/24
hours confers increased risk irreversible graft loss
Risk of preeclampsia about 32% (NTPR registry data)
Armenti et al. Clin Transpl 103-119, 2005.
Copyright Harvard Medical School, 2010. All Rights Reserved.
765
Pregnancy in Recipients of Renal
Transplants
Risks of Pregnancy to fetus
Preterm delivery rate 50 to 64%
Mainly due to maternal and/or fetal compromise
High risk premature rupture membranes
Low birth weight rate about 50%
Intrauterine growth restriction rate about 50%
Most cases thought due to hypertension and/or Most cases thought due to hypertension and/or
preeclampsia or renal insufficiency
High incidence (~20-26%) developmental delays
reported after age 5 (NTPR)
Stanley et al Transplant Proc 31:241-242, 1999
Pregnancy in Recipients of Renal
Transplants
Safety of immunosuppressive drug use in pregnancy not
clearly established clearly established
All pass through maternal-fetal circulation
Some drugs appear safer than others
FDA classifications not helpfulNone labelled A
Azathioprine and low-dose prednisone
Minimal adverse effects on fetus
Fetal liver lacks enzyme to metabolize azathioprine Fetal liver lacks enzyme to metabolize azathioprine
Placenta metabolizes prednisone
Calcineurin Inhibitors
Monitor levels carefully to prevent toxicity
Avoid sirolimus and mycophenolate mofetil
Copyright Harvard Medical School, 2010. All Rights Reserved.
766
Question #1:
Which Statement(s) is(are) true?
Sh h ld ti h t i i A) She should continue her current immunosuppressive
medications.
B) Preeclampsia occurs in about two thirds of pregnant
recipients of renal grafts.
C) A. and B.
D) None of the Above
Pregnancy and Renal Disease:
Conclusions
Likelihood of pregnancy-related worsening of renal
function depends on degree of impairment at outset of p g p
pregnancy
Fetal survival very good with renal disease but risk of
prematurity and intrauterine growth restriction increased
Preeclampsia risk increased with renal disease
Monitor closely for possible onset in 3
rd
trimester Monitor closely for possible onset in 3 trimester
Fertility restored by transplantation
Know which antihypertensives and immunosuppressive
medications to use or to avoid in pregnancy
Copyright Harvard Medical School, 2010. All Rights Reserved.
767
Disclosures
None
Copyright Harvard Medical School, 2010. All Rights Reserved.
768
Treatment of Glomerulonephritis
J eremy S Duffield MD PhD
Laboratory of Inflammation Research, Renal Division,
Brigham & Womens Hospital, & Harvard Medical School,
Boston, MA
http://duffieldlab.bwh.harvard.edu
Disclosures
Scientific Advisory Board for Promedior
Inc and Regulus Therpeutics
Copyright Harvard Medical School, 2010. All Rights Reserved.
769
Rapidly Progressive Glomerulonephritis
Definition: Clinical syndrome characterized by rapid loss of renal
function (<3months)
With features of glomerulonephritis:
o Dysmorphicred cells, red cell casts, leukocyturia, granular casts
o Glomerularproteinuriaoften >3g/d
Often assoc with oliguria/anuria
A renal biopsy will show >50% Crescents
Also called crescentic GN
Often referred to as nephritic syndrome (overlap)
Most aggressive form of glomerularinjury
Pathol ogy:
a) fibrinoidnecrosis,
b) epithelial cell crescents,
c) destruction of glomerularcapillaries,
d) vasculitis in small vessels,
e) varying degrees of tubular injury (ATN)
RPGN differential
Atheroembolic disease
Acute thrombotic microangiopathy (TMA)
ATN +/- underying chronic GN
AIN
Crystalopathies
Acute myeloma kidney
Obstruction
Copyright Harvard Medical School, 2010. All Rights Reserved.
770
Algorithm for diagnosing the causes of
RPGN
Rapidly progressive glomerulonephritis
Type I Type II Type III
What is severe glomerulonephritis?
Type II: Immune complex deposition triggers activation of
leukocytes (innate immunity) +/- autoimmunity against nuclear
proteins
Type I: Autoimmunity (cell mediated & humoral)
againstglomerularbasementmembrane
Type III: Pauci-immune with leukocyte activation triggered
in the glomerularcapillaries by anti-leukocyte antibodies &
anti-leukocyte cell mediated immunity
Glomerulus: highly specialized vasculature with
fenestrated endothelium unique CBM, high shear
stresses & specialized high density pericyte
(podocyte)
CD68
Copyright Harvard Medical School, 2010. All Rights Reserved.
771
Y Y Y Y Y Y Y Y Y Y Y Y Y Y Y Y Y Y Y Y Y Y Y Y Y
Y Y Y Y Y Y Y Y Y Y Y Y Y Y Y Y Y Y Y Y Y Y Y Y Y
Disease Mechanisms
Y
Y
Y
Y
Y
Y
Y
Y
Y
Y
Y
Y
Y
Y
Y
Y
Type II
RPGN
Type I
RPGN
PMN
Monocyte
CD8 T cell
Type III: Model of the role of ANCAs
in disease initiation
TNF
Secondary
inflammation
H
2
O
2
H
2
O
2
C5a
1. 2. 3.
Primed neutrophil
Primed monocyte
ANCA
Integrin
Integrin receptor
Fc gamma receptor
Granules or lysosomes
Copyright Harvard Medical School, 2010. All Rights Reserved.
772
Frequency of presentation RPGN by
glomerular pathology
Type I
Type II
Type III
Brenners The Kidney, 7th Ed, 2007
Characteristic glomerular pathology
Focal necrosis
Fibrinoidnecrosis
Karryorrhexis (nuclear
fragmentation)
Capillary rupture
Crescent formation
Endocapillaryinflammation
Vasculitis
Leukocyte infiltration
Copyright Harvard Medical School, 2010. All Rights Reserved.
773
Range of clinical presentations
by disease type
Remember RPGN may
be superimposed on
other common
glomerulardiseases
(ischemic vasculopathy
or diabetic nephropathy)
What tests should I order?
Immune panel:
CH50, C3, C4, ANA, dsDNA (Ro, La), CRP, RhF, cryoglobulins, IgE,
anti-GBM, ANCA IF, anti-PR3, anti-MPO, SPEP, UPEP
Infection panel:
HepB, HepC serology and viral load, ASOT, blood cultures, other anti-
sera
Other: eosinophils blood/ urine, FE Na
Urine: microscopy, Upr/UCr
US kidneys:
Copyright Harvard Medical School, 2010. All Rights Reserved.
774
Treatment objectives
Prevent immune complex deposition and or
formation
Prevent myeloid leukocyte activation in
response to immune complexes or
autoantibodies
Prevent autoimmunity (antigen presentation,
autoreactive T cell and B cell clones)
Prevent secondary inflammation
Allow speedy repair of parenchymal cells
Treatment options
Prednisone/methylprednisolone: Anti-inflammatory and leukocyte
cytotoxic
Cytotoxic
Cyclophosphamide
Anti-T cell
Tacrolimus/cyclosporin
Antiproliferative
Azathiaprine
Mycophenolate
Plasmapheresis
Anti-viral
Cytotoxicantibody therapies (eganti CD20, anti CD52, ATG
Supportive therapies
Treatment of infections where appropriate
Copyright Harvard Medical School, 2010. All Rights Reserved.
775
Pauci-immune GN / type III RPGN only
seen in ANCA associated vasculitis (AAV)
Common
Initially thought to be adaptive cell-mediated autoimmunity
ANCA identified in 1988 (Falk & J ennette,Niles, Goldschmeding
1989)
1993 antibodies first described to be potentially pathological
Rat Model 1993
Mouse model 2003
Abs alone induce disease but probably require PMN and endothelial
cell activation
However frequently no PMN seen in the glomerulus. Secondary (?)
inflammation with T cells and monocytes
RPGN manifestations of ANCA
associated Vasculitis (AAV)
Disease Clinical Characteristics %MPO %PR3
ChurgStrauss Syndrome Asthma, eosinophilia, neuopathy, nephritis 30-70 <10
Wegeners Granulomatosis nose bleeds URT, nephritis, lung lesion 10-30 >70
Granulomas
Microscopic Polyangiitis Nephritis, alveolar hemorrhage, purpura 30-70 10-30
No granulomas
Renal Limited Vasculitis Nephritis only 30-70 10-30
BVAS (1994)-clinical activity score
Evidence of vasculitisin 10 organ systems: General (4), cutaneous(5), mucous
membranes/eyes (10), ENT (5), chest, (7) cardiovascular (6), abdominal (3),
renal (7), nervous system (9), other (4)
Copyright Harvard Medical School, 2010. All Rights Reserved.
776
Not all AAV is RPGN
Natural History of Wegeners
Granulomatosis
Hoffman et al, Ann Int Med 1992; 116:488-498
Study of 158 patients
8% of patients: diagnosis made 5-16 yrs after symptoms began
90% of pts 1
st
sought med care due to upper or lower resp tract symptoms
18% with GN at presentation: in all cases renal disease was asymptomatic.
77% of patients eventually developed GN, usu. within 2 years.
11% developed ESRD.
88% C-ANCA; 5% P-ANCA
ANCAs: what are they and how to
interpret antibody results?
Immunofluorescence
pANCA or cANCA pattern
Ethanol vs formalin fixation
1:20 cut off
Exclude ANA
Majority of targets are MPO and PR3
Minority are lactoferrin, CathepsinG, elastase and others
Also there are some IgA and IgM ANCAs
Plasma concentrations of ANCAs (Especially anti-PR3)
correlate with disease activity
Copyright Harvard Medical School, 2010. All Rights Reserved.
777
ELISAs for MPO & PR3
Csernocket al Rheumatology (2002)
41:1313
IIF alone (1:20 cut off) sens 80% (WG)
70% (MPA) spec 100% (WG) 100 (MPA)
Better sens +spec for IIF than 7
commercial ELISAs.
Meta-analysis: Choi et al J Rheum (2001)
28: 1584
MPO sens 37% (CI: 27-47) spec 96% (CI
94-98). Combined with IIF 31% 99%
PR3 +MPO +IIF sens 85% spec 98%
Recommend IIF +ELISA
Anti PR3
Anti MPO
Bolton/Couser/Cameron 1970s
Bolton and Couser, Am J Med 1979(66):495-502
Fauci 1983 Annals Int Med;98(1):76-85.
Pusey et al Kid Int (1991)
Mepex Trial (2007) J ayne et al J ASN 18: 2180
Euvas Trial NEJ M (2003) 349:36
Copyright Harvard Medical School, 2010. All Rights Reserved.
778
Anti-GBM disease/Goodpastures
syndrome: Type I RPGN
Rare
Either pulmonary renal or renal limited disease
Linear deposition of antibody along GBM (>95%
IgG+C3, rarely IgMor IgA)
Plasma concentrations correlated with disease
Eluted antibodies transferred disease to Monkeys
However there are patients with antibody
deposition and no disease
If associated with lung hemorrhage, known as
Goodpastures syndrome
Rapid access to anti-GBM antibody assay required
Frequently associated with ANCA disease
Goodpastures: disease
mechanisms
Disease susceptibility: HLA-DR2 (class II MHC)
The antigen is non-collagenous domain of
Collagen IV of GBM
90% of Abs directed against alpha3(IV)NC1
(majority 7AA segment)
Rarely against 1 or 4 collagen chains
Although autoantibody is pathogenic there is
unequivocal evidence for T cell mediated
autoimmunity as well (Kalluri et al J CI 1997,
Reynolds et al NDT 2002)
Copyright Harvard Medical School, 2010. All Rights Reserved.
779
Demographics of anti-GBM
disease
Savage et al Brit Med J 1986. Study of 71pts
Male: female. No preponderance. Broad range of ages
Goodpastures syndrome in 30%
Antibody Titres correlated with disease severity
2/3 presented dialysis dependent
Hypertension rare.
10% had arthitis/myalgia/vasculiticrash (probably had
renal limited ANCA disease as well)
Not all anti-GBM disease is RPGN
Presentation & outcome for anti-GBM
disease
Levy et al Ann Int Med (2001).
Single centre retrospective analysis 71 Pts. All treated with Pred, CYC and PEx
Proportion with
lung involvement
White bars =lung hemorrhage
Gray bars =no lung disease
%men
Copyright Harvard Medical School, 2010. All Rights Reserved.
780
Outcomes for patients with anti-GBM
disease
Very small chance of renal recovery if dialysis dependent.
If dialysis dependent and no lung hemorrhage consider not
using immunosuppression
Kaplan-Meier Plots
Outcome by %crescents and
presenting renal status
%

c
r
e
s
c
e
n
t
s
Final dialysis dependent
Final dialysis independent
Anti-GBM disease (based on case series data)
Prednisone 1mg/kg ( 80mg) 3mo then
taper
CYC2.5mg/kg/d 3mo then taper.
Titrate against lymphocyte count
Plasma exchange 4L qd for 14 days
MONITOR ANTI-GBM LEVELS
Consider IV methylprednisolone 1gX2-3 for
veryactive disease
Consider AZA 1mg/kg/d
Maintenance Induction
Prednisone & AZA
(Prednisone taper over 3-6mo to
maintenance 0.1mg/kg/d
AZA initially2.0 mg/kg6mo then
taper over 6mo to 1mg/kg/d)
MONITOR ANTI-GBM LEVELS
Evidence of Pulmonary Hemorrhage?
Consider: IV MP, IVIG, ATG, anti-CD52 Ab
Consider:
Prednisone & MMF
Prednisone & Leflunamide
Or continued Rituximabtherapy(q6-12mo)
YES
Dialysis Dependent?
Evidence of PulmonaryHemorrhage? ANCA?
YES NO
NO
SUPPORTIVETREATMENT
ONLY
Copyright Harvard Medical School, 2010. All Rights Reserved.
781
RPGN: Type II disease (with immune
complexes)
Lupus nephritis
Sjogrens Syndrome
Hepatitis virus mediated nephritis
IgA nephropathy
Henoch Schonlein Purpura
MPGN due to cryoglobulins
Post-streptococcal glomerulonphritis
Other post-infectious e.g. subacute endocarditis
Lupus Nephritis - RPGN
No RCT specifically studying clinical presentation in LN. Therefore
no study only looking at RPGN. All studies use histology +
Proteinurialevels
Many recent studies exclude RPGN e.g. MMF Hong Kong study
excluded pts with Cr>3.5 to try to exclude chronic disease.
However very good supportive evidence from lupus studies for
Prednisone and CYC regimes in RPGN
Consistent data support role for MMF in maintenance therapy and
show MMF for induction therapy in mild-to moderate disease
Small open label studies support the role for rituximabin resistant
disease
No role for PExin addition to steroids and CYC in severe LN (Lewis
et al NEJ M 1992 326 1373)
Copyright Harvard Medical School, 2010. All Rights Reserved.
782
Interpretation of clinical trials
in Lupus Nephritis
Ethnicity
Disease severity
Number of patients
Time to see a difference over and
above steroids alone (5 years)
Meta-analysis of MMF in Lupus
Nephritis
MMF for Induction Therapy of LN:
A Systematic Review and Meta-analysis 268 pts in 4 RCTs MMF vs Cytoxan for
induction in LN.
RR of failure to induce remit w MMF vs Cyt 0.7 ( p=.005) ; RR for death or ESRD for
MMF vs Cyt 0.44 ( p=.02).
Adverse events similar except more leukopenia, amenorrhea in Cyt pts.
Conclusion:
MMF compared to Cyt reduces the risk of treatment failure during induction and may
reduce death and ESRD.
MMF recommended as first line therapy for LN in pts w/o severe renal
dysfunction.
Walsh M, James M, Jayne D, et al. Clin J Am Soc Neph 2:968, 2007
Accepted practice for the management of Lupus nephritis has changed
Chan TM,.J AmSoc Nephrol. 2005 1076-84.
Chan TM, et al . N Engl J Med. (2000);343:1156-62.
Contreras et al NEJM
Copyright Harvard Medical School, 2010. All Rights Reserved.
783
CASE of LN with RPGN
22-year-ol d man with l upus and nephri ti s, al ong wi th edema,
hypertension and nephroti c range protei nuri a (12g/d) active uri ne
sediment and a rapi d ri se i n Creati ni ne over 2 weeks from 1.0 (eGFR
70) to 4.0 (eGFR 15) dsDNA >4000 whil e on IV cycl ophosphami de (4/6)
and Predni sone. Had previ ous course of Predni sone and MMF. He
was previ ousl y bi opsi ed 2 years ago and had Class III l upus nephriti s.
Never been i n compl ete remi ssi on
Renal Biopsy showed severe diffuse proliferative GN (Class IV) with >50%
crescents andsevere interstitial inflammation
Management of case
In addition to supportive therapy and prophylaxis
MP 1g x3. No response over 4 days
ATG 5mg/kg/d X4 followed by rituximab 1g X1
Cr improved slowly from peak of 4.6 to <1.0 over 4
weeks.
Transitioned to 60mg pred/d, Cellcept 3g/d and
tacrolimus 1mg bid (level of 5-8)
2 months later persistent 5g prot/d (nephrotic
syndrome), active urine sediment, but normal
complement, dsDNA and normal eGFR
Copyright Harvard Medical School, 2010. All Rights Reserved.
784
What is supportive therapy?
ACEI +/- ARB
BP <130/80 with
antihypertensives/diuretics
GI ulceration prophylaxis
Anti PCP prophylaxis, antiviral prophylaxis
Osteoporosis prophylaxis
Edema control
Glycemic control
LUPUS NEPHRITIS
Consider: IVIG, ATG, anti-
CD52 Ab(Campath) and/or
Rituximab
Consider plasmapheresis
Prednisone 1mg/kg ( 80mg) 3mo
then taper +MMF 3g/d for
12mths
If GFR and/ or concurrent Class
V with Class IV disease consider:
a) Induction with IV MP
500mg/1g q24h X3 and CYC
500-750mg/m
2
q mo 6mo
b) Also consider 250mg/m
2
CYC
q14d for 3mo
c) MMF +/-tacrolimus
Titrate CYCagainst lymphocyte
count
(If CYCis not tolerated consider:
prednisone & Rituximab
[375mg/m
2
X4 or 1gX2-4])
Maintenance Induction Relapse
No clear role for Plasmapheresis
Prednisone & MMF
(Prednisone taper over 3-
6mo to maintenance
0.1mg/kg/d
MMF initially3g/d, taper
over 24mths)
Consider:
Prednisone & Azathiaprine
Prednisone & Leflunamide
Or continued Rituximab
therapy(q6-12mo)
Continueat least 2years
Even if complete remission
asrelapserateishigh
Prednisone & IV CYC
or Prednisone &
Rituximab +/- IV CYC
1g/m
2
x2-4
Alternatives:
Prednisone & MMF
+/- tacrolimus
Consider anti-CD52
antibodies or ATG
Partial Remission
Prednisone & IV CYC
or Prednisone &
Rituximab +/- IV CYC
1g/m
2
x1
Alternatives:
Prednisone & MMF +
tacrolimus
RPGN (>50%Crescents & risingCr) No RPGN
Induction
NO TRIALS--EMPIRICAL
InductionwithIV MP
500mg/1gq24hX3 CYC
500-750mg/m
2
qmonth
6mo
Titrate against lymphocyte
count
Consider transitioningto
+MMF3g/d+/-tacrolimus
No response
Pulmhemorr or cerebritis
Copyright Harvard Medical School, 2010. All Rights Reserved.
785
Rituximab-humanized anti-CD20 antibodies
effective therapy in RPGN
Antigen presenting cells Costimulatory cells
Roles for B cells in more than Abproduction
CD8
AG
CD4
cytotoxicity
AG
cytokines
cytokines
cytokines
B
Current view of B cells in
adaptive immune
response
CD8
CD4
primed
plasma
AG
AG
B AG
Traditional view of B cells
in adaptive immune
response
Plasma cells
Short lived
Copyright Harvard Medical School, 2010. All Rights Reserved.
786
Keogh et al (2006) Mayo: 10pts
with severe active WG or
treatment resistant. All went
into remission with RIT
375mg/m
2
X4) +oral Pred. 4
had a relapse which responded
to repeat RIT
Smith, Jayne et al J Rheumatology 2007
Prospective open label studies
show efficacy in relapse
RCTs in AAV and Lupus nephritis awaited
Rituximab- anti B cell therapy
Effective in reducing remission in patients
who have relapsed with
AAV with nephritis
Lupus nephritis
Effective primary therapy in hepatitis virus
associated glomerulonephritis and RPGN
Excellent safety profile
RCTs still needed
Copyright Harvard Medical School, 2010. All Rights Reserved.
787
Question 1
Randomized controlled trials in AAV:
1. Support a role for methotrexate
2. Show that MMF is superior to azathioprine
3. Indicate that it is safe to replace cyclophosphamide
after 6 months
4. Indicate the plasma exchange does not improve
outcome
5. Show a clear role for cyclophosphamide over
azathioprinein induction
Question 2
Concerning antibody therapies in the management of AAV:
1. There are RCTs supporting the use of antibody
therapies in AAV
2. Anti-TNF antibodies have been used effectively in the
management of resistant AAV
3. Anti-B cell therapy appears to be effective in many
patients resistant to conventional therapy
4. Antibody therapies have few side effects
5. CD54 and CD20 have the same cell expression profile
Copyright Harvard Medical School, 2010. All Rights Reserved.
788
Question 3
The following may be useful in the management of RPGN
due to cryogobulins
A. Prednisone
B. Plasma exchange
C. Rituximab
D. Bactrim
E. Cyclophospamide
Question 4
The following statements about the role of prednisone and
cyclophosphamide in the treatment of AAV are true
A. The use of methylprednisone or prednisone has been
subjected to RCT
B. The use of CYC has been subjected to an RCT
C. CYC is accepted first-line therapy for AAV with
crescentic GN
D. CYC causes thrombocyosis
E. Prednisone is used alone if there is pulmonary disease
as well as renal disease
Copyright Harvard Medical School, 2010. All Rights Reserved.
789
Disclosures
Scientific AdvisoryBoard for Promedior Scientific Advisory Board for Promedior
Inc and Regulus Therpeutics
Copyright Harvard Medical School, 2010. All Rights Reserved.
790
Brigham Board Review Course
Aug 10, 2010
Q&A
Dana Miskulin, MD
Tufts Medical Center
Disclosures
None
Copyright Harvard Medical School, 2010. All Rights Reserved.
791
Case #1
44yo male with developmental delay, deaf HIV on
HAART (undetectable viral load but CD4 counts
consistently <100), HCV 2/2 IVDA, DM, lipodystrophy,
CAD s/p PCIX3, presents to ID clinic with increasing leg
edema, 10 kg weight gain, dyspnea, fatigue over last few
weeks;
admitted to hospital for diuresis and work-up; responds
to lasix/metolozone
24h urine shows 10g proteinuria
sCr 2.33 mg/dl up from 1.0 mg/dl two months earlier
U/A spgrav 1.015, ph 6, 2+blood, 3+protein, leuk
esterase negative
Microscopy- 40 dysmorphic RBC/hpf, no RBC casts,
numerous lipid laden casts and oval fat bodies
After holding ASA for one week, a kidney biopsy is
performed
Copyright Harvard Medical School, 2010. All Rights Reserved.
792
Immuno fluorescent examination could not be performed
on this formalin fixed specimen.
Copyright Harvard Medical School, 2010. All Rights Reserved.
793
Question 1
How would you treat this patient?
A. Pegylated interferon a + plasmapheresis
B. Corticosteroids
C. Pegylated interferon a
D. Pegylated interferon a + ribavirin
E. Pegylated interferon a + rituximab
DIAGNOSIS
MEMBRANOPROLIFERATIVE GLOMERULONEPHRITIS
(MPGN)
OF IMMUNE COMPLES ETIOLOGY
MOST LIKELY HEPATITIS C ASSOCIATED.
OVERLAPPING DIABETIC NEPHROPATHY
VASCULOPATHY, MODERATE
TUBULAR ATROPHY AND INTERSTITIAL FIBROSIS,
MILD
Copyright Harvard Medical School, 2010. All Rights Reserved.
794
Answer to Question 1
The correct answer is C.
The amount of blood on the urine sediment was more suggestive of
MPGN than HIVAN and indeed the biopsy demonstrates characteristic
findings of MPGN. Cryoglobulins were not present on EM. A review of
the evidence for treatment of HCV in CKD is found in the KDIGO HCV
guidelines published in 2009. A 12 month course of pegylated
interferon plus ribavirinis recommended in patients with moderate
proteinuriaand slowly declining kidney function. If CrCl<50 mls/min,
ribavirinshould be avoided due to high risk for hemolytic anemia and
red cell aplasia. A meta-analysis of trials comparing antiviral versus
immunosuppressive therapy (corticosteroids alone or in combination
with cyclophosphamide) in patients with HCV associated
glomerulonephritis, showed proteinuriadecreased more with IFN
therapy although there was no difference on long term renal function
[Alric, KI 2004]. The guidelines suggest adding Rituxamib,
cyclophosphamideor plasmapharesis to anti-viral therapy in patients
with nephrotic range proteinuria, rapidly progressive GN, or in an acute
flare of cryoglobulinemia.
Copyright Harvard Medical School, 2010. All Rights Reserved.
795
Case #2
20 yo female recently diagnosed with ADPKD
History:
12/15- dysuria, frequency and foul smelling urine
12/25-left sided flank pain and fevers to 105
12/27-at outside hospital diagnosed with pyelonephritis and started on
Bactrim
12/30-seen in clinic with temperature 102, nausea, ongoing left sided
flank pain
Labs reveal sCr 0.70, WBC 15.3 with 87% PMNs and 1% bands
U/A specific gravity 1.004, pH 5.5, nitrites negative, leuk esterase
negative, blood negative, 1 WBC, 1 RBC and a few bacteria
-CT: Both kidneys are enlarged with innumerable heterogeneous
appearing cystic lesions, some hypodense, some hyperdense, of
differing sizes. There is a dominant 4.8 X 3.8 cm cyst at the upper
pole of the left kidney with fat stranding in the perinephric fat.
Question #2a (cont)
Which Abx is least likely to be effective for
this condition:
A. Trimethoprim
B. Ciprofloxacin
C. Gentamycin
D. Chloramphenicol
Copyright Harvard Medical School, 2010. All Rights Reserved.
796
Answer to 2A
The answer is C, gentamycin
Urinary tract infections in the ADPKD may consist of cystitis,
pyelonephritis, or cyst infection. Females are more likely than
males to be affected, in keeping with infection developing via
the ascending route. E coli and other gram negatives are the
most common organisms identified. The urine culture may be
negative if the infected cyst(s) is detached from the collecting
system. Cyst infection and pyelonephritis may be difficult to
distinguish as both present with flank pain and fevers and
imaging may not reveal an infected/inflamed cyst. The usual
treatment is fat soluble antibiotics which are able to penetrate
cyst walls. Even then, Abxmay need to be given for a
prolonged course if several cysts are involved. All of the
antibiotics shown, except gentamycinare fat soluble and
reach high concentrations in cyst fluid. A prolonged (E.g. 4-6
week course of Abxmay be needed in severe infections in
which the initial response to Abxis delayed.
Copyright Harvard Medical School, 2010. All Rights Reserved.
797
Case #2 Continued
Urine culture reveals >10,000 cfu/ml E Coli
sensitive to all but ampicillin
12/30 She is started on IV Ciprofloxacin
1/3 The urine has cleared but she continues
to spike temperatures and the WBC
remains elevated with a left shift
Question 2B
How would you manage this patient?
A. Add IV trimethoprim
B. Increase the ciprofloxacin to 400 mg IV
Q8 hours
C. Continue ciprofloxacin at 400 mg IV Q12
hours
D. Obtain a gallium scan
E. Arrange a CT guided percutaneous drain
of the left upper pole kidney cyst
Copyright Harvard Medical School, 2010. All Rights Reserved.
798
Answer to #2B
The correct answer is E.
The CT scan (non-enhanced) shows a large inflamed cyst in
the left upper pole, corresponding to the patients symptoms.
The patient remains febrile despite 4 days of an antibiotic that
penetrates the cyst wall and to which the organism is
sensitive. It thus, would not make sense to switch antibiotics.
A gallium scan would confirm that the left upper pole cyst is
infected / inflamed, although we already know this from the
CT. A trial of higher dose ciprofloxacin could be attempted
but the better treatment would be to drain the infected cyst
(i.e. abscess). The cyst was aspirated for creamy fluid that
was growing E-Coli (sensitive to ciprofloxacin) and the patient
rapidly defervesced. A pigtail drain was placed and kept in
place until the cyst stopped draining. The patient continued a
6 week course of ciprofloxacin. Partial or complete
nephrectomymay be required for persistent infection, despite
antibiotic therapy/ percutaneous cyst drainage.
Copyright Harvard Medical School, 2010. All Rights Reserved.
799
Case #3
36 yo presents to the ER room with blurred vision and is
found to have a BP of 230/130 mm Hg
He has had high BP for years. He has not seen a
physician in one year. He has been taking Chinese
herbal medications for headaches
Retinal exam reveals Grade III hypertensive retinopathy
Labs: serum creatinine 2.3 mg/dL, BUN 30
U/A: 4+protein, 3+blood
UACR: 1.94
Started on labetalol 200 mg BID and lisinopril 20 mg per
day and discharged with follow-up in renal clinic
In renal clinic, BP 140/100
Urine sediment shows dysmorphic RBCs,
RBC casts, lipid-laden casts
A serum creatinine from a year ago is 1.2
mg/dl
A renal biopsy is performed
Case 3 (Cont)
Copyright Harvard Medical School, 2010. All Rights Reserved.
800
Copyright Harvard Medical School, 2010. All Rights Reserved.
801
23
IgA
IgG
Question 3A
What is the diagnosis?
A. FSGS
B. MPGN
C. IgA nephropathy
D. ANCA vasculitis
E. Membranous nephropathy
Copyright Harvard Medical School, 2010. All Rights Reserved.
802
Answer to Question 3A
The correct answer is C.
There is increased mesangium, mesangial
proliferation, and electron dense deposits
in the mesangium consistent with IgAN.
Immunofluoresence was 3+ positive for
IgA, 1+ for IgG, others were negative.
Copyright Harvard Medical School, 2010. All Rights Reserved.
803
Biopsy
Renal biopsy 28 of 36 glomeruli are 28 globally
sclerosed. The viable glomeruli show
mesangioproliferative changes, segmental to global
membranoproliferative changes, 1 glomerulus shows
active crescent. The interstitium show patchy tubular
atrophy and cortical fibrosis. There is moderate
interstitial nephritis. The arterial blood vessels show
severe vasculopathic changes. IF is 3+for mesangial
deposits of IgA.
Serum creatinine remains 2.9 mg/dl
How would you manage this patient?
Question 3B
In addition to controlling BP, how would
you treat this patient?
A.ACE inhibitors/ARB
B.ACE inhibitors/ARB + MMF
C.ACE inhibitors/ARB + Steroids
D.ACE-inhibitors/ARB + Steroids +
cyclophosphamide
Copyright Harvard Medical School, 2010. All Rights Reserved.
804
Answer to Case #3
The correct answer is A, maximize RAAs blockade.
Proteinuria is the most prognostic factor for renal outcomes in IgAN. The exact
threshold (0.5g vs. 1g/d) above which risk of renal progression is increased, is
uncertain but a reduction in proteinuria to <1 g/d has been associated with a better
prognosis. Several trials have shown a benefit of ACEI or ARBs in reducing
proteinuria and improving kidney function. The first step in managing this patient
should be to aggressively lower BP and maximize RAAS blockade, both of which may
significantly reduce proteinuria and slow GFR decline. The role of corticosteroids has
been assessed in patients with >1 g/d proteinuria and GFR >50 mlsmin [Manno C
NDT 2001; Manno C NDT 2009; Lu J et al AJ KD 2009, Pozzi C J ASN 2004]. Three
of 4 RCTs used corticosteroids (different regimens but 0.5-1.0 mg/kg/d) in
combination with ACE inhibitors compared to ACE inhibitors alone. 4/4 studies
showed reduced proteinuria and 3 out of 4 showed a slower decline in kidney function
and /or a smaller number with a 50% decline in GFR with prednisone treatment. No
trials have examined prednisone treatment in patients with GFR<50 mls/min but if
proteinuria remains elevated after optimizing BP and RAAS blockade, it could be
considered. Cyclophosphamide and steroids are reserved for patients with
deteriorating kidney function and severe proteinuria, in the absence of extensive
scarring (a problem in this case) or in the setting of extensive crescents and rapidly
declining GFR. RCTs of MMF in IgAN have shown conflicting results. More studies
with MMF are needed.
Copyright Harvard Medical School, 2010. All Rights Reserved.
805
Case #4
26 yo with a history of antiphospholipid Ab syndrome
(presented with PE), SLE (mainly joint involvement)
treated with low dose steroids X 5 years (discontinued 2
years ago) and plaquenil, which he still takes
Referred by rheumatologist for 4+proteinuria on dipstick
and a 24 h collection found to have 7.6 g/ day of
proteinuria. Prior dipsticks 6 months ago were negative
for protein
P/E BP 144/88; lungs clear, trace peripheral edema
U/A pH 5; specific gravity 1.015; +3 protein; trace blood;
no leukocytes; no nitrites; no other findings
Microscopy- 2-3 RBC/hpf, no casts
A kidney biopsy is performed
Copyright Harvard Medical School, 2010. All Rights Reserved.
806
Copyright Harvard Medical School, 2010. All Rights Reserved.
807
IF
Diffuse
global
Granular
IgG, IgA, IgM
C3, C4
C1q
4A. What is the diagnosis?
A. Thrombotic microangiopathy
B. WHO Class IV lupus nephritis
C. WHO Class III lupus nephritis
D. WHO Class V lupus nephritis
Copyright Harvard Medical School, 2010. All Rights Reserved.
808
Answer to Question #4A
The correct answer is D.
The biopsy shows markedly thickened capillary
basement membranes with spikes on silver stain. There
are diffuse subepithelial electron dense deposits with
"spikes" and "domes" present. The epithelial cell foot
processes are fused. Immunofluorescent studies reveal
granular deposits of IgG, IgA, IgM, and complement
along the glomerular basement membranes.
These findings are diagnostic of membranous
glomerulonephropathy, or Class V lupus nephritis.
Copyright Harvard Medical School, 2010. All Rights Reserved.
809
Question #4B
Which of the following statements about Class V
(membranous) lupus is correct:
A. 50% of patients with nephrotic range proteinuria have
a spontanous remission
B. Treatment of patients with Class III and Class V lupus,
should be directed towards treatment of the Class V
lupus
C. Cyclosporine or MMF are first line treatments for Class
V lupus
D. Patients with subnephrotic range proteinuria do not
need treatment
E. ESRD occurs in about 10% of patients with Class V
lupus after ~10 years
Answer to #4b
The preferred treatment for Class V lupus are cyclophosphamide or
cyclosporine. There is only 1 small RCT (n=15 per am) comparing
cyclophosphamide or cyclosporine against prednisone in patients
with nephrotic range proteinuria. Those treated with either CYC or
CSA were more likely to achieve a complete remission but those
treated with cyclosporine relapsed after therapy was discontinued
(40% within a year). The toxicities of cyclophosphamide need to be
weighed against the harm, especially in young women of
childbearing age. MMF has only been tested in small, observational
studies of a short duration. The complete remission rate is in the
order of 40-60% at 6 months. RCTs are needed before MMF can be
recommended. In patients with Class 3 and 5 lupus, the treatment
should be directed at the more severe, Class 3 disease. The risk of
ESRD is ~10% at 10 years and is more likely in patients with non-
remitting, heavy proteinuria. Spontaneous remissions of nephrotic
range proteinuria are rare.
Copyright Harvard Medical School, 2010. All Rights Reserved.
810
Disclosures
None
Copyright Harvard Medical School, 2010. All Rights Reserved.
811
Glomerulonephritis
take home messages
Ajay K. Singh MD
asingh@partners.org
Disclosures
Consulting Consulting
Amgen, Johnson and Johnson, Sandoz, Fibrogen
Grant Support
Amgen, Johnson and Johnson, Roche
Advisory Boards
Rockwell Rockwell
Copyright Harvard Medical School, 2010. All Rights Reserved.
812
ISN/RPS Classification 2004
Class-I Minimal mesangial LGN
Class-II Mesangial proliferative LGN
Class-III Focal GN (<50% glom. involved) (a/cor a-c)
Class-IV Diffuse GN (a/c or a-c)
( >50% glom. involved)
IV S and IV G
Weening et al Kidney International,65: 521-30,2004
IV-S and IV-G
Class-V Membranous LGN
Class-VI Advanced sclerotic LGN
( 90% sclerosed )
Prognostic Features in LN
Histological Predictors
WHO ISN Histologic Class IV
Activity and Chronicity Index
Crescents and Interstitial fibrosis
Segmental necrotizing lesions
Clinical Predictors
Hypertension
Anemia
High baseline serum creatinine
Higher baseline proteinuria
Delay in therapy Delay in therapy
Epidemiologic Predictors
African American Race
Low socioeconomic status.
Appel Appel G, Cameron JS in Comprehensive Clinical Nephrology 2007. G, Cameron JS in Comprehensive Clinical Nephrology 2007.
Copyright Harvard Medical School, 2010. All Rights Reserved.
813
Treatment
I NDUCTI ON
MAI NTENANCE
Cycl ophosphami de
Azat hi apr i ne
Myc ophenal at e mof et i l
Cycl ophosphami de
Azat hi opr i ne
Myc ophenal at e mof et i l
Pr edni sone
Met hyl pr edni sone
Pr edni sone
Met hyl pr edni sone
FSGS
CP1177455-5
Copyright Harvard Medical School, 2010. All Rights Reserved.
814
FSGS
FSGS is a lesion, not a disease
Incidence of FSGS in biopsy series has risen
Epidemiology
Brigham Renal Board Review Course
Incidence of FSGS in biopsy series has risen
over the last 30 years.
FSGS most common pathology in biopsies for
idiopathic NS in US; 35% in all cases, 50% in
African-Americans
FSGS most common primary glomerular disease FSGS most common primary glomerular disease
causing ESRD in US (USRDS)
Copyright Harvard Medical School, 2010. All Rights Reserved.
815
FSGS is a descriptive term for a pathologic
Pathology
Brigham Renal Board Review Course
FSGS is a descriptive term for a pathologic
lesion, not a disease entity.
Etiological Classification of FSGS
Primary (idiopathic) FSGS
Permeability factor mediated?
S d FSGS
Brigham Renal Board Review Course
Secondary FSGS
1. Familial / genetic
MYH9, -actinin-4, TRPC6,
NPHS1&2, PLCE1, COQ2
2. Virus-associated
HIV, Parvo B-19, CMV, HTLV-1
3. Drug/toxin-mediated
Interferon- Sirolimus
4. Arising from adaptive
structural-functional responses
Reduced renal mass (e.g. renal
agenesis or dysplasia, reflux
nephropathy, cortical necrosis,
surgical ablation) Interferon , Sirolimus,
Bisphosphonates, Lithium,
Heroin
surgical ablation)
Initially normal renal mass (e.g.
hypertension, obesity, OSA,
sickle cell anemia,
atheroembolic disease)
Adapted from DAgati et al (2004) AJKD
Copyright Harvard Medical School, 2010. All Rights Reserved.
816
Histological Variants of FSGS
5 variants:
1 Classic
Brigham Renal Board Review Course
1. Classic
2. Perihilar
Frequently seen with secondary FSGS due to glomerular
hypertension
3. Tip lesion
Abrupt onset, high degree of proteinuria; better response to
steroids, some spontaneous remissions reported; ? less likely to
progress to ESRD
4. Cellular
5. Collapsing
More severe, poorer prognosis
Prognostic value of histological variants remains debated.
Mean age 40-50; male:female ~1:1
Primary FSGS
Clinical Presentation
Brigham Renal Board Review Course
Primary FSGS
Nephrotic range proteinuria: 60-75%
Microscopic hematuria: 30-50%
Hypertension: 45-65%
Impaired renal function: 25-50%
Secondary FSGS
Slowly progressive proteinuria and renal insufficiency
Peripheral edema and hypoalbuminemia uncommon even with Peripheral edema and hypoalbuminemia uncommon even with
nephrotic range proteinuria.
Heavy proteinuria uncommon.
Copyright Harvard Medical School, 2010. All Rights Reserved.
817
Presenting Features in FSGS
Children Adults
No. 459 665 No. 459 665
Nephrotic (%) 90 70
Male (%) 55 60
Hypertensive (%) 30 45
CP1177455-3
Hematuria (%) 55 45
Renal insufficiency (%) 20 30
Pathogenic Classification of FSGS
Primary alterations of GEC
Primary FSGS
HIV-associated nephropathy
Heroin-associated nephropathy Heroin associated nephropathy
Pamidronate
Familial FSGS
Sporadic genetic mutations
Reduced nephron mass/glomerular adaptation
Unilateral renal agenesis
Obesity-related glomerulopathy (+/- OSA)
CP1177455-4
Obesity-related glomerulopathy (+/- OSA)
Secondary to basement membrane defects
Secondary to focal proliferative glomerulonephritis
Aging kidney
Copyright Harvard Medical School, 2010. All Rights Reserved.
818
80
100
Kidney Survival on the Basis of Degree of
Proteinuria at Time of Renal Biopsy
(13)
(9)
(10)
(7) (3) (5)
20
40
60
80
P
A vs B 0.038
P
A vs B 0.038
% with
functioning
kidneys
(A) Asymptomatic proteinuria
<3.5 gm/24 hr
n=15
(B) Nephrotic proteinuria
3.5-14 0 gm/24 hr
n=38
(C) Massive
proteinuria
>14 gm/24 hr
n=10
(29)
(7) (3) (5)
(1)
(20)
(14)
(9) (5)
0
20
0 2 4 6 8 10 12 14
A vs C <0.001
B vs C 0.014
A vs C <0.001
B vs C 0.014
Time from biopsy (yr)
Velosa et al: Mayo Clinic Proc 58, 1983 CP1044973-1
( )
Renal Survival in Nephrotic Adults with FSGS
Based on Remission Status
100
Remission Remission
25
50
75
Survival
(%)
Survival
(%)
No remission No remission
CP1177455-7
Banfi, 1991; Cattran, 1998;
Schwartz & Korbet, 1999
Banfi, 1991; Cattran, 1998;
Schwartz & Korbet, 1999
0
0 2 4 6 8 10 12
Years Years
Copyright Harvard Medical School, 2010. All Rights Reserved.
819
Spontaneous remissions Spontaneous remissions Spontaneous remissions
are rare (<5%) and are most
often partial remissions
Spontaneous remissions
are rare (<5%) and are most
often partial remissions
CP1151810-4
Treatment in Primary FSGS
Proteinuric patient with primary FSGS
ACEi/ARB & BP control (125/75)
Non-nephrotic Nephrotic
CP1151810-5
Follow Begin
immunosuppressive
therapy
Copyright Harvard Medical School, 2010. All Rights Reserved.
820
Persistence of the
Steroid Resistance in Adults
Persistence of the
nephrotic syndrome after
4 months of prednisone
t d f 1 /k /d
CP1151810-19
at a dose of 1 mg/kg/day
Meyrier et al: Kidney Int, 1994
Membranous Nephropathy
CP1195760-12
Copyright Harvard Medical School, 2010. All Rights Reserved.
821
60-80% with nephrotic syndrome (NS)
remainder with subnephrotic proteinuria
Clinical Presentation
Brigham Renal Board Review Course
remainder with subnephrotic proteinuria
detected incidentally
~60% of those with subnephrotic range
proteinuria progress to NS within 1-2 years of
presentation
30-40% with microscopic hematuria p
Only 10-20% with HTN
<20% with significantly impaired renal function
Uniform thickening of glomerular capillary wall
b PAS / li ht i
Pathology
Brigham Renal Board Review Course
by PAS / light microscopy.
Craters and spikes in glomerular basement
membrane on silver stain
IF with diffuse, granular deposits of IgG
(predominantly IgG4) along basement
membrane membrane
Subepithelial electron dense deposits on EM
(stages described by Ehrenreich & Churg)
Copyright Harvard Medical School, 2010. All Rights Reserved.
822
Risk of Progression Categories
Low risk
Laboratory
Normal Function Normal Function
Proteinuria < 4 g/d
Medium risk
Normal function
Persistent proteinuria
> 4<8 g/d
High risk
Abnormal function and/or Abnormal function and/or
Persistent proteinuria
> 8 g/d
Specific Treatment
Low Risk - Normal renal function and Low Risk Normal renal function and
proteinuria < 4g/24h
~ 5% risk of progression
Treatment
i) reduce proteinuria
ii) idealize blood pressure ii) idealize blood pressure
iii) use ACEi-ARB
Continue to monitor
Copyright Harvard Medical School, 2010. All Rights Reserved.
823
Specific Treatment
Medium Risk - Normal renal function and
persistent proteinuria (4-
8g/24h) over 6 months
Corticosteroids alone
- ineffective
Corticosteroids + cytotoxic
effective - effective
Cyclosporine - effective
Specific Treatment
High Risk (10-15%) - >8g/24h High Risk (10 15%) >8g/24h
Proteinuria
Corticosteroids - ineffective
Progression
Cyclosporine - effective
C ti t id + t t i Corticosteroids + cytotoxic
- effective
Copyright Harvard Medical School, 2010. All Rights Reserved.
824
IgA Definition
Dominant or
CCo-dominant IgA deposition in glomeruli
Epidemiology (aka Bergers disease)
Most common cause of nephritis in the world (15 Most common cause of nephritis in the world (15
to 40% of primary GN in world, 20% of primary GN in
USA)
Males > Females (2:1)
Peak occurrence in 2nd & 3rd decades
Asian predominance (up to 40% of biopsies
compared with 20% in European/U S registries) compared with 20% in European/U.S. registries)
?screening bias
Copyright Harvard Medical School, 2010. All Rights Reserved.
825
Clinical Presentation
Synpharyngitic (24 to 48 hrs after URI or GI infection -- Synpharyngitic (24 to 48 hrs after URI or GI infection
in contrast with post-infectious nephritis 1 to 3 weeks)
Low grade fever, loin pain
Serum IgA levels elevated in ~50%
Spectrum of microscopic hematuria + low levels of
albuminuria to RPGN picture
Can be with or w/o Henoch-Schonlein Purpura (HSP)
Children: gross hematuria after URI Children: gross hematuria after URI
Adults: microscopic hematuria and/or proteinuria
Definition of IgA Phenotypes for
Treatment Purposes
Asymptomatic hematuria y p
Recurrent gross hematuria
Significant Proteinuria (>1g) and/or renal
insufficiency
Rapidly progressive glomerulonephritis
Nephrotic syndrome with minimal lesions
Acute Renal Failure
Henoch Schonlein Purpura
Copyright Harvard Medical School, 2010. All Rights Reserved.
826
Acute Renal Failure in IgAN
3% of 865 IgAN patients
25% of 32 pts had SCr >25%
Associated with macroscopic
hematuria and red blood
cells in tubules
4% patient developing
chronic renal failure after a
mean follow-up of 65
months.
p
of baseline
Duration of Macroscopic
Hematuria > 15 d
[OR 12.3; 1.06 to 143.5; P = 0.04]
Packham DClin Nephrol. 1994 Dec;42(6):349-53 Gutierrez E. Clin J Am Soc Nephrol. 2007 Jan;2(1):51-7.
Secondary IgA Nephropathy
HIV infection
T l i L Toxoplasmosis Leprosy
Celiac disease
Dermatitis herpetiformis
Crohn disease
Liver disease
Alcoholic cirrhosis
Ankylosing spondylitis
Reiter syndrome Reiter syndrome
Neoplasia Mycosis fungoides
Lung CA
Copyright Harvard Medical School, 2010. All Rights Reserved.
827
LEE Classification
From: Lee SM, Rao VM, Franklin WA, et al: IgA nephropathy: morphologic predictors of progressive
renal disease. Hum Pathol 13(4):314-322, 1982
Histologic grade Histopathologic features
I Minimal or no mesangial hypercellularity, without g yp y,
glomerular sclerosis, proliferative endocapillary or
extracapillary changes.
II Less than 50% of glomeruli show localized mesangial
proliferation and sclerosis. Rarely, small crescents.
III Diffuse mesangial proliferation with focal and
segmental variation. Occasional small crescents and
adhesions. adhesions.
IV Marked diffuse mesangial proliferation and sclerosis.
Crescents in 45% or less of glomeruli.
V Similar to IV, but more severe changes. Crescents in
more than 45% of glomeruli
Subclass Histopathologic features
HAAS Classification
From: Haas M.: Histologic subclassification of IgA nephropathy: a
clinicopathologic study of 244 cases. Am J Kidney Dis 29(6):829-842, 1997
I Minimal or no mesangial hypercellularity, without
glomerular sclerosis, proliferative endocapillary or
extracapillary changes
II Focal and segmental glomerular sclerosis, minimal
mesangial hypercellularity, no crescents or necrosis
III Focal proliferative GN; 50% or less of glomeruli show
mesangial or endocapillary hypercellularity, crescents,
or necrosis
IV M th 50% f l d l li IV More than 50% of non-sclerosed glomeruli are
hypercellular (mesangial hypercellularity, endocapillary
hypercellularity, crescents, or necrosis)
V 40% or more of glomeruli are globally sclerotic and/or
40% or more of estimated cortical tubular atrophy or
loss
Copyright Harvard Medical School, 2010. All Rights Reserved.
828
Key features that should be reported in pathology reports
for patients with IgAN
Eitner, F. and Floege, J. (2009) The Oxford classificationpredicting progression of IgAN
Nat. Rev. Nephrol. doi:10.1038/nrneph.2009.150
Difficulties in Treatment Studies in IgAN
Slow progression in many requires use of Slow progression in many requires use of
surrogate markers of progression
Variable rate of progression
Heterogeneous population- phenotype
Only a few RTC to define outcome of RX - Recent
meta analysis many of low quality and poorly
reported
Everyone knows how to treat some of the pts
Nobody is certain how to treat others
Copyright Harvard Medical School, 2010. All Rights Reserved.
829
Therapeutic Options
Blockade of the Blockade of the
Renin/Angiotensin/Aldosterone axis
Tonsillectomy
Glucocorticoids
Fish Oils
Immunosuppressives
Azathioprine + steroids
Cyclophosphamide + steroids
Mycophenolate mofetil
Specific Recommendations
Based on Phenotype
Asymptomatic microhematuria y p
Close monitoring
Recurrent gross hematuria
Tonsillectomy if documented tonsillitis
Significant Proteinuria (>1g) and/or renal
insufficiency
ACEi (+ ARB if tolerated)
Consider
Steroids alone Steroids alone
Cytotoxic + steroids
Mycophenolate + Steroids
Copyright Harvard Medical School, 2010. All Rights Reserved.
830
Specific Recommendations
Based on Phenotype
Rapidly progressive glomerulonephritis Rapidly progressive glomerulonephritis
Cytotoxic + Steroids
Nephrotic syndrome with minimal lesions
Steroids
Acute Renal Failure
Steroids if persistent gross hematuria
H h S h l i P Henoch Schonlein Purpura
Steroids +/- cytotoxics in high risk
ADPKD
Most common genetic disease
Incidence 1:500 1:1000 live births
Clinical Manifestations
abdominal mass
chronic flank or back pain
gross hematuria
recurrent UTI
nephrolithiasis (uric acid stones)
Copyright Harvard Medical School, 2010. All Rights Reserved.
831
Autosomal Dominant Polycystic
Kidney Disease
Mutation in ~70% located on short arm of
chromosome 16p 13.3 p13.1 (PKD1 locus)
~30% of mutation is located on chromosome 4q21-
q23 milder phenotype with PKD2 locus. Later age of
diagnosis and hypertension, smaller kidney volume,
fewer kidney cysts later age of ESRD system: fewer kidney cysts, later age of ESRD system:
disorder is kidney, liver, pancreatic and vascular
abnormalities.
Ultrasound Criteria for Diagnosis of PKD1 in
At-Risk Individuals
Positive and negative predictive values 97-100% Positive and negative predictive values 97 100%
Ravine et al, Lancet 343:824, 1994
Age < 30: at least 2 cysts (unilateral or bilateral)
Age 30-59: at least 2 cysts/kidney
Age > 60: at least 4 cysts/kidney
For PKD2 age 30 59 use 4 or more cysts in both For PKD2 age 30-59, use 4 or more cysts in both
kidneys for sensitivity of 96%
Pei et al, JASN 14:107A, 2003
Copyright Harvard Medical School, 2010. All Rights Reserved.
832
Progressive Loss of Kidney Function
Rate of decline of GFR (data from MDRD study,
starting at GFR <55 ml/min) starting at GFR <55 ml/min)
Males 5 - 6 ml/min/year
Females 4 - 5 ml/min/year
Pattern of GFR loss has recently been
established by CRISP study
GFR stable for many years, despite progressive
increase in total kidney volume
GFR decrease not detected until total kidney
volume exceeds 1500 ml
Hateboer et al, THE LANCET 353:103, 1999
Copyright Harvard Medical School, 2010. All Rights Reserved.
833
Consortium for Radiologic Imaging Studies of
Polycystic Kidney Disease (CRISP)
Study Completed
Observational trial (no intervention) to
determine how to assess changes in
polycystic kidneys over a relatively short
period of time (3 years)
232 subjects with normal kidney function 232 subjects with normal kidney function
Emory; U. of Alabama; Kansas University;
Mayo
Kidney International, 64: 10351045, 2003
Kidney International, 64: 22142221, 2003
Principal Extrarenal Manifestations
Hepatic and pancreatic cysts
Asymptomatic in many patients, but can expand and cause pain and Asymptomatic in many patients, but can expand and cause pain and
infection; rarely massive PLD
Cardiac valvular abnormalities
Mitral, tricuspid and aortic valve prolapse and regurgitation
Intracranial aneurysms
Risk of rupture; found in approximately 5% of patients with no family
history and about 22% of patients with family history of ICA or SAH
Seminal vesicle cysts
Found in ~39-60% of men; undefined risk of infertility
Copyright Harvard Medical School, 2010. All Rights Reserved.
834
Treatment of ADPKD (1)
There is no specific therapy
Pain
Diff ti l di i bl d i f ti Differential diagnosis: bleed vs. infection vs.
obstruction vs. stone
Analgesics
Percutaneous drainage; laparoscopic or
surgical unroofing of individual cysts
Infection: lipophilic antibiotics p p
Hypertension
ACE inhibitors thought to be beneficial
Treatment of ADPKD (2)
Progressive kidney insufficiency Progressive kidney insufficiency
Lack of proven benefit of low protein diets or ACE-I
Cyst decompression does not alter progression
Renal replacement therapy
Extrarenal manifestations
Intervene as needed for symptoms
Screen for cerebral aneurysms with + family history; Screen for cerebral aneurysms with + family history;
antibiotic prophylaxis for valvular regurgitation
Avoid estrogen/progesterone in women (effect on liver cyst
disease)
Copyright Harvard Medical School, 2010. All Rights Reserved.
835
Nephrolithiasis in ADPKD
Occurs in ~20% of patients
Uric acid and/or calcium oxalate
Predisposing factors include hypocitraturia,
hyperoxaluria, hyperuricosuria, hyperuricemia,
hypercalciuria, possible distal acidification
defects
Expanding cysts compress the collecting
system producing urinary stasis which system producing urinary stasis, which
predisposes to stone formation and infection
Treatment of Kidney Cyst Infection in
ADPKD
Lipophilic antibiotics such as ciprofloxacin Lipophilic antibiotics such as ciprofloxacin,
norfloxacin, trimethoprim, chloramphenicol
Percutaneous or operative drainage is rarely
needed; only for refractory infection
Resistant organisms
Localization of infected cyst is difficult
Labeled WBC or gallium scan Labeled WBC or gallium scan
MRI with contrast
PET scan
Copyright Harvard Medical School, 2010. All Rights Reserved.
836
Kidney Infection in ADPKD
30 to 50% of patients with ADPKD will have a urinary 30 to 50% of patients with ADPKD will have a urinary
tract infection, either pyelonephritis or cyst infection,
during their lifetime
Urinary tract infections are more common in women
with ADPKD
Fever and flank pain are the presenting symptoms
Urine culture may be negative in cyst infection, as
cysts frequently dont communicate with the
collecting system
Hematuria in ADPKD
Cyst hemorrhage occurs in ~60% of individuals
gross or microscopic hematuria if cyst connects to
collecting system
intracyst or subcapsular hemorrhage without
hematuria
Excessive angiogenesis results in fragile blood
vessels stretched across walls of enlarging
cysts; susceptible to minor trauma with resultant
h h hemorrhage
Patients with recurrent episodes of gross
hematuria have the largest kidneys and
progress more quickly to kidney failure
Copyright Harvard Medical School, 2010. All Rights Reserved.
837
Treatment of Hematuria in ADPKD
Appropriate diagnosis and treatment of specific Appropriate diagnosis and treatment of specific
entity, such as infection or stone
Correction of coagulopathy, if present
Conservative management with hydration, bed
rest, and appropriate use of analgesics
R l i bl di i Rarely, massive bleeding may require
transfusion, or kidney embolization or
nephrectomy
Pregnancy and Renal Disease
Copyright Harvard Medical School, 2010. All Rights Reserved.
838
Mild Renal Insufficiency
(Creatinine 1.4 mg/dl)
Data from several small series with mild renal
insufficiency at baseline: Minimal to no impact
of pregnancy in terms of acceleration renal
insufficiency.
Moderate to Severe Renal Insufficiency
Creatinine 1.4 mg/dl
Largest Series: 82 pregnancies in 67 women with creatinine 1.4 at
conception (mean creatinine 1.9)
Mean creatinine increased to 2.5 by 3
rd
trimester
20% had worse renal function during pregnancy
23% had worse renal function by 6 wks post-partum
8% recovered; 10% declined further
8 women (10%) ESRD within year of pregnancy 8 women (10%) ESRD within year of pregnancy
Creatinine >2.0 outset: Highest likelihood decline (33% had
accelerated decline renal function)
Jones DC, Hayslett JP. N Enl J Med 1996;335:226-32.
Copyright Harvard Medical School, 2010. All Rights Reserved.
839
Impact of Renal Disease on Pregnancy
Outcomes
Fetal Survival Good (70 to 100%)
Lower with uncontrolled hypertension
Especially with MAB > 105 start of conception
Risk of Prematurity estimated 45 -70%
Risk of Intrauterine Growth Restriction estimated 20-50%
Risk of Preeclampsia increased
Greater risk of occurrence during second trimester g
Associated with both prematurity and intrauterine growth
restriction
Preeclampsia
Systemic disorder unique to pregnancy marked by new Systemic disorder unique to pregnancy marked by new
onset hypertension and proteinuria after 20 weeks
gestation
Eclampsia: onset of seizures in preeclampsia
Preeclampsia may be superimposed on chronic
hypertension in pregnancy
I id i 3 14% i ld id Incidence is 3-14% pregnancies worldwide
Copyright Harvard Medical School, 2010. All Rights Reserved.
840
Risk Factors
First pregnancy past history preeclampsia family history First pregnancy, past history preeclampsia, family history
preeclampsia, multiple gestation, obesity, pre-gestational
diabetes, pre-existing renal disease, pre-existing
hypertension, advanced maternal age, antiphospholipid
antibody syndrome.
Cigarette smokers have lower risk preeclampsia v. nonsmakers
Severe Preeclampsia
Defined as preeclampsia accompanied by at Defined as preeclampsia accompanied by at
least one of following:
cns dysfunction; liver capsule distension; hepatocellular
injury (transaminases > twice normal); marked blood
pressure elevation; thrombocytopenia, marked
proteinuria ( 5 g/24 hrs); oliguria; severe fetal growth
restriction; pulmonary edema; stroke ; p y ;
ACOG Practice Bulletin #33, January 2002
Copyright Harvard Medical School, 2010. All Rights Reserved.
841
Preeclampsia: General Management
Principles
Definitive treatment is delivery
Prevents maternal or fetal complications
Timing of deliverydepends on preeclampsia severity,
gestational age, maternal and fetal status
Corticosteroids should be given at 24-34 weeks gestation in
preeclampsia
P t f t l l t ti Promotes fetal lung maturation
Severe preeclampsia: delivery indicated regardless of
gestational age
Preeclampsia:
Seizure prophylaxis
Magnesium sulfate: drug of choice for preventing
eclampsia
Evidence suggests use reduces incidence of eclampsia
But no effect on disease progression aside from seizure
prevention
Dose for renal insufficiency
Observe for toxicity (loss of deep tendon reflexes respiratory Observe for toxicity (loss of deep tendon reflexes, respiratory
paralysis, cardiac arrest)
Calcium gluconate---antidote for toxicity
Copyright Harvard Medical School, 2010. All Rights Reserved.
842
Preeclampsia: Hypertension Treatment
Lack of data about level of blood pressure to treat
Does not alter the course of preeclampsia
Concerns about deleterious effects of blood pressure
lowering on fetal growth need to be weighed with
concern for possible maternal vascular complications
(especially stroke)
Usually treat systolic >150-160; diastolic > 100-105
Medications for acute blood pressure lowering: IV
labetalol, hydralazine (less commonly diazoxide)
Avoid sodium nitroprussidepossible fetal cyanide poisoning
Mild Preeclampsia Management
Frequent monitoring, including labs
Deliver by 38 weeks
unless severe preeclampsia or abruption
placentae, marked fetal growth restriction, non-
i f l i li h d i reassuring fetal testing, or oligohydramnios
develop
Copyright Harvard Medical School, 2010. All Rights Reserved.
843
Preeclampsia Prevention
No clearly established preventive
measures
Some data (very small studies) suggest
possible benefit of low-dose aspirin in
prevention in high risk groups
But weight of evidence not strong
No benefit from calcium supplementation
Other Forms Hypertension in
Pregnancy
Chronic hypertension: Antedates pregnancy (systolic
140 and diastolic 90 mmHg)
Limited data regarding degree of hypertension to treat and target
goals
Expert opinion: treat bp > 155-165/95-100 mmHg
Aim for lower levels if end-organ damage present
Gestational hypertension: mild hypertension latter half
pregnancy (but no proteinuria or other signs
preeclampsia)
Copyright Harvard Medical School, 2010. All Rights Reserved.
844
Antihypertensive Drugs in Pregnancy
Drugs of choice:
Methyldopa: nearly 50 years experience; excellent
long-term safety profile in pregancy
Labetalol: both alpha and beta-adrenergic blocking
properties; may have less impact on uterine/placental
blood flow blood flow
Nifedipine (long-acting): appears safe
Less data on Diltiazem and Verapamil but likely safe
May worsen hypotension and neuromuscular blockade if
magnesium sulfate used for preeclampsis
Antihypertensive Drugs in Pregnancy
Drugs to avoid:
ACE inhibitors and ARBs
First trimester exposure: higher risk major congenital
malformations, esp. cardiovascular and cns malformations
Later pregnancy exposure: oligohydramnios, hypocalvaria
Drugs to use with caution:
Diuretics: avoid volume depletion Diuretics: avoid volume depletion
Beta blockers: may uterine/placental blood flow; may be linked
to intrauterine growth restriction and fetal bradycardia
Copyright Harvard Medical School, 2010. All Rights Reserved.
845
Disclosures
Consulting Consulting
Amgen, Johnson and Johnson, Sandoz, Fibrogen
Grant Support
Amgen, Johnson and Johnson, Roche
Advisory Boards
Rockwell Rockwell
Copyright Harvard Medical School, 2010. All Rights Reserved.
846
Nephr ol ogy
Boar d Revi ew
Ajay K. Singh, M.D.
Associate Professor of Medicine, Harvard Medical
School
Physician, Brigham and Womens Hospital
Di sc l osur es
Consulting Consulting
Amgen, Johnson and Johnson, Sandoz,
Fibrogen
Grant Support
Amgen, Johnson and Johnson, Roche
Advisory Boards
Rockwell
Copyright Harvard Medical School, 2010. All Rights Reserved.
847
Quest i on 1
67 year old man - 1 week history of anorexia 67 year old man - 1 week history of anorexia,
nausea, lassitude, and pedal edema.
Longstanding hypertension, well controlled with
hydrochlorothiazide and amlodipine.
Fenoprofen for osteoarthritis of the hip for the
past 3 months past 3 months.
Quest i on 1 c ont d
Physical examination
BP 142/68mm, HR 72 bpm, Temp of 97.8
O
F. p p
JVP 8 cm; normal cardiac and pulmonary
examinations; and 2+ pitting edema.
Urinalysis showed a specific gravity of 1.017,
protein 4+, no blood, and negative for glucose.
Microscopic examination of the sediment was
bland.
Copyright Harvard Medical School, 2010. All Rights Reserved.
848
BUN 18 mg/dL; Cr 0.8 mg/dL; Na 137, K
Quest i on 1 c ont d
4.4, Cl 95, C02 21, Ca 9.2, Phos 3.6, UA
4.6 mg/dL; Alb 2.9 g/dL; HCT 38%. anti-
dsDNA antibody level 0.
24 h protein excretion 7.7 g.
Renal ultrasound showed normal sized
kidneys bilaterally without obstruction kidneys bilaterally without obstruction.
The renal biopsy on light microscopy will most
Quest i on 1
The renal biopsy, on light microscopy, will most
probably show:
A). Thickened loops with evidence of double
contours
B). Swollen endothelial cells -- glomerular
endotheliosis
C.) No changes
D.) Focal segmental sclerosis
E.) Mesangial proliferation
Copyright Harvard Medical School, 2010. All Rights Reserved.
849
PAS Stain x 400
Copyright Harvard Medical School, 2010. All Rights Reserved.
850
NSAI Ds and t he Ki dney
Prerenal azotemia
Ischemic acute tubular necrosis Ischemic acute tubular necrosis
Allergic interstitial nephritis (AIN)
AIN plus minimal change nephropathy
ARF plus bilateral flank pain
Sodium and water retention
Hyperkalemia
CRF and papillary necrosis
Whelton and Hamilton, J Clin Pharm 31: 588, 1991
Ac ut e I nt er st i t i al Nephr i t i s
Dr ugs
ANTIBIOTICS
Penicillins, cephasporins, others (quinolones, Bactrim)
DIURETICS
Thiazides, furosemide
ANALGESICS
NSAIDS
ANTICONVULSANTS
phenytoin, carbamazepine, phenobarbitol
OTHERS
allopurinol, cimetidine
Copyright Harvard Medical School, 2010. All Rights Reserved.
851
Cl i ni c al Syndr omes
Drug Induced AIN Drug Induced AIN
Variable
Full-blown hypersensitivity reaction to
asymptomatic increase in Scr
Nephritic / Nephrotic / AIN Urine sediment
Fanconis syndrome, hyperkalemeic
hyperchloremic metabolic acidosis,
Nephrogenic DI
Di agnost i c f eat ur es
Urinalysis Urinalysis
Impaired concentrating ability (SG, low urine osmolality)
Leukocytosis eosinophilia (10-30%) common
Increased IgE
Urine eosinophiluria
Gallium high sensitivity, low specificity Gallium high sensitivity, low specificity
Renal Biopsy
Copyright Harvard Medical School, 2010. All Rights Reserved.
852
Eosi nophi l ur i a
>1% of white cells in urine stained
Wrights and Hansels Stain
Both use eosin-methylene blue combinations
CAUSES
Acute Interstitial Nephritis
UTI
pyelonephritis
Both use eosin methylene blue combinations
Hansels 4 fold more sensitive, less pH dependent
Sensitivity 67%, specificity 80%
pyelonephritis
prostatitis
Cystitis
Glomerulonephritis
Atheroembolic disease
Nolan et al, NEJM 1986
Hansel s st ai n
Hansels stain
Eos stain bright red
Copyright Harvard Medical School, 2010. All Rights Reserved.
853
Dr ug I nduc ed AI N
NSAID-induced AIN
+ Nephrotic Syndrome
Methicillin-induced AIN NSAID-induced AIN
Rossert et al KI 60: 804-817, 2001
NSAI D assoc i at ed AI N
Heterogenous picture Heterogenous picture
Typically, no hypersensitivity reaction
Usually middle-aged or elderly
Majority: associated nephrotic syndrome (MCD)
Pyuria, hematuria, eosinophiluria (less than
methicillin-associated)
Copyright Harvard Medical School, 2010. All Rights Reserved.
854
Nephr ot i c Syndr ome & AKI
Characteristics: Characteristics:
Elderly
HTN
Severe Proteinuria
Severe Hypoalbuminemia
Severe Edema
M>F
Fenoprofen higher risk
Mi ni mal Change Di sease & AKI
Reversible renal failure in MCD was first reported in the Reversible renal failure in MCD was first reported in the
1960s by Chamberlain, Pringle and Wrong in QJM and
by Conolly, Wrong and Jones in Lancet.
Reduced Oncotic Pressure
Hypotension
Hypovolemia
Copyright Harvard Medical School, 2010. All Rights Reserved.
855
Mi ni mal Change Di sease & AKI
Etiologies Include: Etiologies Include:
ATN (60%-70%)
AIN
Renal Vein Thrombosis
Interstitial edema
Lowenstein, Schacht, Baldwin, AJM, 1981
R t ti i f 15 t / MCD&ARF Retrospective review of 15 pts w/ MCD&ARF
Postulated interstitial edema produced an in in
hydrostatic pressure in prox tubules and Bowmans
space leading to dec GFR
Quest i on 2
A 27 year old man with the acquired immunodeficiency syndrome
(AIDS) is hospitalized with a cough, fever, and a pulmonary infiltrate
on CXR Therapy is initiated with trimethoprim-sulfamethoxazole on CXR. Therapy is initiated with trimethoprim sulfamethoxazole.
On admission, the serum creatinine is 1.6 mg/dL and blood urea
nitrogen, 21 mg/dL; On re-examination 3 days later, the serum
creatinine is 2.2 mg/dL and blood urea nitrogen, 23 mg/dL. Results
of urinalysis both on admission and 3 days later are normal. Urine
output on day 3 is 1350 mL. The most likely cause of the increased
creatinine is:
A.) AIDS glomerulopathy
B ) Trimethoprim-mediated decrease in creatinine secretion B.) Trimethoprim-mediated decrease in creatinine secretion
C.) Intratubular obstruction secondary to sulfonamide
D.) AIN caused by trimethoprim-sulfamethoxazole therapy
E.) Acute tubular necrosis secondary to sepsis
Copyright Harvard Medical School, 2010. All Rights Reserved.
856
Tr i met hopr i m-Sul f amet hox azol e
Elevated creatinine, no change in BUN, no
evidence of ARF: inhibition of tubular secretion
Allergic interstitial nephritis with fever, rash, and
eosinophilia induced by sulfa moiety
Hyperkalemia with salt wasting due to amiloride-
like action of trimethoprim
Rarely, crystallization of sulfamethoxazole
metabolite and renal stone formation
Copyright Harvard Medical School, 2010. All Rights Reserved.
857
Quest i on 3
A previously healthy 26 year old Japanese-American
man becomes ill with fever (temperature, 38C [100.4]),
malaise, myalgias, and a sore throat. Approximately 8
hours later, gross hematuria and flank pain begin. His
past medical history is notable for several prior episodes
of gross hematuria. Urinalysis shows protein, 3+ and
erythrocyte casts. His blood urea nitrogen (BUN) is 28
mg/dL, serum creatinine is 1.3 mg/dL. Electrolytes are
within normal limits. Serological testing reveals normal
complements, a normal IgA level, a 1:40 ANA, anti-DNA
ab level of 0 and negative ASLO and ANCA titers. His
anti-GBM titers are also negative. Which of the following
is the most likely diagnosis:
Quest i on 3
A.) WHO class IV lupus nephritis
B.) IgA nephropathy
C.) Rapidly progressive glomerulonephritis
secondary to Wegeners granulomatosis
D.) Goodpastures syndrome ) p y
E.) Post-streptococcal glomerulonephritis
Copyright Harvard Medical School, 2010. All Rights Reserved.
858
HSP and I gAN w i t h age
Copyright Harvard Medical School, 2010. All Rights Reserved.
859
I gAN c l i ni c al pr esent at i ons
Macroscopic Hematuria
80% pediatric pts, 40-55% adults
synpharyngitic
30% fl k / l i i 30% flank / loin pain
recurrent gross hematuria
varying levels of proteinuria
slower progression
Microscopic Hematuria
persistent
t i i proteinuria
36% hypertension, 27-43% azotemic
faster progression
RPGN
Nephrotic syndrome
Hi st ol ogi c Subc l asses of
I gAN (Haas, 1997)
Class I Minimal Histologic lesion
Mi i l i l ll l it Minimal mesangial cellularity
Class II FSGS-like
Minimal mesangial cellularity, no cresc
Class III Focal proliferative GN
<50% glos hypercellular, +/- cresc
Class IV Diffuse proliferative GN
>50% glos hypercellular, +/- cresc
Class V Advanced chronic GN Class V Advanced chronic GN
>40% global scl, >40% tubular atrophy
Copyright Harvard Medical School, 2010. All Rights Reserved.
860
Key features that should be reported in pathology reports for patients
with IgAN
Eitner, F. and Floege, J. (2009) The Oxford classificationpredicting progression of IgAN
Nat. Rev. Nephrol. doi:10.1038/nrneph.2009.150
Renal Sur vi val by I gAN c l ass
0.8
1.0
I & II
0.2
0.4
0.6
V
III
All cases
20 40 60 80 100 120 140 160
Time from biopsy (mo)
0.2 V
IV
Copyright Harvard Medical School, 2010. All Rights Reserved.
861
Quest i on 4
A 52 year old African-American female presents to the
emergency room with unstable angina. She is noted to
have a past medical history of mild chronic renal
insufficiency ( creatinine of 1 8 mg/dL) She is insufficiency ( creatinine of 1.8 mg/dL). She is
transferred to the coronary care unit and therapy for
her unstable angina is initiated. A cardiac
catheterization is planned for the next day. Risk factors
that would predispose this woman to contrast
nephrotoxicty include all of the following except:
A.) Diabetes mellitus
B.) Pre-existing renal insufficiency
C.) The volume of IV contrast utilized in the procedure
D.) Presence of extracellular volume contraction
E.) A history of coronary artery disease
Copyright Harvard Medical School, 2010. All Rights Reserved.
862
Quest i on 5
Her cardiologist asks you to recommend a strategy to minimize the
risk of contrast nephrotoxicity All of the following would be risk of contrast nephrotoxicity. All of the following would be
reasonable strategies except:
A.) Start the patient on 0.45% saline and order furosemide 40
mg IV and 10 g mannitol on call to the procedure
B.) Start the patient on 0.9% saline at 1 ml/kg for 12 hours prior to
the procedure
C.) Start the patient on N-acetyl cysteine 1200 mg in divided doses
and pre-hydrate the patient
D.) Stop the ibuprofen the patient is taking for her arthritis
E.) Start isotonic sodium bicarbonate solution IV at 1ml/kg for 12
hours prior to the procedure
F.) Use low osmolality ionic contrast
Copyright Harvard Medical School, 2010. All Rights Reserved.
863
Cont r ast Nephr ot ox i c i t y
(ver sus at her oembol i )
ARF and oliguria within 24-48 hours ARF and oliguria within 24-48 hours
Peak serum creatininine on days 3-5
Low fractional excretion of sodium
Benign sediment or granular casts
Resolution usual within 1 week
Risk factors: CRI, diabetic nephropathy,
dose>120 cc, multiple myeloma, volume
Cont r ast Nephr opat hy
Incidence ARF: 6.4%
Mortality: 30-70%
Radiocontrast nephropathy (RCN)
3rd most common form of ARF 13%
mortality: 29%
Potentially preventable
Copyright Harvard Medical School, 2010. All Rights Reserved.
864
Contrast Nephropathy Prophylaxis
Options:
1.Hydration 0.45% or normal saline (Solomon,
NEJM 1994) NEJM 1994)
1.N-acetyl cysteine
1.Sodium bicarbonate
The image cannot be displayed. Your computer may not have enough memory to open the image, or the image may have been t d R t t t d th th fil i I f
The image cannot be displayed. Your computer may not have enough memory to open theimage or theimagemay havebeen
The image cannot be displayed. Your computer may not have enough memory to open the image, or the image may have been corrupted. Restart your computer, and then open the file again. I f the red x still appears, you may have to delete the image and then insert it again.
The image cannot be displayed. Your computer may not have enough memory to open the image, or the image may have been corrupted. Restart your computer, and then open the file again. I f the red x still appears, you may have to delete the image and then insert it again.
corrupted. Restart your computer, and then open the file again. I f the red x still appears, you may have to delete the image and then insert it again.
The image cannot be displayed. Your computer may not have enough memory to open the image, or the image may have been corrupted. Restart your computer, and then open the file again. I f the red x still appears, you may have to delete the image and then insert it again.
The image cannot be displayed. Your computer may not haveenough memory to open the image, or the image may have been corrupted. Restart your computer, and then open the file again. I
The image cannot be displayed. Your computer may not haveenough memory to open the image, or the image may have been corrupted. Restart your computer, and then open the file again. I
enough memory to open theimage, or theimagemay havebeen corrupted. Restart your computer, and then open the file again. I f the red x still appears, you may have to delete the image and then insert it again.
The image cannot be displayed. Your computer may not have enough memory to open the image, or the image may have been corrupted. Restart your computer, and then open the file again. I f the red x still appears, you may have to delete the image and then insert it again.
Solomon et.al. NEJM 1994
Copyright Harvard Medical School, 2010. All Rights Reserved.
865
Pr event i on of Cont r ast -I nduc ed Nephr opat hy Wi t h
Sodi um Bi c ar bonat e: A Randomi zed Cont r ol l ed Tr i al
Study Objective: To determine whether intravenous infusion of
isotonic NaHCO3 solution is superior to 0.9% NS hydration
in the prevention of RCN
Study Design: Prospective randomized, controlled trial of 119
patients undergoing a non-emergent exposure to iodinated
contrast
Methods:
Patients received 154 mEq/L of NS or Na
+
HCO
3
at
bolus of 3 mL/kg per hour for 1 hour before contrast
1 mL/kg per hour for 6 hours post procedure.
Contrast Agent: Iopamidol (796 mOsm/L) Contrast Agent: Iopamidol (796 mOsm/L)
Entry Criteria: Serum Cr > 1.1 mg/dl
Exclusion: Patients receiving dopamine, FNP, N-AC,
or mannitol
Primary endpoint: Rise in serum Cr by 25% 48 hrs
Merten et.al JAMA 291 (19) 2328-2334, 2004
Pr event i on of Cont r ast -I nduc ed Nephr opat hy Wi t h
Sodi um Bi c ar bonat e: A Randomi zed Cont r ol l ed Tr i al
Sodium Chloride
Hydration
Cr=1.71 mg/dl
Sodium Bicarbonate
Hydration
Cr=1.89 mg/dl
Merten et.al JAMA 291 (19) 2328-2334, 2004
Copyright Harvard Medical School, 2010. All Rights Reserved.
866
Pr event i on of Cont r ast -I nduc ed Nephr opat hy Wi t h
Sodi um Bi c ar bonat e: A Randomi zed Cont r ol l ed Tr i al
R
P<0.02 -0.1 ml/min
%

C
h
a
n
g
e

i
n

G
F
R
P 0.02
+ 8.5
ml/min
0.1 ml/min
Merten et.al JAMA 291 (19) 2328-2334, 2004
Sodium Chloride
Hydratzion
Sodium Bicarbonate
Hydration
%
Ef f ec t of Ac et yl cyst ei ne on CN
Tepel et al , NEJ M: 343: 180-4, 2000
RCT n=83 baseline CRI (Scr 2 4 + 1 3 mg/dL) RCT, n=83, baseline CRI (Scr 2.4 + 1.3 mg/dL)
antioxidant acetylcysteine (600 mg twice daily) or
placebo
All patients hydrated with 0.45% saline before and after;
used non-ionic low osmolality agent
Mean Scr at 48 h decreased in acetylcysteine group (2.5
+ 1.3 to 2.1 + 1.3 mg/dL, P<0.001) vs. increasing in
placebo group (2.4 + 1.3 to 2.6 + 1.5 mg/dL, P<0.001 CN
in Rx group 56% vs. placebo 29%, P=0.18)
Copyright Harvard Medical School, 2010. All Rights Reserved.
867
Ant i -Ox i dant Ther apy Reduc es I nc i denc e of
Radi oc ont r ast Nephr opat hy
21% RCN
2.4 2.6 2.5 2.1
2.0% RCN
1/2 NS Acetylcysteine
Tepel et.al. NEJM 2000
Saline
NAC
N-Ac et yl c yst ei ne f or t he Pr event i on of RCN:
A Revi ew of Pr ospec t i ve Cont r ol l ed Tr i al s
Presented Abstract ASN, San Diego, 2003
Copyright Harvard Medical School, 2010. All Rights Reserved.
868
Quest i on 6
All of the following statements regarding aminoglycoside
nephrotoxicity are true, except:
A ) Aminoglycosides cause a secondary A.) Aminoglycosides cause a secondary
phospholipidosis in lysosomes
B.) Aminoglycosides are proximal tubule toxins
C.) Hypokalemia increases the risk of aminoglycoside
nephrotoxicity
D.) Hypokalemia and hypomagnesemia are
commonly observed in patients with aminolglycoside y p g y
nephrotoxicty
E.) Multidose regimen is preferred to single dose
regimens in order to reduce aminoglycoside
nephrotoxicity.
Copyright Harvard Medical School, 2010. All Rights Reserved.
869
Ami nogl yc osi de Nephr ot ox i c i t y
25% incidence with therapeutic levels
l ti ith l ti d some correlation with cumulative dose
non-oliguric ARF after 5-10 days
Toxicity correlates with cationic charge
Pathology: membrane phospholipidosis
Prevention: once daily dosing y g
Prine et al: Lancet 341: 335, 1993
Levinson Ann Intern Med 117: 694, 1992
48 year old male ESRD patient presents to the
ED with a K= 7 8 mEq/L HC03 of 22 His EKG
Quest i on 7
ED with a K= 7.8 mEq/L, HC03 of 22. His EKG
shows peaked T waves. Recommended initial
treatment include all of the following EXCEPT:
A.) Calcium gluconate 10 mls, IV
B.) Insulin 10 units and 1 amp of 50% dextrose
C.) Albuterol nebulizer (10-20 mg)
D.) IV bicarbonate 8.4%, 1 to 2 amps IV
E.) Emergent dialysis
Copyright Harvard Medical School, 2010. All Rights Reserved.
870
Changes i n pl asma K
8.4% bi c ar b, epi nephr i ne, i nsul i n/dex t r ose, or HD
Blumberg et al
Am J. Med 88:507-512, 1988
Copyright Harvard Medical School, 2010. All Rights Reserved.
871
Changes i n pl asma K dur i ng I V i nf usi on
of bi c ar bonat e i n HD pat i ent s
Values= means + SE
*P<0.5, + P<0..01 vs. baseline
Blumberg et al KI, 41: 369-374, 1992
Quest i on 8
21yr female with 8 day Hx of fever, chills, malaise, nausea, dark urine.
She is initially treated for UTI with ciprofloxacin. SCr 1.2 2.8 over 3
days; t/f to BWH Past History G1 P1; dental abscess drained 2 wks
previously Social history: 2 cigs/day; hairdresser 5 units Etoh/wk previously. Social history: 2 cigs/day; hairdresser, 5 units Etoh/wk
Review of systems: no rash, no arthralgia, no increased menstrual
blood loss; +mild SOB. Examination: P 94, BP 130/70, RR 18, afebrile,
O2 sat 97% (room air). Conjunctiva pale. Resp: BS normal - few bibasal
crackles. Cardiac, abdominal examination unremarkable; no leg edema.
Lab data: UA 3+blood, 2+protein. Used. multiple RBCs (some
dysmorphic), few WBCs, no casts. BUN 32, K 4
.
7, Cr 3
.
9. WBC 8
.
2, Hct
23, MCV 74, Plts 283. Normal complements, ANA negative. CXR:
bilateral lower zone interstitial infiltrates.
Th t lik l di i i The most likely diagnosis is:
A.) Churg Strauss syndrome
B.) Lupus nephritis
C.) Post-infectious GN
D.) Anti-GBM nephritis
E.) Mixed essential cryoglobulinemia
Copyright Harvard Medical School, 2010. All Rights Reserved.
872
Di f f er ent i al dx i n t hi s 21yr f emal e
w i t h ARF, ac t i ve ur i ne sedi ment ,
abnor mal CXR
1. Post-infectious GN (with pulmonary edema) os ec ous G ( pu o a y ede a)
2. Systemic vasculitis:
Microscopic polyarteritis
Wegeners granulomatosis
SLE
Other systemic vasculitides (HSP, cryoglobulin
disease) disease)
3. Anti-GBM disease
4. Legionella / Mycoplasma sp. pneumonia
5. Other?
Copyright Harvard Medical School, 2010. All Rights Reserved.
873
Hypoc ompl ement emi a i n
Gl omer ul ar Di sease
Pathway Complement Disease
Classical Low C3, C4, CH50 Lupus Nephritis
Mixed essential
cryo
Alternate Low C3, Normal C4 Post-strep GN
Post-infect GN
SBE
Shunt
Hep B
MPGN type 2 MPGN type 2
Reduced synthesis Acquired Liver Disease
Hereditary (C2 def) Lupus
Ant i -GBM Nephr i t i s - Pat hol ogy
Copyright Harvard Medical School, 2010. All Rights Reserved.
874
I ni t i al Management
1g of iv methyprednisolone
12 hrs later, renal bx performed 12 hrs later, renal bx performed
Light microscopy: severe crescentic GN (90% of
glomeruli involved); marked inflammation of the
tubulo-interstitium. IF showed intense linear
staining of the glomerular basement membranes
for IgG.
Cr now 5
.
2
Serum aGBM titer by ELISA: 140 [ref 0-5]; ANCA
neg.
Fur t her Management
Started on Hammersmith protocol of Lockwood et al.
(Lancet 1976)
Methylpred 1g iv x3days; then prednisone 1mg/kg/day Methylpred 1g iv x3days; then prednisone 1mg/kg/day
Cyclophosphamide 3mg/kg/day po
Plasmapheresis daily x14days; 4L exchanges (albumin +
FFP).
Day 8 of hospital stay:
Nausea, vomiting; urine output 800ml/24hrs; persistent
anemia despite RBC transfusions (Hct 23.9); BUN 122, Cr 7.6;
serum aGBM titer by ELISA: 20 [ref 0-5]
Copyright Harvard Medical School, 2010. All Rights Reserved.
875
Whi c h of t he f ol l ow i ng def i nes t he
best f ur t her t r eat ment st r at egy?
A St i i d l h i i k t A. Stop immunosuppression and plasmapheresis - risks to
patient (opportunistic infection, neoplasia, ovarian
damage) now outweigh potential benefits
B. Stop immunosuppression but continue plasmapheresis
until aGBM serology negative
C Continue immunosuppression and plasmapheresis C. Continue immunosuppression and plasmapheresis
D. Add rituximab
Copyright Harvard Medical School, 2010. All Rights Reserved.
876
I ndi c at i ons f or w i t hol di ng
i mmunosuppr essi on i n ant i -GBM di sease
Levy et al . Ann I nt er n Med, 2001
Renal limited disease with:
Dialysis dependency (Cr > 7.0 approx.) and severe,
irreversible disease on biopsy (e.g. 100% crescents)
Exceptions:
Al l h h Alveolar hemorrhage
Very acute renal damage (based on Hx & histology)
ANCA+ and antiGBM+
Fol l ow -up
Anemia presumed multi-factorial in origin; further RBC
transfusions given g
Pheresis continued until aGBM titre neg. by ELISA (total
19 exchanges); immunosuppression also contd.
Pt was dialysed twice, then slowly recovered renal fn.
ACE-I prescribed due to ongoing proteinuria
Ad ised to stop smoking and a oid occ pational Advised to stop smoking and avoid occupational
exposure to inhalants
1 year later, off immunosuppression, pt was doing well:
Cr 0.9 and +1 protein on urine dipstick
Copyright Harvard Medical School, 2010. All Rights Reserved.
877
Causes of pul monar y-r enal syndr omes
(ac ut e r enal and r espi r at or y f ai l ur e)
Goodpastures Disease p
Systemic Vasculitis:
-Common: WG, MPA, SLE
-Uncommon: HSP, Churg Strauss syn., cryoglobulin
disease
ARF (any etiology) with pulmonary edema (a y et o ogy) t pu o a y ede a
Severe LVF / cardiogenic shock
Severe pneumonia (esp. Legionella sp.)
Renal vein / IVC thrombosis with pulmonary emboli
Quest i on 9
A 22-year old white male is seen in the emergency
room for evaluation of acute renal failure. He explains
that he has just finished running the Boston marathon that he has just finished running the Boston marathon.
He complains of severe leg cramps. He tells you that
his urine is light pink. He has no significant past
medical history. He is not taking any medications. He
denies recent alcohol consumption. His physical
examination shows a white male. His BP is 120/80
mmHG, heart rate of 110 bpm, afebrile. His JVP is 2-3
cm He has clear lungs a normal cardiovascular and cm. He has clear lungs, a normal cardiovascular and
abdominal examination. He has no edema. His skin
turgor is reduced. Urinalysis reveals a SG of 1020, pH
5.0, 4+ blood, rest negative. His urine sediment shows
2-4 RBCs but is otherwise negative.
Copyright Harvard Medical School, 2010. All Rights Reserved.
878
All of the following statements are correct
Quest i on 9
All of the following statements are correct,
EXCEPT
A.) His CPK level is likely to be elevated
B.) His serum phosphorous is likely to be
normal
C.) He should be rapidly volume repleted
D.) His urine should be alkalinized
E.) Orthotoludine positive urine is typical
Copyright Harvard Medical School, 2010. All Rights Reserved.
879
Feat ur es of Rhabdomyol ysi s
Muscle pain and dark urine Coca-Cola color Muscle pain and dark urine Coca Cola color
Orthotoludine-positive urine without RBCs
Elevated CPK and myoglobin
Increased K, Phos, urate, decreased Ca
Rapid increase in serum creatinine
Mechanism: free radicals ferrihemate reduced Mechanism: free radicals, ferrihemate, reduced
nitric oxide
Treatment: saline repletion, alkaline diuresis,
mannitol. Dialysis once ARF established
Causes of Rhabdomyol ysi s
Excessive muscle activity
seizures, delerium tremens, sport
Direct of ischemic muscle injury
trauma, compression syndrome, vascular
occlusion
Metabolic disorders
hypokalemia, hyponatremia,
hypophosphatemia
Drugs or toxins
ethanol, isopropyl alcohol, heroin, methadone
Infections
tetanus, legionaires, influenza
Copyright Harvard Medical School, 2010. All Rights Reserved.
880
Quest i on 10
A 12-year-old boy consults his family physician A 12-year-old boy consults his family physician
because of the recent onset of edema. He has
no other relevant history and the physical
examination is remarkable only for significant
pitting edema in the lower extremities. His blood
pressure is 135/80. His sediment shows the p
following:
The most appropriate treatment is:
Quest i on 10
a) Prednisone and cyclosporine
b) Prednisone and Rituximab
c) Prednisone
d) Prednisone and Cyclophosphamide
e) Prednsione and Cellcept
Copyright Harvard Medical School, 2010. All Rights Reserved.
881
Usedi ment
BUN 15 mg/dL
Quest i on 10
g
Creatinine 0.9 mg/dL
Albumin 1.7 g/dL (normal = 3.5 to 5 g/dL)
Glucose 92 mg/dL
Urinalysis 4+ protein (by dipstick); no cells or casts
The total protein to creatinine ratio is 10.8, 24 h protein
10 8 /d 10.8 g/day
Copyright Harvard Medical School, 2010. All Rights Reserved.
882
Quest i on 10
The most appropriate treatment is:
a) Prednisone and cyclosporine
b) Prednisone and Rituximab
c) Prednisone
d) Prednisone and Cyclophosphamide
e) Prednsione and Cellcept
Copyright Harvard Medical School, 2010. All Rights Reserved.
883
MCD
Prednisone 1 mg/kg/d
(max 80 mg/d)
Until complete remission or
6 wks of therapy
Prednisone 2 mg/Kg alt day
For 3 months
YES
Complete
NO
REMISSION?
p
Remission
Or relapse
CsA > 1 y
Cytoxan or
Chlorambucil
8-12 w
Steroid taper
Conservative
Rx
61 year old man
Rheumatoid arthritis x 15 years & gold therapy X
Quest i on 11
Rheumatoid arthritis x 15 years & gold therapy X
12 years, with excellent control of his symptoms.
Six months prior he c/o of edema. Cr 1.0 mg/dl,
24-h protein 10 g, albumin 1.8 g/dl.
UA: 4+ albuminuria, bland sediment.
Gold salts D/Cd; proteinuria persists, 24h Gold salts D/C d; proteinuria persists, 24h
protein averaging 14 g/day.
Copyright Harvard Medical School, 2010. All Rights Reserved.
884
On no medications other than lasix in particular
Quest i on 11
On no medications other than lasix, in particular,
no NSAID in the recent past.
Physical examination showed a well-developed
man in no acute distress. Blood pressure was
120/80. There was 4+ edema present.
The most likely diagnosis is:
Quest i on 11
A.) Membranous glomerulopathy secondary
to gold
B.) Idiopathic membranous glomerulopathy
C.) AA amyloidosis secondary to
rheumatoid arthritis
D.) Primary focal segmental
glomerulosclerosis
E.) Rheumatoid vasculitis
Copyright Harvard Medical School, 2010. All Rights Reserved.
885
Gl omer ul ar Di sease and RA
Renal Disease with RA Renal Disease with RA
Membranous
Mesangial proliferative GN +/- IgA deposits
Diffuse proliferative GN
Necrotizing and crescentic GN (rheumatoid
vasculitis)
Amyloidosis
Copyright Harvard Medical School, 2010. All Rights Reserved.
886
Renal Disease with treatment of RA
Renal Di sease and RA
Renal Disease with treatment of RA
Gold: MN, MCD, ATN
Penicillamine: MN, Crescentic GN, MCD
NSAIDs: AIN, MN, MCD, ATN
CsA: chronic vasculopathy and tubulopathy
Methotrexate: ATN / crystal induced ARF
Aza: AIN
Quest i on 12
The following statements about Wegeners
granulomatosus are all true EXCEPT: g
A.) A small fraction of patients have anti-GBM
antibodies present.
B.) C-ANCA pattern is observed on indirect
immunofluorescence.
C.) Serial ANCA measurement has been proven to
predict relapse and should be utilized to intensify
th therapy.
D.) Less than 15% of patients actually have
granulomas on renal biopsy.
E.) Complement levels (C3, C4, CH50) in the serum
are in the normal range.
Copyright Harvard Medical School, 2010. All Rights Reserved.
887
WG
Respiratory and kidney involvement
Commonly presents with the syndrome rapidly
i l l h iti (RPGN) progressive glomerulonephritis (RPGN)
Associated with ANCA
Demographics
Slight male predominance; may occur at any
age of life; peak incidence in the fourth to
sixth decade of life.
I d i ti ith HLA DR2 HLA B7 Increased associations with HLA-DR2, HLA-B7,
and HLA-DR1 and DR1-DQW1.
All sizes of arteries and veins may be affected
but there is a predilection for small and
medium sized vessels.
Copyright Harvard Medical School, 2010. All Rights Reserved.
888
WG
Clinical Features
Presentation may be indolent with progressive involvement of
the respiratory tract and mild renal findings or there may be a
fulminant presentation with acute glomerulonephritis or RPGN.
Predominantly affects respiratory tract but vasculitic Predominantly affects respiratory tract, but vasculitic
multisystemic involvement is not uncommon.
Upper respiratory involvement includes sinusitis, tinnitus, and
hearing loss with otic discharge and pain.
Untreated, leads to tympanic membrane, deafness, chronic
sinusitis, and saddle-nose deformity.
Lower respiratory tract symptoms include cough with dyspnea
progressing to hemoptysis and alveolar hemorrhage.
Multisystemic disease may include skin (e g papules Multisystemic disease may include skin (e.g., papules,
purpura), joints (arthralgias, arthritis), eyes (conjunctivitis,
episcleritis,), nervous system, the liver, the thyroid, the
gallbladder, and the heart.
Rapidly progressive glomerulonephritis and renal failure.
Decline in GFR may be more gradual with non-nephrotic range
proteinuria, hematuria, and red cell casts.
WG
Pathology
Focal segmental necrotizing and crescentic
glomerulonephritis.
Also intracapillary thrombosis with endothelial cell swelling
and polymorphonuclear leukocyte infiltration mural
necrosis in vessel wall, pyknosis or karyorrhexis (nuclear
dust) may be observed.
Sites of active lesion often display non-specific crescents
and Bowman capsule destruction.
Chronic changes include glomerulosclerosis and fibrous
crescents.
Vasculitis may involve the small and medium size renal Vasculitis may involve the small and medium size renal
arteries, veins and capillaries.
Tubules show areas of focal tubular
degenerative/regenerative changes; cortical infarcts may
be observed.
Immunofluorescence shows a pauci immune pattern with
little immunoglobulin deposition.
Copyright Harvard Medical School, 2010. All Rights Reserved.
889
c-ANCA p-ANCA
Quest i on 13
All of the following drugs are removed All of the following drugs are removed
completely or partly by dialysis except:
A.) Cylcophosphamide
B.) Vancomycin
C.) Gentamicin
D.) Metoprolol
E.) Atenolol
Copyright Harvard Medical School, 2010. All Rights Reserved.
890
Removal of dr ugs by di al ysi s
Cyclophosphamide: 30-60% Cyclophosphamide: 30-60%
Prednisone: negligible
Captopril 35-40%
Lisinopril: 50-60%
Atenolol: 50%
Metoprolol: negligible
Copyright Harvard Medical School, 2010. All Rights Reserved.
891
Quest i on 14
All of the following drugs are contraindicated in All of the following drugs are contraindicated in
pregnancy, except:
A.) Cyclophosphamide
B.) ACE inhibitors
C.) Azathioprine
D.) Methyl dopa
E.) Cellcept
Copyright Harvard Medical School, 2010. All Rights Reserved.
892
FDA pr egnanc y
c at egor i es
A human studies (controlled) no adverse risk
B animal studies no adverse risk; no anecdotal
human studies showing risk
C animal studies show risk; no human studies
D human risk but benefits outweigh D human risk but benefits outweigh
X human risk, no benefits outweigh risk
Ef f ec t of I mmunosuppr essi ve
Dr ugs i n Pr egnanc y
Prednisone (category B)
Neonatal lymphopenia
Thymic hypoplasia
Azathiaprine (category D)
Crosses placenta, cannot be converted to active form
by fetal liver early in pregnancy (fetal liver lacks enzyme
inosinate pyrophosphorylase)
No clear increase in congenital anomalies
Neonatal leukopenia and thrombocytopenia
Copyright Harvard Medical School, 2010. All Rights Reserved.
893
ACEi and Di ur et i c s i n pr egnanc y
ACEi are contraindicated in pregnancy
Associated with oligohydramnios Associated with oligohydramnios
Neonatal death from hypoplastic lungs or neonate
renal failure
Higher risk of stillbirth
Dysplastic kidneys, hypocalvaria, and contractures
2 studies -- no ill effects from 1st trimester exposure
Diuretics use with caution b/o increased risk of pre Diuretics -- use with caution b/o increased risk of pre-
eclampsia
Ef f ec t of I mmunosuppr essi ve
Dr ugs i n Pr egnancy (c ont d)
Cyclosporine (category C)
Intrauterine growth retardation
Cyclophsphamide (category D)
Flattened nasal bridge
Palate defect
Single coronary artery Single coronary artery
Imperforate anus
Rectovaginal fistula
Dysmorphic facies etc
Copyright Harvard Medical School, 2010. All Rights Reserved.
894
Ef f ec t of I mmunosuppr essi ve
Dr ugs i n Pr egnancy (c ont d)
Tacrolimus
Hyperkalemia
Neonatal hyperkalemia
MMF
Embryocidal in animals
No human experience No human experience
Warfarin
Teratogenic during 1
st
trimester
Crosses placenta (fetal risk of bleeding)
Quest i on 15
A 62-year old patient is admitted to the hospital
for evaluation of hemoptysis of 2 weeks for evaluation of hemoptysis of 2 weeks
duration. He gives a history of gradually worse
dyspnea over the past 10 years. He has a cough
productive of copious sputum during winter
months and several prior lung infections during
the winter months. He has noticed some general
malaise, loss of appetite, and tiredness, worse
recently.
Copyright Harvard Medical School, 2010. All Rights Reserved.
895
Social history shows moderate beer intake -- 1-2
beers each night. Heavy smoking history 20-30
cigarettes/day x 40 years. He denies g y y
recreational drugs. Works as a janitor.
Medications: Atrovent inhaler and prn ventolin.
Examination: mild dyspnea at rest. He is
confused and agitated. Not cooperative. Blood
pressure on admission is 168/92 mmHg without
orthostasis. HR 84 bpm. Lungs: he has
scattered rhonchi. Reduced right base air-entry
and dullness on percussion. He has no
peripheral edema.
Urinalysis: SG 1010, pH 5.0. Rest negative.
Urine sodium 42 mEq/L
Urine osmolality 615 mOsm
Electrolytes
Na 100, K 3.5, C02 30, Cl 72, BUN 5.0,
Creatinine 0.6, Glucose 108 , Uric acid 2.6
Copyright Harvard Medical School, 2010. All Rights Reserved.
896
All of the following statements are true except:
A.) The patients total body sodium is normal
B.) The calculated serum osmolality is approximately 208
mOs/Kg mOs/Kg
C.) The patient has evidence of either inappropriate or
appropriate ADH secretion
D.) The urine sodium value seen in this patient is atypical of
a patient with SIADH
E.) Use of 3% hypertonic saline would be the best treatment at
this point in this patient
Copyright Harvard Medical School, 2010. All Rights Reserved.
897
Hypovelemia Euvolemia Hypervolemia
20% 35% 20%
Extrarenal Na losses
urine Na < 20
SIADH
Hypothyroidism
Extra-renal Disorders
CHF
urine Na < 20
Vomiting
Diarrhea
Renal Na losses
urine Na > 20
Diuretics
Osmotic diuretics
Hypothyroidism
Glucocorticoid def
Drugs
Pain
Respiratory failure
Positive pressure
breathing
CHF
Cirrhosis
Renal Disorders
NS
ARF
CRF
Cr i t er i a f or di agnosi ng SI ADH
Euvolemia
Uosm > 200 mOsm/Kg water)
Normal renal, adrenal, thyroid function
U Na > 20 mEq/L (usually 40 mEq/L)
Hypouricemia (< 4 mg/dL) Hypouricemia (< 4 mg/dL)
Copyright Harvard Medical School, 2010. All Rights Reserved.
898
Cl i ni c al Pr esent at i on of SI ADH
High risk group
young children
elderly patients
menstruating women
Na < 120
anorexia, lethargy, malaise, confusion, headaches
Na < 100
Confusion, seizures, stupor, coma
Management of Hyponat r emi a
Acute Symptomatic
acute (< 48 hrs)
symptomatic symptomatic
<Na 120 mEq
Hypertonic saline (3%) via central line
Correct at 0.55 mEq/L/hr - 1.0 mEq/L/hr
< 20 mEq change in Na over 24 hrs
Asymptomatic Asymptomatic
acute or chronic
free water restriction
500 ml to 1 liter / day
Demeclocycline 300 mg BID to 300 mg QID
Copyright Harvard Medical School, 2010. All Rights Reserved.
899
Pr i c e f or r api d c or r ec t i on
Rapid correction with acute severe
hyponatremia hyponatremia
Price of too rapid correction is central pontine
myelinolysis
Flaccid quadraplegia, dysphagia, dysarthria
Predisposing factors unclear
Experimental data support magnitude of Experimental data support magnitude of
correction not rate of correction
rate > 2 mEq/L/hr
total > 20 mEq/L/24 hr
A 42-year old white male 8 days post bone
marrow transplantation on treatment with FK506
Quest i on 16
marrow transplantation on treatment with FK506
(tacrolimus), among many other medications is
diagnosed with a type IV renal tubular acidosis.
All of the following features would be compatible
with this diagnosis except:
A.) A urine pH of 5.0
B.) The presence of hyperkalemia
C.) A negative urine anion gap of -22
D.) A serum bicarbonate of 18
E.) A normal anion gap
Copyright Harvard Medical School, 2010. All Rights Reserved.
900
Ur i ne Ani on Gap
Urine anion gap Urine anion gap
=(Na+) + (K+) (Cl-)
= unmeasured anions unmeasured cations
Most cases of MA has negative value due to excretion of an
unmeasured anion NH4+
Normal subjects, 20-40 meq NH4+ is excreted and anion gap = 0
Metabolic acidosis, NH4+ excretion increases and UAG = -20 to
50
In CRF or type 1 and 4 RTA, impaired H+ and NH4 excretion,
UAG is positive
Copyright Harvard Medical School, 2010. All Rights Reserved.
901
Renal Tubul ar Ac i dosi s
3 types
Type 1 (distal) RTA
R d d H+ ti b ll ti t b l Reduced H+ secretion by collecting tubule
Reduced NH4+ and titratable acid excretion
Defective H+-ATPase pump
Reduction in cortical Na+ resorption (voltage-dependent
defect)
Increased membrane permeability, back diffusion of H+
(gradient defect)
Causes Causes
Idiopathic
Autoimmune (SLE, RA etc), drugs and toxins (ampho B,
Toluene, lithium)
Marked volume depletion
Type 2 (proximal)
Proximal NaHC03 reabsorption is reduced
Self limiting disored, HC03 14-20 mEq/L
Renal Tubul ar Ac i dosi s
S g , C03 0 q/
Etiology
Idiopathic
Hereditary (cystinosis, Wilsons disease)
Acquired (multiple myeloma, drugs (acetazolamide, outdated
tetracycline, lead etc), amyloidosis, Sjogrens
Diagnosis
Normal AG metabolic acidosis G
Moderate acidosis (14-20)
Urine pH variable (> or < 5.3 depending on threshold)
Plasma K normal
Treatment
Bicarb replacement
Copyright Harvard Medical School, 2010. All Rights Reserved.
902
Hyper k al emi c Di st al
RTA Type I V
Disturbance of distal nephron function Disturbance of distal nephron function
Impaired excretion of H+ and K+
Normal gap acidosis and hyperkalemia
Type 1 RTA
Hypokalemia
NL renal fxn
Type 4 RTA
Hyperkalemia
CRI
U pH > 5.5
Metabolic acidosis
severe HC03 < 15
U pH <5.5
Metabolic acidosis
mild HC03 > 15
Causes of Type I V RTA
Defect Causes
Mineralocorticoid Deficiency
Low renin low aldo -- Diabetes mellitus
-- Drugs: NSAIDS, CsA, -blockers
High renin low aldo -- Adrenal destruction
-- Congenital enzyme defects
-- Drugs: ACEi, ARBs, heparin
ketoconazole
Abnormal cortical collecting duct -- Absent or defective
mineralocorticoid receptor
D i i l t il id -- Drugs: spirinolactone, amiloride,
pentamidine, trimethoprim
-- Chronic tubulointerstitial disease
UTI, lupus nephritis, sickle cell neph
Copyright Harvard Medical School, 2010. All Rights Reserved.
903
RTA
Type 1
(distal)
Type 2
(proximal)
Type 4
Basic Defect distal
acidification
proximal
reabsorption
Aldo
deficiency or
resistance
Serum
bicarb
Plasma K
Urine pH
Maybe < 10
Low or NL
> 5 3
14-20
Low or NL
Variable
Usually > 15
Elevated
<5 3
Urine pH
Other
> 5.3 Variable <5.3
Serum aldo
level
Dose of
HC03
1-2 mEq/d 10-15 mEq/d 1-2 meq/d
Quest i on 17
You are asked to consult on a 62 year old You are asked to consult on a 62 year old
African-American male with acute on chronic
renal insufficiency secondary to diabetes
mellitus ascribed to contrast nephrotoxicity.
Routine chemistry labs show a potassium of 8.2
mg/dL. All of the following would be changes g g g
seen on the EKG compatible with hyperkalemia,
except:
Copyright Harvard Medical School, 2010. All Rights Reserved.
904
A.) Peaked T waves
B.) Prolonged QRS
C.) Flattened p wave
D.) Sine-wave appearing QRS complex D.) Sine wave appearing QRS complex
E.) U wave
Copyright Harvard Medical School, 2010. All Rights Reserved.
905
EKG c hanges i n hyper k al emi a
Early: tenting of T waves pinch-bottomed T Early: tenting of T waves pinch bottomed T
waves; precordial leads
Prolonged P-R interval
ST segment depression and lengthening of QRS
P wave disappears, further widening of QRS
Ventricular fibrillation Ventricular fibrillation
EKG c hanges i n Hyper k al emi a
Copyright Harvard Medical School, 2010. All Rights Reserved.
906
The most common type of kidney stone
Quest i on 18
yp y
observed in the United States is:
A.) Cystine stone
B.) Triple phosphate stone
C.) Struvite stone
D ) Calcium oxalate stone D.) Calcium oxalate stone
E.) Uric acid stone
Copyright Harvard Medical School, 2010. All Rights Reserved.
907
Nephr ol i t hi asi s
12% of US population affected
Incidence rate (age 30-65)
Male: 3/1000/yr
Female: 1/1000/yr
Calcium oxalate > 75%
Hypercalciuria,hyperoxaluria, hypocitrituria,
hyperuricosuria
Infection stone/Magnesium ammonium Infection stone/Magnesium ammonium
phosphate/struvite/triple phosphate 7-15%
Uric acid 2%
Calcium phosphate 2%
Cystine <1%
Common Cr yst al s
Source: www:medstat.med.utah.edu/WebPath
Copyright Harvard Medical School, 2010. All Rights Reserved.
908
Di sc l osur es
Consulting Consulting
Amgen, Johnson and Johnson, Sandoz,
Fibrogen
Grant Support
Amgen, Johnson and Johnson, Roche
Advisory Boards
Rockwell
Copyright Harvard Medical School, 2010. All Rights Reserved.
909
Kidney Stone Cases
Gary C. Curhan, MD, ScD
Associate Professor of Medicine
and Epidemiology
Disclosure
I have financial relationships or affiliation with:
Name of Organization Relationship
TAKEDA Consultant
ASTELLAS Grant Support
UP-TO-DATE Section Editor
American Society of Nephrology Editorial Board
Copyright Harvard Medical School, 2010. All Rights Reserved.
910
Type of Stones & Frequency in Adults
CaOx
CaOx
CaOx
Coe, 1983
Urinary Risk Factors
Increased risk
Calcium
Oxalate
Uric acid
Decreased risk
Citrate
Volume
But need to rethink our approach--these
are continuous variables, not dichotomous
Copyright Harvard Medical School, 2010. All Rights Reserved.
911
Definitions
Hypercalciuria
Males: >300 mg/d
Females: >250 mg/d
Either: >4 mg/kg/d
Hyperoxaluria: >45 mg/d
Hyperuricosuria
Males: >800 mg/d
Females: >750 mg/d
Case #1Mr. G.O.
52 y.o. male with recurrent nephrolithiasis
Passes a stone every two months
PMH: chronic diarrhea
Meds: Lomotil; allopurinol 300 mg/d
Copyright Harvard Medical School, 2010. All Rights Reserved.
912
Case #1Evaluation
Blood
Bicarbonate 26 meq/L
Uric acid 5.5 mg/dL
KUB
No stones seen
Case #124 hour urine
TV Ca Ox Cit UA Creat Na pH
2.4 290 56 141 503 2599 347 5.6
1.5 172 20 142 231 1528 237 5.3
Copyright Harvard Medical School, 2010. All Rights Reserved.
913
Questions
1. Metabolic abnormalities?
2. Stone type?
3. Next steps?
Question: Metabolic abnormalities?
Which urine is the correct one?
Second collection contains 40% less
creatinine than the first
Copyright Harvard Medical School, 2010. All Rights Reserved.
914
Case #124 hour urine
TV Ca Ox Cit UA Creat Na pH
2.4 290 56 141 503 2599 347 5.6
1.5 172 20 142 231 1528 237 5.3
Question: Metabolic abnormalities?
Assume first urine is the
correct one.
High calcium,
oxalate, sodium
Low citrate
Normal uric acid
Assume second urine is
the correct one.
High, sodium
Low citrate
Normal calcium,
oxalate, uric acid
Copyright Harvard Medical School, 2010. All Rights Reserved.
915
Question: Stone type?
1. Calcium oxalate
2. Calcium phosphate
3. Uric acid
4. Cystine
Uric acid 100%
Question: Next Steps?
Imaging study
Spiral CT
Repeat 24 hour urine
Alkalinizationof urine
Potassium citrate
Patient follow urine pH (aim for ~6.5)
Copyright Harvard Medical School, 2010. All Rights Reserved.
916
Case #2Ms. G.K.
45 y.o. female with recurrent nephrolithiasis
Frequency: every 8-10 months
Previous procedures:
ESWL x 5
Cystoscopyx 6
PCNL x 1
PMHotherwise negative
Case #224 hour urine
TV Ca Ox Cit UA Creat Na pH
2.30 114 28 1102 396 972 168 7.30
Copyright Harvard Medical School, 2010. All Rights Reserved.
917
Question: Stone type?
1. Calcium oxalate
2. Calcium phosphate
3. Uric acid
4. Cystine
Cystine100%
Medications
Potassium citrate 20 meqBID
Occasionally checks urine pH
Allergies
Penicillaminerash
Copyright Harvard Medical School, 2010. All Rights Reserved.
918
CT Report
Right kidney is normal
Left kidney contains 4 mm and 2 mm upper
pole stones; 9 mm lower pole stone; 4 mm
at the left UVJ
Case #224 hour urine
TV Ca Ox Cit UA Creat Na pH
2.30 114 28 1102 396 972 168 7.30
Total cystine: 1877 mg/d (816 mg/L)
Copyright Harvard Medical School, 2010. All Rights Reserved.
919
Treatment
Reduce [cystine] <250mg/L
Increase fluid intake?
Urine output >5 L/d
Tiopronin (Thiola)
Typically need at least 900 mg/d (300 mg TID)
Continue alkalinization
Case #2F/U 24 hour urine
First collection
Total cystine: 1877 mg/d
Second collection on tiopronin 900 mg/d
Total cystine: 2014 mg/d
Copyright Harvard Medical School, 2010. All Rights Reserved.
920
Why Was Urine CystineHigher?
Day-to-day variability
Lab variability
Completeness of collection
What does the lab actually measure?
All existing assays seem to also measure the
cystine-tiopronincomplex
Case #26 Months Later
Calls with left renal colic
Stopped medication because it clearly did
not work
Is she correct?
Copyright Harvard Medical School, 2010. All Rights Reserved.
921
Case #3Ms. J .B.
35 y.o. female nurse with recurrent stones
Presented 5 years ago with colic and
bilateral renal stones
PMH: unremarkable
Question: Stone type?
1. Calcium oxalate
2. Calcium phosphate
3. Uric acid
4. Cystine
Calcium oxalate 95%
Calcium phosphate 5%
Copyright Harvard Medical School, 2010. All Rights Reserved.
922
CT Report
Bilateral renal stones
3 mm stone at left UVJ
Question: Acute Management?
Watchful waiting
For how long?
Pain
Infection
Hydronephrosis
Medications
Alpha blocker
Cystoscopicremoval
ESWL
Copyright Harvard Medical School, 2010. All Rights Reserved.
923
Case #324 hour urine
TV Ca Ox Cit UA Creat Na
1.73 210 26 845 493 1400 ---
1.10 209 32 943 447 1265 ---
Question: Treatment?
Increase fluid intake
How much to prescribe?
If TV does not increase?
Thiazide
Copyright Harvard Medical School, 2010. All Rights Reserved.
924
Curhan, Kidney Int, 2008
24 Hour Urine Calcium
Cases: 2237; Controls: 1113
Question: Follow-Up?
Repeat 24 hour urine
4-6 weeks
Adjust recommendations
Repeat 24 hour urine
Follow-up imaging
Stone growth
New stone formation
Copyright Harvard Medical School, 2010. All Rights Reserved.
925
Case #4Mr. S.L.
49 y.o. male accountant with h/oCrohns
disease
S/p partial ileal resection
Recurrent intestinal obstruction
Presented 6 months ago with abdominal pain
Intestinal obstruction?
Exploratory laparotomyplanned
Case #4Mr. S.L.
Pre-op evaluation: U/A-->100 RBC
Underwent right hemi-colectomy
POD #5abdominal pain
due to tapering of analgesics?
Discharged home on Percocet
Returned two days later to ER
UVJ stone seen on CT
Copyright Harvard Medical School, 2010. All Rights Reserved.
926
Question: Stone type?
1. Calcium oxalate
2. Calcium phosphate
3. Uric acid
4. Cystine
Calcium oxalate 100%
Case #424 hour urine
TV Ca Ox Cit UA Creat Na
1.20 62 117 18 413 1350 ---
1.30 73 73 11 711 1412 ---
Copyright Harvard Medical School, 2010. All Rights Reserved.
927
Question: Treatment?
Increase fluid intake
How much to prescribe?
Reduce oxalate
Increase dietary calcium intake
Increase citrate
What formulation?
Case #5Ms. E.J .
35 y.o. female neurology nurse
Renal colic
9 x 5 mm stone right UVJ
Cysto+laser lithotripsy
Calcium oxalate 100%
Patient reports she is stone free
Copyright Harvard Medical School, 2010. All Rights Reserved.
928
Case #5 Ms. E.J .
Diagnosed with recurrent UTIs
U/A +for RBC, WBC
Questionable relief from Abx
KUB report 2 mm calcification at right
UVJ
Question: Next Steps?
Urine cultures
Cultures are negative
Imaging study
Spiral CT: UVJ stone
Cystofor stone fragment removal
24 hr urine
Copyright Harvard Medical School, 2010. All Rights Reserved.
929
Case #524 hour urine
TV Ca Ox Cit UA Creat Na
1.97 477 42 437 978 1714 191
2.37 491 45 534 894 1510 196
Question: Metabolic abnormalities?
Elevated in urine:
calcium
oxalate
uric acid
sodium
Check PTH
Normal
Copyright Harvard Medical School, 2010. All Rights Reserved.
930
Question: Treatment?
Maintain current fluid intake
Diet
Reduce oxalate
Reduce sodium
Reduce animal protein
Increase dietary calcium intake?
Check bone density
Thiazide
Avoid if pregnancy planned
Case #524 hour urine
on HCTZ 25 mg/d
TV Ca Ox Cit UA Creat Na
3.99 465 44 1095 1005 1700 319
Compare with pre-treatment values
1.97 477 42 437 978 1714 191
2.37 491 45 534 894 1510 196
Copyright Harvard Medical School, 2010. All Rights Reserved.
931
Case #6 Ms. P.T.
27 y.o. female with recurrent nephrolithiasis
PMH: Recurrent UTIs
Passes stones monthly
KUB: bilateral renal calcifications
Question: Stone type?
1. Calcium oxalate
2. Calcium phosphate
3. Struvite
4. Cystine
Calcium phosphate 100%
Copyright Harvard Medical School, 2010. All Rights Reserved.
932
Case #6 Ms. M.T.
IVP
Medullarysponge kidney
CT
Parenchymal calcifications
Few stones in renal pelvis
Serum HCO3 21
Urine pH 6.6
Question: Next Steps?
Urine cultures
Proteus
Cystoand laser for stone fragment removal
3 unit bleed
required embolization
24 hr urine
Copyright Harvard Medical School, 2010. All Rights Reserved.
933
Case #624 hour urine
TV Ca Ox Cit UA Creat Na
0.93 251 18 225 437 978 132
0.87 275 21 189 334 1012 159
Question: Treatment?
Increase fluid intake
Diet
Reduce sodium
Increase dietary calcium intake?
Check bone density
Thiazide
Alkali?
Copyright Harvard Medical School, 2010. All Rights Reserved.
934
Hypocitraturiaand Calcium
Phosphate Stones
CaP form in more alkaline urine
How to lower urine pH in patients with
RTA?
How to raise urine citrate w/o raising urine
pH?
Alkali may help or make the situation worse
Question: Next Steps?
Screening urine cultures
aggressively treat UTIs
Imaging study
Difficult to evaluate stone burden
24 hr urine
Copyright Harvard Medical School, 2010. All Rights Reserved.
935
Wisdom from Experience
Always measure TWO 24 hour urines at
baseline
Find out stone composition by analysis
Repeat 24 hour urine after
recommendations
Helical CT is the most reliable imaging
study
Wisdom from Experience
Be careful when recommending a low
oxalate diet
Explain to patient that a new episode of
colic does not always mean treatment
failure
Not all chronic kidney pain is from stones
Watch out for the addicted stone patient
The urologist is your friend
Copyright Harvard Medical School, 2010. All Rights Reserved.
936
1. A 42 year old male suffered a severed spinal cord at
T10. He has a long history of recurrent urinary tract
infections. As part of the evaluation for abdominal pain,
he had a KUB that showed a staghorncalculus in his
right kidney. One of his urine infections most likely
included which of the following bacteria?
a. E. coli
b. MRSA
c. Klebsiellapneumoniae
d. proteusmirabilis
(Correct answer is D)
2. A child whoseparents are first cousins was found to
have a kidney stone at age 7. The urinalysis showed
blood and many hexagonal crystals. The most likely
stone composition is:
a. cystine
b. struvite
c. calcium oxalate
d. uric acid
(correct answer is A)
Copyright Harvard Medical School, 2010. All Rights Reserved.
937
References
StamatelouKK, Francis ME, J ones CA, Nyberg LM, CurhanGC. Time trends in reported prevalence
of kidney stones in the United States: 1976-1994. Kidney Int 2003; 63:1817-23.
CurhanGC, Willett WC, RimmEB, Stampfer MJ . A prospective study of dietary calciumand other
nutrients and the risk of symptomatic kidney stones. N Engl J Med 1993; 328:833-8.
Taylor EN, Stampfer MJ , Curhan, GC. Dietary factors and the risk of incident kidney stones in men:
new insights after 14 years of follow-up. J Am Soc Nephrol. 2004 Dec;15(12):3225-32.
CurhanG, Willett W, Speizer F, SpiegelmanD, Stampfer M. Comparison of dietary calciumwith
supplemental calciumand other nutrients as factors affecting therisk for kidney stones in women.
Ann Intern Med 1997; 126:497-504.
CurhanGC, Willett WC, Knight EL, Stampfer MJ . Dietary factors and the risk of incident kidney
stones in younger women (Nurses' Health Study II). Arch Intern Med 2004; 164:885-91.
CurhanGC, Willett WC, Speizer FE, Stampfer MJ . Beverage use and risk for kidney stones in
women. Ann Intern Med 1998; 128:534-40.
CurhanG, Taylor E. 24-h uric acid excretion and the risk of kidney stones. Kidney Int 2008;
Feb;73(4):489-496.
Taylor EN, Stampfer MJ , Curhan GC. Obesity, weight gain, and the risk of kidney stones. JAMA.
2005 J an 26;293(4):455-62.
Taylor E, CurhanG. Oxalate intake and the risk for nephrolithiasis. J Am Soc Nephrol. 2007
J ul;18(7):2198-204.
Evan AP, et al. Randall's plaque of patients with nephrolithiasisbegins in basement membranes of
thin loops of Henle. J Clin Invest. 2003 Mar;111(5):607-16.
Taylor EN, Fung TT, Curhan GC. DASH-style diet associates with reduced risk of kidney stones. J
Am Soc Nephrol. 2009 Oct; 20(10):2253-9.
Disclosure
I have financial relationships or affiliation with:
Name of Organization Relationship
TAKEDA Consultant
ASTELLAS Grant Support
UP-TO-DATE Section Editor
American Society of Nephrology Editorial Board
Copyright Harvard Medical School, 2010. All Rights Reserved.
938
Hypertension Board Review
J ohn P. Forman, M.D., M.Sc.
Brigham and Womens Hospital
Harvard Medical School
Disclosures
None
Copyright Harvard Medical School, 2010. All Rights Reserved.
939
Case 1
Case 1, question 1
A 78 year old woman with DM, CAD, and CKD (Cre=2.4 mg/dL), is
referred to you for persistent HTN, despite being treated with 5
medications (lisinopril 40 mg, amlodipine10 mg, furosemide40 mg
bid, metoprolol 50 mg, and losartan 50 mg). She is asymptomatic.
She tells you that she is often dizzy when she wakes up in the
morning. She checks her BP at home, and says it is typically 100
120 systolic. On exam, her BP is 164/70, non-orthostatic, HR 54,
but otherwise unremarkable. Her UA shows 2+albumin. BUN=42,
Cre=2.38, K=4.1.
- Which one of the following next steps is most appropriate?
A.) Increase the dose of her losartan to achieve better BP control
B.) Ask her to continue to do home BP measurements and see her
again in 3 months time.
C.) Do a screen for furosemidein her urine to see if she is being
adherent with her medications
D.) Set her up for ambulatory BP monitoring
E.) Start her on a clonidinepatch.
Copyright Harvard Medical School, 2010. All Rights Reserved.
940
135/85
Ambulatory Pressure
140/90
Clinic
Pressure
Sustained
Hypertension
White Coat
Hypertension
True
Normotension
Masked
Hypertension
10-40%
5-20%
Predictors of white coat HTN
Older (age 60 years)
Female
Morning clinic visit
BP meds taken in AM (vs. PM)
Obesity
Active smoking
Diabetes
Banegas JR et al. Hypertension 2007; 49:62
Copyright Harvard Medical School, 2010. All Rights Reserved.
941
Recommendations for Clinical Use of ABPM:
J NC 7 & WHO-ISH
JNC 7 WHO-ISH
ABPM endorsed Yes Yes
Indications:
White Coat HTN Yes Yes
Labile BP Yes Yes
R/O hypotensive episodes Yes Yes
Resistant HTN Yes Yes
Autonomic dysfunction Yes No
A 78 year old woman with DM, CAD, and CKD (Cre=2.4 mg/dL), is
referred to you for persistent HTN, despite being treated with 5
medications (lisinopril 40 mg, amlodipine10 mg, furosemide40 mg
bid, metoprolol 50 mg, and losartan 50 mg). She is asymptomatic.
She tells you that she is often dizzy when she wakes up in the
morning. She checks her BP at home, and says it is typically 100
120 systolic. On exam, her BP is 164/70, non-orthostatic, HR 54,
but otherwise unremarkable. Her UA shows 2+albumin. BUN=42,
Cre=2.38, K=4.1.
- Which one of the following next steps is most appropriate?
A.) Increase the dose of her losartan to achieve better BP control
B.) Ask her to continue to do home BP measurements and see her
again in 3 months time.
C.) Do a screen for furosemidein her urine to see if she is being
adherent with her medications
D.) Set her up for ambulatory BP monitoring
E.) Start her on a clonidinepatch.
Case 1, question 1
Copyright Harvard Medical School, 2010. All Rights Reserved.
942
Case 1, question 2
The patient returns for follow-up after completing the 24-
hour ABPM. Average daytime BP is 124/65 mmHg.
Mean nighttime BP is 110/50 mmHg. There is an 11%
drop in SBP from daytime to nighttime.
- Which of the following statements about 24-hour ABPM is
false?
A.) ABPM has prognostic value over and above clinic BP.
B.) Home BP monitoring is prognosticallyinferior to 24hr
mean BP.
C.) Nighttime BP is the strongest predictor of CV mortality.
D.) Non-dipping status is associated with greater declines in
GFR
E.) Non-dipping status is associated with a higher risk of CV
events.
Prognostic significance over and above
clinic BP
Clement DL. et al. NEJM 2003;348: 2407
Masked HTN = risk
Additional prognostic
information gained from
ABPM
Copyright Harvard Medical School, 2010. All Rights Reserved.
943
BP metric and risk of CV death
Dolan, E. et al. Hypertension 2005;46:156-161 N=5292
In fully adjusted models, including adjustment for clinic BP:
10 mmHg higher nighttime SBP 21% higher risk of CV death
5 mmHg higher nighttime DBP 9% higher risk of CV death
BP dipping and change in GFR
Davidson, M. B. et al. Arch Intern Med 2006;166:846-852.
329 adults w/o ESRD referred for 24hr ABPM
Copyright Harvard Medical School, 2010. All Rights Reserved.
944
Non-dipping and risk of CV events
Staessen, J. A. et al. JAMA 1999;282:539-546.
Each 10% higher night-
day ratio is associated
with a 41% higher risk
of CV events
Case 1, question 2
The patient returns for follow-up after completing the 24-
hour ABPM. Average daytime BP is 124/65 mmHg.
Mean nighttime BP is 110/50 mmHg. There is an 11%
drop in SBP from daytime to nighttime.
- Which of the following statements about 24-hour ABPM is
false?
A.) ABPM has prognostic value over and above clinic BP.
B.) Home BP monitoring is prognostically inferior to
24hr mean BP.
C.) Nighttime BP is the strongest predictor of CV mortality.
D.) Non-dipping status is associated with greater declines in
GFR
E.) Non-dipping status is associated with a higher risk of CV
events.
Copyright Harvard Medical School, 2010. All Rights Reserved.
945
Case 2
Case 2: question 1
A 37 year old white female referred to you for resistant hypertension.
Occasional headaches, fatigue, and muscle cramps
BP 170/100, BMI 27, exam is otherwise unremarkable.
Enalapril 20 bid, HCTZ/triamterene25/37.5 qd, atenolol 50 qd.
Potassium 2.6 meq/L, other labs normal
Which of the following would most likely ESTABLISHthe correct
diagnosis?
A.) Fractionated plasma metanephrines
B.) Renal angiogram
C.) Plasma ratio of aldosterone to PRA (ARR)
D.) Low dose dexamethasone suppression test
E.) 24-hr urine aldosterone after oral sodium loading
F.) TSH
Copyright Harvard Medical School, 2010. All Rights Reserved.
946
Evaluate for endocrine causes
of secondary hypertension
Resistant hypertension
Eliminate psuedoresistance
Improper measurement
White coat syndrome
Non-adherence
Identify common non-endocrine causes
Obesity sleep apnea
Diabetes and/or kidney disease
Offending drugs or alcohol
Combination of poor lifestyle factors
Specific signs or symptoms
Adrenal incidentaloma
Primary aldosteronism:
the most common endocrine cause
True prevalence is unclear
Prevalence data come from referred
populations
Prevalence data not always confirmed
Lack of a widely accepted gold standard
Population Prevalence
Hypertension 5-13%
Resistant hypertension 17-23%
Hypertension with incidentaloma 1-10%
Copyright Harvard Medical School, 2010. All Rights Reserved.
947
Primary aldosteronism(PA): diagnosis*
Patient with possible PA
Patient with probable PA
Patient with confirmed PA
Screening test: ARR
Confirmatory test: choice of 4
*Endocrine society guidelines 2008
Patient with possible PHA
Patient with probable PHA
Patient with confirmed PHA
Screening test: ARR
Confirmatory test: choice of 4
*Endocrine society guidelines 2008
Problems
Time of day?
Sodium intake?
Position?
Ratio only or ratio plus aldo?
Cutoff for ratio
Do the patients meds matter?
PRA assay sensitivity?
Primary aldosteronism(PA): diagnosis*
Copyright Harvard Medical School, 2010. All Rights Reserved.
948
Problems with the ARR
Timing
Mid-morning draw is recommended
Diet
Liberal sodium intake is recommended
Position
15 minutes seated is recommended
Specimen handling
Maintain at room temperature is recommended
In the real world
Not always convenient
What is the appropriate cutoff?
No guidance from the endocrine society
Ratios used in various studies: 20-50
Should the plasma aldosterone also be
considered?
No guidance from the endocrine society
From various studies: some do, some dont
Problems with the ARR
Copyright Harvard Medical School, 2010. All Rights Reserved.
949
Medication Effect on ARR Effect on Decision
-blocker Lowers PRA (a lot) False positive
Diuretics Increases PRA False negative
ACEI/ARB Lowers aldo, increases PRA False negative
DHP Ca-blocker Increases PRA False negative
Central 2-blocker Lowers PRA False positive
Problems due to medications
Problems due to other factors
Factor Effect on ARR Effect on Decision
Hypokalemia Lowers also Falsenegative
Salt restricted Increases PRA False negative
CKD Lowers PRA False positive
Elderly Lowers PRA Falsepositive
Problems with the ARR
What about medications?
Endocrine society suggests washout of all meds
for 4 weeks, or
switching to combination of verapamil,
hydralazine, and 1-blockers
Endocrine society may not live on a place I like
to call Earth
Problems with the ARR
Copyright Harvard Medical School, 2010. All Rights Reserved.
950
Denominator dependence
The PRA assay varies from lab to lab
0.2 to 2.0 ng/mL per hour
Examples
Patient with PA, no meds, aldo =16 ng/dL
(high), PRA suppressed at 2.0 ng/mL per hour
Ratio =8 (false negative)
Patient without PA, on -blocker, aldo =6
ng/dL (normal), PRA suppressed at 0.2 ng/mL
per hour
Ratio =30 (false positive)
Problems with the ARR
Patient with possible PHA
Patient with probable PHA
Patient with confirmed PHA
Screening test: ARR
Confirmatory test
*Endocrine society guidelines 2008
Options
Outpatient salt suppression
Saline infusion
Fludrocortisone stimulation
Captopril challenge
Primary aldosteronism(PA): diagnosis*
Copyright Harvard Medical School, 2010. All Rights Reserved.
951
Confirmatory tests for PHA
Test Issues
Outpatient salt suppression 3-day high salt diet and a 24 hour urine
collection
Saline infusion Requires at least 7 hours in either hospital or
infusion center
Fludrocortisone stimulation Usually requires a multi-day hospital admission
Captopril challenge 2-hour office-based test; some evidence of
inferiority compared to other tests
Case 2: question 1
A 37 year old white female referred to you for resistant
hypertension.
Occasional headaches, fatigue, and muscle cramps
BP 170/100, BMI 27, exam is otherwise unremarkable.
Enalapril 20 bid, HCTZ/triamterene 25/37.5 qd, atenolol 50
qd.
Potassium 2.6 meq/L, other labs normal
Which of the following would most likely ESTABLISHthe correct
diagnosis?
A.) Fractionated plasma metanephrines
B.) Renal angiogram
C.) Plasma ratio of aldosterone to PRA (ARR)
D.) Low dose dexamethasone suppression test
E.) 24-hr urine aldosterone after oral sodium loading
F.) TSH
Copyright Harvard Medical School, 2010. All Rights Reserved.
952
You intensify her BP meds and give KCL supplements
She does the 24-hr urine with oral sodium loading
Potassium 3.6 meq/L
24hr urine: 1.2 grams cre, 280 mmol of sodium, and 54
mcg aldosterone.
No family history of hypertension.
Youve just diagnosed PHA. What are you going to do now?
a) Order an adrenal CT scan
b) Refer for adrenal vein sampling
c) Start spironolactone
Case 2: question 2
Rare cause of PHA
Only 26 cases between 1955-2002
Usually >4cm and contrast enhancing
Often metastatic at time of presentation
Adrenal carcinoma
Copyright Harvard Medical School, 2010. All Rights Reserved.
953
You intensify her BP meds and give KCL supplements
She does the 24-hr urine with oral sodium loading
Potassium 3.6 meq/L
24hr urine: 1.2 grams cre, 280 mmol of sodium, and 54
mcg aldosterone.
No family history of hypertension.
Youve just diagnosed PHA. What are you going to do now?
a) Order an adrenal CT scan
b) Refer for adrenal vein sampling
c) Start spironolactone
Case 2: question 2

CT scan reveals a 1 cm adenoma in left


adrenal gland
The right adrenal appears normal
What are you going to do now?
a) Refer for surgical resection of left adrenal
b) Refer for adrenal vein sampling
c) Start spironolactone
d) Ask her if she would consider surgery
Case 2: question 3
Copyright Harvard Medical School, 2010. All Rights Reserved.
954
Surgery or MR antagonist for primary
aldosteronism?
54 patients with PA
Treatment
adrenalectomy(N=24)
spironolactone (N=30)
7.4 year mean f/u
Equivalent average BP
Copyrightrestrictions may apply.
Catena, C. et al. Arch Intern Med 2008;168:80-85.
P=0.71
Inci dence of composi te CV endpoi nt
CT scan reveals a 1 cm adenoma in left
adrenal gland
The right adrenal appears normal
What are you going to do now?
a) Refer for surgical resection of left adrenal
b) Refer for adrenal vein sampling
c) Start spironolactone
d) Ask her if she would consider surgery
Case 2: question 3
Copyright Harvard Medical School, 2010. All Rights Reserved.
955
She would like to pursue surgery
What are you going to do now?
a) Refer for surgical resection of left adrenal
b) Refer for adrenal vein sampling
c) Start spironolactone
Case 2: question 4
Is a unilateral adenoma sufficient?
Systematic review of 38 studies (950 patients)
with PA, either CT or MRI, and AVS
If we use CT or MRI alone to guide treatment
Unilateral adenoma surgery
Bilateral or no adenoma spironolactone
Inappropriate exclusion from adrenalectomy
19%
Inappropriate adrenalectomy
19%
Because disease was really bilateral: 15%
Because disease was on the other side: 4%
Kempers M. et al. Ann Intern Med 2009;151:329-37.
Copyright Harvard Medical School, 2010. All Rights Reserved.
956
She would like to pursue surgery
What are you going to do now?
a) Refer for surgical resection of left adrenal
b) Refer for adrenal vein sampling
c) Start spironolactone
Case 2: question 4
PA algorithm
Possible PA
Probable PA Confirmed PA
Screening test
Confirmatory test
*Endocrine society guidelines 2008
Adrenal CT scan
PA, no cancer
Surgery not desired
Treat medically
Surgery desired
Perform AVS
Unilateral PA
Bilateral PA
Operate
Copyright Harvard Medical School, 2010. All Rights Reserved.
957
Case 3
Case 3, question 1
A 55 yo man is referred for chronic resistant hypertension, despite
taking lisinopril, chlorthalidone, amlodipine, and metoprolol. He
drinks 3-4 alcoholic beverages per week, and tries to restrict his salt
intake. He snores, but denies hypersomnolence or headaches, and
his wife is unaware of apneic episodes.
His BP is 160/90, consistent with home readings. He is obese
(BMI=35), and has 1+pitting edema.
Cre=1.0, K=3.8, and urinalysis is negative for albumin or cells.
Which of the following is most correct?
A)His alcohol intake is likely contributing to his resistant
hypertension
B)His chlorthalidone should be replaced by furosemide
C)Polysomnography would most likely be positive for sleep apnea
D)Polysomnography is not indicated since he does not have
specific symptoms of sleep apnea
Copyright Harvard Medical School, 2010. All Rights Reserved.
958
Alcohol and hypertension
Alcohol intake, grams per day (10 grams 1 alcoholic beverage)
Clinical prediction of OSA
Epidemiology
50-90% of obese patients with hypertension
History/exam
Best clinical predictors from the sleep
literature:
Snoring
Obesity
Hypertension
Witnessed gasping/choking
Physician impression using the standard
questions (daytime somnolence, etc)
50% sensitivity
Copyright Harvard Medical School, 2010. All Rights Reserved.
959
Prevalence of OSA in resistant hypertension
Two recent studies of overweight or obese individuals with confirmed
resistant hypertension
Logan AG, et al. J Hypertens 2001
Lozano L, et al. J Hypertens 2010
Case 3, question 1
A 55 yo man is referred for chronic resistant hypertension, despite
taking lisinopril, chlorthalidone, amlodipine, and metoprolol. He
drinks 3-4 alcoholic beverages per week, and tries to restrict his salt
intake. He snores, but denies hypersomnolence or headaches, and
his wife is unaware of apneic episodes.
His BP is 160/90, consistent with home readings. He is obese
(BMI=35), and has 1+pitting edema.
Cre=1.0, K=3.8, and urinalysis is negative for albumin or cells.
Which of the following is most correct?
A) His alcohol intake is likely contributing to his resistant hypertension
B) His chlorthalidone should be replaced by furosemide
C) Pol ysomnography woul d most l i kel y be posi ti ve for sl eep
apnea
D) Polysomnography is not indicated since he does not have specific
symptoms of sleep apnea
Copyright Harvard Medical School, 2010. All Rights Reserved.
960
You refer him for polysomnography, demonstrating an apnea-
hypopnea index (AHI) of 20 events per hour (moderate OSA). He
begins CPAP and is now seeing you 2 months later. He uses CPAP
nightly, sleeps well, and says that he feels more energetic. He still
takes lisinopril, chlorthalidone, amlodipine, and metoprolol. His
exam is unchanged and his BP is now 157/88 (instead of 160/90).
Which one of the following is most likely to result in control of his
hypertension?
A) Spironolactone 25 mg/day titrating to 100 mg/day as needed.
B) An exercise program of walking, 20 minutes three times per week
C) Referral back to the sleep clinic to adjust his CPAP mask or prescription
since the therapy is not working
D) Referral to ENT for laser-assisted uvuloplasty
E) Long-acting diltiazem 120 mg/d titrating to 300 mg/day as needed
Case 3, question 2
Effect of CPAP on blood pressure
C
h
a
n
g
e

i
n

S
B
P

(
m
m
H
g
)
p=<0.05
p=<0.05 P=NS
(N=16) (N=12) (N=10)
Conclusion: effect of CPAP on blood
pressure is modest
Haentjens et al. Arch Intern Med 2007 Bazzano et al. Hypertens 2007 Alajmi et al. Lung 2007
Three meta-analyses published in 2007
Copyright Harvard Medical School, 2010. All Rights Reserved.
961
Relation between OSA and aldosterone
Among 114 patients with resistant HTN
72 had high probability for OSA
42 had low probability for OSA
On a high salt diet, 24hr urine aldosterone was
higher among those with a high probability for
OSA (13.6 vs. 9.8 g, p<0.05)
Calhoun et al. Chest 2004
Among 60 patients with proven OSA and
resistant hypertension
Plasma aldosterone was correlated with AHI
(r=0.44, p<0.01)
Pratt-Ubanama et al. Chest 2007
Effect of Spironolactone in resistant hypertension:
Anglo-Scandinavian Cardiac Outcomes Trials (ASCOT)
ASCOT-BLPA (N=19,257)
Amlodipine & perindopril Atenolol & bendroflumethiazide
Doxazosin if needed
4
th
drug added if BP not at target
Doxazosin if needed
Spironolactone (N=1411)
Other drug
Chapman et al. Hypertension 2007
Copyright Harvard Medical School, 2010. All Rights Reserved.
962
Effect of Spironolactone in resistant hypertension:
Anglo-Scandinavian Cardiac Outcomes Trials (ASCOT)
Chapman et al. Hypertension 2007
Mean BP fell from 157/85 to 135/76 with
spironolactone
BP =22/9 mmHg (other studies have found
similar effects in resistant hypertension)
Effect on BP was similar in both treatment arms
(whether already on diuretic or not)
Effect was somewhat larger if
Age >60 years
Female
diabetic
You refer him for polysomnography, demonstrating an apnea-
hypopnea index (AHI) of 20 events per hour (moderate OSA). He
begins CPAP and is now seeing you 2 months later. He uses CPAP
nightly, sleeps well, and says that he feels more energetic. He still
takes lisinopril, chlorthalidone, amlodipine, and metoprolol. His
exam is unchanged and his BP is now 157/88 (instead of 160/90).
Which one of the following is most likely to result in control of his
hypertension?
A) Spi ronol actone 25 mg/day ti trati ng to 100 mg/day as needed.
B) An exercise program of walking, 20 minutes three times per week
C) Referral back to the sleep clinic to adjust his CPAP mask or prescription
since the therapy is not working
D) Referral to ENT for laser-assisted uvuloplasty
E) Long-acting diltiazem 120 mg/d titrating to 300 mg/day as needed
Case 3, question 2
Copyright Harvard Medical School, 2010. All Rights Reserved.
963
Case 4
An 85 yo F is seeking a second opinion because her PCP advised
her to take lisinopril-HCTZ to lower her BP. She only takes OTC
medications, specifically a multivitamin, fish oil, and vitamin D. Her
clinic and home BPs over the past 2 years have ranged from 158/60
to 180/72, consistent with isolated systolic hypertension (ISH). She
has no diabetes, MI, or stroke, but her mother died from a stroke at
age 77. She eats a healthy diet, walks for exercise 5 times/week,
and does not drink alcohol.
She is a thin and fit elderly woman (BMI=22), BP is 172/65. She
has a 2/6 SEM, an S4, and no edema. Cre=0.8, UA is negative.
You should advise her that:
A. She should have 24-hour ABPM because she likely has white-coat HTN
B. ISH is normal with ageing and requires no therapy
C. Although ISH should be treated in most, there are no data pertaining to women
of her age
D. Taking lisinopril-HCTZ may reduce her chances of dying and/or developing
congestive heart failure during the next 2 years
Case 4, question 1
Copyright Harvard Medical School, 2010. All Rights Reserved.
964
Systolic Hypertension in Europe (Syst-Eur)
Objective: To determine whether antihypertensive treatment reduces
cardiovascular complications in older patients with elevated
SBP
Patients: 4695 patients, 60 years of age, with SBP 160219 mm Hg
and DBP <95 mm Hg
Treatments: Nitrendipine(1040 mg/day) with possible addition or
substitution of:
Enalapril (520 mg/day)
Hydrochlorothiazide (12.525 mg/day)
Placebo
Follow-up: 2 years (median)
Endpoint: Total stroke
Myocardial infarction
Adapted from StaessenJ A et al. Lancet. 1997;350:757-764.
Syst-Eur: Outcomes
*P=.003;

P=.03;

P=.12;

P<.001.
Adapted from StaessenJ A et al. Lancet. 1997;350:757-764.
P
e
r
c
e
n
t

R
e
d
u
c
t
i
o
n

0
5
10
15
20
25
30
35
40
45
42*
CHF Stroke Al l CHD Total CVD MI
26

29

30

31

Risk Reduction
Copyright Harvard Medical School, 2010. All Rights Reserved.
965
Hypertension in the Very Elderly Trial: (HYVET)
Objective: To determine whether treatment of systolic hypertension in
patients 80 years old lowers the risk of stroke
Design: Multicenter, randomized, double-blind, placebo-controlled
Patients: 3845 men and women >80 y (mean, 84 y) with sustained
SBP >160 mmHg
Treatments: Step 1: thiazide
Step 2: ACEI
Goal: SBP <150 mmHg
Placebo
Follow-up: 1.8 years (stopped early)
Endpoint: Fatal and non-fatal stroke
Beckett NS, et al. NEJ M 2008; 358:1887
HYVET: Outcomes
Fatal and
non-fatal
stroke
-80
-60
-40
-20
0
- 30
P
e
r
c
e
n
t

r
e
d
u
c
t
i
o
n
- 39*
- 28
- 64

- 21*
*p<0.05

p<0.001
CHF
Fatal
stroke
Al l -cause
mortal ity
Fatal and
non-fatal MI
Beckett NS, et al. NEJ M 2008; 358:1887
Copyright Harvard Medical School, 2010. All Rights Reserved.
966
An 85 yo F is seeking a second opinion because her PCP advised
her to take lisinopril-HCTZ to lower her BP. She only takes OTC
medications, specifically a multivitamin, fish oil, and vitamin D. Her
clinic and home BPs over the past 2 years have ranged from 158/60
to 180/72, consistent with isolated systolic hypertension (ISH). She
has no diabetes, MI, or stroke, but her mother died from a stroke at
age 77. She eats a healthy diet, walks for exercise 5 times/week,
and does not drink alcohol.
She is a thin and fit elderly woman (BMI=22), BP is 172/65. She
has a 2/6 SEM, an S4, and no edema. Cre=0.8, UA is negative.
You should advise her that:
A. She should have 24-hour ABPM because she likely has white-coat HTN
B. ISH is normal with ageing and requires no therapy
C. Although ISH should be treated in most, there are no data pertaining to women
of her age
D. Taking lisinopril-HCTZ may reduce her chances of dying and/or
developing congestive heart failure during the next 2 years
Case 4, question 1
Case 5
Copyright Harvard Medical School, 2010. All Rights Reserved.
967
A 27 year old woman who is 30 weeks pregnant presents
for a routine OB visit and is found to have peripheral
edema but no other symptoms, a BP of 150/90
(previously normotensive) and 2+protein by dip, later
quantified as 1.3 g protein/g of creatinine(previously
normal). She is placed on bed rest; in addition, which
of the following best describes the next step to
address her high BP:
a. Begin triamterene-hydrochlorothiazide
b. Intravenous bolus of Mg-Sulfate, followed by oral
magnesium supplementation
c. Begin enalapril
d. Advise a low salt (<2000 mg) diet
e. Begin methyldopa
f. No anti-hypertensive medications can be used
Case 5, question 1
Fetopathic effects of AngiotensinBlockade
1
st
Trimester
3.7-fold increased risk of CV malformations (ASD, VSD, patent
ductus, pulmonic stenosis)
4.4-fold increased risk of neurologic malformations (spina bifida,
microcephaly)
Risk not seen among infants exposed to other antihypertensive
medications during the 1
st
trimester
WO Cooper et al. New Engl J Med 2006: 354:2443
2
nd
and 3
rd
Trimesters
Renal failure
Renal dysplasia
Fetal hypotension
Oligohydramnios
Pulmonary hypoplasia
IUGR
A Quan. Early Hum Devel 2006; 82:23
Copyright Harvard Medical School, 2010. All Rights Reserved.
968
Drug Pregnancy class
Methyldopa
Amiloride
B
Labetalol
Metoprolol
Prazosin
Nifedipine
Hydralazine
HCTZ
Furosemide
C
ACE-inhibitors
Angiotensin receptor blockers
Spironolactone
Triamterene
Atenolol
D
Pregnancy risk categories of BP meds
Pregnancy class
A. Good human studies show no increase in risk
B. Animal studies show no increased risk without human data, or animal studies how harm but
human studies do not show harm
C. No human data combined with a lack of animal studies or studies showing harm
D. Human studies show possible harm, but benefit may outweigh the risk
X. Human studies show congenital abnormalities, and drug should not be used
A 27 year old woman who is 30 weeks pregnant presents
for a routine OB visit and is found to have peripheral
edema but no other symptoms, a BP of 150/90
(previously normotensive) and 2+protein by dip, later
quantified as 1.3 g protein/g of creatinine(previously
normal). She is placed on bed rest; in addition, which
of the following best describes the next step to
address her high BP:
a. Begin triamterene-hydrochlorothiazide
b. Intravenous bolus of Mg-Sulfate, followed by oral
magnesium supplementation
c. Begin enalapril
d. Advise a low salt (<2000 mg) diet
e. Begin methyldopa
f. No anti-hypertensive medications can be used
Case 5, question 1
Copyright Harvard Medical School, 2010. All Rights Reserved.
969
Case 6
You are discussing the initiation of an ACEI with a 37 year old African
American man with hypertension and normal kidney function. He
asks you about potential side effects. All of the following are true
about side effects of ACEI except:
a. Cough and angioedema occur at rates of 5-20% and 0.7%,
respectively, depending on the survey.
b. Angioedemais less common in African Americans.
c. Angioedemaoccurs with both ACEI and ARB.
d. Cough results from inhibition of bradykininmetabolism.
e. Refractory hyperkalemiaoccurs with similar frequency
compared to other drugs among individuals with normal
kidney function.
f. Refractory hyperkalemiaoccurs with similar frequency
compared to other drugs among individuals with CKD.
g. Risk factors for hyperkalemiawhen starting an ACEI
include CKD, NSAIDs, and DM
Case 6, question 1
Copyright Harvard Medical School, 2010. All Rights Reserved.
970
Side effects of ACEI
Cough
Incidence is 5-20%, depending on survey
Bradykininmediated
More common in women
Disappears when switched to ARB
AH Israili et al. Ann Intern Med 1992; 117:234
Angioedema
Incidence is ~0.7%
3x higher risk in African Americans, 1.6x higher risk in elderly
Can occur anytime, but usually within the first week
Mechanism: bradykinin, complement system, autoantibodies
May not be safe to switch to ARB
J B Kostis et al. Arch Intern Med 2005; 165:1637
Hyperkalemia
In pooled analysis of 6 randomized trials of CKD patients,
withdrawal because of refractory hyperkalemiawas 1.4% (ACEI)
vs. 0.95% (non-ACEI)
Lancet 2001; 357:1601
You are discussing the initiation of an ACEI with a 37 year old African
American man with hypertension and normal kidney function. He
asks you about potential side effects. All of the following are true
about side effects of ACEI except:
a. Cough and angioedema occur at rates of 5-20% and 0.7%,
respectively, depending on the survey.
b. Angioedema is less common in African Americans.
c. Angioedemaoccurs with both ACEI and ARB.
d. Cough results from inhibition of bradykininmetabolism.
e. Refractory hyperkalemiaoccurs with similar frequency
compared to other drugs among individuals with normal
kidney function.
f. Refractory hyperkalemiaoccurs with similar frequency
compared to other drugs among individuals with CKD.
g. Risk factors for hyperkalemiawhen starting an ACEI
include CKD, NSAIDs, and DM
Case 6, question 1
Copyright Harvard Medical School, 2010. All Rights Reserved.
971
Final Question
An internal medicine colleague has read the recent
ACCORD trial and is wondering what the goal
blood pressure should be for the patients she
follows with chronic kidney disease.
You tell her that:
a. Because of consistent and convincing evidence from
multiple RCTs that aggressive BP control results in lower
mortality and CV events in in patients with CKD, the official
recommendation is a BP target of <130/80.
b. Although we lack consistent and convincing evidence
from multiple RCTs that aggressive BP control results
in lower mortality and CV events in in patients with
CKD, the official recommendation is a BP target of
<130/80.
c. Based upon the results of the ACCORD trial, and other
recent trials, the official recommendation for patients with
CKD has been changed to a target of <140/90.
Final question
Copyright Harvard Medical School, 2010. All Rights Reserved.
972
Disclosures
None
Copyright Harvard Medical School, 2010. All Rights Reserved.
973
Coronary Artery Disease and
Renal Disease
David Charytan, MD, MSc
Associate Staff Physician
Brigham & Womens Hospital
Instructor in Medicine
Harvard Medical School
Disclosures
Dr Charytan receives funding from the
American Society of Nephrology, Paul
Teschan Research Fund (DCI), and the
American Heart Association.
Copyright Harvard Medical School, 2010. All Rights Reserved.
974
USRDS; 2005 ADR.
(A) Adjusted age-
specific MI rates
(B) Adjusted cause-
specific mortality rates
Go, A. S. et al. N Engl J Med 2004;351:1296-1305
Cardiovascular Event Rate in CKD
Copyright Harvard Medical School, 2010. All Rights Reserved.
975
Herzog CA. NEJM 1998; 339: 799-805
Estimated Cumulative Mortality Following First Myocardial
Infarction in Chronic Dialysis Patients
Anavekar NS. N Engl J Med 2004;351:1285-1295
Estimated Cardiovascular Death and Reinfarction Rates Three Years after MI in
Patients with Pre-ESRD CKD
Copyright Harvard Medical School, 2010. All Rights Reserved.
976
Relative Risk of ESRD vs. CV Death in
Elderly Patients with CKD
2 Year Incidence Rates
CV Death ESRD
CKD(+)/DM(+) 32.3% 6.1%
CKD(+)/DM(-) 29.5% 2.9%
CKD(-)/DM(-) 10.3% 0.1%
Collins AJ. KI 2003;64(S87):S24-31
Foley RN. JASN 2005;16:489-95
Summary: MI Risk in CKD
There is a high risk of a cardiovascular events in
patients with CKD that is directly proportional to
the degree of impairment in GFR and is 10-20
fold higher in patients with ESRD compared to
those with normal renal function
MI is a catastrophic event in patients with CKD
Risk of death from CVD markedly exceeds the
risk of progression to ESRD in elderly patients
with CKD
Copyright Harvard Medical School, 2010. All Rights Reserved.
977
Etiology of Increased Risk of CVD
Events CKD
High burden of fixed CAD
Inflammation
Oxidative stress
Coronary artery calcification
Traditional CV risk factors ( lipids,
diabetes, hypertension, age)
Poor performance of diagnostic tests
Underutilization of standard CV therapies
GFR
>60
GFR
45-
59
GFR
30-
44
GFR
<30
Nakano T. AJKD. 2010; 55:21-30
--
0.9
0.3
0.002
P value Calcification P Value Advanced
Atherosclerosis
eGFR
mL/min/1.73m
2
1.00 (Ref)
0.95 (0.46-1.94)
1.43 (0.69-2.95)
4.71 (1.78-12.50)
--
0.3
0.05
0.02
1.00 (Ref)
1.40 (0.76-2.55)
2.02 (0.99-4.15)
3.02 (1.22-7.49)
60
45-59
30-44
<30
Adjusted Odds Ratio for Advanced Atherosclerosis and
Calcified Lesions
Copyright Harvard Medical School, 2010. All Rights Reserved.
978
Incidence of Obstructive CAD in Advanced
CKD
60.2% incidence of obstructive CAD in new
dialysis patients (Joki, N. NDT. 1997;12:718-23)
Nearly with CAD have multi-vessel involvement
42% of transplant candidates have at least one-
vessel disease (Marwick, TH. Transplantation. 1990;49:100-103)
~50% incidence of CAD in asymptomatic
dialysis patients (Ohtake, T. JASN. 2005;16:1141-8. DeFilippi C. JAMA/ 2003;
290:353-9.)
Nearly with CAD have multi-vessel involvement
Inflammation, Oxidative Stress and CKD
All biomarkers of oxidative stress and
inflammation are elevated in patients with
stage 3-5 CKD (Payson BP. KI. 2004;65:1009-1016)
Not linearly correlated with eGFR but levels
generally higher in ESRD than pre-dialysis CKD
<0.001 0.61 0.29 Carbonyls nmol/
mg protein
Adapted from Oberg BP. KI 2004;65:1009-16
0.001 6.5 2.1 IL-6 pg/mL
<0.001 303 415 Thiols mol/L
<0.001 0.046 0.036 F2-isoprostanes
ng/mL
0.02 3.9 1.8 CRP mg/L
P value CKD Patients Healthy Subjects
Inflammatory and Oxidative Stress Markers in Individuals with and without CKD
Copyright Harvard Medical School, 2010. All Rights Reserved.
979
Inflammation, Oxidative Stress and CKD
Both transient and persistent elevations in
inflammatory markers are associated with a
doubling in the risk of CV death in HD
patients
CRP persistently elevated to >10mg/L 20% and
transiently elevated in 26% (Wendy PJ. NDT. 2006;21:1588-1595,
Wanner C. NDT. 2002;17:29-32)
Elevated CRP associated with doubling of
mortality in stage 3-4 CKD patients (Menon V. KI.
2005;68:766-72)
Coronary Calcification ?
More common in patients with ESRD or CKD than
patients with normal renal function, particularly
among those with diabetes (Goodman WG. NEJM. 2000;18;1478-
83, Kramer H. JASN 2005;16:507-13)
Independently linked to risk of CV death in
patients with ESRD as well as in general
population (Matsuoka M. Clin Exp Neph 2004;8:54-58. Block GA. KI 2007;.
Block GA. KI 2007;78:438-41)
Moderately well correlated with presence of
coronary atherosclerosis
Sensitivity and specificity for detection of CAD vary
widely
Depending on cut point test performance may be poor
(Fujimoto N. Clin Chem Acta 2006;367:98-102. Sharple EJ. AJKD 2004;43:313-19)
Copyright Harvard Medical School, 2010. All Rights Reserved.
980
Coronary Calcification ?
No significant effect on mortality when patients randomized
to non-calcium containing binders compared with calcium
containing binders in long-term study (Kidney Int. 2007;72:1130-7. )
No significant relationship of eGFR or Cystatin C with CAC
score among patients with mild to moderate CKD in MESA
study of >6000 participants ( IX JH. JASN 2008;19:579-85)
Coronary calcification associated with other know CV risk
factors such as P04, serum calcium,PTH, fetuin A and FGF-
23 (Adeney KL. JASN 2009; 20:381-7, Guttierez OM. Circ, 2009;119:2545-52)
Coronary calcification is associated with calcification in large
conduit arteries
Resistant hypertension
Increased CV work load
Vascular Calcification Summary
Excellent marker of risk of all-cause and
cardiovascular mortality
Pathogenic role remains uncertain
Experimental studies suggest that calcification
stabilizes coronary plaque (H, Virmani R, Younis H, Burke AP, Kamm
RD, Lee RT. Circ 2001;103:1051-1056)
May be non-pathogenic or extra-coronary
calcification may be more important risk-factor
Copyright Harvard Medical School, 2010. All Rights Reserved.
981
Traditional Risk Factors and CKD
Most traditional
risk factors such
as advanced age,
hypertension,
hyperlipidemia are
common in
patients with CKD
Uhlig K. Sem in Dial 2003;16:118-127
0
10
20
30
40
50
60
70
80
90
T
o
ta
l C
h
o
le
s
t
e
r
o
l >
2
4
0
m
g
/d
L
T
r
ig
ly
c
e
r
id
e
s
>
2
0
0
m
g
/d
L
L
D
L
>
1
3
0

m
g
/
d
L
H
y
p
e
r
te
n
s
io
n
D
ia
b
e
t
e
s
S
m
o
k
in
g
HD
CKD 1-4
%

o
f

P
o
p
u
l
a
t
i
o
n
But Framingham risk
score is poorly predictive
in CKD (5 year-c statistic
only 0.62 in men) and
standard factors only
partly account for
elevated CV mortality
rates in CKD patients
Weiner D. JACC
2007;50:217-24
Fleishman EH. Clin Nephrol
200156:221-230.
Diagnosis of Ischemia in CKD
Differential symptoms of ischemia in CKD vs.
normal
Differential sensitivity and negative predictive
value of non-invasive testing compared with
general population
ECG may be difficult to interpret due to baseline
abnormalities (Zuber, et al. NDT 1989;4:632-4)
ST Elevation MI is ~50% less frequent in patients with CKD than in the
general population (Herzog CA. ASN 2004)
Dialysis-18.5% Moderate CKD-17.4% Normal Function-35.8%
Cardiac enzymes may be less sensitive/specific
in CKD
Copyright Harvard Medical School, 2010. All Rights Reserved.
982
Differential Symptoms of MI: CKD vs.
Normal Renal Function
Sosnov, J. AJKD. 2006;47:378-84
1.08 (0.86-1.37) Dizzyness/lightheadeness/syncope
1.35 (1.13-1.62) Cough, dsypnea
0.87 (0.66-1.13) Chest tightness
0.57 (0.46-0.70) Chest pain/pressure, diaphoresis
0.52 (0.42-0.64) Arm Pain/Shoulder Pain
Multivariate OR
(95% CI)
Symptoms
Stress Testing in ESRD
Maximal exercise stress test frequently non-diagnostic
due to limited functional capacity/mobility
Pharmacologic stress testing with contemporary tools
more frequently diagnostic
Accuracy in HD (Dahan M. AJKD 2002;40:737-44)
Dipyridamole/Stress-ThalliumSensitivity
92%/Specificity-89%
Dipyridamole/Stress-Echo-Sensitivity 83%/Specificity 84%
Good utility as prognostic tools-6-fold increase in risk of MI
and 4-fold increase in risk of cardiac death in presence of
inducible ishemia (Rabbat DG. JASN 2003;14:431-9)
Negative predictive value may be low due to high pre-
test probability of CAD
Copyright Harvard Medical School, 2010. All Rights Reserved.
983
Cardiac Enzymes in CKD
CK and CK-MB
~88-100% of asymptomatic dialysis patients without
clinically suspected ischemia have intermittent
elevations in CK-MB (Green TR et al. Clin Neph 1986;25:22-7)
Troponin
ESRD
~20% of patients with ESRD have elevated Troponin
T (Apple FS. Circulation 2002;106:2941-1945, Illiou MC. AJKD 2003:42:513-23)
~7-18% have Troponin I elevation (Freda BJ. JACC 2002;40:2065-
71, Illiou MC. AJKD 2003:42:513-23)
CKD (Lamb EJ. AJKD 2007; 49:507-16)
Troponin I-elevated in 18% of pre-ESRD CKD patients
Troponin T-elevated in 43% of pre-ESRD CKD
patients
Summary: Dx of CAD
Diagnosis of CAD/MI can be challenging in
CKD patients
Standard non-invasive tests may have low
negative and positive predictive values
Maintenance of a high index of suspicion
is crucial
Copyright Harvard Medical School, 2010. All Rights Reserved.
984
Utilization of Cardiovascular Therapies
in CKD
___________________________________
CKD No CKD P
_______________________________________________
Angiography
All Patients 25.2% 46.8% <0.0001
Appropriate Candidates 29.6% 49.6% <0.0001
Revascularization After
Angiography 54.7% 62.0% <0.0001
_________________________________________________________
Chertow G. JASN 2005;15:2462-2468
Low Utilization of Invasive Diagnostic and Therapeutic Procedures
in Dialysis Patients after MI
Charytan D.
Am Heart J
2006;152:55
8-64
Copyright Harvard Medical School, 2010. All Rights Reserved.
985
Low Use of Medical Therapy in Patients with
ESRD
P
e
r
c
e
n
t

R
e
c
e
i
v
i
n
g

M
e
d
i
c
a
t
i
o
n

w
i
t
h
i
n

9
0

D
a
y
s

o
f

M
I
Winkelmayer W. AJKD. 2006
Treatment of Stable CAD in the
General Population
Standard of care derived from large,
randomized clinical trials
ASA/anti-platelet agents
Statins
ACE-I/ARB
Beta blocker
Percutaneous/surgical revascularization
Copyright Harvard Medical School, 2010. All Rights Reserved.
986
CKD ESRD HTN DM Smoking
Condition
Use of Baseline Conditions as Exclusion Criteria in Large
Cardiovascular Trials
F
r
e
q
u
e
n
c
y
Charytan D. KI 2006;70:2021-30
Anti-Platelet Agents
No randomized evidence for CAD endpoints in ESRD
population
2-fold increase of the risk of bleeding with Clopidogrel +
full dose ASA in ESRD but no change when with
Clopidogrel alone (Kauffman JS. JASN 2003;14:2313-2321, Dember, L. M. et al. JAMA
2008;299:2164-2171)
Risk of CV death/MI decreased in general population of
patients with hx of CVD administered Clopidogrel 75
mg/day + ASA (75-162 mg/day) vs. ASA alone, and
appears to be similar effective in patients with CKD (Bhatt,
DL. NEJM. 2006, Keltai M. Eur J Card Prev Rehab 2007;14:312-18 )
Retrospective analyses suggest that relative efficacy of
ASA is similar in patients with vs. without CKD(McCullough PA.
Am Heart J 2002;144:226-232, Berger AK. JACC 2003;42:201-8)
Recommendations:
ASA-all patients
Clopidogrel-consider with low dose ASA for patients with
documented CAD and low bleeding risk
Copyright Harvard Medical School, 2010. All Rights Reserved.
987
Statins
4D Trial (Wanner, C. NEJM 2005;353:238-248)
1297 Diabetic HD patients with LDL 80-190
mg/dL
Lipitor 20 mg/day vs. placebo
HR for CV events 0.92 (0.77-1.10)
AURORA Trial (Fellstrom BC. NEJM 2009;360:1395-1407)
2776 prevalent HD patients (40% with CAD)
Rosuvastatin 10 mg/day vs. placebo
HR for CV Death 1.0 (0.85-1.16)
No difference in non-fatal MI (2.1% vs. 2.5%)
Statins
Multiple trials/subgroup analyses demonstrating efficacy in
general population and moderate CKD population
Planet 1 and Planet 2 Study
Proteinuric renal disease from diabetic (Planet 1, n=325) or non-
diabetic CKD (Planet 2, n=220)
Atorvastatin 40 mg/daily vs. Rosuvastatin 10 mg or 40 mg/daily-1
year follow-up
Results
Atorvastatin significantly reduces proteinuria compared with baseline
No effect of Rosuvastatin on proteinuria
Greater loss of eGFR at 1 year in Rosuvastatin 10 mg (4 mL/min/1.73
m
2
) and 40 mg (8 mL/min/1.73 m
2
) groups than in Atorvastatin group (1-
2 mL/min/1.73 m
2
) in Planet 1 Study
Greater loss of eGFR in high dose Rosuvastatin group vs. Atorvastatin
group in Planet 2
Copyright Harvard Medical School, 2010. All Rights Reserved.
988
Statins
Recommendation:
Statins for most patients with CKD
May consider withholding in ESRD patients
with LDL 160
Atorvastatin may be preferable to
Rosuvastatin for individuals with pre-ESRD
ACE Inhibitors/ARB
Fosodial Trial (Zannad, F. KI. 2006;70:1318-24)
397 HD patients on HD > 6 months
Fosinopril vs. placebo
HR for CV events 0.93 (0.68-1.26) but underpowered due
lower than expected event rate
Open label trial of ARB vs. no therapy in 360 ESRD
patients (Suzuki, H. AJKD 2008;52:501-6)
HR for fatal and non-fatal CV events 0.51 (0.33-0.79)
HR for mortality 0.64 (0.39-1.06)
Recommendation:
ACE-I/ARB-consider for all patients with stage 3-5 CKD
Efficacy in ESRD uncertain but reasonable first line BP
agent
Copyright Harvard Medical School, 2010. All Rights Reserved.
989
Beta-Blockers
Carvedilol in ESRD (Cice, G. JACC.2003;41:1438-44)
114 HD patients with dilated cardiomyopathy, EF<35% &
NYHA class II or III CHF
Carvedilol 25 mg QD vs. Placebo for 24 months
HR for all-cause mortality 0.51 (0.32-0.82)
CV death HR 0.32 (0.18-0.57)
Non-fatal MI (11% vs. 12%)
Safety: side effects in < 10%
Recommendation
Administer to all patients with low EF (grade 1
recommendation)
Consider for all patients post-MI regardless of EF
CABG
CABG-No randomized evidence of efficacy in
the dialysis population
Surgical mortality in ESRD (Beckermann, J. J Throac Cardivasc Surg
2006;131:1261-6).
Off-pump-11.7% mortality at 30 days
On-pump-12.5% mortality at 30 days
Surgical mortality in general population-3-5%
Absolute 5-year mortality reduction of CABG vs.
medical therapy in general population is 5.6% (Yusuf, S.
Lancet. 1994;344:563-70)
Copyright Harvard Medical School, 2010. All Rights Reserved.
990
Percutaneous Revascularization
Percutaneous Angioplasty-No randomized
evidence of efficacy in the stage 3-5 CKD
population
In general population trials of patients with stable CAD
percutaneous therapy does not decrease mortality or risk
of MI vs. medical therapy in the general population
(Henderson, RA. JACC. 2003:42:1161-70)
May be inferior to CABG for treatment of multi-vessel
disease in general population (Hannan, EL. NEJM. 2006;352:2174-83)
2-Vessels-RR of death CABG vs. PTCA ~0.75
3-Vessels-RR of death CABG vs. PTCA ~0.64-0.74
Operative Mortality
1.7
3.5
7.5
9.8
0
1
2
3
4
5
6
7
8
9
10
N
o
r
m
a
l
/
E
a
r
l
y
C
K
D
S
t
a
g
e

3
S
t
a
g
e

4
S
t
a
g
e
5
/
E
S
R
D
Renal Function
I
n
-
h
o
s
p
i
t
a
l

M
o
r
t
a
l
i
t
y

(
%
)
Yeo K, et al Am J Card. 2008:101;1269-
74
19
7.6
13.2
18.5
8.3
12.3
0
4
8
12
16
20
Death and
Non-Fatal MI
Death Non-Fatal MI
Endpoint
P
e
r
e
c
e
n
t

w
i
t
h

O
u
t
c
o
m
e

a
t

4
.
6

Y
e
a
r
s
Pecutaneous
Intervention
Medical
Therapy
Boden WE. NEJM. 2007:356:1503-16
Copyright Harvard Medical School, 2010. All Rights Reserved.
991
Revascularization in CKD/ESRD
Percutaneous Angioplasty
? More effective than medical therapy
? Less effective than CABG
Post MI-1 year survival in ESRD (Chertow, GM. AJKD.
2000;35:1044-51)
Medical Tx-45%
PTCA-54%
CABG-69%
First Revascularization-RR of 2 year survival
CABG vs PCI -- 0.80 (0.76-0.84) (Herzog CA. Circ.
2002;106:2207-11)
Serruys PW et al. N Engl J Med 2009;360:961-972
Outcomes in Unprotected Left Main Stenting vs. CABG
Copyright Harvard Medical School, 2010. All Rights Reserved.
992
Randomization to Coronary Angiography vs.
Medical Therapy in patients with Acute
Coronary Syndromes and CKD
Hospitalization
Death
Charytan DM. CJASN 2009; 4:1032-43
Revascularization in CKD/ESRD
Recommendation
CABG
Risk benefit ratio must be considered carefully
Indicated for most patients with left main disease
or 3VD with proximal LAD, unless multiple
comorbidities
Percutaneous Interventions
Indicated for medically refractory angina
Role vs. medical therapy remains undefined
Left main PCI
Potentially treatment of choice for ESRD/advanced
CKD, but more data needed
Copyright Harvard Medical School, 2010. All Rights Reserved.
993
Summary: Therapy of CAD in CKD
Quality of available data is poor and further
studies needed
In general, evidence points towards benefits
from standard therapies
Increased risk of interventions and lack of
randomized evidence in CKD limit
recommendation for revascularization to
patients with high-risk anatomy or medically
refractory symptoms
Question #1
A 72 year-old peritoneal dialysis patient with type II diabetes,
elevated CRP and low albumin complains of occasional exertional
chest pain and shortness of breath. An exercise stress test is
ordered and is stopped after 3.5 minutes due to fatigue. EKG is
without ischemic changes. Which of the following statements is
most correct?
A) There is a low likelihood of obstructive coronary disease, no further
evaluation is needed
B) An echocardiogram should be ordered to evaluate ejection fraction
and rule out wall motion abnormalities.
C) Pharmacologic stress testing should be ordered to definitively rule
out obstructive coronary disease
D) Pharmacologic stress testing should be ordered. If non-invasive
testing is negative, consider proceeding to cardiac catheterization
Copyright Harvard Medical School, 2010. All Rights Reserved.
994
Question #1
A 72 year-old peritoneal dialysis patient with type II diabetes,
elevated CRP and low albumin complains of occasional exertional
chest pain and shortness of breath. An exercise stress test is
ordered and is stopped after 3.5 minutes due to fatigue. EKG is
without ischemic changes. Which of the following statements is
most correct?
A) There is a low likelihood of obstructive coronary disease, no further
evaluation is needed
B) An echocardiogram should be ordered to evaluate ejection fraction
and rule out wall motion abnormalities.
C) Pharmacologic stress testing should be ordered to definitively rule
out obstructive coronary disease
D) Pharmacologic stress testing should be ordered. If non-
invasive testing is negative, consider proceeding to cardiac
catheterization
Question #2
A 45 year-old previously stable male with an
estimated GFR of 30 cc/min/1.73m
2
develops
the new onset of diffuse, left-sided chest pain
described as tightness accompanied by
dyspnea. Physical exam is remarkable for blood
pressure of 140/80 and 1+ pitting edema with
clear lungs. EKG shows LVH but is otherwise
unremarkable. The patient is non-diabetic, has
normal cholesterol, and does not smoke. He has
otherwise been stable. Which one of the
following statements about the evaluation and
management is most correct?
Copyright Harvard Medical School, 2010. All Rights Reserved.
995
Question #2
A) Diuretics should be increased to treat
probable pulmonary congestion
B) Beta-adrenergic agonists should be
prescribed to treat asthma
C) Stress-testing and echo-cardiography shoud
be considered to rule out the presence of
structural heart disease
D) The atypical nature of the symptoms and lack
of traditional risk factors make coronary disease
unlikely. Stress testing can be safely deferred
References
Lentine K. Cardiovascular risk assessment among
potential kidney transplant recipients: Approaches and
controversies. AJKD 2010;55:152-67.
Patel T. Cardiovascular complications in diabetic kidney
disease. Sem Dial 2010:23:169-77.
Stenvinkel P. Emerging biomarkers for evaluating
cardiovascular risk in the chronic kidney disease patient:
how do new pieces fit into the uremic puzzle? CJASN
2008;3:505-21.
Fellsrom B. Cardiovascular disease in patients with renal
disease: the role of statins. Curr Med Res Opinion
2009;25:271-85
Copyright Harvard Medical School, 2010. All Rights Reserved.
996
Disclosures
Dr Charytan receives funding from the
American Society of Nephrology, Paul
Teschan Research Fund (DCI), and the
American Heart Association.
Copyright Harvard Medical School, 2010. All Rights Reserved.
997
Garasic 2010
Renovascular
Disease
Joseph M. Garasic, M.D.
Division of Cardiology
Massachusetts General Hospital
Boston, Massachusetts
Somethings Old, Somethings New,
& Lingering Questions
Garasic 2010
Presenter Disclosure
Information
The following relationships exist related to this
presentation:
Consultant / Equity Interest Access Closure,
Inc.
Speakers Bureau Sanofi-BMS Partnership
Copyright Harvard Medical School, 2010. All Rights Reserved.
998
Garasic 2010
Defining the Problem
RAS is an important cause of secondary
hypertension
Renovascular disease under-appreciated as
cause of CRF
23% of malignant hypertension is the result
of renovascular causes
Not all patients with RAS are hypertensive
as a result
Garasic 2010
Progress in Renovascular Disease
1) The disease
2) Clinical diagnosis and medical management
3) Laboratory diagnosis / imaging modalities
4) Patient selection: who benefits from intervention?
5) Limiting contrast-induced nephropathy
6) Atheroembolic protection
7) Expanding the pool of eligible patients / interventions
8) Limiting restenosis
Copyright Harvard Medical School, 2010. All Rights Reserved.
999
Garasic 2010
Progress in Renovascular Disease
1) The Disease
2) Clinical diagnosis and medical management
3) Laboratory diagnosis / imaging modalities
4) Patient selection: who benefits from intervention?
5) Limiting contrast-induced nephropathy
6) Atheroembolic protection
7) Expanding the pool of eligible patients / interventions
8) Limiting restenosis
Garasic 2010
Etiology of Renal Artery Stenosis
Fibromuscular dysplasia
Atherosclerosis
Polyarteritis Nodosa
Radiation-induced
Takayasus arteritis
Copyright Harvard Medical School, 2010. All Rights Reserved.
1000
Garasic 2010
Renal Physiology
Garasic 2010
L. Gabriel Navar and L. Lee Hamm
Copyright Harvard Medical School, 2010. All Rights Reserved.
1001
Garasic 2010
Mark A. Pohl
Garasic 2010
Progress in Renovascular Disease
1) The Disease
2) Clinical diagnosis and medical management
3) Laboratory diagnosis / imaging modalities
4) Patient selection: who benefits from intervention?
5) Limiting contrast-induced nephropathy
6) Atheroembolic protection
7) Expanding the pool of eligible patients / interventions
8) Limiting restenosis
Copyright Harvard Medical School, 2010. All Rights Reserved.
1002
Garasic 2010
Clinical Clues
Onset of diastolic hypertension after age 55
Refractory or malignant hypertension
Development of resistant hypertension in a
previously well-controlled patient
Progressive increase in Creatinine, even if still
normal
Presence of atherosclerotic macrovascular disease
elsewhere heightens suspicion
Left heart failure out-of-proportion to LV
dysfunction or ischemic burden
Clinically silent RAS
Garasic 2010
What is medical therapyfor
renovascular disease?
Copyright Harvard Medical School, 2010. All Rights Reserved.
1003
Garasic 2010
Natural History of Renal Artery Stenosis
Caps et al. Circulation 1998; 98:2866-2872.
Role of lipid lowering and
Aggressive risk factor modification?
Garasic 2010
01/24/2002
Copyright Harvard Medical School, 2010. All Rights Reserved.
1004
Garasic 2010 09/05/2002
Garasic 2010
05/12/2006
Copyright Harvard Medical School, 2010. All Rights Reserved.
1005
Garasic 2010 02/06/2007
Garasic 2010
Trend of Serum Creatinine
0
0.2
0.4
0.6
0.8
1
1.2
1.4
1.6
Creat
9/6/02 to 4/7/07
Copyright Harvard Medical School, 2010. All Rights Reserved.
1006
Garasic 2010
Progress in Renovascular Disease
1) The Disease
2) Clinical diagnosis and medical management
3) Laboratory diagnosis / imaging modalities
4) Patient selection: who benefits from intervention?
5) Limiting contrast-induced nephropathy
6) Atheroembolic protection
7) Expanding the pool of eligible patients / interventions
8) Limiting restenosis
Garasic 2010
Clinical syndrome most important in patient
selection
Various diagnostic modalities:
Serologic markers
Duplex ultrasound - in experienced hands can predict
with great accuracy the presence or absence of
significant RAS
Captopril renal scan - 10-25% false negative
MR angiography - rare false negatives / common false
positives. Equipment/experience dependent
Contrast angiography
Screening for Renovascular Disease
Copyright Harvard Medical School, 2010. All Rights Reserved.
1007
Garasic 2010
The Loss of MR Angiography
Nephrogenic Fibrosing Dermopathy
Mostly in HD Patients
Crippling skin thickening and contracture
Associated with Gadolinium exposure
90 Reported cases USA / 200 Wordwide
Garasic 2010
The Loss of MR Angiography The Loss of MR Angiography
Nephrogenic Fibrosing Dermopathy
Mostly in HD Patients
Crippling skin thickening and contracture
Associated with Gadolinium exposure
90 Reported cases USA / 200 Wordwide
Whats Old is New: Renal
Duplex
Copyright Harvard Medical School, 2010. All Rights Reserved.
1008
Garasic 2010
Duplex U/S for Renovascular Disease
Prospective Duplex U/S evaluation and Renal Angiography in 102 pts
Goal: Validate renal artery U/S as a viable non-invasive modality
Drawbacks:
Time and labor intensive
Technologist dependent
Not available
NPO
Requires a cooperative patient
Olin et al. Ann Intern Med. 1995; 122:833-838.
Degree Stenosis Renal PSV RAR
Normal < 180 cm/sec < 3.5
< 60% > 180 cm/sec < 3.5
> 60%
< or > 180 cm/sec
> 3.5
Occlusion No signal No signal
Garasic 2010
Screening Aortography
RN
LN
Copyright Harvard Medical School, 2010. All Rights Reserved.
1009
Garasic 2010
Onset of hypertension at 30 years of age or severe hypertension at 55 years of
age* (Class I; LOE B)
Accelerated, resistant, or malignant hypertension* (Class I: LOE C)
Unexplained atrophic kidney or size discrepancy 1.5 cm between
kidneys (Class I; LOE B)
Sudden, unexplained pulmonary edema (Class I; LOE B)
Unexplained renal dysfunction, including individuals starting renal
replacement therapy (Class IIa; LOE B)
Development of new azotemia or worsening renal function after
administration of an ACE inhibitor or ARB agent (Class I; LOE B)
Multivessel coronary artery disease or peripheral arterial disease (Class
IIb; LOE B)
Unexplained congestive heart failure or refractory angina (Class IIb; LOE C)
New Recommendations
for RAS Screening: AHA Guidelines
Hirsch AT. Circ 2006; 113:e463
White et al. Circulation 2006;114;1892-1895
Garasic 2010
Progress in Renovascular Disease
1) The Disease
2) Clinical diagnosis and medical management
3) Laboratory diagnosis / imaging modalities
4) Patient selection: who benefits from intervention?
5) Limiting contrast-induced nephropathy
6) Atheroembolic protection
7) Expanding the pool of eligible patients / interventions
8) Limiting restenosis
Copyright Harvard Medical School, 2010. All Rights Reserved.
1010
Garasic 2010
What Are the Goals of
Treatment for RAS?
Control hypertension
Aid in medical management
Prevent deterioration in renal function
Forestall need for dialysis
Defer death and disability
Garasic 2010
Hypertension and RAS
Among 152 patients with Unilateral or Bilateral RAS
undergoing surgical revascularization:
90% had improvement in BP control
Only 15% had cure of hypertension
Among 20 published series of PCI for atherosclerotic
renal artery disease:
54% had improvement in hypertension
9% had cure of hypertension
Hansen et al. J Vasc Surg 1992;16;319-31.
Copyright Harvard Medical School, 2010. All Rights Reserved.
1011
Garasic 2010
Chronic Renal Insufficiency and RAS
Who Benefits From Revascularization?
Trial of 51 patients with Creat>2.0 before
revascularization with >75% Bilateral RAS:
67% had improvement in renal function
27% had stabilization in renal function
Only 6% had worsening in renal function
No demonstrated impact upon mortality
Novick et al. J Urol 1983; 129:907-12.
Garasic 2010
Experimental Data supporting Stenting for
Preservation of Renal Function
61 vessels in 31 patients with global obstructive
atherosclerotic renal disease
All with chronic renal insufficiency (Creat 1.5 4.0)
Stenting with non-articulated Palmaz stents
Follow-up Renal U/S, Serum Creat , BP measurements:
- Improvement in reciprocal slope of serum creatinine
- Improved BP control (SBP from 17021 Pre-stent vs. 148
15mmHg Post-stent; p<0.001)
- Restenosis (>50%) in only 1 of 61 vessels
- Stabilization of pole-to-pole renal dimension
Watson et al. Circulation. 2000; 102:1671-1677.
Copyright Harvard Medical School, 2010. All Rights Reserved.
1012
Garasic 2010
Watson et al. Circulation. 2000; 102:1671-1677.
IMPROVED
STABLE
ALTERED
Garasic 2010
Dutch Renal Artery Stenosis Intervention Cooperative Study
Study Design:
106 hypertensive patients with RAS (>50%) and Creat<2.3 mg/dl
PTA vs. Medical rx with follow-up of BP/meds/ renal fxn
at 3&12 mths
Results:
BP same in both groups
Fewer meds (2.1 vs. 3.2) in the PTA vs. Medical group
Renal function similar between groups
Shortcomings:
Crossover of patients from medical-to-PTA
No stents
Is 50% stenosis physiologically significant?
Pts with elevated creatinine excluded
Is the goal of renal artery revascularization improvement in BP control?
N Engl J Med 2000; 342:1007-14
Copyright Harvard Medical School, 2010. All Rights Reserved.
1013
Garasic 2010
ASTRAL Trial Schema
Diagnosis of significant ARVD
(Unilateral or Bilateral)
Revascularization not contraindicated
Uncertain whether to revascularize
Randomisation
No revascularization
Medical Treatment only
Revascularization
with angioplasty and/or
stent
(and medical treatment)
N Engl J Med 361;20, 19.
Garasic 2010
Eligibility
Any patient with atherosclerotic renovascular
disease (ARVD) and at least one ARVD lesion
that is suitable for balloon angioplasty and/or
stenting confirmed by angiogram.
No prior revascularisation procedure for ARVD.
No definite contraindication to revascularisation
and unlikely that revascularisation will become
indicated within the next 6 months.
Copyright Harvard Medical School, 2010. All Rights Reserved.
1014
Garasic 2010
PATIENT CHARACTERISTICS
GFR
Mean =40 ml/min (Range: 5.4 124.5)
0
50
100
150
200
250
300
<=20 20-30 30-40 40-50 >50
GFR (ml /mi n)
N
o
.

o
f

p
a
t
i
e
n
t
s
Garasic 2010
ANGIOGRAPHIC DATA BY
RANDOMISED TREATMENT
Revasc. Medical P-value
% Stenosis 76% (40 100%) 75% (20 100%) 0.3
Renal length 9.7cm (6 14) 9.7cm (6 20) 0.5
Location of ostial/distal ARVD lesion
Left kidney 24% 20% 0.2
Right kidney 18% 17%
Both 50% 57%
Missing data 8% 6%
Copyright Harvard Medical School, 2010. All Rights Reserved.
1015
Garasic 2010
PATIENT CHARACTERISTICS
Percent Stenosis
Mean =76% (Range: 20% 100%)
0
50
100
150
200
250
300
<50 50-59 60-69 70-79 80-89 >=90
Stenosis (%)
N
o
.

o
f

p
a
t
i
e
n
t
s
* 40% of Revascularization patients had no stenosis >70%.
Garasic 2010
COMPLIANCE WITH
RANDOMISED TREATMENT
N Revasc.
Successful
Attempted
but Failed
Not
Attempted
Revasc. 403 308 (82%)* 17 44
Medical 403 18 (4.4%) 1 1
Complications in Revasc Arm:
5 Renal Embolizations / 4 Renal Artery Occlusions / 4 Renal
Artery Perfs
Copyright Harvard Medical School, 2010. All Rights Reserved.
1016
Garasic 2010
REVASCULARIZATION PROCEDURE
Revasc. Medical
Intervention Performed N=308 N=13
Angioplasty plus stent 201 (65%) 8 (62%)
Stent only 86 (28%) 5 (38%)
Balloon angioplasty 21 (7%) 0 (-)
Unilateral 259 (84%) 11 (85%)
Bilateral 49 (16%) 2 (15%)
Garasic 2010
PLOT OF SCr OVER TIME
Copyright Harvard Medical School, 2010. All Rights Reserved.
1017
Garasic 2010
PLOT OF SYSTOLIC BP OVER TIME
Garasic 2010
PLOT OF DIASTOLIC BP OVER TIME
Copyright Harvard Medical School, 2010. All Rights Reserved.
1018
Garasic 2010
TIME TO FIRST RENAL EVENT
(ARF, Dialysis, Transplant, Nephrectomy, Renal Death)
HR=0.98, 95% CI=0.66 to 1.48
Garasic 2010
TIME TO FIRST OF MI, STROKE, VASCULAR DEATH
OR HOSPITALISATION FOR ANGINA, FLUID
OVERLOAD OR CARDIAC FAILURE
HR=0.90, 95% CI=0.66 to 1.15
Copyright Harvard Medical School, 2010. All Rights Reserved.
1019
Garasic 2010
All that said:
Garasic 2010
Copyright Harvard Medical School, 2010. All Rights Reserved.
1020
Garasic 2010
Study Design: Prospective, multi-center, two-arm randomized trial
Purpose: Medical therapy vs. Stenting for Systolic Htn + RAS
Enrollment: 1080 Patients
Patient Population: Systolic htn / >2 Htn Meds / >60% RAS with gradient or
>80% RAS with no gradient necessary
Endpoints:
Primary Cardiovascular/ renal death, Stroke, MI, hospitalization for CHF,
progressive renal failure, need for permanent renal replacement
Seondary Systolic BP response, Stent patency, Resistive index, all cause
mortality, subgroup analysis (DM vs non-DM), Longitudinal renal fxn
National PI: Christopher J. Cooper, M.D. Medical College of Ohio
U/S Core Lab: Michael Jaff, D.O. MGH
CORAL
Garasic 2010
Resistive
Index
Predicts
Fate of
Renal
Function
Radermacher et al. NEJM 2001 344: 410-17
PSV - MEDV
PSV
( )
=
Resistive
Index
Copyright Harvard Medical School, 2010. All Rights Reserved.
1021
Garasic 2010
Progress in Renovascular Disease
1) The Disease
2) Clinical diagnosis and medical management
3) Laboratory diagnosis / imaging modalities
4) Patient selection: who benefits from intervention?
5) Limiting contrast-induced nephropathy
6) Atheroembolic protection
7) Expanding the pool of eligible patients / interventions
8) Renal artery restenosis
Garasic 2010
Options:
Contrast minimizing maneuvers
Use of low-osmolar, non-iodinated
contrast
CO2 Angiography
Gadolinium contrast
Mucomyst (Acetylcysteine)
Selective DA-1 agonists
HOCM may increase
cellular injury when
potentiated by hypoxemia
c/t LOCM
Useful for localization
> Anatomic definition
-Gadopentatate dimeglumine
-Renally cleared by GF
-Now has its own problems
-As a result, rarely used
AC and hydration
reduce Creatinine >
hydration alone in CT
with CRI (2% vs 21%)
May decrease incidence
of RCN vs. historical
controls (4.7% vs 18.8%)
Copyright Harvard Medical School, 2010. All Rights Reserved.
1022
Garasic 2010
Progress in Renovascular Disease
1) The Disease
2) Clinical diagnosis and medical management
3) Laboratory diagnosis / imaging modalities
4) Patient selection: who benefits from intervention?
5) Limiting contrast-induced nephropathy
6) Atheroembolic protection
7) Expanding the pool of eligible patients / interventions
8) Renal artery restenosis
Garasic 2010
Be Afraid.
Be Very Afraid!
Copyright Harvard Medical School, 2010. All Rights Reserved.
1023
Garasic 2010
Distal Atheroembolic Protection: The Ideal
Capture all debris
Continued renal perfusion during procedure
Limitless reservoir
Atraumatic to vessel wall
Technically easy to use
Low profile
Trackable
Garasic 2010
Pathology
of
Atheroembolism
Plaque / cholesterol
Endothelial cells
Platelet-Fibrin Thrombi
Calcified tissue
Copyright Harvard Medical School, 2010. All Rights Reserved.
1024
Garasic 2010
Evolution of Distal Protection Devices
Initially used in the treatment of patients with
coronary bypass graft disease
These interventions commonly plagued by
angiographic No Reflow phenomenon
Cardiac enzyme leak
Clinical myocardial infarction
Long attributed to RBC lysis and platelet
activation with resultant microvascular spasm
Garasic 2010
Initial Reports: Atheroembolic Protection
Percusurge Guardwire
47% of patients with SVG intervention had
gross, macroscopic evidence of red-yellow
debris
An additional 20% of patients had evidence of
microscopic debris
Carlino et al.Circulation 1999; 99: 3221-3223
Copyright Harvard Medical School, 2010. All Rights Reserved.
1025
Garasic 2010
Randomized comparison of SVG lesions
treated +/- PercuSurge Guardwire
Improved outcomes: 42% decrease MACE
Lower laboratory MIs
Safe
High procedural success
SAFER Trial
Baim et al. Circulation 2002.
Garasic 2010
Distal Protection in Renovascular
Disease: An Opportunity
Most RAS caused by atheromatous disease
Ostial / proximal segments of disease are
common
Kidney will tolerate longer balloon occlusion
time than coronary / cerebral circulation
Atheroembolism has long been viewed as a
major risk / complication of percutaneous
intervention of the renal arteries.
Copyright Harvard Medical School, 2010. All Rights Reserved.
1026
Garasic 2010
FILTER
DEVICES
OCCLUSION
DEVICES
Preserve flow
Limit Ischemic Time
More complete capture
Small debris
Vascular Injury
No antegrade flow
Prolonged Ischemic Time
Vascular Injury
Shoulder Regions
+
-
Garasic 2010
FILTER
DEVICES
Cordis
Copyright Harvard Medical School, 2010. All Rights Reserved.
1027
Garasic 2010
Atheroembolization Protection
Percusurge Guardwire
A. Traverse
B. Inflate
C. Intervene
D. Embolectomize
Garasic 2010
Early Experience: Distal Protection
in Renovascular Intervention
28 patients with 32 renal arteries
29 Lesions ostial location
100% Technical success with GuardWire
Visible debris aspirated: 100% cases
Mean RA occlusion time: 6.55 min (2.29-13.21 min)
Creatinine post-procedure and at follow-up stable or
improved in all cases.
Conclusion: Distal protection against atheroembolism is feasible and safe
But is it effective?
Henry et al.J Endovasc Ther 2001; 8(3): 227-37.
Copyright Harvard Medical School, 2010. All Rights Reserved.
1028
Garasic 2010
Non Randomized Data:
Distal Protection
in Renovascular Intervention
121 hypertensive patients / 148 renal arteries
Both occlusion balloons (n=46) and filters (n=95) used
Visible debris aspirated: 100% cases with Guardwire
Visible debris aspiration: 80% cases with filters
5X volume of particulate matter retrieved with FiberNet
6 mths: 99% of patients had stable or improved renal fxn
2 yrs: 95% of patients had stable or improved renal fxn
Conclusion: Distal protection against atheroembolism is feasible and safe
But is it effective?
Henry et al.J Cardiovasc Surg 2008; 8(3): 227-37.
Garasic 2010
Renal Artery Stent Revascularization with Embolic
Protection in Patients with Ischemic Nephropathy
RESULTS at 6 months
K-DOQI 3A K-DOQI 3B K-DOQI 4
Total
Improved
12(52%) 8(32%) 5(33%) 25(40%)
Stabilized
11(48%) 15(60%) 10(67%) 36(57%)
Unchanged
decline
0(0%) 2(8%) 0(0%) 2(3%)
Total 23 25 15 63
Level of pre-intervention CRI
97% of patients had renal function improved or stabilized at 6 months
94% of patient had renal function improved or stabilized at 16 months
Holden A, Hill A, Jaff M R, Pilmore H. Kid Int 2006;70:830-832.
Copyright Harvard Medical School, 2010. All Rights Reserved.
1029
Garasic 2010
Progress in Renovascular Disease
1) The Disease
2) Clinical diagnosis and medical management
3) Laboratory diagnosis / imaging modalities
4) Patient selection: who benefits from intervention?
5) Limiting contrast-induced nephropathy
6) Atheroembolic protection
7) Expanding the pool of eligible patients /
interventions
8) Renal artery restenosis
Garasic 2010
What are the benefits of PCI over
Surgical revascularization?
Shortened hospital stays
Reduced post-procedural morbidity / mortality
(J Vasc Surg 1994; 20: 76-87)
Early graft failure 5%
Peri-operative mortality 5.6%
43% of patients required aortic grafting
Comparable procedural success and improvement in renal function
(J Vasc Surg 1993; 18:841-52)
Procedural success: PTRA 83% vs. Surgery 97% (p=NS)
Improved or stable renal function: PTRA 83% vs. Surgery 72%
(p=NS)
Broadens pool of patients eligible for revascularization
Copyright Harvard Medical School, 2010. All Rights Reserved.
1030
Garasic 2010
Progress in Renovascular Disease
1) The Disease
2) Clinical diagnosis and medical management
3) Laboratory diagnosis / imaging modalities
4) Patient selection: who benefits from intervention?
5) Limiting contrast-induced nephropathy
6) Atheroembolic protection
7) Expanding the pool of eligible patients / interventions
8) Renal artery restenosis
Garasic 2010
P
a
t
h
o
b
i
o
l
o
g
y
o
f

I
n
-
s
t
e
n
t

R
e
s
t
e
n
o
s
i
s
Deep Arterial Injury: What Vascular Biology Tells Us
Copyright Harvard Medical School, 2010. All Rights Reserved.
1031
Garasic 2010
RA Stenting: Restenosis
van de Ven, 1999 52 50 ( 95%) 100 Pal maz angi o* 6 21%
Rocha-Si ngh, 1999 180 158 ( 88%) 43 Pal maz dupl ex + angi o 13 12%
Tuttl e, 1998 148 49 ( 33%) 100 Pal maz angi o 8 14%
Rundback, 1998 54 28 ( 52%) NA Pal maz angi o* + spi ral CT 12 26%
Whi te, 1997 133 80 ( 60%) 81 Pal maz angi o* 9 19%
Harden, 1997 32 24 ( 75%) 75 Pal maz angi o* 6 12%
Bl um, 1997 74 74 ( 100%) 100 Pal maz angi o* 24 11%
Henry, 1996 64 54 ( 84%) 53 Pal maz angi o* 14 9%
Iannone,1996 83 69 ( 85%) 78 Pal maz dupl ex 11 14%
Dorros, 1995 [ 30 ] 92 56 ( 61%) 100 Pal maz angi o* 7 25%
Hennequi n, 1994 21 20 ( 95%) 33 Wal l stent angi o* 29 20%
Rees, 1994 296 150 ( 51%) 100 Pal maz angi o* 7 33%
10 20%
Arteri es
eval uated
(%ori gi nal total
arteri es)
Stent type
angi o* = protocol -speci fi ed angi ographi c fol l ow-up
Study seri es
# of
Arteri es
wei ghted average
Osti al l esi on
(%)
Restenosi s (of
artery eval uated)
Average ti me to
eval uati on
(month)
Method of
eval uati on
~20%
Lim and Rosenfield, Curr Int Cardiol 2000,2:130-139.
Garasic 2010
Restenosis: Is it a problem in
the Renal Arteries?
Zeller et al. Catheter Cardiovasc Interv 2003; 60: 1-6.
0
2
4
6
8
10
12
14
16
18
3-4 5 6 7-9
Stent Size (mm)
R
e
s
t
e
n
o
s
i
s
R
a
t
e

(
%
)
Copyright Harvard Medical School, 2010. All Rights Reserved.
1032
Garasic 2010
Curing ISR: The Molecular Approach
Garasic 2010
GREAT: 6-Month QA Data
Bare
Mean + SD
N=41 (79%)
SES
Mean + SD
N=45 (85%)
P-value
Reference vessel
diameter
5.58 0.81 mm 5.52 0.73 0.74
Diameter stenosis
(%DS)
23.9 22.89
(-2.3 - 96.8)
18.7 15.58
(0.7 66.6)
0.39
Minimum lumen
diameter (MLD)
4.33 1.44 mm
(0.2 - 6.8)
4.45 0.88 mm
(1.6 6.1)
0.88
In-stent restenosis
(> 50%)
6 (14.3%) 3 (6.7%) 0.30
Late loss 0.92 0.62 0.21
Zahringer et al. J Endovascular Therapy 2007;Vol. 14 (4): 460468.
1 Year TLR: BMS 11.5% / SES 1.9% (P=0.21)
Copyright Harvard Medical School, 2010. All Rights Reserved.
1033
Garasic 2010
Management of Renal Artery ISR:
DES
Drug eluting (Sirolimus) coronary stents for renal re-
stenosis
Renal Duplex: Mean PSV=445 +/- 131 cm/sec
RAR 5.0 +/ - 1.6
Clinical evidence of restenosis
N=22 renal arteries
75% female
71.4% incidence of recurrent restenosis at median f/u
510 d
Female gender an independent predictor of recurrent
ISR
Kiernan T et al. Vasc Med 2009; 15(1): 3-7.
Garasic 2010
The Utilization of PTFE-Covered Stents For The
Treatment of Renal Artery In-Stent Restenosis
Ten men and 16 women (mean age of 70 +/- 11 years).
Twelve (46%) of the patients had DM, 100% had HTN, 100% had
hypercholesterolemia, 16 (62%) were current or former smokers and all
had peripheral arterial disease.
Mean pre-procedure creatinine was 1.58 +/- 0.72 mg/dl (5 patients had a
solitary functioning kidney).
The average RA-ISR by angiography prior to intervention was 84 +/- 1.8 %.
There was 100% procedural success without any recorded procedural
complications.
Median stent diameter was 6 mm.
Median stent length was 16 mm.
Post-dilation was performed in all patients, median balloon diameter of 7 mm.
Mean follow-up of 10.2 +/- 4.7 months, 0% of patients who received a
PTFE-covered stent for RA-ISR had a severe enough renarrowing to
require repeat intervention.
Nicholas J . Ruggiero, II, J oseph Garasic, Michael R. J aff, Andrew B.
McCann, Thomas J . Kiernan, Brian G. Hynes, Douglas E. Drachman,
Robert Schainfeld, Kenneth Rosenfield, Gary M. Ansel. ACC 2010.
Pre
Post
Copyright Harvard Medical School, 2010. All Rights Reserved.
1034
Garasic 2010
In Summary
Renovascular disease is an often-unrecognized contributor to:
Uncontrolled hypertension
Volume overload / CHF
Chronic and progressive renal failure
Medical management of RAS involves avoidance of ACEI and use of
agents commonly used in the management of CAD
Existing literature allows data-driven decision making, helping clinicians
to properly manage their patients with renovascular disease. However,
the optimal treatment of patients with unilateral disease or clinically
silent disease is ill defined.
New technologies have expanded the pool of patients eligible for
percutaneous intervention, and help to limit procedural risk with renal
revascularization.
Atheroembolic distal protection devices are likely to be a mainstay of
therapy in the near future.
Vascular medicine allows cooperation and collaboration across
departmental boundaries.
Garasic 2010
Failure of renal function to improve after renal
intervention
may be predicted by:
A) Significant proteinuria
B) Recent onset or worsening of hypertension
C) Preserved renal span
D) Severe ostial stenosis
E) Creatinine clearance >40 ml/min
Copyright Harvard Medical School, 2010. All Rights Reserved.
1035
Garasic 2010
Which of the following statements regarding
Renovascular disease is false:
A) The rate of renal artery restenosis is ~20%
B) Renal intervention is curative of hypertension
in most cases of bilateral severe RAS
C) A low resistive index is associated with improved
outcomes
after renal intervention.
D) The majority of revascularized patients in the Astral
trial
had unilateral RAS
E) Fibromuscular dysplasia affecting the renal arteries
most typically does not result in a loss of renal span
Garasic 2010
Which is true of MR Angiography in evaluating
Renovascular disese:
A) MRA is the modality of choice in the HD patient
B) False negatives are common, but false positives are
rare
C) Renal duplex provides greater data for plotting
intervention
D) MRA is likely preferable to u/s in the obese patient
E) Pathology involving the mid renal artery such as
FMD is
diffficult to image by MRA
Copyright Harvard Medical School, 2010. All Rights Reserved.
1036
Garasic 2010
Presenter Disclosure
Information
The following relationships exist related to
this presentation:
Consultant / Equity Interest Access Closure,
Inc.
Speakers Bureau Sanofi-BMS Partnership
Copyright Harvard Medical School, 2010. All Rights Reserved.
1037
Acute Kidney Injury:
EPIDEMIOLOGY
Sushrut S. Waikar, MD, MPH
Renal Division
Brigham and Womens Hospital
Disclosures
Investigator-initiated grant support from Pfizer,
Genzyme, Satellite HealthCare, NxStage;
DSMB member for Takeda
Copyright Harvard Medical School, 2010. All Rights Reserved.
1038
Board exam breakdown: ARF
19 to 21 questions total (10% of exam)
Differential diagnosis 3 5
Management 7 9
Prevention 0 2
Complications/Outcomes 4 6
ICU 1 3
Miscellaneous 0 1
From: http://www.abim.org/shared/pdf_blueprint/neph_cert.pdf
Overview
General epidemiologic characteristics of
acute kidney injury (AKI)
Definitions, incidence, mortality, prognostic
significance
Specific disease associations
Copyright Harvard Medical School, 2010. All Rights Reserved.
1039
Evolving terminology
ACUTE RENAL FAILURE
ACUTE KIDNEY INJURY
Experts from nephrology and
intensive care: Acute Kidney
Injury Network
Consensus that term ARF
needed updating in light of:
Failure denotes severe
dysfunction, fails to
capture spectrum of
condition
Injury short of failure (i.e.,
small increase in SCr) is
clinically important
Definitions of AKI
Over 35 distinct definitions in the literature
Early definitions emphasized severe kidney
injury: need for RRT, doubling of SCr
Newer definitions focus on smaller changes
in SCr (increase of >0.3 mg/dL or 50%),
incorporate urine output
Have not yet been validated: unlikely to be
asked about new definitions
Copyright Harvard Medical School, 2010. All Rights Reserved.
1040
New proposed stages of AKI
AKIN proposed criteria
Stage 1
SCr increase 0.3 or 50%
UO: <0.5ml/kg/h for >6h
Stage 2
SCr doubling
UO: <0.5ml/kg/h for >12h
Stage 3
SCr tripling, acute rise 0.5
UO: <0.3ml/kg/h for >24h
or anuria x 12h
Reviewed by Himmelfarb Kidney Int 2007 Molitoris JASN 2007
RIFLE criteria
RISK
SCr increase by 50%
UO: <0.5ml/kg/h for >6h
INJURY
SCr doubling
UO: <0.5ml/kg/h for >12h
FAILURE
SCr tripling, acute rise 0.5
UO: <0.3ml/kg/h for >24h or
anuria x 12h
New proposed stages of AKI
Waikar & Bonventre
Stage 1
SCr increase of 0.3 in 24h OR 0.5 in 48h
Stage 2
SCr increase of 0.5 in 24h OR 1.0 in 48h
Stage 3
SCr increase of 1.0 in 24h OR 1.5 in 48h
Waikar & Bonventre JASN 2009
Copyright Harvard Medical School, 2010. All Rights Reserved.
1041
Question
The risk of acute kidney injury (>0.5 mg/dL
increase in SCr) in hospitalized patients is
approximately:
A) 0.5%
B) 1%
C) 3%
D) 13%
E) 25%
Incidence of AKI
Estimates depend entirely on definition, population
being studied
5 to 7%of hospital admissions (0.5, 1.0, or 1.5 mg/dL
increase in SCr depending on baseline)
13%of hospital admissions (0.5 mg/dL increase)
1%of admissions from community-acquired AKI
Following CABG:
15%of patients have a >25% increase in SCr
1-2%require renal replacement therapy
Sepsis:
Doubling of SCr in 9%with SIRS, 51%with shock
Copyright Harvard Medical School, 2010. All Rights Reserved.
1042
Rising incidence of AKI
Between 1988 and
2002: Four-fold
increase in AKI, six-
fold increase in AKI-D
(Waikar JASN 2006)
Community-based
estimate
1
: 522 AKI,
30 AKI-D per 100,000
A
K
I

i
n
c
i
d
e
n
c
e

(
/
1
0
0
,
0
0
0

p
y
)
1
Hsu Kidney Int 2007
322.7
388.3
453.6
522.4
0
100
200
300
400
500
600
1996-1997 1998-1999 2000-2001 2002-2003
Risk factors for AKI
Best studied in cardiac catheterization and
CABG
Cardiac catheterization:
Age, higher SCr, CHF, diabetes
contrast volume, intra-aortic balloon pump
CABG:
Age, higher SCr, CHF, diabetes, concomitant
valve surgery
urgency of operation, bypass time
Copyright Harvard Medical School, 2010. All Rights Reserved.
1043
Clinical features of AKI
BEST Kidney
Septic shock 48%
Major surgery 34%
Cardiogenic shock
27%
Hypovolemia 26%
Drug-induced 19%
Hepatorenal 6%
Obstructive uropathy
3%
Other 12%
PICARD
ATN (unspecified) 50%
Drug-induced 26%
Sepsis 19%
Cardiac disease 20%
Hypotension 20%
Pre-renal 20%
Liver disease 11%
Uchino JAMA 2005 Mehta Kidney Int 2004
Predictors of dialysis in AKI
PICARD
Age
Oliguria
High BUN
Liver failure
Copyright Harvard Medical School, 2010. All Rights Reserved.
1044
Small changes in SCr
1
Chertow JASN 2005
2
Brown Circulation 2006
Recent reports suggest that even small
increases in SCr increase risk of death
Among ~10,000 inpatients
1
: increase in SCr
of 0.3 or 0.4 mg/dL 70% higher adjusted
odds of death in hospital
Following CABG
2
: 50% to 99% increase in
SCr 6.6-fold higher adjusted risk of death
at 90d
Predictors of death in AKI
Results from multicenter observational studies
BEST Kidney Invest.
(Uchino J AMA 2005)
Overall mortality 60%
Age
Duration between
admission and AKI
Mechanical ventilation
Vasopressor use
Sepsis
Cardiogenic shock
Hepatorenal syndrome
PICARD
(ChertowKidney Int 2006)
Overall mortality 37%,
over 60% for AKI-RRT
Age
Higher BUN
Liver failure
Sepsis
CKD stage IV:
protective
Copyright Harvard Medical School, 2010. All Rights Reserved.
1045
Long-term outcomes of AKI
The kidneys possess a remarkable capacity to
recover after severe injury
Animal studies show permanent damage, however:
tubulointerstitial fibrosis, damage to microvasculature
Mortality
33% increased risk of death at 10y in survivors of AKI
following acute MI compared to non-AKI controls
1
Renal function decline
30% higher risk of ESRD with AKI on CKD
2
1
Parikh et al., Arch Int Med 2008
2
Hsu et al.,cJASN 2009
Costs associated with AKI
Markedly increased
60% increase in costs with AKI following
cardiac surgery (RIFLE Ror 50% increase)
1
>0.3 mg/dL: Additional $5000 in general
hospitalized population
2
Annual costs in United States estimated at
$10 billion
2
1
Dasta et al., NDT 2008
2
Chertow et al.,JASN 2005
Copyright Harvard Medical School, 2010. All Rights Reserved.
1046
AKI in the developing world
Certain causes of AKI are region-specific
Malaria from P. falciparum: AKI in ~5% of
patients in endemic areas, up to 30% of
nonimmune travellers
Leptospirosis: widespread zoonosis, variable
clinical manifestations, AKI in up to 18%
Herbal toxins (e.g., Chinese herbal
nephropathy from Aristolochic acid)
Schistosomiasis: obstructive uropathy
Specific clinical scenarios
Copyright Harvard Medical School, 2010. All Rights Reserved.
1047
Community-acquired AKI
1% of hospital admissions
Differs greatly from hospital-acquired:
70% pre-renal azotemia
17% obstruction
11% intrinsic (mostly drug-induced, only 1 with
ATN)
Kaufman AJKD 1991
AKI following cardiac surgery
1 2% AKI requiring RRT
15% less severe AKI (25% increase SCr)
Major causes
Pre-renal azotemia (hypovolemia,
overdiuresis, cardiac failure)
Intrinsic (ATN, atheroemboli, contrast, AIN
from perioperative antibiotics)
Copyright Harvard Medical School, 2010. All Rights Reserved.
1048
AKI in pregnancy
Marked reduction in incidence (industrialized world)
Causes
Early pregnancy: pre-renal azotemia from hyperemesis
gradivarum, ATN from septic abortion
Severe pre-eclampsia: usually preserved GFR unless
severe bleeding, DIC, etc
TTP-HUS
Mid-pregnancy, peripartum, or postpartum
Cortical necrosis: anuria, gross hematuria, flank pain
Abruption, placenta previa, fetal death, amniotic fluid
embolus
Acute fatty liver of pregnancy
Contrast nephropathy
Incidence
Negligible following radiologic procedures in patients
with normal SCr
50% in pts with advanced CKD
Following PCI
~3% overall (defined as 0.5 mg/dL increase)
25% in patients with SCr >2.0 mg/dL
Clear association with in-hospital and 5-yr mortality
Risk factors: age, higher SCr, CHF, diabetes,
contrast volume, intra-aortic balloon pump
Copyright Harvard Medical School, 2010. All Rights Reserved.
1049
AKI in bone marrow transplant
Incidence (defined as 50% increase SCr)
Myeloablative allo >non-myeloablative allo >
myeloablative auto (75% vs 40% vs 20%)
Common causes in first 3 weeks
ATN from sepsis
Hepatic veno-occlusive disease
Nephrotoxins
Other: tumor lysis, thrombotic
microangiopathy, CNI toxicity
Reviewed by Parikh Kidney Int 2006
AKI in cancer
Tumor lysis syndrome
Most commonly in poorly differentiated lymphoma
(e.g., Burkitts)
ALL >AML
Also described in other solid tumors
Multiple myeloma
Cast nephropathy
Hypercalcemia
Hyperuricemia
Contrast nephropathy
Obstruction
Prostate, bladder, uterus, cervix
Absence of hydronephrosis in retroperitoneal tumors
and retroperitoneal fibrosis
Reviewed by Humphreys JASN 2005
Copyright Harvard Medical School, 2010. All Rights Reserved.
1050
AKI in nephroticsyndrome
Minimal change disease
Risk factors: older age, higher BP, more
proteinuria
Collapsing glomerulopathy
Membranous nephropathy
Superimposed crescentic GN
Drug reaction
Vasoconstriction from NSAIDs
Allergic interstitial nephritis (NSAIDs, others)
AKI and NSAIDs
Hemodynamically mediated
Inhibition of prostaglandin synthesis afferent
arteriole vasoconstriction GFR decline
Risk factors: CHF, cirrhosis, volume depletion,
underlying CKD, diuretic therapy, age
COX-2 inhibitors have similar toxicity
Acute tubulointerstitial disease
Fenoprofen, but all NSAIDs
Absence of fever, rash, eosinophilia is common
Nephrotic syndrome
Mimimal change disease >>membranous nephropathy
Copyright Harvard Medical School, 2010. All Rights Reserved.
1051
AKI from other nephrotoxins
Aminoglycoside antibiotics
Nearly 50% of patients treated >14d; dose related
Proximal tubular injury
Typically nonoliguric AKI, slow onset
Recovery can take weeks, may be incomplete
Cisplatin
Cumulative dose-related
Tubulointerstitial pattern without heavy proteinuria
25% decline in GFR in 20-30% of patients
Incidence and severity increase with subsequent doses, may be
irreversible
Nonoliguric; hypomagnesemia common
Antivirals
Acyclovir: intratubular precipitation (U/A: needle shaped crystals)
Foscarnet: ATN
Tenofovir: ATN
Abdominal compartment syndrome
Increased intra-abdominal pressure can lead to
dysfunction of several organs
Cardiac, pulmonary, renal, GI, CNS
Abdominal pressure >25
Most common settings
Volume resuscitation for trauma
Post-laparotomy
Pancreatitis
Peritonitis
Increased renal venous pressure
Suspect in: tense abdomen, oliguria, azotemia
Copyright Harvard Medical School, 2010. All Rights Reserved.
1052
Question 1
26 year old G1P0 woman presents at 33 weeks gestation with lower
extremity edema and severe right upper quadrant pain. Physical
examination reveals BP 188/94, mild RUQ tenderness, and 3+lower
extremity edema. Labs notable for Hgb 8.6, platelet count 92,000/mm
3
,
INR 1.8, PTT 41, LDH 700, ALT 400, serum creatinine 1.4 mg/dL, 3+
albuminuria, and moderate schistocytes on peripheral blood smear.
Which of the following two are the MOST appropriate interventions:
A) Immediate delivery
B) Intravenous heparin
C) Plasma exchange
D) Activated protein C
E) Intravenous methylprednisolone
Question 2
A 76 year old man
developed a rise in serum
creatinine following cardiac
catheterization and
percutaneous coronary
intervention. His serum
creatinine trajectory was as
follows:
1
1.2
1.4
1.6
1.8
2
2.2
2.4
pre-
cath
24h 48h 72h 96h 120h 144h
S
C
r

(
m
g
/
d
L
)
Which of the following is LEAST LIKELY:
a) False-positive proteinuria at 12h
b) Hypocomplementemia
c) FeNa 0.3% at 24 hours
d) Normal urine output
Copyright Harvard Medical School, 2010. All Rights Reserved.
1053
Disclosures
Investigator-initiated grant support from Pfizer,
Genzyme, Satellite HealthCare, NxStage;
DSMB member for Takeda
Copyright Harvard Medical School, 2010. All Rights Reserved.
1054
Acute Kidney Injury
(aka: Acute Renal Failure)
It is more than just a change in name
J oseph V. BonventreMD PhD
Director, Renal Division
Brigham and Womens Hospital
Harvard Medical School
Disclosure
Dr. Bonventre is co-inventor on KIM-1 patents , licensed to
Johnson and Johnson, Genzyme and BiogenIdec
Copyright Harvard Medical School, 2010. All Rights Reserved.
1055
Questions
1. What is the definition of acute renal failure/acute kidney
injury (AKI) ? What are the RIFLE criteria ?
2. How large do changes in creatininehave to be before
worrying about the effects of renal dysfunction on
mortality?
3. Is the incidence of AKI increasing or decreasing ?
4. Is mortality associated with AKI increasing or decreasing?
5. Does AKI contribute to Chronic Renal Failure?
6. Is dopamine appropriate for the treatment of AKI ?
Questions (2)
7. Does CRRT result in better outcome than IHD in AKI ?
8. Does increasing the dose of dialysis in AKI alter the
outcome?
9. Is serum creatininea good early marker for AKI?
10. What are some other potential biomarkers for early
diagnosis of AKI and prediction of outcome of AKI?
Copyright Harvard Medical School, 2010. All Rights Reserved.
1056
> Diagnosis and New terminology (AKI, RIFLE
Criteria)
> Changing epidemiology
> Pathophysiologyand Implications for
Pharmacologic and DialyticTherapies
Vasoconstriction
Inflammation
> Repair (?Stem cells)
> Biomarkers
Acute Kidney Injury
RIFLE criteria for diagnosis of acute kidney injury
Risk of renal injury
Injury to the kidney
Failure of kidney function
>0.3mg/dL increase
2.0 X baseline
3.0 X baseline
or
Serum creatinine>
4 mg/dL with an
absolute increase of
>0.5 mg/dL
<0.5 mL/kg/hr for >6 h
<0.5 mL/kg/hr for >12 h
<0.3 mL/kg/hr for >24 h
or
Anuriafor >12 h
Loss of kidney function
End-stage disease
Persistent renal failure for >4 weeks
Persistent renal failure for >3 months
Increase in serum
creatinine Urine output
Copyright Harvard Medical School, 2010. All Rights Reserved.
1057
Incidence of AKI depends on definition
1.0% if >2.0 mg/dL increase in SCr
12% if >0.5 mg/dl increase in SCr
Chertow
Burdick
Honour
Bonventre
Bates
J ASN 16:
3365-70, 05
Brigham and Womens: 9210 adults with
2 or more serum creatininedeterminations
during their hospitalization (of 19,982 admissions)
AKI and Mortality (Brigham and Womens, 9210 adults)
AKI 6.5 (5.0 to 8.5) <0.0001
Age(per 10 y) 1.7 (1.1 to 2.6) <0.0001
CKD 2.5 (1.9 to 3.3) <0.0001
DRG weight 1.2 (1.1 to 1.3) <0.0001
CV 1.5 (1.0 to 2.2) <0.04
Respiratory 3.0 (1.9 to 4.8) <0.0001
GI 2.4 (1.4 to 4.1) <0.001
Cancer 2.9 (1.9 to 4.4) <0.0001
Infection 7.5 (4.2 to 13.6) <0.0001
Chertow
Burdick
Honour
Bonventre
Bates
J ASN 16:
3365-70, 05
Multivariable Odds Ratio for Death
AKI: creatinine>+0.5 mg/dl
Copyright Harvard Medical School, 2010. All Rights Reserved.
1058
>0.3 mg/dl 4.1 (3.1 to 5.5)
>0.5 mg/dl 6.5 (5.0 to 8.5)
>1.0 mg/dl 9.7 (7.1 to 13.2)
>2.0 mg/dl 16.4 (10.3 to 26.0)
Chertow
Burdick
Honour
Bonventre
Bates
J ASN 16:
3365-70, 05
Multivariable Odds Ratio for Death
serum creatinine
AKI and Mortality (Brigham and Womens, 9210 adults)
An abrupt (within 48 hours) reduction in kidney
function currently defined as an absolute
increase in serum creatinine of
either > 0.3 mg/dl or a percentage increase
of 50% or a reduction in urine output
(documented oliguria of < 0.5 ml/kg/hr for more
than 6 hours)*
Diagnostic Criteria for Acute Kidney Injury
(AKI)(current consensus - AKI Network)
Copyright Harvard Medical School, 2010. All Rights Reserved.
1059
Waikar SS and Bonventre JV. J Am Soc Nephrol 2009;20:672-679
SCr concentrations after an abrupt 50% reduction in CrCl,
superimposed on four different levels of baseline kidney
function (no CKD and stages 2 through 4 CKD)
Solid squares show the point at which a 100% increase in SCr has occurred.
Open triangles show the point at which a 1.0 mg/dl increase in SCr has occurred.
> New terminology (AKI, Rifle Criteria)
> Changing epidemiology
> Pathophysiologyand Implications for
Pharmacologic and DialyticTherapies
Vasoconstriction
Inflammation
> Repair (?Stem cells)
> Biomarkers
Acute Kidney Injury
Copyright Harvard Medical School, 2010. All Rights Reserved.
1060
Number of cases of AKI per 1000 hospital discharges of US Medicare
beneficiaries (5,403,015 discharges)
Men
Women
White
By sex By race
By age Overall
Other
Black
85+ yr
75-84 yr
75-84 yr
< 64yr
50
45
40
35
30
25
20
15
10
5
50
45
40
35
30
25
20
15
10
5
40
35
30
25
20
15
10
50
45
40
35
30
25
20
15
10
Year
60
65
55
9293 95 94 969798990001 9293 95 94 969798990001
Cases per
1000 discharges
Cases per
1000 discharges
Year
>84 yr
65-74 yr
Xueet al. J ASN
17:1135-42,
2006
Declining mortality in patients with AKI
Waikar,Curhan,Wald,McCarthyand Chertow, J ASN 17:1143-1150, 2006
Copyright Harvard Medical School, 2010. All Rights Reserved.
1061
Ishani, A. et al. J Am Soc Nephrol 2009;20:223-228
Estimated probability of initiating treatment of ESRD
Post-ischemic expression of -smooth muscle actin
Ischemia
1 week
Ischemia
12 weeks
Ischemia
6 weeks
Sham
1 week
Copyright Harvard Medical School, 2010. All Rights Reserved.
1062
> New terminology (AKI, Rifle Criteria)
> Changing epidemiology
> Pathophysiologyand Implications for
Pharmacologic and DialyticTherapies
Vasoconstriction
Inflammation
> Repair (?Stem cells)
> Biomarkers
Acute Kidney Injury
MICROVASCULAR TUBULAR
Vasoconstriction
renal nerves, adenosine
angiotensinII, thromboxaneA
2
endothelin, leukotrienes
Vasodilation
acetylcholine, bradykinin
nitric oxide, PGE
2
Endothelial and vascular smooth
muscle cellular damage
Leukocyte-Endothelial adhesion
vascular obstruction,
leukocyte activation and
inflammation
Cytoskeletal breakdown
Loss of polarity
Apoptosis and Necrosis
Desquamation of viable
and necrotic cells
Tubular obstruction
Backleak
Glomerular Medullary
O
2
Inflammatory
Vasoactive
Mediators
Mechanisms of Kidney Injury and Repair
Copyright Harvard Medical School, 2010. All Rights Reserved.
1063
If arteriolar vasoconstriction is the cause of ARF
then a vasodilator should be therapeutic
Normal Arteriole Post-Ischemic
Renal Perfusion Pressure
Renal Perfusion Pressure
vasoconstrictors
unstimulated
vasodilators
Dopamine
Natriureticpeptides
NO donors
Calcium channel blockers
Endothelinantagonists
Adenosine antagonists
ACE inhibitors
ROS scavengers
Prostaglandins agonists
and antagonists
Platelet activating factor
IGF-1 and basic FGF
Absence of Effect of Dopamine on Survival in Patients with Early
Renal Dysfunction
Bellamoet al. ANZICS Clinical Trial Group. Lancet 2000; 356: 2139-2143
Early Renal Dysfunctionis defined as either: <0.5 ml/kg/hr for 4 hr; or
an increase in serum creatinine>150 mol/l; or
serum creatinine>150 mol/l in absence of CRF
Copyright Harvard Medical School, 2010. All Rights Reserved.
1064
In-hospital mortality 1.68 (1.06-2.64)
Nonrecoveryof renal function 1.79 (1.19-2.68)
Death or nonrecovery 1.77 (1.14-2.76)
Covariant and Propensity
Score Adjusted Odds Ratio (95% CI)
Days from Consult to Dialysis/Death
100
75
50
25
00
0 10 20 30 40 50 60
% Survival
No Diuretics
Diuretic Responsive
Diuretic Nonresponsive
Diuretics, Mortality and Nonrecovery of Function in Acute
Kidney Injury
MehtaChertow,
PICARD
J AMA 288: 2547, 2002
> New terminology (AKI, Rifle Criteria)
> Changing epidemiology
> Pathophysiologyand Implications for
Pharmacologic and DialyticTherapies
Vasoconstriction
Inflammation
> Repair (?Stem cells)
> Biomarkers
Acute Kidney Injury
Copyright Harvard Medical School, 2010. All Rights Reserved.
1065
CD11b/
CD18
CD11c/
CD18
Ischemic Kidney
ICAMs
VCAM
selectins
Release of ROS,
proteases, elastases,
leukotrienes, PAF
Increased expression of adhesion
molecules on endothelial cells
Endothelial Cell
ICAMs
VCAM
selectins
iC3b
CD35
iC3b
Leukocyte Activation in Ischemic AKI
Local production of inflammatory mediators
- cytokines (TNF, IL-1), chemokines(IL-8,
MCP-1)
- complement activation products
- platelet activating factor (PAF)
- metabolites of arachidonicacid
- reactive oxygen species (ROS)
Activated
Leukocytes
procoagulant
effects
Plasma proinflammatory cytokine levels
Himmelfarbet al.
J ASN 15: 2449,2004
PICARD
4.7 1.7 2.9 0.6 4.6 0.7 10.6 1.0
2.5 0.4 290.2 70.3* 32.4 12.2* 23.5 3.0*
2.8 0.3 190.6 32.4* 44.3 13.1* 56.2 5.3*
+
Healthy
Critically ill
ARF
IL-1 (pg/ml) TNF- (pg/ml) IL-8(pg/ml) IL-6(pg/ml)
Copyright Harvard Medical School, 2010. All Rights Reserved.
1066
Outer medullarycongestion in human ischemic acute kidney
injury
Oxygen tension is reduced in the outer medulla Oxygen tension is reduced in the outer medulla
Cortex
Outer Medulla
Inner Medulla
pO
2
50 mm Hg
10- 20 mm Hg
vasarecta
O
2
Copyright Harvard Medical School, 2010. All Rights Reserved.
1067
10
0
30 60 50 40
85
83
20
0.2
1.0
0.8
0.6
0.4
0
Survival (%)
Time (days)
184
175
68
62
58
57
Numbers at risk
IHD
CVVHDF
IHD
CVVHDF
Survival in patients treated with IHD vs CVVHDF in the ICU
Vinsonneauet al.
Lancet 368:379,2006
VA/NIH Acute Renal Failure Trial Network. NEJ M 359:7-20, 2008
Intensive (6X/Week) vsLess Intensive (3X/Week) Dialysis
Copyright Harvard Medical School, 2010. All Rights Reserved.
1068
Meier et al. BMC Medicine 2009 7:23
Meta-analysis of Bicarbonate Therapy in Contrast Induced Nephropathy
> New terminology (AKI, Rifle Criteria)
> Changing epidemiology
> Pathophysiologyand Implications for
Pharmacologic and DialyticTherapies
Vasoconstriction
Inflammation
> Repair (?Stem cells)
> Biomarkers
Acute Kidney Injury
Copyright Harvard Medical School, 2010. All Rights Reserved.
1069
Ki67
67 at various times after ischemic in the mouse
Origin of Cells Involved in Tubular Epithelial Cell Repair
Loss of polarity Normal
Epithelium
Migration
Differentiation &
Reestablishment of polarity
Sloughing of viable and dead
cells with luminal obstruction
Ischemia/
Reperfusion
Apoptosis Necrosis
Cell death
Proliferation
Toxins
?
stem/progenitor
cell
Copyright Harvard Medical School, 2010. All Rights Reserved.
1070
Interstitial cells are not labeled with DsRed
(Macrophages labeled in green)
D
s
R
e
d
/
F
4
/
8
0
/
D
A
P
I
Inner Cortex Outer Medulla
Labeled epithelial cells proliferate
Serum Cr
(mg/dL)
2.0
1.0
0.5
1.5
Day 1 Day 15
Days post-IRI
Copyright Harvard Medical School, 2010. All Rights Reserved.
1071
Injection of MSCscultured on matrigel immediately post
I/R injury protected the kidney at 24 h without implantation
of cells in the kidney
0
0.5
1
1.5
2
2.5
PBS Mesenchymal
Stem Cells
Creatinine
(mg/dL)
**
Ischemic Kidney
BMSCsmay generated anti-inflammatory mediators in
AKI
Local production of inflammatory mediators Local production of inflammatory mediators
- - cytokines ( cytokines (TNF TNF , IL , IL- -1), 1), chemokines chemokines (IL (IL- -8, MCP 8, MCP- -1) 1)
- - complement activation products complement activation products
- - platelet activating factor (PAF) platelet activating factor (PAF)
- - metabolites of metabolites of arachidonic arachidonic acid acid
- - reactive oxygen species (ROS) reactive oxygen species (ROS)
ICAMs
VCAM
selectins
Release of ROS,
proteases, elastases,
leukotrienes, PAF
Increased expression of adhesion
molecules on endothelial cells
Endothelial Cell
ICAMs
VCAM
selectins
iC3b
iC3b
Activated
Leukocytes
BMSCs
()
t -inflammatory
mediators
Copyright Harvard Medical School, 2010. All Rights Reserved.
1072
> New terminology (AKI, Rifle Criteria)
> Changing epidemiology
> Pathophysiologyand Implications for
Pharmacologic and DialyticTherapies
Vasoconstriction
Inflammation
> Repair (?Stem cells)
> Biomarkers
Acute Kidney Injury
Biomarkers of injury in kidney and heart
Creatinine-based
AKIN, RIFLE
Stage dependent on absolute or
relative creatininerise
Urine output-based
AKIN, RIFLE
Stage dependent on duration and
extent of oliguria
Troponin, CPK, EKG
Biomarkers of myocardial cell injury
often associated with
Ischemic symptoms
Modified from SusWaikar
Copyright Harvard Medical School, 2010. All Rights Reserved.
1073
Damage Normal Epithelium
Toxic/ischemic
Injury
Apoptosis Necrosis
Cell death
Death
Death
Complications
Complications
Normal
Normal
Increased
risk
Increased
risk
Kidney
failure
Kidney
failure
Damage
Damage
GFR
GFR
Kidney Injury Molecule-1 (KIM-1)
Neutrophil Gelatinase Associated Lipocalin (NGAL)
N-acetyl--D-glucosaminidase (NAG)
Microalbuminuria
Cystatin C
Interleukin-18 (L-18)
Serum Creatinine
Blood Urea Nitrogen
Potential biomarkers for earl y diagnosis of AKI
AKIN Scheme,
2006
GFR
GFR
Delayed biomarkers for kidney
injury
Vaidya et al., Ann Rev Pharm Tox., 48, 2008
Copyright Harvard Medical School, 2010. All Rights Reserved.
1074
Nickolas, T. L. et. al. Ann Intern Med 2008;148:810-819
Study of Urine Biomarkers and Serum Creatinine in ER
on Admission
NGAL 0.948
Serum Creatinine 0.921
NAG 0.713
1-Microglobulin 0.887
1-Acid glycoprotein 0.832
Fract. Excret. Sodium 0.708
Nickolas et al. Ann Intern Med 2008;148:810-819
Test characteristics of Novel Biomarkers and Standard DiagnosticMarkers
in patients being admitted to the Columbia U. Hospital: ROC analysis
Urinary NGAL as determined by Western blot was equivalent
to serum creatininein predicting AKI
Biomarker AUC
Copyright Harvard Medical School, 2010. All Rights Reserved.
1075
Kim-1
BUN
SCr
AUC from ROC
ALL 0-3 0-2 0-1
A
Kim-1
BUN
SCr
Sensitivity
Histopathology Grade Subset
B
0.7
0.8
1.0
0.9
0.8
0.6
0.4
0.2
1.0
ALL 0-3 0-2 0-1
Comparison of Urinary KIM-1 to BUN or Creatinineas Predictors of
Pathology
Protective Safety
Testing
Consortium
(PSTC)
0.01
0.1
1
10
100
1000
KIM-1
Healthy AKI CKD UTI AAA CC ICU
0.0001
0.001
0.01
0.1
1
(n=50) (104) (50) (21) (13)
NAG
(70) (14)
NAG (U)
Creat (mg)
KIM-1 (ng)
Creat (mg)
Copyright Harvard Medical School, 2010. All Rights Reserved.
1076
Healthy Volunteers AKI CKD UTI
0.1
1
10
100
1000
10000
(n=50) (100) (51) (20)
Healthy AKI CKD UTI AAA CC ICU
0.01
0.1
1
10
100
1000
10000
100000
1000000
(n=50) (103) (21) (50) (11) (70) (14)
NGAL
IL-18
NGAL(ng)
Creat (mg)
IL-18 (pg)
Creat (mg)
Heated Cisplatin
Urinary NAG (U/mg creat.)
Pre-op 2-8 24 48 72 96
0.0000
0.0025
0.0050
0.0075
0.0100
0.0125
0.0150
0.0175
0.0200
0.0225
0.0250
0.0275
0.0300
0.0325
0.0350
0.0375
0.0400
0.0425
AKI
No AKI
Time Point (hrs)
Urinary KIM-1 (ng/mg creat.)
Pre-op 2-8 24 48 72 96
0.00
0.25
0.50
0.75
1.00
1.25
1.50
1.75
2.00
2.25
2.50
2.75
3.00
3.25
3.50
3.75
4.00
Time Point (hrs)
Urinary Protein (mg/mg creat.)
Pre-op 2-8 24 48 72 96
0.0
0.1
0.2
0.3
0.4
0.5
0.6
0.7
0.8
0.9
1.0
Time Point (hrs.)
AKI
No AKI
AKI
No AKI
Copyright Harvard Medical School, 2010. All Rights Reserved.
1077
0
2
4
6
8
10
12
14
16
C
1
D
1
C
1
D
1
P
O
S
T
C
1
D
2
C
1
D
3
C
1
D
4
C
1
D
5
C
2
D
1
C
2
D
1
P
O
S
T
C
2
D
2
C
2
D
3
C
2
D
4
C
2
D
5
C
3
D
1
C
3
D
1
P
O
S
T
C
3
D
2
C
3
D
3
C
3
D
4
C
3
D
5
C
4
D
1
C
4
D
1
P
O
S
T
C
4
D
2
C
4
D
3
C
4
D
4
C
4
D
5
Cycle and Day
N
o
r
m
a
l
i
z
e
d

b
i
o
m
a
r
k
e
r

v
a
l
u
e
KIM-1/creat
NAG/creat
Mean Urinary KIM-1 and NAG Levels During CisplatinTreatments
In Patients with Testicular Cancer
5 4 3 2 1 0
1.0
0.9
0.8
0.7
0.6
0.5
T1 (0.01 -0.48)
Tertiles of
urinary KIM -1
excretion (ng/24h)
Follow -up (years)
T2 (0.49 -1.09)
T3 (1.15 -10.04)
5 4 3 2 1 0
1.0
0.9
0.8
0.7
0.6
0.5
T1 (0.01 -0.48)
Tertiles of
urinary KIM -1
excretion (ng/24h)
C
u
m
u
l
a
t
i
v
e

s
u
r
v
i
v
a
l

(
%
)
Follow -up (years)
T2 (0.49 -1.09)
T3 (1.15 -10.04)
(1)
(2)
(3)
Kaplan-Meier survival curve of tertiles of urinary KIM-1 excretion (in ng/24 hr)
at baseline for death-censored graft survival. P=0.001
Van Timmeran,VanGoor,Bonventre,
Bakker et al. Transplantation, 2007
Copyright Harvard Medical School, 2010. All Rights Reserved.
1078
Board Question (1)
The dialytictherapy which is most effective in
reducing mortality in patients in the ICU
with acute kidney injury is:
a. Continuous hemodialysis
b. Intermittent hemodialysis- 3 times per week
c. Intermittent hemodialysis- 6 times per week
d. Slow extended daily but intermittent
hemodialysis
e. None of the above
Board Question (2)
Which of the following has been associated with
increased risk for AKI:
1. Colonoscopy preps
2. Rice from fields located near battery factories
3. The chineseherb Aristolochia
4. Infant formula made in China
5. None of the above
a. 1 and 2 c. 1,2, and 4
d. 1,3, and 4 e. All of the above
Copyright Harvard Medical School, 2010. All Rights Reserved.
1079
Summary
1. The term acute kidney injury (AKI) is replacing
acute renal failure to reflect the importance of the
entire spectrum of kidney injury to outcome
2. The incidence of AKI is increasing
3. AKI contributes to Chronic Renal Failure?
4. Dopamine or diuretics do not affect outcome in
AKI but diuretics may be helpful prognostically.
5. Continousrenal replacement therapies have not
been shown to have a better outcome than
intermittent dialysis.
Summary (2)
6. Bone marrow derived cells have antiinflammatory
effects to enhance recovery from AKI but do not
replace dead epithelial cells.
7. Proximal epithelial cells are replaced by replication
of proximal cells that have survived the insult, not
by an interstitial stem cell.
8. New biomarkers of kidney injury will transform the
way that we diagnose kidney disease, monitor
patients with chronic kidney diseases, and make
clinical trials much more efficient.
Copyright Harvard Medical School, 2010. All Rights Reserved.
1080
> New terminology (AKI, Rifle Criteria)
> Changing epidemiology
> Pathophysiologyand Implications for
Pharmacologic and DialyticTherapies
Vasoconstriction
Inflammation
> Repair (?Stem cells)
> Biomarkers
Acute Kidney Injury
Disclosure
Dr. Bonventre is co-inventor on KIM-1 patents , licensed to
Johnson and Johnson, Genzyme and BiogenIdec
Copyright Harvard Medical School, 2010. All Rights Reserved.
1081
KennethChristopher,MD,FASN,FCCP
RenalDivision
ICUNephrologyService
BrighamandWomensHospital
CVVH:AnOverview
Disclosures
I do not accept honorarium nor do consulting work
for the pharmaceutical or device industries.
I do not serve as a member of a speaking bureau
for pharmaceutical or device companies.
2009 Awarded competitive Research Grant
support from NxStageMedical
Copyright Harvard Medical School, 2010. All Rights Reserved.
1082
Case
34yearolddiabeticfemaleonLisinopril
WitnessedPEAarrest
Resuscitated
Progressivevolumeoverload(+15L)
Oliguric,diureticresistant
BP100/60onmoderatedoselevophed
Mechanicallyventilatedandsedated
Na129,K5.7,CO
2
14,BUN69,Cr4.7
BestRenalReplacementmodality?
Case
BestRenalReplacementmodality?
A.IHD
B.CVVH
C.CVVHD
D.CVVHDF
E.SLED
F.Alloftheabove
Copyright Harvard Medical School, 2010. All Rights Reserved.
1083
Optimizehemodynamicandvolumestatus
Minimizefurtherrenalinjury
Correctmetabolicabnormalities
RemovalofUremictoxins
Permitadequatenutrition
PrimaryTherapeuticGoalsinARF
Luyckx VASemin Dial.2004;17(1):306.
Hypercatabolic state
Hemodynamicinstability
ControlofVolumestatus
LimitationsonDialysisDeliveryinARF
Copyright Harvard Medical School, 2010. All Rights Reserved.
1084
CVVHContinuousVenoVenousHemofiltration
CVVHDContinuousVenoVenousHemodialysis
CVVHDFContinuousVenoVenousHemodiafiltration
SCUFSlowContinuousUltraFiltration
SLED SlowLowEfficiencyDialysis
ContinuousRenalReplacement
Therapy(CRRT)
SoluteRemoval
DiffusionandConvection
SoluteAddition
ReplacementFluid
FluidRemoval
Convection
Detoxification
PrimaryTherapeuticGoalsinCRRT
Copyright Harvard Medical School, 2010. All Rights Reserved.
1085
Convection
Ultrafiltration coefficient
Sievingcoefficient(Sc)
SolventDrag
Diffusion
Solventgradient
Effluent
SecondaryMembraneFormation
ReplacementSolution
Clearance
BasicCRRTConcepts
FluidRemovalandSoluteRemoval
Movementoffluidacrossthemembranevia
transmembrane pressure
Watermovesacrossthemembraneandcarries
dissolvedsoluteswithitviasolventdrag
Ultrafiltration isfluidremoval
Hemofiltration involvespartialortotalreplacementoffluid
removed
Convection
LucianoA. Pedrini J Nephrol 2003; 16 (suppl 7): S57-S63
Copyright Harvard Medical School, 2010. All Rights Reserved.
1086
Convection
H2O
Transmembrane Pressure
Convection
Transmembrane Pressure
H2O
Glucose
Cr,BUN
Toxins
Na,Cl
K,PO4,Mg
Ca,HCO3
Copyright Harvard Medical School, 2010. All Rights Reserved.
1087
Ultrafiltration plasmawater
Ultrafiltration coefficient ofthemembrane
Potentialtoremovewateradjustedforthetransmembrane
pressure
Soluteremovalfromplasma
Sievingcoefficient forthatsolute
Ratiobetweenthesoluteconcentrationintheultrafiltrate
anditsaverageplasmaconcentrationwithinthedialyzer
Inverselyrelatedtothesolutemolecularweight
~1.0withsmallsoluteandhighlypermeablemembranes
Convection
LucianoA. Pedrini J Nephrol 2003; 16 (suppl 7): S57-S63
ChelamcharlaM, SeminNephrol. 2005;25(2):81-9
Movementofsolutesdownaconcentrationgradient
acrossasemipermeable membrane.
Solutescrossthemembranefromthebloodtothe
dialysisfluidcompartment.
Fluidinthedialysiscompartmentmovesina
countercurrentdirection,therebymaintaininga
concentrationgradient.
Diffusion
Luciano A.Pedrini JNephrol 2003;16(suppl 7):S57S63
Copyright Harvard Medical School, 2010. All Rights Reserved.
1088
Diffusiveclearanceisdeterminedby
Molecularweightofthesolute
Concentrationgradientacrossthemembrane
Membranesurfacearea
Thicknessandporesize
Diffusion
Representstheendproductofthefiltrationprocess
CVVH: Ultrafiltrate
CVVHD: Dialysate plusvariableultrafiltrate
CVVHDF: Dialysate plusultrafiltrate
Effluent
Copyright Harvard Medical School, 2010. All Rights Reserved.
1089
CVVH
Continuous
VenoVenous
Hemofiltration
Access
Return
Effluent
Replacement
P
R
I
S
M
A
Prefilter
Postfilter
CVVHD
Continuous
VenoVenous
HemoDialysis
Access
Return
Effluent
P
R
I
S
M
A
Dialysate
Copyright Harvard Medical School, 2010. All Rights Reserved.
1090
CVVHDF
Continuous
VenoVenous
HemoDiaFiltration
Dialysate
Access
Return
Effluent
Replacement
P
R
I
S
M
A
Prefilter
Postfilter
SCUF CVVH
Hemofiltration
CVVHD
HemoDialysis
CVVHDF
HemoDiaFiltration
Blood filter
Middle Molecule
Clearance
Replacement Fluid
Dialysate
High
Permeability
+
None
None
High
Permeability
+++
RF
None
Low
Permeability
-
None
D
High
Permeability
+++
RF
D
TechnicalAspects
AdaptedfromJohnsonRJ,Feehally J,ComprehensiveClinicalNephrology2
nd
Edition
Copyright Harvard Medical School, 2010. All Rights Reserved.
1091
Replacestheultrafiltrate removedbyhemofiltration
andhemodiafiltration.
Buffers:Lactate,bicarbonateorcitrate
LactateandCitratemetabolizedbyliverandmuscle
tobicarbonate.
Bicarbonateismosteasilytolerated
canbeunstableinsolution
Lactateismorestable
maycontributetoanexistinglacticacidosisinsepticorliver
failurepatients
Citrate
Regionalanticoagulation
ReplacementSolution
P HeeringIntensiveCareMed 1999, 25:1244-1251
Tan HK Int J Artif Organs. 2003 J un;26(6):477-83.
CitrateRegionalAnticoagulation
Citratecausesanticoagulationbychelation of
calciumintheextracorporealsystem
Systemicanticoagulationdoesnotoccur
asionizedcalciumlevelisrestoredwhenbloodreturning
fromtheextracorporealsystemismixedwithvenousblood
Rapidmetabolismofcitraterestoresbicarbonatelevel
andreleasescalcium
Patientswithsevereliverfailureandlacticacidosis
mayhavedifficultymetabolizingcitrateanddevelop
citratetoxicity
Copyright Harvard Medical School, 2010. All Rights Reserved.
1092
CitrateReplacementsolution
SodiumCitrate=40mEq/Liter,
Dextrose=2gm/Liter,
SodiumChloride=105mEq/Liter,
MagnesiumSulfate=1.5mEq/Liter.
CVVHwithmaximumreplacementof2000cc
citrate/hour.
CVVHDFadddialysate toimproveclearance.
Calciumreplacementscalesadjusteddependingon
clearance
CitrateToxicity
LowionizedCalciumdespiterepletion
ElevatedtotalserumCalciuminresponseto
repletion
Exacerbationofserumacidosis
Elevationofaniongap
DecreaseCitratereplacementsolutionrate
Switchreplacementsolutions
Copyright Harvard Medical School, 2010. All Rights Reserved.
1093
Pre vs.postdilution
Postdilution:
Highsmallsolute
clearance/ml
UFratelimitedby
Hematocrit
QB
MaximalUFis25%plasma
flow
Predilution:
Lessclearance/ml(diluted)
Reducedefficiencyby1015%
UFrateisnot limited
Access
Return
Effluent
Replacement
P
R
I
S
M
A
Prefilter
Postfilter
Replacementfluidinfusedattheproximalside
Relativelylowviscosityofthebloodwithinthefilter
Efficiencyofultrafiltration iscompromised
Reducesnetfiltrationfraction
Minimizesconcentrationofclottingfactors
Prolongsfilterlifespan
PreDilution
J oseph V. DiCarlo, MD ALL-NET Pediatric Critical Care Textbook
Copyright Harvard Medical School, 2010. All Rights Reserved.
1094
Replacementfluidinfusedviadistalside
Mostefficienttechnique
Maximumclearances
Maximallydehydratesthebloodinthehemofilter
Higherviscosityofthebloodwithinthefilter
Clottingoffilter
PostDilution
Hemofilters allowthepassageofmoleculeswitha
molecularweightoflessthan50,000Daltons.
Smallmoleculesfreelyfiltered
sodium,potassium,bicarbonate,glucoseandammonia.
Largersolublesubstancesfiltered
myoglobin,insulin,andinterleukins
medications(vancomycin,heparin)
Proteinboundmoleculesarenotfilteredeffectively
Ultrafiltration Membranes
Copyright Harvard Medical School, 2010. All Rights Reserved.
1095
Pressureexertedonthemembraneduringoperation
Primaryfactordeterminingfiltrationrate
Pressuredifferencebetweenbloodandfluid
compartment
Usualrange100 500mmHg
IncreasedTMPassociatedwithincreasedUFrate
TMP=[(FilterPressure+ReturnPressure)/2] EffluentPressure
TMPTransMembrane Pressure
Clearanceistherateatwhichsolutesarecleared
excretionrateofsolute/bloodconcentrationofsolute
SoluteclearanceinCVVHdependsonUFRate
ClearanceinPredilution 1015%<Postdilution
Predilution modeClearanceimproved
largehemofilters
highUFRate
Clearance
Copyright Harvard Medical School, 2010. All Rights Reserved.
1096
StrategiestoIncreaseSoluteClearance
IncreaseTransmembrane pressure
Increasefilterpermeabilitytowater
Increasefiltersurfacearea
Increasebloodflow
CVVHDF
Adddialysistoconvection
J ohnson RJ , FeehallyJ , ComprehensiveClinical Nephrology 2
nd
Edition
Proteinlayerformedoversurfaceofdialyzermembranewith
transmembrane pressure
Reducestheeffectivesoluteandwaterpermeability.
Adsorptionoccursinallmembranes
MagnifiedinCRRT
Prepumpincreasesfilterlife
Dilutebloodgoingintothefilter
Diluteproteinandredcellconcentration
SecondaryMembraneFormation
gellayerorproteincake
Ronco C,Contrib Nephrol.1991;93:1758
MehtaR,AtlasofDiseasesoftheKidney1999
Copyright Harvard Medical School, 2010. All Rights Reserved.
1097
Patientswith/atriskforhypotension:
severehemodynamicinstability
hepaticfailure
CHF
sepsisormultipleorganfailure
Patientsatriskofcerebralcomplications:
hepaticfailure,strokeorheadtrauma
highriskforcerebraledema
IndicationsforCRRT
Increasedmetabolic needs
massiveburns
sepsis
multipleorganfailure
Volumeoverload
massivevolumeoverload
Patientsreceivinglargeamountsoffluidsorbloodproducts
Whenvolumemanagementiscritical
IndicationsforCRRT
Copyright Harvard Medical School, 2010. All Rights Reserved.
1098
NonrenalIndicationsforCRRT
Lacticacidosis
Ongoingproduction
Crushinjury
Myoglobin Removal
Tumorlysis syndrome
Temperaturecontrol
Relativehyperorrelativehypothermia*
MassivevolumeoverloadwithoutARF
HighNH
3
*Kopcke J.Anaesthesiol Reanim.1996;21(6):15962.
PotentialAdvantagesofCRRT
Increasedtotalsoluteclearance
Gradualclearancemaybebettertolerated
Decreasesfrequencyofhypotension
Decreasesriskofcerebraledema
Continuousclearancemayhelpinremovaloftoxins
withhighintracellularconcentrations
Increasedclearanceofmiddlemolecules
Preciseadaptablevolumecontrol
Copyright Harvard Medical School, 2010. All Rights Reserved.
1099
Anticoagulationrequirement
Lackofrapidfluidandsoluteremoval
Limitedroleinoverdosesetting
RelativeHypothermia
ElectrolyteDepletion
K,PO
4
,Ca
Potential disadvantagesofCRRT
TechnicalComplicationsofCRRT
Vascularaccessmalfunction
Bloodflowreductionandcircuitclotting
Lossoffilterefficiency
Linedisconnection
Airembolism
FluidandElectrolytebalanceerrors
Copyright Harvard Medical School, 2010. All Rights Reserved.
1100
ClinicalComplicationsofCRRT
Bleeding
Thrombosis
LineInfectionandsepsis
RelativeHypothermiamaskinglowgradefever
Nutrientlosses
Inadequatebloodpurificationduetodowntime
median3hperdaydowntime
1
Thrombocytopenia
destructionorretentionofplateletspassingthroughhemofilter.
2
1.UchinoS,Intensive CareMed.2003Apr;29(4):5758.
2.Mulder J,Int JArtif Organs.2003;26(10):90612.,
Lifethreateninghyperkalemia
ContraindicationsofCRRT
Copyright Harvard Medical School, 2010. All Rights Reserved.
1101
Minimalremovalofproteinbounddrugs
Somemayberemovedviamembraneadsorption
DrugclearancesincreasedwithCVVHDF
HigherdosesmaybeneededwithhigherUFrates
GFR=10ml/minLowUFrate
GFR=30ml/minMediumUFrate
GFR=50ml/minHighUFrate
DrugDosageinCRRT
MehtaR,AtlasofDiseasesoftheKidney1999
Bugge JF.Acta Anaesthesiol Scand.2001;45(8):92934.
eGFR =ReplacementSolutionmlperhour
Idealbodyweight
eGFR =3000ml/hr
70kg
eGFR=42
EstimateGFRinCRRT
Copyright Harvard Medical School, 2010. All Rights Reserved.
1102
Increasedazotemia inducedbyproteinoramino
acidsinTPN
1
FluidoverloadcausedbytheadministrationofTPN
1
Negativebalancesofselenium,copper,thiamine
2,
LossofMagnesiumandcalcium
3
HighproteindietsafeforpatientswithARFon
CRRT
4
Proteinintakeof2.5g/kg/day
increasesthelikelihoodofpositivenitrogenbalance
5
correctsaminoaciddeficiencies.
6
NutritionandCRRT
1.Bellomo R.BloodPurif.2002;20(3):296303.2.BergerMM.AmJClin Nutr.2004;80(2):4106.
3.KleinCJJPENJParenter Enteral Nutr.2002;26(2):77924.Bellomo R.Ren Fail.1997;19(1):11120.
5.Scheinkestel CDNutrition.2003;19(1112):90916.6.Scheinkestel CD.Nutrition.2003;19(9):73340.
Manyimmunemediatorsarewatersolubleandfall
intothemiddlemolecularweightcategory
Theoreticallyremovedviahemofiltration.
adsorptiontothefiltermembrane
CRRTinanimalmodelsofsepsisandhave
demonstratedsomebeneficialeffects.
CVVHreducesplasmaTNF,butnotIL6,IL10.
Earlyisovolemic CVVHat2l/hourinestablished
sepsis
Noreductionofseveralcytokinesandanaphylatoxins
Organdysfunctionnotimproved.
CVVHhasnotbeenshowntobenefitSIRS/Sepsisin
theabsenceofARF
CRRTandSepsis
Venkataraman R.CriticalCare2003,7:139145
Copyright Harvard Medical School, 2010. All Rights Reserved.
1103
HVHF:ultrafiltration ofmorethan>35ml/kg/hour
sepsisdosesofultrafiltration 3.86l/hour
increasedsurvival
decreasedvasopressor requirements
animalstudies
significanthemodynamicbenefit
improvementinimmunecellresponsiveness
reducedmortality
MorestudiesareneededtoclarifytheroleofHVHF
hyperdynamic septicshock
withorwithoutacuterenalfailure
sepsisandSIRS.
HighVolumeHemoFiltration andSepsis
ColeL.IntensiveCareMed2001,27:978986
OudemansvanStraaten HM,IntensiveCareMed1999,25:814821.
Honore PMInt JArtif Organs.2004,27(12):107782.
Noconsistentstatisticallysignificantdifferencein
survival
Vinsonneauet al. Lancet 2006; 368: 379-85
Bothmethodsarecomplementary
IHD
FasterPotassiumelimination
FasterDrug/Toxinelimination
Betterforoverdose
CRRT
Regulationofhighercaloriesrequirements
Hemodynamically unstablepatients
Preciseadaptablevolumecontrol
CRRTvs Hemodialysis
Copyright Harvard Medical School, 2010. All Rights Reserved.
1104
AllcriteriaforinitiatingCRRTareabsent
Urineoutputaverages1ml/kg/hrovera24hrperiod
Fluidbalancecanbekeptapproximatelyneutralwith
currenturineoutput
ThereisacomplicationrelatedtoCRRT
Whencriteriaarefulfilledbegina1224hourperiod
withoutCRRT
ReevaluateforindicationsforCRRT
CRRTshouldbeinitiatedearlyandceasedlate
BellomoR, RoncoC. Intensive Care Med 1999;25:781-9
CessationofCRRT
InContinuousVeno VenousHemodiafiltration
(CVVHDF)solutesareclearedbythefollowing
processes:
A.ultrafiltration
B.convection
C.diffusion
D.adsorbtion
E.A,BandC
F.BandC
BoardQuestion
1
Copyright Harvard Medical School, 2010. All Rights Reserved.
1105
InContinuousVeno VenousHemodiafiltration
(CVVHDF)solutesareclearedbythefollowing
processes:
A.ultrafiltration
B.convection
C.diffusion
D.adsorbtion
E.A,BandC
F.BandC
BoardQuestion
1
AdvantagesofCVVHoverHemodialysis include(T/F)
A.PreciseAdaptableVolumeControl
B.EnhancedMiddleMoleculeclearance
C.HigherFilterPermeability
D.EnhancedSurvival
E.Higherclearanceperunittime
F.A,BandC
BoardQuestion
2
Copyright Harvard Medical School, 2010. All Rights Reserved.
1106
AdvantagesofCVVHoverHemodialysis include(T/F)
A.PreciseAdaptableVolumeControl T
B.EnhancedMiddleMoleculeclearance T
C.HigherFilterPermeability T
D.EnhancedSurvival F
E.Higherclearanceperunittime F
BoardQuestion
2
Sources
MehtaR,SupportiveTherapiesAtlasofDiseasesoftheKidneyChapter19Eds TBerl and
JVBonventre.BlackwellScience1999
Luciano A.Pedrini JNephrol 2003;16(suppl 7):S57S63
Chelamcharla M,Semin Nephrol.2005;25(2):819
JohnsonRJ,Feehally J,ComprehensiveClinicalNephrology2ndEd
Venkataraman R.CriticalCare2003,7:139145
ColeL,Crit CareMed.2002Jan;30(1):1006.
Luyckx VASemin Dial.2004;17(1):306.
Humes HD.KidneyInt.2004;66(4):157888.
ColeL.IntensiveCareMed2001,27:978986
Bugge JF.Acta Anaesthesiol Scand.2001;45(8):92934
OudemansvanStraaten HM,IntensiveCareMed1999,25:814821.
Honore PMInt JArtif Organs.2004Dec;27(12):107782.
Bellomo R.BloodPurif.2002;20(3):296303.
BergerMM.AmJClin Nutr.2004;80(2):4106.
KleinCJJPENJParenter Enteral Nutr.2002;26(2):7792
Bellomo R.Ren Fail.1997;19(1):11120.
Scheinkestel CDNutrition.2003;19(1112):90916.
Scheinkestel CD.Nutrition.2003;19(9):73340.
UchinoS,Intensive CareMed.2003Apr;29(4):5758.
Mulder J,Int JArtif Organs.2003;26(10):90612.,
Splendiani G.Artif Organs.2000Apr;24(4):3058
vanBommel EF.AmJNephrol.2000;20(5):40811.
Agostini M.HumExpToxicol.2003;22(3):1657.
Kopcke J.Anaesthesiol Reanim.1996;21(6):15962.
Dicarlo JV.ALLNETPediatricCriticalCareTextbook
Copyright Harvard Medical School, 2010. All Rights Reserved.
1107
Disclosures
I do not accept honorarium nor do consulting work
for the pharmaceutical or device industries.
I do not serve as a member of a speaking bureau
for pharmaceutical or device companies.
2009 Awarded competitive Research Grant
support from NxStageMedical
Copyright Harvard Medical School, 2010. All Rights Reserved.
1108
AKI Syndromes
Sushrut S. Waikar, MD, MPH
Assistant Professor, HMS
Associate Physician, Renal Division,
Brigham and Womens Hospital
Disclosures
Investigator-initiated grant support from
Pfizer, Genzyme, Satellite HealthCare,
NxStage; DSMB member for Takeda
Copyright Harvard Medical School, 2010. All Rights Reserved.
1109
Case 1
A 65 year old man with known alcohol dependence
was found unconscious on his living room floor.
He was brought to the emergency department
where he was intubated for airway protection.
Vitals signs following intubation: T 98.3 BP
86/54 HR 116 O2 sat 99% on FI02 0.40
140 108 67 Calcium 7.6 Phos 8.5
6.3 15 4.6 Uric acid 4.3
Case 1 cont.
The urine was noted to be red. After
centrifugation, the sample remained red.
Centrifuged plasma sample was yellow.
Copyright Harvard Medical School, 2010. All Rights Reserved.
1110
Case 1 question
What is the most likely cause of his acute
kidney injury?
A. Hemolysis
B. Rhabdomyolysis
C. Acute intermittent porphyria
D. Paroxysmal nocturnal hemoglobinuria
Case 1 question
What is the most likely cause of his acute
kidney injury?
A. Hemolysis
B. Rhabdomyolysis
C. Acute intermittent porphyria
D. Paroxysmal nocturnal hemoglobinuria
Urine supernatant CLEAR in hematuria, but remains RED in rhabdomyolysis
and hemoglobinuria. Plasma supernatant may remain RED in hemoglobinuria
due ot hemoglobins large size compared to myoglobin (which is filtered and
cleared)
Copyright Harvard Medical School, 2010. All Rights Reserved.
1111
Rhabdomyolysis
Dissolution of skeletal muscle leading to leakage of
contents into circulation
Creatine kinase, aldolase, LDH, ALT, AST
AKI in approximately 50% of cases
1
Causes
Crush syndrome, limb compression
Strenuous exercise, seizures
Disorders of glycolysis, glycogenolysis, lipid metabolism
Infections (Influenza A and B), coxsackie-virus, HIV, EBV, Strep,
Staph
Heat stroke, malignant hyperthermia, malignant neuroleptic
syndrome
Hypokalemia, hypophosphatemia, hypocalcemia, DKA
Drugs: fibrates, statins, alcohol, heroin, cocaine
1
Melli et al. Medicine 2005
Rhabdomyolysis pathogenesis
Myoglobin
17.8 kDa protein freely filtered by glomerulus
Appears in urine when renal threshold of 0.5 to 1.5
mg/dL is exceeded, usually when serum levels >100
mg/dL
Intrarenal vasoconstriction
Direct tubular injury
Reactive oxygen species, free radicals
Tubular obstruction
Precipitates in tubules with Tamm-Horsfall protein,
especially in acidic urine
Copyright Harvard Medical School, 2010. All Rights Reserved.
1112
Rhabdomyolysis Dx
Weak correlation between CK and AKI
Rare to have AKI with admission CK <15,000
Unless concomitant volume depletion, sepsis, etc
Heme positive urine with absent RBCs
Myoglobin
True pathogenic factor but early peak, rapid
metabolism: low sensitivity for diagnosis
Low FeNa is common!
May reflect primacy of vasoconstriction and tubular
occlusion rather than tubular necrosis
Rhabdomyolysis Rx
Early and aggressive volume repletion
Lack of trials comparing different fluids
Sodium bicarbonate
Minimizes precipitation of myoglobin:TH protein
Inhibits redox cycling of myoglobin, lipid peroxidation
May minimize vasoconstriction
BUT may worsen hypocalcemia
Normal saline
Disadvantage: hyperchloremic metabolic acidosis
Diuretics (only after volume repletion)
Mannitol: antioxidant, may relieve tubular obstruction, may help
improve hypovolemia due to osmotic gradient with injured muscle
cells
Loop diuretics: not well studied, frequently used
Copyright Harvard Medical School, 2010. All Rights Reserved.
1113
Rhabdomyolysis treatment
guidelines
Normal saline 400 ml/h
If urine pH <6.5, alternate each NS liter with 1L of D5W / NS plus 100
mmol bicarb
Target urine output ~200 ml/h
Frequent K checks
Correct hypocalcemia only if tetany, seizures, or if severe
hyperkalemia occurs
Consider mannitol up to 200 g/d, cumulative 800 g
Discontinue if diuresis not established
Check plasma osms and gap if used, stop >55 mosm/kg
Hyperkalemia management (usual)
Loop diuretics
Hemodialyis for rising hyperkalemia, anuria, volume overload
Adapted from Bosch NEJ M 2009
Case 2
A 48 year old previously health man
presented to the ED complaining of
fatigue, anorexia, and weight loss. He took
no prescription or over the counter
medications. On exam, BP 168/94 HR 98
RR 16 O2 sat 95% RA
128 99 271 Calcium 7.1 Phos 5.3
8.4 7 24.6 Uric acid 4.3
Copyright Harvard Medical School, 2010. All Rights Reserved.
1114
Case 2 (cont.)
Moderate bilateral hydronephrosis, hydroureter, and
markedly distended bladder
Obstructive uropathy
Patient was admitted, underwent hemodialysis, and
received bilateral nephrostomy tubes and a suprapubic
catheter. Foley catheter placement was impossible due
to urethral stricture.
Urine output was brisk following relief of obstruction. HD
stopped after a single session.
Two weeks after presentation urine output was 6 liters per
day, reflective of severe postobstructive polyuria.
Discharged home with PICC for 2L normal saline/d plus
KCl, sodium bicarbonate supplementation. Required IVF
for 8 weeks.
Copyright Harvard Medical School, 2010. All Rights Reserved.
1115
Case 2 question
Metabolic complications during post-
obstructive polyuria include impairment in:
A. Urinary concentrating ability
B. Urinary diluting ability
C. Sodium reabsorption
D. Urinary acidification
Post-renal
Stones, blood
clots, external
compression,
tumor, RP
fibrosis
BPH, blood
clots, cancer
Strictures
FOLEY
Copyright Harvard Medical School, 2010. All Rights Reserved.
1116
Obstructive uropathy
Early phase
Elevated intratubular pressure increases P
BS
, thereby reducing single
nephron GFR
Compensatory hyperemia (afferent vasodilation) that serves to increase
GFR
Later phase
Vasoconstriction, reduced renal perfusion
Reduction in Kf (ultrafiltration coefficient)
Recovery after obstruction
Dependent on duration, severity (partial/complete)
Complete obstruction: relatively complete recovery even after 1 week;
little or no recovery after 12wks
1
Tubular dysfunction during recovery
Progressive interstitial fibrosis following severe obstruction
Better et al. Am J Med 1973
Case 3
65 yr old man admitted for atrial fibrillation with
rapid ventricular response, hospital course
complicated by catheter associated urosepsis
Treated with ampicillin-sulbactam
Seven days into course of antibiotics:
diffuse erythematous rash across chest, arms, legs
Rise in serum creatinine: 0.9 1.7 on day 7
Progressive rise over next 5 days to BUN 96,
creatinine 6.8
Copyright Harvard Medical School, 2010. All Rights Reserved.
1117
Case 3 (cont.)
PMHx: A fib, HTN, T2DM (oral
hypoglycemics), normal baseline kidney
function
Meds: ASA, metoprolol, lisinopril,
ampicillin-sulbactam, glipizide, intermittent
dopamine during ICU stay for SBPs ~ 90
Allergies: none known
Case 3 (cont.)
Exam:
T 101.5, BP 88/50, HR 110, RR 22, O
2
93% 2L
Notable findings:
Diffuse erythematous rash over chest, arms, back
Bilateral crackles on lung auscultation
Tachycardic, no pericardial friction rub, J VP ~12
2+dependent edema
Labs: WBC 19, 12% eosinophils; BUN 96
and creatinine 6.8
Copyright Harvard Medical School, 2010. All Rights Reserved.
1118
Case 3 (cont.)
Requests the following biomarkers to formulate
a differential diagnosis of acute kidney injury
Fractional excretion of sodium
Urinalysis and sediment
Renal ultrasound
Urine eosinophils
Case 3 (cont.)
Requests the following biomarkers to formulate
a differential diagnosis of acute kidney injury
Fractional excretion of sodium
Urinalysis and sediment
Renal ultrasound
Urine eosinophils
1.6%
Positive: 10%
No hydro
Copyright Harvard Medical School, 2010. All Rights Reserved.
1119
Presumptive Dx, Rx
Allergic interstitial nephritis due to
ampicillin
Treatment recommended:
methylprednisolone 1gm IV x 1, then
prednisone 60mg per day
Presumptive Dx, Rx
Allergic interstitial nephritis due to
ampicillin
Treatment recommended:
methylprednisolone 1gm IV x 1, then
prednisone 60mg per day
Creatinine continues to rise after 1 week of
steroids; dialysis initiated
Kidney biopsy performed
Copyright Harvard Medical School, 2010. All Rights Reserved.
1120
Kidney biopsy
Acute tubular necrosis
No evidence of acute/chronic tubulointerstitial nephritis
Case summary
Late recognition of acute kidney injury
Reasonable interpretation of biomarkers
suggesting allergic interstitial nephritis
No response to aggressive treatment for
presumed diagnosis; gold standard
diagnosis obtained (kidney biopsy)
Copyright Harvard Medical School, 2010. All Rights Reserved.
1121
Acute interstitial nephritis
Acute interstitial nephritis is found in the infectious
diseases of children, particularly in diptheria and scarlet
fever, but may be met with in other infectious diseases
Acute interstitial nephritis
Common cause of unexplained AKI
Single series of 109 patients: 27% with AIN
1
Drug related in >70%
NSAIDs including COX-2 inhibitors
Penicillins and cephalosporins
Rifampin
TMP-SMX, other sulfonamides
Ciprofloxacin
Cimetidine
Allopurinol
Proton pump inhibitors
5-aminosalicylates
1
Farrington et al. QJ Med 1989
Copyright Harvard Medical School, 2010. All Rights Reserved.
1122
Acute interstitial nephritis
Infections
Pneumonia with AIN: legionella
Multisystem dysfunction, hemorrhage, confjunctival
suffusion: leptospirosis
Cytomegalovirus
Streptococcus
Auto-immune disorders
SLE
SJ ogrens
TINU (tubulointerstitial nephritis +uveitis) syndrome
Acute interstitial nephritis
Timing of onset is variable
3 to 5 days after second exposure, several weeks with first
exposure to drug
1 day following rifampin exposure
1
Classic triad: fever, rash, eosinophilia
Seen in only 10%
Can see:
WBCs, RBCs, WBC casts; rarely RBC casts
Subnephrotic proteinuria
Fanconis syndrome (glucosuria, aminoaciduria, phosphate
wasting), Type 2 RTA
Gallium scan: conflicting reports regarding sens/spec
Kidney biopsy: interstitial edema, infiltrate with T cells,
monocytes; also eosinophils, plasma cells, neutrophils
1
Ten et al. Mayo Clin Proc 1988
Copyright Harvard Medical School, 2010. All Rights Reserved.
1123
Acute interstitial nephritis
Treatment approach
Careful review of medication list, OTCs
Discontinuation of offending agent
+/- steroids
No definitive study has ever been (perhaps never
will be) performed
Some suggestion that early steroid therapy speeds
recovery
Largest study to date:
1
retrospective, N =61 only 9 of
whom were not treated with steroids
1
Gonzalez et al. Kidney Int 2008
Steroids for AIN?
Delay (days) in steroid therapy
F
i
n
a
l

S
C
r
Gonzalez et al. Kidney Int 2008
PRO CON
Clarkson et al. NDT 2004
Months
F
i
n
a
l

S
C
r
Copyright Harvard Medical School, 2010. All Rights Reserved.
1124
Case 4
47 year old woman previously healthy, on no
medications developed recurrent uveitis
starting 6 months prior to a rise in SCr.
Both episodes of uveitis were treated with
topical steroids and improved rapidly.
SCr was 0.9 mg/dL in February and
increased to 2.5 mg/dL in November,
prompting nephrology consultation
Case 4 (cont.)
Exam: BP 155/93 HR 82 Weight 100 kg
HEENT: mild conjunctivitis bilaterally
Lungs: clear
CV: RRR nl s1 s2 no rub, no J VD
Abd: obese, soft, ND, NT
Ext: no pretibial edema
Skin: no rashes on trunk, legs, or arms
J oints: no active tenosynovitis or arthritis
Back: no CVAT
143
3.5
104 14
23 2.5
Ca 9.8 Phos 2.2 glucose 88
Urine pr:cr 0.8 u/a: +glucose +LE - nit
Urine sediment: WBCs no casts
Copyright Harvard Medical School, 2010. All Rights Reserved.
1125
Case 4 (cont.)
ANCA neg
Anti GBM neg
C3 C4 normal
SPEP / UPEP negative
Negative ANA, dsDNA, anti-Ro, anti-La
Renal ultrasound: normal size, echogenicity
CXR: no abnormalities
Case 4 question
The most likely diagnosis is:
A. Sarcoidosis
B. Bechets syndrome
C. Lupus nephritis
D. TINU syndrome
E. Sjogrens disease
F. Wegeners granulomatosis
Copyright Harvard Medical School, 2010. All Rights Reserved.
1126
Case 4 question
The most likely diagnosis is:
A. Sarcoidosis
B. Bechets syndrome
C. Lupus nephritis
D. TINU syndrome
E. Sjogrens disease
F. Wegeners granulomatosis
TINU syndrome
Tubulointerstitial nephritis and uveitis: first described in
1975
In addition to eye and kidney, may have:
Fever, weight loss, fatigue, malaise, anorexia, abdominal pain,
arthralgias, headache, polyuria
Uveitis: bilateral, usually anterior chamber, can precede
or follow nephritis
Females >males; younger age predominance
Treatment: steroids for severe cases. Many reports of
spontaneous resolution
Uveitis can be chronic and relapsing whereas ATIN usually self-
limited
Pathogenesis unknown: ? Common antigen in uveal and
tubular cells
Copyright Harvard Medical School, 2010. All Rights Reserved.
1127
Case 4 followup
0
0.5
1
1.5
2
2.5
3
0 14 56 63 77 84 105 133 203 259
Days from diagnosi s
S
C
r

(
m
g
/
d
L
)
Prednisone 60mg / day
REFUSED
Case 5
31 year old woman with morbid obesity, bipolar disease on
chronic lithium therapy was brought to an outside
hospital with altered mental status and respiratory
failure. Her initial presentation was notable for tremors,
confusion, hypoxemic and hypercarbi respiratory failure
requiring intubation, and AKI. SCr was 2.5 mg/dL; lithium
level was 3.9 meq/L.
She was intubated and underwent hemodialysis, after
which lithium levels fell to below 1 meq/L. SCr recovered
to baseline (0.6 mg/dL). A 2 week ICU course was
notable for difficulty in weaning from mechanical
ventilation, catheter associated bacteremia with
vancomycin-sensitive enterococcus and persistent
fevers to 104. She was transferred to BWH for continued
care and the availability of a CT scan with adequate
capacity for her weight of ~500lbs.
Copyright Harvard Medical School, 2010. All Rights Reserved.
1128
Case 5 (cont.)
She underwent contrast-enhanced CT of the
chest, abdomen, and pelvis for continued fevers
despite antibiotic therapy for enterococcal
bacteremia.
0
1
2
3
4
1 2 3 4 5 6 7 8
Day
S
C
r

(
m
g
/
d
L
)
CT +contrast
Case 5 (cont)
Relevant to her acute kidney injury:
Meds on transfer: cefepime, diflucan, linezolid,
vancomycin
Exam
BP 100/60 (no pressors) HR 112 Temp 104, CVP 12 after 8+
liters volume in 72h
Urine output 10 ml/hr
Coarse breath sounds, tachycardic without rub, obese belly, ?
edema, no rashes, small decubitus ulcer on back
Labs
Na 140, K 4.5, Cl 108, Bicarb 16, BUN 49, SCr 3.3
Ca 8.1, Ph 4.3, LFTs normal, CPK 370
WBC 4.5, Hgb 7.7, Plt 86
Vancomycin 58
UNa <assay, UCr 350, Uosms 290, +++muddy brown casts
CT imaging: no hydronephrosis
Copyright Harvard Medical School, 2010. All Rights Reserved.
1129
Case 5 (cont)
Relevant to her acute kidney injury:
Meds on transfer: cefepime, diflucan, linezolid,
vancomycin
Exam
BP 100/60(no pressors) HR 112 Temp 104, CVP 12 after 8+
liters volume in 72h
Urine output 10 ml/hr
Coarse breath sounds, tachycardic without rub, obese belly, ?
edema, no rashes, small decubitus ulcer on back
Labs
Na 140, K 4.5, Cl 108, Bicarb 16, BUN 49, SCr 3.3
Ca 8.1, Ph 4.3, LFTs normal, CPK 370
WBC 4.5, Hgb 7.7, Plt 86
Vancomycin 58
UNa <assay, UCr 350, Uosms 290, +++muddy brown casts
CT imaging: no hydronephrosis
Case 5: our formulation
Differential diagnosis
Contrast nephropathy
Sepsis +AKI
TTP-HUS
Vancomycin
nephrotoxicity
Allergic interstitial
nephritis (cefepime)
Abdominal
compartment
syndrome
Management issues
Dialysis?
Diuretics?
Fluids?
Copyright Harvard Medical School, 2010. All Rights Reserved.
1130
Case 5 teaching points
Low FeNa = pre-renal azotemia (only)
Vancomycins nephrotoxicity?
Role of diuretics in management of oliguria
FeNa
Percentage of filtered sodium that is
excreted
Copyright Harvard Medical School, 2010. All Rights Reserved.
1131
FeNa
Percentage of filtered sodium that is
excreted
What is your FeNa?
FeNa
Percentage of filtered sodium that is
excreted
What is your FeNa?
Depends on sodium intake, GFR
Copyright Harvard Medical School, 2010. All Rights Reserved.
1132
FeNa
Percentage of filtered sodium that is
excreted
What is your FeNa?
Depends on sodium intake, GFR
Intake = 150 meq/d = Excreted
Filtered = 140 meq/L * GFR (180L/d) = 25,200 meq
FeNa
Percentage of filtered sodium that is
excreted
What is your FeNa?
Depends on sodium intake, GFR
Intake = 150 meq/d = Excreted
Filtered = 140 meq/L * GFR (180L/d) = 25,200 meq
Ratio = 0.6%... youre pre-renal!
Copyright Harvard Medical School, 2010. All Rights Reserved.
1133
What the FeNa tries to answer
If your urine output is LOW, is it because:
Youre dry as a bone, holding on to salt and
water?
OR
Your tubules are dead, clogged up, low GFR:
cannot reabsorb sodium even if you wanted
to?
It should not be used indiscriminately
It should not alone guide volume management
Can be helpful in: cardiorenal, hepatorenal
FeNa -- data
Urinary Diagnostic Indices in ARF
Miller et al., Ann Int Med 1978
Copyright Harvard Medical School, 2010. All Rights Reserved.
1134
FeNa -- data
Urinary Diagnostic Indices in ARF
Miller et al., Ann Int Med 1978
Pre-renal Oliguric ATN
U
osm
518 369
U
Na
18 68
FeNa 0.4% 7.0%
FeNa -- data
Urinary Diagnostic Indices in ARF
Miller et al., Ann Int Med 1978
Pre-renal Oliguric ATN Acute GN
U
osm
518 369 385
U
Na
18 68 22
FeNa 0.4% 7.0% 0.6%
Copyright Harvard Medical School, 2010. All Rights Reserved.
1135
FeNa -- exceptions
Low FeNa frequently seen in
Early sepsis (ATN)
Rhabdomyolysis (ATN)
Contrast nephropathy (ATN)
Urinary tract obstruction
High FeNa despite being pre-renal
Diuretic use (Fe
urea
one study)
Chronic kidney disease (=excreted/filtered)
Case 5 teaching points
Low FeNa = pre-renal azotemia (only)
Role of diuretics in management of
oliguric AKI
Copyright Harvard Medical School, 2010. All Rights Reserved.
1136
Case 5 teaching points
Low FeNa = pre-renal azotemia (only)
Role of diuretics in management of
oliguric AKI
Furosemide in oliguric AKI
Pathophysiological rationale
Sodium reabsorption =major energy-requiring
activitiy of the kidney
Inhibition of Na/K/Cl pump in medullary thick
ascending limb
Decrease renal tubular cell oxygen demand
Popular!
70% of ICU patients with AKI given a diuretic at
enrollment (Uchino Crit Care Med 2004)
Clinical equipoise with respect to efficacy
Observational data suggest harm (Mehta JAMA 2002)
Randomized controlled trials fail to show mortality
benefit (Cantarovich AJKD 2004)
Copyright Harvard Medical School, 2010. All Rights Reserved.
1137
Ongoing RCT
Furosemide drip (titrated to urine output) vs. placebo
Target enrollment: 216 ICU pts with early SIRS, early AKI
Primary endpoint: progression to more severe AKI
(doubling of SCr)
Disclosures
Investigator-initiated grant support from
Pfizer, Genzyme, Satellite HealthCare,
NxStage; DSMB member for Takeda
Copyright Harvard Medical School, 2010. All Rights Reserved.
1138
AKI and Cancer
Benjamin D. Humphreys MD, PhD
Renal Division Renal Division
Brigham and Womens Hospital /
Harvard Medical School
Disclosures
None relevant to AKI and cancer None relevant to AKI and cancer
Copyright Harvard Medical School, 2010. All Rights Reserved.
1139
Case 1: Myeloma
50 yo previously healthy woman developed
fatigue and weight loss. Rejected for life
insurance due to new finding of S Cr 3.2
mg/dL.
Delayed follow up 6 mo later, Cr 5.3, Hct 28.8,
free light chain 0.7 g/dL free kappa light chain.
Urine Pr:Cr =8 g/g, fine granular casts
Renal biopsy
Micrographs courtesy of Helmut Rennke, MD
Copyright Harvard Medical School, 2010. All Rights Reserved.
1140
Lambda Kappa
Copyright Harvard Medical School, 2010. All Rights Reserved.
1141
Light Chain Deposition Disease
Aka Monoclonal Ig Deposition Disease
Monoclonal Ig deposition in glomeruli, tubular BM
and arteries (unlike Amyloid, also Congo Red
negative)
Glomeruli: nodular glomerulosclerosis, nephrotic
proteinuria
Tubules: BM thickening, dramatic interstitial
fibrosis
Arterioles: PAS positive deposits, arteriolar
sclerosis
Copyright Harvard Medical School, 2010. All Rights Reserved.
1142
Multiple Myeloma
Kyle et al., 2003
1027 pts
1985-1998
0.2
0.4
1.0
F
r
a
c
t
i
o
n

A
l
i
v
e
2 4 6 8 10
Time (mo)
0.6
0.8
12 0
Survival after diagnosis
MGUS
No MGUS
Renal Involvement in MM
50% of pts have increased Cr at diagnosis
9% will require dialysis at some point
5,739 patients developed ESRD from MM in 2007,
representing 48% of patients with cancer-related
ESRD.
20% will develop progressive renal failure during course
of disease
Patients with renal failure have poorer prognosis, and
are frequently excluded from aggressive or high dose
chemotherapy protocols due to expected toxicities
USRDS, 2007. Goldschmidt, H. et al. Nephrol. Dial. Transplant. 2000 15:301-304;
Copyright Harvard Medical School, 2010. All Rights Reserved.
1143
Bence J ones Protein
J ones, H. On a new substance occurring in the urine of a patient with mollities ossium. Phil Trans r Soc London
1848;138:55.
Saturday, November 1, 1845
Dear Dr. J ones,
This tube contains urine of a very high specific gravity. When boiled, it
becomes highly opake. On the addition of nitric acid it effervesces, assumes
a reddish hue, and becomes quite clear, but as it cools, assumes the
consistence and appearance which you see. Heat re-liquefies it. What is it?
Henry Bence J ones
AKI in Multiple Myeloma
Cast Nephropathy
Amyloidosis (AL)
LCDD
Interstitial nephritis
Hypercalcemic AKI
Urate nephropathy
Plasma cell infiltration
Hyperviscosity syndrome
Other paraprotein diseases:
Fanconi disease
Goldschmidt, H. et al. Nephrol. Dial. Transplant. 2000
15:301-304; doi:10.1093/ndt/15.3.301
Light
chains
Ca2+
Tubular
Injury
Casts
Dehydration
THP
GFR
Plasma Cell
Copyright Harvard Medical School, 2010. All Rights Reserved.
1144
Cast Nephropathy: Pheresis or Not?
Standard therapy: Correct volume depletion, treat hypercalcemia,
discontinue NSAIDS, treat infection, etc
??Renal Biopsy - usually not in acute setting
Treat Myeloma: high dose Dex +/- melphalan, thalidomide
?Plasma exchange,
Conflicting data
How acute is renal insult?
What was baseline renal function?
What is the concentration of the circulating M-component?
Treatment of choice in hyperviscosity syndromes
Dialysis
Renal Failure in Hematopoietic
Stem Cell Transplantation
In US in 2007, more HSCT were
performed (18,000) than renal transplants
(16,629)
At least 5-15% of HCT recipients develop
ARF
And 5-20% ultimately develop CKD
Copyright Harvard Medical School, 2010. All Rights Reserved.
1145
Organ Transplantation in US
0
2500
5000
7500
10000
12500
15000
17500
20000
25 60
750
2500
3700
18,000
Annual Numbers of
Blood and Marrow Transplantations,
1970-2006
- Worldwide -
N
u
m
b
e
r

o
f

T
r
a
n
s
p
l
a
n
t
s
0
5,000
10,000
15,000
20,000
25,000
30,000
35,000
40,000
1970 1975 1980 1985 1990 1995 2000 2005
Year
Allogeneic
Autologous
Source: IBMTR
Copyright Harvard Medical School, 2010. All Rights Reserved.
1146
Typical HSCT
W
B
C
Conditioning Engraftment
Time (days)
HCT infusion
GCSF
Neutropenia, thrombocytopenia
HSCT Basics
Myeloablative HSCT
Goal is to eradicate disease completely
This also ablates the marrow, requiring HCT rescue
Non-myeloablative HSCT
Less aggressive preparative regimens, does not ablate
marrow, allows mixed chimerism.
Uses the graft vs. tumor effectfor disease eradication
Used for older patients, those with co-morbidities, or those
with indolent diseases such as CLL
Copyright Harvard Medical School, 2010. All Rights Reserved.
1147
HSCT Associated AKI and Mortality
Grade 1 ARF: >1.25 fold but <2-fold rise in serum Cr
Grade 2 ARF >2-fold rise in serum Cr but no HD
Grade 3 ARF =grade 2 parameters and requiring dialysis.
Parikh, CR Kid. Int, 2005
HSCT Associated AKI, Renal Replacement
and Mortality
HSCT Type
Mod-
severe
AKI
ARF
requiring
RRT
Mortality if
RRT
Myeloablative
Allogeneic 30-60% 20-30% 80%
Autologous 15-20% 5-10% 80%
Non-
myeloablative 40% 3-5% >70%
Copyright Harvard Medical School, 2010. All Rights Reserved.
1148
AKI after HSCT: Timeline
Adapted from Zager 1994
0
10
20
30
40
50
60
Time after HCT (days)
Tumor lysis
syndrome,
marrow
infusion
toxicity
TMA, CNI
toxicity
HRS/VOD,
ATN
AKI (no RRT)
AKI (RRT)
P
e
r
c
e
n
t

o
f

p
a
t
i
e
n
t
s
d
e
v
e
l
o
p
i
n
g
r
e
n
a
l

c
o
m
p
l
i
c
a
t
i
o
n

a
f
t
e
r

H
C
T
0 7 14 21 28 1 yr
Early AKI
Hypovolemia / hypotension
(diarrhea, third space losses, sepsis)
Functional pre-renal syndromes
HRS (see below)
CyA / tacrolimus, amphotericin
Copyright Harvard Medical School, 2010. All Rights Reserved.
1149
Case 2: VOD
45 yo M undergoes myeloablative allo-BMT
for AML
Between days +3 to +7 he gains 8 Kg with
dependent edema
Beginning on day +7 bilirubin rises to 2.1
mg/dL and rose daily thereafter; he becomes
jaundiced and has RUQ tenderness and
ascites. Liver U/S showed reversal of flow.
Creatinine rose slowly starting day +8, FeNa
0.05% despite IV lasix and sediment was bland
HRS/VOD: Pathogenesis
90% of HRS is due to VOD (other causes
include viral or drug-induced hepatitis)
Conditioning-related toxicity (Radio-chemoRx)
Hepatic endothelial and sinusoidal damage
cause sinusoidal obstruction, portal HTN and
intrahepatic porto-systemic shunting
Copyright Harvard Medical School, 2010. All Rights Reserved.
1150
HRS/VOD: Diagnosis
Weight gain (edema, ascites), painful
hepatomegaly and jaundice precede renal failure
Liver U/S often shows reversal of flow in the
portal vein.
BP is often low-normal, FeNa is very low and
sediment bland initially, +/- muddy brown granular
casts later.
Timing is helpful in diagnosis - VOD onset occurs
in days +3 to +30.
Concurrent sepsis syndrome is often trigger for
ARF
HRS/VOD: Treatment
Supportive care - reversible
Volume mgmt: very high daily fluid intake
Paracentesis often required, lactulose for
encephalopathy
Diuretics, keep Hb
>
11, avoid SPA
Dialysis: 50% will require; HD vs. CRRT
tPA, Heparin, LMWH have been tried with
little success. Defibrotide appears very
promising.
Copyright Harvard Medical School, 2010. All Rights Reserved.
1151
Late CKD: CNI toxicity
Most HCT recipients are not on CNI long
term - discontinued after 6 months.
A subset develop chronic GVHD and receive
longtermsteroids and CNI
At risk for chronic CNI toxicity
Case 3: TMA
45 yo M underwent myeloablative allo-BMT 10
months ago for CML
Complicated by stage 4 hepatic GVHD
Recently developed anemia, worsening HTN,
Cr from 1.6 to 2.8 mg/dL
Sediment: hyaline casts, 500 mg proteinuria/
24 hrs
1-2 schistocytes, occ. low haptoglobin, plt 130
(down from 320), LDH 550
Copyright Harvard Medical School, 2010. All Rights Reserved.
1152
Late AKI/CKD: TMA
Also called BMT Nephropathy,
radiation nephropathyand
resembles HUS-TTP
Hypertension, hypervolemia
common
Disproportionate anemia, often
evidence of mild microangiopathic
hemolytic anemia (LDH, Hapto,
schistos, Plt)
Putative pathogenesis of TMA
Chemotherapy
Radiation
Kidney
+
Renal endothelial damage and
inflammation
TMA syndrome
Loss of EC thromboresistance
Platelet and fibrin deposition
Microvascular obstruction
Microangiopathic
hemolytic anemia
Hypertension
Renal
failure
Glomerular
ischemia
Genetic
factors? CNI?
Pro-inflammatory
cytokines?
GVHD
Infection
+ +
+
Renal
shielding
-
Pro coagulant
state?
+
Adapted from Humphreys, Soiffer and Magee, J ASN, 2005
Copyright Harvard Medical School, 2010. All Rights Reserved.
1153
TMA: Treatment
Goal is to limit further damage
Aggressive BP control <130/80
ACEI likely provide additional benefits
(hyperkalemia common)
Reduce/discontinue CNI if possible
40% will ultimately progress to ESRD
TMA: Treatment
Plasma exchange of unclear benefit, not used
unless florid Sx (eg TTP)
The Oklahoma TTP-HUS Registry has had 23
patients with HCT-related TTP-HUS
12
None of these had severe ADAMTS13 activity
deficiency
None of them responded to plasma exchange
Copyright Harvard Medical School, 2010. All Rights Reserved.
1154
Case 4
59 yo F underwent non-myeloablative
allo-BMT for relapsed AML
Complicated by self-limited oral and
ocular GVHD (stage I)
At 1 year post-BMT, she received HiB,
DPT and Pneumovax vaccines per
protocol. Cr =0.8.
Beginning 5 days later she developed
fatigue and swelling
She presented 14 days later with
anasarca, ARF (Cr =3.4) and 3+
proteinuria
Sediment
Copyright Harvard Medical School, 2010. All Rights Reserved.
1155
Copyright Harvard Medical School, 2010. All Rights Reserved.
1156
Response to steroids
Secondary causes of minimal
change nephropathy
Drugs
Nonsteroidal anti-inflammatory drugs
Ampicilli n
Gold
Allergens
Hymenoptera stings
Food
Pollen
Poison ivy and poison oak
Immunization
Infections
Viral
Cancer
Hodgkins disease
Copyright Harvard Medical School, 2010. All Rights Reserved.
1157
Nephrotic Syndrome after BMT:
How frequent?
Series of 163 consecutive non-myeloablative
HSCTs*
Found seven cases of nephrotic syndrome
Of these, 4 underwent renal Bx and all
showed membranous nephropathy
?Role of earlier withdrawal of
immunosuppression, marrow chimerismin
increased susceptibility to NS?
*Srinivasan R, BalowJ E, Sabnis S, et al., (2005) Nephrotic syndrome: an under-recognised
immune-mediated complication of non-myeloablative allogeneic haematopoietic cell
transplantation. Br J Haematol 131:74-79
Urate Nephropathy/TLS
Risk factors
Poorly differentiated lymphomas (Burkitts)
Acute lymphoblastic leukemia
Timing
Usually associated with induction chemotherapy
Occasionally spontaneous
Diagnosis
AKI +
Hyperkalemia
Hyperphosphatemia
Hypocalcemia
Hyperuricemia
Often high LDH
Urine uric acid : creatinine ratio is useful. If greater than 1, urate
nephropathy likely. If less than 0.6, unlikely.
Rhomboid crystals on sediment (absence does not rule out)
Copyright Harvard Medical School, 2010. All Rights Reserved.
1158
TLS: Pathophysiology
Purines
Xanthine
Uric acid
Tumor Cell lysis
Allopurinol
Urate oxidase
(Rasburicase IV)
Allantoin
Rapid cell
turnover
Chemoradiotherapy
Crystal
nephropathy
K
+
release PO
4
2-
release
Volume
expansion
Urinary
excretion
ARF
Adapted from Humphreys, Soiffer and Magee, J ASN, 2005
Urate Nephropathy/TLS
Treatment
Prevention through volume expansion and allopurinol
Alkalinizing urine - controversial, as alkaline pH
decreases urate crystalization but increases Calcium
phosphate crystalization.
Hemodialysis
- for severe TLS and life-threatening electrolyte
disturbances
- Intermittent HD followed by CRRT may be effective
to avoid rebound
Rasburicase
- Increasing use in USA
Copyright Harvard Medical School, 2010. All Rights Reserved.
1159
Rasburicase for Hyperuricemia
-120 -80 -40 0 40 80 120 160
0
10
20
30
Uric acid (mg/dL)
S Cr (mg/dL)
1
2
3
4
5
IVF
Hours post rasburicase
6 mg Rasburicase
Thrombotic Microangiopathy and
Chemotherapy: Gemcitabine
HumphreysMagee, Cancer, 2004
Cumulative incidence: 0.31 %
Most patients with
mild MAHA, sub
nephrotic
proteinuria and
bland sediment
Copyright Harvard Medical School, 2010. All Rights Reserved.
1160
Thrombotic Microangiopathy: Biopsy
Findings
HumphreysMagee, Cancer, 2004
Gemcitabine-TMA: Summary
More common than appreciated
HTN is an early predictor of patients at risk
Discontinuation of drug usually results in improvement
in renal disease
Plasmapheresis likely of no benefit
ACE/ARB are beneficial when tolerated (fibrinolytic
effects)
Copyright Harvard Medical School, 2010. All Rights Reserved.
1161
Case 5
A 41-year-old woman with no history of hypertension (BP
108/74mmHg) was started on sunitinib for GIST in 2002.
In 2005 she developed hypertension (BP 140/98mmHg
worsened to154/102mmHg) and required three
antihypertensives. Her S. Cr. rose from 0.8 to 2.0 mg/dl
and she was noted to have proteinuria (1.8gm/gm).
The washout period for sunitinib was increased to 4 weeks
(instead of the usual 2 weeks) followed by reduction in
dose to 37.5mg alternating with 25mg every other day.
Over the course of twenty-four weeks, BP improved to
108/72mmHg, urine protein to 0.4 gm/gm and S. Cr. to
0.9mg/dl.
Hypertension on Sutent
Azizi M et al. N Engl J Med 2008;358:95-97
Diastolic
B
P

a
t

H
o
m
e

(
m
m
H
g
)
Sunit. Sunit.
Systolic
Normotensive patients
180
60
120
0 5 10
Copyright Harvard Medical School, 2010. All Rights Reserved.
1162
Rapid Development of HTN by anti-VEGF
chemo
A B
RobinsonHumphreys CJ ASN, 2010
Hypertension induced by anti-VEGF
agents
Copyright Harvard Medical School, 2010. All Rights Reserved.
1163
Case 6
YS, a 79-year-old type 2 diabetic and
hypertensive woman, was started on sunitinib for
metastatic GIST.
She had no proteinuria and her S. Cr. was
1.2mg/dl. She subsequently developed nephrotic
range proteinuria and AKI to 2.4mg/dl, both of
which resolved after discontinuing sutent.
Two years later she was rechallenged with
Sorafenib, a different anti-angiogenic
chemotherapy, and proteinuria and renal failure
recurred.
Proteinuria on anti-Angiogenic Therapies
0 25 50 75 100 125 150
0
2
4
6
8
10
12
Weeks since starting protocol
Sunitinib
Sorafenib
SBP>200
Retaspimycin, erlotinib
nilotinib
SBP>200
Fatal MI
Imatinib
Copyright Harvard Medical School, 2010. All Rights Reserved.
1164
Maharaj et al., Am J Path 2006
VEGF
LacZ
Podocytes strongly express VEGF
Glomerulus
Arteriole lumen is
Completely occluded
J ust as seen in HUS/TTP
Copyright Harvard Medical School, 2010. All Rights Reserved.
1165
Almost all capillary lumens are obliterated by reactive
endothelium
Glomerular basement membrane is thickened
Called double contours reflecting new bsmt
Membrane secreted by injured endothelial cells
Thickened endothelial cell cytoplasm
with new rim of
Bsmt membrane that it is secreting
The capillary lumen is very narrowed
Copyright Harvard Medical School, 2010. All Rights Reserved.
1166
Reversible Renal TMA If Diagnosed in Time
Managing anti-VEGF renal toxiciteis
Control BP, use ACE/ARB first, followed by diuretics
and CCB
Monitor urine protein and kidney function monthly
If sub nephrotic proteinuria, may continue therapy
with aggressive BP and ACE/ARB
If renal failure, nephrotic proteinuria or hypertensive
emergency occurs, need dose reduction, washout or
discontinuation (in consultation with oncology)
Copyright Harvard Medical School, 2010. All Rights Reserved.
1167
Review Question 1
All of the following are possible causes of AKI related
to multiple myeloma EXCEPT:
A. Hypercalcemia
B. Postrenal obstruction due to retroperitoneal
lymphadenopathy
C. Direct plasma cell infiltration of the kidney
D. Intratubular obstruction from crystal formation
E. Light chain deposition disease
Review Question 1
All of the following are possible causes of AKI related
to multiple myeloma EXCEPT:
A. Hypercalcemia
B. Postrenal obstruction due to retroperitoneal
lymphadenopathy
C. Direct plasma cell infiltration of the kidney
D. Intratubular obstruction from crystal formation
E. Light chain deposition disease
Copyright Harvard Medical School, 2010. All Rights Reserved.
1168
Review Question 2
All of the following renal diseases are associated with
bone marrow transplantation EXCEPT:
A. Membranous nephropathy in the setting of GVHD
B. Hepatorenal syndrome due to veno-occlusive
disease
C. Chronic TMA syndrome
D. Interstitial nephritis
E. Tumor lysis syndrome
Review Question 2
All of the following renal diseases are associated with
bone marrow transplantation EXCEPT:
A. Membranous nephropathy in the setting of GVHD
B. Hepatorenal syndrome due to veno-occlusive
disease
C. Chronic TMA syndrome
D. Interstitial nephritis
E. Tumor lysis syndrome
Copyright Harvard Medical School, 2010. All Rights Reserved.
1169
References
1. S.M. Korbet and M.M. Schwartz. Multiple Myeloma. J ASN, 2006 17:2533-45.
2. Magee C, Pascual M. The growing problem of chronic renal failure after transplantation of a nonrenal organ. N Engl J
Med. 2003 Sep 4;349(10):994-6.
2. Griffiths M et al. Cyclosporin nephrotoxicity in heart and lung transplant patients. QJ M. 1996 Oct;89(10):751-63.
3. Broekroelofs J et al. Long-term renal outcome after lung transplantation is predicted by the 1-month postoperative renal
function loss. Transplantation. 2000 Apr 27;69(8):1624-8.
4. Ojo A et al. Chronic renal failure after transplantation of a nonrenal organ. N Engl J Med. 2003 Sep 4;349(10):931-40.
5. Angermann C et al. Reduction of cyclosporine after introduction of mycophenolate mofetil improves chronic renal
dysfunction in heart transplant recipients--the IMPROVED multi-centre study. Eur Heart J . 2004 Sep;25(18):1626-34.
6. Cohen E. Renal failure after bone-marrow transplantation. Lancet. 2001 J an 6;357(9249):6-7.
7. Zager R. Acute renal failure in the setting of bone marrow transplantation. Kidney Int. 1994 Nov;46(5):1443-58.
8: Humphreys B, Soiffer R, Magee C. Renal failure associated with cancer and its treatment: an update. J Am Soc Nephrol.
2005 J an;16(1):151-61.
9. Cohen E. Radiation nephropathy after bone marrow transplantation. Kidney Int. 2000 Aug;58(2):903-18.
10. Parikh C et al. Acute renal failure after nonmyeloablative hematopoietic cell transplantation. J Am Soc Nephrol. 2004
J ul;15(7):1868-76.
11. Cohen E. ESRD after bone marrow transplantation: poor survival compared to other causes of ESRD. Nephron. 1998
Aug;79(4):408-12.
12 George J N Et al The OklahomaTTP-HUS registry: A communityperspective of patients with clinicallydiagnosedTTP- 12. George J N. Et al. The Oklahoma TTP-HUS registry: A community perspective of patients with clinically diagnosed TTP-
HUS. Sem Hematol.2004 J an;41(1):60-67.
13. E.S. RobinsonB.D. Humphreys. Rapid Development of Hypertension and Proteinuria with Cediranib, an Oral Vascular
Endothelial Growth Factor Receptor Inhibitor. CJASN, 5(3):477-83, 2010.
14. V. EreminaS.E. Quaggin. VEGF Inhibition and Renal Thrombotic Microangiopathy. NEJ M, 13;358(11), 2008.
15. Patel TV, J .A. Morgan, and B.D. Humphreys. A Preeclampsia-like Syndrome Characterized by Reversible Hypertension
and Proteinuria Induced by Multifunctional Tyrosine Kinase Inhibitors. Journal of the National Cancer Institute.
100:282-84, 2008.
16. Humphreys BD, C.M. Magee. Gemcitabine-Associated Thrombotic Microangiopathy. Cancer 100: 2664-70, 2004.
Disclosures
None relevant to AKI and cancer None relevant to AKI and cancer
Copyright Harvard Medical School, 2010. All Rights Reserved.
1170
Nephrology
Board Review-3
Bradley M. Denker, M.D.
Associate Professor of Medicine, Harvard
Medical School
Physician, Brigham and Womens Hospital
Chief Nephrology Harvard Vanguard Medical Chief Nephrology, Harvard Vanguard Medical
Associates
BWH Renal Board Review 8/11/2010
Disclosures
Dr Denker has reported no financial Dr. Denker has reported no financial
relationships with commercial entities
producing, marketing, re-selling, or
distributing health care goods or services
consumed by, or used on, patients.
Copyright Harvard Medical School, 2010. All Rights Reserved.
1171
Question 1
67 year old man - 1 week history of anorexia 67 year old man - 1 week history of anorexia,
nausea, lassitude, and pedal edema.
Longstanding hypertension, well controlled
with hydrochlorothiazide and amlodipine.
Fenoprofen for osteoarthritis of the hip for the
past 3 months past 3 months.
Question 1 contd
Physical examination
BP 157/93mm, HR 72 bpm, Temp of , p , p
97.8
O
F. JVP 8 cm; normal cardiac and
pulmonary examinations; and 2+ pitting
edema.
Urinalysis showed a specific gravity of
1.017, protein 4+, 1+ blood, and
negative for glucose Microscopic negative for glucose. Microscopic
examination of the sediment showed 2-4
erythrocytes and 15-20 leukocytes/hpf,
and occasional granular casts.
Copyright Harvard Medical School, 2010. All Rights Reserved.
1172
BUN 93 mg/dL; Cr 7.8 mg/dL; Na 137, K 4.4, Cl
Question 1 contd
95, C02 21, Ca 9.2, Phos 7.8, UA 7.7 mg/dL;
Alb 2.9 g/dL; HCT 29%. ANCA (-) Antinuclear
(+) 1:40 titer, anti-dsDNA antibody level 0.
24 h protein excretion 7.7 g.
Renal ultrasound showed normal sized kidneys
bilaterally without obstruction. y
Three months previously his serum creatinine
was 1.7 mg/dL.
The nephrotic-range proteinuria and renal
Question 1
The nephrotic range proteinuria and renal
failure are most likely the result of:
A). Lupus nephritis
B). Multiple myeloma
C.) Systemic small vessel vasculitis
D.) Fenoprofen-induced nephrotic D.) Fenoprofen induced nephrotic
syndrome and interstitial nephritis
E.) Renal vein thrombosis secondary to
membranous nephropathy
Copyright Harvard Medical School, 2010. All Rights Reserved.
1173
The nephrotic-range proteinuria and renal
Quest i on 1
The nephrotic-range proteinuria and renal
failure are most likely the result of:
A). Lupus nephritis
B). Multiple myeloma
C.) Systemic small vessel vasculitis
D.) Fenoprofen-induced nephrotic
syndrome and interstitial nephritis
E.) Renal vein thrombosis secondary to
membranous nephropathy
Copyright Harvard Medical School, 2010. All Rights Reserved.
1174
NSAIDs and the Kidney NSAIDs and the Kidney
Prerenal azotemia Prerenal azotemia
Ischemic acute tubular necrosis
Allergic interstitial nephritis (AIN)
AIN plus minimal change nephropathy
ARF plus bilateral flank pain
Sodium and water retention
Hyperkalemia
CRF and papillary necrosis
Whelton and Hamilton, J Clin Pharm 31: 588, 1991
Question 2
A 27 year old man with the acquired immunodeficiency
syndrome (AIDS) is hospitalized with a cough, fever, and a
pulmonary infiltrate on CXR. Therapy is initiated with
trimethoprim-sulfamethoxazole. On admission, the serum
ti i i 1 6 /dL d bl d it 21 /dL creatinine is 1.6 mg/dL and blood urea nitrogen, 21 mg/dL;
On re-examination 3 days later, the serum creatinine is 2.2
mg/dL and blood urea nitrogen, 23 mg/dL. Results of
urinalysis both on admission and 3 days later are normal.
Urine output on day 3 is 1350 mL. The most likely cause of
the increased creatinine is:
A.) AIDS glomerulopathy
B.) Trimethoprim-mediated decrease in creatinine
secretion
C.) Intratubular obstruction secondary to sulfonamide
D.) AIN caused by trimethoprim-sulfamethoxazole therapy
E.) Acute tubular necrosis secondary to sepsis
Copyright Harvard Medical School, 2010. All Rights Reserved.
1175
Quest i on 2
A 27 year old man with the acquired immunodeficiency
syndrome (AIDS) is hospitalized with a cough, fever, and a
pulmonary infiltrate on CXR. Therapy is initiated with
trimethoprim-sulfamethoxazole. On admission, the serum
creatinine is 1 6 mg/dL and blood urea nitrogen 21 mg/dL; creatinine is 1.6 mg/dL and blood urea nitrogen, 21 mg/dL;
On re-examination 3 days later, the serum creatinine is 2.2
mg/dL and blood urea nitrogen, 23 mg/dL. Results of
urinalysis both on admission and 3 days later are normal.
Urine output on day 3 is 1350 mL. The most likely cause of
the increased creatinine is:
A.) AIDS glomerulopathy
B ) Trimethoprim mediated decrease in creatinine B.) Trimethoprim-mediated decrease in creatinine
secretion
C.) Intratubular obstruction secondary to sulfonamide
D.) AIN caused by trimethoprim-sulfamethoxazole
therapy
E.) Acute tubular necrosis secondary to sepsis
Copyright Harvard Medical School, 2010. All Rights Reserved.
1176
Trimethoprim-Sulfamethoxazole Trimethoprim-Sulfamethoxazole
Elevated creatinine, no change in BUN, no g
evidence of ARF: inhibition of tubular secretion
Allergic interstitial nephritis with fever, rash, and
eosinophilia induced by sulfa moiety
Hyperkalemia with salt wasting due to amiloride-
like action of trimethoprim p
Rarely, crystallization of sulfamethoxazole
metabolite and renal stone formation
43 yo white male is referred for evaluation of abnormal
renal function. The patient has fatigue, dry skin, and the
absence of sweating even in hot weather. He has severe
burning pain in the palm and soles during episodes of fever
Question 3
burning pain in the palm and soles during episodes of fever.
Medical history is otherwise neg. The patients mother has
skin lesions on lower trunk. Two male first cousins have
renal transplants for ESRD.
Exam-BP-130/90, P-76, Resp-16, Afebrile. Thumbnails are
normal There are many small dark purple-red papules on normal. There are many small, dark purple red papules on
the lower abdomen, genital area, hips and upper thighs
Labs: Bun-29, creat-1.9, CrCl-45cc/min with 1.0gms/24h
protein.
Copyright Harvard Medical School, 2010. All Rights Reserved.
1177
Which of the following EM pictures would
be most likely on renal biopsy of this patient?
A
B C
Which of the following EM pictures would
be most likely on renal biopsy of this patient?
A-Amyloid
B-Fabrys C-Thin BM
Copyright Harvard Medical School, 2010. All Rights Reserved.
1178
This patient has Fabrys disease and these are myeloid
bodies in the glomerular visceral epithelial cells.
Fabrys Disease
X-linked deficiency of -galactosidase A (1:40,000)
Substrate accumulates in endothelial, periendothelial
and smooth muscle cells of vascular system, renal
epithelial cell, myocardial cells, dorsal root ganglia and
cells of autonomic nervous system.
Clinical manifestations by age 10, ESRD by 30-40
Renal Manifestations: polyuria, polydipsia, Fanconis
syndrome, renal sinus cysts, proteinuria, unexplained
ckd, vacuolization of podocytes, distal tubules; (less
commonly proximal tubules) and vasculature
Copyright Harvard Medical School, 2010. All Rights Reserved.
1179
Question 4
A 52 year old African-American female presents to the
emergency room with unstable angina. She is noted to
have a past medical history of mild chronic renal
insufficiency ( creatinine of 1 8 mg/dL) She is insufficiency ( creatinine of 1.8 mg/dL). She is
transferred to the coronary care unit and therapy for
her unstable angina is initiated. A cardiac
catheterization is planned for the next day. Risk factors
that would predispose this woman to contrast
nephrotoxicty include all of the following except:
A.) Diabetes mellitus
B.) Pre-existing renal insufficiency
C.) The volume of IV contrast utilized in the procedure
D.) Presence of extracellular volume contraction
E.) A history of coronary artery disease
Quest i on 4
A 52 year old African-American female presents to the
emergency room with unstable angina. She is noted to
have a past medical history of mild chronic renal
insufficiency ( creatinine of 1 8 mg/dL) She is insufficiency ( creatinine of 1.8 mg/dL). She is
transferred to the coronary care unit and therapy for
her unstable angina is initiated. A cardiac
catheterization is planned for the next day. Risk factors
that would predispose this woman to contrast
nephrotoxicty include all of the following except:
A.) Diabetes mellitus
B.) Pre-existing renal insufficiency
C.) The volume of IV contrast utilized in the procedure
D.) Presence of extracellular volume contraction
E.) A history of coronary artery disease
Copyright Harvard Medical School, 2010. All Rights Reserved.
1180
Question 4
Her cardiologist asks you to recommend a strategy to minimize
the risk of contrast nephrotoxicity. Which would be most
appropriate?
A.) Start the patient on 0.45% saline and order furosemide
40 mg IV and 10 g mannitol on call to the procedure
B.) Start the patient on 0.45% saline at 1 ml/kg for 12 hours
pre and post procedure and give N-acetyl cysteine 1200 mg
po bid, day before and day of procedure
C.) Give N-acetyl cysteine 1200 mg po bid the day before
and day of the procedure and hydrate with isotonic sodium
bicarbonate at 3ml/kg for 1h pre and 1ml/kg for 6h post
procedure
D.) Stop the ibuprofen the patient is taking for her arthritis
E.) Use low osmolality ionic contrast
Copyright Harvard Medical School, 2010. All Rights Reserved.
1181
Question 4
Her cardiologist asks you to recommend a strategy to minimize
the risk of contrast nephrotoxicity. Which would be most
appropriate?
A.) Start the patient on 0.45% saline and order furosemide
40 mg IV and 10 g mannitol on call to the procedure
B.) Start the patient on 0.45% saline at 1 ml/kg for 12 hours
pre and post procedure and give N-acetyl cysteine 1200 mg
po bid, day before and day of procedure
C.) Give N-acetyl cysteine 1200 mg po bid the day before
and day of the procedure and hydrate with isotonic sodium
bicarbonate at 3ml/kg for 1h pre and 1ml/kg for 6h post
procedure
D.) Stop the ibuprofen the patient is taking for her arthritis
E.) Use low osmolality ionic contrast
Copyright Harvard Medical School, 2010. All Rights Reserved.
1182
Contrast Nephrotoxicity
(versus atheroemboli)
Contrast Nephrotoxicity
(versus atheroemboli)
ARF and oliguria within 24-48 hours ARF and oliguria within 24 48 hours
Peak serum creatininine on days 3-5
Low fractional excretion of sodium
Benign sediment or granular casts
Resolution usual within 1 week
Risk factors: CRI diabetic nephropathy Risk factors: CRI, diabetic nephropathy,
dose>120 cc, multiple myeloma, volume
Prevention: hydration 75 cc 0.9% saline, non-
ionic for high risk patients or NaHCO3
Risk of CN
20
25
Probability
of CN %
5
10
15
20
Source: Davidson et al, Ann Intern Med: 110, 119-124, 1989
0
44.2 88.4 132.6 176.8
mcmol/L 1.2 mg/dL 2.0 mg/dL
Copyright Harvard Medical School, 2010. All Rights Reserved.
1183
Contrast Nephropathy
Risk Factors
Pre-existing renal insufficiency Pre existing renal insufficiency
Diabetes mellitus
CHF
Volume Depletion
Dose/osmolality of Contrast agent
Question 5
What treatment resulted in the decreased risk of stone
formation in the following patient? 24 hour urine values
are given before and after therapy. Supersaturations are
given as SS Caox SSCaP and SSUA for calcium oxalate
Vol SSCAOX CA OX CIT SSCAP UpH SSUA UA
1.60 8.66 223 40 398 0.41 5.5 2.47 .876
2.45 3.93 205 39 1082 0.8 6.3 0.33 .767
< 6 0.5-2 0-1
given as SS Caox, SSCaP and SSUA for calcium oxalate,
calcium phosphate and uric acid.
A. Increased hydration alone
B. Na. Citrate
C. K. Citrate
D. Dietary Na Restriction
E. Allopurinol
Copyright Harvard Medical School, 2010. All Rights Reserved.
1184
Question 5
Vol SSCAOX CA OX CIT SSCAP UpH SSUA UA
1.60 8.66 223 40 398 0.41 5.5 2.47 .876
2.45 3.93 205 39 1082 0.8 6.3 0.33 .767
< 6 0 5 2 0 1 < 6 0.5-2 0-1
A. Increased hydration alone
B. Na. Citrate
C. K. Citrate
D. Dietary Na Restriction
E Allopurinol E. Allopurinol
Urinary citrate increased suggesting alkali therapy
K Citrate is preferred in most cases; Na increases Ca
+2
exchange and worsens hypercalciuria
SS CaP increased and SSUA decreased because of
increased U pH
Copyright Harvard Medical School, 2010. All Rights Reserved.
1185
Question 6
All of the following statements regarding
aminoglycoside nephrotoxicity are true, except:
A.) Aminoglycosides cause a secondary
phospholipidosis in lysosomes
B.) Aminoglycosides are proximal tubule toxins
C.) Hypokalemia increases the risk of
aminoglycoside nephrotoxicity
D.) Hypokalemia and hypomagnesemia are
commonly observed in patients with commonly observed in patients with
aminolglycoside nephrotoxicty
E.) Multidose regimen is preferred to single
dose regimens in order to reduce
aminoglycoside nephrotoxicity.
Quest i on 6
All of the following statements regarding
aminoglycoside nephrotoxicity are true, except:
A.) Aminoglycosides cause a secondary
phospholipidosis in lysosomes
B.) Aminoglycosides are proximal tubule toxins
C.) Hypokalemia increases the risk of
aminoglycoside nephrotoxicity
D.) Hypokalemia and hypomagnesemia are
commonly observed in patients with commonly observed in patients with
aminolglycoside nephrotoxicty
E.) Multidose regimen is preferred to single
dose regimens in order to reduce
aminoglycoside nephrotoxicity.
Copyright Harvard Medical School, 2010. All Rights Reserved.
1186
Aminoglycoside Nephrotoxicity Aminoglycoside Nephrotoxicity
25% incidence with therapeutic levels 25% incidence with therapeutic levels
some correlation with cumulative dose
non-oliguric ARF after 5-10 days
Toxicity correlates with cationic charge
Pathology: membrane phospholipidosis
Prevention: once daily dosing y g
Prine et al: Lancet 341: 335, 1993
Levinson Ann Intern Med 117: 694, 1992
Copyright Harvard Medical School, 2010. All Rights Reserved.
1187
Question 7
A 69-year-old man without any past medical
history. Two months ago he developed mild
f joint pains and muscle aches for which he
took a non-steroidal anti-inflammatory drug
(Voltaren, or diclofenac). He developed
generalized edema which has persisted for
the last 2 weeks. His physical examination is
unremarkable except for 3+ pitting edema of unremarkable except for 3+ pitting edema of
the lower extremities. Blood pressure is
normal. No skin lesions are noted and his
joint and muscles are non-tender.
His laboratory work-up reveals a 24 hour
Quest i on 7
His laboratory work-up reveals a 24 hour
urinary protein excretion of 16g. The urinary
sediment is remarkable for 3-4 red blood
cells/hpf without casts. His BUN is 25mg/dl,
creatinine 1.2mg/dl, and his creatinine
clearance was measured at 80ml/min.
Serologic tests show an ANA of 1:160, an
anti-DNA ab level of 26, and borderline low
complements.
Copyright Harvard Medical School, 2010. All Rights Reserved.
1188
Renal Biopsy is Performed:
IgG g
Th t lik l di i i
Question 7
The most likely diagnosis is:
A.) Class V (membranous) lupus nephritis
B.) Idiopathic membranous glomerulopathy
C.) Focal segmental glomerulonephritis
D.) Class III lupus nephritis ) p p
E.) Class IV lupus nephritis
Copyright Harvard Medical School, 2010. All Rights Reserved.
1189
Quest i on 7
The most likely diagnosis is:
A.) Class V (membranous) lupus nephritis
B.) Idiopathic membranous glomerulopathy
C.) Focal segmental glomeuonephritis
D ) Call III lupus nephritis D.) Call III lupus nephritis
E.) Class IV lupus nephritis
Copyright Harvard Medical School, 2010. All Rights Reserved.
1190
Renal Disease with SLE:
Minimal Mesangial Lupus Nephritis (Class I)
Mesangial Proliferative Lupus Nephritis
(Class II) (Class II)
Focal Lupus Nephritis (Class III)
Diffuse Lupus Nephritis (Class IV)
Membranous Lupus Nephritis (Class V)
Advanced Sclerosing Lupus Nephritis (Class
VI) VI)
Tubulointerstitial and Vascular
Membranous (Class V)
Normal
Membranous LN (V): Subepithelial immune deposits, diffuse basement
membrane thickening. Similar to idiopathic MN but if tubuloreticular
structures, highly suggestive of SLE
LM findings of subendothelial deposits indicates V+ III or IV.
IgG
10-20% of patients; Nephrotic syndrome, may have some hematuria.
Pure Membranous may not require immunosuppression but if
associated with III or IV have a poorer prognosis.
Copyright Harvard Medical School, 2010. All Rights Reserved.
1191
Question 8
All of the following are true regarding
pentamidine nephropathy except: p p p y p
A.) Often presents with a distal RTA
B.) Hypomagnesemia is common
C.) ECF volume contraction is a risk factor
D.) Nebulized drug virtually never causes
acute renal failure
E.) Acute renal failure is reversible on
discontinuation of the drug
Quest i on 8
All of the following are true regarding
pentamidine nephropathy except: p p p y p
A.) Often presents with a distal RTA
B.) Hypomagnesemia is common
C.) ECF volume contraction is a risk factor
D.) Nebulized drug virtually never causes
acute renal failure
E.) Acute renal failure is reversible on
discontinuation of the drug
Copyright Harvard Medical School, 2010. All Rights Reserved.
1192
Pentamidine Nephrotoxicity Pentamidine Nephrotoxicity
Risk factors: hypovolemia and CRF yp
25-95% incidence of ARF after 7 to 10 days
Several reports of ARF after nebulized drug
Often pyuria, proteinuria, hematuria, casts
Hypomagnesemia / Hypocalcemia, distal RTA
D t t b d d d t Does not appear to be dose dependant
Mechanism unknown
Recovery usually within weeks
Copyright Harvard Medical School, 2010. All Rights Reserved.
1193
Question 9
A 22-year old white male is seen in the emergency
room for evaluation of acute renal failure. He explains
that he has just finished running the Boston marathon. j g
He complains of severe leg cramps. He tells you that
his urine is light pink. He has no significant past
medical history. He is not taking any medications. He
denies recent alcohol consumption. His physical
examination shows a white male. His BP is 120/80
mmHG, heart rate of 110 bpm, afebrile. His JVP is 2-3
cm. He has clear lungs, a normal cardiovascular and g ,
abdominal examination. He has no edema. His skin
turgor is reduced. Urinalysis reveals a SG of 1020, pH
5.0, 4+ blood, rest negative. His urine sediment shows
2-4 RBCs but is otherwise negative.
All of the following statements are correct
Question 9
All of the following statements are correct,
EXCEPT
A.) His CPK level is likely to be elevated
B.) His serum phosphorous is likely to be
normal
C.) He should be rapidly volume repleted ) p y p
D.) His urine should be alkalinized
E.) Orthotoludine positive urine is typical
Copyright Harvard Medical School, 2010. All Rights Reserved.
1194
All of the following statements are correct
Quest i on 9
All of the following statements are correct,
EXCEPT
A.) His CPK level is likely to be elevated
B.) His serum phosphorous is likely to be
normal
C.) He should be rapidly volume repleted
D.) His urine should be alkalinized
E.) Orthotoludine positive urine is typical
Copyright Harvard Medical School, 2010. All Rights Reserved.
1195
Features of Rhabdomyolysis
Muscle pain and dark urine Coca-Cola color Muscle pain and dark urine Coca Cola color
Orthotoludine-positive urine without RBCs
Elevated CPK and myoglobin
Increased K, Phos, urate, decreased Ca
Rapid increase in serum creatinine
Mechanism: free radicals ferrihemate Mechanism: free radicals, ferrihemate,
reduced nitric oxide
Treatment: saline repletion, alkaline diuresis,
mannitol. Dialysis once ARF established
Causes of Rhabdomyolysis
Excessive muscle activity
seizures, delerium tremens, sport
Direct of ischemic muscle injury Direct of ischemic muscle injury
trauma, compression syndrome, vascular
occlusion
Metabolic disorders
hypokalemia, hyponatremia,
hypophosphatemia yp p p
Drugs or toxins
ethanol, isopropyl alcohol, heroin, methadone
Infections
tetanus, legionaires, influenza
Copyright Harvard Medical School, 2010. All Rights Reserved.
1196
Question 10
70 yo A M attorney develops pedal edema and foamy
urine. PMH-HTN, HypercholesPx BP 140/92, 1+ pedal
edema. Pt smokes 1 PPD and denies any medications
except rare ASA or tylenol.
Lab: BUN 28 mg/dl, Pcreat 1.6 mg/dl, U/A 3+ protein 5-
10 rbcs, P cholesterol 525 mg/dl, Palb 1.4 g/dl, 24 hr
UV prot10 g/d
ANA, Complement, VDRL, HBV, HCV
Bx- Stage II-III MN w segmental sclerosis
EM extensive subepithelial and some mesangial
deposits
Question 10
What is the chance that this patient will be
found to have a malignancy underlying his
membranous nephropathy?
A) 2%
B) 12%
C) 18% )
D) 25%
E) 40%
Copyright Harvard Medical School, 2010. All Rights Reserved.
1197
Question 10
What is the chance that this patient will be
found to have a malignancy underlying his
membranous nephropathy?
A) 2%
B) 12%
C) 18% )
D) 25%
E) 40%
Copyright Harvard Medical School, 2010. All Rights Reserved.
1198
Membranous Nephropathy and
Malignancy
MN has been associated with malignancy,
but unclear how often. but unclear how often.
Retrospective survey of 232 MN patients (148
M, 84 F) from 1994-2000.
24 pts (10.3%) had a known malignancy or
developed one within a year of diagnosis. In
pts > 65 years old, 25% had malignancy.
Only difference between the Idiopathic and
the Malignancy associated MN was increase
in infiltrating leukocytes.
Lefaucheur C, et al Kidney Int 2006
27 year old male from the country of Cape Verde
who wishes to be a kidney donor for his aunt, who is an
ESRD patient from polycystic kidney disease on dialysis
for several years. The patient is healthy male with no
hi t f HTN di b t d i h it li ti
Question 11
history of HTN or diabetes, and no prior hospitalizations.
PMH: none All: NKDA, Current Medications: none,
denies NSAIDs.
SH: no tob, social EtOH use, no drugs. Works as
musician in Africa.
FH: +FH of ADPKD on fathers side of family FH: +FH of ADPKD on father s side of family.
His father has HTN but no known kidney disease.
He has two paternal aunts both with ADPKD, (one is
the potential recipient).
He has 6 sisters and 3 brothers none of whom have
known kidney disease (ages 24-36).
Copyright Harvard Medical School, 2010. All Rights Reserved.
1199
Physical Exam:
BP 112/66, P 58, BMI 21
Benign physical exam
Question 11
Benign physical exam.
Data:
Cr 0.72, normal lytes
T-bili 1.8, ALT 13, AST 15, ALKP
57
No microalbuminuria No microalbuminuria
HIV, hepatitis B, hepatitis C
negative
Question 11
Which of the following recommendations are
most appropriate?
A. Obtain renal ultrasound and if no evidence of cystic
kidney disease, approve him as a donor
B. Obtain renal ultrasound and if no evidence of cystic
kidney disease, advise him to have MRI, and if negative
approve him as a donor
pp p
C. Advise against donation until he has a negative
ultrasound at age 40.
D. Advise waiting until age 30 and if ultrasound and
MRI shows one or less cysts, then go ahead with
donation
Copyright Harvard Medical School, 2010. All Rights Reserved.
1200
Question 11
Which of the following recommendations are
most appropriate?
A. Obtain renal ultrasound and if no evidence of cystic
kidney disease, approve him as a donor
B. Obtain renal ultrasound and if no evidence of cystic
kidney disease, advise him to have MRI, and if negative
approve him as a donor
pp p
C. Advise against donation until he has a negative
ultrasound at age 40.
D. Advise waiting until age 30 and if ultrasound and
MRI shows one or less cysts, then go ahead with
donation
Copyright Harvard Medical School, 2010. All Rights Reserved.
1201
Recommended Screening for ADPKD:
Ultrasound Criteria for Diagnosis of ADPKD:
15 to 39 years of age: at least three unilateral or
bilateral cysts bilateral cysts
40 to 59 years of age, two cysts in each kidney
>60 years or older, four cysts in each kidney.
To Exclude ADPKD:
>40, ultrasonographic evidence of zero or only one
renal cyst excludes the disease (negative PV=100%)
30-39 years of age, False negative rate of normal
ultrasound is ~2% (Normal CT or MRI would exclude
the disease.)
<30 years of age, ultrasonographic imaging is limited
in its ability to help exclude the disease.
61 year old man
Rheumatoid arthritis x 15 years & gold
therapy X 12 years with excellent control of
Question 12
therapy X 12 years, with excellent control of
his symptoms.
Six months prior he c/o of edema. Cr 1.0
mg/dl, 24-h protein 10 g, albumin 1.8 g/dl.
UA: 4+ albuminuria, bland sediment.
Gold salts D/Cd; proteinuria persists, 24h Gold salts D/C d; proteinuria persists, 24h
protein averaging 14 g/day.
Copyright Harvard Medical School, 2010. All Rights Reserved.
1202
On no medications other than lasix in
Question 12
On no medications other than lasix, in
particular, no NSAID in the recent past.
Physical examination showed a well-
developed man in no acute distress. Blood
pressure was 120/80. There was 4+ edema
present. present.
The most likely diagnosis is:
Question 12
A.) Membranous glomerulopathy
secondary to gold
B.) Idiopathic membranous glomerulopathy
C.) AA amyloidosis secondary to
rheumatoid arthritis
D.) Primary focal segmental
glomerulosclerosis
E.) Rheumatoid vasculitis
Copyright Harvard Medical School, 2010. All Rights Reserved.
1203
The most likely diagnosis is:
Quest i on 12
A.) Membranous glomerulopathy
secondary to gold
B.) Idiopathic membranous glomerulopathy
C.) AA amyloidosis secondary to
rheumatoid arthritis
D.) Primary focal segmental
glomerulosclerosis
E.) Rheumatoid vasculitis
Copyright Harvard Medical School, 2010. All Rights Reserved.
1204
Glomerular Disease and RA
Renal Disease with RA Renal Disease with RA
Membranous
Mesangial proliferative GN +/- IgA deposits
Diffuse proliferative GN
Necrotizing and crescentic GN
(rheumatoid vasculitis)
Amyloidosis
Renal Disease with treatment of RA
Renal Di sease and RA
Renal Disease with treatment of RA
Gold: MN, MCD, ATN
Penicillamine: MN, Crescentic GN, MCD
NSAIDs: AIN, MN, MCD, ATN
CsA: chronic vasculopathy and tubulopathy
Methotrexate: ATN / crystal induced ARF
Aza: AIN
Copyright Harvard Medical School, 2010. All Rights Reserved.
1205
65-year-old woman
Quest i on 13
65-year-old woman
Adult onset diabetes mellitus for the last 2
years.
Blood sugar levels have been controlled very
well on oral hypoglycemics.
2 th hi t f t h ti d 2-month history of overt nephrotic syndrome.
PMH: positive serology for hepatitis B.
Currently she is hepatitis B serum antigen
negative.
Exam: moderately obese
Question 13
Exam: moderately obese.
Edema of the lower extremities (+++) & the
lumbosacral area.
VS within normal limits, including a systemic
blood pressure of 140/85.
F d NL Fundoscopy NL.
Neurologic examination NL.
Copyright Harvard Medical School, 2010. All Rights Reserved.
1206
Labs
Quest i on 13
BUN 18, Cr 1
Ccr 90ml/min.
24 h urine 7.3g of protein
UA benign sediment. Only rare hyaline casts are
seen.
Lupus serology negative
RF factor +. Cryo (-).
Complements levels normal
Coags normal parameters
The most likely diagnosis is
Quest i on 13
y g
A.) Stage 4 diabetic nephropathy
B.) Minimal change disease with early
diabetic nephropathy
C.) Class V lupus nephropathy
D ) Post infectious glomerulonephritis D.) Post-infectious glomerulonephritis
E.) Henoch-Schonlein nephritis
Copyright Harvard Medical School, 2010. All Rights Reserved.
1207
The most likely diagnosis is
Quest i on 13
y g
A.) Stage 4 diabetic nephropathy
B.) Minimal change disease with early
diabetic nephropathy
C.) Class V lupus nephropathy
D ) Post infectious glomerulonephritis D.) Post-infectious glomerulonephritis
E.) Henoch-Schonlein nephritis
Copyright Harvard Medical School, 2010. All Rights Reserved.
1208
Natural History of Diabetic Nephropathy
ESRD
Stage 4
Stage 5
UAE
mg/d
3000
Overt
Nephropathy
Microalbuminuria
Stage 1 Stage 2
Stage 3
30
300
Early
DM
Structural
Changes
g g
0 2
5 12
25
years
30
Physiologic changes in DN
100-
50-
25-
75-
Copyright Harvard Medical School, 2010. All Rights Reserved.
1209
42 yo male with recurrent calcium oxalate stones. No
meds, No h/o of GI disease,UTI, or hypercalcemia
Exam - mild obesity, 100kg, BP-130/70 otherwise WNL
Urine - pH 5.0, trace blood,otherwise neg
Chemistry: electrolytes-normal, Ca-9.8, PO4-2.7, Creat-
Question 14
Chemistry: electrolytes normal, Ca 9.8, PO4 2.7, Creat
0.9
24h Urine:
Volume -1266mls (varies with intake)
Sodium - 358meq/d (varies with intake)
Potassium - 78meq/d (varies with intake)
Calcium - 320mg/d (<250mg/d) g ( g )
Oxalate - 42mg/d (<40mg/d)
Citrate - 250mg/d (450-600mg/d)
Urate - 870mg/d (<800mg/d)
Urea - 14.4g/d;Estimate DPI (6.25(14.4)+0.03 Wt(kg)
~93gms
Question 14; Which of the following might
reduce new stone formation?
A. HCTZ
B. Low Calcium Diet
C. Sodium Citrate
D. Potassium Bicarbonate
E Low Protein Diet E. Low Protein Diet
F. Allopurinol
H. Increased Fluid Intake
Copyright Harvard Medical School, 2010. All Rights Reserved.
1210
Answers/Pearls
A. Yes - Hctz will lower urinary calcium excretion A. Yes Hctz will lower urinary calcium excretion
B. NO - Low calcium diet (inc. oxalate absorb.)
C. NO - Sodium citrate - Inc. Na will inc. Ca exc.
D. Yes - K-bicarbonate - Inc. citrate exc, alk blood and
urine
E. Yes - Low protein diet - Decrease Ca, urate
excretion; Increase citrate excretion ;
F. Yes - Allopurinol - reduces urate excretion
G. Yes - Increase Volume to >2L/d; reduces conc.
Copyright Harvard Medical School, 2010. All Rights Reserved.
1211
42 yo WM referred for HTN, Renal
Insufficiency and crampy abdominal pain.
Question 15:
y py p
HTN diagnosed 2 years ago; on multiple
meds. PMH notable for gout involving right
knee and feet-last episode about 8 weeks ago
PE notable for BP-168/100; otherwise WNL
Laboratory Studies:
Hct 35 2% Hb 12 g/dl Hct 35.2%, Hb-12 g/dl
BUN/Creat - 42/2.9
Electrolytes;Na-140, K-4.8, Cl-22, HCO3-22
Uric acid - 9.4
UA- pH 5.0,1+ protein, no cells, casts
Question 15: Which study is most likely to
reveal the diagnosis?
A Urine porphobilinogen excretion A. Urine porphobilinogen excretion
B. 24h urine creatinine and beta2-
microglobulin excretion
C. Urine lead excretion following
administration of EDTA
D R l Ult h D. Renal Ultrasonography
E. Renal Biopsy
Copyright Harvard Medical School, 2010. All Rights Reserved.
1212
Question 15: Which study is most likely to
reveal the diagnosis?
A Urine porphobilinogen excretion A. Urine porphobilinogen excretion
B. 24h urine creatinine and beta2-
microglobulin excretion
C. Urine lead excretion following
administration of EDTA
D R l Ult h D. Renal Ultrasonography
E. Renal Biopsy
Copyright Harvard Medical School, 2010. All Rights Reserved.
1213
Question 15 Answer
C - Urine lead excretion following EDTA C Urine lead excretion following EDTA
Classic Triad of Renal Insufficiency, HTN
and gout
HTN duration too short to explain renal
insuff
Porphyria - GI and Neuro Sx, no renal p y
Without family history unlikely to have PKD
Potentially large numbers of patients and
treatable with chelation therapy.
Question 16
A Family of Patients is determined to have
Autosomal Dominant FSGS
f f ? Which of the following is True?
A. They likely have a mutation in the nephrin gene
B. If they have a podocin genetic defect it is likely to be
steroid responsive
C They are likely to have a mutation in a actinin IV or C. They are likely to have a mutation in a-actinin IV or
TrpC6 ion channel
D. Almost 50% of adults with FSGS have been found to
have genetic mutation encoding for structural
proteins or transport channels of the visceral
epithelial cell.
Copyright Harvard Medical School, 2010. All Rights Reserved.
1214
Question 16
A Family of Patients is determined to have
Autosomal Dominant FSGS
f f ? Which of the following is True?
A. They likely have a mutation in the nephrin gene
B. If they have a podocin genetic defect it is likely to be
steroid responsive
C They are likely to have a mutation in actinin IV or C. They are likely to have a mutation in -actinin IV or
TrpC6 ion channel
D. Almost 50% of adults with FSGS have been found to
have genetic mutation encoding for structural
proteins or transport channels of the visceral
epithelial cell.
Copyright Harvard Medical School, 2010. All Rights Reserved.
1215
Classification of FSGS
DAgati. 2008 Curr Opin Nephrol Hypertens 3;271
Idiopathic - Mediated by permeability factor? p y p y
Secondary:
Familial/Genetic (next slide)
Virus Associated: HIV, Parvovirus, SV40, CMV
Drug Induced: Heroin, Interferon-, Li, Pamidronate,
Sirolimus
Hemodynamic/Structural:
Reduced Renal Mass: agenesis, dysplasia, reflux, surgery,
chronic allograft nephropathy
Normal Mass (Initially): HTN, atheroemboli, obesity, SSD
Genes Mutated in Familial FSGS
SD
Cyto
BM
DAgati 2008 KI 73;399
Nuc
Copyright Harvard Medical School, 2010. All Rights Reserved.
1216
Molecular anatomy of the podocyte foot
process cytoskeleton
Podocalyxin
NHERF2
MAGI-1
NEPH-1?
Ezrin
V
T
P
ILK
PLC1
Somlo and Mundel, Nature Genetics 2000
FAT
TRPC6
Question 17
57 yo black female with h/o mild hypertension,
presents to the ED with headache and BP-240/140.
The evening before she had consumed a raw
hamburger and she c/o non bloody bowel movements hamburger and she c/o non bloody bowel movements
the morning of admission. Her physical exam was
notable for T=100.4
o,
S4 gallop, petechiae but no
edema or other abnormalities.
Labs notable Hct-22%, plts-95k, creat 4.5mg/dl
(3months prior 1.2mg/dl). Urinalysis - heme +, few ( p g ) y ,
rbcs, +1 protein, no casts. Admitted to ICU for IV
nipride.
Copyright Harvard Medical School, 2010. All Rights Reserved.
1217
Question 17
Patient Normal
The Most Appropriate Next Step is:
A Check ADAMTS-13 Level A. Check ADAMTS-13 Level
B. Obtain Stool Culture for E.Coli 01357
C. Perform Kidney Biopsy
D. Initiate Therapy with Steroids
E. Initiate Therapy with Steroids and
Plasmapheresis
Copyright Harvard Medical School, 2010. All Rights Reserved.
1218
Question 17: The Most Appropriate Next
Step is:
A Check ADAMTS-13 Level A. Check ADAMTS-13 Level
B. Obtain Stool Culture for E.Coli 01357
C. Perform a Kidney Biopsy
D. Initiate Therapy with Steroids
E. Initiate Therapy with Steroids and
Plasmapheresis
Copyright Harvard Medical School, 2010. All Rights Reserved.
1219
Question 17
All of the following conditions can present with similar
findings: Scleroderma Crisis, Hemolytic uremic
syndrome Thrombotic Thrombocytopenia Purpura syndrome, Thrombotic Thrombocytopenia Purpura,
Malignant Hypertension, Disseminated Intravascular
Coagulation.
Kidney Biopsy will not distinguish these etiologies
Incubation time too short for Ecoli associated HUS
The presence of findings of neurologic symptoms,
h l ti i th b t i l f il hemolytic anemia, thrombocytopenia, renal failure
and low grade fever suggests TTP.
Summary of Indications for
Plasmapheresis in Renal Disease
Disease Category
Anti-GBM disease I
TTP I TTP I
Rapidly Progressive Glomerulonephritis II
Cryoglobulinemia II
Hemolytic Uremic Syndrome III
Desensitization for Renal II
transplantation
Recurrent FSGS III
Renal transplant rejection IV
Systemic Lupus Erythematosus III
Category I: Standard Primary Therapy; Category II: Supportive
Therapy; Category III: When the evidence of benefit is unclear;
Category IV: No current evidence of benefit or for research protocols
Copyright Harvard Medical School, 2010. All Rights Reserved.
1220
Question 18: For Which of the Following
Conditions is the use of Plasmapheresis
Least Indicated
A Guillain-Barre Syndrome A. Guillain-Barre Syndrome
B. Myasthenia Gravis
C. Multiple Myeloma with Cast Nephropathy
D. Chronic inflammatory demyelinating
polyneuropathy
E. Hyperviscosity in monoclonal
gammopathies (Waldestrm
macroglobulinemia)
Question 18: For Which of the Following
Conditions is the use of Plasmapheresis
Least Indicated
A Guillain-Barre Syndrome A. Guillain-Barre Syndrome
B. Myasthenia Gravis
C. Multiple Myeloma with Cast Nephropathy
D. Chronic inflammatory demyelinating
polyneuropathy
E. Hyperviscosity in monoclonal
gammopathies (Waldestrm
macroglobulinemia)
Copyright Harvard Medical School, 2010. All Rights Reserved.
1221
Question 18
All the other indications are CATEGORY I All the other indications are CATEGORY I
Although early studies demonstrated improvement in
renal function and patient survival, the largest study
to date of 104 patients with multiple myeloma and
acute renal failure randomly assigned to conventional
therapy plus plasma exchanges or conventional
therapy alone did not show differences in composite therapy alone did not show differences in composite
outcome of death, dialysis dependence or severely
reduced renal function.
Clark WF, et al., Plasma exchange when myeloma presents as acute renal failure: a
randomized, controlled trial. Ann Intern Med 143:777-84. 2005.
Szczepiorkowski ZM, et al. Guidelines on the use of therapeutic apheresis in clinical
practiceEvidence-based approach from the Apheresis Applications
Committee of the American Society for Apheresis. J Clin Apher 22:106175.
2007
Copyright Harvard Medical School, 2010. All Rights Reserved.
1222
A 45-year-old woman is referred for recurrent oxalate
stone formation. The following is her 24-hr urine result.
Question 19
Vol SSCAOX CA OX CIT SSCAP UpH SSUA UA
1.52 10.04 40 143 23 0.09 5.5 0.53 .202
6-10 < 200 20-40 >550 0.5-2 0-1 <0.75
What is the likely cause of her disorder?
A. Renal tubular acidosis resulting in hypocitraturia
B. Dietary excess of nuts, chocolate, and berries
C. Hyperparathyroidism
D. Crohns disease
E. Oxalosis
A 45-year-old woman is referred for recurrent oxalate
stone formation. The following is her 24-hr urine result.
Question 19
Vol SSCAOX CA OX CIT SSCAP UpH SSUA UA
1.52 10.04 40 143 23 0.09 5.5 0.53 .202
6-10 < 200 20-40 >550 0.5-2 0-1 <0.75
What is the likely cause of her disorder?
A. Renal tubular acidosis resulting in hypocitraturia
B. Dietary excess of nuts, chocolate, and berries
C. Hyperparathyroidism
D. Crohns disease
E. Oxalosis
Copyright Harvard Medical School, 2010. All Rights Reserved.
1223
Answer: D
Vol SSCAOX CA OX CIT SSCAP UpH SSUA UA
1.52 10.04 40 143 23 0.09 5.5 0.53 .202
There is low citrate, high oxalate, and low volume in this
intestinal malabsorptive condition.
RTA does not explain increased U oxalate
6-10 < 200 20-40 >550 0.5-2 0-1 <0.75
Nuts etc do not explain degree of hyperoxaluria
Hyperparathyroidism only explains low U Ca
Oxalosis does lead to low U citrate, low U Calcium
Copyright Harvard Medical School, 2010. All Rights Reserved.
1224
A 55 yo woman is admitted to the hospital with 4 day h/o
tea colored urine and renal failure. 8 weeks prior she
received PCN for streptococcal pharyngitis and completed
10 days of therapy. Based upon UA showing rbcs, wbcs,
Question 20
she was given bactrim for 2 days prior to admission for
presumed UTI. Routine labs done 2 days ago return with
creat. 2.9. She denies any respiratory or sinus symptoms.
6 months ago renal function and UA were normal.
On admission she is afebrile, BP-146/96 with Pulse-90 and
R 20/ i Ad i i l b l t 4 2 d UA Resp-20/min. Admission labs reveal creat 4.2 and UA
shows many dysmorphic rbcs and rbc casts with 3+
protein. CXR is unremarkable.
Which of the following approaches is
most appropriate at this time?
A Obtain serologies for ANCA Anti-GBM ANA A. Obtain serologies for ANCA, Anti GBM, ANA,
ASLO, Hepatitis B, C, C3, C4 ESR and schedule
renal biopsy for the following morning.
B. Obtain serologies and initiate methylprednisolone
1 gm IV while awaiting renal biopsy.
C. Obtain emergent renal biopsy, serologies, and
wait for results before initiating therapy. g py
D. Obtain emergent renal biopsy, serologies, and
begin plasmapheresis and IV solumedrol while
awaiting test results.
Copyright Harvard Medical School, 2010. All Rights Reserved.
1225
Which of the following approaches is
most appropriate at this time?
A Obtain serologies for ANCA Anti-GBM ANA A. Obtain serologies for ANCA, Anti GBM, ANA,
ASLO, Hepatitis B, C, C3, C4 ESR and schedule
renal biopsy for the following morning.
B. Obtain serologies and initiate methylprednisolone
1 gm IV while awaiting renal biopsy.
C. Obtain emergent renal biopsy, serologies, and
wait for results before initiating therapy. g py
D. Obtain emergent renal biopsy, serologies, and
begin plasmapheresis and IV solumedrol while
awaiting test results.
Copyright Harvard Medical School, 2010. All Rights Reserved.
1226
H and E
Fibrin
Anti-glomerular basement membrane antibody
Copyright Harvard Medical School, 2010. All Rights Reserved.
1227
Correct Answer - D, begin therapy with
plasmapheresis and steroids.
Although the differential includes post-strept and g p p
bactrim mediated acute renal failure, this is too
long a latency period for post-strep and renal
failure already present prior to starting bactrim.
Therefore, this is a rapidly progressive
glomerulonephritis and a true emergency.
Literature supports poor prognosis if creat >4 and
very aggressive therapy indicated even before the
diagnosis is confirmed.
Question 21
Anterior Lenticonus is Pathognomonic for
Which of the Following Kidney Diseases?
A. Alports Syndrome
B. Meckel Gruber Syndrome
C. ARPKD
D Nephronophthisis (NPHP) D. Nephronophthisis (NPHP)
E. ADPKD with Mutations in Polycystin-2
Copyright Harvard Medical School, 2010. All Rights Reserved.
1228
Question 21
Anterior Lenticonus is Pathognomonic for
Which of the Following Kidney Diseases?
A.Alports Syndrome
B.Meckel Gruber Syndrome
C.ARPKD
D Nephronophthisis (NPHP) D.Nephronophthisis (NPHP)
E. ADPKD with Mutations in Polycystin-2
Copyright Harvard Medical School, 2010. All Rights Reserved.
1229
Nephronophthisis
Meckel Gruber: NPHP 6,8 and MKS Genes
ARPKD - Fibrocystin
Morphology of NPHP
Hildebrandt, F. et al. J Am Soc Nephrol 2007;18:1855-1871
Copyright Harvard Medical School, 2010. All Rights Reserved.
1230
Autosomal dominant polycystic kidney
disease: The genes
PKD1
Polycystin 1 Polycystin 1
Huge Integral membrane
protein->400kDa-11 TM
Domains
Cilia, Cell-cell interactions,
Cell Matrix: Regulates Cell
Cycle and Planar Cell
Polarity
Expressed in many
tissues: kidney, brain, tissues: kidney, brain,
heart, bone, muscle
PKD2
Polycystin 2
Interacts with polycystin 1
Calcium Channel
Alport syndrome: The genes and
inheritance
Mutations of type-IV collagen chains
X-linked
X-Chromosome: COL4A5
85% of Alport Syndrome families
New mutations in 10 to 15%
X-linked Alport with leiomyomatosis COL4A5
and COL4A6
Autosomal recessive
Chromosome 2: either COL4A3 or COL4A4
Majority of non-X-linked families
Autosomal dominant
Chromosome 2: COL4A3 and COL4A4?
Rare
Copyright Harvard Medical School, 2010. All Rights Reserved.
1231
Alport syndrome
Progressive glomerulonephritis: X-linked
Severe in males
100% with hematuria from birth; 60-70% with
gross hematuria gross hematuria
Proteinuria develops in all; nephrotic range in
30%
ESRD in 90% by age 40yrs; <31 yrs in 75%
Less symptomatic in females (carriers)
95% with hematuria, may be intermittent; 33%
gross hematuria
Progressive sensorineural hearing loss
Ocular abnormalities
gross hematuria
Proteinuria develops in 2/3
End Stage Renal Failure: 12% by age 40yrs;
30% by age 60yrs
Alport syndrome: Pathology
Normal
Alport syndrome
Copyright Harvard Medical School, 2010. All Rights Reserved.
1232
Alport syndrome: Ocular defects
Ocular abnormalities in ~50%
Anterior lenticonus -a regular
conical protrusion on the anterior
aspect of the lens due to thinning aspect of the lens due to thinning
of the lens capsule
Never at birth; in 22% by 25 yrs
Pathognomonic
Associated with rapid
progression to End Stage
Renal Failure and hearing loss
M l fl k i 37% Macular flecks in 37%
May be only extra renal
manifestation
Recurrent corneal erosions in
23%
In severe AS
Fovea
A 50-year-old woman with recurrent mixed calcium oxalate
and calcium phosphate stones has the following serum
chemistries:
Question 22
Ca Phos Cr UA Mg Na K C02 Cl
10.86 2.74 1.11 6.8 1.9 137 3.8 20 110
Which of the following is least likely?
A Sh i h l i A. She is hypercalcuric
B. Elevated phosphate excretion is a risk factor for her
stones
C. She has a proximal renal tubular acidosis
D. Surgical management is required
E. Bone demineralization has occurred
Copyright Harvard Medical School, 2010. All Rights Reserved.
1233
A 50-year-old woman with recurrent mixed calcium oxalate
and calcium phosphate stones has the following serum
chemistries:
Question 22
Ca Phos Cr UA Mg Na K C02 Cl
10.86 2.74 1.11 6.8 1.9 137 3.8 20 110
Which of the following is least likely?
A Sh i h l i A. She is hypercalcuric
B. Elevated phosphate excretion is a risk factor for her
stones
C. She has a proximal renal tubular acidosis
D. Surgical management is required
E. Bone demineralization has occurred
Copyright Harvard Medical School, 2010. All Rights Reserved.
1234
Answer: B
The answer is B, elevated phosphate excretion.
Phosphate excretion is not a major risk factor for stone
disease.
This patient has primary hyperparathyroidism with its
associated renal t b lar acidosis and bone associated renal tubular acidosis and bone
demineralization.
Disclosures
Dr Denker has reported no financial Dr. Denker has reported no financial
relationships with commercial entities
producing, marketing, re-selling, or
distributing health care goods or services
consumed by, or used on, patients.
Copyright Harvard Medical School, 2010. All Rights Reserved.
1235
Sushrut S. Waikar, MD, MPH
Renal Division
Brigham and Womens Hospital /
Harvard Medical School
Take home messages:
ACUTE KIDNEY INJURY
Disclosures
Investigator-initiated grant support from Pfizer,
Genzyme, Satellite HealthCare, NxStage;
DSMB member for Takeda
Copyright Harvard Medical School, 2010. All Rights Reserved.
1236
New proposed definition of AKI
AKIN proposed criteria
Stage 1
SCr increase 0.3 or 50%
UO: <0.5ml/kg/h for >6h
Stage 2
SCr doubling
UO: <0.5ml/kg/h for >12h
Stage 3
SCr tripling, acute rise 0.5
UO: <0.3ml/kg/h for >24h
or anuria x 12h
Reviewed by Himmelfarb Kidney Int 2007 Molitoris JASN 2007
Rising incidence of AKI
4 to 6-fold increase from 1988 to 2002
Hsu CY KI 2007
Copyright Harvard Medical School, 2010. All Rights Reserved.
1237
Importance of small changes in SCr
70%increased odds of death with an
increase of just 0.3 to 0.4 mg/dL
6.6-fold increased risk of death after 50-99%
increase in SCr following CABG
Mortality after severe AKI-RRT
60% + in recent multicenter epidemiology
studies
Copyright Harvard Medical School, 2010. All Rights Reserved.
1238
Contrast nephropathy
Incidence
~3% risk overall following PCI
25% risk if SCr >2.0 mg/dL
Implications
Increased in-hospital mortality
Increased 5-year mortality
Risk factors
age, higher SCr, CHF, diabetes, contrast
volume, intra-aortic balloon pump
Atheroembolic renal disease
Typical triad:
Presenting event (catheterization, anticoagulation)
Acute or subacute renal failure
Signs of peripheral emboli
Renal presenation: AKI, subacute AKI, proteinuria,
hematuria, renal infarction
Non-renal signs: fever, myalgia, weight loss, neuro
smptoms, livedo reticularis, GI bleeding
Lab findings: hyperamylasemia, elevated CPK,
decreased complements, eosinophilia/eosinophiluria
Copyright Harvard Medical School, 2010. All Rights Reserved.
1239
Secondary causes of minimal
change nephropathy
Drugs
Nonsteroidal anti-inflammatory drugs
Ampicilli n
Gold
Allergens
Hymenoptera stings
Food
Pollen
Poison ivy and poison oak
Immunization
Infections
Viral
Cancer
Hodgkins disease
AKI in cancer
Tumor lysis syndrome
Most commonly in poorly differentiated lymphoma
(e.g., Burkitts)
ALL >AML
Also described in other solid tumors
Multiple myeloma
Cast nephropathy
Hypercalcemia
Hyperuricemia
Contrast nephropathy
Obstruction
Prostate, bladder, uterus, cervix
Absence of hydronephrosis in retroperitoneal
tumors and retroperitoneal fibrosis
Reviewed by Humphreys JASN 2005
Copyright Harvard Medical School, 2010. All Rights Reserved.
1240
AKI in Multiple Myeloma
Cast Nephropathy
Amyloidosis (AL)
LCDD
Interstitial nephritis
Hypercalcemic AKI
Urate nephropathy
Plasma cell infiltration
Hyperviscosity syndrome
Other paraprotein diseases:
Fanconi disease
Goldschmidt, H. et al. Nephrol. Dial. Transplant. 2000
15:301-304; doi:10.1093/ndt/15.3.301
Light
chains
Ca2+
Tubular
Injury
Casts Dehydration
THP
GFR
Plasma Cell
Light Chain Deposition Disease
Aka Monoclonal Ig Deposition Disease
Monoclonal Ig deposition in glomeruli, tubular BM
and arteries (unlike Amyloid, also Congo Red
negative)
Glomeruli: nodular glomerulosclerosis, nephrotic
proteinuria
Tubules: BM thickening, dramatic interstitial
fibrosis
Arterioles: PAS positive deposits, arteriolar
sclerosis
Copyright Harvard Medical School, 2010. All Rights Reserved.
1241
AKI from other nephrotoxins
Aminoglycoside antibiotics
Nearly 50% of patients treated >14d; dose related
Proximal tubular injury
Typically nonoliguric AKI, slow onset
Recovery can take weeks, may be incomplete
Cisplatin
Cumulative dose-related
Tubulointerstitial pattern without heavy proteinuria
25% decline in GFR in 20-30% of patients
Incidence and severity increase with subsequent doses, may be
irreversible
Nonoliguric; hypomagnesemia common
Antivirals
Acyclovir: intratubular precipitation (U/A: needle shaped crystals)
Foscarnet: ATN
Tenofovir: ATN
Aminoglycoside nephrotoxicity
Non-oliguric, slow to recover
Proximal tubule dysfunction
Enzymuria, proteinuria, glucosuria, aminoaciduria
Electrolyte abnormalities: hypoMg, hypoCa, hypoK
Pathology
Proximal tubule: myeloid bodies in lysosomes,
loss of microvilli of luminal surfaces
Glomerular capillary: endothelial cell damage
Copyright Harvard Medical School, 2010. All Rights Reserved.
1242
Acute interstitial nephritis
Antecedent illness or exposure
Fever, rash [can be absent]
Labs:
Eosinophiia/-uria, hematuria, WBC casts, sub-
nephrotic proteinuria
Biopsy: interstitial monocytic infiltrate with edema,
tubular damage, normal glomeruli
Treatment: remove inciting agent, possible role for
steroids if no improvement after removal
Prednisone 1 mg/kg x 1 mo, then taper
Prognosis: up to 40% with permanent reduction in
GFR; interstitial fibrosis on biopsy may predict poor
outcome
Diffusive vs. convective clearance
Diffusion: movement of solutes down concentration
gradient across semipermeable membrane
Convection: solutes passively carried along with
water as it passes across a semipermeable
membrane
Hemodialysis -- Diffusive
CVVH -- Convective
CVVHD -- Diffusive
CVVHDF -- Both
Copyright Harvard Medical School, 2010. All Rights Reserved.
1243
Contraindication to CVVH
Life threatening hyperkalemia
On a per-minute basis, HD is ~10 times
more effective than CVVH in small solute
clearance
Patients with/ at risk for hypotension:
severe hemodynamic instability
hepatic failure
CHF
sepsis or multiple organ failure
Patients at risk of cerebral complications:
hepatic failure, stroke or head trauma
high risk for cerebral edema
Indications for CRRT
Copyright Harvard Medical School, 2010. All Rights Reserved.
1244
Increased metabolic needs
massive burns
sepsis
multiple organ failure
Volume overload
massive volume overload
Patients receiving large amounts of fluids or blood products
When volume management is critical
Indications for CRRT
Non-renal Indications for CRRT
Lactic acidosis
Ongoing production
Crush injury
Myoglobin Removal
Tumor lysis syndrome
Temperature control
Relative hyper or relative hypothermia*
Massive volume overload without ARF
High NH
3
*Kopcke J.Anaesthesiol Reanim.1996;21(6):15962.
Copyright Harvard Medical School, 2010. All Rights Reserved.
1245
Disclosures
Investigator-initiated grant support from Pfizer,
Genzyme, Satellite HealthCare, NxStage;
DSMB member for Takeda
Copyright Harvard Medical School, 2010. All Rights Reserved.
1246
Board Preparation: ESRD
August 2010
Daniel W. Coyne, M.D.
Professor of Medicine
Director of Hemodialysis
Chromalloy American Kidney Center
Washington University School of Medicine, St. Louis, MO
Conflict of interest statement: consultant, advisor, and speaker for
Abbott; Consultant: INEOS, Watson; Consultant and Speaker, AMAG,
Pharmacosmos; PI in multicenter studies designed and funded by
Abbott, AMAG, Amgen, Genentech, Roche, and Watson
Copyright Harvard Medical School, 2010. All Rights Reserved.
1247
Question 1
The dialysis facility you supervise has an older water treatment
system, but has routinely achieved AAMI water quality
standards. Recently, a new water technician has been hired,
and the city has altered it water supply while performing
treatment plant upgrades.
You note that the mean Hgb is significantly lower in the unit, and
several patients have new severe anemia. Which of the
following water quality problems would not account for these
findings?
A. Chloramines
B. Copper
C. Fluoride
D. Nitrates
Copyright Harvard Medical School, 2010. All Rights Reserved.
1248
Water Delivery Circuits
Idealized Water Treatment System
Water Treatment Components
Prefilters
particulate removal
Activated Carbon
organic,chlorine removal
Ion Exchanger/Softener
exchanges Na for Ca/Mg
Reverse Osmosis
organic, non-organic removal
clears bacteria, pyrogens, particulates
Deionizer
H+ or OH- for other ions
UV light
Ultrafilter
Kills, destroys or removes bacteria, endotoxin
Copyright Harvard Medical School, 2010. All Rights Reserved.
1249
Water Treatment
Reverse Osmosis
Water-Contamination Problems
Sign/Symptom Possible contaminant / Cause
Anemia Aluminum, chloramines, copper, zinc
Bone Disease Aluminum, Fluoride
Hemolysis Chloramines, copper, nitrates
Hypertension Calcium, sodium
Hypotension Bacteria, endotoxin, nitrates
Metabolic acidosis Low pH, sulfates
Encephalopathy Aluminum
Nausea/Vomiting Bacteria, endotoxin, low pH, nitrates, sulfates,
zinc
Copyright Harvard Medical School, 2010. All Rights Reserved.
1250
Question 2
Question 2: A 48 y.o. woman on hemodialysis for 2 years transfers
to your facility. During the first 10 minutes of her first treatment,
she develops cough, acute dyspnea, diffuse warmth, and
marked anxiety. She is hypotensive in the 70s and tachycardic.
She denies any history of similar symptoms. Which action
would be most appropriate?
A. Administer NS, give epinephrine 1:1000 SQ, and provide
oxygen NC until transport arrives. Continue dialysis with no
ultrafiltration to remove possible toxin
B. Stop dialysis, and administer NS, consider use of
antihistamines, steroids, and epinephrine.
C. Stop dialysis, administer NS and NC 0
2
. Admit to hospital for
probable pulmonary embolus.
D. Clamp the lines, remove the patient from the machine without
returning the blood. Administer NS, O2, antihistamines,
epinephrine and steroids as needed.
Copyright Harvard Medical School, 2010. All Rights Reserved.
1251
Types of Dialyzers
By Composition
Synthetic - PAN, AN-
69, polysulfone,
PMMA
Substituted Cellulose
cellulose triacetate
Semi-synthetic
Hemophan
Cellulose cuprophan
By permeability
Low Flux
High Flux
Characteristics of Dialyzers
1. KoA: quantitative measure of a dialyzer's
efficiency of clearance
1. defined by membrane porosity and thickness, solute size
and flow rate of blood and dialysate
2. Ultrafiltration coefficient (KUf) - the volume of
fluid (in mL/h) that is transferred across the
membrane per mmHg of pressure gradient
3. Surface Area: Membrane surface within a class
of dialyzers corresponds to the clearance
Copyright Harvard Medical School, 2010. All Rights Reserved.
1252
Definitions: High-flux Hemodialysis
1. Used in most dialysis units
2. Dialyzers with K
uf
> 15 ml/mm Hg/hr
3. Enhanced middle molecule clearance, including beta-2
microglobulin
4. Can have more efficient drug removal
1. e.g. Vancomycin
Dialyzer Reactions
1. TYPE A REACTIONS
2. four of every 100,000 dialysis treatments
3. Begin in the first few minutes of dialysis, immediately after the
return of blood from the dialysis circuit to the patient
4. Occasionally onset is delayed
5. Symptoms
1. itching, burning sensation at the access site, urticaria, flushing, cough,
sneezing, wheezing, abdominal cramps, diarrhea, headache, back and
chest pain, nausea, vomiting, fever, and chills.
2. More severe reactions lead to dyspnea, a sense of impending doom,
and hypotension, potentially resulting in cardiac arrest and death.
6. Patients with an allergic history and eosinophilia appear to be
predisposed to this type of reaction
CAUSES
1. classic cause is ethylene oxide, via preformed IgE antibodies
2. Other leachable agents also can result in this type of reaction
Copyright Harvard Medical School, 2010. All Rights Reserved.
1253
Dialyzer Reactions
1. TYPE A Anaphylactic type
Causes
1. Ethylene Oxide
2. AN69 (formerly negatively charged membranes)
3. Contaminated dialysate
4. Reuse problems
5. Heparin
6. Pre-existing eosinophilia
Treatment
1. Stop dialysis immediately
2. Do NOT return the blood to the patient
3. Use antihistamines, steroids, epinephrine,
bronchodilators, and pressors as needed based on
symptoms
Dialyzer Reactions
TYPE B Reactions
1. More common and less severe than type A reactions
2. Seen in 3 to 5 percent of patients dialyzed with new
cellulosic membranes
3. Much lower incidence with newer synthetic membranes
Symptoms
1. Delayed, begin within the first 15 to 30 minutes of the
dialysis treatment, occasionally later
2. Chest and back pain, dyspnea, nausea, vomiting, and
hypotension
3. Anaphylaxis is extremely rare.
4. There is generally an improvement in symptoms with
continuation of the dialysis treatment.
Cause: Thought to be complement mediated
Copyright Harvard Medical School, 2010. All Rights Reserved.
1254
Q3. A 28yo woman with bipolar disorder is admitted
with a lithium overdose. She took ~90 pills of LiCO3
300 mg. Her initial lithium level is 8.1 meq/L. Her BP is
100/60, R 98, RR 22. Cr is 0.8 mg/dL
Which is the appropriate definitive treatment?
1. Establish high urine flow of 200-300 ml/hr with
NS
2. Initiate HD of 6 hrs; Qb ~400, Qd ~700
3. Initiate CVVHDF with Qb 200, ~35ml/Kg/hr of
dialysate + replacement fluid
4. Initiate PD with exchanges every 2 hours
Copyright Harvard Medical School, 2010. All Rights Reserved.
1255
Common Poisons Amenable to removal by
Hemodialysis (HD) or Hemoperfusion (HP)
Drug General Conc. for intervention Method of Choice
mg/L mcmol/L (HD, HP)
Phenobarbitol 100 430 HP, HD
Glutethimide 40 180 HP
Methaqualone 40 160 HP
Salicylates 800 4.4 mmol.L HD
Theophylline 40 220 HP, HD
Paraquat 0.1 0.4 HP > HD
Methanol 500 16 mmol/L HD
Meprobamate 100 460 HP
From: Handbook of Dialysis, 4
th
edition, Ed: Daugirdas, Blake, and Ing
Modalities Recommended for Intoxications
Bayliss Hemodialysis International 2010; 14:158167
Copyright Harvard Medical School, 2010. All Rights Reserved.
1256
General Rules Concerning Drug Removal
by Dialysis
1. Molecular weight
1. < 500 D well cleared
2. 500 2000 D somewhat cleared
2. Vd > 1 L/kg, very poorly removed
3. High protein binding, poorly cleared
4. Greater lipid solubility, poorly cleared
Q4. A 54yo man on HD for 5 years has undergone a
carpal tunnel release surgery. You prescribed a
narcotic for pain relief. Which of the following is
incorrect?
1. A dose reduction of at least 50% is required for
narcotics in dialysis patients
2. Metabolites of meperidine and propoxyphene
accumulate in renal failure and should be
avoided
3. Evidence of narcotic toxicity in ESRD include
nausea, myoclonic jerking, and delerium
4. Pain after carpal tunnel surgery should be
manageable by acetominephen alone
Copyright Harvard Medical School, 2010. All Rights Reserved.
1257
Question 5
A 48 y.o. man with Type I Diabetes has been on hemodialysis
for 7 years. He has a tunneled catheter, receives 3.5 hr
treatments TIW using a high flux 1.8m
2
polysulfone
dialyzer. A recent single pool Kt/V was 1.27. His typical
intradialytic weight gain is 2.7 kg. His predialysis BP is
142/64 and HR 88. Exam reveals no JVD, clear lungs, an
S4 gallop, and trace ankle edema. He has had recurrent
sudden mid-treatment hypotension despite 2 Kg increase
in his dry weight and a reduction in antihypertensives.
Which of the following actions you recommend to deal with
his intradialytic hypotension?
1. Increase in dry weight
2. Withhold antihypertensives prior to dialysis
3. Initiate sodium modeling with initial Na = 150 meq/L
4. Lower dialysate temperature to 35
o
C
Copyright Harvard Medical School, 2010. All Rights Reserved.
1258
Recurrent Intradialytic Hypotension
Causes
1. Easily corrected
1. Antihypertensives
2. Eating prior to dialysis
2. Dialyzer/Dialysate related
1. Dialyzer Reactions
2. Low sodium dialysate
3. High dialysate magnesium
4. Use of Acetate dialysate
3. Rapid fluid removal/excessive fluid gains
4. Autonomic neuropathy
5. Diminished cardiac reserve
6. Arrhythmias or pericardial effusion with tamponade
Copyright Harvard Medical School, 2010. All Rights Reserved.
1259
Intradialytic Hypotension Therapy
1. Acute Therapy
1. Trendelenburg position
2. Saline
3. Stop Ultrafiltration
4. Debatable: Reduce blood flow rate
2. Chronic Therapy
1. Dry Wgt adjustment
2. Adjust antiHTNives; hold short acting antiHTNives
3. Limit Wgt gains
4. Lengthen time; more treatments
5. Cool Dialysate (35.0 35.5
o
C)
6. Midodrine (10mg PO preHD)
7. ?Sertraline an SSRI (50 to 100 mg QD)
Hemodialysis Induced Acute
Reduction in Cardiac Blood Flow
Dialysis reduced myocardial blood
flow
Reductions of >30% induced regional
wall motion abnormalities
McIntyre CJ ASN 2007
Copyright Harvard Medical School, 2010. All Rights Reserved.
1260
Hemodialysis Induced Cardiac
Stunning
Serial echoes show induction and resolution of
cardiac stunning due to hemodialysis
Note induction of regional wall motion abnormalities
Selby and McIntyre AJ KD 2007
Intradialytic Hypotension
Cool Temp. Dialysis and Midodrine
1. Prospective crossover study of 11
patients with symptomatic intradialytic
hypotension
1. Low temperature dialysis (35.5
o
)
2. Midodrine (10 mg preHD); -1 adrenergic agonist
3. Combined therapy
2. Cool temperature dialysis
1. Significant increases in the lowest SBP during and after HD
2. Midodrine was equally effective
3. No increased benefits with combination therapy
Copyright Harvard Medical School, 2010. All Rights Reserved.
1261
Cool Temp Dialysate Ameliorates
Cardiac Stunning during HD
Selby et al. CJ ASN 2007
Question 6
1. Your dialysis patient has been scheduled for a
MRI with gadolinium contrast to assess a
suspicious renal mass seen on abdominal CT.
Which of the following is FALSE about
Nephrogenic Systemic Fibrosis?
1. No cases of NSF have been reported in patients with GFR
greater than 60ml/min
2. Radiologic societies recommend never using Gd as a
contrast agent for radiography, computed tomography, or
angiography as a method of avoiding Contrast-induced
nephropathy
3. The differences in incidence of NSF with different Gd
products is most likely due to variance in usage
4. Risk has not been shown to be lowered by prednisone
before and after the procedure
Copyright Harvard Medical School, 2010. All Rights Reserved.
1262
Clinical Characteristics of
Nephrogenic Fibrosing Dermopathy
(Nephrogenic Systemic Fibrosis )
AKA: Dialysis-associated or Nephrogenic
systemic fibrosis
Hardened papules/plaques
Symmetrical
Invol ve limbs and trunk
Advancing edge of lesions described as
amoeboid
Gradual restriction of range of motion
Copyright Harvard Medical School, 2010. All Rights Reserved.
1263
NFD-Clinical Characteristics
Skin Findings
Hardened
papules/plaques
Symmetrical
Involve limbs and trunk
Advancing edge of
lesions described as
amoeboid
Gradual restriction of
range of motion
Other common complaints
pruritis, burning pain,
restricted motion
No arthralgias, fever, or
myalgias
Often mistaken for cellulitis
Edema becomes difficult to
appreciate
Scleral Plaques have been
seen
Systemic Involvement of NFD
Ting et al. Reported fibrotic
infiltration of the psoas, rete
testes, and diaphragm in a
patient with NFD (A and B)
Jimenez et al., Cardiac (D) and
Lung involvement (E)
Ting et al. Arch Dermatol 139: 903-906 (2003) Jimenez et al. Arthritis&Rheumatism 50 (8): 2660-2666 (2004)
Copyright Harvard Medical School, 2010. All Rights Reserved.
1264
NSF
Causes, Course, and Prognosis
1. Probably related to low affinity gadolinium complex
use with MRA
1. Three dialysis treatments removes about 95% of gadolinium
administered
2. Progressive induration, restricted range of motion,
flexion contractions
3. Chronology of visceral involvement is not clear
Q7. Urea blood concentrations rebound following
dialysis. Which of the following do not influence
urea rebound?
1. Urea generation rate of the individual
2. Dialysis access recirculation
3. Cardiopulmonary recirculation
4. Urea compartmentalization
Copyright Harvard Medical School, 2010. All Rights Reserved.
1265
Q8. Which of the following is incorrect about
obtaining a post-dialysis Urea for determination of
URR and Kt/V?
1. Obtain it from the arterial limb of the dialysis
tubing
2. Slow the blood pump to 50-100 ml/min for 30-60
seconds prior to sampling
3. Sampling from another access site provides
better URR accuracy than sampling from the
dialysis ciruit
4. It is minimally affected by the ultrafiltration
volume achieved during the dialysis session
Copyright Harvard Medical School, 2010. All Rights Reserved.
1266
Q9. Which of these factors is not a major influence
on the removal of urea during a dialysis session?
1. The generation rate of urea during the dialysis
session
2. The volume of distribution of urea
3. The treatment duration
4. The urea clearance of the dialyzer (K)
Copyright Harvard Medical School, 2010. All Rights Reserved.
1267
Question 10
A 65 y.o. man with a left forearm graft for 17
months is noted to have a reduction in his
recent Kt/V from 1.4 to 1.2. Which of the
following statements about a change in Kt/V in
a hemodialysis patient is false?
1. It could indicate non-compliance with prescribed time
2. It could indicate a change in protein intake
3. It could indicate incorrect needle placement
4. It could indicate dialyzer dysfunction
5. It could indicate improper sample collection
Copyright Harvard Medical School, 2010. All Rights Reserved.
1268
Question 10
A 65 y.o. man with a left forearm graft for 17
months is noted to have a reduction in his
recent Kt/V from 1.4 to 1.2. Which of the
following statements about a change in Kt/V in
a hemodialysis patient is false?
1. It could indicate non-compliance with prescribed time
2. It could indicate a change in protein intake
3. It could indicate incorrect needle placement
4. It could indicate dialyzer dysfunction
5. It could indicate improper sample collection
Copyright Harvard Medical School, 2010. All Rights Reserved.
1269
Question 11
A 69 kg man in HD for 3 yrs has a left arm loop
graft of 13 months duration. He has had no
recent change in his dialysis prescription. The
last monthly Kt/V values are June 1.5; July 1.4;
August 1.3.
Which of these would be the next appropriate
action?
1. Fistulogram
2. Recirculation study
3. Review treatment flow sheets
4. No action; Kt/V remains acceptable
5. Repeat Kt/V
Causes for Fall in Dialysis Adequacy
Hemodialysis Patients
1. Purpose: determine relative frequencies of
causes of decreasing KT/V
2. Kinetic modeling and recirculation tested for 3
consecutive months
3. Baseline Kt/V = average Kt/V in prior 4 mon.
4. Clinically important fall in Kt/V:
1. decline of 0.2 if the baseline Kt/V was > 1.2
2. decline of 0.1 if the baseline Kt/V was <1.2.
Coyne, J ASN, 1996
Copyright Harvard Medical School, 2010. All Rights Reserved.
1270
Causes of Reduced Adequacy of Hemodialysis
No Cause
Found
32%
Reci rcul ati on
25%
Reduced
Bl ood
Processi ng
42%
Di al yzer
Dysfuncti on
1%
Coyne, J ASN, 1996
n=93/375 sessions
3 consecutive months
Outcomes of Patients With Elevated
Recirculation Results
Coyne, J ASN, 1996
n=22
Copyright Harvard Medical School, 2010. All Rights Reserved.
1271
Physical Examination Features of Vascular
Access Suggesting Dysfunction
1. Fistulas
1. Pulsatile vein
2. Multiple collaterals
2. Grafts
1. Pulsatile flow
2. Palpable transition from pulsatile to thrill
3. Auscultatory transition
Question 11
A 69 kg man in HD for 3 yrs has a left arm loop
graft of 13 months duration. He has had no
recent change in his dialysis prescription. The
last monthly Kt/V values are June 1.5; July 1.4;
August 1.3.
Which of these would be the next appropriate
action?
1. Fistulogram
2. Recirculation study
3. Review treatment flow sheets
4. No action; Kt/V remains acceptable
5. Repeat Kt/V
Copyright Harvard Medical School, 2010. All Rights Reserved.
1272
Question 12
A 62 y.o. man has been on hemodialysis for 3 years due to
ESRD from chronic GN. He recently had an episode of
chest pain, elevated troponin of 6.1, and 2 vessel cardiac
disease at catheterization.
Medical therapy is recommended. Lipid testing showed his
total cholesterol was 185 mg/dL, LDL 117 mg/dL, HDL 36
mg/dL, and TG 138. The Cardiologist recommends
atorvastatin at 20 mg per day, but the patient reports
some muscle soreness in the past with simvastatin.
Which of the following statements is NOT correct
Copyright Harvard Medical School, 2010. All Rights Reserved.
1273
Question 12 (continued)
Which of the following statements is NOT correct
1. Statins have not been shown to reduce CV events or
deaths in HD patients
2. Use of statin in this patient is not indicated because of
the lack of proven efficacy in trials and the history of
rhabdomyolysis with statin in this patient
3. Uremic dyslipidemia is characterized by a shift in LDL
size to small dense LDL (dsLDL); statin therapy does
not alter LDL particle size
4. There is a higher incidence of dyslipidemia in PD
patients than HD patients
5. Combined statins and fibrates increase the risk of
myalgia and rhabdomyolysis in HD
Die Deutsche Diabetes Dialyse (4D)
Study population: 1255 type II diabetic
haemodialysis patients
Treatment: Atorvastatin 20 mg vs placebo
LDL-cholesterol difference: 1 mmol/l (40 mg/dl)
Follow-up: 4 years
Primary endpoint: Cardiac death, non-fatal MI
or stroke
Copyright Harvard Medical School, 2010. All Rights Reserved.
1274
4D Study Results
Atorvastatin
(619)
Placebo
(636)
Risk ratio
(95% CI)
Primary endpoint 226 243 0.92
(0.77 to 1.10)
P=0.37
Cardiac death 104 129
Non-fatal MI 69 76
Stroke 53 38
AURORA STUDY
Study population: 2776 patients
50 80 y.o, on ESRD for > 3
months (mean 3.5 yrs)
Treatment: Rosuvastatin vs placebo
LDL-cholesterol difference: 43%
Follow-up: 1296 patients died in follow up;
~1/2 from CV event
Primary endpoint: Cardiac death, non-fatal MI
or non-fatal stroke
Aurora Study, NEJ M, 2009
Copyright Harvard Medical School, 2010. All Rights Reserved.
1275
AURORA: Principal results
Rosuvastatin
(1389)
Placebo
(1384)
Risk ratio
(95% CI)
1
o
Endpoint
(cardiac death, non-
fatal MI or stroke)
396 408 0.96
(0.84 - 1.11)
P=0.59
All-cause
mortality
13.5 per
100/pt-yrs
14.0 per
100/pt-yrs
0.96;
(0.86 -1.07)
P = 0.51
Aurora Study, NEJ M, 2009
Question 12 (continued)
Which of the following statements is NOT correct
1. Statins have not been shown to reduce CV events or
deaths in HD patients
2. Use of statin in this patient is not indicated because of
the lack of proven efficacy in trials and the history of
rhabdomyolysis with statin in this patient
3. Uremic dyslipidemia is characterized by a shift in LDL
size to small dense LDL (dsLDL); statin therapy does
not alter LDL particle size
4. There is a higher incidence of dyslipidemia in PD
patients than HD patients
5. Combined statins and fibrates increase the risk of
myalgia and rhabdomyolysis in HD
Copyright Harvard Medical School, 2010. All Rights Reserved.
1276
Question 13
A. Which of the following bone result is most likely
with the this laboratory finding:
25D <10 ng/ml for the past 2 years
1. Osteitis Fibrosa
2. Osteomalacia
3. Adynamic Bone Disease
Copyright Harvard Medical School, 2010. All Rights Reserved.
1277
Question 14
A. Which of the following bone results is most
likely with the this laboratory finding:
PTH < 100 ng/ml for 1 year
1. Osteitis Fibrosa
2. Osteomalacia
3. Adynamic Bone Disease
Copyright Harvard Medical School, 2010. All Rights Reserved.
1278
UpToDate 2005
Adynamic Bone
Osteomalacia
Copyright Harvard Medical School, 2010. All Rights Reserved.
1279
Osteomalacia vs Normal Bone biopsy
http://www.endotext.org/parathyroid/parathyroid8/parathyroid8.html
The 3 figures on the left show
defective mineralization, irregularity
of mineralization fronts (Figure 5A)
high number of osteoid lamellae
(Figure 5C)
broad single tetracycline fluorescent
labels or no label at all, in contrast to
the normal double tetracycline
fluorescent labels (Figures 5E and
5F).
Osteitis Fibrosa
High Bone Turnover
Low Power showing
fibrosis and
disordered bone
formation and
mineralization
Fibrosis and marked
osteoclastic resorption
Fibrosis and
intratrabecular tunnels http://courses.washington.edu/bonephys/opgallery.html
Copyright Harvard Medical School, 2010. All Rights Reserved.
1280
Question 15
A 63-year-old diabetic man has been on hemodialysis for 5
years. He recently transferred to your unit, and you
review his labs (below) and find they are similar to his
previous facilities labs.
He denies bone pain, fractures, heart disease, or problems
with phosphorus control. He dialyzes on a 2.5 meq/L
Calcium bath, and recent Kt/V was 1.55.
Medications include calcium acetate 667 mg, two tabs with
each meal, and cholecalciferol 800 IU per day.
Laboratory studies: Serum: Sodium 140 meq/L, K+ 4.9
meq/L, Chloride 98 meq/L, HCO3 19 meq/L, Calcium 9.7
mg/dL, Phosphorus 4.8 mg/dL, Intact PTH 45 pg/mL,
25(OH)D 47 ng/ml. Which of the following statements
represents appropriate management of this patient?.
Question 15 (continued)
Which of the following statements represents
appropriate management of this patient?
1. The patient should continue on his present
medications. No changes are needed.
2. The patient should stop his cholecalciferol; Calcium
acetate should be stopped in 3 months if his PTH is <
100 pg/ml.
3. The patient should be started on a low dose of an
active vitamin D (such as paricalcitol) to reduce the
risk of a cardiovascular event or death. Cholecalciferol
can be stopped.
4. The patient should have calcium acetate stopped and
sevelamer 1600 mg TID started.
5. Both answer C and D.
Copyright Harvard Medical School, 2010. All Rights Reserved.
1281
Guideline 13C. Adynamic Bone Disease
13C.1 Adynamic bone disease in Stage 5 CKD (as
determined either by bone biopsy or intact PTH
<100 pg/mL [11.0 pmol/L]) should be treated by
allowing plasma levels of intact PTH to rise in order
to increase bone turnover. (Opinion)
13C.1a This can be accomplished by decreasing doses
of calcium-based phosphate binders and vitamin D
or eliminating such therapy. (Opinion)
Copyright Harvard Medical School, 2010. All Rights Reserved.
1282
Phosphate
retention
Low levels of
calcitriol
Hypocalcemia
Hyperparathyroidism
Chronic Renal Failure
1) Avoid Calcium-based Binders
(Use Sevelamer)
2) Avoid Vitamin D
3)? Use 1.0 to 2.0 Ca bath?
4)? Keep PO
4
near 5.5?
*Avoid Vitamin D
Supplementation
?Use Low Ca++
Bath (2.0 or 1.75)
*Use
Sevelamer
Low PTH and Adynamic Bone Disease
(iPTH <100; biPTH < 50-60)
Management of Low PTH in Dialysis
Patients: K/DOQI
Effect of Chronically low PTH: Adynamic Bone Disease
Management
1. Avoid Calcium-based Binders
1. Use Sevelamer (Renagel) or Lanthanum Carbonate (Fosrenol)
2. Use 2.5 meq/L Calcium bath or PD fluid
3. Consider short term use of a lower calcium bath in
hemodialysis (e.g. 2.0 meq/L)
4. Avoid the use of Active Vitamin D
5. Supplement low serum phosphorus
6. ?Allow predialysis Phosphorus to be closer to 5.5
Copyright Harvard Medical School, 2010. All Rights Reserved.
1283
Evidence ALERT!
Use your Noggin!
1. We have no trial evidence that raising Bone formation
rate in the previous patient improves outcomes (lowers
fractures, CV events, mortality)
2. Virtually all PTH, Calcium, and Phosphate Guidelines are
NOT based on Randomizied trials showing better
outcomes with the recommended therapy.
3. Consider the status and the goals of the patient when
changing therapy in Phosphate, Ca and PTH control
1. Does a patient with lung cancer need a higher PTH?
2. Does a demented Pt need better (and more expensive) phosphate control?
Question 16
A 38 y.o. woman has been on hemodialysis for 5
months. Despite epoetin 10,000 units 3x/week ,
her Hgb remains less than 9 g/dL.
1. Which of the following most likely explains her
persistent anemia?
1. Anti-erythropoietin antibodies
2. Iron deficiency
3. Inadequate dialysis
4. Heavy menses
Copyright Harvard Medical School, 2010. All Rights Reserved.
1284
Question 16
A 38 y.o. woman has been on hemodialysis for 5
months. Despite epoetin 10,000 units 3x/week ,
her Hgb remains less than 9 g/dL.
1. Which of the following most likely explains her
persistent anemia?
1. Anti-erythropoietin antibodies
2. Iron deficiency
3. Inadequate dialysis
4. Heavy menses
Copyright Harvard Medical School, 2010. All Rights Reserved.
1285
Question 17
A 47 y.o. man has been a on dialysis for 2.7 yrs. His Hgb is
generally 11-12 g/dl with IV Epoetin doses of less than
5000u per HD. He recently spent four months in France
where his management was similar except he received
his Epoetin SQ. He was well for 2 months after returning,
but in the last month his Hgb has fallen steadily to 7.5
g/dl despite 2 increases in Epo (now 12,000u/HD). His
ferritin is 685 and TSAT 46%. Stool guaiac is repeatedly
negative, and no other source of bleeding is identified. A
reticulocyte count is 0.1%, while WBC and platelets are
normal.
Which is the likely diagnosis?
1. Iron Deficiency Anemia
2. Megaloblastic Anemia
3. Pure Red Cell Aplasia
4. Retroperitoneal bleeding
Copyright Harvard Medical School, 2010. All Rights Reserved.
1286
Pure Red Cell Aplasia (PRCA)
1. Clinical presents as
1. rapidly declining Hbg (~ 1g/dL per 7-10 days)
2. Very low Reticulocyte counts
3. Lack of Iron requirement leads to increase in TSAT
4. Bone Marrow confirms absence of Red Cell Precursors
5. Anti-Epo antibodies detected
2. ~160 cases in Europe around 2000
1. All in Eprex-treated dialysis patients
2. All were using SQ Eprex
Pure Red Cell Aplasia (PRCA)
1. Eprex is Epoetin-alpha manufactured by J&J
1. Manufacturing methods may have changed in 1998
2. Anti-epo antibodies will cross react against all other
erythropoietic proteins
3. Treatment:
1. Stop All Epoetin and Darbepoetin injections
2. Immunosuppressi ve therapies have been variably effective (but are
superior to no therapy)
3. Transfuse as necessary
4. Renal transplantation may well correct condition
Copyright Harvard Medical School, 2010. All Rights Reserved.
1287
Question 18
Which of the following hemodialysis patients
should be dialyzed in an isolation room in an
outpatient dialysis center?
1. HIV positive, non-compliant with HAART therapy
2. Hepatitis C positive, CAH on liver biopsy
3. Hepatitis B surface Antibody positive
4. Hepatitis B surface Antigen positive
5. Viral Meningitis with West Nile Virus
Question 18
1. Which of the following hemodialysis patients
should be dialyzed in an isolation room in an
outpatient dialysis center?
1. HIV positive, non-compliant with HAART therapy
2. Hepatitis C positive, CAH on liver biopsy
3. Hepatitis B surface Antibody positive
4. Hepatitis B surface Antigen positive
5. Viral Meningitis with West Nile Virus
Copyright Harvard Medical School, 2010. All Rights Reserved.
1288
Question 19
1. Which water treatment component is
specifically intended to remove calcium?
1. Carbon tank
2. Water Softener
3. Reverse Osmosis
4. Deionizer
5. 0.2 u filter
Copyright Harvard Medical School, 2010. All Rights Reserved.
1289
Water Delivery Circuits
Idealized Water Treatment System
Copyright Harvard Medical School, 2010. All Rights Reserved.
1290
Water Treatment Components
Component
Prefilters
particulate removal
Activated Carbon
organic,chlorine removal
Ion Exchanger/Softener
exchanges Na for Ca/Mg
Question 20
Which of the following is true regarding
intradialytic hypotension?
1. Use of bicarbonate based dialysate and volumetric
ultrafiltration control has eliminated dialysis induced
hypotension
2. On-line monitoring of Hematocrit is the best method
for prevention of hypotension
3. Minimizing the rate of ultrafiltration is the best method
for prevention of hypotension
4. Sodium modeling enhances fluid removal but does not
reduce the incidence of intradialytic hypotension
Copyright Harvard Medical School, 2010. All Rights Reserved.
1291
Question 20
1. Which of the following is true regarding
intradialytic hypotension?
1. Use of bicarbonate based dialysate and volumetric
ultrafiltration control has eliminated dialysis induced
hypotension
2. On-line monitoring of Hematocrit is the best method
for prevention of hypotension
3. Minimizing the rate of ultrafiltration is the best method
for prevention of hypotension
4. Sodium modeling enhances fluid removal but does not
reduce the incidence of intradialytic hypotension
Copyright Harvard Medical School, 2010. All Rights Reserved.
1292
Question 21
Which of the following is not a cause of
intradialytic hypotension?
1. Dilated cardiomyopathy
2. Cardiac diastolic dysfunction
3. Clonidine withdrawal
4. Increased caloric intake without adjustment in dry
weight
Question 21
1. Which of the following is not a cause of
intradialytic hypotension?
1. Dilated cardiomyopathy
2. Cardiac diastolic dysfunction
3. Clonidine withdrawal
4. Increased caloric intake without adjustment in dry
weight
Copyright Harvard Medical School, 2010. All Rights Reserved.
1293
Q22. A 50 yo diabetic woman has been on CAPD for 2 years (4xch,
2L, 1.5% alt with 2.5%).
She has had 2 peritonitis episodes, the last 4 months ago. She has
had progressive problems achieving her dry weight. She was
hospitalized for fluid overload 1 month ago. A new PET test
shows:
Initial PET Now
Creatinine D/P 4hr 0.71 0.68
Glucose D/P 4hr 0.39 0.40
Drain volume (ml) 2250 2200
Her problem is most likely:
1. Idiopathic peritoneal fibrosis
2. Loss of residual renal function
3. Loss of peritoneal UF d/t an increase in solute transport rate
4. Loss of peritoneal UF d/t an decrease in solute transport rate
5. Temporary loss of UF d/t recent peritonitis
Copyright Harvard Medical School, 2010. All Rights Reserved.
1294
Question 23
A 28 y.o. woman with ESRD from lupus is on
peritoneal dialysis. She is using an oral
contraceptive and reports during her menstrual
periods her PD effluent is bloody.
Which of the following statements is true?
1. Heparin should be added to the exchange fluid as long
as the effluent is bloody
2. Heparin should be added to the exchange fluid unless
she is on warfarin, as heparin will increase systemic
bleeding risks
3. Heparin should not be added to the instilled
exchanges as this will increase bleeding
4. Such bleeding episodes are only seen in women on
oral contraceptives: the OCP should be stopped
Copyright Harvard Medical School, 2010. All Rights Reserved.
1295
Question 23
A 28 y.o. woman with ESRD from lupus is on
peritoneal dialysis. She is using an oral
contraceptive and reports during her menstrual
periods her PD effluent is bloody.
Which of the following statements is true?
1. Heparin should be added to the exchange fluid as long
as the effluent is bloody
2. Heparin should be added to the exchange fluid unless
she is on warfarin, as heparin will increase systemic
bleeding risks
3. Heparin should not be added to the instilled
exchanges as this will increase bleeding
4. Such bleeding episodes are only seen in women on
oral contraceptives: the OCP should be stopped
Copyright Harvard Medical School, 2010. All Rights Reserved.
1296
Question 24
A 58 y.o. hemodialysis patient has lost 8% of his
body weight over the past year. He has
undergone colonoscopy and other tests which
have failed to reveal any cancer. His recent
blood work indicates his Kt/V urea is 1.25.
Which statement best describes Kt/V urea.
1. The Kt/V urea is satisfactory based on KDOQI
recommendations, so dialysis is adequate
2. The Kt/V urea may be spuriously elevated because
protein catabolism results in non-urea end products
3. The Kt/V may be spuriously elevated because of
increased protein intake by the patient
4. The Kt/V may be spuriously elevated because of low V
Question 24
A 58 y.o. hemodialysis patient has lost 8% of his
body weight over the past year. He has
undergone colonoscopy and other tests which
have failed to reveal any cancer. His recent
blood work indicates his Kt/V urea is 1.25.
1. Which statement best describes Kt/V urea.
1. The Kt/V urea is satisfactory based on KDOQI
recommendations, so dialysis is adequate
2. The Kt/V urea may be spuriously elevated because
protein catabolism results in non-urea end products
3. The Kt/V may be spuriously elevated because of
increased protein intake by the patient
4. The Kt/V may be spuriously elevated because of low V
Copyright Harvard Medical School, 2010. All Rights Reserved.
1297
Question 25
A 56 y.o. Harvard educated attorney has
progressive renal failure. He expresses the view
that residual renal function (RRF) is essential
for prolonged survival on dialysis. He asks you
for your opinion.
Which of the following statements is true?
1. Decline in RRF is slower with CAPD than with HD
2. Loop diuretics maintain RRF in CAPD patients
3. In PD patients, survival correlates equally well with
RRF and PDF clearance
4. A Harvard educated attorney would never ask for
another persons opinion
Copyright Harvard Medical School, 2010. All Rights Reserved.
1298
Contribution of RRF to Clearance of Small vs Large Solutes
Bammens Bammens et al, 2003 et al, 2003
RRF: Clearance of Larger MW Uremic Toxins
Copyright Harvard Medical School, 2010. All Rights Reserved.
1299
Distribution of PET results in Adults (%)
2003 Annual Report, CMS
4 hour D/P Ratio
Low = 0.35-0.50
Low Av = 0.50-0.65
High Av = 0.65-0.81
High = 0.81-1.03
0
10
20
30
40
50
Low Low Av High Av High
7
29
48
16
Guideline 15: Weekly Dose of CAPD
(Evidence)
1. For CAPD, the delivered PD dose should be a
total Kt/Vurea of at least 2.0 per week and a
total creatinine clearance (CCr) of at least 60
L/wk/1.73 m2 for high and high-average
transporters, and 50 L/wk/1.73 m2 in low and
low-average transporters.
K/DOQI 2000
Copyright Harvard Medical School, 2010. All Rights Reserved.
1300
ADEMEX: Randomized trial of more Clearance in CAPD
965 pts randomized to CrCl of 60L/wk/1.73 m2 or to four 2L exchanges daily.
Paniagua et
al. 13 (5):
1307. (2002)
JASN
ADEMEX Study: Summary of Results
1. increasing the peritoneal dialysis
prescription to DOQI Kt/V urea had no effect
on survival in this cohort
2. this finding persisted regardless of age, sex,
nutritional or diabetic status
3. residual renal function, however, was an
important predictor of survival
KDOQI guidelines have repeatedly exceeded available evidence, and
recommended therapies that are not beneficial or even harmful to patients.
Always consider whether your pt. truly needs what KDOQI recommends, and
what harm, benefit, or cost you are placing on the pt. by following the
Guideline.
Copyright Harvard Medical School, 2010. All Rights Reserved.
1301
Question 26
A 63 y.o. diabetic man initiates hemodialysis after presenting to
the hospital with fluid overload, severe hypertension and
new ESRD. On exam in the outpatient unit, his BP is 160/105,
P84, lungs clear, and Cor RRR with an S4 gallop, no edema.
He is taking Norvasc 10 mg and Lisinopril 10 mg daily.
Which management of his hypertension is the best option?
1. Increase the lisinopril to 40 mg, add beta blocker in 2
weeks if still hypertensive
2. Progressively lower dry weight over 3 to 4 weeks until
predialysis BP < 140/90
3. Add metoprolol 25 mg BID and titrate to P <60 or BP <
140/90
4. Start clonidine 0.1 mg TID; hold the immediate predialysis
dose
Question 26
A 63 y.o. diabetic man initiates hemodialysis after presenting
to the hospital with fluid overload, severe hypertension
and new ESRD. On exam in the outpatient unit, his BP is
160/105, P84, lungs clear, and Cor RRR with an S4 gallop,
no edema. He is taking Norvasc 10 mg and Lisinopril 10
mg daily.
Which management of his hypertension is the best option?
1. Increase the lisinopril to 40 mg, add beta blocker in 2
weeks if still hypertensive
2. Progressively lower dry weight over 3 to 4 weeks until
predialysis BP < 140/90
3. Add metoprolol 25 mg BID and titrate to P <60 or BP <
140/90
4. Start clonidine 0.1 mg TID; hold the immediate
predialysis dose
Copyright Harvard Medical School, 2010. All Rights Reserved.
1302
Question 27
A 42 y.o. man presents with a creatinine of 17.0 mg/dL, BUN
180 mg/dL, nausea, malaise, and a 3 component
pericardial friction rub. His blood pressure is 180/110. US
reveal small kidneys and he is initiated on dialysis. He
has stable blood pressure despite 2 liter ultrafiltration,
and has an uneventful first treatment, with a URR of 69%.
His post dialysis blood pressure is 150/95. Later that day
is complains of headaches, nausea and vomiting, and
then has a grand mal seizure.
Which of the following most likely explains his deterioration?
1. Cardiac tamponade
2. Stroke (ischemic or hemorrhagic)
3. Cerebral hypoperfusion due to sudden reduction in
blood pressure.
4. Dysequilibium syndrome
Copyright Harvard Medical School, 2010. All Rights Reserved.
1303
Dialysis Dysequilibrium Syndrome
1. Characterized by neurologic symptoms of varying
severity that are thought to be due primarily to cerebral
edema
2. Risk factors
1. New patients just being started on hemodial ysis
1. particularl y if the BUN >175 mg/dL
2. Severe metabolic acidosis
3. Older age or pediatric patients
4. Other CNS disease such seizure disorder
3. Symptoms are caused by H2O movement into brain,
leading to cerebral edema. Two theories proposed:
1. A reverse osmotic shift induced by urea removal
2. A fall in cerebral intracellular pH
Mailloux, L. UpToDate, 2009
Dialysis Dysequilibrium Syndrome
1. Classic acute symptoms develop during or immediately
after hemodialysis
1. Headache, nausea, disorientation, restlessness, blurred vision, and
asterixis
2. May progress to confusion, seizures, coma, and even death
3. Many milder S&S associated with dial ysis such as muscle
cramps, anorexia, and dizziness developing near the end of a
dial ysis treatment are also part of this syndrome
2. Differential diagnosis includes: uremia itself, subdural
hematoma, cerebral infarction, intracerebral hemorrhage,
meningitis, hyponatremia, hypoglycemia, and drug-
induced encephalopathy
Mailloux, L. UpToDate, 2009
Copyright Harvard Medical School, 2010. All Rights Reserved.
1304
Dialysis Dysequilibrium Syndrome
Prevention
1. Slow reduction in BUN
1. two hours of dialysis at a relatively low blood flow rate of 150
to 250 mL/min with a small surface area dialyzer (0.9 to 1.2
m2), and concurrent rather than countercurrent blood and
dialysate flow
2. If fluid overload is present, use sequentially ultrafiltration
with short hemodialysis treatment
3. DDS has not been reported with CAPD
Treatment of DDS
1. Supportive; stop HD is seizures or obtundation occurs.
Mannitol can rapidly raise plasma osmolality and improve
symptoms
Mailloux, L. UpToDate, 2009
Question 28
Which of the following is not a hemodynamic
effect of placement of an arteriovenous fistula
1. Diastolic dysfunction
2. Increased pulmonary bed blood pressure
3. Increased end left ventricular diastolic diameter
4. Increased cardiac output
5. Decreased BNP levels
Copyright Harvard Medical School, 2010. All Rights Reserved.
1305
Effects of AV Fistula Creation
1. Increase in cardiac output
2. Increase in left ventricular end-diastolic diameter
3. Left ventricular (LV) diastolic dysfunction in association
with a restrictive filling pattern
4. Elevation in ANP release, correlated with volume loading
5. Increased BNP release, stimulated by LV diastolic
dysfunction
6. Pulmonary hypertension also appears immediately after
the creation of an arteriovenous access
1. Affected patients have significantly higher cardiac output.
2. Associated with a statistically significant survi val
disadvantage
3. It tends to regress after AV access closure.
Beathard G. UpToDate, 2009
Question 28
Which of the following is not a hemodynamic
effect of placement of an arteriovenous fistula
1. Diastolic dysfunction
2. Increased pulmonary bed blood pressure
3. Increased end left ventricular diastolic diameter
4. Increased cardiac output
5. Decreased BNP levels
Copyright Harvard Medical School, 2010. All Rights Reserved.
1306
Question 29
Which of the following is FALSE about ultrapure
dialysate?
1. It is the quality standard that is required for US
hemodialysis centers
2. It may reduce markers of inflammation, such as CRP
3. It may reduce epoetin requirements
4. Regular disinfection of the water system is required to
maintain ultrapure dialysate
Copyright Harvard Medical School, 2010. All Rights Reserved.
1307
Ultrapure Dialysate
1. Use of dialysate of much higher microbiological purity than is
required for routine hemodialysis
2. Commonly defined as having a bacterial count less than 100
CFU/L and an endotoxin content less than 0.03 EU/mL
3. Use of ultrapure dialysate is required for on-line
hemodiafiltration and hemofiltration, where dialysate is used to
prepare the substitution solution
4. Some studies show use of Ultrapure dialysate leads to
1. Reduction in inflammatory markers (e.g. CRP)
2. Anemia correction,
3. Reduced beta2-microglobulin
4. Improved nutritional status
Richard Ward, UpToDate, 2009
Production of Ultrapure Dialysate
1. Attention to the design of the treated water distribution
system to avoid areas of stagnation
2. Attention to the preparation of concentrates, with
particular emphasis on bicarbonate concentrate, which
provides a good medium for bacterial proliferation
3. Frequent disinfection of the treated water distribution
system and dialysis machines on a schedule designed to
prevent bacterial proliferation and biofilm formation in
the fluid pathways
Richard Ward, UpToDate, 2009
Copyright Harvard Medical School, 2010. All Rights Reserved.
1308
Question 29
Which of the following is FALSE about ultrapure
dialysate?
1. It is the quality standard that is required for US
hemodialysis centers
2. It may reduce markers of inflammation, such as CRP
3. It may reduce epoetin requirements
4. Regular disinfection of the water system is required to
maintain ultrapure dialysate
Copyright Harvard Medical School, 2010. All Rights Reserved.
1309
Conflict of interest statement: consultant, advisor, and speaker for
Abbott; Consultant: INEOS, Watson; Consultant and Speaker, AMAG,
Pharmacosmos; PI in multicenter studies designed and funded by
Abbott, AMAG, Amgen, Genentech, Roche, and Watson
Copyright Harvard Medical School, 2010. All Rights Reserved.
1310
Dialysis: An Overview for the
Boards
J . Kevin Tucker, M.D.
Brigham and Womens Hospital
Massachusetts General Hospital
Harvard Vanguard Medical
Associates
Conflicts of Interest
Consultant
Baxter Healthcare
Genzyme
Copyright Harvard Medical School, 2010. All Rights Reserved.
1311
Outline
Clinical Vignettes with Discussion
Review of Current Guidelines and
Relevant Data:
Survival/Dialysis Adequacy
Complications (Acute)
Bone Disease
Cardiovascular Disease
Outline
Clinical Vignette with Discussion
Review of Current Guidelines and
Relevant Data:
Survival/Dialysis Adequacy
Complications (Acute)
Bone Disease
Cardiovascular Disease
(Anemia)
Copyright Harvard Medical School, 2010. All Rights Reserved.
1312
Case 1
67 year old woman with ERSD on HD for 3
years admitted to the hospital with
progressive dyspnea and chest pressure of 3-
4 weeks duration. She reported low grade
fever at home 2 days prior to admission.
There was no associated chill, cough, night
sweats, or weight loss.
Case 1
Past Medical History
ESRD Secondary to HUS/TTP on HD x 3
years
History of 3 clotted AVF/AVG
History of tunneled catheter infection
Currently dialyzing via a tunneled catheter
Optiflux 160, 3 hours, Qb 350, Qd 500
URR 77%
Good residual renal function; weight close to dry
weight prior to dialysis
Copyright Harvard Medical School, 2010. All Rights Reserved.
1313
Case 1
Past Medical History
Non-Hodgkins lymphoma
Cardiomyopathy secondary to
doxorubicin
Hypertension
Breast cancer: Active and metastatic
Physical Exam
Notable for BP: 115/75 mm Hg HR-80 T-
98.0F RR-20 99% RA; no pulsus, no rub,
bibasilar rales, trace edema
Data
Labs: Electrolytes normal, Bun 30, Creat 3.7, Ca
9.1, Phos 2.1,
CPK 16, trop I 0.07
WBC 8.4, Hct 43.1, Plt 219
Alb 2.8, PTH 241, Tsat >30%, Ferritin 2797
Copyright Harvard Medical School, 2010. All Rights Reserved.
1314
Questions:
1. What is the cause of her pericardial
effusion?
2. What is the next step of
management?
Copyright Harvard Medical School, 2010. All Rights Reserved.
1315
Hospital Course:
On 1st day, she received dialysis with dry
weight challenge, tolerated well.
On 2
nd
day, she became hypotensive,
responded to IV fluid; repeat echo
revealed increased pericardial effusion,
and mild diastolic inversion of right
ventricular wall, no tamponade.
She then underwent semi-emergent
pericardiacentesis; 300 mL hemorrhagic
fluid drained; a pigtail catheter placed
Pericardial fluid analysis: cytology:
negative for malignant cells; Culture:
negative; adenosine deaminase 8.8
Survival in ESRD
USRDS 2005
E
x
p
e
c
t
e
d

Y
e
a
r
s

R
e
m
a
i
n
i
n
g
*
US
Population
0
35
10
20
30
Colon
Ca
ESRD Lung
Ca
Prostate
Ca
21.6
13
8
4.5
2.5
*Based on adult, age 59 years
Ca=Cancer
ESRD patients lose 79.2% of years
that the general population is
expected to live.
Copyright Harvard Medical School, 2010. All Rights Reserved.
1316
48%
6%
15%
4%
20%
7%
Cause of Death in ESRD
Source: USRDS
Unknown
Other Known
Malignancy
Infection
Cardiovascular
dysfunction
Cardiac
disease
Mortality Risk Factors Not Related to
Dialysis
Comorbid Conditions: DM (42%)
10y survival lower in DM (4 vs 11-14%)
CAD, LVH
Enhanced CAD with Dialysis due to HTN, Metabolic
abnormalities (Ca, Phos), LVH with anemia,
hyperlipidemia
Underlying renal disease, age, country, race,
nutrition, inflammation, anemia, late referral,
fractures, dialysis adequacy
Copyright Harvard Medical School, 2010. All Rights Reserved.
1317
NCDS and Kt/V
Landmark 1981 study showing that time-
averaged urea and PCR were important
determinants of morbidity and mortality.
Urea reflects Kt/V and dietary protein intake
Kt/V acceptableat 1.0 - Increased Mortality
Kt/V is another marker of dialysis
adequacy first described from NCDS data
Calculating Kt/V
Computer Models; pre- and post-HD weight,
UF volume, Hct, pre- and post-HD BUN
Statistical Models:
URR=(1-[post BUN/pre-BUN])
More complex models add urea generation during
dialysis and changes in body water
Single pool vs. double pool
Single pool, nonequilibrated
Kt/V =ln(R-0.03+[4-3.5R) x UF/W
Copyright Harvard Medical School, 2010. All Rights Reserved.
1318
Nomogram
Data from
Daugirdas, J T, J
Am Soc Nephrol
1993; 4:1205.
HEMO Study
Prospective; 1846 pts - Primary Outcome-
Death
Secondary Outcomes-hospitalizations (non-
access), first hospitalization for cardiac
problem or death, infectious cause or death,
and the first decline of >15% in albumin or
death
Results:
Good separation 1.32 (66.3) v 1.71 (75%)
No difference in death or secondary outcomes
No difference high vs low flux dialyzers (CV)
Copyright Harvard Medical School, 2010. All Rights Reserved.
1319
Survival Curves for High and Standard
Dialysis Doses
Eknoyan, G, Beck, GJ , Cheung, AK, et al. N Engl J Med 2002; 347:2010.
Improved Survival with Longer
Dialysis Session
Effect of time independent of conventional
markers is unclear:
DOPPS (Dialysis Outcomes and Practice Patterns
Study) of 22,000 HD patients found >240 min. had
lower mortality (RR-0.81). 7% lower risk of death
with each 30min (Saran et al., KI 2006)
Similar findings in Australia (Marshall et al., KI 2006)
?Better Control of Fluids and HTN
So, independent of URR is longer time beneficial?
(nocturnal HD, Tassin France)
Copyright Harvard Medical School, 2010. All Rights Reserved.
1320
K/DOQI-Adequacy
K/DOQI: If GFR <2 mL/min;
Single Pool Kt/V of 1.2 or 65% URR is minimally
adequate dose
Target recommended dose 1.4 or 70% URR
If Below Target:
Assess Fistula (recirculation)
Treatment time
Method of obtaining BUN
Patient specific variables: Qb, hypotension,etc
Our Patient: Increased risk of death in some studies
with high URR (>75%); low body volume and poor nutrition
(Chertowet al., KI 1999)
Outline
Clinical Vignette with Discussion
Review of Current Guidelines and
Relevant Data:
Survival/Dialysis Adequacy
Complications (Acute)
Bone Disease
Cardiovascular Disease
(Anemia)
Copyright Harvard Medical School, 2010. All Rights Reserved.
1321
48 year old woman on vacation on Cape Cod
receives routine dialysis at a local unit. She has
ESRD from PCKD on HD for 2 years. A polysulfone
dialyzer is used and ~5 min into the session she c/o
severe dyspnea, chest tightness and back pain.
Pulse increased from 70 to 125 and BP falls from
130/70 to 80/50 mm Hg. She becomes tachypneic
with diffuse wheezing. Dialysis is discontinued
without returning blood from the circuit.
Stat Labs: Hb: 9.3 g/dL
WBC: 1100/cu mm 90% lympho
Lytes: Normal
EKG: Sinus tach; no ischemia
CXR: Clear
The patient is given NS, O
2
and symptoms resolve
over next half-hour.
Case 2
Which of the following is the most likely explanation for
this patients symptoms:
A. Dialysis associated autonomic dysfunction
B. Air Embolus
C. Pulmonary Embolus
D. First Use Syndrome
E. Pyrogen Reaction
Copyright Harvard Medical School, 2010. All Rights Reserved.
1322
Correct Answer - D; First Use Syndrome
Air Embolus - to cerebral circulation if upright (seizures,
syncope); Supine - cardiac/pulmonary with dyspnea. For
both place patient left lateral and head down.
Pulmonary Embolus unlikely in this setting.
Pyrogen reaction is usually from endotoxin in water or
dialyzer and associated with fever, chills, n/v and
hypotension with negative blood cultures.
Complications
Hypotension
Hypertension
Dialyzer reactions
Infection and
pyrogen reactions
Air embolism
Vascular access
bleeding
Seizures
Hemolysis
Arrhythmias
Muscle cramping
Copyright Harvard Medical School, 2010. All Rights Reserved.
1323
1. Hypotension During
Hemodialysis (IDH)
Causes of hypotension in HD
patients
Medications
Autonomic
dysfunction
Sepsis
Endogenous
vasodilators
Underestimation of
dry weight
Cardiac causes
Reduced diastolic
filling
Atrial fibrillation
Tachycardia
Left ventricular
hypertrophy
Ischemia
Pericardial effusion
Copyright Harvard Medical School, 2010. All Rights Reserved.
1324
Interventions that may be
Effective
Decrease fluid gains (salt intake)
Increase treatment time
Increase dialysate sodium
Raise dry weight
Lower dialysate temperature
Administer midodrine
Stop eating during dialysis
Supplement L-carnitine
Interventions that are Probably
Not Effective
Oxygen
Isolated ultrafiltration
Fludrocortisone
Decrease in dialysis machine blood flow
rate
Hct monitoring
Copyright Harvard Medical School, 2010. All Rights Reserved.
1325
Midodrine
Oral -1 adrenergic agonist
Highly selective (no -1 activity)
Peak level of active metabolite achieved
90 minutes after administration
Among patients with autonomic
dysfunction, toxicities include urinary
retention and supine hypertension
L-Carnitine Supplementation
Carnitine deficiency is common in
hemodialysis patients
Facilitates fatty acid entry into mitochondria;
cofactor for fatty acid utilization as energy
substrate
Randomized, placebo-controlled trial of 82
patients found reduction in IDH (Ahmad et al,
Kidney Int 1990)
Mechanism underlying effect is not known
Copyright Harvard Medical School, 2010. All Rights Reserved.
1326
Sodium modeling
As solutes are removed from the
extracellular compartment, plasma
osmolality falls
Plasma osmolality decreases relative to
the intracellular compatment
Fluid moves into the intracellular
compartment at the same time that it is
being removed from the vascular
compartment
Sodium modeling
Sodium modeling helps to maintain the
plasma osmolality as ultrafiltration takes
place.
The plasma osmolality is gradually
decreased over the course of the
treatment
Linear
Stepwise
Copyright Harvard Medical School, 2010. All Rights Reserved.
1327
Cool temperature dialysis
Cool temperature dialysate induces
vasoconstriction, which helps to
maintain BP
Disadvantage: Patients may complain of
feeling cold.
Sequential UF/HD
Isolated UF followed by HD is often
referred to as sequential.
Isolated UF is iso-osmotic and therefore
less likely to lead to hypotension.
Isovolemic hemodialysis can be
performed after isolated UF.
Disadvantage: Total treatment time is
longer.
Copyright Harvard Medical School, 2010. All Rights Reserved.
1328
Intradialytic blood volume monitoring
Devices that prospectively monitor blood volume
have been advocated for better volume management.
The Crit-Lineis one such device that noninvasively
monitors hematocrit by optical transmission.
The CLIMB study compared conventional monitoring
to Crit-Linemonitoring to test the hypothesis that
Crit-Linemonitoring would decrease morbidity
associated with ultrafiltration.
There was no difference in hypotensive events
between the two groups.
There was a higher mortality and hospitalization rate
in the Crit-Linegroup.
Reddan et al, J Am Soc Nephrol 16: 2162-2169, 2005
Which treatment works best to reduce
hypotension in HD?
Dheenan and Henrich (Medical College of Ohio)
Single center study
10 stable chronic outpatients
Single-blinded, crossover study design of five different
protocols
High sodium dialysate (144 mEq/L)
Sodium modeling (152 mEq/L 140 mEq/L in the last
half hour)
Sequential (One hour of isolated UF followed by 3 hours
of isovolemic dialysis)
Cool temperature dialysis (35 C)
Standard dialysis (dialysate sodium 138 mEq/L)
Dheenan and Henrich, KI 59: 1175-1181, 2001
Copyright Harvard Medical School, 2010. All Rights Reserved.
1329
Number of hypotensive episodes per HD
treatment
Hypotensive symptoms and
signs per dialysis treatment
Copyright Harvard Medical School, 2010. All Rights Reserved.
1330
2. Chest Pain During Dialysis
Causes of Dialysis-Associated
Chest Pain
Myocardial ischemia or infarction
Hemolysis
Dialyzer Reaction
Air embolism
Pulmonary thromboembolism
Copyright Harvard Medical School, 2010. All Rights Reserved.
1331
Hemolysis
Rare with current equipment and
monitoring devices
Manifestations
chest pain/tightness, back pain, dyspnea
hyperkalemia, fall in hematocrit
port-wine appearance of blood in venous
line
pink appearance of plasma in centrifuged
blood
Causes of Hemolysis
Hypotonic or over-heated dialysate
(>55C)
Contamination of dialysate with
formaldehyde, hydrogen peroxide, bleach,
chloramine, nitrate, copper
Blood pump malfunction or defective
tubing
Copyright Harvard Medical School, 2010. All Rights Reserved.
1332
Dialyzer Reactions
Broad group of events with multiple etiologies
Type A - anaphylactic
occur early in treatment, 1st 15-20 minutes
ethylene oxide was frequent cause in past but less
frequent now with better removal of ETO by
manufacturer
Type B - milder, complement activation?
Occur any time during dialysis, usually 1st hour
back pain, chest pain
Management of Type A Reactions
Immediate: Stop Dialysis and Treat with Epi,
steroids, antihistamines
Subsequent:
Increase volume of saline for dialyzer rinsing
Replace ETO-treated dialyzer with dialyzer
sterilized by other methods
Use biocompatible membrane
Switch heparin type
Avoid combination of PAN dialyzer & ACE I
Consider latex allergy
Copyright Harvard Medical School, 2010. All Rights Reserved.
1333
Management of Type B Reactions
Immediate:
Can continue dialysis
Symptoms resolve without specific
intervention
Oxygen, saline, diphenhydramine often used
Subsequent:
Use biocompatible membrane
Reuse may help (without bleach)
This is what our patient
had - first usesyndrome
Air Embolism
Rare but need to be aware!
Multiple potential sites for air to enter circuit
Manifestations depend on patient position
Supine: dyspnea, chest pain, hypotension
Upright: loss of consciousness, seizure
Trendelenburg: lower extremity ischemia
Signs
foamingof blood in venous line
churningsound with auscultation of heart
Treatment
clamp venous line and turn off blood pump
position patient on left side, chest and head down
100% Oxygen
Aspiration of air from ventricle
Copyright Harvard Medical School, 2010. All Rights Reserved.
1334
Pulmonary Embolism
Not really a dialysis-associated complication
and not particularly likely to occur during the
dialysis session
But the lore that patients with ESRD do not
develop pulmonary embolismappears not to
be true in current era
Tveit et al, AJ KD 2002; Analysis of USRDS and
NCHS data
Age-adjusted rates of hospitalization for PE greater in
patients with ESRD than general population
3. Fever
Copyright Harvard Medical School, 2010. All Rights Reserved.
1335
Pyrogen-Reaction versus
Infection
Pyrogen reaction
Afebrile pre-dialysis, fever during dialysis
Blood cultures negative, no infection
identified
Dialysis-associated fever can occur with
use of infected catheter
More common with high-flux dialyzers
Outbreak of Pyrogen-
Reactions or Infections
Many possible sources
Inadequate disinfection of dialysis machines
Defect in water processing system
Bicarbonate concentrate (powder form eliminates
this)
Reuse process
Mishandling of intravenous medications
Copyright Harvard Medical School, 2010. All Rights Reserved.
1336
Gram Negative Infection Outbreaks
Medication Contamination
Serratia liquefaciens bloodstream infections from
contamination of epoetin alfa at a hemodialysis
center. N Engl J Med 2001; 344:1491.
WHO Contamination
An outbreak of gram-negative bacteremia in
hemodialysis patients traced to hemodialysis
machine waste drain ports. Infect Control Hosp
Epidemiol 1999 20: 746.
Reuse Procedure
An outbreak of gram-negative bacteremia traced to
contaminated O-rings in reprocessed dialyzers. Ann
Intern Med 1993; 119:1072.
Infected HD Catheter
Management
Incidence of Catheter Related Bacteremia (CRB)
Author # CRB/1000d CRB/yr
Marr,1997 (Annals) 102 3.9 1.4
Beathard, 1999(J ASN) 387 3.4 1.2
Saad, 1999 (AJ KD) 101 5.5 2.0
J ean, 2002 (Nephron) 129 1.1 0.4
Copyright Harvard Medical School, 2010. All Rights Reserved.
1337
Risk Factors for CRB
Most Significant Risks (Single Study: J ean, Nephron, 2002
91:399)
Nasal Staph Aureus Carriage
Previous Bacteremia
Peripheral Vascular and CAD
Diabetes
Others Include: Low albumin, High Ferritin,
Immunosuppression, prior bacteremia, ?higher iron,
low Hb, poor hygiene, catheter care
Treatment of CRB
Antibiotics; broad spectrum typically for 3
weeks
Salvage Rate: Prospective: 62 CRB (Marr; Ann Intern Med
1997; 127:275)
24 Immediate Removal; 38 Left In
Only 12 (32%) of catheters left in were salvaged but no
increase in complications
Catheter Exchange: Afebrile x 48h; stable w/o tunnel infection
(Robinson-Kidney Int 1998; 53:1792, Tanriover-Kidney Int 2000;
57:2151.
50% required removal within 48h
Guidewire Replacement; Ab x 3 weeks
80% 90 day Infection Free Rate
Copyright Harvard Medical School, 2010. All Rights Reserved.
1338
Approach to CRB
Fever, Sx(BCs; Empiric Abx)
Catheter Removal
Afebx48h; Stable
Abx x 3 weeks
+/-Guidewire Change
Stable? (BP; Comps)
Outline
Clinical Vignette with Discussion
Review of Current Guidelines and
Relevant Data:
Survival/Dialysis Adequacy
Complications (Acute)
Bone Disease
Cardiovascular Disease
(Anemia)
Copyright Harvard Medical School, 2010. All Rights Reserved.
1339
Case 3
An 80-year-old woman with ESRD
secondary to diabetic nephropathy is
admitted with extreme pain in her lower
extremities.
Case 3
Past Medical History
ESRD secondary to diabetic nephropathy, on
hemodialysis for 5 years
Diabetes mellitus type 2 with nephropathy,
retinopathy, neuropathy
Hypertension
Hypercholesterolemia
DVT/PE
Catheter-related bacteremia with Staph aureus,
complicated by lumbar diskitis
Hyperparathyroidism
Copyright Harvard Medical School, 2010. All Rights Reserved.
1340
Medications: Insulin, Lisinopril, Amlodipine,
Atorvastatin Gabapentin,
Acetaminophen/Hydrocodone, Warfarin
Exam: 140/80, clear lungs, S4, decreased
distal pulses
Labs: Hemoglobin 13.0 g/dL, Bun 53 mg/dL,
Creatinine 7.7 mg/dL, K+5.0 mEq/L Calcium
11.1 mg/dL, Phosphorous 5.6, Albumin 3.8
g/dL
What is the source of her pain?
Vascular surgery evaluation unremarkable
What next?
iPTH returns at 10,317 pg/mL
Alk phos-290
Should she undergo
parathyroidectomy?
Should she be treated with cinacalcet?
K/DOQI:
PHOSPHOROUS:
Phosphorus should be maintained between 3.5 and 5.5 mg/dL
(1.13 and 1.78 mmol/L).
CALCIUM:
Serum levels of corrected total calcium should be maintained
within the normal range; preferably toward the lower end
(8.4 to 9.5 mg/dL [2.10 to 2.37 mmol/L])
Calcium x Phosph: <55
iPTH: 150-300 pg/mL
Copyright Harvard Medical School, 2010. All Rights Reserved.
1341
Mortality with Disorders of Mineral
Metabolism in ESRD
Retrospective Analysis of >40,000 HD Pts:
Block GA, et al., J ASN 2004
Lowest Death Risk with:
Phosphorous 3-5 mg/dl
Calcium <8.0 mg/dl
Calcium x Phos product <45-50
iPTH<600ng/L
Similar findings in DOPPS+Others
Prospective Studies: Phosphorous elevation
correlated with increased mortality at baseline
and as time dependent variable; Calcium and
PTH only time dependent (Melamed et al., KI 2006;
Kalantar-Zadeh et al., KI 2006)
Bone Disease in CKD
Osteitis Fibrosa
PTH mediated high bone turnover
TREAT by suppressing PTH
Adynamic Bone
Low bone turnover
Pathologically the same as osteoporosis
Usually due to low PTH
TREAT by:
avoid Calcium binders
Avoid Active Vitamin D
Use low Calcium bath
Osteomalacia
Low bone turnover with large amounts of unmineralized
osteoid
Usually due to Vitamin D deficiency
In past seen commonly due to Aluminum
Suspect in dialysis patients with low bone mass and
frequent fractures
TREAT with Ergocalciferol with or without Active
Vitamin D
Copyright Harvard Medical School, 2010. All Rights Reserved.
1342
Chronic Renal Failure
Phosphate Retention Low 1,25(OH)
2
D
3
Hypocalcemia
Secondary Hyperparathyroidism
Increased PTH secretion
Chief cell hyperplasia
Pathogenesis of Secondary
Hyperparathyroidism
Phosphaturia 1-OHase
Declining Calcitriol and Increasing PTH
With CKD Progression
Kates et al. Am J Kidney Dis. 1997;30:809-813; Martinez et al. Am J
Kidney Dis. 1997;29:496-502; Martinez et al. Nephrol Dial Transplant.
1996;11(suppl 3):22-28; St. John et al. Nephron. 1992;61:422-427.
GFR (mL/min/1.73 m
2
)
50
40
30
20
10
0
105 95 75 85 65 45 35 15 55 25
1,25Vit D

Lower Limit
10
20
30
40
50
60
70
80
90
100
P
a
t
i
e
n
t
s

W
i
t
h

E
l
e
v
a
t
e
d

P
T
H

(
%
)

CKD Stage 2 Stage 3


C
a
l
c
i
t
r
i
o
l
1
,
2
5
(
O
H
)
2
D
3
(
p
g
/
m
L
)
Stage 4

Copyright Harvard Medical School, 2010. All Rights Reserved.


1343
Phosphate Restriction Prevents
Hyperparathyroidism
Slatopolsky, E, Caglar, S, Pennell, J P, et al, J Clin Invest 1971; 50:492.
Vitamin D Actions in Humans
Calcitriol
Suppress PTH
Enhance Gut
absorption of
Calcium,
Phosphorus
Enhance Bone
resorption of
Calcium,
Phosphorus
Vit D
Analog
Calcimimetic
Copyright Harvard Medical School, 2010. All Rights Reserved.
1344
Treatments-Phosphorous
Control Serum Phosphorous - Dietary! But
Malnutrition risk in HD
Binders (Calcium based, Sevelamer, Lanthanum)
Treat to Goal Study- Sevelamer vs Calcium (Chertow, KI 2002)
Similar Phos control; less hypercalcemia, low PTH levels, lower
LDL, and less coronary artery calcification with Sevelamer, Lower
CRP (US)
CARE Study (calcium acetate Renagel Eval) found better phos
control with calcium acetate (Quinibi, KI 2004)
DCOR (Dialysis Clinical Outcomes Revisited); no difference in
mortality at 3y as of 11/05 (still not published)
Limit Total Calcium from binders to <1500mg/d
(Calcium carbonate has 500mg/1250 and acetate
has 167/667)
Lanthanum Carbonate
Effective in controlling phosphate in several
studies, less hypercalcemia
A prospective randomized study showed
improved bone histology c/wcalcium
carbonate (DHaese, KI 2006)
No significant adverse events over 2y; No
hepatotoxicity up to 4y
Concern for organ accumulation (animals)
New Agents? nicotinamide, ferric
citrate,phosphonoformic acid
Copyright Harvard Medical School, 2010. All Rights Reserved.
1345
Treatments-Vitamin D
Retrospective analyses show improved
survival with Vit D in dialysis patients and
paricalcitol was better than calcitriol (Teng M., et
al, NEJ M 2003)
Three Choices (US); calcitriol, paricalcitol,
doxercalciferol
All increase serum calcium, analogues may be
better than calcitriol
1 randomized prospective trial (calcitriol vs paricalcitol);
no difference in PTH, hypercalcemia or CaxP product
(Sprague et al., KI 2003)
Months
6.0
5.6
4.8
4.4
4.0
3.6
3.2
5.2
4
150
0
75
Normal PTH
1 2 3
Control Control Calcitriol
P
T
H

(

L

e
q
/
m
L
)
I
o
n
i
z
e
d

C
a

(
m
g
/
d
L
)
Ionized Ca
PTH
n = 6
Slatopolsky et al. J Clin Invest. 1984;74:2136-2143.
Effects of IV Calcitriol on PTH and Ionized
Ca in Uremic Patients
10
Copyright Harvard Medical School, 2010. All Rights Reserved.
1346
Treatments-Cinacalcet
Calcimimetic; binds to CaSR
3 Prospective phase 3 studies have shown
increased % of patients achieving K/DOQI
endpoints
Phase 2 study showed reduced risk for PTX,
fracture and cardiovascular hospitalization.
No data on CV outcomes
Monitor: Risk of hypocalcemia, N/V (self-
limited)
200
300
400
500
600
700
800
0 2 4 6 8 10 12 14 16 18 20 22 24 26
P
a
r
a
t
h
y
r
o
i
d

H
o
r
m
o
n
e

L
e
v
e
l
,

p
g
/
m
L
Efficacy Assessment Dose Titration
P<.001*
Changes in intact PTH with Cinacalcet or
Placebo in Hemodialysis on Standard Therapy
Cinacalcet
Placebo
*Block GA, et al.
N Engl J Med 2004;350:1520-9..
week
Copyright Harvard Medical School, 2010. All Rights Reserved.
1347
Treatments-Indications for
parathyroidectomy
Severe hyperparathyroidism (persistent
serum levels of intact PTH >800 pg/mL)
associated with hypercalcemia and/or
hyperphosphatemia that is refractory to
medical therapy (KDOQI)
Musculoskeletal complications
Bone pain
Fractures
Presenting Symptoms in Patients
Undergoing PTX
Neonakis et al Arch Surg 1995; 130: 643-648
Copyright Harvard Medical School, 2010. All Rights Reserved.
1348
Increased death risk associated
with high PTH
Block et al 2004; J ASN 15: 2208-2218
Outline
Clinical Vignette with Discussion
Review of Current Guidelines and
Relevant Data:
Survival/Dialysis Adequacy
Complications (Acute)
Bone Disease
Cardiovascular Disease
(Anemia)
Copyright Harvard Medical School, 2010. All Rights Reserved.
1349
A 57 year old woman with ESRD secondary to
polycystic kidney disease on hemodialysis for 5
years has new onset of repetitive intra-dialytic
hypotension. Hypotension occurs during the last 1
hour of a 4 hour treatment (to <60/p). Her weight is
stable at 72 kg. She has had no intercurrent illness
and her intradialytic weight gains are rarely more
than 2.5 kg. She has hypertension managed by
manipulation of extracellular volume. She takes no
antihypertensive or cardiovascular medications.
Case 4
Exam notable for a BP of 150/88, no elevated
jugular venous pressure, a quiet precordium
without heaves, and a rare S4 gallop. There
is no edema and remainder of exam is
unremarkable.
Copyright Harvard Medical School, 2010. All Rights Reserved.
1350
On the basis of this clinical history, the best
next step is to:
(A) Increase intensity of dialysis
(B) Do echocardiography
(C) Increase estimated dry weight
(D) Provide linear sodium modeling
(E) Increase Hct from 36 to 40%
Recent Laboratories notable for:
Hemoglobin 11.5 g/dL
Hct 36.2%
WBC 8200, normal diff
BUN 84 mg/dl
Creat 11.1 mg/dl
Albumin 3.9 g/dl
EKG-mild LVH
Correct Answer - B
Adequately dialyzed, long term dialysis
patient has good response to EPO, minimal
weight gains and mild hypertension. The
presence of an S4 with mild hypertension
suggests abnormal ventricular compliance. Echo
would be appropriate to exclude pericardial
effusion (although unlikely in this clinical
scenario) and to assess wall motion and EF.
Echo is likely to show decreased ventricular
distensibility, or impaired relaxation, resulting in
reduced tolerance to ultrafiltration. If echo
unrevealing, one should consider stress echo or
cath.
Copyright Harvard Medical School, 2010. All Rights Reserved.
1351
There are no data to suggest increasing
dialysis intensity would be of value, nor
would increasing dry weight be
appropriate with borderline
hypertension. Providing linear sodium
modeling ignores investigation of
underlying pathophysiology. No
evidence that increasing hematocrit
would be of benefit.
CVD mortality in ESRD patients
Source:Foley et al, AJKD 32, D112-S119, 1998
Copyright Harvard Medical School, 2010. All Rights Reserved.
1352
CKD and CVD
Age
Male gender
Hypertension
Dyslipidemia
Diabetes
Smoking
Physical Inactivity
Menopause
Family History of CVD
LV Hypertrophy
Anemia
Proteinuria
Elevated Ca.P Product
Oxidative Stress
Inflammation
CKD
Kidney specific
Risk Factors
CVD
Traditional
CVD Risk Factors
Risk Factors
CHOICE Study (Longenecker J ASN 2002)
Traditional Risk Factors at Initiation:
Diabetes (54%), low HDL (33%), HTN (96%), LVH
(22%), advanced age (~60)
Other risk factors; anemia, homocysteine
Unique Risk Factors: CKD alone, RRT,
uremia, oxidant stress, calcium and mineral
metabolism, abnormal nitric oxide
Copyright Harvard Medical School, 2010. All Rights Reserved.
1353
Prevention
HTN; Pre-dialysis BP <140/90 mm Hg
Hypertriglyceridemia - common finding; no data
Hypercholesterolemia:
4D Trial; 1255 HD pts with type 2 DM; placebo vs
atorvastatin (20mg/d)
LDL lowered at 4 weeks (120 vs 72)
At 4 years, no difference in composite (CV death, MI, CVA
Reduction in MI (RR-0.82) offset by increase CVA (2.03)
Ongoing trials SHARP and AURORA
Hypocholesterolemia - associated with
increased mortality (malnutrition)
Incidence of composite end point in 4D
trial
Wanner, C., N Engl J Med 2005; 353:238.
Copyright Harvard Medical School, 2010. All Rights Reserved.
1354
Prevention-2
Homocysteine - large doses of folic acid were
minimally effective (29.5 to 21.9 mol/L)
Calcium and Phosphorous
Oxidative Stress - Vitamin E vs Placebo;
lower CV endpoints (16 v 33%) at 1.5y (Boaz, M
et al., Lancet 2000)
Similar finding with n-acetylcysteine
Fish oil - fewer MIs, no effect on
cardiovascular events and death over-all
(Svensson et al., J ASN 2006)
LVH/CHF
CHF mortality 83% at 3 years (USRDS)
Prospective Study of 431 pts;
31% incidence at initiation; 7%/year (Harnett, J D., KI 1995)
K/DOQI - Baseline ECHO and repeat at dry weight; q
3 years thereafter or if symptoms
LVH in 75-80% of dialysis patients
Major risk for morbidity; 2/3 die from CHF or sudden death
Prevention: BP!, Anemia and ?ACEI/ARB
Consider: Valvular disease, AVF shunting, Anemia,
Carnitine deficiency, Volume control
ACEI, -blockers and digoxin are under utilized and
literature supports reductions in onset of new CHF
and death (Cice J Am Coll Cardio 2003).
Our Patient - most likely LVH; probable diastolic dysfunction,
consider use of -blockers
Copyright Harvard Medical School, 2010. All Rights Reserved.
1355
Conclusions
Clinical Vignette with Discussion
Review of Current Guidelines and
Relevant Data:
Survival/Dialysis Adequacy
Complications (Acute)
Bone Disease
Cardiovascular Disease
Question 1
Which of the following strategies is least
effective in preventing intradialytic
hypotension?
A) Cool temperature dialysate
B) Sodium Modeling
C) Sequential Ultrafiltration
D) Carnitine supplementation
Copyright Harvard Medical School, 2010. All Rights Reserved.
1356
Question 2
Mrs. S. is a 65-year-old woman with ESRD
from diabetic nephropathy. She is admitted
to a hospital with chest pain. Within 10
minutes of her first dialysis treatment in the
hospital, she complained of itching, became
diaphoretic, and short of breath. Her BP was
90/50 mm Hg. She had diffuse wheezes.
Question 2- continued
Mrs. S symptoms and signs are most
consistent with
A) Acute coronary syndrome
B) Pulmonary embolism
C) Dialysis-induced pulmonary embolism
D) Air embolism
E) Type A dialyzer reaction
Copyright Harvard Medical School, 2010. All Rights Reserved.
1357
Question 3
A 38-year-old man with ESRD of uncertain
etiology has been dialyzing via a tunneled
catheter while his AV fistula matures. He
presents to dialysis with a temperature of 102
degrees F. He clinically looks well, and there
is no purulence at the catheter exit site. Blood
cultures are drawn, and he is treated
empirically with vancomycin and ceftazidime,
and the catheter is also locked with an
antibiotic solution.
Question 3-continued
Blood cultures grow methicillin resistant
Staphylococcus aureus. At his next
dialysis treatment, vancomycin is
administered and ceftazidime is
discontinued. 72 hours after his initial
presentation, he is again febrile with a
temperature of 103 degrees F.
Copyright Harvard Medical School, 2010. All Rights Reserved.
1358
Question 3-continued
At this point, appropriate steps in the
management of this patient would include:
A) Removal of the tunneled catheter
B) Discontinuing vancomycin and substituting
nafcillin
C) Discontinuing vancomycin and substituting
linezolid
D) Ultrasound of the AV fistula looking for a fluid
collection
Question 4
The patient has his catheter removed and
is afebrile thereafter. He is prescribed a
3-week course of vancomycin. At the
end of his second week of therapy, he
remains afebrile but is complaining of
debilitating back pain to the point that
he cannot walk.
Copyright Harvard Medical School, 2010. All Rights Reserved.
1359
Question 4- Continued
Of the following, which is the most appropriate
next step in management of this patient?
A) Prescribe celecoxib and physical therapy
B) Prescribe a narcotic
C) MRI of lumbosacral spine
D) Add gentamicin to antibiotic regimen
Answers
Question 1: C
Question 2: E
Question 3: A
Question 4: C
Copyright Harvard Medical School, 2010. All Rights Reserved.
1360
Conflicts of Interest
Consultant
Baxter Healthcare
Genzyme
Copyright Harvard Medical School, 2010. All Rights Reserved.
1361
Vascular Calcification:
Clinical Implications
Eduardo Slatopolsky M.D. F.A.C.P.
Joseph Friedman Professor of
Renal Diseases in Medicine
Washington University
Boston August 12, 2010
Abbott : Lectures and grant support
Genzyme: Lectures and grant support
Eduardo Slatopolsky M.D. F.A.C.P.
Washington University, School of Medicine St. Louis, MO.
Disclosures
Professor of Medicine
Vascular Calcification: Clinical Implications
Copyright Harvard Medical School, 2010. All Rights Reserved.
1362
Vascular calcification after 20 years of dialysis
carotids
coronary arteries
thoracic aorta
Agatstonscore
5,683 0
6,010 0
62,159 0
Phosphorus
Progression of Renal Disease
V
a
s
c
u
l
a
r

C
a
l
c
i
f
i
c
a
t
i
o
n
S
e
c
o
n
d
a
r
y

H
y
p
e
r
p
a
r
a
t
h
y
r
o
i
d
i
s
m
The Killer
Copyright Harvard Medical School, 2010. All Rights Reserved.
1363
Arterial Medial Calcification
Hruska KA et al, Kidney International, 2008
Copyright Harvard Medical School, 2010. All Rights Reserved.
1364
Serum Phosphorusand Cardiovascular Risk
Dhingra R et al. Arch Intern Med 2007
CVD =fatal/nonfatal MI, angina, cerebrovascular events, peripheral
vascular disease, heart failure. *HR adjusted for age, sex, BMI,
diabetes, BP, treatment of hypertension, smoking, alcohol consumption,
total cholesterol/HDL ratio, hemoglobin, serumalbumin, eGFR,
proteinuria, hsCRP
P (mg/dL)
A
d
j
u
s
t
e
d

h
a
z
a
r
d

r
a
t
i
o

f
o
r

C
V
D
*
0.0
0.4
0.8
1.2
1.6
1.62.8 2.93.1 3.23.4 3.56.2
2.0
1.55
1.27
Referent
The Framingham Offspring Study
Coronary death or
non-fatal MI
Trend p = 0.03
Fatal or non-
fatal MI
Trend p = 0.03
New heart failure
Trend p = 0.03
P (mg/dL)
1.52 1.43
1.32
0.88
0.77
0.84
Referent
1.12
1.25
1.22
< 2.5 2.53.4 3.53.9 4.0
0.0
0.5
1.0
1.5
2.0
A
d
j
u
s
t
e
d

H
a
z
a
r
d

R
a
t
i
o
N =
2632
N =
1044
N =
320
N =
131
The CARE Trial
Tonelli M Circulation 2005;
Adjusted for baselineage, sex, race, smokingstatus, diabetes, waist to
hip circumference, fastingglucose, GFR, hemoglobin, serumalbumin,
aspirin use, left ventricular ejectionfraction.
N=3,368 N=4,127
(Patients with prior myocardial infarction) (Patients with no history of CVD or CKD)
Serum Phosphorus and Mortality Risk
KestenbaumB J Am Soc Nephrol. 2005;16:520
*Adjusted for baselineage, sex, race, cerebrovascular disease,
diabetes, ischemic heart disease, HF, acuterenal failure, calcium
intakefrommedications, serumcalcium, inverseof baseline
creatinine, time-averaged creatinine, slopeof creatinine, maximal
creatinineconcentration, hemoglobin
CKD (CrCl 50.439.5 mL/min)
Block GA, et al. J Am Soc Nephrol 2004;15:2208
*Multivariable adjusted for age, gender, race, diabetes,
dialysis vintage, body weight, urea reduction ratio,
serum albumin, creatinine, predialysis blood urea
nitrogen, bicarbonate, cholesterol, hemoglobin,
ferritin, and aluminum
7%
1.00
1.15
1.32 1.32 1.34
1.83
1.90
0.0
1.0
2.0
<
2
.5
2
.5

3
.4
9
3
.5

3
.9
9
4
.0

4
.4
9
4
.5

4
.9
9

5
.0
P
(
m
g
/d
L
)
A
d
j
u
s
t
e
d

H
a
z
a
r
d

R
a
t
i
o

f
o
r

M
o
r
t
a
l
i
t
y
5%
88%
N = 6,730
Not on dialysis
P
(
m
g
/d
L
)
N = 40,538
R
e
l
a
t
i
v
e

r
i
s
k

o
f

d
e
a
t
h
*
<
3
3

4
4

5
5

6
6

7
7

8
8

9
>
9
0.00
1.0
1.4
1.6
2.0
2.2
0.08
1.2
1.8
Referent
On dialysis
Copyright Harvard Medical School, 2010. All Rights Reserved.
1365
Pathogenesis??
PO
4
CBfa1
BMP2
ALP
Osteocalcin
Osteonectin
Leptin
Collagen I
Fibronectin
LDLox
TNF-
Dexamethasone
Klotho-/-
PTH 7-84
+
+ +
+
+
+
+
+
+
+
+
Vit D3
Ca
+
Oncostatin
-
Osteoprotegerin
MGP Osteopontin
BMP7
Collagen IV
Fetuin
PTHrP
PTH 1-34
-
-
-
-
-
-
-
-
Pyrophosphate
+
Inductors (+) and Inhibitors () of Vascular
Calcifications
Copyright Harvard Medical School, 2010. All Rights Reserved.
1366
FETUIN

2
-Heremans-Schmid glycoprotein / fetuin-A
(genetic symbol Ahsg)
Fetuin is a 59-kDa glycoprotein, and
is a member of the superfamily of
cysteine protease inhibitors.
Fetuin is synthesized in the liver and is
found in high concentrations in
mammalian serum and bone due to a
high affinity to hydroxyapatite.
(cont.)
The serum fetuin concentration
in adult mammals ranges from 0.5 -
1.5 mg/ml.
Fetuin is one of the most abundant
noncollagenous proteins found in
bone, with a concentration of ~ 1 mg
of fetuin/g of bone in rat, bovine and
humans.
Copyright Harvard Medical School, 2010. All Rights Reserved.
1367
The Precipitation of a Calcium Phosphate
Mineral in Solutions Containing 5mM Calcium
and Phosphate is Biphasic and is Inhibited by
Fetuin
Price P.
Calcification of Ahsg-Deficient DBA/2
Mice Precedes Kidney Failure
Copyright Harvard Medical School, 2010. All Rights Reserved.
1368
Matrix GlaProtein Study
+ / + /
MGP Knockout Mouse
Calcified
arteries
Stained with
alizarin red
after alkaline
digestion of
soft tissues
Luo G, Karsenty G et al, Nature, 1997
Kaplan-Meier Curve Showing All-Cause Mortality
by Tertileof Serum FetuinA Concentration
High (0.70-1.53 g/l)
Medium (0.56-0.69 g/l)
Low (0.25-0.55 g/l)
All Cause Mortality (396 Events)
Follow-up Years
S
u
r
v
i
v
a
l

(
%
)
Hermans MMH, et al, Kidney Internat, 2007
Copyright Harvard Medical School, 2010. All Rights Reserved.
1369
Inducers Inhibitors
1. Pyrophosphate
2. Matrix Gla Protein
3. Fetuin-A
4. Osteoprotegerin
5. Osteopontin
6. BMP7
7. Others
1. Uremic Milieu
2. Hyperphosphatemia
3. PTH
4. Calcium Load
5. Diabetes
6. Dyslipidemia
7. Inflamation
8. Oxidative Stress
9. Other Factors
+ -
Vascular
Calcification
Inducers Inhibitors
1. Pyrophosphate
2. Matrix Gla Protein
3. Fetuin-A
4. Osteoprotegerin
5. Osteopontin
6. BMP7
7. Others
1. Uremic Milieu
2. Hyperphosphatemia
3. PTH
4. Calcium Load
5. Diabetes
6. Dyslipidemia
7. Inflamation
8. Oxidative Stress
9. Other Factors
+ -
Vascular
Calcification
Hyperphosphatemia
Calcium Load
PTH
Copyright Harvard Medical School, 2010. All Rights Reserved.
1370
Effect of Phosphate on HVSMCs
Calcification
0
100
200
300
0 3 6 9
Time (days)
C
a

D
e
p
o
s
i
t
i
o
n

(

g
/
m
g

p
r
o
t
e
i
n
)
Jono et al, Circ. Res. 2000
0
100
200
1.4 1.6 1.8 2
Pi Concentration (mM)
C
a

D
e
p
o
s
i
t
i
o
n

(

g
/
m
g

p
r
o
t
e
i
n
)
*
*
*
Calc.M. (2 mMPi)
Grow M. (1.4 mMPi)
Day 6
Calcification of Cultured Human Vascular
Smooth Muscle Cells
Jono et al Circ. Res. 87: e10, 2000
Copyright Harvard Medical School, 2010. All Rights Reserved.
1371
Vascular Smooth
Muscle Calcification
Phosphate Regulation of Vascular
Smooth Muscle Cell Calcification
Jono S, et al.,Circul Res, 2000
Pi
Na
+
Pi
PO
4
2
-
Pit-1
Osteocalcin
Cbfa-1/ Runx 2
?
What about calcium ??
Copyright Harvard Medical School, 2010. All Rights Reserved.
1372
Calcium-Phosphate Induces Calcification
in HVSMC
C
a
l
c
i
u
m

a
m
o
u
n
t
(

g
/
m
g
)

P
r
o
t
e
i
n
Phosphate/Ca treatment mMor (mg/dl)
Yang H, Giachelli C Kidney Int. 2004
250
150
100
50
0
200
Normal P / High Ca High P / High Ca High P / Normal Ca
2/2.0 2/2.2 2/2.4
(6.19/9.64) (6.19/8.82) (6.19/8.02)
2/1.8
(6.19/7.21)
1.2/1.8
1.2/2.8
(3.71/11.22)
1.2/2.4
(3.71/9.64) (3.71/7.21)
mM
(mg/dL) (7.43/7.21)
1.6/1.8 2.4/1.8
(4.96/7.21)
Gene expression(RT-PCR) in HNBSMC in 10 day culture
0
50
100
150
200
250
300
350
Pit1 ALP
%

e
x
p
r
e
s
s
i
o
n

Normal Calcium(2mM)
High Calcium(3mM)
Calcium induces Pit-1 and Alk Phos mRNA in
HSMC
Copyright Harvard Medical School, 2010. All Rights Reserved.
1373
Model for the Effects of Elevated Ca and
P on VSMC Matrix Mineralization
SMC phenotype modulation SMC phenotype modulation
P P
Ca/P loading of Ca/P loading of
matrix vesicles matrix vesicles
Pit-1
Na P
Elevated Phosphorus
Elevated Calcium
Mineral
Matrix vesicle
Collagen
- -Runx2 Runx2
- -Alkaline Alkaline phosphatase phosphatase
- -Osteocalcin Osteocalcin
Ca P Matrix Mineralization
Giachelli et al. J Am Soc Nephrol. 2004;15:2961.
Media PO
4
directly stimulates osterixexpression
Inhibitableby phosphonoformicacid
Giachellisgroup can show inhibition of PO
4
effect by RNAi
to PIT-1
Therefore, PO
4
is a signaling molecule
Serum phosphate
signal
Cbfa1(Runx 2),osterix
mineralization
Mechanisms of Phosphorus Action
Copyright Harvard Medical School, 2010. All Rights Reserved.
1374
Mizobuchi M, Towler D and Slatopolsky E J Am Soc Nephrol 2009
Mizobuchi M, Towler D and SlatopolskyE J AmSoc Nephrol 2009
Urine toxins
Ca
++
PO
4
3-
PiT1
Apoptotic
bodies
Matrix
vesicles
VSMC
DYING VSMC
VSMC VESICLE
PHAGOCYTOSIS
VSMC osteochondrocytic
transdifferentation
Oxidized
LDL
H
2
O
2
Runx2, Sox9
PPi
PO
4
3-
Bone ALP
(TNAP)
OSTEOBLAST OR
HYPERTROPHIC
CHONDROCYTE
Skeletal matrix
synthesis
Elastin
fragmentation
Osx
Runx2
Msx2
Osteogenic
Lineage allocation
BMP2
Canonical Wnt
Inflammatory
cytokines
Nucleation
OPN, PPi Fetuin, MGP
VSMC
Adipocyte
MULTIPOTENT VASCULAR MESENCHYMAL
PROGENITOR CVC, PERYCITE
Nucleation Propagation
Copyright Harvard Medical School, 2010. All Rights Reserved.
1375
What about PTH ???
Massive Medial Aortic Calcification in high-PTH Rats
Neves KR et al, Kidney International, 2007
Sham +NP U +HP U +LP Sham +HP Sham +NP
100X
400X
High-PTH
(0.7% P) (1.2% P) (0.2% P)
Copyright Harvard Medical School, 2010. All Rights Reserved.
1376
The Effects of Calcium Containing
Phosphate Binders
Experimental Studies:
The Effect of Sevelamer on Calcification of the Thoracic Aorta
Katsumata K et al, Kidney Int, 2003
Copyright Harvard Medical School, 2010. All Rights Reserved.
1377
Effects of Sevelamer and CaCO
3
on Serum Phosphorus
Control
Sevelamer
CaCO
3
0
3
6
9
12
15
Basal Uremic -6 mo.
mg/dl
Normal
p<0.01
p<0.01
Effects of Effects of Sevelamer Sevelamer and CaCO and CaCO
3 3
on on Creatinine Creatinine Clearance Clearance
(Uremia = 6 months) (Uremia = 6 months)
0.2
0.4
0.6
0.8
U-HP U-HP+S U-HP+C
ml/min
p<0.05 p<0.05
Copyright Harvard Medical School, 2010. All Rights Reserved.
1378
Effects of Sevelamer and CaCO
3
on Urinary Protein
Excretion in Rats after 6 Months of Uremia
0
100
200
300
Ctr S 3% C 3%
mg/24h
p<0.05 p<0.05
Effects of Calcium Carbonate and
Sevelamer
on Kidney Calcification
HP+CaCO
3
3% HP+Sevelamer 3%
Uremia = 3 months
HP
Copyright Harvard Medical School, 2010. All Rights Reserved.
1379
Effects of Effects of Sevelamer Sevelamer and CaCO and CaCO
3 3
on on
Calcium Deposition in Rat Myocardium Calcium Deposition in Rat Myocardium
0
50
100
150
200
250
Uremia 6 months

g
/
g

w
e
t

t
i
s
s
u
e
Normal Control Sevelamer CaCO
3
p<0.05
p<0.01
p<0.01
Uremia = 6 months
Effects of Calcium Carbonate and Effects of Calcium Carbonate and Sevelamer Sevelamer
on on Intramyocardium Intramyocardium Arterial Calcification Arterial Calcification
U-HP+C
U-HP+C
U-HP+S
U-HP+S
Copyright Harvard Medical School, 2010. All Rights Reserved.
1380
X
PO
4
PTH
PO
4
PTH
PO
4
PTH
UHP UHP + C 3% UHP + S 3%
GFR GFR GFR
UProt UProt UProt
UCa
UCa
UCa
Clinical Studies
Copyright Harvard Medical School, 2010. All Rights Reserved.
1381
Multivariable-Adjusted Relative Risks of Serum
Phosphorus
Referent Range, 4.0-5.0 mg/dl (1.29-1.61 mmol/L)
0.0
0.4
0.8
1.2
1.6
2.0
2.4
R
e
l
a
t
i
v
e

R
i
s
k

o
f

D
e
a
t
h
Serum Phosphorus Range (mg/dl)
<3 3-4 4-5 5-6 6-7 7-8 8-9 >9
Block GA et al, J Am Soc Nephrol, 2004
mmol/L 1.61-1.94
25% 8%
1.94-2.26
N=40,538
P<0.0001
Phosphorus Balance
Secretion
150 mg/day
Absorption
950 mg/day
Excrete urine
800 mg/day
Blood < 1%
Phosphorus
Pool
Excrete feces
400 mg/day
FOOD
1200 mg/day
1350 mg/day
Formation
300 mg/day
Resorption
300 mg/day
Total adult body stores 700 g, 85% in:
Copyright Harvard Medical School, 2010. All Rights Reserved.
1382
Hyperphosphatemia
Dietary Phosphate Restriction
Effective Dialysis
3. Phosphate Binders:
a. Aluminum e. Nicotinamide
b. Ca Carbonate f. Sevelamer
c. Ca Acetate g. La Carbonate
d. Mg Carbonate h. Iron
i.Chitosan
Dietary Phosphorus by Dietary
Protein Intake in 60 HD Patients
0
300
600
900
1200
1500
D
i
e
t
a
r
y

P

(
m
g
/
d
a
y
)
<.6 .6-.8 .8-1.0 1.0-1.2 >1.2
Protein Intake (g/kg/day)
Rufino NDT 1998
Copyright Harvard Medical School, 2010. All Rights Reserved.
1383
Phosphate Balance in Dialysis
Patients
0
2000
4000
6000
8000
10000
1200
720
450
8400
5000
1800
3200
Daily Weekly
PO
4
Ingested PO
4
Absorbed
60%
PO
4
Removed
4 HD
PO
4
Balance
P
O
4
(
m
g
)
Hyperphosphatemia
Dietary Phosphate Restriction
Effective Dialysis
3. Phosphate Binders:
a. Aluminum e. Nicotinamide
b. Ca Carbonate f. Sevelamer
c. Ca Acetate g. La Carbonate
d. Mg Carbonate h. Iron
i. Chitosan
Copyright Harvard Medical School, 2010. All Rights Reserved.
1384
Dietary Phosphorus and Elemental
Calcium Ingestion from Calcium
Carbonate
1
0
4
8
12
16
20
0.5 0.6 0.7 0.8 0.9 1 1.1 1.2 1.3 1.4
Dietary Phosphorus (g/d)
C
a
C
O
3
I
n
t
a
k
e

(
g
/
d
)
1
Slatopolsky et al., Kidney Int 1989;30:397
Normal
Intake
Net Calcium Balance
-200
0
200
400
600
800
1000
C
a
l
c
i
u
m

(
m
g
/
d
a
y
)
Control
CaCarb.
Pt 1 Pt 2 Pt 3 Normal Normal
Cr Clearance:
5 18 28
Clarkson et al., Clin Sci, 1966
Copyright Harvard Medical School, 2010. All Rights Reserved.
1385
Conclusions
It seems from the evidence presented here that the
administration of calcium carbonate to patients
with renal failure may occasionally be of some
small temporary therapeutic value.
Unless all of the calcium which is
absorbed is laid down in bone, the
calcium content of soft tissues
may rise

Clarkson et al., 1966


Coronary Artery Calcification in Young
Dialysis Patients
0.1
1
10
100
1000
10000
0 5 10 15 20 25 30 35
Age (years)
C
a
l
c
i
f
i
c
a
t
i
o
n

S
c
o
r
e
*
N=39
*Determined by EBT.
Goodman WG, et al. N Engl J Med. 2000;342:1478-1483.
Calcification scores
doubled in patients with
positive initial scan
when rescanned at 20
months
Copyright Harvard Medical School, 2010. All Rights Reserved.
1386
Risk Factors Associated With Increased Risk for
Cardiac Calcification in Young Dialysis Patients
Serum calcium was not significant.
Goodman WG, et al. N Engl J Med. 2000;342:1478-1483.
Coronary No
Calcification Calcification
Factor (n=14) (n=25) P Value
Ca from 6456 4278 3325 1490 0.02
calcium binders (mg/day)
Serum P (mg/dL) 6.9 0.9 6.3 1.2 0.06
Ca P product (mg
2
/dL
2
)* 65.0 10.6 56.4 12.7 0.04
Age (years) 26 3 15 5 <0.001
Mean duration
of dialysis (years) 14 5 4 4 <0.001
High Calcium Intake Increases
Calcification Score
*Determined by ultrasonography.
Gurin AP, et al. Nephrol Dial Transplant. 2000;15:1014-1021.
1.35 1.35
1.5
1.84
2.18
0.5
1.0
1.5
2.0
0 1 2 3 4
E
l
e
m
e
n
t
a
l

C
a
l
c
i
u
m

I
n
t
a
k
e

(
g
/
d
a
y
)
Calcification Score*
2.5
N=120
P=0.001 (ANOVA)
Copyright Harvard Medical School, 2010. All Rights Reserved.
1387
Arterial Calcification* Increases
Mortality Risk
*Determined by ultrasonography.

Carotid artery, abdominal aorta, iliofemoral axis, and legs.


P<0.0001 for each increase in number of arteries calcified.
Blacher J , et al. Hypertension. 2001;38(4):938-942.
N=110
1
Follow-Up (months)
0
0.25
0.5
0.75
0 20 40 60 80
P
r
o
b
a
b
i
l
i
t
y

o
f

S
u
r
v
i
v
a
l
0 Arteries

Calcified
1 Artery Calcified
2 Arteries Calcified
3 Arteries Calcified
4 Arteries Calcified
Arterial Stiffness in Dialysis Patients
and Probability of Survival
140 0 35 70 105
1.00
0.75
0.50
0.25
0
PWV < 9.4 m/s
9.4 PWV 12.0 m/s
PWV > 12.0 m/s
Blacher J, et al. Circulation. 1999;99:2434-2439
<--Months
of f/up
Copyright Harvard Medical School, 2010. All Rights Reserved.
1388
The Effect of Sevelamer on Urinary
Phosphorus Excretion
0
100
200
300
400
0 0.5 1 3 9
T
o
t
a
l

U
r
i
n
a
r
y

P


(
m
g
)
% Dietary Sevelamer
*
*
*
*p<0.01 vs 0%
GTC-49-301: Treat to Goal/
Calcification Study
Randomize
Washout
Washout
Sevelamer
Calcium
Binder
Extended Treatment
Extended Treatment
2 weeks 12 weeks 40 weeks
CT
Scan
Titrate Dose
Pi 3.0 to 5.0 mg/dL
Ca
++
<10.5 mg/dL
CT
Scan
Titrate Dose
Pi 3.0 to 5.0 mg/dL
Ca
++
<10.5 mg/dL
PTH 150-300 pg/mL
CT
Scan
Chertow GM et al, Kidney Inter. 2002
Copyright Harvard Medical School, 2010. All Rights Reserved.
1389
The Effect of Sevelamer and Calcium Carbonate/
Acetate on Serum Phosphorus Levels
4.5
5
5.5
6
6.5
7
7.5
8
-2 2 6 10 14 18 22 26 30 34 38 42 46 50
Time (Weeks)
P
h
o
s
p
h
o
r
u
s

(
m
g
/
d
l
)
Calcium
Sevelamer
S=7.6
C=7.4
S=5.1
C=5.1
The Effect of Sevelamer and Calcium on
Hypercalcemia
0
5
10
15
20
25
-2 2 6 10 14 18 22 26 30 34 38 42 46 50
Time (Weeks)
Calcium
Sevelamer
%

o
f

P
a
t
i
e
n
t
s
Hypercalcemia: Ca>10.5 mg/dL (>2.63mmol/L)
Copyright Harvard Medical School, 2010. All Rights Reserved.
1390
Percent Coronary Change in
52 weeks
0
5
10
15
20
25
30
35
Calcium RenaGel
25%
6%
M
e
d
i
a
n

P
e
r
c
e
n
t

C
h
a
n
g
e
* within treatment p<0.0001
*
Percent Change in Aortic Scores
at 52 weeks
28%
*
5%
M
e
d
i
a
n

P
e
r
c
e
n
t

C
h
a
n
g
e
* p<0.0001
Copyright Harvard Medical School, 2010. All Rights Reserved.
1391
Median Percentage Change in Coronary
And Aorta Scores at 104 Weeks
* Between treatment groups P<0.0001 (Patients with a baseline score >30)
-30
-10
10
30
50
70
90
M
e
d
i
a
n

P
e
r
c
e
n
t
a
g
e

C
h
a
n
g
e
83%*
20%*
-7%*
66%*
Coronary Aorta
Bommer J, EDTA, 2002
Ca Sevelamer
52 weeks
52 weeks
Ca Sevelamer
The Effects of Sevelamer or Calcium
on LDL Cholesterol
60
80
100
120
140
-2 0 12 24 52
S=102
C=102
Calcium
Sevelamer
Study Week
L
D
L

C
h
o
l
e
s
t
e
r
o
l

(
m
g
/
d
l
)
Copyright Harvard Medical School, 2010. All Rights Reserved.
1392
Multivariable-Adjusted Relative Risks of Serum Calcium
Referent Range, 9.0-9.5 mg/dl (2.25-2.38 mmol/L)
0.0
0.5
1.0
1.5
2.0
R
e
l
a
t
i
v
e

R
i
s
k

o
f

D
e
a
t
h
Serum Calcium Range (mg/dl)
<8 8-8.5 8.5-9 9-9.5 9.5-10 10-10.5 10.5-11 >11
Block GA et al, J Am Soc Nephrol, 2004
mmol/L 2.5-2.65
0.78
0.86 0.89
1
1.14
1.2
1.28
1.48
2.1-2.3 2.3-2.38 2.0-2.1 <2.0 2.38-2.5 2.65-2.75 >2.75
n=40,531
p<0.0001
DOPPS I Versus DOPPS II
DOPPS I DOPPS II
Countries 7 12
Facilities 309 340
Patients 19,197* 16,200
Patient years 23,595* 21,000
*Includes replacement patients
Dialysis Outcomes and Practice Patterns Study (DOPPS). General information and objectives. 2003. Available at: www.dopps.org.
University Renal Research and Education Association (URREA). TheDOPPSReport. 2002;4:1-8.
Copyright Harvard Medical School, 2010. All Rights Reserved.
1393
Association between Study Outcomes
(All-Cause and Cardiovascular Mortality) and
Markers of Mineral Metabolism
Outcome Measure
Predictor
All-Cause Mortality
Cardiovascular
Mortality
Phosphorus (per 1 mg/dl) 1.10 (p <0.0001) .04 (p<0.0001)
Ca x P Product (per 5 mg
2
/dl
2
) 1.03 (p<0.0001) 1.06 (p=0.0001)
PTH (per 100 pg/ml) 1.01 (p=0.03) 1.02 (p=0.007)
Calcium (per 1 mg/dl)
1.12(p=0.0001)
Effects of Sevelamer and calcium
on coronary artery calcification in
patients new to hemodialysis
Block, G.A. et al.
Kidney International, Vol68(4): 1815-1824 (2005)
RIND trial
Copyright Harvard Medical School, 2010. All Rights Reserved.
1394
Calcium
binder
Effects of sevelamer and calcium on coronary artery
calcification in patients newto hemodialysis
Sevelamer Extended treatment
RANDOMIZE
w/in 90 days
of hemodialysis
0 EBCT scan
Titrate dose
P<6.5 mg/dL
Ca
2+
<10.2 mg/dL
USUAL clinical practice
Extended treatment
6 mo 12 mo 18 mo EBCT scans
Titrate dose
P<6.5 mg/dL
Ca
2+
<10.2 mg/dL
PTH 150-300 pg/mL
Maintain dialysate Ca=2.5 mEq/L
Renagel patients can receive Ca supplementation at night
Dialysate Ca concentration was maintained at 2.5 mEq/L (1.25 mmol/L) throughout the
study period Block GA. Kidney Int 2005;68:1815-1824
n=109
Average Phosphorus Control by Binder
Mean Serum Phosphorus by Study Month
Block, GA, Kidney Int; Vol68(4): 1815-1824, 2005
5.1mg/dL
5.2 mg/dL
Elemental Calcium = 2.3 g/day
Sevelamer = 8 g/day (10 pills)
n = 109
Copyright Harvard Medical School, 2010. All Rights Reserved.
1395
Mean Serum Calcium by Study Month
Mean Serum Calcium Level by Binder
8.5
8.7
8.9
9.1
9.3
9.5
9.7
0 1 2 3 4 5 6 7 8 9 10 11 12 13 14 15 16 17 18
Study Month
S
e
r
u
m

C
a
l
c
i
u
m

(
C
o
r
r
e
c
t
e
d
)
Renagel Calcium
8.5
8.7
8.9
9.1
9.3
9.5
9.7
0 1 2 3 4 5 6 7 8 9 10 11 12 13 14 15 16 17 18
Study Month
S
e
r
u
m

C
a
l
c
i
u
m

(
C
o
r
r
e
c
t
e
d
)
Renagel Calcium
Block, GA, Kidney Int; Vol68(4): 1815-1824, 2005
9.6 mg/dL
9.1 mg/dL
Effects of Sevelamer and Calcium on
Coronary Artery Calcification
0
50
100
150
200
250
300
350
Baseline 6 months 12 months 18 months
M
e
d
i
a
n

C
A
C
S
Sevelamer Calcium
N=54 N=55 N=51 N=53 N=45 N=47 N=40 N=45
In Patients New to Hemodialysis
Block, GA, Ki dney Int; Vol68(4): 1815-1824, 2005
Copyright Harvard Medical School, 2010. All Rights Reserved.
1396
0
20
40
60
80
100
120
140
Sevelamer Ca Salts
M
e
d
i
a
n

I
n
c
r
e
a
s
e

C
A
C
S
0
20
40
60
80
100
120
140
Sevelamer Ca Salts
M
e
d
i
a
n

I
n
c
r
e
a
s
e

C
A
C
S
Effects of Sevelamer and Calcium on
Coronary Artery Calcification
The median absolute
increase in CACS at
18 months was 11-
Fold greater in the
calcium treated
group compared to
the sevelamer-treated
group
P=0.01
In Patients New to Hemodialysis
Block, GA, Kidney Int; Vol 68(4): 1815-1824, 2005
Mortality effect of coronary calcification and
phosphate binder choice in incident hemodialysis
patients
GA Block, P Raggi, A Bellasi, L Kooiengaand
DM Spiegel
Kidney International. Advanced Online Access, 3 January 2007
Copyright Harvard Medical School, 2010. All Rights Reserved.
1397
Mortality Analysis Comparing
Baseline Calcium Score
Months
0
S
u
r
v
i
v
a
l

D
i
s
t
r
i
b
u
t
i
o
n

F
u
n
c
t
i
o
n
0.00
0.25
0.50
0.75
1.00
6 12 18 24 30 36 42 48 54 60 66
CCS=0
CCS< 400
CCS >= 400
No. at Risk Mortality Rate
CCS = 0 46 3.3 /100 patient years (.4 6.1)
CCS < 400 42 7.0 /100 patient years (2.7 11.4)
CCS >= 400 39 14.7/100 patient years (8.1- 21.4)
Block G et al Kid. Int. 2007
Adjusted Survival by Binder Type
Cox Proportional Hazard
Regression Model
Age
Race
Sex
Diabetes
Baseline CCS
Ca vs. Sevelamer p=0.02
(Hazard Ratio 3.1)
Block G et al Kidney Inter. 2007
Mortality Analysis Comparing CalciumVersus Sevelamer
Months
0 6 12 18 24 30 36 42 48 54 60 66
S
u
r
v
i
v
a
l

D
i
s
t
r
i
b
u
t
i
o
n

F
u
n
c
t
i
o
n
0.00
0.25
0.50
0.75
1.00
No. atRisk
Calcium 60 57 54 49 38 22 5
Renagel 54 52 52 46 42 25 4
Calcium
Renagel
11
death
23 death
Copyright Harvard Medical School, 2010. All Rights Reserved.
1398
Lanthanum Carbonate
LANTHANUM CARBONATE
Lanthanum is a rare element
Atomic weight 139
Binds phosphate ionicallyat all pH
values
Lanthanum phosphate very insoluble
Low intestinal absorption
Copyright Harvard Medical School, 2010. All Rights Reserved.
1399
The Effects of Lanthanum or Calcium
Carbonate on Mean Serum Phosphorus
Levels
1.1
1.6
2.1
2.6
3.1
0 2 4 6 8 10 12 14 16 18 20 22 24 26
Treatment Weeks
S
e
r
u
m

P
h
o
s
p
h
o
r
u
s

(
m
m
o
l
/
l
) S
e
r
u
m

P
h
o
s
p
h
o
r
u
s

(
m
g
/
d
l
)
Hutchinson AJ, Nephrol Dial Transplant, 2004
3.40
9.60
8.05
6.50
4.95
Lanthanum Carbonate
Calcium Carbonate
Titration Phase Dose-Maintenance Phase
Tissue Distribution of Lanthanum in
Normal and Uremic Rats
0
500
1000
1500
2000
2500
3000
3500
L
a
n
t
h
a
n
u
m

(
n
g
/
g

w
e
t

w
t
)
Kidney Bone Liver Kidney Bone Liver
Normal Uremic
SlatopolskyE et al, Kidney Internat. 2005
Copyright Harvard Medical School, 2010. All Rights Reserved.
1400
Lanthanum Content of Liver in Normal
and Uremic Rats fed Lanthanum (La) or
Control (C) Diets
0
500
1000
1500
2000
2500
3000
0 20 40 60 80 100 120
Time (days)
L
a
n
t
h
a
n
u
m

(
n
g
/
g

w
e
t

w
t
)
NC
N+La
UC
U+La
Niacinamide
(nicotinamide)
(Vitamin B
3
)
Works on intestinal Na-PO
4
co-transporter NaPi II -b
Copyright Harvard Medical School, 2010. All Rights Reserved.
1401
P <0.001
Takahashi, et al Kidney Int 65: 1099 1104, 2004
Chitosan
A cationic polymer.
The OH and the NH2 polar groups in the structure
act as electron donors and interacts with inorganic
salts.
Chewing Gum
The 20-mg chitosanloaded chewing gum was manufactured
as cold compressed three-layer tablets, 1.7g in weight
Copyright Harvard Medical School, 2010. All Rights Reserved.
1402
Savica, V. et al. J Am Soc Nephrol 209;20:639-644
Serum and salivary phosphate levels during treatment
with chewing gum containing Chitosan
Phosphate Binder Algorithm
PO
4
> 5.0 mg/dl
Prescribe a binder
Evidence of
Vascular
Calcification
PTH <150 pg/ml
Serum Ca >10
mg/dl
or or
Calcium
Up to 1.0 to 1.5
g/day
NO
Add Sevelamer or
Lanthanum
Sevelamer or
Lanthanum
or
Does Patient
Have:
Copyright Harvard Medical School, 2010. All Rights Reserved.
1403
Board Questions!!!!
The inducers of vascular calcification
are:
a. Alkaline phosphate
b. Pyrophosphate
c. Fetuin
d. GLA protein
Copyright Harvard Medical School, 2010. All Rights Reserved.
1404
Serum phosphorus affects vascular
smooth muscles cells (VSMC) by:
a. Increasing Cbfa1 (Runx2)
expression
b. Increasing osteocalcin
expression
c. Increasing calcium uptake
d. All of the above
An increase in pulse wave velocity
(PWV) will induce:
a. A propensity to develop left
ventricular hypertrophy (LVH)
b. Decreased myocardial oxygen
demand
c. Wide brachial arterial pulse
pressure
d. a and c
Copyright Harvard Medical School, 2010. All Rights Reserved.
1405
Sevelamer will decrease:
a. Serum phosphorus
b. LDL cholesterol
c. Fetuinlevels
d. a and b
Vascular Calcification Meta-Analysis:
All-Cause Mortality
Rennenberg RJMW, et al. Vasc Health Risk Manag. 2009.
Odds Ratio
(95% CI)
12.30 (3.47, 43.67)
Study
3.16 (2.19, 4.55)
1.43 (1.22, 1.68)
2.14 (1.64, 2.81)
Blacher 2001
Fox 2003
Kemmeren1998
Lehto1996
15.16 (5.74, 40.06)
4.51 (2.24, 9.07)
3.04 (1.40, 6.61)
3.54 (2.13, 5.89)
London 2003
Wang2003
Wilens2006
Okuno2007
3.52 (2.25, 5.50)
Overall
% Weight
7.1
15.0
16.3
15.7
9.3
11.8
11.1
13.7
1 0.1 10
Odds Ratio
Vascular calcification
increases
all-cause mortality
219,000 patients ,follow up 10 years
Copyright Harvard Medical School, 2010. All Rights Reserved.
1406
Tissue Mineralization in FGF-23
Deficiency
1
Razzaque MS and Lanske B J Endocrinol 2007;194(1)110
The Fgf23 null phenotype is
also characterized by
hyperphosphatemia
Rescue of the null phenotype in
terms of the heterotopic
mineralization is accomplished
by breeding with 1-alpha
hydroxylase deficient mice or by
feeding a low phosphate diet
1
Thus, Fgf23 mediates
osteoblastic mineralization and
phosphate excretion
Abbott : Lectures and grant support
Genzyme: Lectures and grant support
Eduardo Slatopolsky M.D. F.A.C.P.
Washington University, School of Medicine St. Louis, MO.
Disclosures
Professor of Medicine
Vascular Calcification: Clinical Implications
Copyright Harvard Medical School, 2010. All Rights Reserved.
1407
Iron Metabolism and Iron Therapy in Anemia
Management
Daniel W. Coyne, M.D.
Professor of Medicine
Director of Hemodialysis
Chromalloy American Kidney Center
Washington University School of Medicine, St. Louis, MO
Conflict of interest statement: consultant, advisor, and speaker for
Abbott; Consultant: INEOS, Watson; Consultant and Speaker, AMAG,
Pharmacosmos; PI in multicenter studies designed and funded by
Abbott, AMAG, Amgen, Genentech, Roche, and Watson
Copyright Harvard Medical School, 2010. All Rights Reserved.
1408
Inflammation and Chronic Kidney Disease
Relative or absolute
Erythropoietin
Erythropoiesis
Anemia/Hypoxia
Epoetin
Administration

Erythropoiesis
Resolution of Anemia
Iron Deficiency
Folate/B12 Deficiency Hemolysis
GI Bleeding
Worsening
Inflammation
Iron Deficiency
Clinical Manifestations (I)
Fatigue
Decreased exercise tolerance
Tachycardia
Dermatologic manifestations
Decreased intellectual performance
Dysphagia
Depression, increased incidence of infections
Restless legs syndrome
Hoffman, ed. Hematology: Basic Principles and Practice, 4th ed. 2005.
Trost LB, et al. J Am Acad Dermatol. 2006;54:824.
Copyright Harvard Medical School, 2010. All Rights Reserved.
1409
Iron Deficiency
Clinical Manifestations (II)
Skin and conjuctival pallor
Koilonychia
Angular cheilosis
Burning tongue
Glossitis
Hair loss (alopecia areata)
Top figure accessed from: www.nature.com/bdj/v194/n12/images/4810265f1, with permission from
Nature Publishing Group.
Bottom figure accessed from: www.dentistry.leeds.ac.uk/biochem/lectures/nutrition.org. Modern
Nutrition
in Health & Disease. 9
th
ed. Editors: Shils, Olsen, Shike & Ross. Williams & Williams, pub.
Diagnosis of Iron Depletion in
the BodyHaematology
Iron deficiency
Thalassaemia syndromes
Haemoglobinopathies (E,C,CS, Lepore)
Anaemia of chronic diseases
Familial sideroblastic anaemia
Miscellaneous (lead intoxication)
Hypochromic, microcytic anaemia usually with high platelets
Differential diagnosis of microcytosis
Hoffman, ed. Hematology: Basic Principles and Practice, 4th ed. 2005.
Copyright Harvard Medical School, 2010. All Rights Reserved.
1410
Two Types of Dietary Iron
Heme and nonheme
iron are absorbed
through distinct
pathways
Andrews NC. Intestinal iron absorption: current concepts circa 2000. Dig Liver Dis. 2000;32:56-61.
Dietary iron exists in
two forms
Heme (in meat, poultry
and fish)
Nonheme (in other
foods)
Heme Iron
Heme iron can be
directly absorbed
via the heme-
carrier protein 1
(HCP-1) via apical
enterocytes of the
duodenum
Agarwal R. Epoetin and Iron Deficiency. In: Nissenson AR, Fine RN, eds. Clinical Dialysis. 4th
Edition. Elsevier, Burlington, MA. In press.
HCP-1
Copyright Harvard Medical School, 2010. All Rights Reserved.
1411
Iron Transport Mechanisms in the
Duodenal Enterocyte
Most iron absorption
takes place in the
proximal duodenum,
near the gastric
outlet
Andrews NC. Intestinal iron absorption: current concepts circa 2000. Dig Liver Dis. 2000;32:56-61.
Agarwal R. Epoetin and Iron Deficiency. In: Nissenson AR, Fine RN, eds. Clinical Dialysis. 4th Edition.
Elsevier, Burlington, MA. In press.
Dcytb
At the brush border, a ferric
reductase (Dcytb)
facilitates conversion of
dietary nonheme iron
from Fe
3+
to Fe
2+
Iron Transport Mechanisms in the
Duodenal Enterocyte
Ferroportin, in turn, is
regulated by hepcidin
Increased hepcidin
causes decreased
circulating iron
Andrews NC. Intestinal iron absorption: current concepts
circa 2000. Dig Liver Dis. 2000;32:56-61.
Ferroportin
Ferroportin transport
regulates the amount
of iron that leaves the
duodenal enterocytes
and goes into the
circulation
Copyright Harvard Medical School, 2010. All Rights Reserved.
1412
Ferroportin
Ferroportin is the protein transporter on the
surface of every cell which exports iron
Intestinal Cells
Reticuloendothelial cells
Macrophages
Internalized by Hepcidin
Complex is degraded in lysosomes
Effectively reducing iron absorption and RES iron release
Hepcidin Internalizes Ferroportin,
Preventing Iron Efflux from Cells
Ferroportin on cell
surface (no hepcidin)
Ferroportin internalized
in presence of Hepcidin
Elizabeta Nemeth, et al. Science 306, 2090 (2004);
Copyright Harvard Medical School, 2010. All Rights Reserved.
1413
Factors Affecting Hepcidin Expression?
Transferrin Receptor
1
Hemojuvelin
2
Oral iron
1
Iron stores
1,2
Lipopolysaccharide
2
IL-6
2
HFE
1
-
Erythropoiesis
1
Anemia
1
Hypoxia
1
Non-transferrin bound iron?
+
Hepcidin
Tf =transferrin; TfR =transferrin receptor; HJ V =hemojuvelin; LPS =lipopolysaccharide;
IL =interleukin; NTBI =nontransferrin-bound iron.
1. Leong W, Lnnerdal B. J Nutr. 2004;134:1.
2. Lee P, et al. Proc Natl Acad Sci U S A. 2004;101:9263.
Hepcidin is increased in CKD and
Dialysis Patients
Zaritsky et al. Clin J
Am Soc Nephrol 4:
10511056, 2009
Copyright Harvard Medical School, 2010. All Rights Reserved.
1414
Hepcidin is Increased by Inflammation
and CKD
Ganz, T et al. Blood. 2008;112:4292-4297
Transferrin Cycle
Transferrin circulates in the
blood and functions as the
intercellular iron shuttle
Each transferrin molecule can
deliver two Fe
3+
ions
Agarwal R. Epoetin and Iron Deficiency. In: Nissenson AR, Fine RN, eds.
Clinical Dialysis. 4th Edition. Elsevier, Burlington, MA. In press.
The transferrin protein loaded
with iron binds to a transferrin
receptor on the surface of a cell
and undergoes receptor-
mediated endocytosis
Transferrin
Receptor
2
3
Endosome
Diferric Transferrin
1
Copyright Harvard Medical School, 2010. All Rights Reserved.
1415
Transferrin Cycle (cont)
The cell acidifies the endosome,
causing transferrin to release
its iron ions
Agarwal R. Epoetin and Iron Deficiency. In: Nissenson AR, Fine RN,
eds. Clinical Dialysis. 4th Edition. Elsevier, Burlington, MA. In press.
The receptor is then
transported through the
endocytic cycle back
to the cell surface,
ready for another round of
iron uptake
5
4
Iron release
Mitochondria
Transferrin
Receptor
2
Apotransferrin
Endosome
3
Diferric Transferrin
1
Hepcidin Actions
Hepcidin
RES System
Iron Sequestration
Intestinal Iron
Absorption
Serum Iron
TSAT
Iron Deficiency
Copyright Harvard Medical School, 2010. All Rights Reserved.
1416
Inflammation and Chronic Kidney Disease
Relative or absolute
Erythropoietin
---
Decreased Intestinal
Iron absorption
(long term response)
Decreased Mobilization
of Iron from RE stores
(short term response)
Erythropoiesis
Anemia/Hypoxia
Hepcidin Expression
Epoetin
Administration
Erythropoiesis
Resolution of
Anemia
KDOQI Recommendations
for IV Iron Use
Insufficient Iron During ESA Therapy
TSAT <20%
CHr <29 pg/cell
Serum ferritin <200 ng/mL for HD-CKD
<100 ng/mL for ND-CKD and PD-CKD
KDOQI. Am J Kidney Dis. 2006;47(suppl 3):S11-S145.
When serum ferritin is >500 ng/mL, decisions about IV iron
treatment should weigh factors such as patients clinical status,
ESA dose / responsiveness, Hb level, and iron indices
Copyright Harvard Medical School, 2010. All Rights Reserved.
1417
Oral vs IV Iron in Peritoneal Dialysis:
EPO and Hct Changes
*P<.05 vs oral iron.
Reprinted with permission from Ahsan. J Am Soc Nephrol. 1998;9:664-668.
Month 6 Baseline
3000
4000
5000
6000
7000
8000
9000
M
e
a
n

E
P
O

D
o
s
e
,

U
/
w
k
10000
IV Iron Oral Iron
28
29
30
31
32
33
34
M
e
a
n

H
c
t
,

%
36
35
*
*
Month 6 Baseline
8 9 10 11 12 13 14 15
9 -------Entry------11 12.6 13.4
Target
TREAT Study
The Actual Results of the Dont Treat Arm
1. TREAT to target of 12.6 to 13.4 g/dL, compared to
2. Dont TREAT if Hgb is 9.0 g/dL, then target 9.0 g/dL
TREAT
GOAL 9 g/dL
ESA only if Hgb < 9g/dL
Hgb (g/dL)
Dont TREAT
Dont TREAT
Copyright Harvard Medical School, 2010. All Rights Reserved.
1418
TREAT Trial: Mean Hemoglobin Levels
through 48 Months
Baseline Hgb
Hgb = 11.0 g/dL
Pfeffer et al. NEJ M 2009
Baseline Iron Parameters Anemic CKD Stage 4
and 5 pts nave to IV Iron and ESA
Baseline Data
N=100
Median Interquartile
Range (IQR)
Ferritin (ng/ml) 176 79 300 ng/ml
TREAT Ferritin 134 67 258 ng/ml
TSAT (%) 23% 13 30 %
TREAT TSAT 23% 18 29 %
48% of patients had no stainable Iron in their Bone Marrow
1.Response to IV iron occurred in 63% of iron-deplete and 30% of iron-replete
patients.
2.No Ferritin or TSAT cut-off could adequately predict the response to IV iron.
They concluded the therapeutic trial to IV iron seems to be a useful tool in
the management of anemia in nondialysis patients with CKD.
Stancu et al. CJ ASN 2010 Pfeffer et al. NEJ M 2009
Copyright Harvard Medical School, 2010. All Rights Reserved.
1419
What Do These Trials Tell Us About
Anemia in CKD Patients?
1. The natural history of anemia in CKD is NOT
inexorable worsening
2. In CKD, Iron Deficiency is far more common
than we think and plays a major role in severity
of the anemia
1. Ferritin and TSAT are helpful only when low
3. Treatment with iron can raise Hgb and delay or
defer ESA requirements
1. Consistent with several IV iron trials in CKD patients
Use of Iron in CKD Patients Not on
Hemodialysis: Summary
Based on the results of randomized controlled
trials, in Stage 3 or 4 CKD or PD patients
Use oral iron
Switch to IV iron if the expected Hb response is
suboptimal or ESA dose is high or increasing
Agarwal R. Epoetin and Iron Deficiency. In: Nissenson AR, Fine RN, eds. Clinical
Dialysis. 4th Edition. Elsevier, Burlington, MA. In press.
Copyright Harvard Medical School, 2010. All Rights Reserved.
1420
Oral Iron Is Insufficient to Replenish Iron
Stores in CKD Patients on Hemodialysis
Chromagen and Tabron (ferrous fumarate); Niferex (polysaccharide); Feosol (ferrous sulfate).
1. KDOQI. Am J Kidney Dis. 2001;37(suppl 1):S182-S238.
2. Wingard RL, Parker RA, Ismail N, et al. Efficacy of oral iron therapy in patients receiving
recombinant human erythropoietin. Am.J .Kidney Dis. 1995;25:433-439.
Oral iron is generally
insufficient to maintain
adequate Hb/Hct
1,2
Adverse effects may lead
to compliance problems
GI absorption is inadequate
0
10
20
30
40
Chromagen

Feosol

Niferex

Tabron

M
e
a
n

H
c
t
,

%
Hct at Month 6 With 4
Oral I ron Supplements
2

p< 0.05;

p<0.01;

p<0.001
Oral
iron
IV
iron
14 0
Time after starting
treatment over 56 days
C
u
m
u
l
a
t
i
a
v
e

r
e
s
p
o
n
s
e

(
%

a
c
h
i
e
v
i
n
g

H
b

1
.
0

g
/
d
L
)

60
40
20
0
28 42 56
IV Iron Increases Hgb and Results in Lower
ESA Doses over timeReduces
Hb increase: IV versus Oral
Dialysis - 0.83g/dL greater
CKD - 0.31g/dL greater
E
P
O

d
o
s
e

r
e
d
u
c
t
i
o
n


[
%
]
0
10
20
30
40
50
60
70
Clinical Studies
Bhandari et al J Clinical Pharmand Therap 1998 Bhandari et al Haematology 1998
Van Wyck DB et al Kidney Int 2005;68:28462856 Rozen-Zvi et al. Am J Kid Dis 2008;52:897
% Reduction in ESA Dose
Copyright Harvard Medical School, 2010. All Rights Reserved.
1421
Current KDOQI Recommendations for
IV Iron Use in Dialysis Patients
Targets of iron therapy:
Sufficient iron should be administered to generally maintain
the following indices of iron status during ESA treatment:
HD-CKD: Serum ferritin >200 ng/mL
AND TSAT >20%, or CHr >29 pg/cell.
When serum ferritin is >500 ng/mL, decisions about IV iron
treatment should weigh factors such as patients clinical
status, ESA dose / responsiveness, Hb level, and iron
indices
KDOQI Anemia GL, Am J Kidney Dis. 2006;47(suppl 3):S1-S146.
Opinion-based Statement
Problems with the Ferritin of 500 Limit
1. Ferritin of 500 depends on the assay
2. High intrapatient variability
in serum ferritin over 2-6 wks
Ford, Coyne, Eby and Scott, Kidney International (2009) 75, 104110
200-500 501-800 801-1200
Initial Ferritin(ng/ml)
Copyright Harvard Medical School, 2010. All Rights Reserved.
1422
Problems with the Ferritin of 500 Limit
1. Ferritin of 500 depends on the assay
2. High intrapatient variability
in serum ferritin over 2-6 wks
Ford, Coyne, Eby and Scott, Kidney International (2009) 75, 104110
200-500 501-800 801-1200
Initial Ferritin(ng/ml)
If Ferritin is 500 ng/mL the next test must be <340 or >660 to be 95% confident
that it is a true physiologic change Ford et al. KI 2009
Problems with the Ferritin of 500 Limit
3. No solid scientific validity
Studies showing ferritin of 500 separates IV iron
responders from non-responders = 0
Studies showing 500 is a safety breakpoint = 0
4. Halt iron at 500 is not what KDOQI said
When serum ferritin is >500 ng/mL, decisions
about IV iron treatment should weigh factors such
as patients clinical status, ESA dose /
responsiveness, Hb level, and iron indices
Copyright Harvard Medical School, 2010. All Rights Reserved.
1423
Serum Ferritin as a Marker of Iron Status
1. Predictive Value of Ferritin for Iron needs
A. as assessed by Iron BM stores
B. as assessed by IV iron responsiveness
Absolute Iron DeficiencyDiagnosis
Microphotograph
of bone marrow
staining for iron.
Iron is stained
blue and it is
mainly in the
macrophages
(lower left)
Graphic courtesy of Dr. P. Beris.
Copyright Harvard Medical School, 2010. All Rights Reserved.
1424
Rocha et al CJASN 2008
Serum Ferritin Level Remains a Reliable Marker of Bone Marrow
Iron Stores Evaluated by Histomorphometry in Hemodialysis
Patients
Tertiles of #cells with
stainable iron
1
st
: 0 58 ng/ml
2
nd
: 58 111 ng/ml
3
rd
: >111 ng/ml
Note that points near zero
stainable iron cells have
ferritin values that extend
from <100 to >1000 ng/ml
Only the circled points have
low ferritin and few stainable
cells
0 100 200 300 400 500
1600
1200
800
400
0
Bone Marrow Iron (cells/mm2)
F
e
r
r
i
t
i
n
(
n
g
/
m
l
)
Rocha et al. CJ ASN in press doi: 10.2215/CJ N.01630408
Rocha et al CJASN 2008
Serum Ferritin Level Remains a Reliable Marker of Bone
Marrow Iron Stores Evaluated by Histomorphometry in
Hemodialysis Patients
Rocha et al. CJ ASN 2009
A Reliable test tells you
something about the pts iron
stores
1.If F =1000, are the stores
absent, moderate, or replete?
2.If F =500, are the stores
absent, moderate, or replete?
3.If Iron STORES are Low, what
Ferritin value would we see?
This study shows Ferritin
is related to iron stores,
but not reliable enough for
clinical decisions.
0 100 200 300 400 500
160
0
120
0
800
400
0
Bone Marrow Iron (cells/mm2)
F
e
r
r
i
t
i
n
(
n
g
/
m
l
)
R
2
=0.13
Copyright Harvard Medical School, 2010. All Rights Reserved.
1425
Anemic CKD Patients
Use of IV Iron Without Concomitant ESA
*P<.01
Gotloib L, et al. J Nephrol. 2006;19:161-167.
224 135 Serum ferritin, g/L
13 4 TSAT, %
235 54 Serum transferrin,
mg/dL
46.8 14.8 Serum Fe, g/dL
10.16 1.32 Hb, g/dL
Before Fe Variable
275 92
22 7
204 44
64.3 17.0
11.96 1.52
After Fe
+51 102
+8 7
-31 46
+17.5 19.2
+1.80 1.72
Change*
Forty-six of 47 patients had no evidence of iron deposits in the
bone marrow. Then given Ferric Gluconate 250 IV twice per month
x 3 months.
Study Design: Hb 11g/dL, TSAT 25%, ferritin 500-1200 ng/mL,
Epoetin dose 22,500 IU/week
Major inclusion criteria
Hb 11g/dL, TSAT 25%, ferritin 500-1200 ng/mL, EPO dose 225 IU/kg/week or
22,500 IU/week, and 125 mg/week IV iron in any of the 4 weeks preceding
enrollment
Major exclusion criteria
Active infection, recent blood loss, recent inpatient hospitalization
L =lab panel, H =Hb and CHr, X =125 mg Fe gluconate
Control:
No Iron
IV Iron:
1g Fe
Gluconate
L
Randomization
and 25% increase
in baseline EPO
dose
Epoetin dose held
constant
Epoetin and IV iron doses per anemia manager
discretion
H H H H H L
week
7
week
8
wk 12 wk 11 week
9
wk 10 week
1
week
2
week
6
week
5
week
3
week
4
week 0
screening/
baseline
Local lab Hb, TSAT, ferritin
collected
Epoetin dose held
constant
Epoetin and IV iron doses per anemia manager
discretion
week
1
week
2
week
6
week
5
week
3
week
4
week
7
week
8
wk 12 wk 11 week
9
wk 10
H H H H H L
X X X X X X X X X X X X X X X X
DRIVE DRIVE II
Coyne DW et al. J Am Soc Nephrol. 2007;18:975-984.
Copyright Harvard Medical School, 2010. All Rights Reserved.
1426
IV Iron Increased Hgb more than ESA dose increase alone, and Persists at
12 weeks
Despite Lower Epoetin Dose
10.1
11.6
11.4
12.1 11.9
10.3
10.0
10.5
11.0
11.5
12.0
12.5
DRIVE baseline Week 6 (End of DRIVE) Week 12
H
g
b

(
g
/
d
L
)
Control Ferric Gluconate
P =0.062
After week 6, Patients returned to routine anemia management.
Epoetin dose was adjusted per usual dialysis unit practice.
DRIVE DRIVE II
Coyne Dw, Kapoian T, Suki W, et al. J Am Soc Nephrol. 2007;18:975-984. Kapioan, Coyne, et al. JASN 2008
P =0.028
Coyne DW et al. J Am Soc Nephrol. 2007;18:975-984. Kapioan, Coyne, et al. JASN 2008
IV Iron when Ferritin is >500 ng/ml and TSAT 25% Decreased
ESA Dose Significantly
Epoetin dose was significantly lower at 12 weeks in the IV ron group
compared to the Control group (p=0.017)
Control group Epoetin doses remained significantly elevated (p=0.0004)
IV Iron group EPO doses returned to baseline level (p=0.6039)
DRIVE DRIVE II
Coyne DW et al. J Am Soc Nephrol. 2007;18:975-984. Kapioan, Coyne, et al. JASN 2008, Pizzi et al. Kidney Int 2008
Epoetin Difference
~9,600 u/wk
P =0.017
protocol-
mandated 25%
increase
Epoetinchanges
per MD discretion
~$5,000 per 1g
~$3,600 per 1g
Copyright Harvard Medical School, 2010. All Rights Reserved.
1427
DRIVE Trial: Use of Ferritin to Guide IV
iron Use
47% responded to IV iron with >2 g/dl increase in Hgb
Ferritin of 500 to 1200 ng/ml had no predictive value
Withholding IV iron leads to Iron-restricted erythropoiesis as
seen by steadily falling reticulocyte Hgb content (CHr)
Coyne DW et al. JASN 2007;18:975-984. Singh AK et al. Kidney Int. 2007
P=0.011 IV Iron
vs No iron arm
P<0.001 No iron
group BL to wk 6
The Proper use of Ferritin to Guide
Decisions on IV Iron Use
CKD 3 and 4
1. A low Ferritin (<100 ng/ml) usually indicates low Iron
stores.
2. A higher ferritin lacks predictive value. Use clinical
judgment on whether to give IV or oral iron.
3. IV iron can raise Hgb, delay or prevent the need for
ESA therapy, or lower ESA doses.
Patients on Dialysis
1. A low Ferritin (<200 ng/ml) usually indicates low Iron
stores.
2. A higher ferritin lacks predictive value. Use clinical
judgment on whether to give IV iron.
3. IV iron can raise Hgb and lower ESA doses, and cost.
Copyright Harvard Medical School, 2010. All Rights Reserved.
1428
Inflammation and Chronic Kidney
Disease
Relative or
absolute
Erythropoietin
?
Decreased Intestinal
Iron absorption
(long term response)
Decreased Mobilization
of Iron from RE stores
(short term response)
Erythropoiesis
Anemia/Hypoxia
Hepcidin Expression
Epoetin
Administration

Erythropoiesis
Resolution
of Anemia
IV Iron
Therapy
Bypasses
Hepcidin
Mediated
Iron
Blockade
Balanced Treatment of Anemia
Decision Points
Anemic, High or increasing EPO dose?
TSAT < 30%?
When was last IV iron given? How much?
If no or only low dose iron in last 3-6 months, may be
iron deficient
Any evidence of GI losses?
Any evidence of hospital / procedure losses?
If yes to blood losses, Hgb may rise with time, and pt
may also be iron deficient
Copyright Harvard Medical School, 2010. All Rights Reserved.
1429
Summary
Iron Absorption is regulated by Hepcidin
Low hepcidin promoted iron absorption/mobilization
High hepcidin inhibits iron absorption/mobilization
Hepcidin is an Acute Phase Reactant
Ferroportin is the crucial transmembrane
transport protein moving iron from cells to
circulating transferrin
Internalized by high hepcidin levels
IV iron can bypass hepcidin-mediated iron
blockade, raise Hgb, and lower ESA dose
Iron Overload
Copyright Harvard Medical School, 2010. All Rights Reserved.
1430
Feder J N, et al. Nat Genet. 1996;13:399.
Porter J B. Br J Haematol. 2001;115:239.
Causes of Iron Overload
Primary (hereditary)
Resulting from a primary defect in the
regulation of iron balance, eg, hereditary
haemochromatosis
Secondary (acquired)
Caused by another condition or by its
treatment
Anaemias requiring repeated blood transfusion (eg,
thalassaemia, sickle cell disease, and
myelodysplastic syndromes)
Ineffective erythropoiesis
Toxic ingestion
Type 1 Type 2A Type 2B Type 3 Type 4
Gene HFE HJ V HAMP
(hepcidin)
TFR2 Ireg1
Ferroportin 1
Function Interact with
TfR1
Unknown Regulates
iron export
Iron uptake Iron export
Inheritance
Incidence
Recessive
Common
Recessive
Rare
Recessive
Rare
Recessive
Rare
Dominant
Rare
Tissues
affected
Liver;
hepatocytes
macrophages
Skeletal
muscle,
heart,
liver
Liver
hepatocyte
Liver
hepatocyte
Duodenum
Macrophage
Clinical
presentation
Late variable Early onset
Severe
Early onset
Severe
Severe Variable
Hepcidin levels High
With permission fromWorwood M. Blood Rev. 2005;19:69.
Classification of Haemochromatosis
Copyright Harvard Medical School, 2010. All Rights Reserved.
1431
Liver cirrhosis/
fibrosis/cancer
Diabetes
mellitus
Growth failure
Capacity of serum transferrin to
bind iron is exceeded
Iron overload
Cardiac failure
Infertility
Complications of Iron Overload
Nontransferrin-bound iron
circulates in the plasma
Excess iron promotes the
generation of free hydroxyl
radicals, propagators of
oxygen-related tissue
damage
Insoluble iron complexes
are deposited in body
tissues and end-organ
toxicity occurs
Courtesy of Dr. M. D. Cappellini.
Olivieri N, et al. Blood. 1997;89:739.
Liver Iron and Risk from Iron Overload in non-CKD pts
50
30
0
40
10
20
Increased risk of complications
Normal
0 20 50 10 30 40
Age (years)
Thalassaemia major:
transfusion without
chelation
Homozygous
haemochromatosis
Heterozygote
Optimal level in chelated patients
H
e
p
a
t
i
c

i
r
o
n
,

m
g
/
g

o
f

l
i
v
e
r
,

d
r
y

w
e
i
g
h
t
Threshold for cardiac
disease and early death
H
e
p
a
t
i
c

I
r
o
n

(

m
o
l
/
g

w
e
t

w
e
i
g
h
t
)
50
100
150
200
250
0
In a study reporting IV iron leads to Liver iron overload in HD patients,
39 of 40 HD patients had a liver iron less than 2000 mcg/g by SQUID testing
Copyright Harvard Medical School, 2010. All Rights Reserved.
1432
Blood Transfusion Predicts Heart Iron
in Unchelated non-CKD Patients
0
2
4
6
8
10
12
14
16
18
20
E
s
t
i
m
a
t
e
d

H
e
a
r
t

I
r
o
n

(

m
o
l
/
g
)
2
5
5
0
7
5
1
0
0
1
2
5
1
5
0
Blood Units Transfused
J ensen PD, et al. Blood. 2003;101:4632.
Upper
Normal
Limit
Slide courtesy of Dr. J . Porter.
75 units of blood
corresponds to
~17,000 mg iron
without ongoing
loss of blood or
iron
Serum Ferritin and Risk from
Iron Loading in non-CKD pts
Change in serum ferritin over time reflects change
in liver iron concentration
Sequential evaluation of ferritin provides good index of
chelation history
1
Maintenance of serum ferritin <2500 ng/ml
significantly correlates with cardiac disease-free
survival
2-5
1. Gabutti V, et al. Acta Haematol. 1996;95:26. 2. Olivieri NF, et al. N Engl J Med. 1994;331:574. 3.
Telfer PT, et al. Br J Haematol. 2000;110:971. 4. Davis BA, et al. Blood. 2004;104:263.
5. Borgna-Pignatti C, et al. Haematologica. 2004;89:1187.
Copyright Harvard Medical School, 2010. All Rights Reserved.
1433
12 HD Patients with End-Organ Damage from
Transfusion-related Iron Overload in the pre-ESA Era
A 45 y.o. hemodialysis patient has been taking ferrous fumarate
200 mg PO daily for 6 months. He is in good health, has ESRD due
to HTN, and has a well functioning fistula for access. He received
Epoetin 12,000 u IV qHD. Recent tests show: Hgb 11.1 g/dL, iron
58 mcg/ml, TSAT 22%, ferritin 489 ng/ml, black stools, hemoccult
negative.
Which statements is true concerning his present iron status?
1. His iron stores are replete
2. Ferroportin levels are elevated due to Epoetin
3. He is unlikely to be absorbing adequate iron
4. Administration of IV iron is unlikely to improve his
Hgb or TSAT due to his normal TSAT and ferritin
5. His hepcidin level is likely normal or low
Copyright Harvard Medical School, 2010. All Rights Reserved.
1434
Conflict of interest statement: consultant, advisor, and speaker for
Abbott; Consultant: INEOS, Watson; Consultant and Speaker, AMAG,
Pharmacosmos; PI in multicenter studies designed and funded by
Abbott, AMAG, Amgen, Genentech, Roche, and Watson
Copyright Harvard Medical School, 2010. All Rights Reserved.
1435
Dialysis Vascular Access
BWH Board Review Course
Boston, August 12, 2010
Dirk Hentschel, MD
Instructor in Medicine, Harvard Medical School
Director Interventional Nephrology BWH Renal Division Director, Interventional Nephrology, BWH Renal Division

Disclosures
Consultant: Proteon Therapeutics
Off-label use of stents and stent-grafts
Copyright Harvard Medical School, 2010. All Rights Reserved.
1436
Goals for this presentation:
Challenge you to examine dialysis accesses C a e ge you to e a e d a ys s accesses
and think about access flow.
Stimulate you to look at fistulograms and
acquire basic skills in reading them.
Become a knowledgeable partner to your
access surgeon and interventional
nephrologist or radiologist.
No dialysis without access
In 1943 Kolff treated his first
Dr. Willem Kolff
patient with CDK, a 29-year-old
household maid.
Each placement of cannulas for
access required a cut-down to
an artery. After 12 treatments the
patient had no more suitable
access sites and died.
rotating drum kidney
Copyright Harvard Medical School, 2010. All Rights Reserved.
1437
Access Milestones
1960 Scribner shunt (Teflon)
1962 Qunton-Scribner shunt 1962 Qunton Scribner shunt
(siliconized Teflon)
http://www.washington.edu/alumni/columns/sept03/images/extras_scribner.jpg
Belding Scribner, MD 1921-2003
Nature Med 2002; 8:1066-67
Access Milestones
1966 Cimino Fistula
James E. Cimino, MD
Copyright Harvard Medical School, 2010. All Rights Reserved.
1438
1961 Shaldon-catheter
Odman-Ledin plastic tubing
regional heparinisation
First dialysis catheter
Dr. Stanley Shaldon
regional heparinisation
veno-venous dialysis with
catheter
Fig. 1 and 3, Lancet 1961;2:857-859
Nephrologists have been the driving force for
innovations in dialysis access care.
Today, nephrologist seem remote from Today, nephrologist seem remote from
hemodialysis access.
The decision regarding type and location of
access is left to the access surgeon.
Th t i ( t i ) i The maturing (or non-maturing) access is
evaluated by interventional radiologists or
surgeons who rarely see an access during
dialysis.
Copyright Harvard Medical School, 2010. All Rights Reserved.
1439
1. Elements of dialysis access physical
Physical Exam of Dialysis Accesses
1. Elements of dialysis access physical
exam
2. Looking beyond the access
3. Bridging device - tunneled catheters
4 Indications for access study referral 4. Indications for access study referral
5. Pseudoaneurysms
KDOQI Guideline 4 -
Detection of Access Dysfunction
Copyright Harvard Medical School, 2010. All Rights Reserved.
1440
Surveillance - Predicting Stenosis
Expert Physical Exam - PPV 69-93%
Schwab KI 1989, Safa Radiology 1996, Cayco AJKD 1998,
1. Static dialysis venous pressures
- Positive predictive value (PPV) 92% (Besarab KI 1995)
1. Flow monitoring
PPV 87 100% (Schwab KI 1995 Moist JASN 2003)
gy y
Robbin Radiology 1998, Maya AJKD 2004, Robbin KI 2006
- PPV 87-100% (Schwab KI 1995, Moist JASN 2003)
Duplex Ultrasound
- PPV 80% (Robbin KI 2006)
Nephrologists miss abnormal physical exam
of dialysis accesses
BWH HD population, n=72, 8 nephrologists
No time or no expertise?
Copyright Harvard Medical School, 2010. All Rights Reserved.
1441
Fistula First - Changes in Access Prevalence
modified after Pisoni KI 2002 and Spergel Sem Dial 2008
Shift to upper arm accesses
Distribution of Accesses - one Dialysis Unit
Brescia-Cimino Fistula is a vanishing entity !
Copyright Harvard Medical School, 2010. All Rights Reserved.
1442
Historical Experience - Brescia-Cimino-
Fistula
Autogenous
access created in access created in
1990
(>6000 needles
insertions)
Dual Outflow
flow >2000ml/min
radial artery 8mm
What happens with a forearm fistula
without dual outflow?
2004
one PTA, one Thrombectomy
5 years
Copyright Harvard Medical School, 2010. All Rights Reserved.
1443
Left upper arm autogenous brachial-cephalic access created in 9/2007
The 51y-old patient is concerned about increase in access size
6/2008
He returns one year later with the same concern
7/2009
Access is pulsatile and does not soften with arm
elevation.
Copyright Harvard Medical School, 2010. All Rights Reserved.
1444
7/2009
Access flow: 2700 ml/min
Upper arm access: Upper arm access:
single outflow often without collaterals
larger artery
Risk of High Inflow-Limited outflow imbalance
1 P l tilit
Elements of Access Exam
1. Pulsatility
2. Thrill
3. Augmentation
4. Murmur / Bruit
5. Collapse of access against gravity
Copyright Harvard Medical School, 2010. All Rights Reserved.
1445
Pulsatility
1. changes with blood pressure
2. changes with diameter of vessel
3. changes after branch
points/collaterals points/collaterals
Pulsatility
QuickTime and a
Photo - JPEG decompressor
are needed to see this picture.
Copyright Harvard Medical School, 2010. All Rights Reserved.
1446
occluded
brachial vein
arterial anastomosis
brachial vein
occluded
brachial vein
bovine jump-biograft to cephalic vein
creating dual outflow
Copyright Harvard Medical School, 2010. All Rights Reserved.
1447
Thrill
1. accompanies a bruit
2. sign of turbulent flow
3. presence depends on relation of
blood flow to diameter of vessel blood flow to diameter of vessel
Left upper arm brachial-
cephalic autogenous
access created June 2008
Known left brachio-
cephalic vein stenosis
Progressive enlargement of
access until March 2010
Copyright Harvard Medical School, 2010. All Rights Reserved.
1448
Shoulder
Chest
Elbow
Access flow measured as 4100ml/min
Access pulsatile along palpable segment
No thrill -
except at juxta-anastomotic segment
Augmentation
1. depends on blood pressure
2. decreased/absent in presence of
inflow stenosis or early collateral
3 very important for needle insertion 3. very important for needle insertion
Copyright Harvard Medical School, 2010. All Rights Reserved.
1449
60-year-old ESRD patient with left upper arm autogenous
brachial-cephalic access created 5/29/2009. In September
inability to use access with several infiltrations.
Collapse with Arm elevation
1. indicates adequate outflow in
relation to inflow
2. need to check augmentation
Copyright Harvard Medical School, 2010. All Rights Reserved.
1450
63-year-old ESRD patient
with left forearm
autogenous radial-
cephalic access created
15 years ago y g
Arm Elevation
Copyright Harvard Medical School, 2010. All Rights Reserved.
1451
Flow-Murmur (Bruit)
1. pitch - higher at/after site of stenosis
2. continuity - discontinuous if large flow
only during systole (high pressure)
3. character depends on relation of blood
flow to diameter of vessel
Non-maturing left upper arm autogenous brachial-cephalic access
1. Increased pulsatility at anastmosis
juxta-anastomotic stenosis
2. Thrill just after anastomosis
3. Diminished augmentation
4. Discontinuous flow murmur in juxta-
anastomotic segment
Copyright Harvard Medical School, 2010. All Rights Reserved.
1452
Left upper arm autogenous brachial-cephalic access 7/27/09
9/25/09 - flow <400ml/min
Chest
10/8/09 - flow 1100ml/min
Chest
Elbow
Shoulder
Elbow
Shoulder
Beathard (n=100) Nasser (n=118)
Inflow artery 4 5
Early AVF Failure - surgical technique ?
Arterial Anastomosis 38 48
Juxta-anastomotic 49 64
Peripheral Vein 39 60
C t l V i 9 8 Central Vein 9 8
Accessory Veins 46 30
Beathard Kidney Int 64:1487-1494, 2003
Nasser, ASN 2005 Abstract
Copyright Harvard Medical School, 2010. All Rights Reserved.
1453
Giving up too early ? - assisted
maturation
92% of fistulas were used for initiation of dialysis
at 12 months 68% functional
Beathard Kidney Int 64:1487-1494, 2003
Location of Stenoses in Autogenous Accesses
radial-cephalic brachial-cephalic
Nephrol Dial Transplant 2000; 15:2029-2036
Copyright Harvard Medical School, 2010. All Rights Reserved.
1454
67-year-old ESRD patient with left forearm radial-cephalic
autogenous access created in 9/2004. Recurrent juxta-
anastomotic stenosis, even after surgical patch angioplasty,
now undergoing angioplasty in about 6 months intervals for
flow < 450ml/min.
Maximum flow after angioplasty 650-800ml/min
Copyright Harvard Medical School, 2010. All Rights Reserved.
1455
True / False
1. Cannulation of a fistula can be performed 6
weeks after it is created weeks after it is created.
2. Maturation of AV fistulas is associated with
pre-operative venous diameters > 2.5mm and
arterial diameters >2 mm.
3. Twelve weeks after creation a fistula should
have a diameter >6mm, be less than 5mm
below the skin surface, and have a blood-flow
of > 800ml/min.
True / False
1. Cannulation of a fistula can be performed 6
weeks after it is created
T
weeks after it is created.
2. Maturation of AV fistulas is associated with
pre-operative venous diameters > 2.5mm and
arterial diameters >2 mm.
3. Twelve weeks after creation a fistula should
T
have a diameter >6mm, be less than 5mm
below the skin surface, and have a blood-flow
of > 800ml/min.
F
Copyright Harvard Medical School, 2010. All Rights Reserved.
1456
1. AVFs can safely be cannulated 1-2
months after creation
Saran Nephrol Dial Transplant (2004) 19: 23342340
But: reduce blood flow rate to below 300ml/min and
extend dialysis time.
2. AVF Duplex-evaluation
- arterial diameter > 2.0mm
- venous diameter > 2 5mm
Silva MB Jr, J Vasc Surg 27:302-307
venous diameter > 2.5mm
- blood flow increases within weeks to
500-800ml/min
Malovrh Nephrol Dial Transplant 13;125-129
Yerdel Nephrol Dial Transplant 12; 1684-1688
- 80% accuracy of experienced
examiner to predict AVF success
Robin Radiology 225; 59-64
Copyright Harvard Medical School, 2010. All Rights Reserved.
1457
K/DOQI CPG 3 - Cannulation of AVF and AVG
1. Aseptic technique [A]
2. AVF [B]
- Rule of 6s: at 6 weeks Q>600ml/min d
AVF
> 6mm < 6mm deep Rule of 6s: at 6 weeks Q>600ml/min, d
AVF
> 6mm, < 6mm deep
- hand-arm exercises
- failure to mature at 6 weeks: imaging/fistulogram
3. AVG [B]
PTFE - wait 2 weeks/swelling down
composite/PU - > 24 hours
rotate cannulation sites (pseudoaneurysms)
4. CVC
- examine tunnel site [B]
- dressing change each HD treatment [A]
- aseptic technique: mask, gloves [A]
K/DOQI CPG 1 - Vein Preservation
1. GFR < 30ml - education (HD, PD, Tx) [A]
2 CKD S 4 d i /IV/PICC NOT 2. CKD Stage 4 and 5 - venipuncture/IV/PICCs NOT on
upper or forearm [B] [hands are site of venipuncture!]
3. Timing at least [B]
- AVF 6 months prior
- AVG 3-6 weeks prior
- PD cath 2 weeks prior
4. Evaluations before HD access placement:
- H and P [B]
- Duplex US upper extremity [B]
- central vein evaluation if prior CVC or PM [A]
Copyright Harvard Medical School, 2010. All Rights Reserved.
1458
Physical Exam - Look Listen Feel
AVF Graft
nl nl nl nl
Look
straight main vein
no aneurysm
collaps c elevation
aneuryms
surface collaterals
uniform sized
no aneurysm
site rotation
Listen
low pitch continuous
high pitch
discontinuous
systolic
low pitch continuous
high pitch
discontinuous
systolic
Feel
thrill throughout
easy to compress
water-hammer
pulse at stenosis
loss of thrill
thrill arterial
anstomosis strong
but throughout
easy to compress
water-hammer
pulse
thrill may be
increased at
stenosis
Physical exam ideally
performed prior to dialysis
without needles
Bruit, pulsatility, thrill,
collapse against gravity
often can be tested during
dialysis with the pump
speed dialed to near zero.
Copyright Harvard Medical School, 2010. All Rights Reserved.
1459
Complications of dialysis accesses
1. Central vein stenosis
2. Steal
3. Pseuodaneurysm
74-year-old chronic hemodialysis paitent with left upper arm prosthetic
brachial-axillary c-loop configuration access
He noted enlarge neck veins as well as humming noise in left ear.
Copyright Harvard Medical School, 2010. All Rights Reserved.
1460
Shoulder Sternum
Copyright Harvard Medical School, 2010. All Rights Reserved.
1461
39-year-old ESRD patient
over past months increasing
h d h f i d t d headaches, friends noted
darker face
left upper arm axillary-axillary
biograft access
Venous stasis
pigmentation pigmentation
Total occlusion of left brachiocephalic vein
Copyright Harvard Medical School, 2010. All Rights Reserved.
1462
Collateral flow through
intracranial venous
sinuses sinuses
Access flow 4200ml/min
Banding of graft inflow to flow
of 1600ml/min
Headaches vanished,
i t i i t ti improvement in pigmentation
Central vein stenosis/obstruction
1. Distended neck veins
2. Arm swelling
3. Changes in skin pigmentation
Copyright Harvard Medical School, 2010. All Rights Reserved.
1463
Right upper arm autogenous brachial-basilic direct access
2008 2008
DM II, HTN, PVD
Right upper arm autogenous brachial-basilic direct access
In 2010 the access has enlarged and flow is
between 1600-2400ml/min
Copyright Harvard Medical School, 2010. All Rights Reserved.
1464
Patient fell with
small scab on
2nd digit of
right hand,
over next
weeks and weeks and
months this
developed....
Finger
amputated
Hand
amputation
th t d threatened
Flow guided
banding of
access to 700-
800ml/min
Gravity and repeated needle injury
Autogenous access
created in 1990
(>6000 needles insertions)
Copyright Harvard Medical School, 2010. All Rights Reserved.
1465
Needle insertion sites
81-year-old women right upper arm
t b hi l h li autogenous brachial-cephalic
access created 11/30/04
In 7/2008 notable blanching of
arterial needle insertion site
1/2009
presents
ith with
drainage
from
arterial
needle
insertion
site
Copyright Harvard Medical School, 2010. All Rights Reserved.
1466
1/2009 bovine bypass biograft
in 2010 access in use
successfully
Summary 1
1. Physical exam skills of dialysis accesses are valuable
tools in determining access dysfunction and function
2. Dialysis care providers need to identify dedicated and
knowledgeable interventionlists (radiology/nephrology)
and access surgeons to tailor custom solutions for each
patient
3. The ideal dialysis access balances outflow limits and
potential for steal with sufficiently high flow for dialysis. p y g y
4. Challenge from here on: Who and How to train access
competency?
Copyright Harvard Medical School, 2010. All Rights Reserved.
1467
Tunneled catheters
Cant live with them, cant live without
them......
Pisoni KI 2001
6479 patients from DOPPS, observational study
incident pts with permanent access: 430 Europe and 428 US
one-year survival: AVF Europe 83%, AVF US 68%, AVG US 49%
Copyright Harvard Medical School, 2010. All Rights Reserved.
1468
Subclavian vein
tunneled catheter
Tip position
Courtesy Ted
Saad
Exposed cuff
Itching at exit site
Copyright Harvard Medical School, 2010. All Rights Reserved.
1469
Itching around exit site
Exit site infection
Fever and chills for one
day
Tender area over clavicle
tunnel infection
Copyright Harvard Medical School, 2010. All Rights Reserved.
1470
Role of the Surgeon
Dember JAMA 2008;299(18):2164-2171: 60% failure to
attain suitability for dialysis
877 AVFs by 71 surgeons over the course of
the study for 3.2 fistulae per surgeon per year
Konner Kidney Int 2002;62(1):329-338: >100 fistulas per
year, failure to mature rate <11%
Fassiadis Semin Dial 2007;20(5):455-457: >100 fistulas Fassiadis Semin Dial 2007;20(5):455-457: >100 fistulas
per year per surgeon, <7% failure to mature
rate
It must be the goal of patient and nephrologist to build a
fruitful relationship with a dedicated access surgeon.
Right upper arm prosthetic brachial-basilic
access
Stenosis
Elbow
Shoulder
Chest
Arterial
anastomosis
Venous
anastomosis
Elbow Chest
PTFE ft i t d l th 3 th S t
Intraprocedure: Access flow before angioplasty measured
270 and 280ml/min, after angioplasty 1510 and 1560ml/min.
PTFE graft inserted less than 3 months ago. Spontaneous
bleeding from needle insertion sites during the night after
dialysis.
Copyright Harvard Medical School, 2010. All Rights Reserved.
1471
Location of Stenoses in Grafts
Courtesy TM Vesely
When to refer....?
1 Catheters 1. Catheters
flow / function / position
cuff-exposure
catheter damage / material breakdown
infection infection
Copyright Harvard Medical School, 2010. All Rights Reserved.
1472
When to refer....?
2. Non-maturing autogenous access
evaluate at 6-8 weeks
image early
in CKD 4 patients studies with limited
contrast safe (Kian Kidney International 2006)
Access may be lost during interventional procedure
- surgeon and patient need to be aware
When to refer....?
3. Mature Non-autogenous and autogenous
Accesses Accesses
CHANGE in physical exam
RECURRENT clinical abnormalities in access
use (bleeding, Venous pressure)
functional CONTINUOUS decrease in functional CONTINUOUS decrease in
objective parameters (K, URR, flow)
Copyright Harvard Medical School, 2010. All Rights Reserved.
1473
Access Flow and Intra-Access Pressure
are inversely correlated
Modified after Besarab Blood Purif 2006
Final thoughts on access flow...
Prosthetic accesses thrombose with increased
likelihood at lower flows likelihood at lower flows
referral in 600-700ml/min range
Autogenous Access
Outflow stenosis - prevent aneurysm and
thrombosis
Inflow stenosis - prevent thrombosis
Copyright Harvard Medical School, 2010. All Rights Reserved.
1474
Question 1
In your office you are examining a chronic
hemodialysis patient who had a new fistula
created 8 weeks ago. Which of the following
i f h h i l ? is not part of the physical exam?
1. Arm-Elevation Test
2. Allens Test
3. Audible Bruit along the
access
4. Palpable thrill
5. Augmentation
Question 1
In your office you are examining a chronic
hemodialysis patient who had a new fistula
created 8 weeks ago. Which of the following
i f h h i l ? is not part of the physical exam?
1. Arm-Elevation Test
2. Allens Test
3. Audible Bruit along the
access
4. Palpable thrill
5. Augmentation
Copyright Harvard Medical School, 2010. All Rights Reserved.
1475
Question 2
Which of the following has no influence on
successful use of a newly created fistula?
1. Assisted maturation
2. Hand exercise
3. Inhibitors of coagulation 3. Inhibitors of coagulation
4. Pre-operative mapping
5. Surgical experience
Question 2
Which of the following has no influence on
successful use of a newly created fistula?
1. Assisted maturation
2. Hand exercise
3. Inhibitors of coagulation 3. Inhibitors of coagulation
4. Pre-operative mapping
5. Surgical experience
Copyright Harvard Medical School, 2010. All Rights Reserved.
1476
Inflow - Outflow Balance
R L P
2 10 50 2.6 959.76
3.5 30 50 2.6 3000.4996875
3 40 50 2 6 1214 69625 3 40 50 2.6 1214.69625
3 30 50 2.6 1619.595
3 10 50 2.6 4858.785
3 5 50 2.6 9717.57
Disclosures
Consultant: Proteon Therapeutics
Off-label use of stents and stent-grafts
Copyright Harvard Medical School, 2010. All Rights Reserved.
1477
Peritoneal Dialysis: An
Overview
J oanne M Bargman MD
FRCPC
Director, Home Peritoneal Dialysis Unit
University Health Network, Toronto
Disclosure
Consultant Consultant
Amgen
Hospira
European Commission
Speaker Fees / Honoraria p
Amgen
Baxter Healthcare
Copyright Harvard Medical School, 2010. All Rights Reserved.
1478
Lecture Outline: Case-Based
Peritoneal transport
solute flux
ultrafiltration
Peritoneal dialysis solutions
dextrose-based
icodextrin icodextrin
Adequacy of PD
Approach to volume management
The Rapid Transporter so what?
A 67 year old woman with type II diabetes is y yp
started on peritoneal dialysis. Two months
later she undergoes a peritoneal
equilibration test (PET) that shows that the
D/P creatinine at 4 hours is 0.90 (high or
rapid).
Copyright Harvard Medical School, 2010. All Rights Reserved.
1479
The Rapid Transporter so what?
Which ONE of the following statements is
TRUE?
A. Since urea is smaller and more diffusible than
creatinine, the D/P urea will be lower.
B. The rapid transport status is a contraindication
to the use of intraperitoneal insulin for glycemic
control.
C. The PET test was performed too soon after the
start of PD.
D. There will be problems with ultrafiltration, D. There will be problems with ultrafiltration,
especially during the long dwell of dialysate.
E. Icodextrin is not useful for this type of transport
status.
The Peritoneal-Vascular Interface
dialysate
capillary
Peritoneal
membrane
Copyright Harvard Medical School, 2010. All Rights Reserved.
1480
The Peritoneal-Vascular Interface
(closeup)
dialysate
capillary
Peritoneal
membrane
The Peritoneal-Vascular Interface
Important transport occurs here
Dialysate
side
Blood
side
endothelium
interstitium
peritoneal
membrane
mesothelium
Copyright Harvard Medical School, 2010. All Rights Reserved.
1481
Solute Transport in PD
How does solute enter peritoneal fluid?
I. Diffusion
II. Convection (during ultrafiltration)
Principles of Solute Transport
Convection
Diffusion
Slide courtesy of Dr. O. Heimburger
Copyright Harvard Medical School, 2010. All Rights Reserved.
1482
Diffusion Kinetics - from blood to
dialysate
diffusive flux is highest in first hour and diffusive flux is highest in first hour and
lessens over time
by 4 hours, urea is >90% equilibrated,
creatinine > 60% equilibrated
further small solute removal is minimal u e s a sou e e o a s a
after that
long dwells more important for removal of
larger MW solutes
Diffusion Curves a Schema
Dialysate-to-plasma (D/P) ratios
urea
creatinine
middle
molecules
1.0
0.8
0.6
0.4
molecules
Dwell time (hours)
0.2
Copyright Harvard Medical School, 2010. All Rights Reserved.
1483
Diffusion Kinetics - from dialysate
to blood
What can you add to dialysate?
antibiotics (not just for peritonitis)
insulin
KCl (up to 10 mEq/l)
xylocaine, NaHCO3 (infusion pain)
metoclopramide erythromycin metoclopramide, erythromycin
(gastroparesis)
erythropoietin
Ultrafiltration in PD
done by osmotic pressure (compared to HD
where done by hydraulic pressure)
results of ultrafiltration:
fluid removal
convective removal of solutes, especially
middle molecules middle molecules
Copyright Harvard Medical School, 2010. All Rights Reserved.
1484
Composition of Peritoneal Dialysate:
Osmolality
1.5% dextrose - 347 mOsm/l (isotonic) ( )
2.5% dextrose - 397 mOsm/l
(hypertonic)
4.25% dextrose - 485 mOsm/l
(more hypertonic)
Ultrafiltration
E l 4 25% d t di l t Example: 4.25% dextrose dialysate
320 mOsm 480 mOsm
blood peritoneal cavity
Copyright Harvard Medical School, 2010. All Rights Reserved.
1485
Ultrafiltration Example of 4.25%
Dialysate
Water will move from lower to higher
osmolality
320 mOsm 480 mOsm
HH
22
00
blood peritoneal cavity
H20 H20
Ultrafiltration Example of 4.25%
Dialysate
Water will move from lower to higher
l lit d t k l t l osmolality and take solute along
320 mOsm 480 mOsm
H20
blood peritoneal cavity
H20
H20
Copyright Harvard Medical School, 2010. All Rights Reserved.
1486
Ultrafiltration
Glucose itself will diffuse out of peritoneal cavity
l it t ti di t along its own concentration gradient
320 mOsm 480 mOsm
blood peritoneal cavity
Ultrafiltration in PD
Ultrafiltration
volume
osmotic equilibrium
Time (hours)
0
Net
reabsorption
Copyright Harvard Medical School, 2010. All Rights Reserved.
1487
Typical Ultrafiltration in Peritoneal
Dialysis
1.5 % Dialysate
maximum UF 330 +/- 187 ml
time to maximum UF 140 +/- 48 minutes
4.25 % Dialysate
maximum UF 1028 +/- 258 ml
time to maximum UF 247 +/- 61
minutes
Typical Ultrafiltration Curves for Each
Strength of Dialysate

2200
2400
2600
2800
n
e
d

v
o
l
u
m
e
,

m
l
4.25% Dextrose
2.5% Dextrose
1.5% Dextrose
4.25%
1.5%
2.5 %
1600
1800
2000
100 200 300 400 500 600 700
Time in Minutes
D
r
a
i
n
840
APD
Night
Dwell
(2hrs
)
CAPD
Day
Dwells
(4hrs)
CAPD
Long
Dwell
(8hrs
)
APD
Long
Dwell
(14h)
Rippe, et.al.,, 2000.
Copyright Harvard Medical School, 2010. All Rights Reserved.
1488
Ultrafiltration in Peritoneal Dialysis
The amount of fluid ultrafiltered depends
upon: upon:
tonicity of dialysate
4.25% > 2.5% > 1.5%
duration of dialysate dwell
after osmotic equilibration, fluid starts to be
absorbed
permeabilityof peritoneal membrane to permeability of peritoneal membrane to
glucose
osmotic gradient dissipates faster across a more
permeable membrane
The Peritoneal Equilibration Test (PET):
A Way to Characterize the Peritoneal
Membrane
( ti i ) ( l )
At time t =0:
(creatinine) (glucose)
blood Peritoneal
cavity
Copyright Harvard Medical School, 2010. All Rights Reserved.
1489
The Peritoneal Equilibration Test (PET):
A Way to Characterize the Peritoneal Membrane
After 4 hours:
(creatinine) (glucose)
blood Peritoneal
cavity
The Peritoneal Equilibration Test
How easily does creatinine cross from y
blood to the peritoneal cavity?
quantified as Dialysate [creatinine]
Plasma [creatinine]
or
D/P creatinine (at T =4 hours) ( )
the leakier the peritoneal membrane, the
higher the D/P creatinine
Copyright Harvard Medical School, 2010. All Rights Reserved.
1490
The Peritoneal Equilibration Test (PET):
A Way to Characterize the Peritoneal Membrane
leaky peritoneal membrane leaky peritoneal membrane
(rapid transporter)
(creatinine) (glucose)
Higher
D/P
Creat.
blood Peritoneal
cavity
The Peritoneal Equilibration Test (PET):
A Way to Characterize the Peritoneal Membrane
tight peritoneal membrane tight peritoneal membrane
(slow transporter)
(creatinine) (glucose)
Lower
D/P
Creat
blood Peritoneal
cavity
Creat.
Copyright Harvard Medical School, 2010. All Rights Reserved.
1491
Peritoneal Equilibration Test
UREA
D/P
Creatinine
D/P
0.4
0.6
0.8
1
rapid
H. Ave
L. Ave
slow
0.4
0.6
0.8
1
1.2
Twardowski et al. Perit Dial Bull
0
0.2
0 1 2 3 4
0
0.2
0 1 2 3 4
Membrane Permeability and
Ultrafiltration - rapid transporters
the leakier the peritoneal membrane
( l b d ) (more vascular beds are open)
the faster glucose will diffuse out of the
peritoneal cavity
the faster the osmotic gradient will
dissipate
Copyright Harvard Medical School, 2010. All Rights Reserved.
1492
Why is Someone a Rapid
Transporter from the Start?
association with higher CRP, lower serum
albumin, less residual renal function
in some studies, more common in
diabetics
lower serum albumin seen before the start
f PD of PD
This suggests that rapid transporter status
may be a marker of inflammation
Membrane Permeability and
Ultrafiltration - slow transporters
the tighter the peritoneal membrane
the slower glucose will diffuse out of the
peritoneal cavity
the osmotic gradient will be maintained
longer
Copyright Harvard Medical School, 2010. All Rights Reserved.
1493
Transport Status Implications for
Ultrafiltration
Drain Volume
(2000 ml infused)
Creatinine
D/P
( )
1000
1500
2000
2500
3000
3500
ml
slow
L Ave
H Ave
Rapid
0.4
0.6
0.8
1
1.2
0
500
1000
0
0.2
0 1 2 3 4
Rapid vs Slow Transporters: Why
Solute Removal Isnt that Different
the better UF in the slow transporters will
increase solute removal through convective increase solute removal through convective
transport
D
C
small
solute
removal
C=convective flux
C
D
Rapid transporter Slow transporter
removal
D
D=diffusive flux
Copyright Harvard Medical School, 2010. All Rights Reserved.
1494
The High Transporter so what?
Which ONE of the following statements is TRUE?
A Si i ll d diff ibl th D/P A. Since urea is smaller and more diffusible, the D/P urea
will be lower than 0.90.
B. The high transport status is a contraindication to the
use of intraperitoneal insulin for glycemic control.
C. The PET test was performed too soon after the start
of PD.
D. There will be problems with ultrafiltration, especially
during the long dwell of dialysate.
E. Icodextrin is not useful for this type of transport
status.
Which ONE of the statements is FALSE?
a. Aquaporins are found in the endothelial cells at
the peritoneal-vascular interface. the peritoneal vascular interface.
b. Aquaporins are channels that allow water and
electrolyte flux through the cell membrane.
c. Aquaporin channels are responsible for the
sodium sieving found in peritoneal dialysis.
d. Intercellular small and large pores also
participate in transperitoneal flux participate in transperitoneal flux.
e. Fluid and macromolecules can leave the
peritoneal cavity by lymphatic absorption.
Copyright Harvard Medical School, 2010. All Rights Reserved.
1495
Transport in Peritoneal Dialysis
The Three Pore Model
Peritoneal Cavity Capillary
Transcellular pore (aquaporin)
Small pore
Endothelial cell
Large pore
The Concept of Sodium Sieving
blood dialysate
Endothelial cells
Copyright Harvard Medical School, 2010. All Rights Reserved.
1496
The Concept of Sodium Sieving
H2O
H2O
Water movement
through aquaporins
blood dialysate
The Concept of Sodium Sieving
H2O
H2O
Na
+
and
H
2
0 movement
through small pores
blood dialysate
Copyright Harvard Medical School, 2010. All Rights Reserved.
1497
Sodium Sieving
the fall in dialysate Na+ concentration y
over time is mainly the result of
dilution
dilution is the result of osmotically-driven
water-only transport across aquaporins
in the endothelium in the endothelium
sodium is held back or sieved at the
aquaporin
Sodium Sieving: The Fall in
Dialysate [Na+] vs. Time
115
120
125
130
135
140
S
o
d
i
u
m

m
e
q
/
L
High D/P Cr
Normal 2.5%D
Normal 4.25%D
110
115
0 30 60 90 120 150 180 210 240
Dwel l ti me (mi n)
S
(Heimburger)
Copyright Harvard Medical School, 2010. All Rights Reserved.
1498
The Consequence of Na+ Sieving: Rapid
Exchanges on APD Impair Na
+
Removal
100
150
200
250
CAPD
APD
mmol
0
50
Na removal
(Rodriguez-Carmona 2002)
Which ONE of the following is FALSE?
a. Aquaporins are found in the endothelial cells at
the peritoneal-vascular interface. the peritoneal vascular interface.
b. Aquaporins are channels that allow water and
electrolyte flux through the cell membrane.
c. Aquaporin channels are responsible for the
sodium sieving found in peritoneal dialysis.
d. Intercellular small and large pores also
participate in transperitoneal flux participate in transperitoneal flux.
e. Fluid and macromolecules can leave the
peritoneal cavity by lymphatic absorption.
Copyright Harvard Medical School, 2010. All Rights Reserved.
1499
Icodextrin: Which ONE of the statements
is FALSE?
a) The osmolality of icodextrin is
approximately that of plasma approximately that of plasma
b) Absorbed byproducts of Icodextrin can
falsely elevate blood glucose readings by
certain kinds of glucometers
c) Icodextrin is more effective than a 4.25%
dextrose solution for UF over a 4 h dwell
d) The UF is postulated to work by colloid
osmosis osmosis.
e) Icodextrin may be helpful for ultrafiltration
(UF) during PD peritonitis
Icodextrin
Mechanismof Action: Mechanism of Action:
colloid osmosis - analogous to the Starling
force of albumin causing fluid flux from the
interstitial to vascular compartment
P Plasma osm
=
Interstitial osm
Copyright Harvard Medical School, 2010. All Rights Reserved.
1500
Osmolality of the PD Solutions
m
o
l
a
r
i
t
y
(
m
O
s
m
/
K
g
)
600
500
400
300
200
346
396
485
282
Normal plasma
285 mOsm
O
s
m
100
0
Dextrose
1.5%
Dextrose
2.5%
Dextrose
4.25%
Icodextrin
Dextrose vs Icodextrin
Crystalloid osmosis with dextrose Crystalloid osmosis with dextrose
H20 movement H20 movement
lower osmolality
higher osmolality
Copyright Harvard Medical School, 2010. All Rights Reserved.
1501
Dextrose vs Icodextrin
Colloid osmosis with Icodextrin Colloid osmosis with Icodextrin
H20 movement
no significant aquaporin
ti ti ith i d t i
equal osmolality
equal osmolality
activation with icodextrin
Peritoneal Ultrafiltration: Glucose vs
Icodextrin (Computer Simulation)
4.25%
Rippe and Levin Kidney Int 2000
Copyright Harvard Medical School, 2010. All Rights Reserved.
1502
Remember the Hare and the
Tortoise?
the hare ran quickly the hare ran quickly
but eventually
got tired
the tortoise, slow
and steady, won y
the race
Remember the Hare and the
Tortoise?
the hare ran quickly q y
but eventually
got tired
the tortoise, slow
and steady, won
the race
dextrose
the race
icodextrin
Copyright Harvard Medical School, 2010. All Rights Reserved.
1503
Range of Ultrafiltration with
Icodextrin
Ultrafiltration
Absorption
11hrs
12hrs
13hrs
500 0 500 1000 1500
10hrs
Jeloka et al Perit Dial Int 2006
Maintenance of UF during Peritonitis
(Posthuma, 1997)
No
P
P
No
P
P
Copyright Harvard Medical School, 2010. All Rights Reserved.
1504
Icodextrin: Which ONE Statement is
FALSE?
a) The osmolality of Icodextrin is
approximately that of plasma approximately that of plasma
b) Absorbed byproducts of Icodextrin can
falsely elevate blood glucose readings by
certain kinds of glucometers
c) Icodextrin is more effective than a 4.25%
dextrose solution for UF over a 4 h dwell
d) The UF is postulated to work by colloid
i osmosis.
e) Icodextrin may be helpful for ultrafiltration
(UF) during PD peritonitis
Marvin on APD (Part I)
Marvin is a 35 year old man with chronic GN
h t t APD 2 5L X 3 h who starts on APD, 2.5L X 3 exchanges over
8 hours at night, last fill 2L. Residual renal
function is GFR 9 ml/min, U
out
960 ml/24h.
Typical UF on the cycler is 800 ml, average
initial drain volume of his day dwell is 1700
ml when he goes on the cycler at night ml when he goes on the cycler at night.
Copyright Harvard Medical School, 2010. All Rights Reserved.
1505
Which ONE Statement is FALSE?
a) He is protected from ECF volume overload in
part by the residual urine volume.
b) He probably has borderline adequacy and
should have his dialysis prescription increased,
or be converted to hemodialysis.
c) The duration of the day dwell contributes to
absorption of dialysate, accounting for the low
drain volume.
d) He should be advised to avoid nephrotoxic
insults such as NSAIDs and COX-2 inhibitors insults, such as NSAIDs and COX-2 inhibitors.
e) Eight hours of APD is appropriate for many
patients.
Which ONE Statement is FALSE?
a) He is protected from ECF volume overload in
part by the residual urine volume. p y
b) He probably has borderline adequacy and
should have his dialysis prescription increased,
or be converted to hemodialysis.
c) The duration of the day dwell contributes to
absorption of dialysate, accounting for the low
initial drain volume.
d) He should be advised to avoid nephrotoxic
insults such as NSAIDs and COX 2 inhibitors insults, such as NSAIDs and COX-2 inhibitors.
e) Eight hours of APD is appropriate for many
patients.
Copyright Harvard Medical School, 2010. All Rights Reserved.
1506
Adequacy of Dialysis in PD
The strength of PD lies in
continuous therapy 24/7
preservation of RRF compared to HD
good middle molecule clearance (by RRF
and the peritoneal membrane)
None of these is adequately None of these is adequately
measured by Kt/V urea
Adequacy of Dialysis in PD
randomized, controlled trials have not shown ,
a survival benefit for any Kt/V urea >1.5
lower limit for Kt/V urea unknown
Copyright Harvard Medical School, 2010. All Rights Reserved.
1507
Adequacy of Dialysis in PD
The KDOQI Guidelines 2006
minimum total (renal +peritoneal) Kt/V urea
of 1.7
monitor and protect RRF
careful attention to volume status
trial of increaseddialysis is indicatedif trial of increased dialysis is indicated if
patient not doing well without another
explanation
Fluid Balance
I t k
Na+ and water Urine and UF
Intake
Output
==
Ultrafiltration
RRF
Copyright Harvard Medical School, 2010. All Rights Reserved.
1508
Fluid Output in PD
Ultrafiltration
RRF
Volume Overload
Intake
excessive salt and water consumption
Output
loss of residual renal function
use of the wrong dialysis fluid
failure of peritoneal membrane to respond failure of peritoneal membrane to respond
(true ultrafiltration failure)
mechanical problems like leaks
Copyright Harvard Medical School, 2010. All Rights Reserved.
1509
Volume Overload Volume Overload
Intake Intake excessive salt and water excessive salt and water
consumption consumption
PD has often been advertised as PD has often been advertised as
allowing a more liberal dietary intake allowing a more liberal dietary intake
patients with high salt intake are protected patients with high salt intake are protected
from volume overload while they have from volume overload while they have
id l l f ti (RRF) id l l f ti (RRF) residual renal function (RRF) residual renal function (RRF)
once urine volume diminishes, once urine volume diminishes,
patient may develop fluid overload patient may develop fluid overload
Volume Overload
Output: Loss of Residual Renal Function p
probably the commonest cause of
progressive fluid overload
rate of loss of RRF is variable and
unpredictable from patient to patient
Copyright Harvard Medical School, 2010. All Rights Reserved.
1510
Volume Overload
Loss of Residual Renal Function
protect RRF
avoid NSAIDs, COX 2-inhibitors, dye studies,
aminoglycosides, volume depletion
use diuretics to augment urine Na
+
&
water output
eg furosemide metolazone eg furosemide, metolazone
continue immunosuppression for failed
transplant kidneys that still have function
Volume Overload
Use of the wrong type of PD fluid Use of the wrong type of PD fluid
usually this means failure to account for
the long dwell
Copyright Harvard Medical School, 2010. All Rights Reserved.
1511
Temporal Profiles of APD and CAPD
Cycle
3
Cycle
2
Cycle
4
Cycle
1
*
APD
*
nighttime (9 hrs) daytime (15 hrs)
*
CAPD
Volume Overload
Tacklingthe long dwell: Tackling the long dwell:
1.use icodextrin or a more hypertonic
dialysate (e.g. 2.5%)
2. break up the long dwell
day dry (only if there is a lot of RRF)
mid day exchange in APD mid-day exchange in APD
drain out day exchange in APD after a few
hours
Copyright Harvard Medical School, 2010. All Rights Reserved.
1512
Fluid Absorption During the Long
Dwell
Or, it may not need any intervention
if there is a lot of urine volume, may forgive
fluid absorption
e.g. patient on APD
last fill 2L
initial drain 1.5 L (so .5L fluid absorption) initial drain 1.5 L (so .5L fluid absorption)
urine output 1.0 L
patient is clinically euvolemic
No Need to Change the Prescription
Volume Overload
Output dependent
failure of the peritoneal membrane to
respond to UF conditions (true UF failure)
mechanical failure of dialysis procedure
Copyright Harvard Medical School, 2010. All Rights Reserved.
1513
Ultrafiltration Failure: Peritoneal
Membrane Failure
Definition: Inability to maintain volume Definition: Inability to maintain volume
homeostasis despite the use of
hypertonic dialysate solutions (3 or
more daily)
or
Failure to ultrafilter >400 ml using a
4.25% bag for 4 hours (the Rule of 4s)
Ultrafiltration Failure
on PET test, D/P creatinine is high g
these high transporters have rapid
absorption of glucose across peritoneal
membrane
rapid dissipation of osmotic gradient
poor ultrafiltration
Copyright Harvard Medical School, 2010. All Rights Reserved.
1514
Ultrafiltration Failure
Management of rapid transporters (I):
reinforce salt and water restriction
use more hypertonic dialysate
icodextrin can be quite helpful here (as
effective in high transporters as other
transport types) p yp )
Icodextrin: Similar Ultrafiltration in
All Transporter Types
1000
L
-200
200
600
N
e
t

U
F

(
m
L
)
Low
Low-Avg
High-Avg
High
CAPD
Overnight
APD
D ti
-600
0 2 4 6 8 10 12 14 16
Time (hr)
Daytime
Mujais S, Vonesh E. Kidney Int. 2002;62(suppl 81):S17-S22.
Copyright Harvard Medical School, 2010. All Rights Reserved.
1515
Ultrafiltration Failure
Management of rapid transporters(II):
h id l i t t (di ti ) push residual urine output (diuretics)
APD with dry day, or drain out last fill at lunch
(if enough RRF)
once anuric, watch closely for volume
overload
id t f t h di l i if consider transfer to hemodialysis if
patient is chronically overloaded (start
talking about fistula placement with the patient)
Volume Overload
Output dependent p p
mechanical failure of dialysis procedure
Copyright Harvard Medical School, 2010. All Rights Reserved.
1516
Part II Marvin Gets Puffy
1 year later, Marvin comes to clinic y
complaining of increasing ankle edema.
The BP, which had been normal, is
now 150/100.
The dialysis prescription is unchanged.
Serumcreatinine which had been 10 6 Serum creatinine, which had been 10.6
mg/dl at the start of dialysis, is now
13.8 mg/dl.
Which ONE fits Marvin BEST?
a) The increased serum creatinine reflects a
failure of solute transport across the
it peritoneum.
b) He most likely has peritonitis and the
acquisition of a high transporter state.
c) The new onset hypertension is likely the
result of acquired renal cystic disease.
d) Both the increased serum creatinine and
i h l d b l i d b peripheral edema can be explained by
decreased residual renal function.
Copyright Harvard Medical School, 2010. All Rights Reserved.
1517
Which ONE fits Marvin BEST?
a) The increased serum creatinine reflects a
failure of solute transport across the
it peritoneum.
b) He most likely has peritonitis and the
acquisition of a high transporter state.
c) The new onset hypertension is likely the
result of acquired renal cystic disease.
d) Both the increased serum creatinine and
i h l d b l i d b peripheral edema can be explained by
decreased residual renal function.
Part II Marvin Gets Puffy
How to help Marvin (APD 2.5L x3, 2L
l t fill) last fill)
dietary salt restriction
push diuretics
last fill:
mid-day exchange, or
icodextrin last fill, or
both
Copyright Harvard Medical School, 2010. All Rights Reserved.
1518
Summary of Important Points
Transport characteristics determined by a
Peritoneal Equilibration Test
rapid transporter has increased
peritoneal vascularity and transports small
solutes quickly; but loses glucose osmotic
gradient quickly and so has problems with
ultrafiltration
slow transporter has slower removal of
small solutes but excellent ultrafiltration
PD peritonitis can lead to transient rapid
transporter state because of inflammation
Summary of Important Points
in all PD patients (except rapid transporters)
short PD dwells leads to removal of more short PD dwells leads to removal of more
water than sodium
avoid short dwells except in rapid
transporters
residual renal function is a more important
predictor of outcome than dose of PD
d b ll l t ki ti measured by small solute kinetics
try to protect residual function
dont obsess about Kt/V get at least to
minimum target and obsess about RRF and
volume status
Copyright Harvard Medical School, 2010. All Rights Reserved.
1519
Disclosure
Consultant Consultant
Amgen
Hospira
European Commission
Speaker Fees / Honoraria p
Amgen
Baxter Healthcare
Copyright Harvard Medical School, 2010. All Rights Reserved.
1520
Dialysis Pearls for the Boards
J Ke in T cker MD J . Kevin Tucker, M.D.
Brigham and Womens Hospital
Massachusetts General Hospital
Harvard Vanguard Medical
Associates Associates
Conflicts of Interest
Consultant Consultant
Baxter Healthcare
Genzyme
Copyright Harvard Medical School, 2010. All Rights Reserved.
1521
Case 1
Patient BJ is a 33 year oldwoman with Patient BJ is a 33-year-old woman with
ESRD secondary to diabetic
nephropathy. Her other medical
problems include hypertension, HIV,
peripheral vascular disease, and
h h idi Sh h b hyperparathyroidism. She has been on
hemodialysis for about 3 years.
Case 1
Dialysis Prescription
Left arm AV fistula
Optiflux 200
4 hours
Qb 450 mL/min
Qd A1 5 Qd A1.5
2.0K+, 2.5 Ca++, 35 HCO3-
Dry weight 69 kg
Copyright Harvard Medical School, 2010. All Rights Reserved.
1522
Case 1
Medications
TMP-SMX 1 po qod
Labetalol 100 mg po bid
TMP SMX 1 po qod
Epivir 50 mg po q d
Ziagen 300 mg po q d
Kaletra 200-50 mg po q
d
Mirtazapine 30 mg po
Labetalol 100 mg po bid
Nifedipine 90 mg po q d
Nephrocaps
Calcium acetate 667
mg tid with meals
Cinacalcet 30 mg po q
d
qhs
Losartan 100 mg po q d
Doxercalciferol (IV)
EPO (IV)
Monthly labs
spKt/V 1 47 sp Kt/V 1.47
Hemoglobin 12.5 g/dL
Albumin 3.1 g/dL
Calcium 8.0 mg/dL
Phosphorous 8 1 mg/dL Phosphorous 8.1 mg/dL
Intact PTH 1083 pg/mL
Copyright Harvard Medical School, 2010. All Rights Reserved.
1523
Case 1
At her outpatient dialysis unit the nurse notes At her outpatient dialysis unit, the nurse notes
that BJ has been developing paroxysmal
coughing about 30 minutes to one hour into
her treatment. The coughing spells are so
distressing that she has signed off dialysis
early on several occasions. Her pre-dialysis y p y
systolic blood pressure, which is usually 120-
140 mm Hg, rises to more than 200 mm Hg
during these episodes.
What is the cause of her
symptoms?
Differential diagnosis Differential diagnosis
Hypertension-induced pulmonary edema
Pneumonia
Allergic reaction associated with the
dialysis procedure
Heparin
Dialyzer reaction
Copyright Harvard Medical School, 2010. All Rights Reserved.
1524
Hypertension-induced
pulmonary edema
Blood pressure was well controlled prior Blood pressure was well controlled prior
to dialysis.
No coughing or shortness of breath
prior to or after dialysis
Pneumonia
Patient is immunocompromisedbut Patient is immunocompromised but . . .
No fever
No symptoms except on dialysis
Copyright Harvard Medical School, 2010. All Rights Reserved.
1525
Allergic reaction
No wheezing or urticaria No wheezing or urticaria
Not receiving heparin
Dialyzer: biocompatible, polysulfone,
ETO sterilized
Case 1
Patients dialyzer was changed fromthe Patients dialyzer was changed from the
Optiflux 200 to the AM BIO HX 100. The first
3 times she was dialyzed with the AM BIO HX
100, she had no symptoms, but on the fourth
session, she developed coughing, wheezing,
and shortness of breath. She was then tried
on the CT190G and the first time she was on the CT190G, and the first time she was
tried on that dialzyer, she had her most
severe symptoms, including wheezing, such
that the treatment had to be terminated after
30 minutes.
Copyright Harvard Medical School, 2010. All Rights Reserved.
1526
Case 1
At that point no other dialyzers were At that point, no other dialyzers were
available in the outpatient unit, and she
was referred to a hospital dialysis unit
for further treatments.
Dialyzer Symptoms Severity Onset
O tifl C h ++ 1 h Optiflux
200
Cough
Dyspnea
++ 1 hour
BIO HX
100
Cough
Dyspnea
+ 1 hour
CT 190G Cough ++++ 10-15 g
Dyspnea
Wheezing
minutes
Copyright Harvard Medical School, 2010. All Rights Reserved.
1527
Characteristics
Optiflux AMBIO CT 190G Optiflux
200
AM BIO
HX 100
CT 190G
Membrane
Polysulfone Polysulfone
Cellulose
triacetate
Membrane
Type
Synthetic Synthetic
Substituted
cellulose
Type
cellulose
Sterilization
ETO Gamma Gamma
Mechanisms of dialyzer reactions
Complement activation
Pulmonary leukostasis
Hypersensitivity to ETO
Interaction between ACE-I and Interaction between ACE I and
polyacrylonitrile membranes
Dialysate contamination
Copyright Harvard Medical School, 2010. All Rights Reserved.
1528
Complement activation
Unsubstitutedcellulose dialyzers cause Unsubstituted cellulose dialyzers cause
complement activation, leading to a
syndrome of symptoms on dialysis.
These reactions would often abate with
dialyzer re-use.
Hypersensitivity to ETO
Ethylene oxide (ETO) is a sterilant used Ethylene oxide (ETO) is a sterilant used
in dialyzer manufacturing.
Hypersensitivity to ETO may lead to an
allergic reaction shortly after the patient
is begun on dialysis.
Thorough rinsing of the dialyzer with Thorough rinsing of the dialyzer with
saline may in some instances prevent
the reaction.
Copyright Harvard Medical School, 2010. All Rights Reserved.
1529
Pulmonary leukostasis
Complement activation by dialyzer Complement activation by dialyzer
membranes may lead to pulmonary
leukostasis, thus causing dyspnea.
ACE-I and polyacrylonitrile
membranes
Polyacrylonitrile (PAN) membranes Polyacrylonitrile (PAN) membranes
stimulate bradykinin production.
ACE-I inhibit bradykinin degradation.
Use of ACE-I with PAN dialyzers is
associated with anaphylactoid assoc a ed a ap yac od
reactions.
Copyright Harvard Medical School, 2010. All Rights Reserved.
1530
Dialysate contamination
Endotoxins and other bacterial products Endotoxins and other bacterial products
can move across the dialyzer
membrane via back-transport,
activating cytokines and causing
inflammatory symptoms.
Dialyzer reactions
Type A reactions
Within the first 30
minutes; often
Type B reactions
Later in the course of
dialysis but within first
immediately after
initiation of dialysis
Symptoms: itching,
urticaria, wheezing,
dyspnea, chest pain,
back pain, hypotension
IgE mediated; in
response to ethylene
15-30 minutes
Symptoms improve over
the course of dialysis
Symptoms: chest pain,
back pain, nausea, and
vomiting
Pathophysiology:
Complement activation
p y
oxide, a sterilant
Rx: Terminate dialysis;
steroids, bronchodilators,
etc.
Complement activation
More common with less
biocompatible
membranes
Generally do not recur
with re-use
Daugirdas and Ing KI 24:S37-S43, 1988
Copyright Harvard Medical School, 2010. All Rights Reserved.
1531
Incidence of dialyzer reactions
True incidence difficult to gauge given True incidence difficult to gauge given
non-uniformity in classification and
diagnosis
Reported incidence:
3.5 per 100000 for severe reactions 3 5 pe 00000 o se e e eac o s
3-5 per 100 for type B reactions
Yang and Lindsay, Hemodialysis International 9: 120-126, 2005
Case 1
Patient had symptoms with both ETO Patient had symptoms with both ETO
and gamma sterilized dialyzers.
She had early-onset and late-onset
symptoms.
She had symptoms with both synthetic S e ad sy p o s bo sy e c
and cellulosic dialyzers.
Copyright Harvard Medical School, 2010. All Rights Reserved.
1532
Case 1
Outcome
Patient was sent to a local hospital for further Patient was sent to a local hospital for further
dialysis until her dialysis regimen was stable.
She was dialyzed with the AM BIO HX 100
following pre-treatment with corticosteroids,
diphenydramine, and ranitidine. With this
regimen she was symptom free.
Sh t d t th t ti t it She returned to the outpatient unit on a
tapering dose of oral steroids, oral ranitidine,
and IV diphenhydramine and has been
without symptoms.
Case 2
KDis a 31-year-old woman with ESRD who KD is a 31 year old woman with ESRD who
started hemodialysis about 6 months ago.
She had first been diagnosed with renal
disease following the birth of twins 6 years
prior. A biopsy had shown focal segmental
glomerulosclerosis with a chronic thrombotic
microangiopathy She was lost to followup microangiopathy. She was lost to followup
and presented with uremic symptoms and a
creatinine of 7.6 mg/dL. A tunneled catheter
was placed, and the patient was begun on
hemodialysis.
Copyright Harvard Medical School, 2010. All Rights Reserved.
1533
Past Medical History
Chronic kidney disease with nephrotic Chronic kidney disease with nephrotic
syndrome
Diabetes mellitus type 2
Hypertension
Medications
ASA 81 mg po q day ASA 81 mg po q day
Insulin
Losartan 100 mg po q day
Nephrocaps 1 po q day
Calciumacetate 667 mg po tid Calcium acetate 667 mg po tid
EPO
Copyright Harvard Medical School, 2010. All Rights Reserved.
1534
Dialysis Prescription
Tunneled catheter Tunneled catheter
Optiflux 200 NR
4 hours
Qb 400 mL/min
Qd A1.5
3 0K+ 2 5 Ca++ 3.0K+, 2.5 Ca++
Heparin 3000 units IV bolus
Dry weight 101 kg
Labs at initiation of dialysis
Hemoglobin 11 0 g/dL Hemoglobin 11.0 g/dL
Platelets 349K
Potassium 4.4 mEq/L
Albumin 3.7 g/dL
Calcium 8 2 mg/dL Calcium 8.2 mg/dL
Phosphorous 7.1 mg/dL
Copyright Harvard Medical School, 2010. All Rights Reserved.
1535
Case 2
Patient is referredfor an AV fistula The Patient is referred for an AV fistula. The
nephrologist receives a call from the
surgeon stating that her surgery is being
cancelled because of thrombocytopenia
(platelet count 64K). She is referred to a
h l i h h h hematologist who requests that the
patient not receive any heparin for
presumed HIT.
HIT Clinical presentation
Decrease in platelet count of 30-50% or more
from pre-treatment levels
Thrombocytopenia usually develops between
the 5th and 14th day of therapy
Onset may be much more rapid in patients
who received an earlier course of heparin
(within preceding 100 days)
Elderly patients undergoing orthopedic or
cardiovascular surgery appear to be at
greatest risk.
Copyright Harvard Medical School, 2010. All Rights Reserved.
1536
Clinical consequences of HIT
Thrombosis is the major complication of Thrombosis is the major complication of
HIT.
Both arterial and venous thromboses
are associated with HIT.
Venous thrombosis is the most common
clinical complication clinical complication.
Pulmonary embolism is the most
common fatal complication.
Epidemiology
Incidence varies depending upon study Incidence varies depending upon study
design, route of heparin administration, dose
of heparin, and type of heparin.
Incidence <5% with IV bovine heparin; <1%
with IV porcine heparin and <1% in
individuals on heparin prophylaxis
Th b ti li ti i 20 30% f Thrombotic complications occur in 20-30% of
patients with HIT.
Differing patient populations have different
risk for HIT, e.g, orthopedic patients (4.9%) >
cardiac surgery patients (1%).
Copyright Harvard Medical School, 2010. All Rights Reserved.
1537
Studies of HIT in dialysis patients
Country #patients
Thrombotic
events
Thrombocytopenia
J apan 154 ? Yes J apan 154 ? Yes
Germany 165 No No
Germany 70 No No
Italy 50 No 1 of 6 with
antibodies
Netherlands
261 No No
US 88 No No
From Reilly, Seminars in Dialysis 16: 54-60, 2003
Clinical scoring system
(Greinacher)
Criterion Score
Decrease in platelet count by
30-50%
+1
Decrease in count by >50% of
pretreatment value
+2
Onset >5 days in case of first
exposure
+2
exposure
Onset <4 days in case of re-
expsoure
+2
Thromboembolic complications
during heparin treatment
+2
Copyright Harvard Medical School, 2010. All Rights Reserved.
1538
Clinical scoring system
(Greinacher)
Inflammatory skin reaction at +1 y
injection site
Normalization of platelets within
10 days of stopping heparin
+2
Septic events at time of
diagnosis
-1
Recent treatment with cytotoxic
or cytostatic drugs
-1
or cytostatic drugs
Pre-existing thromboembolic
complications
-1
Clinical scoring system
(Greinacher)
0 3HIT unlikely 0-3 HIT unlikely
4-5 HIT possible
6-8 HIT very likely
Greinacher et al, Transfusion 34: 381-385, 1994
Copyright Harvard Medical School, 2010. All Rights Reserved.
1539
Functional HIT Antibody
Assays
HIT antibodies can be detected using both HIT antibodies can be detected using both
functional and antigenic assays.
Functional assays rely upon the ability of HIT
IgG to activate platelets via their FcIIA
receptors.
Functional assays include the
14
C-serotonin Functional assays include the C-serotonin
release assay, heparin-induced platelet
activation assay, and platelet aggregation
test.
Antigenic HIT Antibody
Assays
Antigenic assays detect HIT-Ig binding to the Antigenic assays detect HIT-Ig binding to the
heparin PF4 target antigen by ELISA, biotin-
labeled PF4, or fluorescent-labeled heparin.
ELISA most commonly used
ELISA has poor specificity: antibodies
detected in many patients without detected in many patients without
thrombocytopenia or thromboembolic
complications.
Copyright Harvard Medical School, 2010. All Rights Reserved.
1540
Indications for consideration of
alternative anticoagulants
Thrombocytopenia Thrombocytopenia
Systemic thrombosis
Excessive clotting of the extracorporeal
circuit
Vascular access thrombosis Vascular access thrombosis
Direct thrombin inhibitors
Inhibit thrombin generation by binding Inhibit thrombin generation by binding
with the active site of thrombin,
preventing interaction with substrate
Do not require a plasma cofactor such
as antithrombin III
Able to inactivate soluble and fibrin clot Able to inactivate soluble and fibrin-clot
bound thrombin
Copyright Harvard Medical School, 2010. All Rights Reserved.
1541
Direct thrombin inhibitors
Lepirudin argatroban and bivalirudin Lepirudin , argatroban and bivalirudin
available in the US
Lepirudin and bivalirudin are both partially
renally excreted and require careful
monitoring in renal impairment.
Argatroban is hepatically metabolized Argatroban is hepatically metabolized.
Case reports and small series of direct
thrombin inhibitors for dialysis anticoagulation
Lepirudin Argatroban
Mechanism Direct
thrombin
inhibitor
Direct
thrombin
inhibitor
Cost AWP
$138/50
AWP
$750/100 $138/50 mg $750/100 mg
Copyright Harvard Medical School, 2010. All Rights Reserved.
1542
Case 2
Outcome
Patient was dialyzed without heparin Patient was dialyzed without heparin.
Catheter was locked with a citrate
solution.
Heparin antibodies were negative.
Platelet counts fromdialysis laboratory Platelet counts from dialysis laboratory
have all been > 100 K.
Case 3
CS is a 34 year oldman with ESRD CS is a 34-year-old man with ESRD
secondary to FSGS. He has been on
hemodialysis for about 6 years.
Copyright Harvard Medical School, 2010. All Rights Reserved.
1543
Case 3
Past Medical History
Medications
Past Medical History
FSGS
Hypertension
Renal cell carcinoma
s/p bilateral
nephrectomies
Medications
Calcium carbonate
with meals
Multivitamin
EPO IV
Hyperparathyroidism
s/p parathyroidectomy
Paracalcitol IV
Case 3
Dialysis Prescription
AV fistula AV fistula
Polyflux 210
4.5 hours
Qb 500 mL/min
Qd 800 mL/min
2 0K+ 2 5 Ca++ 2.0K+, 2.5 Ca++
Heparin 7000 units IV bolus; 800 units/hour
Dry weight 108 kg
Copyright Harvard Medical School, 2010. All Rights Reserved.
1544
Case 3
Monthly Labs
Hemoglobin 12 1 g/dL Hemoglobin 12.1 g/dL
Calcium 8.7 mg/dL
Phosphorous 8.6 mg/dL
Albumin 4.5 g/dL
Intact PTH 266 pg/mL Intact PTH 266 pg/mL
sp Kt/V 1.37
Case 3
Patient was in his usual state of good Patient was in his usual state of good
health until the day prior to admission
when he developed abdominal
discomfort and chills during dialysis. He
stayed for his entire treatment, went
home, and went to bed. He developed o e, a d e o bed e de eoped
nausea, vomiting, and diarrhea, and
presented to the Emergency
Department at his mothers insistence.
Copyright Harvard Medical School, 2010. All Rights Reserved.
1545
Case 3
Emergency Department Labs
WBC 21 2K WBC 21.2K
Hematocrit 13%
Platelets 288K
Potassium 6.7 mEq/L
Total bilirubin 11 7 Total bilirubin 11. 7
LDH 7689
Phosphorous < 1.0 mg/dL
Symptoms/signs of hemolysis in
dialysis patients
Hypertension Hypertension
Chest pain
Abdominal pain
Nausea/vomiting
Shortness of breath
Back pain Back pain
Diarrhea
Generalized erythema
Port-wine appearance of blood in venous line
Spry, Seminars in Dialysis 12:205, 1999
Copyright Harvard Medical School, 2010. All Rights Reserved.
1546
Management
When hemolysis is suspected on dialysis: When hemolysis is suspected on dialysis:
Blood pump should be stopped.
Blood lines clamped.
Potassium rich blood should not be
reinfused.
Tubing should be saved to check for Tubing should be saved to check for
defects.
Dialysate sample should be kept for
analysis.
Dialysis-related causes of
hemolysis
Copper Copper
Chlorine, Chloramine
Nitrates, nitrites
Overheated dialysate
Hypo osmoloar dialysate Hypo-osmoloar dialysate
Kinked or defective tubing
Formaldeyhde
Copyright Harvard Medical School, 2010. All Rights Reserved.
1547
Other causes of hemolysis
Medications Medications
G6PD deficiency
Hypersplenism
Sickle cell anemia and other
hemoglobinopathies hemoglobinopathies
Hypophosphatemia
Multistate Outbreak of
Hemolysis, 1998
30 patients in 3 states developed 30 patients in 3 states developed
hemolysis with or without chest pain,
shortness of breath, nausea, or
abdominal pain, while on hemodialysis
2 deaths were associated with this
outbreak.
MMWR 47: 483-484
Copyright Harvard Medical School, 2010. All Rights Reserved.
1548
Nebraska outbreak of hemolysis
Case defined as Case defined as
Hypertension (increase in SBP of >30 mm Hg
above baseline)
Evidence of hemolysis (+pink test)
13 of 118 patients at two dialysis centers in
Lincoln met the definition
O t f t d t di f Onset of symptoms occurred at a median of
120 minutes (range 20-272) into the session.
Case patients ranged in age from 46 to 84.
All patients were dialyzed using the same
tubing lot.
Maryland outbreak of hemolysis
12 of 298 patients at four hemodiaysis clinics 12 of 298 patients at four hemodiaysis clinics
developed abdominal pain, nausea, and/or
erythroderma.
All had evidence of hemolysis upon hospital
admission.
Symptom onset: 114 minutes (range 22-227)
i t th t t t into the treatment.
All were dialyzed with tubing from the same
lot.
Copyright Harvard Medical School, 2010. All Rights Reserved.
1549
Multistate outbreak of hemolysis
Investigation
Examination of implicated blood tubing Examination of implicated blood tubing
revealed narrowing of the aperture through
which blood was pumped during the
treatment.
Analysis of water at one of the hemodialysis
centers in Nebraska was within normal limits centers in Nebraska was within normal limits
for chlorine, chloramine, endotoxin, bacteria
and trace elements as defined by AAMI
standards.
Case 3
Outcome
Patient was admitted to the intensive Patient was admitted to the intensive
care unit. He was transfused 6 units of
PRBCs and given IV phosphorous, and
over the next 24-48 hours, his signs of
hemolysis resolved and his hematocrit
i d bl Bl d l remained stable. Blood cultures were
negative. Stool cultures showed no
pathogens.
Copyright Harvard Medical School, 2010. All Rights Reserved.
1550
Case 3
Outcome
Tests for G6PD deficiency were negative Tests for G6PD deficiency were negative.
The dialysis provider removed the lot of
tubing that was used on that patient. Tests of
dialysate did not reveal any contaminants that
would explain hemolysis. The hemodialysis
machine on which he dialyzed was tested machine on which he dialyzed was tested
and found to be fully functional.
Conflicts of Interest
Consultant Consultant
Baxter Healthcare
Genzyme
Copyright Harvard Medical School, 2010. All Rights Reserved.
1551
Complications of Peritoneal
Dialysis
Joanne Bargman MD FRCPC
Professor of Medicine, University of Toronto
Director, the Home Peritoneal Dialysis
Program
University Health Network
Toronto
Disclosure
Consultant Consultant
Amgen
Hospira
European Commission
Speaker Fees / Honoraria p
Amgen
Baxter Healthcare
Copyright Harvard Medical School, 2010. All Rights Reserved.
1552
Why Choose PD?
better quality of life compared to in-centre better quality of life, compared to in centre
hemodialysis
daily dialysis
keeps the patient at home
patient autonomy
better preservation of residual kidney function
may even be renoprotective
Why Not Choose PD?
more work to establish the patient on PD more work to establish the patient on PD
than hemodialysis
catheter placement
patient training
dialysis prescription
constant glucose loading constant glucose loading
can be a good thing or a bad thing
limited technique survival in many centers
Copyright Harvard Medical School, 2010. All Rights Reserved.
1553
Mrs. B.D. Karma
33 year old woman with postpartumrenal failure 33 year old woman with postpartum renal failure
no appropriate kidney donor
attends dialysis education class and chooses home
peritoneal dialysis
PD catheter inserted by blind surgical technique
Mrs. B.D. Karma
comes to peritoneal dialysis unit to begin training comes to peritoneal dialysis unit to begin training
dialysate infuses easily, but very slow outflow
first 500 mls takes about 30 minutes, and then the flow
stops
Copyright Harvard Medical School, 2010. All Rights Reserved.
1554
Catheter-Related Problems
Usually soon after implantation: Usually soon after implantation:
2-way obstruction
problem with inflow and outflow
1-way obstruction
good inflow poor outflow good inflow, poor outflow
painful inflow or outflow
What is Not Necessarily a Problem
First ever exchange: First ever exchange:
1 liter good inflow
500 ml good outflow
WHY? WHY?
WHAT WOULD YOU NEXT?
Copyright Harvard Medical School, 2010. All Rights Reserved.
1555
Catheter-Related Problems
2-way obstruction
kink or bend in catheter
intraluminal obstruction (blood clot, fibrin)
treatment:
radiologic insertion of trochar to straighten
catheter catheter
vigorous flush with heparinized saline,
insertion of brush into catheter lumen
Fibrin Plug
Copyright Harvard Medical School, 2010. All Rights Reserved.
1556
Catheter-gram: Kink
Courtesy Dr. Martin Simons
Catheter-gram: Un-kinked
Courtesy Dr. Martin Simons
Copyright Harvard Medical School, 2010. All Rights Reserved.
1557
Catheter-Related Problems
1-way obstruction
constipation ***
catheter migration into upper quadrants or into a
loculated pocket
omental wrap
treatment
careful attention to bowel cleanout ***
radiologic or laparoscopic manipulation of
catheter
surgical removal of omentumor laparascopic
omentopexy
Omental Wrap
Slide courtesy of Dr. J . Crabtree
Copyright Harvard Medical School, 2010. All Rights Reserved.
1558
Mrs. B.D. Karma
Her abdominal flat plate shows: Her abdominal flat plate shows:
tons of stool in the large bowel
catheter tip in the left middle quadrant
Mrs. Karma
she is given lactulose and a tap water she is given lactulose and a tap water
enema, and you are told that there
are good results
Are you happy?
a. Yes
b. No
Copyright Harvard Medical School, 2010. All Rights Reserved.
1559
Mrs. Karma
she is sent home for the weekend with she is sent home for the weekend with
lactulose and senna
over the weekend she has 8 more bowel
movements
still no improvement in outflow
Catheter-grams
inject enough dye to fill catheter and spill inject enough dye to fill catheter and spill
into peritoneal cavity
what you can diagnose:
kink in catheter
obstruction within lumen
omental wrap omental wrap
peritoneal compartment problem
Copyright Harvard Medical School, 2010. All Rights Reserved.
1560
Catheter-gram: Compartment
Problem
Courtesy Dr. Martin Simons
Mrs. B.D. Karma
Catheter-gramsuggested omental wrap around Catheter gram suggested omental wrap around
catheter
discussed with patient: patient chose to continue
with PD
laparoscopic removal of omentumfrom around
catheter
Fi ll th t tfl i d d ti ll ! Finally: catheter outflow improved dramatically!
Copyright Harvard Medical School, 2010. All Rights Reserved.
1561
Approach to Post-Insertion Catheter
Dysfunction
Find out if it is one-way or two-way Find out if it is one-way or two-way
obstruction
If two-way: flat plate +/- catheter dye study
(kink, intraluminal clot)
If one-way: abdo flat plate
you will likely see +++stool and catheter may or you will likely see +++stool and catheter may or
may not be out of the pelvis
Approach to Post-Insertion Catheter
Dysfunction (contd)
First of all: clean out the bowels (this may
entail a lot of bowel movements!) entail a lot of bowel movements!)
recheck catheter outflow
if no improvement
another abdo flat plate
if catheter tip is in the right place
? omental wrap ? omental wrap
? tip not connecting with peritoneal cavity
make sure no more stool
proceed to catheter dye study
Copyright Harvard Medical School, 2010. All Rights Reserved.
1562
Painful Inflow and Outflow
unusual unusual
both can usually be treated by tidal
PD:
leave in a residual dialysate volume and
cycle a smaller volume
eg 2L inflow, 1.7 L exchanges
The Yucky Exit Site
Mr EZ is on PDX 2 years Mr. EZ is on PD X 2 years
usual exit site care, including daily
mupirocin
presents with redness and pus around the
exit site
b h S ti M swab shows Serratia Marcesens
Copyright Harvard Medical School, 2010. All Rights Reserved.
1563
The Yucky Exit Site, continued
What is The Best Course of Action?
1. Remove the PD catheter and change to hemo for 4-
6 weeks
2. Remove the PD catheter and replace with a new
catheter at the same OR
3. Change the exit site ointment to gentamicin
4 Change the exit site ointment to canesten 4. Change the exit site ointment to canesten
5. Continue with the mupirocin at the exit site
The Yucky Exit Site, continued
What is The Best Course of Action?
1. Remove the PD catheter and change to hemo for 4-
6 weeks
2. Remove the PD catheter and replace with a new
catheter at the same OR
3. Change the exit site ointment to gentamicin
4 Change the exit site ointment to canesten 4. Change the exit site ointment to canesten
5. Continue with the mupirocin at the exit site
Copyright Harvard Medical School, 2010. All Rights Reserved.
1564
Gentamicinvs Mupirocinat the Catheter
Exit Site
double-blind randomized controlled study of gentamicin double blind randomized controlled study of gentamicin
vs mupirocin cream at the exit site
incident and prevalent PD patients
some statistical problems
gentamicin associated with equivalent staph aureus
infection, but much lower gram negative infection rate
Bernardini et al J Am Soc Nephrol 2005
Gentamicinvs Mupirocinat the Exit Site:
Time to First Exit Site Infection
Bernardini et al, J Am Soc Nephrol 2005
Copyright Harvard Medical School, 2010. All Rights Reserved.
1565
So Is Gentamicin the Answer?
certainly it is less expensive than mupirocin certainly it is less expensive than mupirocin
there seems to be a higher incidence of
yeast/fungal exit site infections
will aminoglycoside resistance develop?
will low-grade absorption lead to ototoxicity
h t i it ? or nephrotoxicity?
Prevention of Peritonitis and Exit Site
Infection
regular application of antibiotic ointment to the regular application of antibiotic ointment to the
catheter exit site is associated with a significant
reduction in exit site infection and (in some
studies) peritonitis
the unknowns:
development of antibiotic resistance p
effect of ointment/cream on catheter integrity
overgrowth of fungus
Copyright Harvard Medical School, 2010. All Rights Reserved.
1566
Gentamicinat the Exit Site: A New
Concern?
2 of my patients who received gentamicin
at the exit site
for gram negative cultures
Mr. G.W.
68 year old Type II diabetic 68 year old Type II diabetic
3 recurrent episodes of coagulase negative
staph peritonitis in 5 months
re-trained
2 weeks later presents with peritonitis
Copyright Harvard Medical School, 2010. All Rights Reserved.
1567
Fungal Peritonitis A Serious
Complication
accounts for 1 15%of peritonitis episodes accounts for 1 15% of peritonitis episodes
technique failure in many (>40%)
mortality rate 5 53%
trend over time for non-Albicans Candida
Risk Factors for Fungal Peritonitis
frequent peritonitis ** frequent peritonitis
antibiotic therapy **
immunosuppression
low serum albumin concentration
total parenteral nutrition
peritoneo-vaginal communication
Copyright Harvard Medical School, 2010. All Rights Reserved.
1568
Antibiotic Therapy as a Risk Factor for
Fungal Peritonitis
bacteria
fungi
Th dd ibi i h i Then add antibiotics to the mixture
Antibiotic Therapy as a Risk Factor for
Fungal Peritonitis Post antibiotics
bacteria
fungi
Fungal overgrowth of the intestine (and other sources?) Fungal overgrowth of the intestine (and other sources?)
Copyright Harvard Medical School, 2010. All Rights Reserved.
1569
Antibiotic Therapy as a Risk Factor for
Fungal Peritonitis Post antibiotics
Peritoneal cavity Peritoneal cavity
Anti-Fungal Prophylaxis
success rate of prophylaxis will depend on success rate of prophylaxis will depend on
baseline rate of fungal peritonitis
units with high rates have demonstrated
reduction of fungal peritonitis rates with
prophylactic anti-fungal therapy
oral nystatin may be inconvenient, but
safe medication
resistance not reported, but not likely a problem
even if it were
Copyright Harvard Medical School, 2010. All Rights Reserved.
1570
Mrs. Karmas Breathing Problems
Mrs. Karma
after her laparoscopic unwrap of omentum she is after her laparoscopic unwrap of omentum, she is
trained and discharged on APD, 2L X 3 exchanges
over 8 h, 1.5L last fill
3 days later, she calls the unit, complaining of
progressive shortness of breath
mild cough, but no fever or sputum
weight is increased1kg but no edema nor change in weight is increased 1 kg, but no edema nor change in
blood pressure
a chest x-ray is ordered
Copyright Harvard Medical School, 2010. All Rights Reserved.
1571
Mrs. Karmas CXR
Hydrothorax
Definition: The presence of peritoneal dialysis Definition: The presence of peritoneal dialysis
fluid in the pleural cavity
Incidence: Probably <5%
Pathogenesis: Movement of dialysate, under
increased intra-abdominal pressure, from
peritoneal to pleural cavity through peritoneal to pleural cavity through
congenital or acquired defects in the
diaphragm
Copyright Harvard Medical School, 2010. All Rights Reserved.
1572
Hydrothorax (contd)
Presentation: Presentation:
may be asymptomatic
shortness of breath
diminished effluent return
right-sided pleural effusion on CXR
Hydrothorax (contd)
Diagnosis: g
thoracentesis for relief of symptoms and/or diagnosis
pleural fluid analysis:
-transudate
-very high glucose concentration (usually, but not
always)
-cell count variable -cell count variable
no intraperitoneal methylene blue !
Copyright Harvard Medical School, 2010. All Rights Reserved.
1573
Hydrothorax (contd)
Treatment: Treatment:
thoracentesis may be helpful if very SOB
stop PD
temporary hemodialysis, if dialysis
necessary
Hydrothorax (contd)
Treatment
trial of re-introducing low pressure PD (the
dialysate in pleural cavity may have
functioned as a sclerosing agent)
pleurodesis (talc, tetracycline, bleomycin,
autologous blood)
operative or pleuroscopic repair operative or pleuroscopic repair
(diaphragmatic defects identified and
patched or oversewn)
Copyright Harvard Medical School, 2010. All Rights Reserved.
1574
Peritonitis: Cloudy Effluent
Peritonitis - Diagnosis
The diagnosis of peritonitis requires at least g p q
2 of the following 3 features:
peritoneal fluid leukocytosis (>100/mm
3
,
and at least 50% polymorphonuclear
cells)
the fluid should dwell 2 to 4 hours
abdominal pain
positive culture of the dialysis effluent
Copyright Harvard Medical School, 2010. All Rights Reserved.
1575
Hematogenous seeding
bacteremia can cause peritoneal bacteremia can cause peritoneal
seeding and peritonitis
but
***peritonitis hardly ever causes
bacteremia*** bac e e a
(keep this in mind if you have a patient with a
mechanical heart valve or prosthetic hip)
Hematogenous seeding
Use antibiotic prophylaxis at times of Use antibiotic prophylaxis at times of
anticipated bacteremia
eg dental work, colonoscopy, colposcopy or
GU instrumentation (and drain effluent
before colonoscopy/colposcopy/GU
instrumentation) instrumentation)
Copyright Harvard Medical School, 2010. All Rights Reserved.
1576
Peritonitis - Principles of Treatment
start antibiotic treatment quickly start antibiotic treatment quickly
cover for both gram positive and gram
negative organisms until cultures available
adjust antibiotics according to culture results
re-evaluate the treatment if no
i t* i 36 48 h improvement* in 36-48 hours
* less abdominal pain, falling peritoneal
fluid WBC count
Peritonitis - Principles of Treatment (II)
Other suggestions: Other suggestions:
avoid vancomycin if possible (VRE)
avoid prolonged courses of aminoglycosides
because of concern for residual renal
function
(b h f h i l) (both of these are controversial)
Copyright Harvard Medical School, 2010. All Rights Reserved.
1577
Peritonitis - Principles of Treatment
consider removal of the PD catheter if
little or no improvement in 4-5 days
(especially if staph. aureus or
pseudomonas)
fungal peritonitis: catheter removal as
soon as possible
watch for acute problems with UF
dont let peritonitis drag on for days !
Most Recent Guidelines: Find them at
www.ispd.org
Copyright Harvard Medical School, 2010. All Rights Reserved.
1578
Special Cases: Pseudomonas
peritonitis
combine 2 anti pseudomonal antibiotics for 21 combine 2 anti-pseudomonal antibiotics for 21
days
if pseudomonas peritonitis is in association
with pseudomonas exit site or tunnel infection,
remove catheter
Treatment of Other Special Cases
Multiple gramnegative organisms: add Multiple gram negative organisms: add
Metronidazole 500 mg q 8 hrs. iv, po, per
rectum - 21 days.
Get a surgical opinion
Copyright Harvard Medical School, 2010. All Rights Reserved.
1579
Mr. C the Building Superintendent
48 year old man with polycystic kidney disease is 48 year old man with polycystic kidney disease is
trained on cycler dialysis. Current prescription is
2.5 L X 4 exchanges over 8 hours at night, with
2.5 L day dwell
one year later: he is doing well
residual renal GFR is 8 ml/min
hi if i l t fi i h d b i k d his wife is almost finished being worked up as a
kidney donor and is likely to be accepted
Mr C (contd)
at clinic he reports a newlumpin his left groin at clinic, he reports a new lump in his left groin.
He had been bent over looking under a sink and
felt a pop and some tenderness in the groin
on physical exam, there is a left inguinal hernia
Copyright Harvard Medical School, 2010. All Rights Reserved.
1580
PD and Increased Intra-abdominal
Pressure (IAP)
instillation of dialysate into the peritoneal cavity
l d t i d IAP leads to increased IAP
the magnitude of the increase depends upon:
-volume of dialysate instilled
-position of the patient (sitting>standing>supine)
-age, body mass index
-coughing, lifting, straining at stool, aerobics class
(!), chopping wood (!)
Relationship among Intra-Abdominal Pressure
(IAP), Position & Dialysate Volume
IAP
sitting
IAP
(mmHg)
sitting
standing
supine
VOLUME OF DIALYSATE (L)
1 2
3
0
after
Twardowski
Copyright Harvard Medical School, 2010. All Rights Reserved.
1581
Ventral Hernia
Other Hernias: Umbilical Hernia
Biggest risk for
incarceration
Copyright Harvard Medical School, 2010. All Rights Reserved.
1582
60 year old woman on PD presents with
localized abdominal pain
Hernias
Treatment:
warn patient about signs of incarceration
surgical repair:
-dialysis around repair depends on renal
function and condition of the patient
-dont usually have to put them on HD ! y p
-reintroduce PD with low volumes, supine
posture, increase volume over 2 weeks
Copyright Harvard Medical School, 2010. All Rights Reserved.
1583
Show This to Your Surgeon
So What Happened to Mr C?
continued night cycler dialysis continued night cycler dialysis
dry during day (RRF 8 ml/min)
elective hernia repair
no PD for 2 days
back to night cycler 1.5L volume X 2 weeks,
2Lvolume X 2 weeks, then 2.5 L volume 2Lvolume X 2 weeks, then 2.5 L volume
day dwell re-introduced 2 months later
Copyright Harvard Medical School, 2010. All Rights Reserved.
1584
Abdominal Wall and Genital Edema
Presentation:
abdominal swelling or bogginess, scrotal or
labial edema
diminished effluent return
weight gain without peripheral edema
pericatheter leak: wetness or swelling at exit pericatheter leak: wetness or swelling at exit
site
Abdominal Wall & Genital Edema
Diagnosis:
physical exam (have patient stand in front of
you)
unchanged PET results
CT scan
Copyright Harvard Medical School, 2010. All Rights Reserved.
1585
Peau dorange of Abdominal Wall
Edema
Abdominal Wall and Genital Edema
Diagnosis by CT Scanning: Diagnosis by CT Scanning:
add 100-150 ml Omnipaque to dialysis bag
infuse dialysate into patient
have patient ambulatory for 30 to 60
minutes to increase intra-abdominal
*
pressure
send for CT scan - discuss with the
radiologist !
*
Copyright Harvard Medical School, 2010. All Rights Reserved.
1586
CT Dye in a Young Man With
Scrotal Edema
Abdominal Wall & Genital Edema
Management: Management:
reintroduce low pressure PD (eg APD with
low volumes)
temporary HD to allow healing
Abdominal wall: CT scan for occult hernia
Genital: CT scan for patent processus
vaginalis, which is easily repaired
Copyright Harvard Medical School, 2010. All Rights Reserved.
1587
SM: An Unfortunate Young Woman
Wegeners Granulomatosis at age 10
ESRD, on CAPD X 7 years (refused TP)
severe S. aureus peritonitis catheter
removal and interim hemodialysis
re-insertion of PD catheter severe intra-
abdominal adhesions
SM: An Unfortunate Young Woman
transferred to adult hospital age 18 on
chronic hemodialysis
1 year later: recurrent bowel obstruction,
weight loss, malnutrition
Copyright Harvard Medical School, 2010. All Rights Reserved.
1588
Encapsulating Peritoneal Sclerosis
(EPS)
typically diagnosed in the later stages typically diagnosed in the later stages
long-term (>60 months) PD patient with
recurrent bowel obstruction
anorexia, weight loss, malnutrition
Clinical Presentation: The
Inflammatory Phase
there is an inflammatory phase that often there is an inflammatory phase that often
precedes the fibrosing phase
this inflammation can be missed or
misdiagnosed
symptoms can be non-specific
Copyright Harvard Medical School, 2010. All Rights Reserved.
1589
Clinical Presentation
Inflammatory Phase
anorexia, nausea, vomiting
fever
weight loss, malnutrition
hemoperitoneum
increased serum CRP
rapid increase in transport
status
hypoalbuminemia hypoalbuminemia
anemia and resistance to
ESA
Clinical Presentation
Inflammatory Phase Sclerosing Phase
anorexia, nausea, vomiting
fever
weight loss, malnutrition
hemoperitoneum
increased serum CRP
rapid increase in transport
status
hypoalbuminemia
recurrent bowel obstruction
progressive malnutrition
hypoalbuminemia
anemia and resistance to
ESA
Copyright Harvard Medical School, 2010. All Rights Reserved.
1590
Epidemiology and Risk Factors
incidence 0 5%to 2 8% incidence 0.5% to 2.8%
(our unit: 1.1%)
mortality 38-63%
both incidence and mortality increase with
increased time on PD therapy
Epidemiology and Risk Factors
at-risk patients include
long-term patients
rapid transporters, especially those who evolve into
rapid transport status
unresolving peritonitis, with removal of the PD
catheter
cessation of PD even without peritonitis cessation of PD, even without peritonitis
? vasculitis as underlying cause of renal failure (my
observation)
? renal transplant
Copyright Harvard Medical School, 2010. All Rights Reserved.
1591
Post-Transplant EPS
observational studies suggest that incidence
f EPS ft l t l t b of EPS after renal transplant may be
increasing
this may be the result of
discontinuation of PD
+
d i i i id d reduction in corticosteroid dose
+
use of calcineurin inhibitors
Is the Incidence of Post-Transplant EPS
Increasing?
retrospective reviewof retrospective review of
PD patients with a
kidney transplant at 2
transplant centers in the
Netherlands
1998-2005
mean time on dialysis mean time on dialysis
almost 72 months
Open bars: PD patients with a transplant
Shaded bars: patients with EPS
Korte et al. Neph Dial Transplant 2007
Copyright Harvard Medical School, 2010. All Rights Reserved.
1592
Clinical Diagnosis: Fibrosing Phase
change in bowel change in bowel
habits
recurrent bowel
obstruction
Macroscopic: Whitened peritoneal
membrane encapsulating the bowel
Kawanishi et al Perit Dial Int 2005
Copyright Harvard Medical School, 2010. All Rights Reserved.
1593
Radiologic Features of EPS
peritoneal thickening with encapsulation p g p
(peritoneal thickening alone not sufficient for the
diagnosis)
peritoneal calcification (not sufficient for the
diagnosis)
small/large bowel obstruction
abdominal cocooning
Advanced EPS and Ascites in a
Former PD Patient
Copyright Harvard Medical School, 2010. All Rights Reserved.
1594
Peritoneal Calcification
11 months
later
George C et al. Clin Radiol 2007
MRI of Patient with EPS
Slide courtesy of Dr. R. Dunst
Copyright Harvard Medical School, 2010. All Rights Reserved.
1595
EPS: Prevention
Who Knows? Who Knows?
prevention of frequent peritonitis?
timely transfer of patients with frequent peritonitis
from PD to HD?
biocompatible PD solutions?
plannedtransition of long termPDpatients to planned transition of long-term PD patients to
HD?
Treatment: Immunosuppressive and
Anti-Inflammatory Therapy
corticosteroids corticosteroids
probably more useful in the early,
inflammatory phase
both pulse steroids and daily oral therapy
have been used
38.5% rate of remission in the largest
study (Kawanishi Am J Kidney Dis 2004)
Copyright Harvard Medical School, 2010. All Rights Reserved.
1596
Treatment: Anti-fibrotic Therapy
Tamoxifen Tamoxifen
upregulates production of TGF 1, leading to
degradation of type IV collagen
4 case reports and a case series of 4 patients
suggest it helps recovery from, or prevents, EPS
risk of thrombosis and association with endometrial
cancer
role of m-Tor inhibitors (sirolimus, everolimus) under
investigation
Surgical Treatment
surgical lysis of intestinal adhesions and surgical lysis of intestinal adhesions and
stripping of fibrous cocoon
Indications for surgery:
persistent or recurrent bowel obstruction
failing nutritional status
failure to respond to medical therapy p py
recurrent peritonitis from bowel compromise
Copyright Harvard Medical School, 2010. All Rights Reserved.
1597
Surgical Treatment (continued)
this can be very risky very bloody surgery this can be very risky, very bloody surgery
high risk of bowel tearing and perforation
should be done by a surgeon familiar with
both the condition and the surgical literature
surrounding EPS
ti t lit t bl (4%) i operative mortality acceptable (4%) in
experienced hands (Kawanishi Perit Dial Int
2008)
SV and Her Scary Episode
28 year old woman, CKD of unknown
ti l etiology
predialysis education: chooses and starts on
PD
Copyright Harvard Medical School, 2010. All Rights Reserved.
1598
SV and Her Scary Episode
doing great on PD doing great on PD
menses resume
with 2
nd
menses: painless bloody dialysate
effluent
to ER: hemodynamically stable, but urgent
CBC PTT d t CBC, PTT, cross and type
Hemoperitoneum
Definition: Definition:
Bloody peritoneal effluent
Presentation:
scary ! (not as bad as it looks) scary ! (not as bad as it looks)
must consider benign and serious
causes
Copyright Harvard Medical School, 2010. All Rights Reserved.
1599
Hemoperitoneum
Benign Causes:
menstruation (retrograde menses or
endometriosis)
ovulation
ruptured renal or ovarian cysts
trauma trauma
coagulopathy
Hemoperitoneum
Serious Causes:
ischemic bowel
hepatic or colon cancer
pancreatitis
encapsulating peritoneal sclerosis
kid kidney cancer
Copyright Harvard Medical School, 2010. All Rights Reserved.
1600
Hemoperitoneum
Treatment:
IP heparin to avoid clotting of catheter
flushes
dialysate at room temperature
investigations depend on whether benign or
serious type of presentation serious type of presentation
During training, warn patients in advance of this
complication!
And What Do You Think About This?
Copyright Harvard Medical School, 2010. All Rights Reserved.
1601
So What Have We Learned? (1)
constipation is the #1 cause of outflow constipation is the #1 cause of outflow
obstruction in new catheters
having a bowel movement doesnt
necessarily mean its cured
think of hydrothorax if a PD patient gets y p g
short of breath early on
So What Have We Learned? (2)
Long-termantibiotics is a risk for fungal Long term antibiotics is a risk for fungal
peritonitis
fungal prophylaxis may or may not work, but its
low-risk therapy
Staph aureus or Pseudomonas infection of
BOTH exit site and PD fluid wont get better
with antibiotics remove catheter
Try to find an ID specialist who knows about
PD infections
Copyright Harvard Medical School, 2010. All Rights Reserved.
1602
So What Have We Learned? (3)
Think of encapsulating peritoneal sclerosis if Think of encapsulating peritoneal sclerosis if
a patient has recurrent bowel obstruction
think of inflammatory phase if patient gets
hemoperitoneumand GI symptoms
Commonest cause of hemoperitoneumis
menstruation and needs no further
investigation in that setting
So What Have We Learned? (4)
Hernias are common
try to prescribe day dry or small day
volumes in at-risk patients
they can be repaired operatively without
switching to hemodialysis
they may lead to secondary bowel
incarceration and even strangulation incarceration and even strangulation
they can be a source of leak of PD fluid
into surrounding tissues
Copyright Harvard Medical School, 2010. All Rights Reserved.
1603
Disclosure
Consultant Consultant
Amgen
Hospira
European Commission
Speaker Fees / Honoraria p
Amgen
Baxter Healthcare
Copyright Harvard Medical School, 2010. All Rights Reserved.
1604
Poisonings and Intoxications
J. Kevin Tucker, M.D.
Brigham and Womens Hospital
Harvard Vanguard Medical
Associates Associates
Massachusetts General Hospital
Conflicts of Interest
Consultant Consultant
Baxter Healthcare
Genzyme
Copyright Harvard Medical School, 2010. All Rights Reserved.
1605
Case Presentation
60 year-old male with history of alcohol abuse
and hypertension.
Brought to a local hospital after the patients ex-
wife called EMS for slurred speech confusion wife called EMS for slurred speech, confusion
and unsteadiness.
Reportedly the patient had been drinking until
3AM the previous night. He was found by his ex-
wife at 4PM with symptoms which she initially
thought were due to drunkenness. The slurred
speech, confusion, and unsteady gait p , , y g
progressively worsened over the following 3
hours and were not typical of his intoxicated
behavior.
Case Presentation
On arrival of EMS, SBP was > 220 mm Hg with , g
slurring of his speech and confusion. Patient was
given labetalol IV and brought to the local hospital,
where he had an unremarkable head CT. Patient
became increasingly confused and then obtunded
and was intubated for airway protection before being
transferred to MGH EW.
In the MGH EW, patient was found to have a K
+
of
6 9 mEq/L He was given intravenous fluids and 6.9 mEq/L. He was given intravenous fluids and
standard treatments for hyperkalemia, then
transferred to ICU.
The patients ex-wife was sent home due to
intoxication.
Copyright Harvard Medical School, 2010. All Rights Reserved.
1606
PMHx
EtOH abuse drinking about 1 pint a day EtOH abuse, drinking about 1 pint a day
HTN
NKDA
No known medications
Family and Social histories could not be Family and Social histories could not be
obtained.
Physical Exam
(12 hours after presentation)
VS: T 100 F; BP 170/70 mm Hg; HR 80s
Obese male unresponsive to any stimuli; ETT in place Obese male, unresponsive to any stimuli; ETT in place,
no sedation being given
JVP difficult to appreciate given body habitus
Clear conjunctivae, pupils are equal and round,
responsive to light
RR with normal S1 and S2, no extra heart sounds,
murmurs, or rubs present murmurs, or rubs present
Breath sounds are clear to auscultation bilaterally
Abdomen is obese, soft. Bowel sounds present; no
masses appreciated
Scant LE edema bilaterally, DP pulses 2+ bilaterally
Copyright Harvard Medical School, 2010. All Rights Reserved.
1607
Labs

143
69
96
77
16
125
145 146
51
98
5
18
132 AG 43
154
6.9 7.7 1.25
AG 39, Albumin 4.7
iCa 1.03
Lactate 7.3, Serum osm 375
ABG: 6.89/26/135
Normal LFTs
26.6
15.9
53.2
424
8.6
2.1
5.7
5.1 <5 1.32 AG >43
Normal amylase and lipase
CK normal, troponin negative
Case Presentation
Question
What is the differential diagnosis What is the differential diagnosis
What would be the initial interventions
as we gather further data?
Is there any role for extracorporeal
therapy in the management of this e apy e a age e o s
patient?
Copyright Harvard Medical School, 2010. All Rights Reserved.
1608
Poisonings and Intoxications:
Scope of the Problem
1261 fatal exposures compiled by the 1261 fatal exposures compiled by the
American Association of Poison Control
Centers in its 2005 report
1730 cases of exposures treated with
hemodialysis
30 cases of exposures treated with
hemoperfusion
Initial Management of the Poisoned
or Overdosed Patient
Airway Airway
Breathing
Circulation
Therapies for patients with altered mental
status
Hypertonic dextrose (0.5 to 1 g/kg as D50W) yp ( g g )
Thiamine 100 mg IV
Naloxone 2 mg IV
Oxygen (100% FIO
2
at 8-10L/min)
Goldfranks Toxocologic Emergencies, Sixth Edition
Copyright Harvard Medical School, 2010. All Rights Reserved.
1609
Removing the drug or
enhancing its elimination
Removal of the drug from the GI tract Removal of the drug from the GI tract
Gastric emptying
Gastric lavage
Enhancing elimination through the GI tract
Activated charcoal
Cathartics
Enhancing renal elimination of the drug
Manipulating urinary pH
Forced diuresis
Removing the drug or
enhancing its elimination
Removal of the drug by extracorporeal Removal of the drug by extracorporeal
methods
Hemodialysis
Hemoperfusion
Copyright Harvard Medical School, 2010. All Rights Reserved.
1610
Gastric Emptying
Should be considered for patients with a Should be considered for patients with a
recent ingestion
Risks of gastric emptying
Availability and utility of alternative
methods to limit the absorption
Smilkstein in Goldfranks Toxicologic Emergencies, Sixth Edition
Factors that favor gastric
emptying
Recent ingestion Recent ingestion
Ingested agent not adsorbed by activated
charcoal (lithium)
Antidotal or adjunctive therapy ineffective or
non-existent (calcium channel blockers,
colchicine, iron)
Ingested agent likely to form a durable mass
after overdose (large amounts of ASA, enteric
coated agents)
Ingestion of sustained-release agents
Copyright Harvard Medical School, 2010. All Rights Reserved.
1611
Activated Charcoal
Activated charcoal has two benefits Activated charcoal has two benefits
Preventing the absorption of toxic agents from the
GI tract
Enhancing the elimination of agents already
absorbed
Initial dose usually 0.5-1.0 g/kg
May be repeated q 6 hours
Usually given with a cathartic such as sorbitol
Drugs that are not removed by
charcoal
Iron Iron
Lithium
Methanol
Ethylene Glycol
Copyright Harvard Medical School, 2010. All Rights Reserved.
1612
Manipulation of Urinary pH
Many toxic compounds are weak acids or Many toxic compounds are weak acids or
bases.
The ionization of such compounds is
dependent upon pKa of the compound and
pH of the solution.
As these substances pass through the
kid th b filt d t d d kidney, they may be filtered, secreted, and
reabsorbed.
If the urinary pH is manipulated to favor the
ionized form, the drug is trapped in luminal
fluid and not reabsorbed.
Manipulation of Urinary pH
For this strategy to be effective the renal For this strategy to be effective, the renal
excretion of the compound must be sufficient
to make increased clearance a major route of
elimination.
Alkalinization can be achieved by
administration of a bicarbonate containing
solution solution
Goal is to increase urinary pH to 7-8
Carbonic anhydrase inhibitors should be
avoided because of systemic metabolic
acidosis
Copyright Harvard Medical School, 2010. All Rights Reserved.
1613
Drugs for which urinary
alkalinization may be effective
Salicylates Salicylates
Phenobarbital
Chlorpropramide
Potential complications of
alkalinization
Volume overload Volume overload
Hypernatremia
Hypocalcemia (ionized)
Hypokalemia (from bicarbonaturia)
Copyright Harvard Medical School, 2010. All Rights Reserved.
1614
Forced Diuresis
Increasing urinary flow rate by volume Increasing urinary flow rate by volume
expansion with normal saline may be
effective
Most beneficial for substances in which
GFR is an important determinant of
excretion, and especially where the
extracellular volume is contracted
(lithium, for example)
Hemodialysis for intoxications
and poisonings
Hemodialysis is most beneficial for Hemodialysis is most beneficial for
drugs that have specific characteristics:
Low volume of distribution (<1 L/kg)
Water soluble
Low protein binding
Rapid equilibration between tissue and
blood compartments
Copyright Harvard Medical School, 2010. All Rights Reserved.
1615
Dialysis procedure for
poisoning and intoxication
Choice of dialyzer: High urea clearance; Choice of dialyzer: High urea clearance;
high flux, depending upon molecular weight
of substance to be removed
Bicarbonate concentration may need to be
lowered if there is concurrent metabolic
alkalosis. alkalosis.
Phosphorous may need to be supplemented.
Drugs/Toxins efficiently
removed by hemodialysis
Drug
Molecular
Weight
Volume of
distribution
Water
soluble
(daltons)
distribution soluble
Ethylene
glycol
62 0.6 Yes
Isopropanol
60 0.6 Yes
Lithium
7 0.6-1.0 Yes
Methanol
32 0.7 Yes
Salicylate
138 0.2 Yes
Theophylline
180 0.5 Yes
Copyright Harvard Medical School, 2010. All Rights Reserved.
1616
Hemoperfusion
A technique in which blood is passed A technique in which blood is passed
through a cartridge packed with
charcoal or carbon
Charcoal in the cartridge competes with
plasma protein for the drug, adsorbs it,
and eliminates it from the circulation and eliminates it from the circulation
More efficient at removing lipid soluble
drugs than is hemodialysis.
Hemoperfusion Cartridges
Copyright Harvard Medical School, 2010. All Rights Reserved.
1617
Hemoperfusion
Requires more heparin than Requires more heparin than
hemodialysis
Qb typically 250 to 400 mL/min
Some charcoal cartridges require
priming with D5W to load the cartridge p g 5 o oad e ca dge
with glucose
Drugs/Toxins efficiently
removed by hemoperfusion
Drug
Molecular
Weight
Volume of
distribution
Water
soluble
(daltons)
distribution soluble
Theophylline
180 0.5 Yes
Carbamazepine
236 1.4 No
Disopyramide
340 0.6 No
Phenobarbital
232 0.5 No
Phenytoin 252 0.6 No
Copyright Harvard Medical School, 2010. All Rights Reserved.
1618
Number of cases receiving
hemodialysis and hemoperfusion
Holubek et al, Kidney Int 74: 1327-34, 2008
Trends for commonly accepted
indications for HD and/or HP
Holubek et al, Kidney Int 74: 1327-34, 2008
Copyright Harvard Medical School, 2010. All Rights Reserved.
1619
Availability of Hemoperfusion
Charcoal hemoperfusion cartridges in Charcoal hemoperfusion cartridges in
only one-third of New York City
hospitals receiving emergency patients
Only three in-hospital hemodialysis
units in New York City had performed
hemoperfusion in the last five years, on
three cases.
Shalkham et al, Am J Kidney Dis 48: 239-241, 2006
Lithium
Lithium is a monovalent cation that is Lithium is a monovalent cation that is
used to treat bipolar disorder
It has a narrow therapeutic window.
Rapidly absorbed from the GI tract
Peak levels occur at 2-4 hours
Large volume of distribution (0.6-0.9
L/kg)
Excreted by the kidney
Copyright Harvard Medical School, 2010. All Rights Reserved.
1620
Signs of lithium intoxication
Prolonged QT interval Prolonged QT interval
Nausea, vomiting
Leucocytosis
Urine concentrating defects
Neurologic signs and symptoms of
lithium intoxication
Weakness Tinnitus Weakness
Lightheadedness
Fine tremor
Muscle twitching
Drowsiness
Tinnitus
Confusion
Clonus
Muscular irritability
Extrapyramidal
Hyperreflexia
Slurred speech
symptoms
Coma
Copyright Harvard Medical School, 2010. All Rights Reserved.
1621
Treatment options for lithium
intoxication
Saline diuresis Saline diuresis
Sodium polystyrene sulfonate (?)
Dialysis
Indications for hemodialysis
Altered mental status Altered mental status
Renal dysfunction
Inability to tolerate a sodium load
Level > 4.0 mEq/L in acute toxicity
Level > 2.5 mEq/L in chronic toxicity
Watch for rebound; a second treatment may
be required
Copyright Harvard Medical School, 2010. All Rights Reserved.
1622
Toxic Alcohols
The three toxic alcohols most commonly The three toxic alcohols most commonly
associated with toxicologic
emergencies:
methanol
ethylene glycol
isopropanol
Compound Molecular
Weight
SOsm
(mOsm/L) per
10 mg/dL
serum alcohol
concentration
M th l 32 04 3 09 Methanol 32.04 3.09
Ethanol 46.07 2.12
Isopropanol 60.02 1.66
Ethylene glycol 62.07 1.60
Propylene
glycol
76.09 1.31
Diethylene
glycol
106.12 0.90
From Kraut and Kurtz, Clin J Am Soc Nephrol 3: 208-225
Copyright Harvard Medical School, 2010. All Rights Reserved.
1623
Ethylene glycol
Commonly used as an antifreeze Commonly used as an antifreeze
Also used in
fire extinguishers
inks
pesticides
dh i adhesives
air conditioning systems
solar energy systems
Ethylene glycol
Absorption and metabolism
Rapidly absorbed from the GI tract Rapidly absorbed from the GI tract
Reaches peak levels within 1-4 hours
Rapidly distributed throughout the body
because of its high water solubility
Metabolized by alcohol dehydrogenase to
glycoaldehyde, glycolic acid, and oxalic acid g y y g y
(all of which are effectively removed by
hemodialysis)
Copyright Harvard Medical School, 2010. All Rights Reserved.
1624
Clinical findings in ethylene
glycol intoxication
Nausea/vomiting Anion-gap metabolic Nausea/vomiting
Lethargy
Coma
Nystagmus
Ataxia
Areflexia
Anion gap metabolic
acidosis
Osmolal gap
Oxalate cystals in
urine
Hippurate crystals in
Myoclonic movements
Seizures
pp y
urine
Hypocalcemia
ATN
Examination of the urine
Crystalluria can persist for up to
40 hrs with normal renal function,
and 4 days with renal failure
Urine fluorescence under UV light
(wavelength 360 nm) with a
Woods lamp is pH dependent
and 4 days with renal failure
Dihydrate crystals (4-5hrs)
Mix of dihydrate and monohydrate
crystals (5-7 hrs)
Only monohydrate crystals (> 7
hrs)
Wood s lamp is pH dependent
Fluorescence will be minimal or
absent with urine pH 4.5
Not all brands of anti-freeze have
sodium fluorescein added
Copyright Harvard Medical School, 2010. All Rights Reserved.
1625
Methanol
A commonly used alcohol found in A commonly used alcohol found in
Windshield washing and de-icing solutions
Carburetor cleaners
Duplicating fluids
Solid canned fuels
Shellac Shellac
Paint removers and thinners
Embalming fluids
Methanol
Absorption and metabolism
Peak levels occurs within 30-60 minutes Peak levels occurs within 30 60 minutes
Often a latent period of about 24 hours
before development of either toxic
symptoms or metabolic acidosis
Volume of distribution of 0.6-0.7L/kg
8 % i b li d i h li b 75-85% is metabolized in the liver by
alcohol dehydrogenase; 10-20%
excreted unchanged by the lungs
Copyright Harvard Medical School, 2010. All Rights Reserved.
1626
Methanol
Metabolites
Alcohol dehydrogenase metabolizes Alcohol dehydrogenase metabolizes
methanol to formaldehyde.
Formaldehyde is then oxidized to formic
acid, which produces the metabolic
acidosis and the retinal toxicity.
Clinical findings in methanol
intoxication
Tachycardia
Blurred vision
Tachycardia
Hypotension
Seizures
Coma
Headache
Blurred vision
Hyperemic optic
discs
Papilledema
Blindness
Hemorrhagic and
Somnolence
Abdominal pain
Nausea/vomiting
Hemorrhagic and
non-hemorrhagic
necrosis of the
putamen
Copyright Harvard Medical School, 2010. All Rights Reserved.
1627
Ethylene Glycol
Glycoaldehyde
Alcohol
dehydrogenase
NAD
NADH
lactate
Pyruvate
Glyoxylic
acid
Adepate Glycine
thiamine pyridoxine
Glycolic acid
Oxalate
ARF, crystalluria (Ca
oxalate)
hypocalcaemia Tissue deposition
Treatment of methanol and
ethylene glycol intoxications
Blocking alcohol dehydrogenase Blocking alcohol dehydrogenase
Ethanol
Fomepizole
Hemodialysis
Correction of metabolic abnormalities
Removal of toxic metabolites
Copyright Harvard Medical School, 2010. All Rights Reserved.
1628
Treatment of methanol and
ethylene glycol intoxications
Thiamine and Pyridoxine (ethylene glycol) Thiamine and Pyridoxine (ethylene glycol)
Thiamine 100 mg IV q 6 hours (shunts metabolism
of glyoxylic acid to alpha-hydroxy-beta-ketoadipic
acid)
Pyridoxine 50 mg IV q 6 hours (shunts
metabolism from glyoxylic acid to glycine)
Folate (ethanol)
50-75 mg IV q 4 hours for 24 hours to enhance
elimination of formic acid
Indications for treatment of ethylene
glycol with an antidote
Ethylene glycol concentration in plasma > 20
mg/dL or mg/dL or
Documented recent history of ethylene glycol
ingestion and osmolal gap > 10 mOsm/kg or
History or strong clinical suspicion of ethylene
glycol poisoning and at least two of the
following: g
Arterial pH < 7.3
Serum bicarbonate < 20 mEq/L
Osmolal gap > 10 mOsm/kg
Urinary oxalate crystals
From Winchester and Kitiyakara in Handbook of Dialysis
Copyright Harvard Medical School, 2010. All Rights Reserved.
1629
Indications for hemodialysis in
ethylene glycol poisonings
Severe metabolic acidosis (pH 7 25 7 3) Severe metabolic acidosis (pH 7.25-7.3)
unresponsive to therapy
Renal failure
Ethylene glycol levels > 50 mg/dL
unless fomepizole is being administered u ess o ep o e s be g ad s e ed
and patient is asymptomatic with a
normal pH
From Winchester and Kitiyakara in Handbook of Dialysis
Indications for dialysis in
methanol poisoning
Methanol level > 50 mg/dL Methanol level > 50 mg/dL
Visual, fundoscopic, or mental status
changes
Severe acidosis
Serum formic acid levels elevated Serum formic acid levels elevated
Consumption of the equivalent of more
than 30 mL of pure methanol
Copyright Harvard Medical School, 2010. All Rights Reserved.
1630
Use of ethanol in toxic alcohol
poisonings
Loading dose 0 6 g/kg IV (10% ethanol in Loading dose 0.6 g/kg IV (10% ethanol in
D5W) or orally (20% ethanol in water)
Maintenance dose
In alcoholic patients 154 mg/kg/hr
In non-alcoholic patients 66 mg/kg/hr
During hemodialysis double ethanol infusion rate During hemodialysis, double ethanol infusion rate
Adjust to maintain serum ethanol level 100-
200 mg/dL
From Winchester and Kitiyakara in Handbook of Dialysis
Use of fomepizole in ethylene
glycol poisonings
Loading dose 15 mg/kg IV in 100 mL 0 9% Loading dose 15 mg/kg IV in 100 mL 0.9%
saline over 30 minutes to 1 hour
Maintenance dose: 10 mg/kg every 12 hours
for 4 doses, then
15 mg/kg q 12 hours until ethylene glycol
concentration < 20 mg/dL and patient is concentration < 20 mg/dL and patient is
asymptomatic with normal arterial pH
From Winchester and Kitiyakara in Handbook of Dialysis
Copyright Harvard Medical School, 2010. All Rights Reserved.
1631
Salicylates
The active ingredient in the common The active ingredient in the common
pain preparation aspirin
May be found in combination with other
drugs, e.g., caffeine in some
preparations
Salicylate toxicity
Stimulates respiratory centers in the brain Stimulates respiratory centers in the brain,
leading to respiratory alkalosis
Interferes with Krebs cycle, limiting ATP
production, and producing a metabolic (lactic)
acidosis
Respiratory alkalosis predominates initially
By the time most adults reach the hospital,
they have a combined respiratory alkalosis
and metabolic acidosis.
Copyright Harvard Medical School, 2010. All Rights Reserved.
1632
Salicylate toxicity
Patients who have a respiratory Patients who have a respiratory
acidosis and metabolic acidosis
Salicylate induced pulmonary edema
Fatigue
CNS depression from a mixed overdose
Very poor prognosis
Clinical features of salicylate
poisoning
Anion-gap metabolic
Nausea/vomiting
Anion gap metabolic
acidosis
Respiratory alkalosis
Tinnitus
Deafness
Vertigo
Hallucinations
Nausea/vomiting
Gastritis
Pylorospasm
Abnormal LFTs
Hypoglycemia
Hallucinations
Stupor
Seizures
Coma
Hyperthermia
Copyright Harvard Medical School, 2010. All Rights Reserved.
1633
Bedside testing for salicylates
Add several drops of FeCl to 1 mL of Add several drops of FeCl
3
to 1 mL of
urine
Purple color indicates the presence of
salicylic acid, acetoacetic acid, or
phenylpyruvic acid
Highly sensitive for small quantities of
salicylic acid
Alkalinization of urine and
blood
Salicylic acid is a weak acid (pKa 3 5) Salicylic acid is a weak acid (pKa 3.5)
It is thus trapped in an alkaline environment
Alkalinization of the urine enhances its
excretion
Alkalinzation of the blood keeps it out of the
CNS
Alkalinization should be considered whenever
the level is > 35 mg/dL
Goal urine pH 7.5-8.0
Copyright Harvard Medical School, 2010. All Rights Reserved.
1634
Indications for hemodialysis in
salicylate poisoning
Renal failure Renal failure
Congestive heart failure
Noncardiogenic pulmonary edema
Persistent CNS disturbances
Progressive deterioration in vital signs
Severe acid-base or electrolyte disturbances
Hepatic compromise with coagulopathy
Salicylate level (acute) > 100 mg/dL
Amphetamine and related
drugs of abuse
Methamphetamine is a sympathomimetic Methamphetamine is a sympathomimetic
amine that belongs to the class of
phenethylamines.
Amphetamine was first synthesized in
Germany in 1887.
Plant-derived stimulant ephedra (ma huang) Plant-derived stimulant ephedra (ma huang)
containing the alkaloids ephedrine and
pseudoephidrine has been used in traditional
Chinese medicine for more than 5000 year to
treat asthma and the common cold.
Copyright Harvard Medical School, 2010. All Rights Reserved.
1635
History of Amphetamines
A nasal decongestant containing A nasal decongestant containing
amphetamine was marketed in 1932.
Widespread abuse led the FDA to ban
amphetamine inhalers in 1959.
The passage of the Controlled Substances
Act in 1970 reduced the rates of Act in 1970 reduced the rates of
amphetamine abuse.
The 1990s saw the explosion of designer
amphetamines.
Methamphetamine
After cannabis the most widely used drug of After cannabis, the most widely used drug of
abuse in the United States.
About 5% of the US population are estimated
to have used meth, with about 300,000 new
users each year.
Used clinically for the treatment of ADHD Used clinically for the treatment of ADHD,
narcolepsy and short-term treatment of
obesity.
Can easily be synthesized in home labs from
over-the-counter cold preparations.
Copyright Harvard Medical School, 2010. All Rights Reserved.
1636
Methamphetamines effects
Meth is taken up by cytoplasmic vesicles of Meth is taken up by cytoplasmic vesicles of
presynaptic adrenergic neurons, thus displacing
epinephrine, norepinephrine, dopamine, and
serotonin.
and adrenergic receptors:
Hypertension, tachycardia, vasospasm,
hyperthermia hyperthermia
Dopamine receptor stimulation:
Drug craving and psychiatric symptoms
Serotonergic stimulation:
Mood alterations; abnormal response to hunger and thirst
Clinical Features of Meth
Intoxication
Tachycardia Tachycardia
Hyperthermia
Agitation
Psychosis
Diaphoresis Diaphoresis
Minimally reactive mydriasis
Copyright Harvard Medical School, 2010. All Rights Reserved.
1637
Clinical Features of Meth Intoxication for
Which Nephrology May be Consulted
Hypertension Hypertension
Avoid blockers, which would lead to
unopposed stimulation
Acute Renal Failure
Rhabdomyolysis
Renal hypoperfusion/ischemia
Hyponatremia
Excess water intake
SIADH
Summary
Removal of drug via the GI tract
Gastric emptying Gastric emptying
Charcoal adsorption
Enhanced renal elimination
Saline diuresis
Alkalinization
Extracorporeal therapies
Hemodialysis
Hemoperfusion
Specific antidotes
Fomepizole
Ethanol
Copyright Harvard Medical School, 2010. All Rights Reserved.
1638
Serum concentrations of commons poisons in
excess of which hemodialysis or hemoperfusion
should be considered
Drug Concentration Method of Drug Concentration Method of
Choice
Theophylline 40 mg/L HP > HD
Phenobarbital 100 mg/L HP > HD
Salicylates 1000 mg/L HD
Ethylene Glycol
500 mg/L HD
Methanol 500 mg/L HD
Lithium acute 4.0 mEq/L HD
Lithium chronic 2.5 mEq/L HD
Case Presentation:
Ethylene glycol ingestion
Urine sediment: needle shaped calcium Urine sediment: needle shaped calcium
oxalate monohydrate crystals
Osmolar gap = 68
Calculated Sosm = 2 x serum Na + [glucose]/18 +
BUN/2.8
= 307
Serum osmolality 375 mosm/kg of water Serum osmolality 375 mosm/kg of water
Ethylene glycol level 38.3 mEq/L
Methanol, Ethanol, Isopropanol negative
Empty bottle of anti-freeze found in the
patients car
Copyright Harvard Medical School, 2010. All Rights Reserved.
1639
Case Presentation
Patient was emergently started on dialysis on
presentation
Rx: 8 hrs F180 Qb best Qd 800 Rx: 8 hrs, F180, Qb best, Qd 800
Post HD ethylene glycol level: 6 mmol/L
36hrs later ethylene glycol level was
undetectable without subsequent HD
treatments
24 hours after ethylene glycol levels were 24 hours after ethylene glycol levels were
undetectable, renal failure necessitated RRT
x 10 days
Patient discharged with Cr 2, still trending
down, off RRT
Acknowledgements
Dr Belinda Lee Dr. Belinda Lee
BWH/MGH Renal Fellow
Copyright Harvard Medical School, 2010. All Rights Reserved.
1640
Question 1
Hemodialysis is most effective at Hemodialysis is most effective at
removing drugs/toxins with which of
the following characteristics?
a) Highly protein bound
b) Low volume of distribution
c) Lipid soluble ) p
d) Slowly equilibrates between
blood and tissue compartments
Question 1 Answer is:
b.
Copyright Harvard Medical School, 2010. All Rights Reserved.
1641
Question 2
Which of the following may be used to block Which of the following may be used to block
the metabolism of ethylene glycol to toxic
metabolites?
a) Ethanol
b) Fomepizole
c) Thiamine
d) Pyridoxine ) y
e) All of the above
Question 2 Answer is:
e.
Copyright Harvard Medical School, 2010. All Rights Reserved.
1642
Conflicts of Interest
Consultant Consultant
Baxter Healthcare
Genzyme
Copyright Harvard Medical School, 2010. All Rights Reserved.
1643
Epo i n Anemi a Management Epo i n Anemi a Management
Ajay K. Singh, MB, FRCP, MBA
Brigham and Womens Hospital
Harvard Medical School
email: asingh@rics.bwh.harvard.edu
Di sc l osur es
Consulting Consulting
Amgen, Johnson and Johnson, Sandoz, Fibrogen
Grant Support
Amgen, Johnson and Johnson, Roche
Advisory Boards
Rockwell Rockwell
Copyright Harvard Medical School, 2010. All Rights Reserved.
1644
Obj ec t i ves
Physiology of Epo Physiology of Epo
Review RCTs
Different types of epo
Er yt hr opoei t i n
1984 Amgen develops epoetin
1986 FDA grants epo orphan drug status (expired 1996) -- protection
against competitors against competitors
1989 Treatment of Anemia associated with CRF
1993 Treatment of Anemia associated with cancer chemotherapy
1996 Presurgical administration to reduce risk of blood transfusion
1996, 2004 Addition of a new subsection in Warnings - risk in CRF (96),
solid tumors (04)
Most expensive drug for Medicare
Widely used in CKD anemia, anemia of chronic disease (cancer, HIV).
Newer applications: anemia in elderly, anemia in chronic inflammation,
heart failure, critically ill patients
1983 Fu-Kuen
Lin clones the
gene for human
erythropoietin
(EPO)
1977, Eugene
Goldwasser, first
isolated epo: 2550 liters
of urine from pts w/
aplastic anemia to
obtain 8 mg of epo
Copyright Harvard Medical School, 2010. All Rights Reserved.
1645
Er yt hr opoi et i n Ti me-l i ne
2001
2006 1980 1989 1984
Darbepoetin alfa launched
Androgens,
iron therapy,
Epoetin alfa
available for use in
Amgen develops
Epoetin alfa
Management of anemia
continues to evolve
and RBC
transfusion
CKD
and Cancer
Epogen for dialysis patients
Procrit for CKD and Cancer patients
Hypoxia:
Epo mRNA
plasma Epo
KIDNEY MARROW
Normoxia:
Epo mRNA
Copyright Harvard Medical School, 2010. All Rights Reserved.
1646
Epo and t he Epo Rec ept or
Epo receptors
Homodimeric
Bone Marrow
Heterodimeric Heterodimeric
Brain
Kidney
Myocardium
Endothelial cells
Adapted from Bunn, H. F. Blood 2007;109:868-873
Pr oduc t i on of r ed bl ood c el l s
Normal RBC production: proliferation, death, and differentiation of erythroid progenitor cell
Yoshimura, A. et al. Oncologist 1996;1:337-339
Normal RBC production: proliferation, death, and differentiation of erythroid progenitor cell
Epo controls RBC production by regulating apoptosis of erythroid progenitor cells
Insufficient Epo results in CFU-Es and proerythroblasts undergoing apoptosis
In BM epo binds receptor to regulate a family of genes bcl-2, ( inhibition of apoptosis RBCs)
Epo receptors in brain, kidney, heart
Copyright Harvard Medical School, 2010. All Rights Reserved.
1647
Obj ec t i ves
Physiology of Epo Physiology of Epo
Review RCTs
Different types of epo
Trials of Anemia Targets in CKD
Normal Hematocrit
1233 subjects, High risk symptomatic pts w/ CVD on chronic HD
Epoetin-alfa
Hb 9-11 g/dL vs. 13-15 g/dL.
Study stopped by Data and Safety and Monitoring Board
CREATE study
603 subjects, 100 centers. Non dialysis CKD, 22 countries
Epoetin-beta
Hb 13.49g/dL vs. 11.6g/dL. Early treatment vs. Late Treatment
CHOIR study
1432 subjects non dialysis CKD patients, US
Epoetin-alfa
Hb 13.5 g/dL vs 11.3 g/dL g g
Study stopped by Data and Safety and Monitoring Board
TREAT study
4,038 subjects, CKD and type 2 diabetes, 623 centers, 24 countries
Darbepoietin
Placebo-controlled with rescue arm: Hb 9.0 g/dL vs 13.0 g/d
Copyright Harvard Medical School, 2010. All Rights Reserved.
1648
The Nor mal Hemat oc r i t St udy
Tested hypothesis that patients with normal Hb 13-15
g/dL will have better outcomes than patients with Hb 9-
11 g/dL 11 g/dL
1233 HD patients with CAD or CHF
Primary end-point: Death or MI
Study terminated early due increased risk
Higher rate of vascular access thrombosis in normal Hct
group: (243 patients, or 39 percent, vs. 176 patients, or g p ( p , p , p ,
29 percent; P=0.001).
Besarab et al. N Engl J Med 339:584-590, 1998
Nor mal Hemat oc r i t St udy
Low Hct Normal Hct
n 618 615 6 8 6 5
Hct 30 42 (achieved 39%)
Epoetin dose 160 460
Total deaths 150 183
Non-fatal MI 14 19
RR 1.3 (0.9-1.9)
Besarab et al. N Engl J Med 339:584-590, 1998
Copyright Harvard Medical School, 2010. All Rights Reserved.
1649
Car di ovasc ul ar r i sk Reduc t i on by Ear l y Anaemi a
Tr eat ment w i t h Epoei t i n- (CREATE)
600 patients, 100 centers, from 22 countries
Epoeitin-
Randomisation
Early Rx gp Late Rx gp Early Rx gp
High target Hb
(1315 g/dl)
Late Rx gp
Standard target Hb
(10.511.5 g/dl)
Group 1 Group 2
Drueke TB et al. N Engl J Med 2006;355:2071-2084
Hemogl obi n over t i me
Drueke TB, et al N Engl J Med. 355:2071-84, 2006
Copyright Harvard Medical School, 2010. All Rights Reserved.
1650
Pr i mar y endpoi nt
Ti me t o f i r st CV event s (105 event s)
Events: 58 vs 47
HR=0.78 (0.531.14)
Log rank test p=0.20
Drueke TB, et al N Engl J Med. 355:2071-84, 2006
Sec ondar y endpoi nt s
Ti me t o di al ysi s
1.0
0 9
Probability
Events: 127 vs 111
HR=0 76 (0 59 0 98)
0.9
0.8
0.7
0.6
0.5
0.4
0.3
0.2
Group 1
Group 2
HR=0.76 (0.590.98)
Log rank test p=0.034
Gp 1
Gp 2
n at risk
Group 1 301 281 255 211 162 115 62 35 0
Group 2 302 293 269 243 199 138 82 33 0
0 6 12 18 24 30 36 42 48
0.1
0
Study month
Group 2
Copyright Harvard Medical School, 2010. All Rights Reserved.
1651
CREATE Resul t s: QOL
Drueke TB, et al N Engl J Med. 355:2071-84, 2006
Patients knew which arm they were randomized to
1
st
yr: 98% of patients in high arm received injections ;only 32% in the low arm.
Low Hb arm pts had to develop worsening anemia prior to epoetin therapy
CHOI R: Pat i ent Fl ow
1432 patients, 130 centers, US only
Epoetin-alfa
Median f/u 16 months
Randomization
High target Hgb
(13.5 g/dl)
n=715
Low target Hgb
(11.3 g/dl)
n=717
Median f/u 16 months
Singh et al,New Engl J Med 2006; 355:2085-98
Copyright Harvard Medical School, 2010. All Rights Reserved.
1652
Mean Week l y Doses of Epoet i n al f a and Hgb
RandomizedTreatment
HemoglobinTarget 13.5g/dL .
1
8
0
0
0
2
0
0
0
0
RandomizedTreatment
HemoglobinTarget 13.5g/dL .
1
8
0
0
0
2
0
0
0
0
RandomizedTreatment
HemoglobinTarget 13.5g/dL .
1
8
0
0
0
2
0
0
0
0
RandomizedTreatment
HemoglobinTarget 13.5g/dL .
1
8
0
0
0
2
0
0
0
0
Mean Weekly Dose
11,215 units
RandomizedTreatment
HemoglobinTarget 13.5g/dL
HemoglobinTarget 11.3g/dL .
1
4
.5
1
5
1
5
.5
RandomizedTreatment
HemoglobinTarget 13.5g/dL
HemoglobinTarget 11.3g/dL .
1
4
.5
1
5
1
5
.5
RandomizedTreatment
HemoglobinTarget 13.5g/dL
HemoglobinTarget 11.3g/dL .
1
4
.5
1
5
1
5
.5
RandomizedTreatment
HemoglobinTarget 13.5g/dL
HemoglobinTarget 11.3g/dL .
1
4
.5
1
5
1
5
.5
Mean Hb 12.6 g/dL
NNNN
HemoglobinTarget 13.5g/dL
HemoglobinTarget 11.3g/dL
M
e
a
n
E
p
o
ie
tin
-
a
lf
a
D
o
s
e
(
U
)
a
n
d
9
5
%
C
.I
2
0
0
0
4
0
0
0
6
0
0
0
8
0
0
0
1
0
0
0
0
1
2
0
0
0
1
4
0
0
0
1
6
0
0
0
St d M th
0 2 4 6 8 10 12 14 16 18 20 22 24 26 28 30 32 34 36
HemoglobinTarget 13.5g/dL
HemoglobinTarget 11.3g/dL
M
e
a
n
E
p
o
ie
tin
-
a
lf
a
D
o
s
e
(
U
)
a
n
d
9
5
%
C
.I
2
0
0
0
4
0
0
0
6
0
0
0
8
0
0
0
1
0
0
0
0
1
2
0
0
0
1
4
0
0
0
1
6
0
0
0
St d M th
0 2 4 6 8 10 12 14 16 18 20 22 24 26 28 30 32 34 36
HemoglobinTarget 13.5g/dL
HemoglobinTarget 11.3g/dL
M
e
a
n
E
p
o
ie
tin
-
a
lf
a
D
o
s
e
(
U
)
a
n
d
9
5
%
C
.I
2
0
0
0
4
0
0
0
6
0
0
0
8
0
0
0
1
0
0
0
0
1
2
0
0
0
1
4
0
0
0
1
6
0
0
0
St d M th
0 2 4 6 8 10 12 14 16 18 20 22 24 26 28 30 32 34 36
HemoglobinTarget 13.5g/dL
HemoglobinTarget 11.3g/dL
M
e
a
n
E
p
o
ie
tin
-
a
lf
a
D
o
s
e
(
U
)
a
n
d
9
5
%
C
.I
2
0
0
0
4
0
0
0
6
0
0
0
8
0
0
0
1
0
0
0
0
1
2
0
0
0
1
4
0
0
0
1
6
0
0
0
St d M th
0 2 4 6 8 10 12 14 16 18 20 22 24 26 28 30 32 34 36
Mean Weekly Dose
6276 units
NNNN
g g g
M
e
a
n
H
e
m
o
g
lo
b
in
(
g
/d
L
)
a
n
d
9
5
%
C
.I
.
9
.5
1
0
1
0
.5
1
1
1
1
.5
1
2
1
2
.5
1
3
1
3
.5
1
4
St d M th
0 2 4 6 8 10 12 14 16 18 20 22 24 26 28 30 32 34 36
g g g
M
e
a
n
H
e
m
o
g
lo
b
in
(
g
/d
L
)
a
n
d
9
5
%
C
.I
.
9
.5
1
0
1
0
.5
1
1
1
1
.5
1
2
1
2
.5
1
3
1
3
.5
1
4
St d M th
0 2 4 6 8 10 12 14 16 18 20 22 24 26 28 30 32 34 36
g g g
M
e
a
n
H
e
m
o
g
lo
b
in
(
g
/d
L
)
a
n
d
9
5
%
C
.I
.
9
.5
1
0
1
0
.5
1
1
1
1
.5
1
2
1
2
.5
1
3
1
3
.5
1
4
St d M th
0 2 4 6 8 10 12 14 16 18 20 22 24 26 28 30 32 34 36
g g g
M
e
a
n
H
e
m
o
g
lo
b
in
(
g
/d
L
)
a
n
d
9
5
%
C
.I
.
9
.5
1
0
1
0
.5
1
1
1
1
.5
1
2
1
2
.5
1
3
1
3
.5
1
4
St d M th
0 2 4 6 8 10 12 14 16 18 20 22 24 26 28 30 32 34 36
Mean Hb 11.3 g/dL
N
Hb13.5 709 693 659 623 578 530 500 452 370 310 258 189 132 97 79 65 52 27 11
Hb11.3 707 691 655 621 577 549 526 479 393 333 262 189 141 95 73 54 43 27 12
N
Hb13.5 709 693 659 623 578 530 500 452 370 310 258 189 132 97 79 65 52 27 11
Hb11.3 707 691 655 621 577 549 526 479 393 333 262 189 141 95 73 54 43 27 12
N
Hb13.5 709 693 659 623 578 530 500 452 370 310 258 189 132 97 79 65 52 27 11
Hb11.3 707 691 655 621 577 549 526 479 393 333 262 189 141 95 73 54 43 27 12
N
Hb13.5 709 693 659 623 578 530 500 452 370 310 258 189 132 97 79 65 52 27 11
Hb11.3 707 691 655 621 577 549 526 479 393 333 262 189 141 95 73 54 43 27 12
StudyMonth StudyMonth StudyMonth StudyMonth N
Hb13.5 710 667 632 600 558 507 485 433 367 306 252 194 139 95 81 67 49 31 13
Hb11.3 707 672 625 603 549 528 510 471 384 334 250 182 141 101 75 60 45 30 13
N
Hb13.5 710 667 632 600 558 507 485 433 367 306 252 194 139 95 81 67 49 31 13
Hb11.3 707 672 625 603 549 528 510 471 384 334 250 182 141 101 75 60 45 30 13
N
Hb13.5 710 667 632 600 558 507 485 433 367 306 252 194 139 95 81 67 49 31 13
Hb11.3 707 672 625 603 549 528 510 471 384 334 250 182 141 101 75 60 45 30 13
N
Hb13.5 710 667 632 600 558 507 485 433 367 306 252 194 139 95 81 67 49 31 13
Hb11.3 707 672 625 603 549 528 510 471 384 334 250 182 141 101 75 60 45 30 13
StudyMonth StudyMonth StudyMonth StudyMonth
Singh et al,New Engl J Med 2006; 355:2085-98
Kapl an-Mei er Pl ot of t he Ti me t o t he Pr i mar y
Composi t e Event bet ween Randomi zat i on and
Ter mi nat i on: I TT Popul at i on
Randomized Treatment
HemoglobinTarget 13.5g/dL
HemoglobinTarget 11.3g/dL
25%
30%
125
K
a
p
l
a
n
-
M
e
i
e
r

F
a
i
l
u
r
e

E
s
t
i
m
a
t
e

(
%
)
5%
10%
15%
20%
Hazard ratio 1.337 (1.025, 1.743)
P= 0.0312
97
Primary Composite Endpoint:
Death, MI, CHF hosp (no RRT) and/or stroke
0%
Months fromRandomization
0 6 12 18 24 30 36
715
717
587
594
457
499
270
293
55
44
101
107
0
3
Copyright Harvard Medical School, 2010. All Rights Reserved.
1653
10%
15%
20%
10%
15%
20%
CHF Hospitalization (where RRT
did not occur)
Death
p = 0.0727
p = 0.0674
Component s of t he Pr i mar y Endpoi nt
Hazard ratio 1.483
(0.969, 2.268)
Hazard ratio 1.409
(0.967, 2.054)
65 deaths
0%
5%
10%
0 6 12 18 24 30 36
0%
5%
10%
0 6 12 18 24 30 36
15%
20%
15%
20%
Myocardial Infarction
Stroke
p = 0.9803 p = 0.7836
Hazard ratio 1.010
(0 454 2 249)
Hazard ratio 0.915
(0 484 1 729)
Randomized Treatment Hemoglobin Target 13.5 g/dL Hemoglobin Target 11.3 g/dL
0%
5%
10%
Months fromRandomization
0 6 12 18 24 30 36
0%
5%
10%
Months fromRandomization
0 6 12 18 24 30 36
(0.454, 2.249) (0.484, 1.729)
CHOI R: QOL
3 instruments
LASA
KDQ
SF-36
QOL Increased in both groups
No difference between groups
Singh et al,New Engl J Med 2006; 355:2085-98
Copyright Harvard Medical School, 2010. All Rights Reserved.
1654
Tr i al t o Reduc e Car di ovasc ul ar
Event s w i t h Ar anesp Ther apy
Hypothesis
In patients with type 2 diabetes, chronic kidney
disease not requiring dialysis, and concomitant
anemia, raising hemoglobin with darbepoetin
alfa would lower the rates of death and
cardiovascular morbidity and/or death and end- y
stage renal disease
Pfeffer MA, et al N Engl J Med. Oct 30. 2009
Desi gn
International (24) multi-center (623), double-blind,
placebo-controlled randomized trial
Randomization in a 1:1
A ti d b ti lf t tt t t hi d i t i Active: darbepoetin alfa to attempt to achieve and maintain a
hemoglobin (Hb) of 13 g/dL
Control: placebo: with rescue therapy if
Hb <9.0 g/dL with resumption of placebo once Hb9.0 g/dL
Blinding maintained during computer directed
adjustments assigning pre-filled syringes
2 composite primary endpoints* co pos te p a y e dpo ts
Cardiovascular composite: death, MI, myocardial ischemia, CHF,
stroke. alpha 0.048
Renal composite: death or ESRD.
alpha 0.002
Pfeffer MA, et al N Engl J Med. Oct 30. 2009
Copyright Harvard Medical School, 2010. All Rights Reserved.
1655
Hemogl obi n Level s
/
d
L
)
13.0
13.5
Placebo
Darbepoetin alfa
e
a
n

H
e
m
o
g
l
o
b
i
n

(
g
/
10.0
10.5
11.0
11.5
12.0
12.5
Median dose: 176 g IQR [104 305]
Mean: 225 g 208
Median dose: 0 g IQR [0 5]
Mean: 5 g 11
Hb Median: 12.5 IQR [12.0
12.8]
Hb Median: 10.6 IQR [9.9
11.3]
M
9.5
0 0
Month
No. of Patients
Placebo
Darbepoetin alfa
0
0 6 12 18 24 30 36 42 48
2019 1742 1460 1221 887 620 356 216 79
2004 1768 1503 1300 946 635 404 253 97
TREAT Composi t e endpoi nt s
Pfeffer MA, et al N Engl J Med. Oct 30. 2009
Copyright Harvard Medical School, 2010. All Rights Reserved.
1656
50
Al l Cause Mor t al i t y
Darbepoetin alfa 412 (20 5%)
20
30
40
n
t
s

W
i
t
h

E
v
e
n
t
s

(
%
)
HR: 1.05 (0.92-1.21)
Darbepoetin alfa 412 (20.5%)
Placebo 395 (19.5%)
0
10
0 6 12 18 24 30 36 42 48
P
a
t
i
e
n
Months
( )
P = 0.48
50
Fat al and Nonf at al St r ok e
Darbepoetin alfa 101 (5 0%)
20
30
40
n
t
s

W
i
t
h

E
v
e
n
t
s

(
%
)
HR: 1.92 (1.38
2.68)
P < 0 001
Darbepoetin alfa 101 (5.0%)
Placebo 53 (2.6%)
0
10
0 6 12 18 24 30 36 42 48
P
a
t
i
e
n
Months
P < 0.001
Copyright Harvard Medical School, 2010. All Rights Reserved.
1657
Mal i gnanc y i n TREAT
Darbepoetin alfa Placebo P-value
Overall
Cancer-related AE
139/2012
6.9%
130/2026
6.4%
0.53
Deaths attributed to
cancer
39/2012
1.9%
25/2026
1.2%
0.08
Subgroup: Baseline History of malignancy (n = 348) Subgroup: Baseline History of malignancy (n = 348)
All cause mortality
60/188
31.9%
37/160
23.1%
0.13
Deaths attributed to
cancer
14/188
7.4%
1/160
0.6%
0.002
Pat i ent Repor t ed Out c omes
FACT-Fat i gue Sc or e at 25 Week s
P = 0.002
54.7%
4.2 10.5 2.8 10.3 P < 0.001
49.5%
L
e
s
s

f
a
t
i
g
u
e
M
e
a
n

C
h
a
n
g
e
30.2
Darbepoetin alfa
n = 1762
30.4 Placebo
n = 1769
FACT-Fatigue range: 0: most fatigued, to 52: least fatigued
Copyright Harvard Medical School, 2010. All Rights Reserved.
1658
Pat i ent Repor t ed Out c omes
Suppor t i ve Anal ysi s
Short Form 36 Mean Change at
Domains
Darbepoetin alfa
n = 1138
Placebo
n = 1157 P-value
Energy 2.6 9.9 2.1 9.7 0.20
Short-Form 36 Mean Change at
25 Weeks in 2 Domains
Physical Function 1.3 9.2 1.1 8.8 0.51
Ri sk s and Benef i t s
Risks
Cardiovascular
Thromboembolism
Cancer progression
No benefit in renal progression ? Higher rate of
ESRD
Hypertension
Benefits
Reducing blood transfusion rate
Improvement in Fatigue and QOL
Copyright Harvard Medical School, 2010. All Rights Reserved.
1659
What ex pl ai ns het er ogenei t y
of adver se out c omes?
Class effect
Epoetin-alfa
Darbepoetin-alfa
Epoetin-beta
Exposure to higher and different doses of ESA
CHOIR higher hemoglobin arm received median of 10,952 units per
week.
TREAT median dose of 8800 units per week in the darbepoetin
treated arm
CREATE, median dose of 5000 units of epoetin-beta per week was
used in the higher hemoglobin arm used in the higher hemoglobin arm.
Enrollment of different populations?
TREAT, all diabetics
CHOIR 50% diabetics
CREATE 25% diabetes mellitus
What ex pl ai ns I nc r eased Ri sk
High Hb hyperviscosity? Activation of
endothelial cells/ platelets endothelial cells/ platelets
Rise in Hb
Activation of heterodimeric ESA receptor
Exposure to high doses of ESA
Iron exposure
Hypertension in ESA treated patients Hypertension in ESA treated patients
Copyright Harvard Medical School, 2010. All Rights Reserved.
1660
Mec hani sms of ESA t ox i c i t y
Singh AK, CJASN. Apr 2010; 5: 553 - 556
Copyright Harvard Medical School, 2010. All Rights Reserved.
1661
r HuEPO st i mul at es er yt hr opoi esi s by
ac t i vat i ng EPO r ec ept or s
rHuEPO
membrane
Homodimeric
EPO Receptor
Heterodimeric
EPO Receptor
Bone Marrow
Heart, Brain, Kidney,
endothelial cells
Mec hani sms of ESA t ox i c i t y
Copyright Harvard Medical School, 2010. All Rights Reserved.
1662
Assoc i at i on bet w een dose and pr i mar y
out c ome i n Gr oup A (9 mont h Landmar k )
Szczech LA, et al: Kidney Int 74: 791798, 2008
Obj ec t i ves
Physiology of Epo Physiology of Epo
Review RCTs
Different types of epo
Copyright Harvard Medical School, 2010. All Rights Reserved.
1663
I. EPO-receptor agonists
Protein-based ESA therapy
Epoetin (alfa, beta, delta, omega)
Bi i il EPO ( ti t ) Biosimilar EPOs (epoetin zeta)
Darbepoetin alfa
C.E.R.A. (methoxy polyethylene glycol epoetin beta)
Synthetic erythropoiesis protein (SEP)
EPO fusion proteins
~ EPOEPO
~ GMCSFEPO
~ FcEPO
~ CTNO 528
Small molecule ESAs
Peptide-based (e.g. Hematide)
Non-peptide based
Additional
carbohydrate
side chains
Aranesp
TM rHuEPO
carbohydrate
side chains
Molecular Comparison of
Aranesp (darbepoetin alfa) and rHuEPO
3 N-linked (CHO) chains 5 N-linked (CHO) chains
Receptor 1
Receptor 2
Receptor 1
Receptor 2
Maximum 14 sialic acids
MW ~ 30,400 daltons
40% carbohydrate
Maximum 22 sialic acids
MW ~ 37,100 daltons
51% carbohydrate
Copyright Harvard Medical School, 2010. All Rights Reserved.
1664
Compar i son of Mean Hal f -Li ves of
ESAs
First-generation: recombinant
human erythropoietin (epoetin alfa
and epoetin beta)
Meal Half-Lives of ESAs
p )
Second-generation: darbepoetin
alfa
2 more carbohydrate chains
and up to 8 more sialic acid
residues than epoetin
Third-generation: continuous
erythropoietin receptor activator
(C.E.R.A.)
Integrated a large polymer
chain into the molecule chain into the molecule
Unexplained loss of effect
Anemia (Hb decreases by approx 0.1 g/d
Low reticulocyte count
Ant i Ant i - -Epo Epo Abs and Abs and
Pur e Red Cel l Pur e Red Cel l Apl asi a Apl asi a
Low reticulocyte count
Plats/WBCs normal
Bone marrow
Normal cellularity except erythroblasts very rare
Positive Epo antibody test
In Epo treated PRCA p
Virtually all cases observed in renal patients (2
cases in MDS patients)
No cases in cancer patients
No cases with exclusive IV exposure, high
correlation w/ SC exposure w/ Eprex
Copyright Harvard Medical School, 2010. All Rights Reserved.
1665
Ant i Ant i - -Epo Epo Abs and Abs and
Pur e Red Cel l Pur e Red Cel l Apl asi a Apl asi a
First reported by
Bergrem et al, 1993
Median duration of
treatment with Epo 7
months (range 1 month
5 years)
Approx 191 cases 1998-
2004
Estimates of the Worldwide Exposure-Adjusted Incidence of Epoetin-Associated Pure Red-Cell Aplasia
According to the Product, between January 1, 2001, and December 31, 2003
Bennett, C. L. et al. N Engl J Med 2004;351:1403-1408
Copyright Harvard Medical School, 2010. All Rights Reserved.
1666
Di st r i but i on of Epr ex -assoc i at ed PRCA
Cases of Antibody-Positive, Eprex-Associated Pure Red-Cell Aplasia Identified
in the Database of the Adverse Event Reporting System of the Food and Drug
Administration between January 1998 and April 2004
Bennett, C. L. et al. N Engl J Med 2004;351:1403-1408
Quest i on 1
58 years old male with ESRD for past 10 years on 3 times/week
dialysis Complaining of weakness and feeling blah Self reported dialysis. Complaining of weakness and feeling blah. Self-reported
fevers at night when at home. ESRD from diabetes mellitus. H/O HTN,
PUD, CAD. s/p kidney transplant (living related) back on dialysis for 4
months. Medications: Epogen 22000 units TIW, maintenance iron,
Zamplar 4 mcg per treatment, ASA, statin, beta blocker, PPI,
prednisone 5 mg/d. Examination: pale,162 cm, 75,8 Kg (BMI 28,9
Kg/m
2
), Pre-HD Blood pressure: 150/78. post-HD 135/70. Left AV
Fistula functioning, mild abdominal tenderness . Kt/V 1.32, PTH 248
Hemoglobin 9.2 g/dL,
Reticulocytes 1.9%,
Ferritin 430 g/L,
Transferrin Saturation 26%
Copyright Harvard Medical School, 2010. All Rights Reserved.
1667
Whi c h one of t he f ol l ow i ng
st at ement s i s c or r ec t
A) The dose of his vitamin D analogue should
be increased be increased
B) The patient should be treated with a
course of IV iron
C) The patient requires a blood transfusion in
order to get his Hb into the 11 to 12 g/dL
range recommended by the NKF range recommended by the NKF
D) The most likely cause of his ESA
resistance is a failed kidney allograft
E) He should be treated with an anabolic
steroid
Whi c h one of t he f ol l ow i ng
st at ement s i s c or r ec t
A) The dose of his vitamin D analogue should
be increased be increased
B) The patient should be treated with a
course of IV iron
C) The patient requires a blood transfusion in
order to get his Hb into the 11 to 12 g/dL
range recommended by the NKF range recommended by the NKF
D) The most likely cause of his ESA
resistance is a failed kidney allograft
E) He should be treated with an anabolic
steroid
Copyright Harvard Medical School, 2010. All Rights Reserved.
1668
Causes of Epo Resi st anc e
Iron deficiency
Inflammation infectious and chronic microinflammatory
Failed kidney allograft
Chronic infection (TC, diabetic ulcer, amputation stump
infection) infection)
Oxidative stress
Inadequate dialysis
Hyperparathyroidism
Carnitine deficiency
Drugs: ACE-I/ARB
Aluminum overload
N t iti l F l t it i B12 it i C d fi i Nutritional: Folate, vitamin B12, vitamin C deficiency
Malignancy
Hematological disorders
Quest i on 2
65 years old female, is referred for evaluation of diabetic
nephropathy. Feels tired. Medications: Lisinopril 40 mg/d,
amlodipine 10 mg/d, atorvostatin 20 mg/d, glipizide, ASA. Physical
examination: 114/82 mmHg, height 1.57 m, weight 51.5 Kg, BMI
20.9 Kg/m
2
. Otherwise normal. Labs show a Cr 1.1, BUN 20,
UACR 312 mg/dg Hb of 9.8 g/dL, MCV 92,3 fL, MHC 30,5 pg,
MCHC 33,0 g/dL. ferritin 212, Tsat 26%, normal folate, negative
stool guaic
The most likely cause of anemia in this patient is:
A.) Anemia of CKD
B.) Increased levels of AcSDKP
C.) Iron deficiency
D.) Diabetes associated ESA resistance
E.) A bone marrow process
Copyright Harvard Medical School, 2010. All Rights Reserved.
1669
Quest i on 2
65 years old female, is referred for evaluation of diabetic
nephropathy. Feels tired. Medications: Lisinopril 40 mg/d, nephropathy. Feels tired. Medications: Lisinopril 40 mg/d,
amlodipine 10 mg/d, atorvostatin 20 mg/d, glipizide, ASA. Physical
examination: 114/82 mmHg, height 1.57 m, weight 51.5 Kg, BMI
20.9 Kg/m
2
. Otherwise normal. Labs show a Cr 1.1, BUN 20,
UACR 312 mg/dg Hb of 9.8 g/dL, MCV 92,3 fL, MHC 30,5 pg,
MCHC 33,0 g/dL. ferritin 212, Tsat 26%, normal folate, negative
stool guaic
The most likely cause of anemia in this patient is:
A ) Anemia of CKD A.) Anemia of CKD
B.) Increased levels of AcSDKP
C.) Iron deficiency
D.) Diabetes associated ESA resistance
E.) A bone marrow process
Di f f er ent i al Di agnosi s of
Anemi a
Epo deficiency Epo deficiency
Functional Iron deficiency
ACEi therapy (lisinopril)
Inflammation (?diabetic ulcer/infection)
Folate and B12 deficiency
Hyperparathyroidism
Occult blood loss
Hemoglobinopathy
Copyright Harvard Medical School, 2010. All Rights Reserved.
1670
Erythropoiesis in CKD
EPO production
EPO
+ +
Iron
Fas Ag
Apoptosis

Pro-inflammatory
cytokines
(IL-1, TNF, IL-6, IFN)
hepcidin
Fe absorption
Fe transport
Fe availability
(EPO-R, Tf, TfR,
Ferriportin, DMT-1)

ACEi assoc i at ed anemi a


Circulating natural inhibitor of bone marrow
(N-acetyl-seryl-aspartyl-lysyl-proline
(AcSDKP) (AcSDKP)
The ACE enzyme is responsible for the
degradation of AcSDKP, thus blockade of ACE
with ACEI leads to increased circulating levels of
AcSDKP and therefore bone marrow inhibition (Le
Meur et al 2001).
E th id it ll li b Erythroid progenitor cell lines can be
stimulated via angiotensin type II receptors
therefore blockade of these receptors by ARB
may inhibit differentiation of erythroid cell
lines (Mohanram et al 2008).
Copyright Harvard Medical School, 2010. All Rights Reserved.
1671
Conc l usi ons
Epo is a an anti-apoptotic growth factor
RCTs demonstrate increased risk with
targeting a higher Hb, however reducing
blood transfusion rate and improving fatigue
are important benefits
Mechanism of risk unclear, but several
hypotehses are being considered including
ESA d ESA dosage
Several ESAs on market and in development
Di sc l osur es
Consulting Consulting
Amgen, Johnson and Johnson, Sandoz, Fibrogen
Grant Support
Amgen, Johnson and Johnson, Roche
Advisory Boards
Rockwell Rockwell
Copyright Harvard Medical School, 2010. All Rights Reserved.
1672
Intradialytic Complications:
Case Studies
Bertrand L. Jaber, MD, MS, FASN
Associate Professor of Medicine
St. Elizabeths Medical Center
Tufts University School of Medicine
The Brigham Renal Board Review Course
The Fairmont Copley Plaza, Boston, MA
August 12, 2010
Faculty Disclosure Information
Member, Scientific Advisory Board, NxStage
Medical, Inc.
Copyright Harvard Medical School, 2010. All Rights Reserved.
1673
Case 1
Five minutes after starting dialysis, a
45-year-old man complains of
burning sensation at the site of the
arteriovenous fistula followed by the y
acute onset of dyspnea
What Should You Not Do?
(A) Immediately stop dialysis
(B) Return extracorporeal blood to patient
(C) Provide supportive measures (e.g., oxygen,
h d ti ti hi t i ) hydrocortisone, anti-histamines)
(D) Investigate the incident immediately
Copyright Harvard Medical School, 2010. All Rights Reserved.
1674
Onset
Investigating a Dialyzer Reaction
Symptoms
Predisposing factor(s)
Dialyzer characteristics
Medication history
Copyright Harvard Medical School, 2010. All Rights Reserved.
1675
Determine Dialyzer Characteristics
Membrane
Type
Manufacturers
Sterilant
Reuse Reuse
Germicide
Cellulose
Cellulose acetate
ETO
Steam
Yes
No
Renalin
Formaldehyde Cellulose acetate
Synthetic polymer
Steam
-irradiated
No Formaldehyde
Glutaraldehyde
Copyright Harvard Medical School, 2010. All Rights Reserved.
1676
Case 1 The Investigation
The patient had consented for the
dialyzer reuse program, and had been
receiving dialysis with a reprocessed
high-flux polysulfone dialyzer. The reuse
i id f th f ilit germicide of the facility was
formaldehyde and on the day of the
incident, the dialyzer was at its 5
th
reuse.
(A) Reaction due to reverse passage of bacterial
With this Additional Knowledge, Select the
Most Likely Cause of this Incident
(A) Reaction due to reverse passage of bacterial
products across the dialyzer
(B) Formaldehyde-mediated reaction
(C) Reaction due to complement activation
(D) Bradykinin-mediated reaction
Copyright Harvard Medical School, 2010. All Rights Reserved.
1677
Case 1 Conclusion
Direct blood exposure to formaldehyde, which
was improperly rinsed from the dialyzer
The burning sensation at the site of the AVF is The burning sensation at the site of the AVF is
highly suggestive of a germicide-related
reaction
Copyright Harvard Medical School, 2010. All Rights Reserved.
1678
Case 2
Five minutes after starting dialysis, a 65-year-
old woman complains of a scratchy throat and
light headedness. Dialysis is immediately
stopped and she requires 1 mg of epinephrine.
Upon further review, she was initiated on
hemodial sis in the pre io s eek and had hemodialysis in the previous week, and had
been using a single-use polysulfone dialyzer.
(A) Thi I E di t d i d t th l id
The Following Statements Regarding Dialyzer First
Use Reactions are
True Except
(A) This IgE-mediated response is due to ethylene oxide
(ETO), the manufacturer's dialyzer sterilant
(B) ETO is released from the dialyzer potting compound and
becomes allergenic after binding to albumin
(C) Anti-ETO-albumin IgE antibodies are detected in all ( ) g
patients with these reactions
(D) Reprocessing ETO-sterilized dialyzers prior to first use
or switching to non-ETO-sterilized dialyzers can prevent
future reactions
Copyright Harvard Medical School, 2010. All Rights Reserved.
1679
Manufacturers Dialyzer Sterilization Method
Manufacturer Dialyzer model Sterilization method
Gambro Polyflux

Steam Gambro Polyflux

Steam
Fresenius F

Optiflux

ETO
ETO, E-beam
Baxter CA

CT

ETO
-irradiation
Asahi AM

PAN

-irradiation
ETO
Toray BK

-irradiation
Copyright Harvard Medical School, 2010. All Rights Reserved.
1680
Case 3
The nurse calls you to assess a 60 year-
old woman with ESRD and COPD, who
complains of acute back pain and mild
shortness of breath, 20 minutes after shortness of breath, 20 minutes after
starting dialysis.
All of the Following Statements
Regarding This Incident are True Except
(A) The absence of allergic symptoms is reassuring
(B) Dialysis should be stopped immediately
(C) Treatment is supportive with oxygen and analgesics
(D) Symptoms usually abate after the first hour of dialysis
(E) Complement activation may have played a role
Copyright Harvard Medical School, 2010. All Rights Reserved.
1681
These Mild Dialyzer Reactions are Prevented by
Use of the Following Dialyzer Types Except
(A) Synthetic polymers
(B) Unsubstituted cellulose reprocessed with bleach
(C) Substituted cellulose
(D) Unsubstituted cellulose reprocessed without bleach
Copyright Harvard Medical School, 2010. All Rights Reserved.
1682
Over a 4-month period 12 anaphylactoid reactions occured in
Dialysis Outbreak 1
Over a 4 month period, 12 anaphylactoid reactions occured in
10 patients at a dialysis facility. One patient required
hospitalization but no patients died. These reactions occurred
within minutes of initiating dialysis and were characterized by
peripheral numbness and tingling, laryngeal edema or
angioedema, facial or generalized sensation of warmth,
and/or nausea or vomiting. All 12 reactions occurred with g
dialyzers that had been reprocessed with an automated
reprocessing system.
Copyright Harvard Medical School, 2010. All Rights Reserved.
1683
The CDC Investigates and Calculates
Attack Rates
17% 25
50
75
100
A
t
t
a
c
k

R
a
t
e

(
%
)
70%
25
50
75
100
A
t
t
a
c
k

R
a
t
e

(
%
)
0%
0
Renalin-
Reprocessed
Dialyzer
Single Use
Dialyzer
Pegues: Kidney Int 42:1232-7, 1992
10%
0
25
ACEI+ ACEI-
A
What Conclusion Can be Drawn?
(A) Blood exposed to renalin resulted in complement
activation and an anaphylactoid reaction
(B) Interaction between renalin-reprocessed dialyzers, blood
and use of ACEI resulted in an anaphylactoid reaction
(C) The investigation is inconclusive
Copyright Harvard Medical School, 2010. All Rights Reserved.
1684
Bradykinin-Mediated Reactions:
Membrane Surface Charge
Membrane Charge
Cuprophan 0
Polyamide 0
Cellulose Acetate 3 Cellulose Acetate -3
Polyacrylonitrile (AN69) -154
Pr ot ei n-c oat ed neut r al membr ane r enal i n ex posur e
pr ot ei n ox i dat i on Negat i vel y char ged membr ane
Copyright Harvard Medical School, 2010. All Rights Reserved.
1685
Negatively
Surface Activation
Kinin Generation
charged
renalin-
reprocessed
dialyzer
membrane
Surface Activation
Factor XII (HF) XIIa
Prekallikrein Kallikrein
Kininogen
Kinins
HMWK
Kinins
Degradation
Kininase
ACEI
X
400 m
l
)
Kinin Generation
328
100
200
300
400
r
a
d
y
k
i
n
i
n

(
f
m
o
l
/
m
5
17
40
0
100
Cellulose Hemophan Polysulfone AN69 P
l
a
s
m
a

B
r
Verresen: Kidney Int 45:1497-1503, 1994
Copyright Harvard Medical School, 2010. All Rights Reserved.
1686
Case 4
Two hours after starting dialysis, a 25
year-old man with ESRD develops chills
followed by a recorded temperature of
38.5
o
C. The patient has a functioning 38.5 C. The patient has a functioning
arteriovenous fistula and the dialyzer is
at its 20
th
reuse.
Which of the Following Statements Regarding
Pyrogenic Reactions is Not True?
(A) They can occur at any time during dialysis
(B) Blood cultures and broad spectrum antibiotics are often
required
(C) This is a diagnosis of exclusion
(D) Pyrogenic reactions are due to endotoxin contamination
(E) They are more common in dialysis facilities that practice
dialyzer reuse and use low-flux dialyzers
Copyright Harvard Medical School, 2010. All Rights Reserved.
1687
A 75 year old diabetic male with end stage renal
Case 5
A 75-year-old diabetic male with end-stage renal
disease has recurrent intradialytic hypotension due
to autonomic dysfunction. All of the following
preventive strategies are acceptable except:
A Use of sodium modeling A. Use of sodium modeling
B. Use of cool dialysate
C. Use of acetate dialysate
D. Use of L-carnitine
Copyright Harvard Medical School, 2010. All Rights Reserved.
1688
Pathogenesis and Causes of Intradialytic
Hypotension
Copyright Harvard Medical School, 2010. All Rights Reserved.
1689
BP (mm Hg) Control Midodrine Cool Dialysate
Combination
Therapy P
Intradialytic Hypotension
Midodrine and Cool Dialysate
( g) Co t o dod e Coo a ysate e apy
Pre-HD
SBP 132.7 3.4 136.1 2.5 132.0 3.3 133.5 3.3 0.73
DBP 74.9 3.0 75.9 2.0 73.7 2.3 74.8 2.0 0.66
Lowest ID
SBP 90.6 2.5 103.9 4.1 102.5 2.9 103.7 4.2 <0.001
DBP 54.9 2.2 62.3 2.9 61.7 2.3 62.6 2.6 0.02
Post-HD
SBP 109.0 2.1 116.9 4.0 118.1 3.5 122.1 4.6 0.009
DBP 63.6 1.9 66.9 2.5 69.2 2.6 68.4 3.0 0.01
Midodrine = 10 mg pre-dialysis; Cool dialysate = 35.5
o
C
Cruz: Am J Kidney Dis
33:920-926, 1999
() Control
Intradialytic Mean Arterial Pressures
() Midodrine
() Cool dialysate
() Combination
Cruz: Am J Kidney Dis
33:920-926, 1999
Copyright Harvard Medical School, 2010. All Rights Reserved.
1690
K-DOQI Guidelines:
L-Carnitine for Intradialytic Hypotension
1. Administration of L-carnitine (20 mk/kg i.v. post-dialysis)
is recommended for hemodialysis patients who, without
clinically identifiable causes, repeatedly experience
symptomatic intradialytic hypotension that requires a
therapeutic intervention
2. Response to L-carnitine should be evaluated at 3-month
intervals. Patient evaluation of the response to L-carnitine
should include:
Reduced frequency of hypotensive episodes, OR
Reduced clinical symptoms
3. L-carnitine therapy should be discontinued if no clinical
improvement has occurred within 9 to 12 months
Ek noyan: Am J Ki dney Di s 41:868-876, 2003
A 67-year-old diabetic male with end-stage renal
Case 6
A 67 year old diabetic male with end stage renal
disease has persistent intradialytic hypertension.
Which of the following preventive strategies are
acceptable?
A. Use of isonatric and isocalcic dialysate A. Use of isonatric and isocalcic dialysate
B. Change to SC Epogen
C. Use of a short acting ACE inhibitor pre-dialysis
D. All of the above
Copyright Harvard Medical School, 2010. All Rights Reserved.
1691
Vexing problem
Definition: sudden increase in SBP > 30 mm Hg from
baseline
Intradialytic Hypertension
baseline
Less frequent than hypotension
Potential risk factor for cardiovascular morbidity/mortality
Causes:
Sympathetic/RAS hormonal surge during UF
Underlying renovascular disease
Intra-dialytic removal of anti-hypertensive drugs
Hypernatric dialysate
Hypercalcic dialysate
IV Epogen (direct endothelin-1 release)
Copyright Harvard Medical School, 2010. All Rights Reserved.
1692
Use anti-hypertensive drugs pre-dialysis:
Intradialytic Hypertension:
Treatment
Use anti-hypertensive drugs pre-dialysis:
ARB
ACEI
Centrally-acting agents (clonidine patch)
Adjust sodium modeling: 145-140-135
Change to iso-calcemic dialysate
Extend dialysis session to minimize hormonal
surge during UF
Change to SC Epogen
A 77 year-old woman is admitted to the ICU with a
Case 7
A 77 year old woman is admitted to the ICU with a
suspected diagnosis of hemolytic uremic
syndrome and was initiated on HD and
plasmapheresis. During her 2
nd
HD, she becomes
cyanotic and her O
2
saturation drops to 80% 1 hour
into the treatment while receiving oxygen 3 L/min
by nasal cannula by nasal cannula.
Arterial blood gas analysis obtained on 50%
oxygen reveals a PO
2
of 103 mm Hg and an O
2
saturation of 95%. Peripheral blood smear
discloses Heinz bodies.
Copyright Harvard Medical School, 2010. All Rights Reserved.
1693
Which of the following is the most likely diagnosis?
Case 7
Which of the following is the most likely diagnosis?
A. Kinked dialysis tubing-induced hemolysis
B. Complement activation-induced hemolysis
C. Chloramine-induced methemoglobinemia
D. Dialysis-induced exacerbation of hemolytic
uremic syndrome
E. Pulmonary air embolism
Copyright Harvard Medical School, 2010. All Rights Reserved.
1694
Chloramines are inorganic substances normally added to
i i l t t t b t i l t i ti
Chloramine Removal for
Dialysis Water
municipal water to prevent bacterial contamination
Acceptable chloramine levels in municipal tap water < 0.1
mg/L
In dialysis facilities, chloramine removal from water is
mandatory to prevent RBC oxidant damage
Chloramine removal achieved through adsorption by
passing water through granulated activated carbon filter
This step depends on the water contact time with the
carbon filter:
Empty Bed Contact Time (EBCT) = 7.28 x volume of
carbon (ft
3
)/water flow rate (gal/min) = set to a minimum
of 5 minutes per carbon filter
Should be suspected in a cyanotic patient with normal arterial oxygen levels.
When dialysis patients are exposed to chloramine levels of 0 2 0 3 mg/L
Methemoglobinemia in
Dialysis Patients
When dialysis patients are exposed to chloramine levels of 0.2-0.3 mg/L,
methemoglobin levels are detectable, and at levels greater than 0.5 mg/L,
hemolysis occurs
Clinical manifestations include headache, fatigue, dyspnea, and lethargy.
Cyanosis clinically detectable when methemoglobin levels reach 8%-12%,
but may be apparent at lower levels in anemic patients
Respiratory depression, altered consciousness, shock, seizures, and death
occur in severe cases
Diagnosis established by an ABG with co-oximetry g y y
Pulse oximetry unreliable because O
2
saturation drops 0.5% for every 1%
rise of methemoglobin
Low methemoglobin levels generally resolve spontaneously
Greater clinical significance when methemoglobin levels > 10% or patient
with cardiac disease or anemia
Treatment with intravenous 1% methylene blue (starting dose 1 mg/kg)
Copyright Harvard Medical School, 2010. All Rights Reserved.
1695
Installment of 2 carbon adsorption beds in a series configuration
U f l ti t d b ith i di b 900 ( ti l f
Prevention of Chloramine
Contamination
Use of granular activated carbon with iodine number > 900 (optimal for
chlorine/chloramine removal)
Means to sample product water from first and second bed to monitor for
carbon bed exhaustion
EBCT set to a minimum of 5 minutes (per carbon bed) at maximum product
water flow rate
Total EBCT of at least 10 minutes (for 2 carbon beds)
Verification of removal of chloramines to < 0.1 mg/L before each treatment
Other Causes of Intradialytic Hemolysis
Copyright Harvard Medical School, 2010. All Rights Reserved.
1696
A 55 year-old man develops AKI after undergoing
Case 8
y p g g
bone marrow transplantation for aplastic anemia.
Over the ensuing 10 days, he develops progressive
azotemia with a BUN of 110 mg/dl and serum
creatinine of 5.3 mg/dl. Hemodialysis is initiated for
worsening azotemia and deterioration of mental
status.
1 hour after completion of his first 4-hour HD
session, he becomes acutely lethargic and
unresponsive.
Which of the following is the most likely
Case 9
Which of the following is the most likely
cause of these symptoms?
A. Acute hyponatremia
B. Intracranial hemorrhage
C. Acute hypoglycemia
D. Dialysis disequilibrium syndrome
E. Acute aluminum intoxication
Copyright Harvard Medical School, 2010. All Rights Reserved.
1697
Can occur in the setting of AKI
Dialysis Disequilibrium Syndrome
Can occur in the setting of AKI
Usually self-limited, but full recovery may
take several days
Cerebral edema is a consistent finding on
CT-scan
EEG findings are usually non specific
May be prevented by targeting initial URR of
~30%
Over a 1-month period 10 Serratia liquefaciens
Dialysis Outbreak 2
Over a 1 month period, 10 Serratia liquefaciens
bloodstream infections and 6 pyrogenic reactions
without bloodstream infection occur at a free-
standing dialysis facility.
The CDC investigates the outbreak The CDC investigates the outbreak.
Copyright Harvard Medical School, 2010. All Rights Reserved.
1698
Patient Demographics
Investigation of a Dialysis Outbreak
- Review of Medical Records -
Co-morbidities
Medication history
Signs and symptoms of illness
Laboratory tests
Dialysis schedule Dialysis schedule
Dialysis machine
Dialyzer used
Staff caring for patient during incident
W t t t t t d ti
Investigation of a Dialysis Outbreak
- Procedural Review -
Water treatment system and practices:
Disinfection
Distribution
Storage procedures
Disinfection & maintenance of reprocessed Disinfection & maintenance of reprocessed
dialyzers
Disinfection and maintenance of dialysis machines
Review of patient dialysis sessions
Copyright Harvard Medical School, 2010. All Rights Reserved.
1699
Factors Associated with Serratia Bacteremia or
Pyrogenic Reactions
Grohskopf, L. A. et al. N Engl J Med 2001;344:1491-1497
Review of procedures:
Dialysis Outbreak 2:CDC
Investigates
Single-use vials of epoetin alfa punctured multiple times
Residual epoetin alfa from multiple vials pooled and administered to
patients
S. liquefaciens isolated from:
Pooled epoetin alfa
Empty vials of epoetinalfa
Antibacterial soap and hand lotion Antibacterial soap and hand lotion
All isolates identical by pulsed-field gel electrophoresis
Pooling epoetin alfa discontinued and contaminatedsoap/lotion replaced
No recurrence of S. liquefaciens bloodstreaminfections or pyrogenic
reactions
Grohskopf LA: N Engl J Med 2001;344:1491-1497
Copyright Harvard Medical School, 2010. All Rights Reserved.
1700
1) A 43 year-old female complains of
recurrent intradialytic headache.
2) Twenty-four hours following a total hip
l t 68 ld l ith
(A) Meperidine toxicity
(B) Kinked dialysis tubing
For the patients described below, select the most appropriate cause
(D)
replacement, a 68 year-old male with
end-stage renal disease develops
generalized seizures.
3) A 75 year old diabetic male with end-
stage renal disease and a history of
peripheral vascular disease complains
of recurrent abdominal pain during
dialysis.
( ) y g
(C) Digoxin toxicity
(D) Caffeine withdrawal
(A)
(C)
4) At a dialysis facility, 30 minutes after
initiating hemodialysis, a patient
complains of nausea, vomiting and
dyspnea, and is found to be
hypotensive. Similar episodes are
noted among other patients during the
same dialysis shift.
(B)
Faculty Disclosure Information
Member, Scientific Advisory Board, NxStage
Medical, Inc.
Copyright Harvard Medical School, 2010. All Rights Reserved.
1701
Dialysis Dose: Dialysis Dose:
Pearls for the Boards
J . Kevin Tucker, M.D.
Brigham and Womens Hospital
Massachusetts General Hospital
Conflicts of Interest
Consultant Consultant
Baxter Healthcare
Genzyme
Copyright Harvard Medical School, 2010. All Rights Reserved.
1702
Outline
Adequate dialysis defined Adequate dialysis defined
National Cooperative Dialysis Study
Quantitative concepts: URR and Kt/V
The HEMO Study
Kt vs Kt/V Kt vs. Kt/V
What is adequate dialysis?
The minimum amount or dose of
dialysis needed to maintain the patient
free of uremic signs and symptoms and
free of volume overload
General sense of well being
Blood pressure
Nutritional parameters
Copyright Harvard Medical School, 2010. All Rights Reserved.
1703
Urea as toxin
Most abundant of
Easily dialyzed
Most abundant of
organic solutes that
accumulate in renal
failure
Distribution volume is
total body water
MW=60 daltons
Easily dialyzed
Source
Produced by the liver
End-product of protein
nitrogen metabolism
Transport
Passive diffusion in vitro
andin vivo
MW 60 daltons
Polar, water solube
Uncharged
and in vivo
Facilitated diffusion in
vivo
Relatively nontoxic
From Kumar and Depner in Principles and Practice of Dialysis
National Cooperative Dialysis Study
First large-scale study of dialysis dose
Patients were randomized to one of four arms
based upon time on dialysis and time-
averaged urea concentrations:
TAC 100
mg/dL
TAC 100
mg/dL g
2.5-3.5 hr
g
4.5-5.5 hr
TAC 50 mg/dL
2.5-3.5 hr
TAC 50 mg/dL
4.5-5.5 hr
Copyright Harvard Medical School, 2010. All Rights Reserved.
1704
National Cooperative Dialysis Study
Patients in the lowBUN group had Patients in the low BUN group had
fewer hospitalizations than those in the
high BUN group:
More dialysis is better?
Quantitation of dose of dialysis
Urea reduction ratio (URR) Urea reduction ratio (URR)
(Pre BUN- Post BUN)/Pre BUN
Easy to calculate
Mortality predictor
Does not account for ultrafiltration and
convective clearance
Does not account for residual renal
clearance
Copyright Harvard Medical School, 2010. All Rights Reserved.
1705
Adjusted Risk of Death According to the Urea Reduction
Ratio or Serum Albumin Concentration in Patients with End-
Stage Renal Disease
Owen W et al. N Engl J Med 1993;329:1001-1006
Odds Ratios for Death for a Range of Urea
Reduction Ratios and Serum Albumin
Concentrations in 13,473 Patients with End-Stage
Renal Disease Treated with Hemodialysis
Owen W et al. N Engl J Med 1993;329:1001-1006
Copyright Harvard Medical School, 2010. All Rights Reserved.
1706
Concept of Kt/V
First proposedby Gotch and Sargent First proposed by Gotch and Sargent
from a secondary analysis of data from
the NCDS
Probability of dialysis failure was a
constant step function of spKt/V
Higher for Kt/V <0 8 Higher for Kt/V <0.8
Decreased for Kt/V >0.9
Adequate Dialysis Defined by Kt/V in NCDS
Gotch et al, Kidney Int 2000; 58 (Suppl. 76): S3-S18
Copyright Harvard Medical School, 2010. All Rights Reserved.
1707
Components of Kt/V
K =dialyzer urea clearance K = dialyzer urea clearance
Dialyzer properties (KoA)
Qb (blood flow)
Qd (dialysate flow)
t = time on dialysis y
V = urea volume of distribution
Relationship of Kt/V to URR
Kt/V and URR are mathematically related Kt/V and URR are mathematically related
Lowrie et al:
Kt/V =(0.024 x URR) -0.276
Daugirdas et al
Kt/V =-ln (R-0.008 x T) +(4- 3.5 x R) x UF/W
R =Post BUN/Pre BUN
T =Treatment time in hours
UF =Ultrafiltration in liters
W =Post- dialysis weight
Copyright Harvard Medical School, 2010. All Rights Reserved.
1708
Single-pool vs. Double-pool kinetics
The single-pool model assumes that urea is The single-pool model assumes that urea is
contained within a single-compartment.
However, post-dialysis urea rebound
indicates that urea removal follows double
pool kinetics.
Urea levels in the blood compartment may be Urea levels in the blood compartment may be
lower than urea levels in the peripheral
compartments and in the intracellular
environment.
eKt/V
The equilibrated Kt/V is a measure of The equilibrated Kt/V is a measure of
hemodialysis adequacy that uses a
double pool kinetic model.
May be derived from the spKt/V
eKt/V=spKt/V-0.6 x (spKt/V)/hours +0.03 p ( p )
Copyright Harvard Medical School, 2010. All Rights Reserved.
1709
Methods of calculating Kt/V
Formal urea kinetic modeling (UKM) Formal urea kinetic modeling (UKM)
Equation (Daugirdas)
Online clearance
Formal UKM
Height Height
Pre and Post HD BUN
Pre and Post HD Weights
Dialyzer KoA
Qb Qb
Qd
Start and end times
Copyright Harvard Medical School, 2010. All Rights Reserved.
1710
Online clearance
Kt/V determined fromionic dialysance is Kt/V determined from ionic dialysance is
easily integrated into the dialysis machine at
relatively low cost
Based on the assumption that sodium
clearance is equal to urea clearance, K
K can be calculated fromthe conductivity K can be calculated from the conductivity
difference between dialysate inlet and outlet
of the dialyzer after inducing changes in
conductivity
Schematic Depiction of Online Clearance Monitor
Gotch et al, Kidney Int 2004 ; 66 (Suppl 89); S3-S24
Copyright Harvard Medical School, 2010. All Rights Reserved.
1711
Correlation of eKt/V measured by ionic dialysance and URR
McIntyre, C. W. et al. Nephrol. Dial. Transplant. 2003 18:559-562;
doi:10.1093/ndt/18.3.559
eKt/V by four methods
Uhlin, F. et al. Nephrol. Dial. Transplant. 2006 21:2225-2231;
doi:10.1093/ndt/gfl147
Copyright Harvard Medical School, 2010. All Rights Reserved.
1712
Optimizing Kt/V
Vascular access factors Vascular access factors
Inadequate Qb
Recirculation
Dialysate flow
Most efficient at 1.5-2x Qb
Dialyzer size
Inadequate exposure to dialyzer surface
Insufficient anticoagulation
Time
HEMO Study
High Flux High Flux High Flux
High Dose
High Flux
Standard Dose
Low Flux Low Flux
High Dose Standard Dose
Copyright Harvard Medical School, 2010. All Rights Reserved.
1713
Kt/V Goals in HEMO
StandardDose Standard Dose
eKt/V 1.05
High Dose
eKt/V 1.45
Base-Line Characteristics of HEMO Study Participants
Eknoyan G et al. N Engl J Med 2002;347:2010-2019
Copyright Harvard Medical School, 2010. All Rights Reserved.
1714
Mean Characteristics of Treatment during Follow-up of
HEMO Study Participants
Eknoyan G et al. N Engl J Med 2002;347:2010-2019
Survival Curves for the Treatment Groups
Eknoyan G et al. N Engl J Med 2002;347:2010-2019
Copyright Harvard Medical School, 2010. All Rights Reserved.
1715
HEMO Study Conclusions
No evidence that higher dialysis doses No evidence that higher dialysis doses
improved mortality
No evidence that high flux dialysis
improved mortality relative to low flux
dialysis
What about t?
Copyright Harvard Medical School, 2010. All Rights Reserved.
1716
Treatment time and mortality in DOPPS
Saran et al, Kidney Int 2006; 69: 1222-8
Kt/V or Kt?
Some experts in the field of dialysis adequacy Some experts in the field of dialysis adequacy
argue that dimensionless parameters such as
Kt/V and URR are ill-suited for predicting
survival
They treat V simply as a diluent for urea without
other biological or epidemiological properties
Body size (V) is strongly associated with death
risk in dialysis patients risk in dialysis patients
Kt/V divides one measure positively associated
with survival by another
A high Kt/V may indicate a high Kt or a low V
Lowrie et al, Kidney Int 2006; 70: 211-217
Copyright Harvard Medical School, 2010. All Rights Reserved.
1717
Survival by URR Groups
Li et al, Am J Kidney Dis 2000; 35: 598-605
Survival by Kt groups
Li et al, Am J Kidney Dis 2000; 35: 598-605
Copyright Harvard Medical School, 2010. All Rights Reserved.
1718
Summary
Adequate dialysis is the minimumdose Adequate dialysis is the minimum dose
of dialysis needed to maintain the
patient free of uremic signs and
symptoms.
The dose of dialysis may be measured
Formal urea kinetic modeling Formal urea kinetic modeling
Equations for spKt/V and eKt/V
Online clearance
Summary
Factors affecting K: Factors affecting K:
Dialyzer characteristics
Qb
Qd
Exposure to dialyzer surface area
Recirculation
Access problems
Copyright Harvard Medical School, 2010. All Rights Reserved.
1719
Summary
The HEMO Study was the largest study The HEMO Study was the largest study
of dialysis dose performed to date.
No survival advantage for the high dose
group
No survival advantage for high flux dialysis
There is increasing interest in Kt rather
than Kt/V as a predictor of survival.
Question 1
All of the following statements are true regarding All of the following statements are true regarding
hemodialysis adequacy except:
a) Inadequate anticoagulation may adversely affect
urea clearance.
b) The amount of ultrafiltration per treatment does not
affect Kt/V.
) Kt/V i l t t d l f c) spKt/V assumes a single compartment model for
urea.
d) Online urea clearance is an effective method for
measuring dialysis adequacy.
Copyright Harvard Medical School, 2010. All Rights Reserved.
1720
Question 2
A 30-year-old muscular man who weighs 100 A 30 year old muscular man who weighs 100
kg develops ESRD and begins hemodialysis
with a catheter. His catheter is problematic
and often will achieve a blood flow rate of
250-300 mL/min. His prescribed dialysate
flow rate is 800 mL/min, and his prescribed
time is 3 5 hours He is dialyzing with a low time is 3.5 hours. He is dialyzing with a low
flux, high-surface area dialyzer. He feels
better since starting dialysis but his initial sp
Kt/V was 1.0.
Question 2 (contd)
Which of the following statements regarding this Which of the following statements regarding this
patients dialysis adequacy is true?
a) Formal urea kinetic modeling that includes residual
renal function may indicate an adequate Kt/V.
b) His dialysate flow rate should be increased to 1000
mL/min.
) I i hi ti di l i ill i hi c) Increasing his time on dialysis will increase his
Kt/V.
d) Both a and c
e) None of the above.
Copyright Harvard Medical School, 2010. All Rights Reserved.
1721
Conflicts of Interest
Consultant Consultant
Baxter Healthcare
Genzyme
Copyright Harvard Medical School, 2010. All Rights Reserved.
1722
Take-Home Messages
Chronic Kidney Disease and Chronic Kidney Disease and
Dialysis
J . Kevin Tucker, M.D.
Brigham and Womens Hospital
Massachusetts General Hospital
Harvard Vanguard Medical Associates
Conflicts of Interest
Consultant Consultant
Baxter Healthcare
Genzyme
Copyright Harvard Medical School, 2010. All Rights Reserved.
1723
DIALYSIS OVERVIEW AND
DOSE
Survival in ESRD
m
a
i
n
i
n
g
*
35
30
ESRD patients lose 79.2% of years
that the general population is
expected to live.
p
e
c
t
e
d

Y
e
a
r
s

R
e
10
20
30
21.6
13
8
p
USRDS 2005
E
x
p
0
Prostate
Ca
4.5
2.5
*Based on adult, age 59 years
Ca=Cancer
Copyright Harvard Medical School, 2010. All Rights Reserved.
1724
Complications
Hypotension Vascular access Hypotension
Hypertension
Dialyzer reactions
Infection and
pyrogenreactions
Vascular access
bleeding
Seizures
Hemolysis
Arrhythmias pyrogen reactions
Air embolism
Arrhythmias
Muscle cramping
Causes of hypotension in HD
patients
Medications Cardiac causes Medications
Autonomic
dysfunction
Sepsis
Endogenous
dil t
Cardiac causes
Reduced diastolic
filling
Atrial fibrillation
Tachycardia
Left ventricular
hypertrophy
vasodilators
Underestimation of
dry weight
hypertrophy
Ischemia
Pericardial effusion
Copyright Harvard Medical School, 2010. All Rights Reserved.
1725
Interventions that may be Effective
for hypotension
Decrease fluid gains (salt intake) Decrease fluid gains (salt intake)
Increase treatment time
Increase dialysate sodium
Raise dry weight
Lower dialysate temperature
Administer midodrine Administer midodrine
Stop eating during dialysis
Supplement L-carnitine
Hemolysis
Rare with current equipment and
monitoring devices
Manifestations
chest pain/tightness, back pain, dyspnea
hyperkalemia, fall in hematocrit
port-wine appearance of blood in venous
li line
pink appearance of plasma in centrifuged
blood
Copyright Harvard Medical School, 2010. All Rights Reserved.
1726
Dialyzer Reactions
Broad group of events with multiple etiologies
T A h l ti Type A - anaphylactic
occur early in treatment, 1st 15-20 minutes
ethylene oxide was frequent cause in past but less
frequent now with better removal of ETO by
manufacturer
Type B - milder, complement activation?
Occur any time during dialysis, usually 1st hour
back pain, chest pain
What is adequate dialysis?
The minimum amount or dose of
dialysis needed to maintain the patient
free of uremic signs and symptoms and
free of volume overload
General sense of well being
Blood pressure
Nutritional parameters
Copyright Harvard Medical School, 2010. All Rights Reserved.
1727
Quantitation of dose of dialysis
Urea reduction ratio (URR) Urea reduction ratio (URR)
(Pre BUN- Post BUN)/Pre BUN
Easy to calculate
Mortality predictor
Does not account for ultrafiltration and
convective clearance
Does not account for residual renal
clearance
Concept of Kt/V
First proposedby Gotch and Sargent First proposed by Gotch and Sargent
from a secondary analysis of data from
the NCDS
Probability of dialysis failure was a
constant step function of spKt/V
Higher for Kt/V <0 8 Higher for Kt/V <0.8
Decreased for Kt/V >0.9
Copyright Harvard Medical School, 2010. All Rights Reserved.
1728
Relationship of Kt/V to URR
Kt/V and URR are mathematically related Kt/V and URR are mathematically related
Lowrie et al:
Kt/V =(0.024 x URR) -0.276
Daugirdas et al
Kt/V =-ln (R-0.008 x T) +(4- 3.5 x R) x UF/W
R =Post BUN/Pre BUN
T =Treatment time in hours
UF =Ultrafiltration in liters
W =Post- dialysis weight
Optimizing Kt/V
Vascular access factors Vascular access factors
Inadequate Qb
Recirculation
Dialysate flow
Most efficient at 1.5-2x Qb
Dialyzer size
Inadequate exposure to dialyzer surface
Insufficient anticoagulation
Time
Copyright Harvard Medical School, 2010. All Rights Reserved.
1729
Survival Curves for High and Standard
Dialysis Doses in HEMO
Eknoyan, G, Beck, GJ , Cheung, AK, et al. N Engl J Med 2002; 347:2010.
Treatment time and mortality in DOPPS
Saran et al, Kidney Int 2006; 69: 1222-8
Copyright Harvard Medical School, 2010. All Rights Reserved.
1730
VASCULAR
CALCIFICATION
Vascular Calcification and
Cardiac Disease
Hruska et al, Kidney Int 2008
Copyright Harvard Medical School, 2010. All Rights Reserved.
1731
Multivariable-Adjusted Relative Risks of Serum Phosphorus
Referent Range, 4.0-5.0 mg/dl (1.29-1.61 mmol/L)
h
N=40,538
P<0.0001
a
t
i
v
e

R
i
s
k

o
f

D
e
a
t
h
25% 8%
R
e
l
a
Serum Phosphorus Range (mg/dl)
<3 3-4 4-5 5-6 6-7 7-8 8-9 >9
Block GA et al, J Am Soc Nephrol, 2004
mmol/L 1.61-1.94
25% 8%
1.94-2.26
Coronary artery calcification scores
in children and young adults treated
with dialysis
Goodman et al NEJ M 2000; 342: 1478-1483
Copyright Harvard Medical School, 2010. All Rights Reserved.
1732
Arterial Calcification*
Increases Mortality Risk
N=110
1
0 Arteries

Calcified
0
0.25
0.5
0.75
P
r
o
b
a
b
i
l
i
t
y

o
f

S
u
r
v
i
v
a
l1 Artery Calcified
2 Arteries Calcified
3 Arteries Calcified
4 Arteries Calcified
*Determined by ultrasonography.

Carotid artery, abdominal aorta, iliofemoral axis, and legs.


P<0.0001 for each increase in number of arteries calcified.
Blacher J , et al. Hypertension. 2001;38(4):938-942.
Follow-Up (months)
0
0 20 40 60 80
Arterial Stiffness in Dialysis Patients
and Probability of Survival
1.00 1.00
0.75 0.75
0.50 0.50
0.25 0.25
PWV < 9.4 m/s PWV < 9.4 m/s
9.4 PWV 12.0 m/s 9.4 PWV 12.0 m/s
PWV > 12 0 m/s PWV > 12 0 m/s
140 140 00 35 35 70 70 105 105
00
PWV > 12.0 m/s PWV > 12.0 m/s
Blacher J, et al. Circulation. 1999;99:2434-2439
<--Months of
f/up
Copyright Harvard Medical School, 2010. All Rights Reserved.
1733
ANEMIA AND IRON
MANAGEMENT
Iron Transport Mechanisms in the
Duodenal Enterocyte
Ferroportin transport
regulates the amount of
Ferroportin, in turn, is
regulated by hepcidin
Ferroportin
regulates the amount of
iron that leaves the
duodenal enterocytes
and goes into the
circulation
g y p
Increased hepcidin
causes decreased
circulating iron
Andrews NC. Intestinal iron absorption: current concepts
circa 2000. Dig Liver Dis. 2000;32:56-61.
Copyright Harvard Medical School, 2010. All Rights Reserved.
1734
Ferroportin
Ferroportin is the protein transporter on the Ferroportin is the protein transporter on the
surface of every cell which exports iron
Intestinal Cells
Reticuloendothelial cells
Macrophages
Internalized by Hepcidin
Complex is degraded in lysosomes
Effectively reducing iron absorption and
RES iron release
Hepcidin Internalizes Ferroportin, Preventing
Iron Efflux from Cells
Ferroportin on cell p
surface (no hepcidin)
Ferroportin
internalized in
presence of Hepcidin
Elizabeta Nemeth, et al. Science 306, 2090 (2004);
Copyright Harvard Medical School, 2010. All Rights Reserved.
1735
Hepcidin is increased in CKD and Dialysis
Patients
27
Zaritsky et al. Clin J
Am Soc Nephrol 4:
10511056, 2009
Hepcidin is Increased by Inflammation and CKD
Ganz, T et al. Blood. 2008;112:4292-4297
Copyright Harvard Medical School, 2010. All Rights Reserved.
1736
Inflammation and Chronic Kidney Disease
Relative or absolute

---
Hepcidin
Epoetin
Ad i i i Erythropoietin
Decreased Mobilization
of Iron from RE stores
(short term response)
Erythropoiesis
Hepcidin
Expression
Administration
Erythropoiesis
Decreased
Intestinal
Iron absorption
(long term
response)
Anemia/Hypoxia
Resolution of
Anemia
Based on the results of randomized
Use of Iron in CKD Patients Not on
Hemodialysis: Summary
Based on the results of randomized
controlled trials, in Stage 3 or 4 CKD or
PD patients
Use oral iron
Switch to IV iron if the expected Hb
30
response is suboptimal or ESA
dose is high or increasing
Agarwal R. Epoetin and Iron Deficiency. In: Nissenson AR, Fine RN, eds. Clinical
Dialysis. 4th Edition. Elsevier, Burlington, MA. In press.
Copyright Harvard Medical School, 2010. All Rights Reserved.
1737
IV Iron Is More Effective Than Oral Iron
in CKD Patients on Hemodialysis
14
H
b

(
g
/
d
L
)
*
*
*

8
10
12
IV Iron
Oral Iron
No Iron
*P<.05 vs IV iron.

P<.005 vs IV iron.
Adapted with permission from Macdougall et al. Kidney Int. 1996;50:1694-1699.
All 37 patients entered the study iron replete with Hb <8.5 g/dL
Week
6
0 4 8 12 16
Summary
Iron Absorption is regulated by Hepcidin Iron Absorption is regulated by Hepcidin
Low hepcidin promoted iron
absorption/mobilization
High hepcidin inhibits iron absorption/mobilization
Hepcidin is an Acute Phase Reactant
Ferroportin is the crucial transmembrane transport
protein moving iron from cells to circulating transferrin
Internalized by high hepcidin levels
IV iron can bypass hepcidin-mediated iron blockade,
raise Hgb, and lower ESA dose
Copyright Harvard Medical School, 2010. All Rights Reserved.
1738
So What Should We Do?
A lowferritin indicates lowiron stores A low ferritin indicates low iron stores
Give IV iron
A high ferritin relatively uninformative
Iron stores may be high, normal or low iron stores
If TSAT is >50%, this is on the way to overload
Dont give iron
Maintenance iron: to replace last weeks iron losses
Usually 25 62 mg/wk or 1 to 3 grams per year
Loading Dose: to replete stores and test needs
Usually 1 gram of iron
VASCULAR ACCESS
Copyright Harvard Medical School, 2010. All Rights Reserved.
1739
Physical Examination of Vascular
Access
Look Look
AVF Normal: Collapses with elevation
Listen
Normal: Low pitch, continuous
Feel Feel
Normal: Thrill easy to compress
Five criteria to assess
hemodialysis access
Pulsatility Pulsatility
Flow murmur
Thrill
Augmentation
Venous collapse Venous collapse
Copyright Harvard Medical School, 2010. All Rights Reserved.
1740
Number of months from placement to first cannulation of
fistulae as a facility-level practice pattern
Saran, R. et al. Nephrol. Dial. Transplant. 2004 19:2334-2340;
doi:10.1093/ndt/gfh363
Pathogenetic hypothesis of AVF failure
Brunori, G. et al. Nephrol. Dial. Transplant. 2005 20:684-687;
doi:10.1093/ndt/gfh777
Copyright Harvard Medical School, 2010. All Rights Reserved.
1741
PERITONEAL DIALYSIS
Adequacy of Dialysis in PD
The strength of PD lies in
continuous therapy 24/7
preservation of RRF compared to HD
good middle molecule clearance (by RRF
and the peritoneal membrane)
None of these is adequately None of these is adequately
measured by Kt/V urea
Copyright Harvard Medical School, 2010. All Rights Reserved.
1742
Adequacy of Dialysis in PD
randomized, controlled trials have not ,
shown a survival benefit for any Kt/V urea
>1.5
lower limit for Kt/V urea unknown
Adequacy of Dialysis in PD
The KDOQI Guidelines 2006
minimum total (renal +peritoneal) Kt/V
urea of 1.7
monitor and protect RRF
careful attention to volume status
trial of increaseddialysis is indicatedif trial of increased dialysis is indicated if
patient not doing well without another
explanation
Copyright Harvard Medical School, 2010. All Rights Reserved.
1743
Failure of Fluid Volume Management
Volume Overload
Intake
excessive salt and water consumption
Output
loss of residual renal function
use of the wrong dialysis fluid
failure of peritoneal membrane to respond failure of peritoneal membrane to respond
(true ultrafiltration failure)
mechanical problems like leaks
The diagnosis of peritonitis requires at
Peritonitis
The diagnosis of peritonitis requires at
least 2 of the following 3 features:
peritoneal fluid leukocytosis (>100/mm
3
, and at
least 50% polymorphonuclear cells)
the fluid should dwell 2 to 4 hours
abdominal pain
positive culture of the dialysis effluent
Copyright Harvard Medical School, 2010. All Rights Reserved.
1744
start antibiotic treatment quickly
Treatment of Peritonitis
start antibiotic treatment quickly
cover for both gram positive and gram
negative organisms until cultures available
adjust antibiotics according to culture
results
re-evaluate the treatment if no
improvement* in 36-48 hours
* improvement: less abdominal pain, falling
peritoneal fluid WBC count
Urgent:
Peritonitis: Urgent Indications
for Catheter Removal
Urgent:
unresolving or worsening peritonitis after 3 -
5 days
surgical peritonitis (eg bowel perforation)
unresolving peritonitis associated with exit
i l i f i site or tunnel infection
fungal peritonitis
Copyright Harvard Medical School, 2010. All Rights Reserved.
1745
Electi e
Peritonitis: Elective Indications
for Catheter Removal
Elective:
relapsing or recurrent peritonitis with same
organism
continuing culture positivity of PD fluid
ongoing exit site infection, especially if
recent peritonitis with same organism
METABOLIC BONE
DISEASE
Copyright Harvard Medical School, 2010. All Rights Reserved.
1746
Renal Osteodystrophy
Renal osteodystrophyis an alteration of Renal osteodystrophy is an alteration of
bone morphology in patients with CKD
It is one measure of the skeletal
component of the systemic disorder of
CKD-MBD that is quantifiable by
histomorphometry of bone biopsy.
Spectrum of Renal Osteodystrophy
Adynamic Bone
Osteomalacia
Normal
Mild
Osteitis Fibrosa
Osteomalacia
Mixed
Hyperparathyroidism
Low
t
High
t
Calcium, calcitriol
Aluminum
turnover turnover
,
Copyright Harvard Medical School, 2010. All Rights Reserved.
1747
CKD-MBD
A systemic disorder of mineral and bone
metabolism due to CKD manifested by either
one or a combination of the following:
Abnormalities of calcium, phosphorus, PTH,
or vitamin D metabolism
Abnormalities in bone turnover,
mineralization, volume, linear growth, or
strength
Vascular or other soft tissue calcification
KDIGO 2006, Kidney International 69:1945.
Chronic Kidney Disease
Phosphate
Retention
Low Levels
1,25(OH)
2
D
3
FGF-23
, ( )
2 3
Resistance of
Bone to PTH
Hypocalcemia
Hyperparathyroidism
Decreased
Ca
Sensor
Decreased
1,25(OH)
2
D
3
Receptors
Copyright Harvard Medical School, 2010. All Rights Reserved.
1748
Paradigm Shift to Early Monitoring of
PTH, Ca, and P Metabolism in CKD
CKD
Stage
GFR*
Range
Measurement
of PTH
Measurement
of Ca and P
3 30-59 Every 12 months
Every 12
months
4 15-29 Every 3 months Every 3 months
5 <15 or dialysis Every 3 months Every month
*GFR in mL/min/1.73 m
2
.
K/DOQI Guidelines for Bone Metabolism and Disease.
NKF. Am J Kidney Dis. 2003;42(4 suppl 3):1-201.
19-Nor Placebo
Changes in the Levels of Intact PTH in
19-Nor -Treated and Placebo Groups
f
r
o
m

I
n
i
t
i
a
l

P
T
H
-40
-20
0
20
D
o
s
e

g
/
K
g

x
3
/
w
e
e
k
0.4
0.5
0.6
0.7
0.8
0.9
1.0
Weeks
1 2 3 4 5 6 7 8 9 10 11 12
%

C
h
a
n
g
e

f
-80
-60
0.0
0.1
0.2
0.3
0.4
Martin K. et al, AJ KD 2001
Copyright Harvard Medical School, 2010. All Rights Reserved.
1749
800
Effect of treatment with cinacalcet on PTH
200
400
600
P
T
H

(
p
g
/
m
l
)
Cinacalcet
Placebo
P<0.001
Block et al, NEJ M, 2004
0
0 2 4 6 8 10 12 14 16 18 20 22 24 26
Time (Weeks)
Dose Titration Efficacy Assessment
POISONINGS AND
INTOXICATIONS
Copyright Harvard Medical School, 2010. All Rights Reserved.
1750
Hemodialysis for intoxications
and poisonings
Hemodialysis is most beneficial for Hemodialysis is most beneficial for
drugs that have specific characteristics:
Low volume of distribution (<1 L/kg)
Water soluble
Low protein binding
Rapid equilibration between tissue and
blood compartments
Dialysis procedure for
poisoning and intoxication
Choice of dialyzer: High urea clearance; Choice of dialyzer: High urea clearance;
high flux, depending upon molecular weight
of substance to be removed
Bicarbonate concentration may need to be
lowered if there is concurrent metabolic
alkalosis. alkalosis.
Phosphorous may need to be supplemented.
Copyright Harvard Medical School, 2010. All Rights Reserved.
1751
Drugs/Toxins efficiently
removed by hemodialysis
Drug
Molecular
W i ht
Volume of Water g
Weight
(daltons)
distribution soluble
Ethylene
glycol
62 0.6 Yes
Isopropanol
60 0.6 Yes
Lithium
7 0 6-1 0 Yes
t u
7 0.6-1.0 Yes
Methanol
32 0.7 Yes
Salicylate
138 0.2 Yes
Theophylline
180 0.5 Yes
Hemoperfusion
A technique in which blood is passed A technique in which blood is passed
through a cartridge packed with
charcoal or carbon
Charcoal in the cartridge competes with
plasma protein for the drug, adsorbs it,
and eliminates it fromthe circulation and eliminates it from the circulation
More efficient at removing lipid soluble
drugs than is hemodialysis.
Copyright Harvard Medical School, 2010. All Rights Reserved.
1752
Drugs/Toxins efficiently
removed by hemoperfusion
Drug
Molecular
Weight
Volume of
distribution
Water
soluble
(daltons)
distribution soluble
Theophylline
180 0.5 Yes
Carbamazepine
236 1.4 No
Disopyramide
340 0.6 No
Phenobarbital
232 0.5 No
Phenytoin 252 0.6 No
Ethylene Glycol
Glycoaldehyde
Alcohol
dehydrogenase
NAD
NADH
lactate
Pyruvate
Glyoxylic
acid
Adepate Glycine
thiamine pyridoxine
Glycolic acid
Oxalate
ARF, crystalluria (Ca
oxalate)
hypocalcaemia Tissue deposition
Copyright Harvard Medical School, 2010. All Rights Reserved.
1753
Conflicts of Interest
Consultant Consultant
Baxter Healthcare
Genzyme
Copyright Harvard Medical School, 2010. All Rights Reserved.
1754
Nephrology Board Review 5
(Dialysis)
Steven M. Brunelli, MD, MSCE
Director of Dialysis & Associate Physician, Brigham and Womens Hospital
Lecturer, Harvard Medical School
August 13, 2009
Financial disclosures
Consulting fees from C B Fleet Co Amgen Consulting fees from C.B. Fleet Co, Amgen
Spouse employed by Genzyme
Copyright Harvard Medical School, 2010. All Rights Reserved.
1755
Case 1
A 42 year old man with ESRD (ADPKD), who has been on
HD for 2 years, presents to your hospital with extreme y p y p
fatigue and exertional dyspnea. Symptoms began ~2-3
weeks ago, and have grown progressively worse.
ROS: -Chest pain, -Orthopnea, -Fever, -Rash, -Joint pain, -
Melena
FHx: Northern European descent, +ADPKD (Father &
Sister), -Hemoglobinopathy, -Autoimmune disease
Meds: felodipine 10 mg daily, nephrocaps 1 daily
Case 1
Exam:
T 98.9F; BP 152/94; HR 96; RR 16 98 9 ; 5 /9 ; 96; 6
Sclera icteric
Lungs clear
Heart +S4, otherwise normal
No dependent edema
Left radio-cephalic AV fistula + bruit and thrill
Skin jaundiced
Chest X-ray demonstrates mild cardiomegally,
stable from 1 year prior. Lung fields are clear.
Copyright Harvard Medical School, 2010. All Rights Reserved.
1756
Case 1
Labs:
Troponin I < lower limit of detection
Electrolytes within reference range
Hemoglobin 5.4 g/dL
WBC, platelets: within reference range
Total bilirubin 4.1 mg/dL
Direct Coombs test negative
Peripheral blood smear with many schistocytes
R i f d f hi di l i f ilit Review of records from his dialysis facility:
Hemoglobin 10.9 8.7 6.2 g/dL over the prior 6 weeks
Escalating erythropoietin alpha dose (currently 16,000 u IV q HD)
Serum ferritin: 210 ng/mL
Transferrin saturation: 20%.
Case 1
The appropriate next diagnostic step in this case is?
A. Therapeutic trial of intravenous iron.
B. Test for anti-erythropoietin antibodies.
C. Contact the dialysis center to inquire as to whether
other patients have had similar hemoglobin trends.
D. Send blood culture and examine for occult sources of
infection. infection.
E. Measure serial cardiac biomarkers, and nuclear stress
test if negative.
Copyright Harvard Medical School, 2010. All Rights Reserved.
1757
Worsening anemia on HD
Impairments in ESA responsiveness would not explain p p p
the patients hemolysis (A, D, E). More on this later
Pure red blood cell aplasia
Anti-erythropoietin antibodies
Cases attributed to haptenization of erythropoietin by packaging
materials
Subcutaneous administration seems to be required
May have disappeared May have disappeared
Rare, but severe
Although antibody mediated, it is not a hemolytic process (B)
Antibodies against erythropoietin, not RBCs
Copyright Harvard Medical School, 2010. All Rights Reserved.
1758
Hemolysis in hemodialysis patients
Autoimmune
Congenital (thalassemias, Hemoglobin S) g ( g )
Mechanical
Catheter
Needles
Roller pump
Drugs
Calcineurin inhibitors (prior transplant)
Those with strong oxidizing potential (G6PD deficiency)
Dialysate
Overheated
Hypotonic
Water impurities
Typical HD water treatment system
Ca++, Mg++ Chloramine
Particulates, chemical,
bacteria, endotoxin
Softener Carbon filter Reverse osmosis
Deionizer
Source water
Mono-valent ions
UV light
Water
tank
UV light
Bacteria
HD machine
Copyright Harvard Medical School, 2010. All Rights Reserved.
1759
Clinical consequences of dialysis water
impurities
Contaminant Consequence(s)
Al i D i i l d h i f Aluminum Dementia, seizures, myoclonus, dysarthria, fractures
Bacteria/ endotoxin Fever, hypotension, sepsis (bacteria only)
Ca/ Mg/ Lead Nausea/vomiting, muscle weakness, hypertension (Ca),
hypotension (Mg)
Chloramine Hemolytic anemia, methemoglobinemia
Copper Nausea, chills, hemolytic anemia
Fluoride Pruritis, chest pain, cardiac arrest, bone disease
Clinical consequences of dialysis water
impurities
Contaminant Consequence(s)
Hydrogen peroxide Hemolytic anemia
Nitrates Nausea/vomiting, hypotension, methemoglobinemia
Sodium Hypertension
Sulfate Nausea/vomiting, metabolic acidosis
Zinc Hemolytic anemia
Copyright Harvard Medical School, 2010. All Rights Reserved.
1760
Case 2
A 36 year old presents to your center for dialysis initiation.
She has stage V CKD from diabetic (type I) nephropathy, g ( yp ) p p y
and has recently developed anorexia and fatigue
attributed to uremia.
Approximately 10 minutes into the first treatment (using a
highly biocompatible AN-69 membrane), she reports
mid-sternal chest pain and dyspnea.
Recent nuclear stress test (performed in the context of
transplant evaluation) demonstrated no evidence of
myocardial ischemia.
Case 2
Exam:
Moderate respiratory distress.
Labs (1 week prior):
BUN 90
T 99.0 F
BP 90/60 (pre-HD 156/84)
HR 121
RR 20
End-expiratory wheezing
No stridor
Tachycardia, regular rhythm
Skin:
Cr 6.3
K 5.6
Na 140
Hgb 11.0
Medications:
Amlodipine 10 mg daily
Lisinopril 40 mg daily
Skin:
Cool and diaphoretic, with
urticarial eruptions seen on
her exposed L forearm (the
site of her AV fistula).
Lisinopril 40 mg daily
Calcium acetate 667 mg tid
AC
Copyright Harvard Medical School, 2010. All Rights Reserved.
1761
Case 2
The most appropriate next step in this patients The most appropriate next step in this patient s
management is?
A. Discontinue dialysis immediately. Refer for urgent
cardiac evaluation.
B. Continue hemodialysis. Refer for cardiac evaluation
upon completion of the treatment.
C. Continue hemodialysis. Check serum K stat and titrate
dialysate K appropriately.
D. Continue hemodialysis. Avoid further ultrafiltration.
E. Discontinue dialysis immediately. Administer
epinephrine and corticosteroids.
Copyright Harvard Medical School, 2010. All Rights Reserved.
1762
Case 2
Cardiac source of symptoms and signs is possible, and may be
due to a primary coronary event or from a reduction in global due to a primary coronary event, or from a reduction in global
perfusion due to aggressive ultrafiltration. Coronary event is less
likely given results from the patients recent stress test (A, B). This
soon into dialysis, it is unlikely that a hemodynamically meaningful
amount of fluid has been removed (D).
Hyperkalemia, as well as acute reductions in serum potassium,
may precipitate dysrhythmia. There is little reason why
hyperkalemia would suddenly be of issue, and so early into
treatment not much potassium has been removed (C) treatment, not much potassium has been removed (C).
None of the above would explain the patients wheezing or
urticaria.
Type A dialyzer reactions
Rare (4 / 100,000 treatments)
IgE mediated
Signs/symptoms:
Itching
Burning at the access site
g
Membrane
Cellulose > Substituted cellulose>
Cellulosynthetic > Synthetic
Ethylene oxide
Bacterial peptide contamination
Heparin
Formaldehyde
Timing
Usually within first 10 minutes
Always within first 30 minutes
Urticaria
Sneezing
Wheezing
Abdominal cramps
Headache
Back pain
Chest pain
Dyspnea
Eosinophilia suggestive
Always within first 30 minutes
Treatment:
Immediate discontinuation (no rinse
back)
Epinephrine, corticosteroids, histamine
antagonists
Copyright Harvard Medical School, 2010. All Rights Reserved.
1763
Special case of type A dialyzer reaction:
AN-69 membrane + ACEI
Prekallikrein
AN-69 membrane Hageman factor
Prekallikrein
Kallikrein
+
Kininogen Bradykinin
+
Bradykinin degradation by angiotensin converting enzyme inhibited in the
presence
of an ACE-I.
Type B dialyzer reactions
Common (3-5%
treatments with cellulose
Signs/symptoms:
Back pain
membranes)
Complement mediated
Membrane
Cellulose > Substituted
cellulose> Cellulosynthetic
> Synthetic
p
Chest pain
Dyspnea
Nausea/ vomiting
Neutropenia suggestive
No allergic
symptoms
Timing
Often begin 15-30 minutes
following start of treatment
May occur later
Treatment supportive
Membrane-OH
C5a (often pulmonary deposition)
Neutrophil chemotaxis
and activation
Copyright Harvard Medical School, 2010. All Rights Reserved.
1764
Case 3
A 52 year old male presents to the ED with complaints of worsening
shortness of breath and exertional dyspnea over 2 days. He has
end-stage kidney failure due to hypertensive nephrosclerosis end stage kidney failure due to hypertensive nephrosclerosis.
Several days prior, he had 3 or 4 episodes of emesis, as did his wife
and several of their dining guests.
He is managed on cycled PD: 8 exchanges per night, dwell time 1 hour.
In response to his dyspnea, the patient has changed from a 1.5% to
a 2.5% dextrose PD solution. Effluent PD solution has been clear,
and the patient denies fever or abdominal pain.
By report from the ED physician, the patient has a blood pressure of
148/96 and 1+ bipedal edema.
Case 3
As the nephrologist on call, you are asked to provide PD orders to treat
suspected pulmonary edema suspected pulmonary edema.
You prescribe a regimen of 4.25% dextrose solution, and recommend
that the patients regimen remain otherwise unchanged.
On morning rounds, you note the patient is hypotensive to 88/40. He
reports symptomatic orthostasis; his shortness of breath is
unresolved. He ultrafiltered 3.4 L overnight.
On exam, the patient has decreased breath sounds and dullness to
percussion of the right middle and lower lung fields with no crackles
or rales.
Copyright Harvard Medical School, 2010. All Rights Reserved.
1765
Case 3
Testing is likely to reveal which of the following?
Chest X-ray Other
A Alveolar infiltration Sputum cultures positive for
streptococcal pneumoniae
B Right pleural effusion High pleural fluid glucose
C Interstitial edema Elevated pulmonary artery pressures
on echocardiography on echocardiography
D No acute disease Pulmonary embolus on CT
angiography
E No acute disease PD fluid culture positive for mixed
gram positive and negative flora
Copyright Harvard Medical School, 2010. All Rights Reserved.
1766
Case 3
Pneumonia (A) is unlikely given the history and physical
examination findings.
Cardiogenic pulmonary edemaeither primary (B), or
due to peritonitis-related impairment in ultrafiltration
capacity (E) should have improved with aggressive
ultrafiltration.
None of the above would necessarily explain the
physical exam findings (absent crackles/ rales, present
evidence of pleural effusion).
Mechanical complications of PD
Hernia
Abdominal wall leak
Genital edema
Hydrothorax
Back pain
GERD/ delayed gastric emptying
Copyright Harvard Medical School, 2010. All Rights Reserved.
1767
Hydrothorax in PD
Incidence unknown
Defects in hemi-diaphragm
Congenital
Acquired
Increased intra-abdominal pressure (cough, vomiting)
Typically right sided
Heart and pericardium buttress left side
May worsen with hypertonic dialysate
Further increases in intra-abdominal pressure
Hydrothorax in PD
Symptoms range:
Asymptomaticsevere shortness of breath y p
Diagnosis:
High pleural fluid glucose
Radio-nuclear scanning
Treatment:
Discontinue PD
Mechanical repair of diaphragm
Pleurodesis
Resume with low volume PD
Copyright Harvard Medical School, 2010. All Rights Reserved.
1768
Case 4
You see a 63 year old male on routine rounds in an outpatient hemodialysis
unit. He has been on dialysis for 3 years for management of ESRD due to
complications of an abdominal aortic aneurysm repair. p y p
On review of systems, he feels well and denies any specific complaints. Review
of the record reveals no recent fever; pre-dialysis blood pressure has
ranged ~ 140/90.
Routine laboratory tests, done 1 month prior demonstrate:
Hemoglobin 9.6 g/dL (10.5 g/dL 1 month prior, between 11-12 g/dL in the six
months before that)
Ferritin 275 ng/ml
Transferrin Saturation 17%
Medications include:
Erythropoietin 6,600 U IV q HD
Iron sucrose 100 mg IV weekly
Case 4
The most appropriate next step in this patients The most appropriate next step in this patient s
management is?
A. Iron sucrose 100 mg IV q HD x 10 treatments
B. Increase erythropoietin to 8,800 U IV q HD
C. Blood cultures and empiric therapy for gram positive
cocci bacteremia
D. Tagged white blood cell scan to evaluate for infection D. Tagged white blood cell scan to evaluate for infection
of senescent AV grafts
E. Referral for endoscopic evaluation of the GI mucosa
Copyright Harvard Medical School, 2010. All Rights Reserved.
1769
Case 4
Target hemoglobin in HD: 11-12 g/dL Target hemoglobin in HD: 11 12 g/dL.
Suboptimal hemoglobin levels may reflect:
Under-dosing of erythropoiesis stimulating agent
(ESA)
ESA hypo-responsiveness / resistance
The patients prior hemoglobin levels suggest
that his ESA was not under-dosed (B), pointing
to decreased responsiveness.
Copyright Harvard Medical School, 2010. All Rights Reserved.
1770
Etiology of poor ESA responsiveness in HD
Hypo-responsiveness:
Patient requires erythropoietin 150-300 U/kg IV tiw (or equivalent) to
achieve target hemoglobin achieve target hemoglobin
Resistance
Failure to achieve hemoglobin target despite doses in this range
Etiologies
Absolute or functional iron deficiency
Inflammation/ infection
Nutritional deficiencies
Secondary hyperparathyroidism Secondary hyperparathyroidism
Marrow infiltration
Of these iron deficiency is by far the most common.
Optimization of iron stores is the most cost-effective means of
correcting anemia among HD patients
Iron management in HD
Target iron storage parameters
Serum ferritin > 200 ng/ml
Transferrin saturation >20% Transferrin saturation >20%
Absolute iron deficiency: ferritin and Tsat below target
Blood loss via HD circuit
Occult blood loss
Severe and prolonged nutritional deficiency
Relative iron deficiency: Tsat below target, ferritin > 200 ng/ml
Blood loss (early) Blood loss (early)
Infection/ inflammation
Clinically manifest (bacteremia, MI)
Occult (senescent AV graft infection, peridontitis)
Copyright Harvard Medical School, 2010. All Rights Reserved.
1771
Iron management in HD
Routine maintenance iron
25-100 mg weekly g y
Replace losses via the dialytic circuit
Overcome the influence of chronic, low-grade inflammation.
Many patients still become iron deficient despite
Iron repletion
100 mg IV q HD to a total of 1 g
Recommended for all patients with absolute iron deficiency Recommended for all patients with absolute iron deficiency
(ferritin < 200 ng/ml)
Also recommended as diagnostic and therapeutic maneuver in
patients with relative iron deficiency (Tsat<20%) provided
ferritin not >500 ng/ml (A)
Iron management in HD
Overt bacteremia unlikely given the absence of
suggestive symptoms (C). gg y p ( )
Evaluation for occult sources of inflammation/ infection
should be reserved for patients failing to correct with
empiric iron repletion (D).
Endoscopic evaluation of the GI mucosa (E) would be
appropriate if frank hemorrhage was suspected, or if
iron storage indices fell below the lower limit of normal.
Note: target ranges for ferritin and transferrin
saturation in HD are supra-normal.
Copyright Harvard Medical School, 2010. All Rights Reserved.
1772
Case 5
A 55 year old man is managed on continuous ambulatory
peritoneal dialysis for ESRD due to hypertensive
h l i nephrosclerosis.
On routine follow up, he feels well and denies specific
complaints.
On physical exam, BP is 150/93. Chest is clear to
auscultation. Heart is regular with no extra sounds
appreciated. He has 1+ bilateral lower extremity edema.
Weight is 70 kg; BSA 1.73 m
2
Net ultrafiltration has been 1 L/day. Urine output ~300-400
ml/day.
Case 5
The patient performs 4 manual exchanges (2000 cc) at four-hour
intervals during the day, and has an additional long 8 hour dwell
overnight overnight.
He has been using 2.5% dextrose containing dialysis solution.
Results of a peritoneal equilibration test are as follows:
Dialysate/plasma creatinine ratio 0.75
Dialysate/plasma urea nitrogen ratio 0.9
Dialysate/plasma glucose ratio 0.35
Drain volume 2100 ml Drain volume 2100 ml
24 hour urine:
Urea nitrogen clearance 2 ml/min
Creatinine clearance 4 ml/min
Copyright Harvard Medical School, 2010. All Rights Reserved.
1773
Case 5
The most appropriate next step in this patients The most appropriate next step in this patient s
management is?
A. Increase the number of exchanges to a total of seven.
B. Substitute 7.5% icodextrin in the patients long dwell.
C. Increase exchange volumes to 2500 ml.
D. Convert from 2.5% to 4.25% dextrose PD solution.
E Transition to hemodialysis E. Transition to hemodialysis.
Copyright Harvard Medical School, 2010. All Rights Reserved.
1774
PD adequacy
Current guidelines recommend a combined
weekly Kt/V of 1 7 weekly Kt/V of 1.7.
Urine volume > 100 ml/day PD + renal
Urine volume < 100 ml/day PD only
Changes from past recommendations:
Lower target
Results from several clinical trials (Hong Kong, ADEMEX)
Frequent modality failure with higher target
No consideration of creatinine clearance
Not stratified by transport status.
Calculating weekly Kt/V
Dialysis removal:
Daily drain volume
V
Dialysate urea nitrogen
Plasma urea nitrogen
1.73 m
2
BSA
X X
Drain volume = (2 L /exchange /d x 5 exchanges) + 1 L UF = 11 L/d = 77 L/ week
V 0 6 X b d i ht (k ) 0 6 X 70 42 L (U 0 55 f )
Kt/V urea =
V = 0.6 X body weight (kg) = 0.6 X 70 = 42 L (Use 0.55 for women)
Kt/V urea = (77/42) X 0.9 X (1.73/1.73) = 1.65
Copyright Harvard Medical School, 2010. All Rights Reserved.
1775
Calculating weekly Kt/V
Renal removal (mean creatinine + urea):
Clearance X L x 1440 min x 7 days x 1.73 m
2
=
1000 ml day week BSA
3ml/min X L x 1440 min x 7 days x 1.73 m
2
= 30 L
1000 ml day week 1.73 m
2
Renal Kt/V = Clearance = 30 = 0.71
V 42
Total weekly Kt/V = 1.65 + 0.71 = 2.36
Case 5
Si Kt/V i t t d d d ti t d Since Kt/V is up to standard, and patient does
not report uremic symptoms, there is no need to
increase dialysis intensity:
Increasing number of exchanges may decrease
patient satisfaction (A)
Increasing fill volume may further compromise UF (C) Increasing fill volume may further compromise UF (C)
by increasing intra-abdominal hydrostatic pressure.
Copyright Harvard Medical School, 2010. All Rights Reserved.
1776
Peritoneal transport characteristics
Peritoneal equilibration test:
Instill 2000 ml warmed 2.5% dextrose PD fluid
After 4 hour dwell, measure:
Serum and dialysate urea nitrogen
Serum and dialysate creatinine
Effluent PD fluid glucose concentration
Drain volume
D/P Cr D/P Urea Effluent
glucose
Drain volume
(ml)
High >0.82 >0.95 <501 <35
High average 0.65-0.81 0.85-0.94 502-722 36-320
Low average 0.50-0.64 0.77-0.84 723-944 321-600
Low <0.49 <0.76 >945 >601
Effects of membrane transport characteristics
More rapid transporters will more quickly dissipate p p q y p
dextrose gradients.
Can lead to difficulties with UF
Outweighs any benefit of higher clearance
Epidemiological studies suggest that low and low-
average transporters have greater modality survival and
overall survival.
Our patient is a high-average transporter, and shows
signs of ultrafiltration difficulties (hypertension, edema).
Copyright Harvard Medical School, 2010. All Rights Reserved.
1777
Choices to increase ultrafiltration in PD
Increase number of exchanges with commensurate
reduction in dwell time.
Would likely require change to automated PD in this case
May decrease patient satisfaction
Reduce exchange volume
Modest effects
Decreases clearance
Increase dextrose tonicity Increase dextrose tonicity
Chronic use of 4.25% dextrose-containing PD solutions may
lead to peritoneal membrane scarring and promote modality
failure (D). Might be the most appropriate option if patient could
have transitioned from 1.5 to 2.5%.
Icodextrin (7.5%)
Glucose polymer Glucose polymer
Lower osmolality than 1.5% dextrose
Well tolerated
Less membrane sclerosis
Less gradient dissipation
Greater ultrafiltration than 4.25% dextrose when given
over 8-12 hour dwell
Copyright Harvard Medical School, 2010. All Rights Reserved.
1778
Case 6
A 52 year old male with stage V CKD (not yet on
RRT) is admitted to the hospital for chest pain RRT) is admitted to the hospital for chest pain.
He undergoes a cardiac catheterization w/ PTCA
and stent placement. Afterwards, he becomes
anuric.
By hospital day 6, he still makes minimal urine. On
h i l h i i d t t physical exam he is in moderate-to-severe
respiratory distress; oxygen saturation is 92% on
55% face mask; rales are appreciated 2/3 up
both lung fields.
Case 6
On hospital day 6, the
patient is dialyzed as
Serum biochemistries
patient is dialyzed as
follows:
4 hours via TDC
Qb 400
Qd 600
2 K/ 2.5 Ca/ 40 HCO3
No heparin
Day Cr BUN Na K HCO3
0 5.9 115 137 4.5 19
No heparin
UF 5 L 3 6.7 137 135 4.9 19
6 7.0 159 132 4.9 17
Copyright Harvard Medical School, 2010. All Rights Reserved.
1779
Case 6
Aft th t t t th ti t i t t t After the treatment, the patients respiratory status
is much improved.
However, he complains of headache and blurred
vision. On exam, he is agitated and listless, but
h f l l i l d fi it has no focal neurological deficits.
Case 6
Each of the following strategies may have reduced Each of the following strategies may have reduced
the likelihood of the patients symptoms
except?
A. Shortened treatment time.
B. Reduced blood flow rate.
C. Administration of 12.5g of hypertonic mannitol C. Administration of 12.5g of hypertonic mannitol
each hour of dialysis
D. Reduced ultrafiltration goal
E. Use of a small surface area dialyzer.
Copyright Harvard Medical School, 2010. All Rights Reserved.
1780
Dialysis disequilibrium syndrome
Neurological syndrome seen in patients initiating
hemodialysis. y
No reports in CAPD
Occurrence usually in first few treatments
Risk factors:
High pre-dialysis BUN
Severe metabolic acidosis
Older age
Pediatric patients
Pre-existing neurological disease
Copyright Harvard Medical School, 2010. All Rights Reserved.
1781
Pathogenesis of dialysis disequilibrium
syndrome
CNS Blood CNS Blood CNS Blood CNS Blood
Rapid removal
of blood osmoles
by dialysis
Osmole (e.g., urea)
Relative hyper-tonicity of CNS;
free water shift; cerebral edema
H
2
O
Clinical manifestations of dialysis
disequilibrium syndrome
Headache Asterixis Headache
Nausea
Disorientation
Restlessness
Blurred vision
M l
Asterixis
Confusion
Seizures
Coma
Death
Muscle cramps
Anorexia
Dizziness
Copyright Harvard Medical School, 2010. All Rights Reserved.
1782
Prophylaxis for and treatment of dialysis
disequilibrium syndrome
Prophylaxis
Reduce solute removal
Treatment
Mild symptoms Reduce solute removal
Decreased blood flow rate
(B)
Decreased treatment time (A)
Small surface dialyzer (E)
Concurrent (vs
countercurrent) dialysate flow
Mild symptoms
Slow blood flow rate
Symptomatic management
Severe symptoms
Discontinue treatment
M it l 12 5
Sequential IUFHD if fluid
removal goals require
Hypertonic mannitol 12.5g q
1 hr (C)
Mannitol 12.5 g
Hypertonic saline (5 ml
23%)
Supportive care
Case 7
A 70 year old woman presents for maintenance hemodialysis
complaining of malaise and chills over the preceding 24 hours.
Detailed review of systems is otherwise negative Detailed review of systems is otherwise negative.
She receives hemodialysis thrice weekly via a tunneled central
catheter. On examination, her temperature is 100.0 F; blood
pressure 148/88. She appears well and has no localizing signs of
infection. Her dialysis tunnel site is neither warm nor tender; there is
no expressible pus.
Blood cultures are drawn at the start of dialysis, and she receives a
dose of vancomycin and gentamycin at the end of treatment.
She returns for dialysis 48 hours later; she is afebrile and looks and
feels well. Cultures drawn at her prior treatment are positive for
coagulase negative staphylococcus (3/4 bottles).
Copyright Harvard Medical School, 2010. All Rights Reserved.
1783
Case 7
According to KDOQI guidelines, which is the most
appropriate management? pp p g
A. Prolonged (>3week) administration of broad spectrum
parenteral antibiotics.
B. Prolonged administration of vancomycin and referral
for prompt guidewire catheter exchange.
C. Prolonged administration of vancomycin and referral
for guidewire catheter exchange once blood cultures
h l d have cleared.
D. Prompt admission to the hospital for tunneled catheter
removal and continued parenteral antibiotics.
Copyright Harvard Medical School, 2010. All Rights Reserved.
1784
KDOQI guidelines for management of
catheter related bactermia (CRB)
All CBRs, except exit-site infections, should be treated with
parenteral antibiotics directed at suspected organisms parenteral antibiotics directed at suspected organisms
Definitive antibiotic therapy should be based on the isolated
organism(s) (A)
Guidewire exchange is the preferred strategy in most cases (more to
follow)
Follow up cultures are needed 1 week after cessation of antibiotic Follow-up cultures are needed 1 week after cessation of antibiotic
therapy
Hospitalization is needed only for clinical sepsis (D)
KDOQI guidelines for management of
catheter related bactermia (CRB)
Options for catheter management:
Catheter salvage in situ with parenteral antibiotics (not preferred) Catheter salvage in situ with parenteral antibiotics (not preferred)
Frequent relapses
Not cost effective
Guidewire exchange
Most cost effective
80-88% relapse free rate
Preferred strategy in many cases
Exchange need not wait until cultures have cleared (C)
Removal and subsequent replacement
Recommended if
Patient clinically unstable
Concomitant tunnel infection
Similar efficacy to and less cost effective than guidewire exchange
Copyright Harvard Medical School, 2010. All Rights Reserved.
1785
KDOQI guidelines for management of
catheter related bactermia (CRB)
C th t l i it ith bi d Catheter salvage in situ with combined
parenteral antibiotics + antibiotic locks promising
Head-to-head studies with guidewire exchange
needed
Insufficient evidence to support prophylactic Insufficient evidence to support prophylactic
strategies other than good catheter care.
Case 8
The patient is a 55 year old man with ESRD (IgA nephropathy) who has
been maintained on chronic hemodialysis for 12 years. He dialyzes
via a left forearm AV fistula via a left forearm AV fistula.
The patient has complained of increasingly worse left shoulder pain
over the past year, and now notes decreased ability to move the
arm.
Detailed review of systems reveals he also has pain and paresthesias
in his right thumb and index finger.
On examination, the patient has decreased active and passive range of
motion in his left shoulder. There is also wasting of his right thenar
eminence and a positive Hoffman-Tinel sign.
Copyright Harvard Medical School, 2010. All Rights Reserved.
1786
Case 8
Review of his chart demonstrates that his serum PTH level
has been meticulously maintained in the target range
i di l i i iti ti C t l l 285 / l ( since dialysis initiation. Current level 285 pg/ml (assay
upper limit of normal 65 pg/ml) on paricalcitol 2mcg IV q
HD.
Radiographs of the hand and shoulder reveal no evidence
of fracture. One cystic lesion with sclerotic margins of the
proximal humerus is noted incidentally. There was no
subperiosteal reabsorption seen subperiosteal reabsorption seen.
Non-contrast MR of the shoulder reveals thickening of the
rotator cuff, without tear.
Case 8
Which of the following therapies would likely
t b h l f l i thi ti t not be helpful in this patients
management?
A. Ibuprofen 600 mg as needed for pain
B. Carpal tunnel release surgery
C Transition to high flux dialyzer C. Transition to high flux dialyzer
D. Renal transplantation
E. Increase paricalcitol to 4 mcg IV q HD
Copyright Harvard Medical School, 2010. All Rights Reserved.
1787
Case 8
Ab t b i t l b ti d Absent subperiosteal reabsorption and
supra-recommended PTH, osteitis
fibrosa cystica is unlikely (E).
Renal osteodystrophy (either osteitis
fibrosa cystica or low turn over bone
disease) would not explain patients
carpal tunnel syndrome.
Copyright Harvard Medical School, 2010. All Rights Reserved.
1788
Renal osteodystrophy (for completeness
sake)
PTH should be maintained at 3-5 times the upper limit
of normal
Range supra-physiological due to competing effects of PTH
and PTH-degradation products
Higher levels predispose to osteitis fibrosa cystica
Subperiosteal reabsorption
Bone cysts
Pathological fractures
Lower levels predispose to low turnover bone disease
Pathological fractures
E t i l ifi ti Ectopic calcification
Formal diagnosis by bone biopsy, but serum
biomarkers used nearly universally
Pathogenesis of dialysis related amyloidosis
2-microglobulin:
usually filtered at the glomerulus and reabsorbed and degraded in
proximal tubule proximal tubule.
dialytic removal limited and cannot keep pace with production
blooddialyzer contact may promote synthesis, but this effect is
probably negligible
serum levels invariably rise upon dialysis initiation
Dialysis related amyloidosis results from tissue deposition (perhaps
facilitated by glycosylation)
preferential deposition in bones, joints and synovium
macrophage chemotaxisinability to phagocytose cytokine
production
also direct stimulation of osteocytes
Incidence increases with time on dialysis
Copyright Harvard Medical School, 2010. All Rights Reserved.
1789
Clinical sequelae of dialysis related amyloidosis
Musculoskeletal deposition
Extra-skeletal deposition
p
Carpal tunnel
Scapulohumeral periarthritis
Bone cysts
Spondyloarthropathy
Pathologic fractures
Subcutaneous tissues
Skin
Rectal mucosa
Liver
Spleen
Blood vessels
Effusive arthropathy
Trigger finger/ tenosynovitis
Visceral involvement rarely
causes clinical manifestations.
Diagnosis of dialysis related amyloidosis
Tissue histology***
Congo red staining
Apple-green birefringence under
X-ray
Radiolucent lesions
Bone cysts with thin sclerotic
Apple green birefringence under
polarized light
2-microglobulin, not
immunoglobulin light chains or
serum amyloid A
Abdominal fat pad biopsy not
useful
2-microglobulin levels
Dialyzed patients 30 to 50 mg/L
Bone cysts with thin sclerotic
margins (enlarge over time)
CT
Useful for detection of small
lesions, those in the axial
skeleton and soft tissue
deposition
Ultrasound
Dialyzed patients 30 to 50 mg/L
Normal value of 0.8 to 3.0 mg/L
Non-specific: does not establish
the diagnosis
Ultrasound
Thickened rotator cuff or
biceps synovial sheath
Copyright Harvard Medical School, 2010. All Rights Reserved.
1790
Treatment of dialysis related amyloidosis
Symptomatic y p
Analgesics (A)
Carpal tunnel release (B)
Removal of shoulder synovium
Excision of bone cysts + bone grafting
Joint replacement
Disease modifying
R l t l t (D) Renal transplant (D)
Extension of dialysis session length
Increased dialysis frequency
High flux dialyzer (C)
Case 9
A 43 year old woman with ESRD secondary to lupus
nephritis presents to the ED complaining of abdominal p p p g
pain which was present upon awakening and grew
worse throughout the day. She has been maintained on
CAPD for 18 months.
On examination: T 101.1 F; BP 130/80. Abdomen is
diffusely tender with voluntary and involuntary guarding.
At your instruction, the ED physician drains her PD fluid
(last exchange 4 hours prior). She sends the cloudy fluid
for cell count, gram stain and culture.
Copyright Harvard Medical School, 2010. All Rights Reserved.
1791
Case 9
P it l fl id h 600 b / i L (80% Peritoneal fluid has 600 wbc/microL (80%
neutrophil predominance).
Gram stain is negative. Culture is pending.
Empiric therapy with intra-peritoneal cephazolin
and ceztazidime is begun.
Case 9
Which of the following would not be an indication for PD
catheter removal?
A. Initial peritoneal fluid cultures grow out CNS; patient
had a prior CNS peritonitis one month ago.
B. Positive peritoneal fluid cultures taken on day 5 of
therapy
C. Initial peritoneal fluid cultures grow out S. aureus
D. CT scan positive for perforated appendicitis
E. Initial peritoneal fluid cultures grow out candida
albicans
Copyright Harvard Medical School, 2010. All Rights Reserved.
1792
Peritonitis in PD
Pathogenic organisms
Gram positive 50% G a pos t e 50%
Gram negative 15%
Culture negative 20%
Polymicrobial <10%
Fungal <5%
Leading cause of modality change in the first Leading cause of modality change in the first
year
Death rate ~3-5% of cases
Copyright Harvard Medical School, 2010. All Rights Reserved.
1793
Diagnosis of PD peritonitis
Diagnosis (2 of 3):
Signs/symptoms g y p
Abdominal pain
Nausea/vomiting
Fever/ chills
Change in bowel habits
Abdominal tenderness/ rebound
Peripheral leukocytosis
Peritoneal fluid inflammation
Cl d ffl t Cloudy effluent
>100 leukocytes per microL with >50% neutrophils
Evidence of microorganism
Gram stain
Culture
Therapy for PD peritonitis
Empiric therapy
Coverage for gram positive and gram negatives unless fungi are identified on gram stain
Definitive therapy
Antibiotic choice and treatment duration depend on isolated organism
Adjuvant intra-peritoneal heparin may be useful
Duration typically 14 days in uncomplicated infection
Cell count should be monitored
Surveillance cultures should be obtained
Antibiotics best given continuously or in long dwell
Prophylaxis
Y-system
Topical antibiotic cream
Eradication of S aureus nasal carriage
Copyright Harvard Medical School, 2010. All Rights Reserved.
1794
Indications for PD catheter removal
Relapsing peritonitis (A)
Another episode with the same pathogen within four weeks of antibiotic
cessation cessation
Refractory peritonitis (B)
Failure to respond to appropriate antibiotics within five days
Tunnel infections
Fungal peritonitis (E)
Fecal peritonitis
Peritonitis + other intra-abdominal pathology (D)
Case 10
A 50 year old woman initiated maintenance HD for
treatment of ESRD due to ADPKD 1 week prior. She has p
tolerated dialysis well, and has not had any episodes of
cramping or hypotension. She reports good adherence
to her low potassium low phosphorous diet, which she
began 1 year ago (while at stage IV CKD).
You see her today on routine rounds.
On exam: BP 152/94 (pre-dialysis), HR 90. Well appearing.
Lungs clear. 2+ bilateral lower extremity edema.
Copyright Harvard Medical School, 2010. All Rights Reserved.
1795
Case 10
Labs:
Hgb 11 9 g/dL Hgb 11.9 g/dL
Transferrin saturation 22%
Ferritin 250 ng/ml
PTH 350 pg/ml
Ca 9.9 mg/dL
Phos 6 5 mg/dL Phos 6.5 mg/dL
Albumin 3.2 g/dL
LDL 110 mg/dL
HDL 55 mg/dL
Case 10
Which of the following is the most appropriate next step in
this patients management? p g
A. Addition of valsartan 80 mg daily for blood pressure
control
B. Addition of calcium acetate 667 mg tid AC for
phosphate control
C. Addition of atorvastatin 40 mg daily for lipid control
D. Titration of dialysis intensity to target single pool Kt/V
of 1.4
E. Addition of erythropoietin alpha 2000 U IV q HD
Copyright Harvard Medical School, 2010. All Rights Reserved.
1796
Blood pressure management in HD
Target blood pressure
140/90 pre dialysis 140/90 pre-dialysis
130/80 post-dialysis
The data actually suggest an inverse J-shaped relationship
between BP and mortality with nadir ~160 mmHg
Mortality
Probable confounding
due to cardiomyopathy
and/ or malnutrition
160
BP (mmHg)
Copyright Harvard Medical School, 2010. All Rights Reserved.
1797
Blood pressure management in HD
Initial management should be through attainment of dry
weight (A) g ( )
If BP still above goal then pharmacotherapy
First line (+ compelling indication)
Use corresponding agent.
First line (- compelling indication)
Stage I: ARB
Stage II: ARB +CCB Stage II: ARB +CCB
Second line: -blocker or clonidine
Third line: evaluation for secondary cause (and appropriate
treatment
Fourth line: minoxidil
Blood pressure management in HD
Compelling indications
-blocker:
Angina
Post MI
Diastolic dysfunction
Cardiomyopathy
ACEi/ARB:
Diabetes
Cardiomyopathy
C l i h l bl k
Blood pressure management
recommendations
are based largely on expert opinion.
Likely, they
will change as more data become
available
Calcium channel blockers
Angina
Diastolic dysfunction (centrally acting)
Cyclosporine-induced hypertension
ESA-induced hypertension
Copyright Harvard Medical School, 2010. All Rights Reserved.
1798
Hyperphosphatemia management in HD
Target serum phosphate: 3.5-5.5 mg/dL
The data suggest a U shaped association between The data suggest a U-shaped association between
phosphate levels and mortality with nadir at 4.5 mg/dL
Mortality
Probable confounding
due to malnutrition
4.5
Serum phosphate (mg/dL)
Hyperphosphatemia management in HD
Initial management should be through dietary restriction. g g y
If phosphate levels remain above target, phosphate
binders should be started.
For patients with corrected serum calcium > 10.2 mg/dL,
non-calcium based phosphate binders are
recommended. (B) recommended. (B)
Sevelamer
Lanthanum carbonate
Hyperphosphatemia management recommendations
are based largely on expert opinion. Likely, they
will change as more data become available
Corrected Ca = Serum Ca + 0.8 X (4.4-Serum albumin)
Copyright Harvard Medical School, 2010. All Rights Reserved.
1799
Lipid management in HD
The incidence of high LDL cholesterol is surprisingly low g p g y
in HD
Likely relates to poor nutritional status
Extrapolating from other groups at high risk for CV
disease, one would expect to find a benefit of statin
therapy
However, two large randomized trials have failed to
show any benefit of statin therapy on CV morbidity and
mortality in hemodialysis patients (C).
Anemia management in HD
Target hemoglobin: 11-12 g/dL
Lower levels associated with decreased quality of life Lower levels associated with decreased quality of life
and perhaps mortality
Higher levels associated with increased mortality
Although no specific threshold for ESA initiation
is given in the most recent KDOQI guidelines is given in the most recent KDOQI guidelines,
patients with spontaneous hemoglobin levels in
the high-target range, probably should not be
started (E).
Copyright Harvard Medical School, 2010. All Rights Reserved.
1800
HD adequacy
Single pool Kt/V is among the preferred metrics for measuring
dialysis dose
Evidence suggests a mortality benefit of single pool Kt/V up to ~ 1.2.
Further increases not associated with incremental mortality benefit
Delivery of Kt/V inconsistent
Access recirculation
Decreased UF needs
Premature termination
Guidelines therefore recommend targeting single pool Kt/V > 1.4 to
ensure that at least 1.2 is delivered (D)
Use of Kt/V as a metric of dialysis dose has come
under question. Its use in this regard will be
determined by future research
Case 11
A 40 year old woman with hereditary amyloidosis and related stage V
CKD comes to you for a second opinion CKD comes to you for a second opinion.
She also has near-debilitating orthostatic hypotension due to autonomic
neuropathy. Her past medical history is otherwise negative.
She anticipates receiving a living donor kidney allograft from her
husband when he returns home from military service in 11 months.
In the meantime she has developed symptomatic uremia and has been In the meantime, she has developed symptomatic uremia and has been
advised to initiate renal replacement therapy, and seeks advice as to
which modality to select.
Copyright Harvard Medical School, 2010. All Rights Reserved.
1801
Case 11
Which of the following is not an advantage of
peritoneal dialysis over hemodialysis in this peritoneal dialysis over hemodialysis in this
case?
A. Survival benefit over this patients anticipated
time horizon to transplant
B. Greater preservation of residual kidney
function
C. Decreased incidence of allograft loss upon
eventual transplantation
D. Greater hemodynamic tolerability
Copyright Harvard Medical School, 2010. All Rights Reserved.
1802
Relative benefits of PD versus HD
Survival
Benefit appears limited to the first 1-2 years (A) e e t appea s ted to t e st yea s ( )
Afterwards, benefit is lost, and PD may be associated
with greater mortality
Benefit particularly among younger patients (<55
years) and non-diabetics
Survival (even early) may be better on HD among older
patients and diabetics
Hemodynamic tolerability
More gradual (i.e., physiologic) fluid removal on PD
leads to greater hemodynamic tolerability among
hypotension-prone patients (D)
Relative benefits of PD versus HD
Preservation of residual kidney function
Multiple studies demonstrate increased time
to anuria on PD versus HD.
Likely relates to lower risk of renal ischemia
stemming from episodes of hypotension
Copyright Harvard Medical School, 2010. All Rights Reserved.
1803
Allograft loss following PD versus HD
M (b t t ll) t di l t i k f Many (but not all) studies reveal a greater risk of
death censored allograft loss among patients
treated with PD prior to transplant.
May relate to higher incidence of allograft thrombosis
Loss of anticoagulant factors on PD?
Activation of pro-coagulant factors? p g
Incidence of DGF paradoxically lower following PD
Financial disclosures
Consulting fees from C B Fleet Co Amgen Consulting fees from C.B. Fleet Co, Amgen
Spouse employed by Genzyme
Copyright Harvard Medical School, 2010. All Rights Reserved.
1804
General Nephrology Review
Q & A
David Charytan, MD, MSc
Associate Staff Physician, Renal Division
Brigham & Womens Hospital
Assistant Professor of Medicine
Harvard Medical School Harvard Medical School
Disclosures
D Ch t i f di f th A i Dr Charytan receives funding from the American
Society of Nephrology, Paul Teschan Research
Fund (DCI), and the American Heart Association.
Copyright Harvard Medical School, 2010. All Rights Reserved.
1805
Question 1
A 82-year-old female with type 2 diabetes mellitus,, hypertension,
hyperlipidemia, and CKD (serum creatinine, 2.3 mg/dL) has exertional
chest pain consistent with myocardial ischemia. A stress test is positive.
Prior to cardiac catheterization, she is given intravenous fluids and n-
acetylcysteine to prevent contrast nephropathy. The left anterior y y p p p y
descending artery is stented, and she is started on clopidogrel.
Approximately 4 days later, she develops oliguria, azotemia (creatinine
2.8 mg/dL), abdominal pain and has evidence of GI bleeding. Which of
the following is the most likely cause of AKI?
A) Cholesterol embolization to the renal arteries
B) Pre-renal azotemia
C) Ischemic acute tubular necrosis caused by hypotension during
catheterization
D) Radiocontrast-induced nephrotoxicity
Question 1
A 82-year-old female with type 2 diabetes mellitus,, hypertension,
hyperlipidemia, and CKD (serum creatinine, 2.3 mg/dL) has exertional
chest pain consistent with myocardial ischemia. A stress test is positive.
Prior to cardiac catheterization, she is given intravenous fluids and n-
acetylcysteine to prevent contrast nephropathy. The left anterior y y p p p y
descending artery is stented, and she is started on clopidogrel.
Approximately 4 days post-procedure, she develops oliguria, azotemia
(creatinine 2.8 mg/dL), abdominal pain, fever and tarry stools. Which of
the following is the most likely cause of AKI?
A) Cholesterol embolization to the renal arteries
B) Pre-renal azotemia
C) Ischemic acute tubular necrosis caused by hypotension during
catheterization
D) Radiocontrast-induced nephrotoxicity
Copyright Harvard Medical School, 2010. All Rights Reserved.
1806
AKI occurred at 4 days post-catheterization
Typical time course for contrast nephropathy 24-
48 hours
Abdominal pain, fever and GI bleeding
suggestive of cholesterol embolization
Other signs/symptoms may include peripheral
eosinophilia,depressed complement levels, blue toes
S di t b ti Sediment may be active
Can be diagnosed on renal biopsy (cholesterol cleft)
Copyright Harvard Medical School, 2010. All Rights Reserved.
1807
Question 2
A 25-year-old female with no significant history, is referred by her PCP
for new onset hypertension. On exam she is thin female with normal
heart and lung exam. There is no edema and exam is otherwise normal.
Blood pressure is 190/95 and equal in both arms. Urinalysis is normal
and labs are notable for a creatinine of 1.2, potassium of 4.0 and , p
bicarbonate of 24. She denies intake of over the counter or prescription
medications, and family history is unremarkable. What is the most
appropriate next step in the work-up of the hypertension?
A) No additional testingsecondary hypertension is unlikely.
B) 24 hour collection for catecholamine
C) Renal artery imaging
D) Genetic testing for Gordons syndrome
Question 2
A 25-year-old female with no significant history, is referred by her PCP
for new onset hypertension. On exam she is thin female with normal
heart and lung exam. There is no edema and exam is otherwise normal.
Blood pressure is 198/95 and equal in both arms. Urinalysis is normal
and labs are notable for a creatinine of 1.1, potassium of 4.0 and , p
bicarbonate of 24. She denies intake of over the counter or prescription
medications, and family history is unremarkable. What is the most
appropriate next step in the work-up of the hypertension?
A) No additional testing secondary hypertension is unlikely.
B) 24 hour collection for catecholamine
C) Renal artery imaging
D) Genetic testing for Gordons syndrome
Copyright Harvard Medical School, 2010. All Rights Reserved.
1808
Renal MRI/MRA is ordered and shows the following.What is the most
appropriate next step in the treat of this patients hypertension?
A) Renal artery bypass.
Question 3
B) Renal artery stenting
C) Renal artery angioplasty
D) ACE-inhibition
Copyright Harvard Medical School, 2010. All Rights Reserved.
1809
Renal MRI/MRA is ordered and shows the following.What is the most
appropriate next step in the treat of this patients hypertension?
A)Renal artery bypass.
Question 3
) y yp
B) Renal artery stenting
C) Renal artery angioplasty
D) ACE-inhibition
Copyright Harvard Medical School, 2010. All Rights Reserved.
1810
Secondary hypertension suggested by
young age and apparently abrupt onset of
hypertension
G d d ( i i i i Gordons syndrome (activating mutation in
thiazide channel/wnk-kinases) unlikely to
have onset in 20s. Also not suggested by
absence of familial history or hyperkalemia
Renal artery stenosis is most common
cause of secondary hypertension and may
be present in >40% of patients with
malignant hypertension
Most common cause of renal artery stenosis in
young female is fibromuscular dysplasia
Typical imaging appearance is string of beads
Responds well to native angioplasty with low risk
of recurrence
Consider imaging carotids to rule out stenosis in g g
other large arteries, as can be systemic
condition
Copyright Harvard Medical School, 2010. All Rights Reserved.
1811
A 43-year-old male construction worker presents after being found at home
lethargic and confused. He was last seen in good health approximately 12
hours earlier. He is afebrile with a BP of 102/55 and tachycardia. He is
arousable but lethargic and confused. Respirations are shallow. Lungs are
clear and abdomen is benign Neurologic exam is non-focal and physical exam
is otherwise unremarkable. Labs and studies are notable for the following:
WBC 9 5 Na 141 K 4 2 Cl 102 Bicarbonate 12 B n 44 Creat 1 5 Gl cose 122
Question 4
WBC 9.5 Na 141 K 4.2 Cl 102 Bicarbonate 12 Bun 44 Creat 1.5 Glucose 122
Laboratory urinalysis has a PH of 5.5, negative ketones, trace protein and rare
RBC and WBC. Head CT and CXR are normal
What is the best combination of tests to determine the etiology of this patients
syndrome?
A) Lumbar puncture, blood and urine cultures
B) Blood gas and urine toxicology B) Blood gas and urine toxicology
C) Blood gas, lactate levels and serum osmolality
D) Methanol and ethanol levels
A 43-year-old male construction worker presents after being found at home
lethargic and confused. He was last seen in good health approximately 12
hours earlier. He is afebrile with a BP of 102/55 and tachycardia. He is
arousable but lethargic and confused. Respirations are shallow. Lungs are
clear and abdomen is benign Neurologic exam is non-focal and physical exam
is otherwise unremarkable. Labs and studies are notable for the following:
Question 4
is otherwise unremarkable. Labs and studies are notable for the following:
WBC 9.5 Na 141 K 4.2 Cl 102 Bicarbonate 12 Bun 44 Creat 1.5 Glucose 122
Laboratory urinalysis has a PH of 5.5, negative ketones, trace protein and rare
RBC and WBC. Head CT and CXR are normal
What is the best combination of tests to determine the etiology of this patients
syndrome?
A) Lumbar puncture, blood and urine cultures
B) Blood gas and urine toxicology
C) Blood gas, lactate levels and serum osmolality
D) Methanol and ethanol
Copyright Harvard Medical School, 2010. All Rights Reserved.
1812
1 hour later the patients wife is reached. She states that the patient has
been depressed lately and that she found an empty bottle of antifreeze
in the kitchen.
What is the approriate treatment for this patient
Question 5
What is the approriate treatment for this patient
A) Ethanol
B) CVVH
C) Dialysis
D) Dialysis and fomepizole ) y p
Copyright Harvard Medical School, 2010. All Rights Reserved.
1813
1 hour later the patients wife is reached. She states that the patient
has been depressed lately and that she found an empty bottle of
antifreeze in the kitchen.
Question 5
What is the approriate treatment for this patient?
A) Ethanol
B) CVVH
C) Dialysis
D) Dialysis and fomepizole
Copyright Harvard Medical School, 2010. All Rights Reserved.
1814
Dominant features in presentation are mental status changes and
possible anion gap acidosis without obvious infectionfindings
should raise red flag to possibility of ingestion
Blood gas is needed to confirm presence of acidosis
Lactate level will rule out lactic acidosis as cause of acidosis Lactate level will rule out lactic acidosis as cause of acidosis
Measuring serum osmolality allows rapid calculation of serum
osmolar gap. Gap>20 is suggestive of methanol, ethanol, ethylene
glycol or isopropyl alcohol ingestion. However, only methanol, and
ethylene glycol are likely to cause this degree of AG acidosis in the
absence of ketones or lactate.
Methanol level and ethylene glycol often not measured locally. Methanol level and ethylene glycol often not measured locally.
Turn-around time may be too low for use in rapid decision making
Where suspicion of ingestion is moderate dialysis/fomepazole may
be initiated on the basis of elevated Osm gap and unexplained
anion gap acidosis while awaiting methanol or ethylene glycol levels
Acids
Ethylene glycol-glycolic acid
Methanolformic acid
Osm gap is produced by ethylene glycol not by glycolic acid.
acidosis may not be present until ethylene glycol is metabolized to
glycolic acid
Osm gap may no longer be present once acidosis is present
CVVH will remove toxic alcohols but more slowly than HD. Thus prologned
HD therapy is treatment of choice for rapid reduction in levels
Indications for dialysis/fomepizole
Acidosis (PH<7.3)
Renal failure
Ethylene glycol level > 50 mg/dL
Methanol level > 50 mg/dL
Copyright Harvard Medical School, 2010. All Rights Reserved.
1815
Question 6
A 68 year old female with ESRD secondary to diabetes and hypertension is on
maintenance HD. Current medications include aspirin, atorvastatin, insulin,
lisinopril (held pre-HD), erythropoietin, calcitriol, calcium acetate, and weekly iron
sucrose. On pre-dialysis exam she generally has basilar rales, 2+ peripheral
edema and BP 150/80. Intradialytic weight gains average 2.5 kg and dry weight is
listed as 72 kg. The nephrologist asks the nurses to change the dry weight to 70
kg in an attempt to remove edema and achieve clear lungs. Over the course of
several weeks the patient has repeated cramping and intra-dialytic hypotension
and is unable to achieve a weight of below 72 kg. The patient currently receives 4
hours of dialysis with sodium modeling (145/140/135). Qb is 400 and QD is 800.
What is the most appropriate next step with this patient?
A) Change dry weight back to 72 kg
B) Give albumin infusions during dialysis
C) Change dialysis orders to 1 hour of ultrafiltration followed by 3 hours of dialysis
D) Cool dialysate to 35 degrees
Question 6
A 68 year old female with ESRD secondary to diabetes and hypertension is on
maintenance HD. Current medications include aspirin, atorvastatin, insulin,
lisinopril (held pre-HD), erythropoietin, calcitriol, calcium acetate, and weekly iron
sucrose. On pre-dialysis exam she generally has basilar rales, 2+ peripheral
edema and BP 150/80. Intradialytic weight gains average 2.5 kg and dry weight is
listed as 72 kg The nephrologist asks the nurses to change the dry weight to 70 listed as 72 kg. The nephrologist asks the nurses to change the dry weight to 70
kg in an attempt to remove edema and achieve clear lungs. Over the course of
several weeks the patient has repeated cramping and intra-dialytic hypotension
and is unable to achieve a weight of below 72 kg. The patient currently receives 4
hours of dialysis with sodium modeling (145/140/135). Qb is 400 and QD is 800.
What is the most appropriate next step with this patient?
A) Change dry weight back to 72 kg
B) Give albumin infusions during dialysis
C) Change dialysis orders to 1 hour of ultrafiltration followed by 3 hours of dialysis
D) Cool dialysate to 35 degrees
Copyright Harvard Medical School, 2010. All Rights Reserved.
1816
Intradialytic hypotension (IDH) is common
complication of dialysis that occurs in 20-
30% of treatments and is a repeated issue
in a subset of patients
Frequent IDH is associated with high risk
of long-term morbidity and mortality
Etiology gy
Impaired catecholamine release in ESRD
Autonomic neuropathy/impaired vascular tone
Increased adenosine levels
Reduce myocardial contractility
Copyright Harvard Medical School, 2010. All Rights Reserved.
1817
Treatments for IDH
Cool dialysate (35 or 36 degrees)
Sodium modeling
Ultrafiltration profiling p g
Albumin/hypertonic saline infusion
Blood volume monitoring
High calcium dialysate
Isolated ultrafiltration
Abd i l b d/ i i l (i Abdominal band/raising legs (increases
venous return)
Adenosine antagonists (not available in U.S.)
Midodrine/vasoprasin
Clinical trials support efficacy of cool dialysate, sodium
modeling, abdominal band, adenosine antagonists,
midodrine and vasopresin
Isolated ultrafiltration and albumin infusion appear to be pp
poorly effective
Cool dialysate
well tolerated by most patients (step down to 35C)
may improve myocardial perfusion/contractility
Free
Na modelling may result in relative hypernatremia-
leading to increase intradialytic weight gains
Copyright Harvard Medical School, 2010. All Rights Reserved.
1818
Question 7
A 77 year old male is admitted with 2-3 days of nausea, vomiting and
constipation. On admission he is noted to be confused and lethargic.
PMH is notable for chrohns disease, hypertension, congestive heart
failure, coronary bypass surgery and chronic kidney disease with a
baseline creatinine of 1.6. On admission he is noted to be hypotensive
and bradycardic with abdominal tenderness, marked abdominal
distension and absent bowel sounds. Neurologic exam is notable for
absent deep tendon reflexes and dilated pupils. Which of the following
best explains his presentation?
A) Hypokalemia
B) H k l i B) Hyperkalemia
C) Hypercalcemia
D) Hyperemagnasemia
Question 7
A 77 year old male is admitted with 2-3 days of nausea, vomiting and
constipation. On admission he is noted to be confused and lethargic.
PMH is notable for chrohns disease, hypertension, congestive heart
failure, coronary bypass surgery and chronic kidney disease with a
b li ti i f 1 6 O d i i h i t d t b h t i baseline creatinine of 1.6. On admission he is noted to be hypotensive
and bradycardic with abdominal tenderness, marked abdominal
distension and absent bowel sounds. Neurologic exam is notable for
absent deep tendon reflexes and dilated pupils. Which of the following
best explains his presentation?
A) Hypokalemia
B) Hyperkalemia B) Hyperkalemia
C) Hypercalcemia
D) Hyperemagnasemia
Copyright Harvard Medical School, 2010. All Rights Reserved.
1819
Question 8
Over the course of 4 hours blood pressure declines, bradycardia
worsens, multiple pressors are required and the patient becomes oliguric.
The patient requires intubation. Imaging is notable for marked illeus and
slight pulmonary congestion. Laboratory data reveals the following:
Na 143 K 4 3 Cl 111 CO2 21 BUN 50 Creatinine 2 1 Glucose 150 Calcium Na 143 K 4.3 Cl 111 CO2 21 BUN 50 Creatinine 2.1 Glucose 150, Calcium
10.0 Phos 3.1, Magnesium 8.9 WBC 8.9, Hemoglobin 11.3
Treatment for this patient should include
A) Dialysis
B) Calcium infusion followed by dialysis
C) Saline and furosemide
D) CVVH
Copyright Harvard Medical School, 2010. All Rights Reserved.
1820
Question 8
Over the course of 4 hours blood pressure declines, bradycardia
worsens, multiple pressors are required and the patient becomes oliguric.
The patient requires intubation. Imaging is notable for marked illeus and
slight pulmonary congestion. Laboratory data reveals the following: slight pulmonary congestion. Laboratory data reveals the following:
Na 143 K 4.3 Cl 111 CO2 21 BUN 50 Creatinine 2.1 Glucose 150, Calcium
10.0 Phos 3.1, Magnesium 8.9 WBC 8.9, Hemoglobin 11.3
Treatment for this patient should include
A) Dialysis
B) Calcium infusion followed by dialysis ) y y
C) Saline and furosemide
D) CVVH
Copyright Harvard Medical School, 2010. All Rights Reserved.
1821
Patient has hypermagnesemia
Symptoms
Nausea/vomitting
Ileus Ileus
Depressed reflexes
Decreased parasympathetic tone (dilated
pupils)
Paralysis/apnea/repsiratory depression
B d di Bradycardia
PR and QT prolongation
Hypotension
Ventricular fibrillation/bradycardic arrest
Most filtered magnesium is reabsorbed therefore toxicity
rarely occurs except when Mg administration is
combined with renal failure
Mild Toxcity < 6 mg/dL
Moderate 6-10 mg/dL
Severe >10 mg/dL
Toxicity more common with oral than intravenous.
magnesium
Severe symptoms have been described with levels
between 6-10 and bradycardia/hypotension not unusual
at this level mg/dL
Copyright Harvard Medical School, 2010. All Rights Reserved.
1822
Mild toxicity can be treated with fluid and
diuretics to promote excretion (if renal function
adequate)
Dialysis is indicated in severe cases, moderate a ys s s d cated se e e cases, ode ate
cases with evidence of significant complications,
or when renal failure is present
CVVH may clear Mg too slowly to be useful in
moderate to severe cases
Calcium administration indicated when cardiac
toxicity present
Mg blocks Ca receptors
Question 9
A 72 year-old hemodialysis patient presents with fever, chills and
positive blood cultures for staph aureus. He dialyzes via a right internal
jugular tunneled line. Medical history is notable for defunct right and j g y g
left upper arm fistulas and stenosis of the left internal jugular and
subclavian veins. Temperature is 38C, HR 100 and BP 140/90. The
patient looks uncomfortable but not toxic. What is the appropriate
management?
A) Immediate line removal and vancomycin for 3 weeks
B) Vancomycin for 3 weeks without line removal
C) Vancomycin for 3 weeks with exchange of line over wire in 2-3 days
D) Gentamycin lock and intravenous vancomycin for 3 weeks
Copyright Harvard Medical School, 2010. All Rights Reserved.
1823
Question 9
A 72 year-old hemodialysis patient presents with fever, chills and
positive blood cultures for staph aureus. He dialyzes via a right internal
jugular tunneled line. Medical history is notable for defunct right and
left upper arm fistulas and stenosis of the left internal jugular and
subclavian veins. Temperature is 38C, HR 100 and BP 140/90. The
patient looks uncomortable but not toxic. What is the appropriate
management?
A) Immediate line removal and vancomycin for 3 weeks
B) Vancomycin for 3 weeks without line removal
C) Vancomycin for 3 weeks with exchange of line over wire in 2 3 C) Vancomycin for 3 weeks with exchange of line over wire in 2-3
days
D) Gentamycin lock and intravenous vancomycin for 3 weeks
Copyright Harvard Medical School, 2010. All Rights Reserved.
1824
Question 10
Over 48 hours the patient develops a pressor requirement and continues
to spike high-grade temperatures. What is the most appropriate step?
A) Echocardiogram to rule out endocarditis A) Echocardiogram to rule out endocarditis
B) Pull the catheter and dialyze via temporary catheter until clinically stable
C) Add gram negative and fungal coverage
D) Citrate lock
Question 10
Over 48 hours the patient develops a pressor requirement and continues
to spike high-grade temperatures. What is the most appropriate step?
A) Echocardiogram to rule out endocarditis
B) Pull the catheter and dialyze via temporary catheter until clinically
stable
C) Add gram negative and fungal coverage
D) Citrate lock
Copyright Harvard Medical School, 2010. All Rights Reserved.
1825
Patient with line infection but clinically stable
Extended antibiotic course is appropriate
Treating through lines is associated with high recurrence and incidence of
complications (enodcarditis, osteo).
Immediate removal is safe but requires temporary access and may burn
through access sites more quickly than catheter lock or guidewire exchange
Guidewire exchange in stable patients preserves access sites, avoids need
for temporary lines and is associated with high cure rates and low recurrent
rates in non randomized series (recurrence <10%)
Not appropriate for unstable patients
Gentamycin/antiobiotic locks highly effective in gram negative infections but
may be less effective in gram positive infections
Copyright Harvard Medical School, 2010. All Rights Reserved.
1826
Question 11
An 35-year-old man presents with acute flank pain and is noted to have a
non-obstructing 6 mm calculus in the right uretero-pelvic junction and
several smaller stones. He is sent home from the emergency room with a
narcotic prescription and told to follow-up with a nephrologist. In narcotic prescription and told to follow up with a nephrologist. In
addition to analgesia, appropriate steps at this point include.
A) Urology referral for lithotripsy
B) 24 hour urine collection and metabolic work-up
C) Straining the urine and analysis of any retrieved stones
D) No work-up is necessary in a first stone episode
E) B & C
Question 11
An 35-year-old man presents with acute flank pain and is noted to have a
non-obstructing 6 mm calculus in the right uretero-pelvic junction and
several smaller stones. He is sent home from the emergency room with a
narcotic prescription and told to follow up with a nephrologist In narcotic prescription and told to follow-up with a nephrologist. In
addition to analgesia, appropriate steps at this point include.
A) Urology referral for lithotripsy
B) 24 hour urine collection and metabolic work-up
C) Straining the urine and analysis of any retrieved stones
D) No work-up is necessary in a first stone episode
E) B & C
Copyright Harvard Medical School, 2010. All Rights Reserved.
1827
Question 12
The patients medical history is remarkable for hypertension. Family
history is negative for stones. Dietary history is unremarkable. His only
medication is atenolol. Urinalysis and serum chemistries are
unremarkable. PTH is within normal limits. A 24 hour urinary collection
shows the following: shows the following:
Volume 1.7 liters, Calcium 400 mg/day, Oxalate 45, Citrate 150 mg/day,
urate 145 mg/day, sodium 190 mg/day PH 6.0
What interventions should you recommend?
A) Increase urine volume, start allopurinol
B) Potassium citrate )
C) Increase fluid intake, decrease salt intake
D) Substitute hydrochlorthizide for atenolol
Copyright Harvard Medical School, 2010. All Rights Reserved.
1828
Question 12
The patients medical history is remarkable for hypertension. Family
history is negative for stones. Dietary history is unremarkable. His only
medication is atenolol. Urinalysis and serum chemistries are
unremarkable. PTH is within normal limits. A 24 hour urinary collection
shows the following:
Volume 1.7 liters, Calcium 400 mg/day, Oxalate 45, Citrate 150 mg/day,
urate 145 mg/day, sodium 190 mg/day PH 6.0
What interventions should should you recommend?
A) Increase urine volume, start allopurinol
B) Potassium citrate
C) Increase fluid intake, decrease salt intake
D) Substitute hydrochlorthizide for atenolol
Copyright Harvard Medical School, 2010. All Rights Reserved.
1829
Need for work-up is equivocal after 1
st
stone, However more than 50%
of stone formers recur within 10 years. Risk of recurrence in this case
higher due to presence of multiple stones on imaging
Calcium oxalate is most common form of stone (70%), followed by
calcium phosphate (10%)
Stone analysis should be performed wherever feasible so that therapy
can be tailored to preventing super-saturation of urine with stone
constituents
Stones less than 8 mm usually pass spontaneously
24 hour urine reveals low volume high calcium excretion 24 hour urine reveals low volume, high calcium excretion,
hypocitrituria and high sodium intake. Supersaturations may be more
important than absolute excretion
Increasing urine volume to > 2L/day is cornerstone of therapy
High Na intake promotes calcium excretion. Thus, it may be
reasonable to increase urine volume and decrease Na intake
as first step with repeat collection in 4-6 weeks.
All i l ff ti i h i i l i t f Allopurinol effective in hyperuricosuric calcium stone formers
Thiazides can decrease calcium excretion and prevent
recurrence
Citrate shown to prevent recurrent stones when given as K-
citrate but not as Na-citrate
A low calcium diet may promote osteopenia, in hypercalcuric
patients and may increase increase urinary oxalate excretion
Copyright Harvard Medical School, 2010. All Rights Reserved.
1830
Question 13
A 82-year-old man is admitted to the intensive care unit following
exploratory laparotomy and resection of a perforation at the illeo-cecal
junction. Over the 1st 4 post-operative days his pressors are weaned and
ventilatory requirements are weaned. On hospital day 5 the patient
becomes lethargic, hypotensive and oliguric and is noted to have g , yp g
intermittent atrial arrhythmias and ventricular ectopy. Abdominal exam is
notable for marked tenderness, and labs are notable for an amylase of
225 mg/dL, creatinine of 1.9 mg/dL bicarbonate of 14, anion Gap of 18 and
CPK of 5800 . What is the most likely explanation of this patients
presentation?
A) Propofol toxicity
B) Propylene glycol toxicity
C) ATN from pancreatitis
D) Dehissance at suture line with intra-abdominal sepsis
Question 13
A 82-year-old man is admitted to the intensive care unit following
exploratory laparotomy and resection of a perforation at the illeo-cecal
junction. Over the 1
st
4 post-operative days his pressors are weaned and
ventilatory requirements are weaned. On hospital day 5 the patient
becomes lethargic, hypotensive and oliguric and is noted to have
intermittent atrial arrhythmias and ventricular ectopy. Abdominal exam is
notable for marked tenderness, and labs are notable for an amylase of
225 mg/dL, creatinine of 1.9 mg/dL bicarbonate of 14, anion Gap of 18 and
CPK of 5800 . What is the most likely explanation of this patients
presentation?
A) Propofol toxicity
B) Propylene glycol toxicity
C) ATN from pancreatitis
D) Dehissance at suture line with intra-abdominal sepsis
Copyright Harvard Medical School, 2010. All Rights Reserved.
1831
Propylene glycol (PG) toxicity can be seen with
prolonged infusion of lorazepam, digoxin, phenytoin,
trimethoprim and diazepam, but most common with
lorazepam.
PG is the carrier for the drug and has higher
concentration in lorazepam infusion than in other p
medications
PG is osmotically active, renally cleared, and
metabolized to lactate
Accumulation can result in osmolar gap, anion gap g p g p
acidosis, and osmotic diuresis with resultant renal failure
Can be dialyzed in severe cases
Copyright Harvard Medical School, 2010. All Rights Reserved.
1832
Multiple potential explanations for patients
symptoms but pancreatitis alone unlikely to
explain arrhythmias, confusion or CPK elevation.
Surgical catastrophe less likely to be associated
with left upper quadrant symptoms, elevated
l CK l k amylase, or CK leak
Patient has propofol infusion syndrome which is
characterized by confusion, cardiac toxicity,
pancreatitis, rhabdomyolysis, lactic acidosis and
kidney injury kidney injury
Risk factors include rapid infusion of propofol
or prolonged infusion (>48 hours)
Treat with propofol withdrawal and supportive
measures
Question 14
A 77-year-old man dialysis patient complains of progressive discomfort
on his arms and legs. Physical examination reveals the following:
n engl j med 351;21
Copyright Harvard Medical School, 2010. All Rights Reserved.
1833
Question 13
presentation?
A) Propofol toxicity
B) Propylene glycol toxicity
What is the most likely
diagnosis?
A) Scleroderma
B) Calciphylaxis
C) ATN from pancreatitis
D) Dehissance at suture line with intra-abdominal sepsis
C) -2 microglobulin
amyloidosis
D) Nephrogenic
fibrosing
dermopathy
n engl j med 351;21
Question 13
presentation?
A) Propofol toxicity
B) Propylene glycol toxicity
What is the most likely
diagnosis?
A) Scleroderma
B) Calciphylaxis
C) ATN from pancreatitis
D) Dehissance at suture line with intra-abdominal sepsis
C) -2 microglobulin
amyloidosis
D) Nephrogenic
fibrosing
dermopathy
n engl j med 351;21
Copyright Harvard Medical School, 2010. All Rights Reserved.
1834
This patient has nephrogenic fibrosing dermopathy
Characterized by diffuse hyperpgimented or yellow brawny
induration of the skin particularly over forearms and shins
May be diffuse and progressive
May lead to disabling flexion contractures
Appearance similar to plaques of scleroderma but distribution is
different and not associated with auto-antibodies.
Biopsy demonstrates CD34 expressing, dermal fibrocytic spindle-
cell proliferation with minimal inflammation
Strongly associated with prior Gadolinium exposure which should be
avoided in patients with ESRD
Role of prophylactic dialysis post-Gadolinium exposure uncertain
Copyright Harvard Medical School, 2010. All Rights Reserved.
1835
Question 15
A 32 year old women is pregnant for the first time. Her past medical
history is notable for bipolar disease currently controlled on
carbamazepine. She is admitted at 11 weeks of pregnancy with nausea
and vomiting. Admission labs are notable for a creatinine 0.5 mg/dL, Na
of 133, K of 4.0, and exam is remarkable for a blood pressure of 100/60, , , p ,
heart rate of 70, moist mucous membranes and brisk capillary refill.
Nephrology consult is requested for evaluation of hyponatremia.
What interventions should you recommend for treatment of
hyponatremia?
A) Start normal saline for volume depletion
B) Fluid restriction to <1.5 L/day while awaiting urine electroltyes for work-up of
hyponatremia
C) Hypertonic saline
D) No therapy is necessary
Question 15
A 32 year old women is pregnant for the first time. Her past medical A 32 year old women is pregnant for the first time. Her past medical
history is notable for bipolar disease currently controlled on history is notable for bipolar disease currently controlled on
carbamazepine carbamazepine. She is admitted at 11 weeks of pregnancy with nausea She is admitted at 11 weeks of pregnancy with nausea
and vomiting. Admission labs are notable for a and vomiting. Admission labs are notable for a creatinine creatinine 0.5 mg/ 0.5 mg/dL dL, Na , Na
of 133, K of 4.0, and exam is remarkable for a blood pressure of 100/60, of 133, K of 4.0, and exam is remarkable for a blood pressure of 100/60,
heart rate of 70, moist mucous membranes and brisk capillary refill. heart rate of 70, moist mucous membranes and brisk capillary refill. p y p y
Nephrology consult is requested for evaluation of Nephrology consult is requested for evaluation of hyponatremia hyponatremia. .
What interventions should you recommend for treatment of
hyponatremia?
A) Start normal saline for volume depletion
B) Fluid restriction to <1.5 L/day while awaiting urine electroltyes for work-up of
h t i hyponatremia
C) Hypertonic saline
D) No therapy is necessary
Copyright Harvard Medical School, 2010. All Rights Reserved.
1836
The sodium level in this patient is normal
for pregnancy. No further testing is
needed
Sodium level decreases approximately 5
meq/L during pregnancy due to changes in
threshold for ADH release production of
placental vasopressinase, and changes in
thirst threshold thirst threshold
Hypertonic saline indicated only for severe
symptomatic hyponatremia
Copyright Harvard Medical School, 2010. All Rights Reserved.
1837
Question 16
The patient is discharged and has an uneventful pregnancy. At 38 weeks
she is admitted with vaginal bleeding. On review of systems, she
complains of frequent urination and thirst. Serum sodium is 138 and
urine volume is 5 liters/day.
Choose the correct answer.
A) The patient most likely has lithium induced diabetes insipidus
B) Serum glucose should be checked to rule out pregnancy induced diabetes
C) dDAVP i lik l t b b tt t t t th AVP C) dDAVP is likely to be a better treatment than AVP
D) A water restriction test should be performed
Question 16
The patient is discharged and has an uneventful pregnancy. At 38 weeks
she is admitted with vaginal bleeding. On review of systems, she
complains of frequent urination and thirst. Serum sodium is 138 and
urine volume is 5 liters/day.
Choose the correct answer.
A) The patient most likely has lithium induced diabetes insipidus
B) Serum glucose should be checked to rule out pregnancy induced diabetes
C) dDAVP i lik l t b b tt t t t th AVP C) dDAVP is likely to be a better treatment than AVP
D) A water restriction test should be performed
Copyright Harvard Medical School, 2010. All Rights Reserved.
1838
The rise in serum sodium at this stage of pregnancy is suggestive of
pregnancy induced diabetes insipidus
Placental vasopresinase increases during pregnancy, and at 22-24
weeks vasopresin clearance is increased 4-fold compared to pre-
pregnancy values. Pre-existing diabetes insipidus may be
exacerbated by this increase in vasopresin clearance
In a minority of patients, the increase in vasopresin may be
exaggerated and cause de novo diabetes insipidus that typically has
an onset during the late 2
nd
or during the 3
rd
trimester of pregnancy
Placental vasopresinase inactivates vasopresinase but has no effect
on its structural analogue ddAVP
In the appropriate setting a failure to increase urine osmolality
with AVP while appropriately responding with the same dose of pp p y p g
ddAVP is diagnostic of pregnancy induced DI

Typically resolves within a few days of delivery


Copyright Harvard Medical School, 2010. All Rights Reserved.
1839
Question 17
A 76 year old PD is patient is admitted with profound hypoglycemia. He
takes 10 units of long-acting insulin twice daily and uses short-acting
insulin at meals according to a sliding scale. His wife states that the
patient has been in his usual state of health. Exam is unremarkable and
temperature is 36.8C. White count is 7 with a normal differential. Chest p
x-ray, abdominal imaging, urinalysis and blood and urine cultures are
negative. The most likely cause of hypoglycemia is:
A) Malfunctioning glucometer
B) Occult infection
C) Icodextrin
D) Reduced insulin clearance
Question 17
A 76 year old PD is patient is admitted with profound hypoglycemia. He
takes 10 units of long-acting insulin twice daily and uses short-acting
insulin at meals according to a sliding scale. His wife states that the
patient has been in his usual state of health. Exam is unremarkable and
temperature is 36.8C. White count is 7 with a normal differential. Chest p
x-ray, abdominal imaging, urinalysis and blood and urine cultures are
negative. The most likely cause of hypoglycemia is:
A) Malfunctioning glucometer
B) Occult infection
C) Icodextrin
D) Reduced insulin clearance
Copyright Harvard Medical School, 2010. All Rights Reserved.
1840
Icodextrin is a glucose polymer that is
metabolized in the circulation and lymphatics to
maltose, maltotriose and maltotetraose
Gl t l b d Glucometers measure glucose based on
ezymatic reactions such glucose oxidase (GOD)
and glucose dehydrogenase using
pyrroloquinolinequinone cofactor (GDH/PQQ)
M lt d th i d t i t b lit Maltose and other icodextrin metabolites may
react in these enzymatic processes and cause
spurious hyperglcyemia that can result in insulin
overdose
Copyright Harvard Medical School, 2010. All Rights Reserved.
1841
Question 18
A 38 year-old male presents with slowly progressive azotemia and
hypertension that is well controlled on a thiazide diuretic and low-dose
ACE-inhibitor. Family history is notable for a mother who developed
ESRD in her 50s of uncertain etiology. He has no other medical history.
Physical exam is notable only for a blood pressure of 135/85. Urinalysis
is bland and other labs are notable for a creatinine of 1 7 mg/dL urea is bland, and other labs are notable for a creatinine of 1.7 mg/dL, urea
nitrogen of 38 mg/dL, potassium of 4.2 meq/L, calcium of 9.6 mg/dL,
phosphorous of 4.3 mg/dL and uric acid of 8.1 mg/dL. Protein:creatinine
ratio is 0.45. Renal ultrasound show 9 cm kidneys with 1 simple cyst on
the left. The most likely diagnosis is:
A) Hypertensive nephropathy
B) Polycystic kidney disease
C) Sarcoidosis
D) Juvenile hyperuricemic nephropathy
Question 18
A 38 year-old male presents with slowly progressive azotemia and
hypertension that is well controlled on a thiazide diuretic and low-dose
ACE-inhibitor. Family history is notable for a mother who developed
ESRD in her 50s of uncertain etiology. He has no other medical history.
Physical exam is notable only for a blood pressure of 135/85. Urinalysis
is bland and other labs are notable for a creatinine of 1 7 mg/dL urea is bland, and other labs are notable for a creatinine of 1.7 mg/dL, urea
nitrogen of 38 mg/dL, potassium of 4.2 meq/L, calcium of 9.6 mg/dL,
phosphorous of 4.3 mg/dL and uric acid of 8.1 mg/dL. Protein:creatinine
ratio is 0.45. Renal ultrasound show 9 cm kidneys with 1 simple cyst on
the left. The most likely diagnosis is:
A) Hypertensive nephropathy
B) Polycystic kidney disease
C) Sarcoidosis
D) Juvenile hyperuricemic nephropathy
Copyright Harvard Medical School, 2010. All Rights Reserved.
1842
This patient has juvenile hyperuricemic nephropathy
Caused by genetic defects in uromodulin gene
(Tamm-Horsfall protein)
Associated with hyperuricemia, gout, and slowly
progressive bland renal failure that first appears in the
3-5
th
decades and progresses over 10+ years
Small kidneys on ultrasound
Medullary cysts may be present at late stages but are
difficult to detect on ultrasound
Proteinuria generally < 1 gm/day
Renal biopsy may show interstitial nephritis but Renal biopsy may show interstitial nephritis but
changes are non-specific on standard biopsy
Distribution of Tamm-Horsfall protein altered
Copyright Harvard Medical School, 2010. All Rights Reserved.
1843
Question 19
What is the appropriate dose of CVVH for patients with AKI?
A) 2000 mL/hour of effluent
B) 35 mL/kg/hour of replacement solution
C) 20 mL/kg/hour of replacement solution
D) 20 mL/kg/hour of effluent solution
Question 19
What is the appropriate dose of CVVH for patients with AKI?
A) 2000 mL/hour of effluent
B) 35 mL/kg/hour of replacement solution
C) 20 mL/kg/hour of replacement solution
D) 20 mL/kg/hour of effluent solution
Copyright Harvard Medical School, 2010. All Rights Reserved.
1844
With convective renal replacement therapy
(CVVH), effluent is fully saturated with
respect to low molecular weight substrates
Equivalent to saying sieving coefficient is 1
Clearance is thus equal to the effluent rate
minus correction for any pre-dilution
Data has been conflicting but larger Data has been conflicting but larger
studies including Australia-New Zealand
trial and VA-NIH trial suggest no benefit
above effluent rate of 20 mL/kg/hour
Copyright Harvard Medical School, 2010. All Rights Reserved.
1845
Question 20
A 71 kg 18 year male with no significant medical history is in an a high
speed motor vehicle accident rupturing his spleen and injuring his bowel.
He receives intravenous contrast for a CT scan while hypotensive in the
emergency room and develops oliguric acute kidney injury. 3 days later,
urine output has increased to 1 liter/day. However he remains intubated p y
and potassium has increased to 7.2 meq/L and renal is called to initiate
hemodialysis. Platelets are 80,000 and INR is 1.5. What is the most
appropriate site and type of dialysis access?
A) Non-tunneled subclavian
B) Tunneled internal jugular
C) Non-tunneled femoral
D) Non-tunneled jugular
Question 20
A 71 kg 18 year male with no significant medical history is in an a high
speed motor vehicle accident rupturing his spleen and injuring his bowel.
He receives intravenous contrast for a CT scan while hypotensive in the
emergency room and develops oliguric acute kidney injury. 3 days later,
urine output has increased to 1 liter/day. However he remains intubated
and potassium has increased to 7.2 meq/L and renal is called to initiate
hemodialysis. Platelets are 80,000 and INR is 1.5. What is the most
appropriate site and type of dialysis access?
A) Non-tunneled subclavian
B) Tunneled internal jugular
C) Non-tunneled femoral
D) Non-tunneled jugular
Copyright Harvard Medical School, 2010. All Rights Reserved.
1846
Tunnelled lines may reduce risk of
infection and may reduce need for later
transition to tunneled access (with 2
nd
procedure) in patients without infection
and those with preexisting CKD
Subclavian access associated with high
risk of venous stenosis. May ruin
ipsilateral side for future AV access and ipsilateral side for future AV access and
should generally be avoided as site of
temporary lines
Copyright Harvard Medical School, 2010. All Rights Reserved.
1847
In RCT no site-related difference in risk of
infection with short term (<1 wk)
placement of dialysis access in jugular or
femoral location
Jugular site may be better in obese patients
No difference in dialysis efficacy rates with
femoral vs. jugular location
Femoral vessels more compressible and
may be better choice in coagulopathic
patients
References
The VA/NIH Acute Renal Failure Trial Network. Intensity of Renal Support in
Critically Ill Patients with Acute Kidney Injury. NEJM. 359:7-20;2008
RENAL Replacement Therapy Study Investigators. Intensity of continuous p py y g y
renal replacement therapy in critically ill patients. NEJM. 361:1627-38;2009
Kraut JA, Kurtz I. Toxic Alcohol Ingestions: Clinical Features Diagnosis, and
management. CJASN. 3:208-25;2008
Parienti JJ. Femoral vs Jugular Venous Catheterization and Risk of
Nosocomial Events in Adults Requiring Acute Renal Replacement Therapy: A
Randomized Controlled Trial JAMA 299: 2413 2422 2008 Randomized Controlled Trial. JAMA 299: 2413 - 2422. 2008.
Parienti JJ. Catheter dysfunction and dialysis performance according to
vascular access among 736 critically ill adults requiring renal replacement
therapy: a randomized controlled study. Crit Care Med. 38:1118-25;2010
Copyright Harvard Medical School, 2010. All Rights Reserved.
1848
Disclosures
Dr Charytan receives funding from the American
Society of Nephrology, Paul Teschan Research
Fund (DCI), and the American Heart Association. Fund (DCI), and the American Heart Association.
Copyright Harvard Medical School, 2010. All Rights Reserved.
1849
Anil Chandraker MB FRCP
Medical Director of Kidney Transplantation
KEY TRANSPLANT STUDIES
What You Should Know for the Boards
Disclosures
T2 Biosystems - Consultant & Scientific
Advisory Board
Novartis - Data Safety and Monitoring
Board
UpToDate- Reviewer of content
Copyright Harvard Medical School, 2010. All Rights Reserved.
1850
Comparison of mortality in all
patients on dialysis, patients on
dialysis awaiting transplantation,
and recipients of a first cadaveric
transplant.
Wolfe RA, Ashby VB, Milford EL, Ojo AO,
Ettenger RE, Agodoa LY, Held PJ , Port FK.
N Engl J Med. 1999 Dec 2;341(23):1725-30
0 106 183 244 365 548
4.00
2.84
1.00
0.32
0.25
Days since Transplantation
R
e
l
a
t
i
v
e

R
i
s
k

o
f

D
e
a
t
h
Risk
equal
Survival
equal
Copyright Harvard Medical School, 2010. All Rights Reserved.
1851
Findings
The relative risk of death during the first 2
weeks after transplantation was 2.8 times
as high as that for patients on dialysis
At 18 months the risk was much lower 0.32
The likelihood of survival became equal in
the two groups within 5 to 673 days after
transplantation
Copyright Harvard Medical School, 2010. All Rights Reserved.
1852
Overall Message
Regardless of age, race or
cause of renal failure
transplantation offers better
long-term results as compared
with dialysis
High survival rates of kidney
transplants from spousal and
living unrelated donors
Terasaki PI, Cecka J M, Gjertson DW,
Takemoto S.
N Engl J Med. 1995 Aug 10;333(6):333-6
Copyright Harvard Medical School, 2010. All Rights Reserved.
1853
Graft survival in living unrelated kidneys
Terasaki et al NEJM 1995; 333:333
Copyright Harvard Medical School, 2010. All Rights Reserved.
1854
Findings
3 Year Survival:
81% Unrelated donors
82% Parental donors
70% Cadaveric donors
87% Spousal donors
76% Wife to husband (previously pregnant) (NS)
90% In untransfused recipients
81% In transfused recipients (p=0.008)
Overall Message
Living donation as compared
with deceased donor kidneys
results in better long-term
survival regardless of HLA
matching
Copyright Harvard Medical School, 2010. All Rights Reserved.
1855
Incidence of BK with Tacrolimus
Versus Cyclosporine and Impact of
Preemptive Immunosuppression
Reduction
Brennan DC, Agha I, Bohl DL, Schnitzler MA,
Hardinger KL, Lockwood M, Torrence S, Schuessler R,
Roby T, Gaudreault-Keener M, StorchGA.
Am J Transplant. 2005 Mar;5(3):582-94
Findings
200 adult renal transplant recipients were
randomized to FK506 (n = 134) or CyA(n =
66). Urine and blood were collected weekly
for 16 weeks & months 5, 6, 9 and 12 for BK
by PCR
By 1 year, 70 patients (35%) developed
viruriaand 23 (11.5%) viremia
After reduction of immunosuppression,
viremiaresolved in 95%, without increased
acute rejection, allograft dysfunction or graft
loss. No BK nephropathy was observed.
Copyright Harvard Medical School, 2010. All Rights Reserved.
1856
5 Year Follow-up
5-year patient survival was 91% & graft
survival was 84%.
Immunosuppression and viremiadid not
influence graft survival.
Acute rejection occurred in 12% by 5-years
after transplant, was less common with
tacrolimus versus cyclosporine (9% vs. 18%;
p= 0.082).
Tacrolimus was associated with better renal
function at 5-years (eGFR 63 FK vs. 52
CsAmL/min, p = 0.001).
Overall Message
Screening for BK virus and
pre-emptive reduction in
immunosuppression prevents
development of BK
nephropathy in the majority of
cases with only a low risk of
acute rejection.
Copyright Harvard Medical School, 2010. All Rights Reserved.
1857
Plasma exchange and tacrolimus-
mycophenolate rescue for acute
humoral rejection in kidney
transplantation.
Pascual M, SaidmanS, Tolkoff-Rubin N, Williams WW,
Mauiyyedi S, Duan J M, Farrell ML, Colvin RB, Cosimi
AB, Delmonico FL.
Transplantation. 1998 Dec 15;66(11):1460-4
Findings
5 out of 73 patients diagnosed with AHR
Severe resistant rejection to steroids and ATG
Typical pathological features
Neutrophils in peritubular capillaries
Glomerulitis,
Fibrin thrombi
Vasculitis
Fibrinoid necrosis in vessel walls
Demonstration of DSA in recipient serum
Copyright Harvard Medical School, 2010. All Rights Reserved.
1858
Treatment
4-7 Plasma exchanges per patient (1.3
volume)
IVIg 0.4 g/Kg after each exchange (to
prevent infectious complication)
Rescue therapy with MMF and Tacrolimus
instead of CsA and azathioprine
Copyright Harvard Medical School, 2010. All Rights Reserved.
1859
Overall Message
Antibody mediated rejection causes acute
allograft rejection after transplantation and
can be treated with plasmapheresis, (IVIG)
and modification of the
immunosuppressive regimen.
Complement activation in acute
humoral renal allograft rejection:
diagnostic significance of C4d
deposits in peritubular capillaries.
Collins AB, Schneeberger EE, Pascual MA,
SaidmanSL, Williams WW, Tolkoff-Rubin N,
Cosimi AB, Colvin RB
J Am Soc Nephrol. 1999 Oct;10(10):2208-14.
Copyright Harvard Medical School, 2010. All Rights Reserved.
1860
Findings
16 biopsies from 10 patients that met the
following inclusion criteria:
Acute graft dysfunction;
Prominent neutrophils in corticalperitubular
capillaries or fibrinoid necrosis of arteries;
A positive antidonor T and/or B cell cross-
match in a concurrentserum.
mononuclear
cells are present
in the
interstitiumand
peritubular cap
abundant
neutrophils are
present
in dilated
peritubular
capillaries
neutrophils are
present in
glomerular
capillaries
Scattered
mononuclear
cells are present
in glomerular
capillaries
Negative C4d
Positive C4d
Acute
cellular
Rejection
Acute
AMR
J Am Soc Nephrol 13: 779787, 2002
Copyright Harvard Medical School, 2010. All Rights Reserved.
1861
abundant
neutrophils are
present
in dilated
peritubular
capillaries
neutrophils are
present in
glomerular
capillaries
Positive C4d
Widespread C4d staining in
PTC was present in
30%(20 of 67) of all acute
rejection biopsies.
Acute
AMR
J AmSocNephrol 13: 779787, 2002
Overall Message
Diagnosis of acute AMR
J AmSoc Nephrol 18: 10461056, 2007
Clin J Am Soc Nephrol 1: 415420, 2006.
Copyright Harvard Medical School, 2010. All Rights Reserved.
1862
Improved graft survival after renal
transplantation in the United
States, 1988 to 1996.
HariharanS, J ohnson CP, BresnahanBA, Taranto
SE, McIntosh MJ , StableinD.
N Engl J Med. 2000 Mar 2;342(9):605-12.
Findings
Analyzed survival for all 93,000 kidney transplants
between 1988-1996
1 year graft survival increased from 88.8 to 93.9%
Half life for living donors increased from 12.7
years to 21.6 years
Half life for deceased donors increased from 7.9
years to 13.8 years
CONCLUSION: a substantial increase in short-
term and long-term survival of kidney grafts from
both living and cadaveric donors
Copyright Harvard Medical School, 2010. All Rights Reserved.
1863
Improved Graft Survival After Renal
Transplantation in the United States 1988-1996
Hariharan et al NEJM 2000;342:605-612
Long-term renal allograft survival:
have we made significant progress
or is it time to rethink our analytic
and therapeutic strategies?
Meier-Kriesche HU, ScholdJ D,
Kaplan B.
Am J Transplant. 2004 Aug;4(8):1289-95.
Copyright Harvard Medical School, 2010. All Rights Reserved.
1864
Long-Term Renal Allograft Survival: Have we Made Significant Progress or is it Time to Rethink our
Analytic and Therapeutic Strategies?. Meier-Kriesche, Herwig-Ulf, Schold, J esse D. & Kaplan, Bruce
American Journal of Transplantation 4 (8), 1289-1295.
Projected half-lives vs. actual half-lives in deceased
donor transplants: 1988-1995
Kaplan-Meier overall graft survival by year of
transplant: deceased donor re-transplants 19881995
Copyright Harvard Medical School, 2010. All Rights Reserved.
1865
Overall Message
Longterm allograft survival has not
increased dramatically despite a reduction
in the rates of acute rejection.
What is causing longterm allograft loss?
N Engl J Med 2003; 349: 2326-2333, 2003
Copyright Harvard Medical School, 2010. All Rights Reserved.
1866
Findings
Prospective study of CAN in 120 kidney-
pancreas recipients
Analysis of 961 kidney biopsy samples over 10
years
Risk of subclinical rejection was common within
the first year (45.7%)
Chronic allograft nephropathy was almost
universal by 10 years
Calcineurin Inhibitor Nephrotoxicity
Nankivell BJ, et al. N Engl J Med. 2003;349:2326-2333. (2b)
80
Acute Rejection
Acute Subclinical Rejection
Borderline Subclinical Rejection
60
40
20
0
0.25 0.5 0.75 1.0 2.0 3.0 4.0 5.0 6.0 7.0 8.0 9.0 10.0
Years After Transplantation
P
r
e
v
a
l
e
n
c
e

(
%
)
100
75
50
25
0.25 0.5 0.75 1.0 2.0 3.0 4.0 5.0 6.0 7.0 8.0 9.0 10.0
C
a
l
c
i
n
e
u
r
i
n

N
e
p
h
r
o
t
o
x
i
c
i
t
y

(
%
)
0
Years After Transplantation
100
Copyright Harvard Medical School, 2010. All Rights Reserved.
1867
Overall Message
Chronic allograft nephropathy represents
cumulative and incremental damage to
nephrons from time-dependent
immunologic and nonimmunologic causes.
Chronic renal failure after
transplantation of a nonrenal
organ.
OjoAO, Held PJ , Port FK, Wolfe RA, Leichtman
AB, Young EW, Arndorfer J , Christensen L,
Merion RM.
N Engl J Med. 2003 Sep 4;349(10):931-40.
Copyright Harvard Medical School, 2010. All Rights Reserved.
1868
CNI use in solid organ transplant, and the burden
of chronic kidney disease
Organ At Discharge At 1 year
1
Kidney 93% 79%
Liver 97% 93%
Heart 98% 93%
Lung 100% 92%
1
H.-U. Meier-Kriesche et al, Am J
Transplant 2006; 6: 1111
Heart Lung Liver
Years after transplant
1 5 10
50
25
0
CKD (%with GFR<30)
2
2
Ojo AO et al, NEJ M
2003;349: 931
%
Findings
Median follow-up of 36 months
Chronic renal failure developed in 11,426 patients
(16.5 percent)
3297 (28.9 percent) required maintenance dialysis
or renal transplantation
The five-year risk of chronic renal failure
6.9 % among heartlung transplants
10.2 % among heart transplants
21.3 % among intestine transplants
18.1 % among liver transplants
15.8% among lung transplants
Copyright Harvard Medical School, 2010. All Rights Reserved.
1869
Cumulative Incidence of Chronic Renal Failure among 69,321 Persons
Who Received Nonrenal Organ Transplants 1990-2000
Chronic Renal Failure after Transplantation of a Nonrenal Organ
Akinlolu O. Ojo, Robert M. Merion, M.D. Volume 349:931-940 September 4, 2003
Overall Message
Calcineurin inhibitors contribute to long
term allograft loss and are an important
factor in limiting allograft survival.
Copyright Harvard Medical School, 2010. All Rights Reserved.
1870
Predicting kidney graft failure by
HLA antibodies: a prospective trial.
Terasaki PI, Ozawa M.
Am J Transplant. 2004 Mar;4(3):438-43..
Findings
Copyright Harvard Medical School, 2010. All Rights Reserved.
1871
Post Transplant Sensitzation
17.8% to 27% have HLA Ab* post transplant
N=2278 Graft Failure
(all Ab)
Graft failure
(de novo Ab)
Antibodies 6.6% 8.6%
No Antibodies 3.3% 3.0%
Terasaki. Am J Trans 2004: 4;1.
* CDC, ELISA and Flow techniques used overall
prevalence is underestimated
Lee. Transplantation 2002: 74; 1192.
De novo HLA Ab Precede Graft Failure by Months
to Years
Copyright Harvard Medical School, 2010. All Rights Reserved.
1872
Overall Message
Development of de novo anti-HLA
antibodies after transplantation is common
Abs associated with increased risk of graft
failure
Least antibodies seen in cohort
transplanted between 1996-1998
Beneficial effects of treatment of
early subclinical rejection: a
randomized study.
Rush D, Nickerson P, Gough J , McKenna R,
Grimm P, Cheang M, TrpkovK, Solez K,
J effery J .
J Am Soc Nephrol. 1998 Nov;9(11):2129-34.
Copyright Harvard Medical School, 2010. All Rights Reserved.
1873
Findings
In CsA era 30% subclinical rejection rate
72 patients, stratified by donor source, were
randomized to biopsies at 1, 2, 3, 6, and 12
m
vs
6- and 12-mo biopsies only
Treatment of subclinical rejection lead to
decrease in early and late acute rejection rates
Copyright Harvard Medical School, 2010. All Rights Reserved.
1874
Factors associated with progression of
interstitial fibrosis in renal transplant
patients receiving tacrolimus and
mycophenolate mofetil.
Rush DN, CockfieldSM, Nickerson PW, Arlen DJ ,
Boucher A, Busque S, GirardinCE, Knoll GA,
Lachance J G, Landsberg DN, Shapiro RJ , Shoker A,
Yilmaz S.
Transplantation. 2009 Oct 15;88(7):897-903.
Findings
Original study in Tacrolimus treated patients
showed that protocol biopsy did not improve
outcomes at 3 years
Follow up study at 24 months 111 PBx and
107 controls completed the study
At 24 months, graft function was excellent
The prevalence of interstitial fibrosis and
tubular atrophy (IF/TA)ci + ct more than or
equal to 2increased from approximately 3%
at baseline to up to 40% to 50%.
Copyright Harvard Medical School, 2010. All Rights Reserved.
1875
Overall Message
Protocol biopsies may be helpful with
CsA/Aza immunosuppression regimen
Incidence of subclinical rejection not as
high on Tac/MMF regimen c/wCsA/Aza.
No benefit in short-term outcomes but still
have a progression of IFTA
Cyclosporine sparing with
mycophenolate mofetil, daclizumab
and corticosteroids in renal allograft
recipients: the CAESAR Study.
Ekberg H, Griny J , NashanB,
VanrenterghemY, Vincenti F, Voulgari A,
Truman M, Nasmyth-Miller C, RashfordM.
Am J Transplant. 2007 Mar;7(3):560-70.
Copyright Harvard Medical School, 2010. All Rights Reserved.
1876
CAESAR Study Design
Ekberg H et al. Am J Transplant 2006
Daclizumab
Low-CsA w/d
MMF
Steroids
Standard CsA
MMF
Steroids
0 6 12 mo
Daclizumab
Low-
CsA
MMF
Steroids
Low
CsA w/d
Stand
CsA
Low
CsA
50-100 ng/mL
150-300 ng/mL, 4 mo.: 100-200
50-100 ng/mL
Withdrawal 4-6 mo.
Summary: CAESAR Study
Low-dose CsA, MMF, steroids and
daclizumab was safe and effecti ve, but did
not improve function
Withdrawal of CsAat 4-6 months
was associated with increased risk of AR
and no benefits of improved function
Ekberg H et al. Am J Transplant 2006
Funded by ROCHE
Copyright Harvard Medical School, 2010. All Rights Reserved.
1877
Reduced exposure to calcineurin
inhibitors in renal transplantation.
Ekberg H, Tedesco-Silva H, Demirbas A, Vtko S,
Nashan B, GrkanA, Margreiter R, Hugo C, Griny
J M, Frei U, VanrenterghemY, Daloze P, Halloran PF.
ELITE-Symphony Study.
N Engl J Med. 2007 Dec 20;357(25):2562-75.
SYMPHONY Study Design
Tx 6 mo 12 mo
Daclizumab
Low dose CsA
MMF
Steroids
B
50100 ng/ml
Low dose SRL
D
MMF
Steroids
Daclizumab
4-8 ng/ml
Low dose TAC
MMF
Steroids
Daclizumab 3-7 ng/ml
C
150-300 ng/ml for 3 months
100-200 ng/ml thereafter
Normal dose CsA
MMF
Steroids
A
Copyright Harvard Medical School, 2010. All Rights Reserved.
1878
The Largest Prospective
De Novo Kidney Transplant Study
1645patients
83 sites
15 countries
316
275
258
179
123
78
71
64 64
59
55
34 32
23
14
N
u
m
b
e
r

o
f

p
a
t
i
e
n
t
s

e
n
r
o
l
l
e
d
Renal Function
Calculated GFR at month 12
(Cockcroft-Gault)
ITT GFR (ml/min)
n median mean SD
Normal-dose CsA 390 57.0 57.1 25.1
Low-dose CsA 399 61.0 59.4 25.1
Low-dose TAC 401 66.2 65.4 27.0
Low-dose SRL 399 57.5 56.7 26.9
p<0.0001
p=0.0011
p<0.0001
Copyright Harvard Medical School, 2010. All Rights Reserved.
1879
Biopsy Proven Acute Rejection
(ITT, Excluding Borderline)
26%
24%
12%
37%
p<0.0001
p<0.0001
0
10
20
30
40
50
12 months post-Tx
B
P
A
R

(
%

o
f

p
a
t
i
e
n
t
s
)
Normal-dose CsA
Low-dose CsA
Low-dose TAC
Low-dose SRL
Graft and Patient Survival
Low-dose TAC Low-dose SRL
p=0.0147 p=0.0143
89%
93%
94%
89%
70
80
90
100
12 months post-Tx
G
r
a
f
t

s
u
r
v
i
v
a
l

(
%
)
70
80
90
100
12 months post-Tx
P
a
t
i
e
n
t

s
u
r
v
i
v
a
l

(
%
)
p = NS
96%
98%
97%
97%
Normal-dose CsA Low-dose CsA
Copyright Harvard Medical School, 2010. All Rights Reserved.
1880
Safety
Overall
infections
*
CMV
infections
*
Lympho
-celes
*
Diarrhoea
*
Diabetes
mellitus*
(post-Tx)
Wound
not
healed

Malig-
nancy
**
Normal-
dose CsA
65.6%
15.3%
6.9% 17.5% 6.3% 10.9% 1.3%
Low-dose
CsA
57.7% 11.5% 7.0% 14.2% 4.8% 11.0% 1.0%
Low-dose
TAC
58.3% 10.2% 3.7%
27.3% 10.6%
9.4% 2.0%
Low-dose
SRL
62.5% 6.5% 15.3% 23.9% 7.3%
16.6% 2.4%
* Kaplan-Meier estimates

at week 2 ** diagnosed within first 12 months post-Tx


Graft and Patient Survival
Low-dose TAC Low-dose SRL
p=0.0147 p=0.0143
89%
93%
94%
89%
70
80
90
100
12 months post-Tx
G
r
a
f
t

s
u
r
v
i
v
a
l

(
%
)
70
80
90
100
12 months post-Tx
P
a
t
i
e
n
t

s
u
r
v
i
v
a
l

(
%
)
p =NS
96%
98%
97%
97%
Normal-dose CsA Low-dose CsA
Copyright Harvard Medical School, 2010. All Rights Reserved.
1881
Disclosures
T2 Biosystems - Consultant & Scientific
Advisory Board
Novartis - Data Safety and Monitoring
Board
UpToDate- Reviewer of content
Copyright Harvard Medical School, 2010. All Rights Reserved.
1882
TRANSPLANTATION IMMUNOBIOLOGY
FOR THE NEPHROLOGIST
Nader Najafian, M.D.
nnajafian@rics.bwh.harvard.edu
Disclosures:
None
Copyright Harvard Medical School, 2010. All Rights Reserved.
1883
DEFINITIONS
Isograft(syngeneic)-identical to self
Identical twins
Inbred mice
Allograft-between individuals of the same
species
Xenograft-between individuals of different
species
STEPS IN ALLOGRAFT REJECTION
1-Recogniti on of alloantigen:
T cell-APC interaction (importance of peripheral
lymphoid organs for nave T cell recognition)
T cell-endothelial cell interaction
2-T cell activati on:
T cell costimulation
Cytokine and chemokine production
Proliferation and clonal expansion
T helper cell function (CD4+ T cells)
Copyright Harvard Medical School, 2010. All Rights Reserved.
1884
STEPS IN ALLOGRAFT REJECTION
3-Effector mechanisms leading to graft
destruction:
Lymphocyte mediated cytotoxicity (CD8+ T cells)
Alloantibodies: complement and cell mediated
cytotoxicity
Delayed type hypersensitivity (macrophages)
4-Resoluti on of the response with residual
memory
STEP ONE
Recognition of Alloantigen
Copyright Harvard Medical School, 2010. All Rights Reserved.
1885
CD4+/CD8+ T CELL
APC/Target Cell
Peptide
Class I / II MHC
CD3
CD4/CD8
TCR
ANTIGEN RECOGNITION
T Cells Play A Key Role in
Rejection
T deficient
T cells
T deficient
Copyright Harvard Medical School, 2010. All Rights Reserved.
1886
A1 B1
CHROMOSOME 6
5' 3'
B1 A1 B3 B4 A B1
DP DQ DR B C A
MHC CLASS I MHC CLASS II
A1
B1
HLA Complex
MHC (HLA) Antigens
Cell Surface
Two Types
Class I
Class II
Several Loci
A,B,C
DP,DQ,DR
Many Alleles
Polymorphism
Co-dominant Expression
Copyright Harvard Medical School, 2010. All Rights Reserved.
1887
The Function of MHC
Molecules
Cell Surface
Cell Proteins
Peptide of Cell Proteins
DIRECT
ALLORECOGNITION
INDIRECT
ALLORECOGNITION
ALLORECOGNITION
T CELL
RECIPIENT
APC
Allopeptide
Endogenous
peptide
T CELL
DONOR
APC
Copyright Harvard Medical School, 2010. All Rights Reserved.
1888
T Cell Selection in the Thymus
The repertoire of mature T cells is selected by two processes:
Positive Selection: only T cells with receptors that recognize
something that looks like a self MHC molecule
Negative Selection: no T cells with receptors that recognize self
MHC molecules too well (protects against autoimmunity)
T cell
receptor
T cell
receptor YES
NO
S S
Allogeneic MHC Antigens
Fundamental principle in immunology: T cells dont
recognize foreign proteins directly, they recognize
peptides of foreign proteins presented by self MHC
molecules on antigen presenting cells
Allogeneic MHC molecules have the unique property
that they look so much like self MHC molecules, but
arent, that T cells can recognize them directly as
foreign
Copyright Harvard Medical School, 2010. All Rights Reserved.
1889
T Cell Precursor Frequency
The precursor frequency for T cells that recognize
environmental pathogens: 1/10,000
The precursor frequency for T cells that recognize
allogeneic MHC antigens: 1/100-1/1000
Because allogeneic MHC antigens can be
recognized directly, they give rise to a
T cell response that is 100x more powerful
than an ordinary T cell response!
MODEL -REJECTION BY INDIRECT
ALLORECOGNITION
B6 II- BALB/C
(Lack MHC Class II)
Auchincloss et al.
Copyright Harvard Medical School, 2010. All Rights Reserved.
1890
MODEL - REJECTION BY DIRECT
ALLORECOGNITION
X
BALB/C
B6 II- Thymic MHC II
B6 II-4
+
Auchincloss et al.
DIRECT & INDIRECT ALLORECOIGNITION
MEDIATE GRAFT REJECTION
0
20
40
60
80
100
0 10 20 30 40 50
BALB/c to B6
B6 to BALB/C
BALB/c to B6II-4+
B6II- to BALB/C
%

S
u
r
v
i
v
a
l
Days after skin transplant
Copyright Harvard Medical School, 2010. All Rights Reserved.
1891
VASCULARIZED CARDIAC ALLOGRAFT
REFECTION BY CD4+ T CELLS
- Indirect pathway alone -
0
20
40
60
80
100
P
e
r
c
e
n
t

s
u
r
v
i
v
a
l
0 10 20 30 40 50 60
Days after transpl antati on
Indirect pathway alone: no treatment
Indirect pathway alone: depleting anti -CD8
Yamada et al., Tx 2001
aorta
#860, day 236
CHRONIC ALLOGRAFT VASCULOPATHY
- Indirect pathway alone: CD4+ T cells -
Yamada et al., Tx 2001
Copyright Harvard Medical School, 2010. All Rights Reserved.
1892
INDIRECT ALLORECOGNITION AND
CHRONIC REJECTION
ELISPOT ANALYSIS OF PERIPHERAL BLOOD
LYMPHOCYTE REACTIVITY TO DONOR HLA-
DR PEPTIDES: A POTENTIAL NOVEL ASSAY
FOR PREDICTION OF OUTCOMES FOR RENAL
TRANSPLANT RECIPIENTS
N Naj afi an, AD Sal ama, EV Fedoseyeva, G Beni chou, MH
Sayegh
JASN, 13:252-259 , 2002
BRIGHAM AND WOMEN S HOSPITAL
CHILDREN S HOSPITAL
HARVARD MEDICAL SCHOOL
BOSTON, MA
Copyright Harvard Medical School, 2010. All Rights Reserved.
1893
IMMUNE REACTIVITY IN STABLE AND
HIGH RISK PATIENTS
23%
77% 73%
27%
Stable >1 rejection
<60 spots/million: 77%
>60 spots/million: 23%
<60 spots/million: 27%
>60 spots/million: 73%
p=0.02 by two tailed Chi squared test
Najafianet al J ASN 2002
Frequency of 60/million IFN-g producing T cells
MINOR TRANSPLANTATION
ANTIGENS
-Minor antigens are non-MHC, donor-deri ved
peptide determinants expressed in the
context of MHC molecules common to the
recipient and the donor
-A recipients can reject a graft matched at all
MHC loci (graft from one MHC-matched
sibling to another)
-Examples: H-Y proteins, mitochondrial
proteins (MTF, MTF), myosin related
protein (HA-2) etc.
Copyright Harvard Medical School, 2010. All Rights Reserved.
1894
ROLE OF LYMPHOID ORGANS IN
GRAFT REJECTION
Vascularized allografts do not initiate
rejection in the absence of secondary
lymphoid tissue (lymph nodes and spleen)
Skin allografts rely primarily on lymph
nodes while vascularized allografts utilize
either lymph nodes or the spleen to initiate
rejection
Secondary lymphoid organs are not
required for effector or memory cells
Lakkis et al. Nature Medicine 2000
STEP TWO
T Cell Activation
Copyright Harvard Medical School, 2010. All Rights Reserved.
1895
MHC APC
T Cell
TCR
IL-2 etc.
CD4
T Cells Require 2 Signals
S1
MHC
APC
T Cell
TCR
IL-2 etc.
CD4
T Cells Require 2 Signals
S1
Costimulatory Signal
S2
Copyright Harvard Medical School, 2010. All Rights Reserved.
1896
APC
T Cell
LFA1
ICAM-1
B7
CD28 LFA1
ICAM-1
CD2
LFA3
CD154
CD40
MHC class II
CD4
TCR
CD3
CD45
?
B7
CTLA4
T CELL COSTIMULATION
The Happy days
Denton et al., Lancet 1999
Clinical Development of T Cell
Costimulatory Blockade
B7: Renal Tx
Lupus
MS
RA
?
CD154: ? Future
CD40: Pre-clinical
Others: ?
Copyright Harvard Medical School, 2010. All Rights Reserved.
1897
LEA29Y Phase II Dose-Finding Study Design
*All patients received MMF, basiliximab, and corticosteroid-tapering regimen
Clinical endpoint
(LEA29Y arms unblinded)
Clinical
endpoint
Randomization
1 month 3 months 6 months 1 year
LEA29Y*
15 29 43 57 71 85
10 mg/kg
113 141 169
5 mg/kg every 4 or 8 weeks
5 mg/kg every 4 or 8 weeks
DAY 1 5
10 mg/kg
LEA29Y Phase II Trial: Results
Equivalent acute rejection rates
Improved GFR
Less CAN by biopsy
LEA29Y is safe
Heralds a new era of chronic
biologicals in transplantation?!
Vincenti et al. NEJM 2005
Copyright Harvard Medical School, 2010. All Rights Reserved.
1898
T Cell Costimulatory Pathways:
Emerging Concepts
Negative and positive signaling pathways
Complex interactions among, and between,
the positive and negati ve pathways
Regulation of naive and effector/memory cells
Molecules expressed on bone marrow derived
cells and parenchymal cells
Rothstein & Sayegh, Imm. Rev 2003
APC
Signal 2
Signal 1
Proliferation
Cytokine production
Prevention of anergy
Thelper cell
differentiation ACTIVATION
APC
Signal 1
INHIBITION
-
Anergy;
Inhibition of
proliferation and
cytokine
production
Apoptosis
? Induction of
regulatory cells
Signal 2
POSITIVE COSTIMULATORY PATHWAYS
NEGATIVE COSTIMULATORY PATHWAYS
+
Clarkson and Sayegh Tx 2005
Copyright Harvard Medical School, 2010. All Rights Reserved.
1899
T cell
Antigen Presenting Cell
CD28
superfamil y
TNF-TNF-R
superfamil y
PD-L1 L2
PD-1
CD28 CTLA4
ICO
S
TC
R
-
+
+
-
+
CD80
CD86
B7RP1
MHC-
peptide
4-1BBL
CD27(CD70)
CD134
L
CD30L
CD70
(CD27
)
+
4-1BB
+
CD30
+
CD13
4
+
?
?
CD40
CD154
+
*
*
*
*
*
*
*
*
APCs T cell Parenchymal Cells
B7-1
B7-2
CD70/CD27
B7-H1
B7-DC
B7-H1 *PD-1
CD28
*CD70/CD27
*CD154
*ICOS
*4-1BB
ICOS-L
B7h
*CD134
4-1BBL
CD40 CD40
CD134L CD134L
TRC
*CTLA4
B7-H3
B7-H4
B7-H3
B7-H4
BTLA/HVEM
?
?
BTLA/HVEM
MHC/Peptide
* Inducible Receptors
Copyright Harvard Medical School, 2010. All Rights Reserved.
1900
New T Cell Costimulatory Pathways
Redundancy insures continuation of immune
responses
Each pathway may have additional unique functions
based on specific cell type (CD4/CD8/NK), phenotypic
differentiation (Th1/Th2), and/or timing of the immune
response (nave, effector, memory)
There are complex interactions among the various
costimulatory pathways
Interactions between positive and negative
costimulatory pathways determine the ultimate fate of
the immune response
Ultimately, multiple pathways may have to be targeted
with blockade of positive and signaling of negative
costimulatory molecules to achieve tolerance
Rothstein & Sayegh, Imm. Rev 2003
De novo
purine synthesis
IL-2
IL-2 receptor
T cell
Ca
2+
P
NFAT
NFAT
Activated
Calcineurin
IL-2 GENE PROMOTOR
+
Steroid
G1
G2
S
M
Rapamycin
FK506 CsA
MMF
Cell
Cycle
Aza
APC
Antigenic
Signal
Costimulatory
Signal
TCR
IL-2-R mAb
OKT3 mAb
Polyclonals
Copyright Harvard Medical School, 2010. All Rights Reserved.
1901
TH1/TH2 PARADIGM
Th0 Cell
Th1 Cell Th2 Cell
IFN g
IL-2
IL-4
IL-5
IL-10
IL-13
IL-4 IL-12
Cell Mediated
Immunity
Humoral
Immunity
TH1/TH2 PARADIGM
Th0 Cell
Th1 Cell Th2 Cell
IFN g
IL-2
IL-4
IL-5
IL-10
IL-13
TCR Signal/Antigen Density/Clone Size
Cytokine Milieu
Costimulatory Signal
? Rejection
? Tolerance
Copyright Harvard Medical School, 2010. All Rights Reserved.
1902
CHALLENGES TO THE TH1/TH2
PARADIGM
Th1 cytoki ne knockout recipients can reject
allografts
Th1 cytoki ne knockout recipients show
resistance to tolerance
Th2 cytoki ne knockout recipients can be
tolerized in some models
Th2 cytoki nes permissive but do not cause
tolerance in fully MHC mismatched models
T cell proliferation
CTL generation
DTH response
Alloantibody
NK cell activation
Afferent Phase Efferent Phase Regulatory Phase
MHC TCR
APC T cell
AICD (apoptosis)
Proliferation arrest
Anergy
DUAL ROLE OF TH1 CYTOKINES IN
GRAFT REJECTION
IL-2
IFN-g
essential redundant
Lakkis et al.
Copyright Harvard Medical School, 2010. All Rights Reserved.
1903
STEP THREE
EFFECTOR MECHANISMS
Copyright Harvard Medical School, 2010. All Rights Reserved.
1904
ALLOGRAFT REJECTION
Indirect
Direct
Cytokine Production
Clonal Expansion
CD4
+
T cell
Allo APC
MHC
Class II
TCR
CD4
+
T cell
Self APC
MHC
Class II
TCR
Activated
Macrophage
DTH PEPTIDE DERIVED
FROM ALLOANTIGENS
Activated
B cell
Alloantibodies
Perforin
Granzyme
CD8
+
T cell
Graft Cell
FAS
FAS L
LMC
Apoptosis
CTL CTL Target Cell
Gz
Gz
Pf
Gz
Pf
Gz
Fas FasL
Non-Secretory
Receptor Mediated
Secretory
Perforin + Granzyme
Mediated
CTL-Mediated Lymphocytotoxicity
Copyright Harvard Medical School, 2010. All Rights Reserved.
1905
Li, B. et al. N Engl J Med 2001;344:947-954
Levels of Cytotoxic Molecules mRNA in
Urinary Cells
APCs T Cell
B cell
Th2
Th1
IL-2
IL-4
IL-4, IL5, IL-6
IL-10, IL-13
IFN-g
IgG1
IgG2
IgG1
IgG4
IgA
IgE
B7/ CD28
B7/ CD28
MHC/pepti de
TCR
MHC/pepti de
TCR
Antibodies
Humoral Responses
Copyright Harvard Medical School, 2010. All Rights Reserved.
1906
ALLOREACTIVE ANTIBODIES
Antibodies recognize polymorphi c (exposed)
regions of MHC molecules
May develop upon exposure to alloantigens
through pregnancy, blood transfusion,
previous transplant or by cross reactivi ty
Cross matches and PRA are used to detect
alloantibodies that are present pre-transplant
(prevent hyperacute rejection)
Alloantibodi es can also devel op post
transplant (contribute to acute and chronic
rejection)
Humoral Alloreactivity
historic current de novo
time
Pre Tx
Preformed Abs
Post Tx
De novo Abs
Copyright Harvard Medical School, 2010. All Rights Reserved.
1907
C4b + C4a C4
C1
Donor-HLA
Anti-HLA-Ab
Humoral Rejection Cellular Rejection
PMN
M
T-cell
T-cell
T-cell
T-cell
T-cell
T-cell
C4d
C4
d
MAC
C2a
C3a + C3b
A Shifting Spectrum
DTH Response
Mediated by CD4+ T cells and monocytes/
macrophages
Endothelial cell role: chemokines, adhesion,
migration, promotion of direct and indirect
allorecogniti on
Starts as an antigen-speci fic immune
response but ends with tissue injury and
repair
Role of apoptosis ?
Copyright Harvard Medical School, 2010. All Rights Reserved.
1908
STEP FOUR
RESOLUTION OF THE RESPONSE
WITH RESIDUAL MEMORY
Figure 1
Fate of T Cells Determines
Immune Response
Signal One
Signal Two
T Cells
Proliferation
Differentiation
Nave T Cell APC
Effector cells
(AR, CR)
Memory cells
(?CR)
Termination (Tolerance):
-Anergy (CTLA4)
-Cell death (PCD, AICD)
-Regulation Najafian, Khoury and Sayegh, 2001
+
-
Copyright Harvard Medical School, 2010. All Rights Reserved.
1909
FOXP3 mRNA Levels are Higher in Patients with
Successfully Reversed Acute Rejection Episode
0
4
8
12
P=0.22
L
o
g
C
D
2
5
m
R
N
A
0
4
8
12
P=0.22
L
o
g
C
D
2
5
m
R
N
A
2
6
10
14
P=0.48
Reversi bl e Non-reversi bl e
Acute Rejection Episode
N=26 N=10
L
o
g
G
r
a
n
z
y
m
e
B
m
R
N
A
2
6
10
14
P=0.43
Reversi bl e Non-reversi bl e
Acute Rejection Episode
N=26 N=10
L
o
g
P
e
r
f
o
r
in
m
R
N
A
-2
2
6
10
Reversible Non-reversible
Acute Rejection Episode
(N=26) (N=10)
P=0.0009
L
o
g

F
O
X
P
3

m
R
N
A
Urine specimens were collected from 36 patients with acute rejection. Twenty six of the 36 acute rejections
qualified as successful reversal and the remaining 10 did not. Urinary cell Foxp3 mRNA was significantly
higher in the group with successful reversal than in the group with out reversal (meanSE: 4.70.5 vs. 1.50.7,
P=0.0009). The levels of CD25 (7.30.4 vs. 6.00.9, P=0.23), CD3 (8.50.5 vs. 7.40.8, P=0.35), and perforin
(7.80.5 vs. 7.30.7, P=0.43) were not informative of acute rejection outcome .
CD4
+
CD25
+
- Mediated
Regulation of the Indirect Alloresponse
<60 spots per million cells
53%
47%
Stable
Patients
60 spots per million cells
P =0.018
High Risk
Patients
Regulation No Regulation
Sal ama et al
JASN, 2003
Copyright Harvard Medical School, 2010. All Rights Reserved.
1910
and what about tolerant humans?
Tolerant
Non
Tolerant
Healthy
Controls
0%
10%
20%
NS
Adult cadaveric liver Tx
0%
0,5%
1%
1,5%
2%
2,5%
Tolerant Non
Tolerant
Healthy
controls
Pediatric living donor liver Tx
Potential role of CD4+CD25+ regulatory T cells in tolerance to
liver allografts in humans
Y. Li et al. Am J Transplant 2004 M.Martnez-Llordella, submitted
*
*
*
* p <0,05
Regulatory Cells and
Suppressor Factors
Effector T cell clone size
Rejection Tolerance
Altering The Balance Of Alloreactivity
How to tolerize a T cell?
-Inactivate (anergy)
-Delete (apoptosis)
-Regulate (cells/factors)
Strom and Turka
Copyright Harvard Medical School, 2010. All Rights Reserved.
1911
QUESTION 1
All the statements regarding Human
Leukocyte Antigens (HLA) are correct
except:
a) HLA system is devidedinto 2 classes, class I and
class II
b) HLA genes are located on the short arm of
chromosome 6
c) HLA antigens are inherited in a mendelian fashion as
codominant alleles
d) HLA-A, HLA-B, HLA DR most important in clinical
transplantation
e) Class II is predominantly expressed by APC, B cells
and T cells
QUESTION 2
The following statements about Belatacept
(LEA29Y) are correct except:
a) Is a fusion protein that combines CTLA4 with Fc
portion of human IgG1
b) Blocks the interaction of B7-CD28 costimulatory
molecules
c) Differs from abatacept only by 2 aminoacids
d) Leads to 10 fold less inhibition of T cell activation
as compared to abatacept in vitro
e) as effective an immunosuppressant as the
calcineurin inhibitor cyclosporine in preventing
acute rejection and is associated with better
kidney function at 6 month
Copyright Harvard Medical School, 2010. All Rights Reserved.
1912
From Abatacept to Belatacept (LEA29Y)
Rational design of a drug
Belatacept (LEA29Y)
CTLA4I
L104E (CDR3)
provides more potent immunosuppression required for transplantation
Leucine
104
GlutamatE
Alanine
29
TYrosine
CTLA4I
g
Mutagenesis
2,300 mutants
L104E (CDR3)
mutant
identified
Repeated with
Mutagenesis
Larsen CP et al. Am J Trans 2005;5:443453
4fold slower
dissociation from
CD86
2fold slower
dissociation from
CD80
screened
+
~10-fold more potent
inhibition of T-cell
activation In vitro
=
Results:
L104E mutant
Disclosures:
None None
Copyright Harvard Medical School, 2010. All Rights Reserved.
1913
Transplant
Immunosuppression
Steven Gabardi, PharmD, BCPS
Abdomi nal Organ Transpl ant Speci al i st
Bri gham and Womens Hospi tal
Instructor i n Medi ci ne
Harvard Medi cal School
Disclosures
NONE
Copyright Harvard Medical School, 2010. All Rights Reserved.
1914
Objectives
Outline the principles of immunosuppression in
kidney transplant recipients
Classify and describe the mechanism of action of
different immunosuppressive agents used in kidney
transplantation
Identify the different uses of transplantation
immunosuppression and the relative benefits and
drawbacks of differentimmunosuppressive agents
Discuss the treatment of acute rejection and
antibody-mediated rejection
Introduction
Advances in transplant immunosuppression
have contributed to:
A decrease in acute rejection and an increase in
graft survival
Longevity for kidney allograft recipients
Proliferation of agents means:
More options
More complicated management
Copyright Harvard Medical School, 2010. All Rights Reserved.
1915
Principles of Immunosuppression - 1
Elimination of acute rejection would be ideal
(if there were no side-effects)
Humar et al
1,2
- studied long-term outcome (rejection vs. no
rejection) of recipients transplanted 1991-2001
- immunosuppression cyclosporine, prednisone,
azathioprine
- excluded graft loss to technical failure, primary
nonfunction, death, recurrent disease
1. Humar A, et al. Transplantation 1999;68:1842-6
2. Arthur Matas, personal communication.
10-Year Death-Censored Allograft Survival Rates
Humar A, et al. Transplantation 1999;68:1842-6
Arthur Matas, personal communication.
p<.001
%

G
r
a
f
t

S
u
r
v
i
v
a
l
Deceased Donor
0
20
40
60
80
100
0 1 2 3 4 5 6 7 8 9 10
Time Posttranspl ant (years)
Group 1 (acute rej ecti on)
n=139
Group 2 (No acute rej ecti on)
n=356
p<.001
Living Donor
0
20
40
60
80
100
0 1 2 3 4 5 6 7 8 9 10
Time Posttranspl ant (years)
%

G
r
a
f
t

S
u
r
v
i
v
a
l
Group 1 (acute rej ecti on)
n=247
Group 2 (No acute rej ecti on)
n=700
Patients were excluded from the analysis if they experienced death with graft
function, primary non-function, technical failure, or recurrent disease.
Copyright Harvard Medical School, 2010. All Rights Reserved.
1916
Actual costs
multiple lab tests (for diagnosis)
return to the transplant center
kidney biopsy
cost of treatment
Long-term costs
increase rate of CAD; decreased allograft survival
increased risk of infection and malignancy (from
anti-rejection treatment)
other side-effects from anti-rejection treatment
Costs of Acute Rejection
Principles of Immunosuppression - 2
Maintenance of excellent kidney function is
important
Meier-Kriesche et al*
- analyzed 58,900 adult US patients who received a primary renal
transplant between 1988 and 1998 and who had at least 1 year
of allograft survival. The primary study endpoint was death from
a cardiovascular event beyond 1 year of transplantation.
- Serum creatinine values at 1 year after transplantation were
strongly associated with the risk for cardiovascular death. Above
a serum creatinine value of 1.5 mg/dL, there was a significant
and progressive increase in the risk for cardiovascular death.
*Meier-Kriesche HU, et al. Transplantation 2003;75:1291-5
Copyright Harvard Medical School, 2010. All Rights Reserved.
1917
Cardiovascular Death and Graft Function
Meier-Kriesche HU, et al. Transplantation 2003;75:1291-5
Principles of Immunosuppression - 3
a) Among transplant recipients, there are
continua of risk - e.g., for acute rejection
episodes, drug side-effects
b) There are also immunosuppressive agents
with greater vs. lesser efficacy and greater
vs. lesser risks (side-effects)
Lowest Risk: Identical twin, BMT Highest Risk: Highly sensitized,
Expanded-criteria
donor
Copyright Harvard Medical School, 2010. All Rights Reserved.
1918
Goals for Achieving the Best Long-
Term Outcome
Minimize acute rejection (AR)
Minimize chronic rejection/chronic
allograft nephropathy: minimize AR, CNI
nephrotoxicity
Minimize cardiovascular risk factors
Minimize other drug-related side-effects
CNI =calcineurin inhibitor
Classification of Immunosuppressi ve Agents
1) Induction agents*
powerful drugs used for a short time at the time
of transplant
2) Maintenance agents*
drugs given long-term (indefinitely) to prevent
rejection
3) Anti-rejection agents*
drugs given to reverse a rejection episode
4) Drugs used in special situations
e.g., ABO incompatible transplants or transplants
across a positive crossmatch
* Agents used in different settings may be the same.
Copyright Harvard Medical School, 2010. All Rights Reserved.
1919
Immunosuppressant Action
and the Immune Cascade
EC-MPA
Basi l i xi mab
ATG
EVL
Al emtuzumab
CD52
Receptor
Critical Questions
1) Should we give the same immunosuppression to all
recipients (one size fits all)?
or, if not
2) Who should receive antibody induction?
-All?
-Selected high risk groups?
-Drug minimization trials?
3) Who should receive aggressive (vs. conservative)
maintenance immunosuppression?
Copyright Harvard Medical School, 2010. All Rights Reserved.
1920
Introduction of Immunosuppressants (US)
Adapted from Zand MS. Semin Dial. 2005;18:511-9
60 65 70 75 80 85 90 95 00 05
Year
0
20
40
60
80
100
P
e
r
c
e
n
t
Radiation
Prednisone
6-MP
Azathioprine
ATGAM
Cyclosporine
OKT3
Cyclosporine Microemulsion
Tacrolimus
Mycophenolate mofetil
Daclizumab
Basiliximab
Thymoglobulin
Sirolimus
Rituximab
Alemtuzumab
Leflunomide
Rejection <12 months
1-year survival
I - Induction
Goals of induction therapy
1. To decrease the rate of acute rejection
2. To permit delayed initiation, minimization or avoidance of some
of the maintenance agents (i.e. CNI, corticosteroids)
Available agents
1) Monoclonal antibodies that react with a single antigen receptor
on the lymphocyte
Basiliximab (Simulect)
Alemtuzumab (Campath)
Recently withdrawn from the market: muromonab-CD3 (OKT3) and
daclizumab (Zenapax)
2) Polyclonal antibodies: react with multiple antigen receptors
Equine polyclonal IgG antibody (ATGAM)
Rabbit polyclonal IgG antibody (Thymoglobulin)
Copyright Harvard Medical School, 2010. All Rights Reserved.
1921
I - Induction
Pharmacologic Classification of the
Induction Agents
1. Non-Depleting Proteins
Basiliximab(Simulect

)
1. Depleting Proteins
Equine antithymocyteglobulin (ATGAM

)
Rabbit antithymocyteglobulin (Thymoglobulin

)
Alemtuzumab(Campath

)
Non-Depleting Proteins: Basiliximab
A chimeric monoclonal antibody that competitively
inhibits the activation of lymphocytes by IL-2
This agent has low immunogenicity potential
because of the incorporation of human protein
sequences.
Human
Mouse
Copyright Harvard Medical School, 2010. All Rights Reserved.
1922
Non-Depleting Proteins: Basiliximab
Dosing:
Loading dose: 20mg ~2hrs prior to transplantation.
Maintenance dose: 20mg dose 4 days post-op.
After this dosing regimen the agent stays bound to
the CD25 receptor for up to 10 weeks
Drug-Drug Interactions (DDI): none reported
Adverse Drug Reactions (ADR): incidence of
ADR in clinical trials were similar to those seen
with placebo.
Depleting Proteins: Antithymocyte
Globulins (Horse and Rabbit)
These agents are purified gamma globulin
obtained by immunizing horses/rabbits w/
human lymphocytes.
Cytotoxic antibodies directed against a broad array of
surface antigens expressed on T- and B-lymphocytes:
CD2, CD3, CD4, CD8, CD11a, CD25, CD28, CD45, Human
Leukocyte Antigen (HLA) Class I and HLA-DR subsets
Causes depletion of peripheral lymphocytes
complement-dependent lysis (primary response)
clearance by the RES
Cellular reconstitution may take up to 3 months
Copyright Harvard Medical School, 2010. All Rights Reserved.
1923
Depleting Proteins: Antithymocyte
Globulins (Horse and Rabbit)
Cytopl asm
CD28
PI-3K
CD3
TCR
CD4
p56
lck
IL-2R
Anti-CD28
Anti-CD3
Anti-TcR
Anti-CD4
Anti-IL-2R
What is the difference between the two agents?
Antithymocyte globulin rabbit (r-ATG) not only causes cell depletion, but
it also has some secondary mechanisms:
immune modulation,
B-cell apoptosis,
actions on adhesion molecules,
dendritic cell depletion
Depleting Proteins: Antithymocyte Globulin
Rabbit
Dosing:
Most common regimens utilize 1.5 mg/kg/day for 3 - 5
days
Ideally first dose is given prior to graft reperfusion
1
DDI: none reported
ADR:
myelosuppression (leukopenia)
cytokine release syndrome (myalgias, hypotension,
tachycardia, fever, etc.)
RARE: serum sickness
1. Goggins WC, et al. Transplantation 2003;76:798-802
Copyright Harvard Medical School, 2010. All Rights Reserved.
1924
Depleting Proteins: Alemtuzumab
Alemtuzumab is an anti-CD52 humanized,
monoclonal antibody that has an FDA indication
for use in B-cell chronic lymphocytic leukemia.
CD52 is present on virtually all B- and T-cells, as
well as macrophages, NK cells and some
granulocytes.
The alemtuzumab-CD52 complex triggers
antibody-dependent lysis.
The depletion of lymphocytes is so marked that it takes
several months, up to one-year, post-administration for
a patients immune system to be fully reconstituted.
Depleting Proteins: Alemtuzumab
Dosing:
30 mg, given as a single dose intraoperatively
In some settings, a second 30 mg dose is given.
DDI: none reported
ADR:
Myelosuppression
Infusion-related reactions: nausea (54%), vomiting
(41%), diarrhea (22%), headache (24%),
dysthesias (15%) and dizziness (12%)
RARE: autoimmune hemolytic anemia
Copyright Harvard Medical School, 2010. All Rights Reserved.
1925
e-ATG vs. r-ATG
1-3
Efficacy: Significantly lower rates of BPAR and improved
allograft/patient survival with r-ATG at 1-, 5- and 10-years
Safety: Significantly lower rates of CMV infection with r-ATG, despite
higher early rates of leukopenia. Similar rates of PTLD.
Basiliximab vs. r-ATG (high risk recipients)
4
Efficacy: Similar composite end point of BPAR, DGF, allograft/patient
survival
Significantly lower rates of BPAR associated with r-ATG
Safety: Significantly higher rates of myelousppressionand overall
infections seen with r-ATG
Significantly fewer cases of CMV infection seen with r-ATG, despite higher early
rates of leukopenia.
Alemtuzumabvs. r-ATG (steroid withdrawal)
5
Efficacy: Similar rates of BPAR, allograft/patient survival, infectious
complications or development of post-transplant diabetes.
Comparative Analyses of Induction
Therapies
1. Brennan DC, et al. Transplantation 1999;67:1011-8. 2. Hardinger KL, et al. Transplantation 2004;78:136-41.
3. Hardinger KL, et al. Transplantation 2008;86:947-52. 4. Brennan DC, et al. N Engl J Med 2006;355:1967-77.
5. Ciancio G, et al. Transplantation 2005;80:457-65.
Biologic Agent Overview
Clinical Use
Agent Class
Induction Maintenance Treatment
Cost
Basiliximab Anti-CD25
Antibodies
(non-
depleting)
20 mg =
$2000
Alemtuzumab Anti-CD52
Antibody
(depleting)
30 mg =
$1700
e-ATG/r-ATG Polyclonal
Antibodies
(depleting)
1.5 mg/kg
x 70 kg =
$1600
Copyright Harvard Medical School, 2010. All Rights Reserved.
1926
Current Trends in Antibody Induction Use in Kidney
Transplantation
Andreoni KA, et al. Am J Transplant 2007; 7:1359-75
0
10
20
30
40
1996 1997 1998 1999 2000 2001 2002 2003 2004 2005
ATG Thymo OKT3 Zenapax Si mul ect Campath
Year
T
r
a
n
s
p
l
a
n
t
s

(
%
)
Calcineurin Inhibitors (CNI)
Cyclosporine (Sandimmune

/ Neoral

)
Tacrolimus (Prograf

)
Inhibitors of T-cell Proliferation
Azathioprine (Imuran

)
Mycophenolate mofetil (CellCept

)
Enteric-Coated Mycophenolic Acid (Myfortic

)
Mammalian Target of Rapamycin (mToR) Inhibitors
Sirolimus (Rapamune

)
Everolimus (Zortress

)
Non-specific immunosuppressants
Corticosteroids
II Maintenance Immunosuppression
Copyright Harvard Medical School, 2010. All Rights Reserved.
1927
Common Combinations of Immunosuppressive
Regimens
Pri mary
agent
Thi rd agent Second
agent
CsA
TAC
Si rol i mus
Everol i mus
AZA
MMF
EC-MPA
Si rol i mus
Everol i mus
Predni sone
AZA = azathioprine, CSA = cyclosporine, EC-MPA=enteric-coated mycophenolate sodium,
MMF = mycophenolate mofetil, TAC = tacrolimus
Common Combinations of Immunosuppressi ve
Regimens
Dual- or triple-therapy (most common)
Benefits: Lower doses of each drug; maximize efficacy
( acute rejection episodes) while
minimizing toxicity of each drug
Risks: Overimmunosuppression
Increased costs
Monotherapy
Benefits: Potential for limiting side-effects of multiple
drugs
Risks: Increased rejection
Increased doses (levels) of the single drug are
needed side-effects
Copyright Harvard Medical School, 2010. All Rights Reserved.
1928
Achievement and Maintenance of
Optimal Immunosuppression
Not just the choice of drugs but how they are
used; blood levels are important for some
Early posttransplant protocols may differ from late:
a) dose reduction over time
b) minimization/elimination of one drug
c) unexpected events may call for changes
d) flexibility is important
Understanding the Importance of
Calcineurin Phosphatase
Nucleus
Cytopl asm
Cytokine
Gene Promoter
IL-2
IL-4
IFN-
TNF-
PPP
NFAT
CNA
CNB
Calmodulin
Ca
2+
Kinase
Cascade Integration
Transcription Factor
Activation
Calcineurin
ZAP70
CD28
PI-3K
CD3
TCR
PLC
fyn
CD4
p56
lck
IL-2R
Transcription
Translation
Copyright Harvard Medical School, 2010. All Rights Reserved.
1929
PPP
Nucleus
Cytopl asm
Cytokine
Gene Promoter
IL-2
IL-4
IFN-
TNF-
PPP
NFAT
CNA CNB Calmodulin
Ca
2+
CD28
PI-3K
CD3
TCR
PLC
fyn
CD4
p56
lck
IL-2R
Transcription
Translation
Down-Regulati on
CsA
Cyclophilin
Understanding CsA and TAC
TAC
FKBP-12
Calcineurin Inhibitors: CsA and TAC
Numerous studies have compared efficacy and side-effects.
in general (though there is some variation between studies):
Efficacy
Patient and allograft survival has been similar for the 2 CNIs
Acute rejection rates have been similar or with TAC
Safety/tolerability
CsA has been associated with greater increases in lipid levels
and blood pressure
TAC has been associated with greater incidence of new-onset
posttransplantdiabetes
Each has drug-specific side-effects
Copyright Harvard Medical School, 2010. All Rights Reserved.
1930
Dosing:
Due to significant inter- and intra-patient variability,
therapeutic drug monitoring (TDM) is employed to
maximize the efficacy of both CNIs.
CsA: overall exposure is best correlated to C
2
or C
3
levels
TAC: overall exposure is best correlated to C
12
levels
Appropriate levels are dependent on institution-
specific protocols and concomitant
immunosuppressants
DDI: both agents are substrates for CYP3A4 and
P-gp
Calcineurin Inhibitors: CsA and TAC
CNI Side-Effects
Cardiovascular Hypertension; Hypercholesterolemia
Glucose intolerance
Neurotoxicity Tremor; Headache; Insomnia;
Paresthesia
Nephrotoxicity perhaps long-term dose and level-
related
Hepatotoxicity
Malignancy related to overall immunosuppression
Physical Gingival Hypertrophy; Hirsutism(CSA);
Alopecia (TAC)
Copyright Harvard Medical School, 2010. All Rights Reserved.
1931
Trends in CNI Use
0
0.1
0.2
0.3
0.4
0.5
0.6
0.7
0.8
0.9
1995 1996 1997 1998 1999 2000 2001 2002 2003 2004 2005
Cycl ospori ne Tacrol i mus
Meier-Kriesche HU, et al, Am J Transplant 2006;6:1111-31
Inhibit lymphocyte
proliferation
Inhibitors of purine and
DNA synthesis
cells other than lymphocytes
have an alternate synthesis
pathway
thus, selective
antiproliferative effect on T
and B cells
MPA
Guanosine
monophosphate
Deoxyguanosine
diphosphate
Deoxyguanosine
triphosphate
DNA
Guanosine
triphosphate
RNA
Clonal proliferation of T- and B-cells
Inosine
monophosphate
IMPDH
Salvage
pathway
can be used
by other cells
Leads to
proliferation of
other cells
Mycophenolic Acid (MPA): MMF and EC-MPA
Copyright Harvard Medical School, 2010. All Rights Reserved.
1932
MPA: MMF and EC-MPA
Dosing:
MMF: 1 gm BID
EC-MPA: 720 mg BID
TDM is not routinely recommended
DDI: divalent/trivalent cation-containing antacids and
supplements, cyclosporine
MMF only appears to interact with proton pump inhibitors
1
ADR:
Myelosuppression
Gastrointestinal disorders, both upper and lower GI tract
Associated with significant teratogenic effects
1. Ruppercht K, et al. J Clin Pharmacol 2010;49:1196-201
AZA
Allograft
Azathioprine
6-mercaptopurine
Thioinosinic acid
(TIMP)
T
Allograft
BM
B
Promyelocyte
Anemi a
B, T Cel l
Prol i ferati on Monocytopeni a
Thrombocytopeni a
Purine synthesis
Thioguanylic acid
Thiodeoxyguanosine
triphosphate
No DNA
Synthesis
AZA is a prodrugof 6-MP.
6-MP is incorporated into DNA where it inhibits purine
synthesis and prevents the formation of RNA.
Inhibits gene replication and subsequent activation of T-cells.
Copyright Harvard Medical School, 2010. All Rights Reserved.
1933
Dosing: 2 - 5 mg/kg/day to start, titrated to
hematological effects (maintenance dose may
be between 1 and 3 mg/kg/day).
DDI: allopurinol, febuxistat
ADR:
Myelosuppression
RARE: pancreatitis
AZA
Both agents binds to
FKBP, but do not inhibit
calcineurinphosphatase
The sirolimus or
everolimus/ FKBP
complex inhibits mToR,
a driver of cell
proliferation
This inhibition results in a
reduction in IL-2 driven
lymphocyte proliferation
Everolimus is a
derivative of Sirolimus
with slightly different PK
parameters.
mToR Inhibitors: Sirolimus and
Everolimus
IL-2
IL-2
Receptor
PTK
PI3-K
mTOR
p70S6k
S6
Si rol i mus /
Everol i mus
FKBP-12
INHIBITION OF T CELL
RESPONSE TO
CYTOKINES
INHIBITS PROLIFERATION
AT THE G1 TO S PHASE
Copyright Harvard Medical School, 2010. All Rights Reserved.
1934
Molecular Inhibition of G1 S Progression
Consequences
Biological Down-Regulation of Proliferation
Consequences
Cellular T Cells B Cells Smooth Endothelial
Targets Muscle Cells
Inci dence of Acute Rej ecti on Epi sodes
Ri sk of Chroni c Al l ograft Nephropathy
Requi rement for Cal ci neuri n Inhi bi tors
Wound Heal i ng / Anti-Cancer Effects
T Cell-Mediated
Humoral?
Clinical
Consequences
Understanding the impact of the mToR
Inhibitors
Dosing:
TDM (C
12
levels) is employed to maximize the efficacy of both mToR
inhibitors
Appropriate levels are dependent on institution-specific protocols and
concomitant immunosuppressants
Common Doses
Sirolimus: 1 2 mg QD
Everolimus: 0.75 mg BID
DDI: both agents are substrates for CYP3A4 and P-gp
ADR:
Cardiovascular (hypercholesterolemia, hypertriglyceridemia)
Myelosuppression
Dermatologic (rash, mouth ulcers)
Musculoskeletal (myalgias, muscle weakness)
Interstitial pneumonitis
Renal (proteinuria)
Hepatotoxicity
Decreased wound healing
mToR Inhibitors: Sirolimus and
Everolimus
Copyright Harvard Medical School, 2010. All Rights Reserved.
1935
2005 OPTN/SRTR Annual Report
1
1. In Meier-Kreische HU, et al. Am J Transplant 2006;6:1111-31
OPTN/SRTR=United States Organ Procurement and Transplantation
Network/Scientific Registry of Transplant Recipients
Corticosteroids
The exact MOA is still not fully understood. Some
believe
High dose: >100 mg of prednisone equivalents.
MOA =directly toxic to T cells
Low dose: <100 mg of prednisone equivalents.
nonspecific immunosuppressive agents - inhibit IL-1, IL-2, IL-3,
IL-6, IL-15, TNF-alpha and INF-gamma at low doses.
Decreased activation of T cells.
What we do know:
Blockade of Cytokine Gene Expression
T-cell and APC cytokine expression
Bind to heat shock protein translocates to nucleus binds to GRE
inhibits transcription of cytokine genes inhibition of IL-1, IL-2, IL-3,
IL-6, INF-, and TNF-
cytokine-receptor expression
Nonspecific Effects
Antiinflammatory effects
Copyright Harvard Medical School, 2010. All Rights Reserved.
1936
Corticosteroids
Dosing: doses vary widely from institution to institution.
Highest doses at time of transplant or as treatment of an acute
rejection episode.
DDI: CYP - 450 inducer (dexamethasone) and inhibitor
(methylprednisolone).
ADR:
Cardiovascular (hypertension, hyperlipidemia)
Endocrine (hyperglycemia)
CNS (mood changes, anxiety)
Osteoporosis
Weight gain
Edema
Lipodistrophy
Relative Side-Effect Profile
CSA TAC mToR Pred MPA
Hypertension ++ + ++
Renal dysfunction +++ ++ +
Hyperlipidemia ++ + +++ ++
Glucose intolerance + ++ ++
Hyperkalemia +++ +++
GI side-effects + + + ++
Tremor + ++
Malignancy + + ? ?
Osteoporosis ++
Hirsutism + +
Gingival hypertrophy +
Alopecia +
Copyright Harvard Medical School, 2010. All Rights Reserved.
1937
Treatment of Acute Rejection
The biggest changes in kidney transplantation since the
turn of the 21
st
century have been in the understanding
of acute rejection.
Historically, antibodies were thought to attack the
allograft immediately (hyperacute rejection) or not be
particularly important. The long-term emphasis has
been on cellular-mediated rejection and its
consequences.
Recent data (and development of new techniques in
pathology) have shown that antibody-mediated rejection
can play an important role both early and late
posttransplant.
Acute Cellular-Mediated Rejection
Acute cellular rejection is most common during the first 6
months posttransplant
Becomes substantially less common over time
Most episodes are not accompanied by symptoms but
present as serum creatininelevel
The transplant center should be notified immediately if
acute cellular rejection is suspected
Because treatment is associated with significant side-
effects, most suspected acute rejection episodes are
biopsied to confirm the diagnosis
Copyright Harvard Medical School, 2010. All Rights Reserved.
1938
Treating Acute Cellular Rejection
Agent Dosage
Pulse
corticosteroids
1
Intravenous methylprednisolone 125
mg to 1 g or 3 to 5 mg/kg body weight
daily for 3 to 5 days
Antithymocyte
globulin, rabbit
(Thymoglobulin)
2
1.5 mg/kg in 500 mL of dextrose or
saline infused over 4 to 8 hours for 4
to 14 days
*Initiation of therapy often requires hospitalization.
1. Pescovitz MD, et al. Am J Transplant 2003;3:1581-6.
2. Thymoglobulin prescribing information. rev. April 2002.
Antibody-Mediated Rejection (AMR)
A significant form of rejection
not amenable to standard
immunosuppressive therapy
aimed at modifying T-cell
function
Acute antibody-mediated
rejection tends to be seen
early posttransplant
Diagnosis is made by kidney
biopsy and special stains
C4d Stai ni ng i n Peri tubul ar Capi l l ari es
Terasaki P, Mizutani K. Clin J Am Soc Nephrol. 2006;1:400-3.
Racusen LC, Haas M. Clin J Am Soc Nephrol. 2006;1:415-20.
Copyright Harvard Medical School, 2010. All Rights Reserved.
1939
1. Becker YT, et al. Am J Transplant 2004;4:996-1001.
2. Alausa M, et al. Clin Transplant 2005;19:137-40.
3. Montgomery R, et al. Am J Transplant 2004;4:1011-2.
Antibody- and cellular-mediated rejection can occur
independently or co-exist
When antibody-mediated rejection is independent,
treatment options include:
1-3
High-dose IVIg
Rituximab(Rituxan

)
Bortezomib(Velcade

)
Eculizumab(Soliris

)
Plasmapheresis
Plasmapheresis and low-dose IVIg
When antibody-mediated and cellular-mediated rejection
coexist, therapy for each must be utilized
Treatment of AMR
Intravenous Immune Globulin
Indication: treatment of primary
immunodeficiency syndrome.
Copyright Harvard Medical School, 2010. All Rights Reserved.
1940
Adverse Events: infusion-related reactions (i.e.,
hypotension, shaking, chills, wheezing, flushing,
nausea, anxiety, chest tightness, back pain,
hypertension)
Patients may require conversion to a different brand
of IVIG if they have continued infusion-related
reactions after decreasing the rate of administration.
Renal dysfunction may occur, especially when using
IVIG products that contain sucrose. Iso-osmotic and
Sucrose-free products are preferred.
Cost: $80 / gm
10 gm =$800.00
2 gm/kg (70 kg patient) =$11,200.00
Intravenous Immune Globulin
Rituximab
Indication: treatment of
B-cell non-Hodgkins lymphoma
CD-20 positive chronic lymphocytic leukemia
Moderately to severely-active rheumatoid arthritis
MOA in Transplantation: chimeric monoclonal
anti-CD20 antibody targeting B-cells
This directly inhibits B-cell proliferation and induces
cellular apoptosis through the binding of complement.
Complement, in turn, mediates antibody-dependent cell-
mediated cytotoxicityand subsequent cell death.
Copyright Harvard Medical School, 2010. All Rights Reserved.
1941
Adverse Events:
BBW: fatal infusion reactions, severe
mucocutaneous reactions (including SJ S) and
PML (also tumor lysis syndrome)
Cardiovascular (hypo- and hypertension, peripheral
edema)
CNS (dizziness)
Derm(urticaria)
Endocrine (hyperglycemia)
GI (diarrhea, vomiting)
Hematologic (leukopenia, anemia)
Muscular (back pain, myalgias, arthralgias)
Respiratory (bronchospasm, dyspnea, sinusitis)
Misc (rise in LDH)
Cost: $6 / mg
375 mg/m
2
(1.73 m
2
) =$3,892.50
1000 mg =$6,000.00
Rituximab
Bortezomib
Indication: treatment of
Multiple myeloma
Refractory mantel cell lymphoma
MOA in Transplantation: a proteasome
inhibitor that induces cell-cycle arrest and
apoptosis of plasma cells.
Copyright Harvard Medical School, 2010. All Rights Reserved.
1942
Adverse Events:
Cardiovascular (hypotension, peripheral edema)
CNS (fever, HA, insomnia, dizziness)
Derm(rash)
GI (nausea, vomiting, diarrhea, contipation)
Hematologic (leukopenia, anemia)
Muscular (weakness, back pain, myalgias)
Respiratory (dyspnea)
Cost: $370 / mg
1.3 mg/m
2
(1.73 m
2
) =$832.13
One cycle (4 doses) =$3,328.52
Bortezomib
Eculizumab
Indication: treatment of paroxysmal
nocturnal hemoglobinuria to reduce
hemolysis
MOA in Transplantation: a humanized
monoclonal antibody against complement
protein C5 that inhibits its cleavage to C5a
and C5b and preventing the generation of
the membrane attack complex (MAC)
Copyright Harvard Medical School, 2010. All Rights Reserved.
1943
Adverse Events:
CNS (HA, fatigue)
GI (nausea, vomiting, diarrhea)
Muscular (back pain, arthralgias)
Respiratory (nasopharygitis)
Cost: $6 / mg
600 mg =$3,600.00
Drug is currently classified as an orphan drug
and has a REMS protocol requiring
prescribing and dispensing restrictions
Eculizumab
Trends in Maintenance Immunosuppression
1) Rapid discontinuation of prednisone (RDP)
Corticosteroids (prednisone) are associated with numerous
side-effects that impair QoL and may reduce long-term
efficacy outcomes
Hence, pts and clinicians would like to reduce their use
Goal: minimization of prednisone-related side-effects
(without an increase in rejection or late graft loss)
Historically, late prednisone withdrawal has been
associated with increased rejection and worse long-term
allograft survival
However, current protocols stop prednisonewithin the first
week posttransplant, and randomized trials have shown
slight increase in early rejection but no difference in allograft
or patient survival
Copyright Harvard Medical School, 2010. All Rights Reserved.
1944
Rapid discontinuation of prednisone (cont.)
Nonrandomized trials have shown excellent 6-year
patient and allograft survival (but concerns remain
about longer-term outcomes)
RDP has been used successfully in low- and high-risk
recipients; adults and children; all ethnicities
Successful in minimizing prednisone-related side-
effects
~80% recipients remain prednisone-free long-term
Trends in Maintenance Immunosuppression
2) Calcineurin Inhibitor (CNI) Minimization
10-20% of extra-renal transplants (heart, lung,
liver) develop renal dysfunction; in many cases
this is attributed to CNI toxicity
CNI toxicity is also seen in some medium- and
long-term kidney allografts
Goal of CNI-minimization protocols is to eliminate
or minimize this nephrotoxicity
Trends in Maintenance Immunosuppression
Copyright Harvard Medical School, 2010. All Rights Reserved.
1945
CNI Minimization (cont.)
Three current experimental approaches:
Transplantation without CNI (eg, SRL, MMF, and
prednisone)
Initial use of CNI, but conversion to a CNI-free
protocol at a fixed time posttransplant
Conversion to a CNI-free protocol for those
recipients developing dysfunction and showing CNI
toxicity on biopsy
Trends in Maintenance Immunosuppression
Conclusions
Proliferation of new agents (plus improvement in
prevention and treatment of infection) has resulted in
significantly better short-term outcomes for kidney
transplant recipients.
Generic medication may make it more difficult to
accurately monitor our patients, but may provide
some financial benefits for drug acquisition.
Major focus of current clinical research is improving
long-term outcomes, in particular, increasing patient
and graft survival, and decreasing morbidity.
Copyright Harvard Medical School, 2010. All Rights Reserved.
1946
SIDE EFFECTS OF IMMUNOSUPPRESSSION THERAPY
Hypertension
Glucose intolerance
Bone disease
Infection
Hyperlipidemia
Cosmetic
Nephrotoxicity
Malignancy
Eye
disorders
Weight
gain
Years after transplant
1 5 10
100
75
50
25
0
CNI nephrotoxicity (%)
Prasad GV et al, Clin Transplant 2003: 17: 135 Nankivell BJ et al NEJM 2003; 349: 2326.
0
20
40
60
80
C
N
I
M
M
F
/
A
Z
A
P
r
e
d
n
is
o
n
e
CNI
MMF/AZA
Prednisone
Patient preference- discontinue:
CNIs
Steroids
Steroid Withdrawal/Avoidance:
Why?
Diabetes
Hyperlipidemia
Bone disease
Cataract formation
Weight gain
Patient quality-of-life (skin changes, edema,
neurological/psychological effects)
Midtvedt K, et al, JASN, 15, 2004, 3233
Opelz G et al, Am J Transplant 2005; 5: 720
Rogers CC et al, Transplantation, 80, 2005, 26
Vanrenterghem Y et al, Transplantation, 70, 2000, 1352
Copyright Harvard Medical School, 2010. All Rights Reserved.
1947
Months Posttranspl ant
Cumulative
incidence of
acute rejection
0.1
0.2
0.3
0.4
0.5
0.6
3 6 9 12 15
95% CI on 12 mo incidence
Ahsan N et al. Transplantation. 1999;68:1865-1874.
Withdrawal /Blacks
Maintenance/Blacks
Withdrawal /Nonblacks
Maintenance/Nonblacks
Study was ended prematurely due
to increased rejection (22% vs. 5%),
particularly in blacks
Late Prednisone Withdrawal at 3 months: unacceptable acute
rejection in kidney transplant recipients on CsA and MMF
Late Steroid Withdrawal is consistently
associated with increased risk of rejection
Kasiske B et al, JASN 11:1910-1917, 2000
Pooled data from two separate eras demonstrate increased acute
rejection when withdrawing prednisone >3 months after transplant
Meta-analysis 2000 Meta-analysis 2004
Pascual J et al, Transplantation 2004; 78: 1548
Copyright Harvard Medical School, 2010. All Rights Reserved.
1948
Early steroid withdrawal (within 7 days from
transplant) may provide acceptable outcomes
U of Minnesota Protocol, 1999-2005:
589 transplant patients, up to 5-year follow up of
prednisone-free maintenance immunosuppression
Rabbit anti-thymocyte globulin : 1.25 to 1.5 mg/kg x 5, first dose
intra-operatively
Steroids given for a total of 5 days:
Methylprednisone 500mg intraoperatively
prednisone 1mg/kg on POD#1
0.5 mg/kg on days 2 and 3
0.25 mg/kg on days 4 and 5
Maintenance immunosuppression: Cyclosporine or Tacrolimus,
MMF or SRL
Matas AJ, Am J Transplant 2005 (5): 2473
U Minnesota-early steroid withdrawal provides
excellent graft survival with low rate of complications
Matas et al , Am J Transpl ant 2005; 5: 2473
5-year graft survival =84%
1-year rejection-free survival =89%
12 months 60 months
CMV 8% 12%
Fractures 2% 6%
Osteonecrosis 0% 0.1%
Cataracts:
Nondiabetics 1% 3%
Diabetics 2% 9%
PTDM 1% 1%
PTLD 0.01% 1%
Non-PTLD
malignancy*
3% 12%
*24 skin,6 other
Compared to hi stori cal control s, si gni fi cantl y l ower rate of CMV
i nfecti on, PTDM, cataracts, AVN and fractures.
Copyright Harvard Medical School, 2010. All Rights Reserved.
1949
Early steroid withdrawal in kidney transplantation: other
strategies*
2-day steroid withdrawal: Methylprednisolone 250 mg d0, 125 d1.
Induction: Basiliximab20 mg (d0,4 x 2,)
Maintenance: Tacrolimus or CSA, and MMF or SRL.
Prospective trial of 300 patients with PRA<10%, 2d steroids vs.
steroid taper to 5 mg/d at 1 month.
Comparable graft survival, rejection rate, renal function with less weight
gain, PTDM
3-day steroid withdrawal: Methylprednisone 500 mg d0, 250 mg
d1, 125 mg d2.
Induction: Basiliximab20 mg (d0,4) x 2 vs. Campath30 mg x 1(d0),
Maintenance: MMF and Tacrolimus.
Retrospective comparison of 278 patients receiving 1st transplant:
Excellent graft survival, low rejection rates of ~15% with either induction
agent Kumar M et al, Transplantation 2006, 81: 832
Kaufman DB, Am J Transplant 2005, 5, 2539
*>100 treated subj ects, >2year fol l ow-up
Steroid use in solid organ transplant has declined
significantly
Organ At Discharge At 1 year
Kidney 77% 68%
Liver 80% 41%
Heart 92% 68%
Lung 97% 95%
H.-U. Meier-Kriesche et al, Am J
Transplant 2006; 6: 1111
0
5
10
15
20
25
30
35
2000 2001 2002 2003 2004
Deceased donor
Living donor
% steroi d-
free at
di scharge
Copyright Harvard Medical School, 2010. All Rights Reserved.
1950
A randomized, double-blind, placebo-controlled trial of early
corticosteroid cessation versus chronic corticosteroids: 4
year results
Woodl e et al , ATC 2007, Abstract #1704
0.5
0.6
0.7
0.8
0.9
1
PS GS Free from
AR
CrCl/100 Free from
DM
ESWD
Steroids
CyA-Induced Nephrotoxicity
Copyright Harvard Medical School, 2010. All Rights Reserved.
1951
CNI Avoidance:
IL-2ra/MMF/Predis not enough
54 DR-matched, 0% PRA recipients of CAD kidneys,
MMF+Pred Randomized to CsA or Daclizumab
Acute rej ecti on at 12 mo: GFR at 12 mo:
Dac: 70% CsA: 30% Dac: 52 ml /mi n CsA: 69
ml /mi n (p=0.006) (p=0.029)
Asberg A et al, Transplantation 2006;82: 62.
CNI avoidance with Sirolimus: initial optimism
Basi l i xi mab/MMF/Pred+
SRL (10-12ng/ml x 6 mo, then 5-10 ng/ml )
vs. CsA (200 ng/ml x 1y, then 150-175 ng/ml )
At I year: SRL group (n=31) CsA group (n=30)
Acute rej ecti on 6.4% 16.6% (p=ns)
Cr @1y 1.32 1.78 (p=0.004)
Cr @2y 1.35 1.81 (p=0.008)
GFR @2 y 60.6ml /mi n 49.2ml /mi n (p<0.02)
Flechner SM et al, Transplantation 2002; 74: 1070; and Am J Transplantation 2004;4: 1776
Protocol bi opsy at 2y:
Normal bi opsy i n 66% of
SRL group
Copyright Harvard Medical School, 2010. All Rights Reserved.
1952
CNI avoidance with Sirolimus: true benefit?
Rabbi t anti-thymocyte gl obul i n x 5/MMF/Pred+
SRL (15-20ng/ml x 4 mo, then 10-15 ng/ml )
vs. TAC (10-12ng/ml x 1 mo, 8-10ng/ml x 3 mo then 6-8 ng/ml )
At I year: SRL group (n=81) TAC group (n=84)
Acute rejection 13.0% 10.0% (p=ns)
GFR @1y 56 ml/min 55 ml/min (p=ns)
GFR @1y (on therapy) 57 ml/min 54 ml/min (p=ns)
GFR @2y 55 ml/min 55 ml/min (p=ns)
Biopsy data @1y: no differences in interstitial fibrosis,
tubular atrophy, or glomerulopathy
chronic vascular changes: 26% 43%
(p=0.03)
(cv1 or greater)
Discontinuati on rate 38% 16%
Larson TS et al, Am J Transplantation 2006;6: 514
Early CNI withdrawal with SRL maintenance:
better GFR, higher rejection rates
6 Randomized controlled trials with a SRL-based regimen
5 CSA withdrawal, 1 TAC withdrawal
5 with CNI withdrawal at 3 mo, 1 with CNI withdrawal at 2 mo
Mul ay AV et al . Am J Transpl antati on 2005; 5: 1748
Babool al 87 8 5%
Gonwa 197 8.9 8%
Johnson 430 6.1 5%
Gri nyo 87 1.1
Jardi nar 206 9.1
Pool ed 1007 +7.49 ml /min +6%(p=0.003)
(p<0.00001)
-5 0 5 10 15
Withdrawal better Withdrawal worse
-10 0 10 20
Withdrawal worse Withdrawal better
Creatinine clearance Acute rejection
Study N DCrCl DAR
Copyright Harvard Medical School, 2010. All Rights Reserved.
1953
Pharmacokinetic interactions to consider in CNI or
steroid withdrawal
CNI withdrawal from: Effect:
CsA/MMF Increased MPA exposure
CsA/SRL Decreased SRL exposure
TAC/MMF No change in MPA exposure
TAC/SRL No change in SRL exposure
Conversion to SRL/MMF from:
CSA/MMF Increased MPA exposure
CSA/SRL Decreased SRL exposure
Steroid withdrawal from: Steroid withdrawal from:
CsA/MMF Increased MPA exposure
TAC/MMF Increased MPA exposure, and
increased TAC exposure
CsA/SRL Increased SRL exposure?
TAC/SRL Increased TAC exposure
Disclosures
NONE
Copyright Harvard Medical School, 2010. All Rights Reserved.
1954
Immunogenetic Testing
Edgar L. Milford
Director HLA Laboratory
Brigham and Womens Hospital Brigham and Women s Hospital
Harvard Medical School
Disclosures
NONE NONE
Copyright Harvard Medical School, 2010. All Rights Reserved.
1955
Immunological Workup
ABO Compatibility
HLA Matching
Less important with current immunosuppression Less important with current immunosuppression
Pre-Existing Antibody
Cytotoxic PRA
Flow PRA
HLA vs Non-HLA
If HLA, Specificity for Donor Antigen?
C t h Crossmatch
Cytotoxic
Flow
Class I vs Class II
Post-Transplant Monitoring for Anti-HLA antibody.
HLA Matching
HLA-DR, B influence short term survival.
HLA-A influences long term survival
Importance in sensitized candidates Importance in sensitized candidates
Copyright Harvard Medical School, 2010. All Rights Reserved.
1956
Histocompatibility Typing
(HLA Typing)
Class I; HLA-A HLA-B HLA-C Class I; HLA A, HLA B, HLA C
Antigens Expressed on All Nucleated Cells
Target of Antibody Response
Target of Killer T-cell Response
Class II: HLA-DR
A ti E d P T b V l Antigens Expressed on Prox.Tub, Venules
Target of Helper T-cell response
Variable Antibody Response.
Graft survival by HLA mm
NotcensoredDWFG CensoredDWFG
1.0
10
0.1
0.2
0.3
0.4
0.5
0.6
0.7
0.8
0.9
S
u
r
v
i
v
i
n
g
0mm
1mm
2mm
3mm
4mm
5mm
6mm
0.1
0.2
0.3
0.4
0.5
0.6
0.7
0.8
0.9
1.0
S
u
r
v
i
v
i
n
g
0mm
1mm
2mm
3mm
4mm
5mm
6mm
0.0
0 1 2 3 4 5 6 7 8 9 10
gtime_ki_yr
0.0
0 1 2 3 4 5 6 7 8 9 10
gtime_ki_yr
Copyright Harvard Medical School, 2010. All Rights Reserved.
1957
Graft survival by Cold Ischemic Time
Distribution of Antigens By Race
Copyright Harvard Medical School, 2010. All Rights Reserved.
1958
Tissue Typing Lab
Typing Assays
HLA Class I-Serology HLA Class I Serology
HLA Class I-DNA (intermediate res)
HLA Class I-DNA (high res)
HLA Class II-Serology
HLA Class II-DNA (intermediate res) ( )
HLA Class II-DNA (high res)
Pre-existing Antibody
Conveys a higher risk of humoral rejection
t t l t post-transplant
Possibly an indicator of general immune
sensitization and a correlate of cellular
responsiveness as well.
May informclinicians about howclosely to May inform clinicians about how closely to
monitor patient for rejection post transplant
and ?? Intensity of immunosuppression
Copyright Harvard Medical School, 2010. All Rights Reserved.
1959
C4d
Copyright Harvard Medical School, 2010. All Rights Reserved.
1960
AMR Addition to Banff 1997
Racusen et al. AJT 2003; 3: 708
Morphologic features:
Type I: ATN-like
Type II: Capillary-glomerulitis, polymorphonuclear
and/or mononuclear leukocytes in peritubular
capillaries
Type III: Arterial-transmural inflammation/fibrinoid
change c a ge
Widespread linear circumferential C4d
deposition in peritubular capillaries
Circulating donor-specific antibodies
Use of Screening and
Crossmatch in Transplantation
Screen candidates monthly
Candidates on national list
High PRA means lower chance for a
crossmatch negative donor (lower chance
for a transplant) for a transplant)
Handicap points given for sentitized
patients with high PRA (>80%) to even out
their chances for getting an organ.
Copyright Harvard Medical School, 2010. All Rights Reserved.
1961
Pretransplant Monitoring
(Screening)
Screening of pretransplant candidate
i d t tit t th t serum is done to quantitate the amount
and anti-HLA specificity of antibodies the
candidate formed as a result of prior
transfusion, transplant, or pregnancies:
Candidate SerumAgainst HLA-Typed Candidate Serum Against HLA Typed
Normals
Quantitate Panel Reactive Antibody (%PRA)
Specificities against HLA
Use of Screening Information
Advisingcandidate about likelihood of Advising candidate about likelihood of
transplant
High PRA > Low likelihood of Tx
Deciding risk of response to cadaver
organ g
Prior anti-HLA antibody > Higher risk
Copyright Harvard Medical School, 2010. All Rights Reserved.
1962
Use of Screening and
Crossmatch in Transplantation
When organ becomes available When organ becomes available,
candidates with historic antibody
against HLA antigens on that donor are
excluded (high predictive value for a
positive wet crossmatch if antibody
present) present).
Wet crossmatch done on the rest of the
candidates.
Graft survival by previous
transplant
Notcensored CensoredDWFG Notcensored CensoredDWFG
01
0.2
0.3
0.4
0.5
0.6
0.7
0.8
0.9
1.0
S
u
r
v
iv
i
n
g
N
Y
01
0.2
0.3
0.4
0.5
0.6
0.7
0.8
0.9
1.0
S
u
r
v
iv
i
n
g
N
Y
0.0
0.1
0 1 2 3 4 5 6 7 8 9 10
gtime_ki_yr
0.0
0.1
0 1 2 3 4 5 6 7 8 9 10
gtime_ki_yr
Copyright Harvard Medical School, 2010. All Rights Reserved.
1963
Graft survival by peak PRA
Notcensored
CensoredDWFG
1.0 1.0
00
0.1
0.2
0.3
0.4
0.5
0.6
0.7
0.8
0.9
S
u
r
v
i
v
i
n
g
0-<10%
10-<80%
80-100%
UK
00
0.1
0.2
0.3
0.4
0.5
0.6
0.7
0.8
0.9
S
u
r
v
i
v
i
n
g
0-<10%
10-<80%
80-100%
UK
0.0
0 1 2 3 4 5 6 7 8 9 10
gtime_ki_yr
0.0
0 1 2 3 4 5 6 7 8 9 10
gtime_ki_yr
Candidate
HLA-A1,A3,B7,B8,DR1,DR4
TRANSPLANT
time
t1 t2 t3 t4 t5 t6
USE OF SCREENING AND CROSSMATCH IN
ORGAN TRANSPLANTATION
PRA 0 0 40 40 40 40
Antibodies n n A2 A2 A2 A2
Donor
HLA-A11 A9 B7 B8 DR1 DR4
C t h
Crossmatch-
Donor
HLA-A2,A3,B7,B8,DR1,DR4
time
HLA A11,A9,B7,B8,DR1,DR4
Crossmatch+
Copyright Harvard Medical School, 2010. All Rights Reserved.
1964
Tissue Typing Laboratory
Crossmatch Assays
Cytotoxic
T ll t h A W h ( dil) T cell crossmatch,Amos Wash (neg,pos; dil)
T cell crossmatch,Anti-Kappa (neg,pos; dil)
B cell crossmatch (neg,pos; dil)
Flow
T cell crossmatch (neg, pos; dfu)
B cell crossmatch (neg, pos; dfu)
IgM vs IgG
Problem 1
Unacceptable variation in PRA%when a Unacceptable variation in PRA% when a
given serum is tested by cytotoxic method
in different laboratories.
This is because the target cell panels
used in each lab are different, and may
reflect local prevalence of HLA in local
pools of volunteers.
Copyright Harvard Medical School, 2010. All Rights Reserved.
1965
Problem 2
UNOS audits found a surprisingnumber of UNOS audits found a surprising number of
candidates with high PRA getting negative
crossmatches (were transplant centers
cheating?)
UNOS Solutions
All transplant centers must do screening All transplant centers must do screening
of candidate serum using a solid state
method with purified HLA antigens.
All centers must list the specific HLA
antigens against which candidate has
antibody and UNOS will calculate the
PRA, not the transplant center.
Copyright Harvard Medical School, 2010. All Rights Reserved.
1966
Problem 3
Positive cytotoxic crossmatches are often Positive cytotoxic crossmatches are often
directed against NON-HLA antigens
expressed on T or B cells.
This is particularly true for patients with
autoimmune disease or on certain
medications.
HLA and MICA antibodies in patients after kidney transplantation
A serial 10-year-follow-up
Rejected Patients Functioning Patients
Mitsutani K. et al. Am J Transplant 2005 Sep;5(9):2265-72
rejected functioning p-value
n=39 n=26
HLA-Abs 72% 46% < 0.05
IgG-HLA + MICA 77% 42% < 0.01
IgG/IgM-HLA + MICA 95% 58% < 0.01
Copyright Harvard Medical School, 2010. All Rights Reserved.
1967
Solid State Screening
Luminex platform Luminex platform
HLA specific
Sensitive (qualitatively)
Sensitive (quantitatively)
Consistent
Alloantibody
Principle of LABScreen

System
Detection by R-Phycoerythrin Conjugated Antibody
PE anti-IgG
Alloantibody
Copyright Harvard Medical School, 2010. All Rights Reserved.
1968
Reporter Laser
PMT
Photo Multiplier Tube/Rep
Classification Laser
Avalanche Photo
Diodes/CL
Copyright Harvard Medical School, 2010. All Rights Reserved.
1969
Example of cPRA Calculation
Sample Caucasian Donor Frequencies:
A2 48% DR4 29% A2 48% DR4 29%
B44 27% DQ2 41%
B49 5%
Sample Patient cPRAs
Antibodies cPRA
Pt 1 No Abs 0%
Pt 2 B49 5% Pt 2 B49 5%
Pt 3 A2 48%
Pt 4 A2,B44 59%
Pt 5 DR4,DQ2 62%
Pt 6 A2,B44,DR4,DQ2 82%
Candidates Excluded Based on
Unacceptable Antigens
Unacceptable antigens entered into UNet used for
two purposes:
-To calculate cPRA for Allocation Points
-To exclude donor who has forbidden
ti antigens
Copyright Harvard Medical School, 2010. All Rights Reserved.
1970
Other Immunogenetic Risk
Factors
CCR59029 and Acute Rejection
0.7
0.8
0.2
0.3
0.4
0.5
0.6
E
p
i
s
o
d
e
s
* p<.05
* p<.05
0
0.1
A/A A/G G/G
Genotype
Copyright Harvard Medical School, 2010. All Rights Reserved.
1971
50
60
ANGIOTENSIN SYSTEM GENOTYPE AND PROGRESSION
OF RENAL TRANSPLANT DYSFUNCTION
(*)
I
O
N
(*) P <0.05
10
20
30
40
50
(*)
(*)
R
C
E
N
T

W
I
T
H

P
R
O
G
R
E
S
S
0
CHRONIC PROGRESSION 33.9 39.7 8.3 53 22.6 21.6 37.5 25 30.7
D/D
ACE
D/I
ACE
I/I
ACE
A/A
AGT
A/G
AGT
G/G
AGT
A/X
AT2
A/G
AT2
G/X
AT2
P
E
R
Questions
Is the overall impact of HLA matching as significant as
cold ischemia time in deceased donor kidney
transplants? transplants?
Are solid state assays for detecting anti-HLA antibody
more sensitive than cytotoxic methods? Are they more
specific? Why?
Which HLA locus is more important for short term kidney
transplant outcome (HLA-A, HLA-DR)?
How can a %PRA (ie an estimate of how many donors ( y
the recipient antibody will react with) be determined if the
method for screening is a solid state one with single
antigens.
Can there be humoral rejection if a donor and recipient
are HLA identical? Explain how this can happen.
Copyright Harvard Medical School, 2010. All Rights Reserved.
1972
Question One
A multiparous woman has failed a first kidney transplant from her
daughter one year ago. She is highly sensitized with cytotoxic
antibodyagainst 80%of a panel of HLA typed individuals antibody against 80% of a panel of HLA typed individuals.
She now has possibility of getting a kidney from her HLA
identical sibling, but the cytotoxic T cell crossmatch is positive.
What test would you recommend? (choose single
best answer).
a. Molecular (DNA) typing of the sister
b. Rh typing of the patient and sister yp g p
c. Solid state (single antigen) screening of patient serum
d. Auto crossmatch on recipient.
Question Two
Which of the following is true? (choose one).
a Cold ischemia time is always more important than HLA matching a. Cold ischemia time is always more important than HLA matching
b. Antibodies against non-HLA antigens are irrelevant for graft
survival
c. Kidneys from African-Americans do not function as well at
those from Caucasians
d. ABO mismatching is always an absolute contraindication to
transplantation.
e. It is impossible to remove pre-existing anti-HLA antibody and
have successful graft survival.
Copyright Harvard Medical School, 2010. All Rights Reserved.
1973
Disclosures
NONE NONE
Copyright Harvard Medical School, 2010. All Rights Reserved.
1974
Dr. M. J aveed Ansari M.D.
Transplant Nephrologist
Northwestern Memorial Hospital
Northwestern University Feinberg School of Medicine
Chicago
Pre-Transplant Evaluati on of Recipients
Disclosure Information
I have no financial relationships to disclose
I will not discuss off-label use and/or
investigational use in my presentation
Copyright Harvard Medical School, 2010. All Rights Reserved.
1975
Purpose of the Evaluation - Overview
Assess patients medical, surgical and
psychological suitability for transplant
Understand the patients reasons for
wanting a transplant
Discuss risks vs benefits of transplant
Discuss donor options
Plan the immunosuppressive protocol,
infectious disease prophylaxis and
additional monitoring ahead of time
The Big Questions
Will this pt benefit from transplant (survival,
quality of life)? Wolfe et al. NEJ M 1999
Prognostic Index: predicting survival benefit
of transplantation in ESRD van Walraven
et al. CMAJ 2010
Is it appropriate to ask a living donor to
donate?
Is it appropriate to take a deceased donor
kidney from the list? (Responsibilities to
individual pt and to the list)
Copyright Harvard Medical School, 2010. All Rights Reserved.
1976
ESRD Survival by Treatment Modality
77.8%
93.7%
97.6%
62.9%
91.6%
96.4%
31.9%
80.6%
90.4%
9.0%
58.9%
77.8%
0%
20%
40%
60%
80%
100%
120%
Dialysis (post day
91 of ESRD)
Posttransplant
survival (deceased
donor)
Posttransplant
survival (living
donor)
1 yr 2 yrs 5 yrs 10 yrs
National Kidney Foundation. Available at: http://www.kidney.org.
Graft Survival in 2405 Paired-Kidney Transplants: Short vs Long ESRD
Time
Adapted from Meier-KriescheHU, et al. Transplantation. 2002;74:1377-1381.
Copyright Harvard Medical School, 2010. All Rights Reserved.
1977
Survival Benefit of Transplant vs Remaining on Waiting List
Adapted from Ojo AO, et al. J Am Soc Nephrol. 2001;12:589-597.
Advantages and Disadvantages of Living-
Donor Transplantati on
Advantages Disadvantages
Preemptive transplant option
Can select donor for haplotype
match, age
Better outcomes
Minimal delayed graft function
No wait for deceased-donor
kidney
Can time transplant for
convenience
Immunosuppressive regimen may
be less aggressive
Emotional gain to donor
Psychological stress to donor
Long donor evaluation process
Operative donor mortality (~1/3000
patients)
Major complications (0.2%-2%)
Minor complications (~50%)
Potential donor hypertension,
proteinuria
Risk of trauma to remaining kidney
Risk of unrecognized covert renal
disease
Kendrick E, et al. In: Danovitch GM, ed. Handbook of Kidney Transplantation. 2005:135-168.
Copyright Harvard Medical School, 2010. All Rights Reserved.
1978
Discussion with Patient - I
What is the pts understanding of risks
and benefits of transplantation?
Is pt aware transplant is not a zero-risk
procedure?
Is pt aware of the types of transplants
and of live donor exchanges?
Discussion with Patient - II
Is pt suitable for / interested in
expanded criteria donors?
Is pt suitable for / interested in HCV Ab+
positive donors?
Is pt suitable for / interested in HBV
cAb+donors?
Copyright Harvard Medical School, 2010. All Rights Reserved.
1979
Referring Patients to the Transplant Center
The referring nephrologistis responsible for coordinating
all pretransplantcare
Point person in coordinating care with transplant
center, specialists (eg, cardiology)
Encouraging patients to learn about transplantation
helps improve outcomes
Transplantation can be preemptive
Identify potential donors
Patient can be listed when GFR <20 mL/min
Kidney Transplant Evaluation Process
Referred for transplant
Initial information session
Still a
candidate?
Potential
barrier?
Evaluate
Barrier
removed?
Proceed with evaluation
Dialysis when
indicated
No
No
Yes
Yes
No
Adapted from Kasiske BL, et al. Am J Transplant. 2001;1 (suppl 2):1-95.
Copyright Harvard Medical School, 2010. All Rights Reserved.
1980
History - I
Cause of ESRD
Previous transplants
Previous pregnancies, RBC transfusions
CV disease (exercise tolerance, evidence
of PVD )
Cancers
Other diseases
Previous immunosuppression (previous
transplant, SLE etc)
History - II
Compliance
Social supports
Family Hx (of renal disease, DM)
Functional status
Urine output; problems with urination
(males)
Possible living donors
Copyright Harvard Medical School, 2010. All Rights Reserved.
1981
Examination
General exam
BMI
Exclude overt CV disease and overt
neoplasia
Femoral pulses volume, bruits
Feet in diabetics
If the pt is frail - assess their walking
Routine Tests
ABO, HLA typing, Panel Reactive
Antibodies (PRA)
EKG, CXR +/- cardiac stress tests +/- TTE
HBV sAg, HBV sAb, HCV Ab, CMV Ab, HIV
Ab, syphilis, VZV Ab
PPD vs. QuantiFERON gold-TB
Pap smear in all women; mammogram
>40yrs
Stool guaic or colonoscopy if >50yrs
Testicular exam, PSA in >50 yrs
Copyright Harvard Medical School, 2010. All Rights Reserved.
1982
Optional Tests
Strongyloides serology
VCUG only if bladder problems
Serum electropheresis if elderly
Immunizations in Adults
Before transplant (at least 4 wks; longer
if live attenuated)
Viral: HAV, HBV*, influenza*, VZV,
H1N1
Bacterial: pneumococcus; Tdap booster
q 5 yr
*May need yearly booster
Stark et al., Lancet 2002; 359: 957
Copyright Harvard Medical School, 2010. All Rights Reserved.
1983
Contraindications to Transplant
Active cancer
Active infection (acute or chronic)
Active psychiatric illness
Ongoing non-compliance / substance
abuse
Major morbidity which would be
worsened by transplant or lead to very
short post-transplant survival
Severe obesity
Cancer
Risks of faster tumor growth and wasted
kidney
Minimum 2 yrs disease free; some
minimum 5yrs (melanoma, breast)
Oncology input
Std screening for breast, colorectal,
prostate, uterine cervix cancers
Copyright Harvard Medical School, 2010. All Rights Reserved.
1984
Acquired renal cystic disease /
renal tumors in dialysis pts
Pathogical analysis (Denton, Magee et al., Kidney
Int 2002; 62: 2201):
ARCD 33%
Adenoma 14%
RCC 4%
Tumors more common in: males, elderly,
longer time on dialysis
But are these tumors clinically significant?
Chronic Infection
Diabetic foot ulcers
Osteomyelitis
HIV not a C/I if controlled; refer to
special center
HCV related advanced cirrhosis (? list for
liver plus kidney)
Copyright Harvard Medical School, 2010. All Rights Reserved.
1985
Psychiatric Illness
Often treatable and not a C/I
Ongoing substance abuse a C/I
Mania: minimize steroids; avoid lithium if
possible (? valproic acid)
Non-Compliance
Common-sense assessment
Compliance to dialysis regimen, meds
Talk to local nephrologist and dialysis unit
Formal psychiatric assessment in some
cases
Many pts improve with time (on dialysis)
Copyright Harvard Medical School, 2010. All Rights Reserved.
1986
Obesity
Increasingly common
Most pts gain weight after transplant
risk of early complications: wound
infection, wound dehiscence
risk of later complications: DM, possibly
poorer allograft survivial
Most evidence suggests transplant still
better than dialysis
Evidence that BMI >41 is not a/wsurvival
benefit compared to dialysis
Obesity Practical Approach
Ideally BMI <30 but difficult to achieve
In general, aim for BMI <35
J udge each case on an individual basis
e.g. the young obese pt with no CAD
may do very well with a transplant
Consider steroid free protocols
Copyright Harvard Medical School, 2010. All Rights Reserved.
1987
Cardiovascular
Disease
CV Disease: Background
Important to distinguish peri-operative
and long-term risk
Assessing peri-operative risk
Risk of the surgery: renal transplant
surgery is low-moderate
Patients risk based on their clinical
status: most patients are intermediate
Copyright Harvard Medical School, 2010. All Rights Reserved.
1988
Clinical Predictors of Peri-oprative Risk in
the Patient
High Intermediate
Recent MI, ESRD / CKD
Unstable angina DM
Decompensated CHF
Severe valvular disease
Severe arrhythmias
Mgt of CV Risk in Noncardiac
Surgery -I
ACC/AHA Practice Guidelines
Eagle et al., J Am Coll Cardiol. 2002; 39: 542-53
The overriding theme of these guidelines
is that preoperative intervention is
rarely necessary simply to lower the
risk of surgery unless such intervention
is needed irrespective of the
preoperative context
Copyright Harvard Medical School, 2010. All Rights Reserved.
1989
Which Test to Use ?
Is it needed at all?
Coronary angiography is gold standard but is
invasive and in pre-dialysis patients, could
precipitate contrast nephropathy
ETT predicts risk in non-ESRD population but
many ESRD pts dont achieve adequate stress
MPS/SE also predict risk in ESRD population
No consensus on the best test in ESRD pts
In practice, depends on your center
Algorithm for cardiac evaluation
High risk renal tx candidate
Myocardial perfusion study
Coronary Angiography
Asymptomatic for CAD Symptomatic IHD
+ve
or
-ve ?
Transplant
-ve
+ve
Adapted from PilmoreH, et al. Am J Transplant. 2006; Apr;6(4):659-65.
Copyright Harvard Medical School, 2010. All Rights Reserved.
1990
Optimizing CV status while on the list
Control of HTN
Control of CHF
Rx of anemia
Rx of hyperlipidemia
Revascularization where appropriate
Re-evaluate higher risk pts (eg diabetics)
every 12m
The Diabetic Recipient
Is whole organ pancreas (simultaneous
or after kidney transplant) potentially of
benefit?
Timing of transplant
CAD
PVD
Gastropathy(medications)
Foot ulcers
Copyright Harvard Medical School, 2010. All Rights Reserved.
1991
Recurrent Disease
Recurrent Disease
Difficult to estimate as original dx may be
unknown and recurrence is probably
underdiagnosed post-transplant
Histological recurrence clinical
recurrence
In 1 series, the third most important
cause of graft loss (after rejection and
death) in those with ESRD due to GN
Risk of renal allograft loss from recurrent glomerulonephritis;
Briganti at al. N Engl J Med. 2002;347(2):103-9
Copyright Harvard Medical School, 2010. All Rights Reserved.
1992
Recurrent Disease
In general, transplant is not
contra-indicated but should be
deferred until the disease is
quiescent
- Anti-GBM disease
- FSGS leading to rapid loss of native
kidneys and first trasplant
Idiopathic MPGN I, II and
Membranous Nephropathy
All may recur and all may be a/w allograft
loss
Important to exclude underlying causes
such as HCV or TMA
MPGN can resemble chronic transplant
glomerulopathy on light microsopy
Membranous Nephropathy can arise de
novo after transplantation
Copyright Harvard Medical School, 2010. All Rights Reserved.
1993
SLE
34 year old south asian women
Lupus Class IV on biopsy
Creatinine 5.2 mg/dl
Complement levels >500
Last flare (joints) 1 month ago
No evidence of APS
Father is worked up as donor
What would you do?
SLE
Now 22 months later
Creatinine 4.8 mg/dl
Complement levels 54
No lupus flares since last being seen
Father is now 63
What would you do?
Copyright Harvard Medical School, 2010. All Rights Reserved.
1994
SLE
Allograft and pt survival same as non-
SLE recipients (Ward; Kidney Int 2000
57:2136)
Allograft survival poorer if
superimposed APS
Ensure disease clinically quiescent
before transplant
High steroid exposure: minimize further
steroids; prevent osteoporosis etc.
High prevalence of CV disease
Systemic (ANCA) Vasculitis
Renal and extrarenal recurrence
One series: recurrence anywhere 17%; in
graft 10% (f/u mean 44m) Nachmanet al, Kidney Int
1999; 56:1544
Ensure disease is clinically quiescent before
transplant
ANCA at time of transplant probably not
predictive of relapse
Overall, allograft survival similar to non-
vasculitis Magee et al, AST, 2003
Copyright Harvard Medical School, 2010. All Rights Reserved.
1995
FSGS
21 year old with primary FSGS
Presents with ESRD
No previous transplants
Mother willing to donate
How would you proceed?
Primary FSGS
Risk of recurrence 20-40% (>75% in Re-Tx)
Circulating factor
Recurs hrs days;
Proteinuria then Cr
Risk factors: white, child, previous
malignant course, recurrence in previous
allograft
Case series suggest benefit from
plasmapheresis, high dose CNIs, steroids
Copyright Harvard Medical School, 2010. All Rights Reserved.
1996
HUS / TTP
Many forms of HUS / TTP essential
to characterize in an individual pt
Classic (diarrhea associated) unlikely
to recur but wait minimum 6 months
Atypical (non-diarrhea) likely to recur
especially if inherited
Prognosis if recurrence, is poor
BK Virus Nephropathy
10-15% recurrence
Allograft nephrectomy ?
Preemptive transplantation ?
Living donor ?
HLA matching ?
Reduced Immunosuppression ?
Copyright Harvard Medical School, 2010. All Rights Reserved.
1997
Other Conditions
(Previously
Contraindications)
HIV
No longer a C/I per se
With HAART, survival of HIV+(ESRD) pts has
dramatically improved
Potential for multiple drug interactions
CNIs, MMF, SRL possibly have anti-HIV effects
Short-term transplant outcomes encouraging;
increased risk of acute rejection
Long-term outcomes being studied in a multi-
center trial
Copyright Harvard Medical School, 2010. All Rights Reserved.
1998
Age per se is no longer a C/I
Selected elderly pts do better with
transplant than remaining on the list
J ohnson at al., Transplantation. 2000; 69: 794-9
Rigorous screening especially for cancer
Strongly consider expanded criteria
donors
Rejection may still be problematic
Advanced Age
Traditionally, very high risk of acute
antibody mediated rejection and graft
loss if transplant across ABO barrier
Now, options include kidney exchanges
or a plasmapheresis /
immunosuppressionprotocol to remove
antibodies
ABO-Incompatability
Copyright Harvard Medical School, 2010. All Rights Reserved.
1999
Previous transplant, BT, pregnancy
Traditionally: very long waiting times,
higher rates of rejection and graft loss
Excellent short-term outcomes with
protocols involving either: high dose IVIg
or PP +IVIg+MMF +tacrolimus
Antibody mediated rejection still common
but can be reversed
Expensive, risk of overimmunosuppression
Highly Sensitized (to HLA antigens)
Caveats with Regard to Desensitization
Long-term data not available
Will low grade / chronic humoral rejection
be problematic?
Quite intense immunosuppression; major
long-term problems?
Expensive (but probably cheaper than
dialysis over long-term)
Copyright Harvard Medical School, 2010. All Rights Reserved.
2000
Mgt of Patients on the List
Becoming a major workload for many
centers (waiting times so long)
How frequently should center re-
evaluate?
Standard screening for cancers
Assess by Hx, Ex for new CV disease;
intervene as appropriate
Communication with dialysis unit,
referring physicians
Interim Medical Examinations
During wait for a deceased-donor, routine medical
evaluations should be conducted
Lipid panels
Diabetes screening
Cancer screening
Pap smears and mammograms for women
Digital rectal exam or PSA test for men
Cardiovascular examination as indicated
The community nephrologist should advise the transplant
center of changes in health that preclude transplantation
Patients who require medical intervention may remain on
the UNOS list, but do not accrue time of waiting points
Copyright Harvard Medical School, 2010. All Rights Reserved.
2001
Role of Primary Nephrologist
Community nephrologists play a key role in the
transplant process
Identification of patients who will benefit from
transplant
Referral to the transplant center
Coordination of specialists in pretransplant
evaluation
Continuation of care while waiting for transplant
Notifying transplant center of health status
changes
Long-term care posttransplant
Summary - Purpose of the Evaluation
Assess patients medical, surgical and
psychological suitability for transplant
Understand the patients reasons for
wanting a transplant
Discuss risks vs benefits of transplant
Discuss donor options
Plan the immunosuppressive protocol,
infectious disease prophylaxis and
additional monitoring ahead of time
Copyright Harvard Medical School, 2010. All Rights Reserved.
2002
Question 1: Which of the following is a risk
factor for HLA sensitization?
a) Previous pregnancies
b) Previous blood transfusions
c) Previous transplants
d) Use of tissue homografts for vascular
reconstruction
e) All the above
Question 2: Which of the following is a
contraindication for transplantation?
a) HIV infection
b) HCV infection
c) Positive PPD
d) All of the above
e) None of the above
Copyright Harvard Medical School, 2010. All Rights Reserved.
2003
Question 3: Which complication of renal
transplantation is most prominent among
patients with HIV infection compared with
patients without HIV infection?
a)Reduced patient survival
b)Reduced graft survival
c)Wound infection
d)Acute graft rejection
References
1. The evaluation of renal transplantation candidates: clinical practice
guidelines.KasiskeBL et al. Am J Transplant. 2001;1 Suppl 2:3-95
2. Survival in recipients of marginal cadaveric donor kidneys compared with
other recipients and wait-listed transplant candidates.OjoAO et al. J Am
Soc Nephrol. 2001 Mar;12(3):589-97
3. Predicting potential survival benefit of renal transplantation in patients with
chronic kidney disease.van WalravenC et al. CMAJ. 2010 Apr
20;182(7):666-72
4. Cardiovascular risk assessment among potential kidney transplant
candidates: approaches and controversies. LentineKL, et al. Am J Kidney
Dis. 2010 J an;55(1):152-67
5. Risk of renal allograft loss from recurrent glomerulonephritis.Briganti EM et
al. N Engl J Med. 2002 J ul 11;347(2):103-9
Copyright Harvard Medical School, 2010. All Rights Reserved.
2004
Disclosure Information
I have no financial relationships to disclose
I did not discuss off-label use and/or
investigational use in my presentation
Copyright Harvard Medical School, 2010. All Rights Reserved.
2005
The Evaluation of Living and
Deceased Donors for Kidney
Transplantation
Didier Mandelbrot
Beth Israel Deaconess Medical Center
Neither I nor my spouse have
any financial relationships to
disclose
Copyright Harvard Medical School, 2010. All Rights Reserved.
2006
2007 SRTR Annual Report, Tables 1.7, 5.1a
Meeting the demand for transplant
kidneys
Living donation
Live donor paired exchange
Extended Criteria Donor (ECD) kidneys
Donation after Cardiac Death (DCD)
kidneys
Copyright Harvard Medical School, 2010. All Rights Reserved.
2007
Living kidney donation in the US
has increased
0
1,000
2,000
3,000
4,000
5,000
6,000
7,000
8,000
9,000
1
9
8
8
1
9
9
0
1
9
9
2
1
9
9
4
1
9
9
6
1
9
9
8
2
0
0
0
2
0
0
2
2
0
0
4
2
0
0
6
2
0
0
8
DeceasedDonor
LivingDonor
Unos.org
Copyright Harvard Medical School, 2010. All Rights Reserved.
2008
Advantages of living donor kidneys
over deceased donor kidneys
No waiting time, versus 3-5 year wait
Longer graft survival
Donors are healthier
Shorter cold ischemia time means less kidney
damage, less delayed graft function and less
rejection
Scheduled surgery is more successful than
emergency surgery
Better kidney function
Laparoscopic donor
nephrectomy
Decrease in hospital length of stay
Less pain
Quicker return to work
Improved cosmetic result
Greater donor acceptance
Equivalent graft function/survival
Copyright Harvard Medical School, 2010. All Rights Reserved.
2009
Survival of living kidney donors in Sweden
compared with expected survival
Fehrman-Ekholm et al.
Ki dney Donors Li ve
Longer
Transplantation 1997
Former US kidney donors are as healthy as
controls matched for age, gender, race and BMI
Ibrahim NEJ M 2009
Copyright Harvard Medical School, 2010. All Rights Reserved.
2010
Living donor paired exchange
programs
Donor - blood type A
Recipient - blood type B
Recipient blood group A
Donor - blood type B
Incompatible
Incompatible
Copyright Harvard Medical School, 2010. All Rights Reserved.
2011
The Evaluation of Potential
Living Kidney Donors
Donor work-up
Blood group and HLA compatibility
History and physical exam
Comprehensive lab screening
Blood chemistry panel and cbc
HBV, HCV, HIV serologies
Urinalysis
24-hour urine creatinine and protein
Chest X-ray
ECG
Cardiac testing as appropriate
CT angiogram
Psychosocial evaluation
Copyright Harvard Medical School, 2010. All Rights Reserved.
2012
Older Age
0
10
20
30
40
50
60
70
80
90
100
No upper
age
exclusion
Excl if >55 Excl if >60 Excl if >65 Excl if >70 Excl if >75-80
1995 2007
Mandelbrot et al. Am J Transplant 2007
Obesity
0
10
20
30
40
50
60
70
80
90
100
No policy No BMI cut-
off
Excl if
BMI>30
Excl if
BMI>35
Excl if
BMI>40
Excl if high
BMI and
other CV
risk factors
2007
1995: 16% exclude if
" moderate obesity"
Mandelbrot et al. Am J Transplant 2007
Copyright Harvard Medical School, 2010. All Rights Reserved.
2013
BMI greater than 30 at
the time of surgery
predicts development
of proteinuria and renal
insufficiency after
unilateral nephrectomy
Praga et al. KI 2000
Creatinine Clearance Cut-off
0
10
20
30
40
50
60
70
80
90
100
No cut-off < 80 ml /mi n/1.73m2 <60ml /mi n/1.73m2 <40ml /mi n/1.73m2 > 2 SD bel ow mean
for age
1995 2007
Mandelbrot et al. Am J Transplant 2007
Copyright Harvard Medical School, 2010. All Rights Reserved.
2014
Average Renal Function by Age
Modified from Davies et al, J CI 1950
Age Mean Creat
Clearance
(mL/min/1.73m
2
)
Std dev Mean Creat
Clearance
minus 2 std dev
25-34 140 21.4 97
35-44 133 20.0 93
45-54 127 17.3 93
55-64 120 16.5 87
65-74 109 16.5 76
Goal kidney function for donors
Donors should have adequate kidney
function for the recipients and for
themselves
Cut-off of 80ml/min is acceptable from
recipient perspective
Cut-off of 2 SD below mean for age is
acceptable for donor
Copyright Harvard Medical School, 2010. All Rights Reserved.
2015
Potential exclusions to donation
Hypertension
wide variations in practice, ranging from pre-HTN to
HTN controlled with two meds
Glucose intolerance
fasting blood sugar, OGTT, HbA1c
Proteinuria
300 mg/day vs 150 mg/day, higher if postural
Hematuria
3 rbc/hpf vs 5 rbc/hpf vs 10 rbc/hpf, higher if
urological w/u and renal biopsy negative
How flexible is your application of the donor
selection criteria identified above, based on:
0
10
20
30
40
50
60
1 2 3 4 5 6 7
1 = not at al l fl exi ble, 7 = extremel y fl exi bl e
Younger Donor Age
Mean = 2.3
Mandelbrot et al. Am J Transplant 2007
Copyright Harvard Medical School, 2010. All Rights Reserved.
2016
How flexible is your application of the donor
selection criteria identified above, based on:
0
5
10
15
20
25
30
35
40
1 2 3 4 5 6 7
1 = not at al l fl exi ble, 7 = extremel y fl exi bl e
Severity of Recipients
Medical Condition
Mean = 3.0
Mandelbrot et al. Am J Transplant 2007
Conclusions
Living kidney donation is increasingly
important in transplantation and
nephrology
Living donation is safe for young healthy
donors
Safety of donation by medically complex
donors is less certain more data is
required
Copyright Harvard Medical School, 2010. All Rights Reserved.
2017
Deceased donor kidneys
Standard criteria donor (SCD) versus
extended criteria donor (ECD)
By initial definition, rate of graft failure of ECD
is 1.7 times that of SCD
Donation after cardiac death (DCD)
kidneys are allocated with SCD
SCD and DCD kidneys have comparable
long-term graft survival
DCD kidneys have more delayed graft
function
Graft survival is comparable with DCD donors
and deceased donors with heartbeats
Cho Y et al. N Engl J Med 1998;338:221-225
Copyright Harvard Medical School, 2010. All Rights Reserved.
2018
Criteria for ECD kidney
Donor >59 years old
or
Donor 50-59 years old plus two of the
following three:
Hypertension
Creatinine >1.5
Death from CVA
Outcomes with ECD (grade B)
kidneys
Shorter graft survival
Higher risk of delayed graft function
Shorter waiting time
Increased patient survival for diabetics
and older recipients (groups with the
highest death rates on dialysis)
Copyright Harvard Medical School, 2010. All Rights Reserved.
2019
Copyright Harvard Medical School, 2010. All Rights Reserved.
2020
Conclusions
For most potential kidney transplant
recipients, a living donor is the best choice
Depending on the potential recipients
demographics, an ECD kidney may be the
next best choice
Because of excellent outcomes, DCD
kidneys are allocated with SCD kidneys
Copyright Harvard Medical School, 2010. All Rights Reserved.
2021
References
Fehrman-Ekholm et al Transpl 1997
Mandelbrot et al Am J Transpl 2007
Ibrahim et al NEJ M 2009
UNOS.org
Neither I nor my spouse
have any financial
relationships to disclose
Copyright Harvard Medical School, 2010. All Rights Reserved.
2022
Types of deceased donor kidneys
Choose the correct response
SCD kidneys have the same rate of delayed
graft function as DCD kidneys
ECD kidneys last as long as SCD kidneys
The wait time for an ECD kidney is the same
as for a SCD kidney
In selected populations, patient survival is
improved by accepting ECD kidneys
Long-term consequences of living
kidney donation
Choose the correct response
Donation increases the risk of ESRD
ESRD never occurs in former living donors
Former donors have similar GFR to matched
controls
Donation is associated with psychological
satisfaction
Copyright Harvard Medical School, 2010. All Rights Reserved.
2023
Neither I nor my spouse have
any financial relationships to
disclose
Copyright Harvard Medical School, 2010. All Rights Reserved.
2024
Early Post-Transplant Early Post Transplant
Complications
Nidyanandh Vadivel MD, MRCP
&&
Sayeed Malek MD
Brigham and Womens Hospital
Harvard Medical School
Disclosures
None
Copyright Harvard Medical School, 2010. All Rights Reserved.
2025
Its all started here!!
1954: Worlds first successful renal
transplant
Early Post-Tx Surgical complications
Bleeding Bleeding
Vascular thrombosis of allograft
Wound complications
Lymphocele
Urine Leaks
Ureteral obstruction Ureteral obstruction
Copyright Harvard Medical School, 2010. All Rights Reserved.
2026
Early Post-Tx Medical complications
Post transplant acute tubular necrosis Post transplant acute tubular necrosis
Electrolyte imbalance
Renal Allograft Rejection
Calcineurin Inhibitor toxicity
Post transplant thrombotic microangiopathy
Recurrent Renal Disease
Approach to allograft dysfunction
Vary markedly based on the timing of allograft a y a ed y based o t e t g o a og a t
dysfunction post transplant
First post op day
First week
First 3 months
Copyright Harvard Medical School, 2010. All Rights Reserved.
2027
Case 1
54 year old male with past history of allogeneic stem
cell transplant for Acute lymphocytic leukemia
He developed ESRD post stem cell transplant from
thrombotic microangiopathy. He was on tacrolimus
and steroid at the time of renal failure. Exact etiology
of TMA was not clear. Native urine output is about a
liter/day
He underwent living unrelated renal transplant with
simulect induction and FK, MMF and steroid
maintenance
Case 1 (contd)
Donor was a 63 year old gentleman with
hypertension (controlled on one medication) yp ( )
Recipients pre transplant BP is 90/50s
Peri-transplant, recipients systolic BP dropped
frequently to 80s in spite of multiple boluses of IV
fluids
Post op day 1, pt made 30cc/hour of blood stained
urine. Creatinine remained unchanged at 6.4
Copyright Harvard Medical School, 2010. All Rights Reserved.
2028
Case 1 - Differential diagnosis
Pre renal state (bleeding hypovolemia) Pre- renal state (bleeding, hypovolemia)
Clotted Foley catheter
Vascular complications
Renal arterial or venous thrombosis
Ureteric leak/obstruction
Hyperacute rejection Hyperacute rejection
Acute tubular injury
Thrombotic microangiopathy
Pre-renal state
Assess for signs of intravascular volume depletion
Monitor HR, BP, tissue perfusion, CVP and UO
Look for evidence of diminished cardiac pump (MI,
failure, arrhythmias post op)
Intravascular volume depletion
Dialysis pre op is a potential risk factor - Dialysis pre-op is a potential risk factor
- Intra-operative / Early post-op bleeding
- Reaction to drugs (thymoglobulin)
Copyright Harvard Medical School, 2010. All Rights Reserved.
2029
Bleeding
Early postop
Stop Aspirin, Plavix and anticoagulants before
transplantation to minimize risk
Correct uremia related platelet dysfunction and abnormal
Bleeding time through dialysis
Early post op bleeding usually results from small hilar
vessels
Cl b ti f it l d HCT i i l Close observation of vitals and HCT is crucial.
Correct with blood products. Surgical exploration may be
necessary
Vascular thrombosis
Abrupt cessation of urine output +/- graft pain & swelling
and raised creatinine raises suspicion
Most arterial and venous thrombosis happen within the
first few days of transplantation.
Reported incidence - Arterial (1%) Venous (1-4%)
Risk increased in patients with atherosclerotic vessels,
anti cardiolipin abs prior thrombotic tendencies and anti-cardiolipin abs, prior thrombotic tendencies and
thrombocythemia
Early diagnosis is crucial in salvaging grafts. Obtain
doppler US or isotope renal scan asap
Copyright Harvard Medical School, 2010. All Rights Reserved.
2030
Ultrasound
Radiographics. May 2000 volume 20
Resistive index
Normal RI
0 6 0 8 0.6 0.8
Elevated RI
>0.9
Zwirewich C. Renal Transplant Imaging and Intervention: practical Aspects - 2 Radiology.co.uk
Copyright Harvard Medical School, 2010. All Rights Reserved.
2031
Renal Vein Thrombosis
Radiographics. May 2000 volume 20
Vascular thrombosis
Prevention through early anticoagulation in high risk
pts
Acute arterial thrombosis is usually due to technical
problem or small embolus
Venous thrombosis may present with gross
hematuria
Surgical repair should be carried out immediately
Late thrombosis (upto 2 months) is mostly from
rejection
Copyright Harvard Medical School, 2010. All Rights Reserved.
2032
Hyperacute rejection
Results from circulating preformed cytotoxic antidonor
antibodies directed to ABO antigens or donor HLA class I
antigens antigens
Antibodies cause complement mediated endothelial
injury, plt aggregation and microvascular thrombosis
Highly sensitized patients are at most risk
Very rare occurrence nowadays due to improved pre Very rare occurrence nowadays due to improved pre
transplant crossmatch techniques
Results in allograft failure and nephrectomy
Post transplant Acute tubular necrosis
About 20-40% incidence among cadaveric organ
transplants. Can last for one to several weeks.
Incidence depends on age, quality of the donor kidney,
cold & warm ischemia time, circumstances of death,
early use of CNI etc.
Rule out other causes of delayed graft function such as
obstruction and vascular causes by US and acute y
rejection by biopsy within 7-10 days
Cautious use of CNI. Take care to avoid hypotensive
injury to allograft on dialysis
Copyright Harvard Medical School, 2010. All Rights Reserved.
2033
Post Transplant TMA
Anemia, Thromobocytopenia, Raising LDH,
Schisotocytes on peripheral smear should raise
suspicion suspicion
Can be localized to kidney or can present as HUS
Diagnosed by glomerular microthrombi on biopsy. C4d
stain should be done to rule out AMR
Initial biopsy may be negative hence high index of Initial biopsy may be negative, hence high index of
suspicion is needed.
Both cyclosporin and tacrolimus can cause TMA. Early
discontinuation or reduction of CNI with
plasmapheresis may help prevent graft loss
TMA involving glomerulus
Changsirikulchai et al. cJASN 2009
Copyright Harvard Medical School, 2010. All Rights Reserved.
2034
Case 2
51 yr old male underwent living related renal
transplant for ESRD secondary to IgA nephropathy
Creatinine settled at 1.8 at discharge
On follow-up visit (day 10 post op):
He felt well. Made 2 liters of urine/day. Tolerated meds
well.
Continued to have about 700cc of Jackson Pratt drain
output per day
Case 2 (contd)
On post op day 14, he developed pain (scale 3-4/10)
at the transplant site at the transplant site
JP drain output was 90cc two days ago and 0 over
last 24 hours
Creatinine 2.3 (was 1.8, four days ago)
What do you do next?
Copyright Harvard Medical School, 2010. All Rights Reserved.
2035
Case 2 (contd)
Renal allograft US revealed
- 11 8cm transplant kidney - 11.8cm transplant kidney
- New minimal hydronephrosis
- Normal resistive indices (RI 0.6 0.7)
- Two collections noted in the medial aspect of the
kidney. Deep collection measured 10 X 6 X 12 cm
(360mls)
How would you approach this further?
Case 2 (contd)
Differential diagnosis:
Lymphocele Lymphocele
Urinary leak (urinoma)
Urinary obstruction
Acute rejection
Percutaneous aspirate and send fluid for Percutaneous aspirate and send fluid for
microscopy, culture, protein and creatinine
estimation
Copyright Harvard Medical School, 2010. All Rights Reserved.
2036
Lymphocele
Result from severed lymphatic vessels that overlie
iliac vessels. Usually presents within weeks y p
Even small lymphocele can occasionally result in
ureteral obstruction
Large lymphoceles can present as abdominal mass,
incontinence, scrotal masses or vena caval
obstruction
Sonolucent fluid collection with septations on US
Lymphocele
Radiographics. May 2000 volume 20
Copyright Harvard Medical School, 2010. All Rights Reserved.
2037
Lymphocele
Early Sirolimus use increase the incidence from 18% to 38%
Fluid is clear with high protein content and creatinine
concentration similar to serum
Treatment
Small No intervention
Large lymphocele with compression Large lymphocele with compression
External drainage
Sclerosing agents
Marsupialization
Lymphocele
pre and post drainage
Garcia et al. Arch esp urol 2009 J une
Copyright Harvard Medical School, 2010. All Rights Reserved.
2038
Urine Leak
Confirm fluid collection with US or CT. Nuclear studies or
retrograde cystogram may be beneficial g y g y
Send aspirate for creatinine estimation. Immediate
provide bladder rest with foley
Percut nephrostomy and nephrogram will be definitive
and also help with management p g
Type of surgical repair depends on the level of leak and
viability of the transplant ureter
Ureteral obstruction
Manifests with elevated creatinine
Low grade pelvi-calyceal dilatation of transplant kidney is
normal. However increasing hydronephrosis is
suggestive of obstruction
Mag 3 scan may be helpful to diagnosis this condition
(least intervention with no nephrotoxicity) ( p y)
Percutaneous antegrade pyelography is the best
diagnostic modality to diagnose the level of obstruction
Copyright Harvard Medical School, 2010. All Rights Reserved.
2039
Renal Tx ultrasound
Normal Pelvi-calyceal dilatation post transplant
ONeill et al. Ultrasonography in renal transplantation, AJKD 2002
Renal Transplant hydronephrosis
(several days post transplant)
ONeill et al. Ultrasonography in renal transplantation, AJKD 2002
Copyright Harvard Medical School, 2010. All Rights Reserved.
2040
Ureteral obstruction
Early post-op obstruction is due to blood clots, technical
failed ureteric reimplantation and ureteral slough.
Intrinsic obstruction results from ureteric stenosis
Extrinsic compression results from Lymphocele, ureteral
kinking
Late causes include calculi
Treatment:
Percutaneous nephrostomy with ureteral stent Percutaneous nephrostomy with ureteral stent
(most cases)
Surgical repair
Ureteral Stenosis
Radiographics. 2000
Copyright Harvard Medical School, 2010. All Rights Reserved.
2041
Ureteral stenosis - Placement of JJ stent
Radiographics 2000
Ureteral stenosis After placement of JJ
stent
Radiographics. 2000
Copyright Harvard Medical School, 2010. All Rights Reserved.
2042
Case 2 (contd)
Creatinine improved to 1.7 after percutaneous drain
placement p
Drain came out after 1 week
3 weeks later, patient presents with slowly raising
creatinine to 2.4. FK levels are optimal.
Repeat US, shows recurrence of the collection
How should we proceed now?
Peri-transplant fluid collections post renal transplant
Mehrabi et al. Clinical transplant 2006
Copyright Harvard Medical School, 2010. All Rights Reserved.
2043
Case 3
62 yr old CAPD patient for ESRD secondary to
congenital solitary kidney transplanted with living
l t d l t l t 11/05/09 Fi l CDC unrelated renal transplant on 11/05/09. Final CDC
crossmatch was negative but positive flow B cell
crossmatch - felt to be due to non- compliment
binding autoantibodies.
Inducted with Simulect and maintained on FK, MMF
and steroids
Day 5 post transplant his creatinine improved to 1.1
Case 3 (contd)
1 month later his creatinine was noted to be elevated
at 2.4.
What is differential diagnosis and how would you
approach the case?
Pre renal state
Calcineurin inhibitor toxicity
Outflow obstruction Outflow obstruction
Rejection
Recurrence of native disease
Copyright Harvard Medical School, 2010. All Rights Reserved.
2044
Calcineurin Inhibitor Toxicity
Causes afferent arteriolar vaso-constriction resulting in
reduced renal blood flow and GFR reduced renal blood flow and GFR
Allograft function improves if CNI toxicity is promptly
corrected
Co-existent CNI toxicity and rejection can be present
If creatinine does not improve, then look for other causes
CNI toxicity presents with renal tubule vacuolization
Case 3 (contd)
Difficult biopsy 4 passes, 2 cores obtained
Biopsy revealed severe necrotizing vasculitis in at
least two arterioles, with fibrinoid necrosis,
lymphocytic infiltration, and nuclear debris in the
vascular walls suggestive of Banff 2a or 3 rejection.
C4d was negative.
Copyright Harvard Medical School, 2010. All Rights Reserved.
2045
Early Renal allograft rejection
Hyperacute rejection (within minutes to hours) Hyperacute rejection (within minutes to hours)
Accelerated acute rejection (within first week of
transplant)
Acute cellular rejection
Acute humoral or antibody mediated rejection
Accelerated acute rejection
Similar mechanism of allograft injury like hyper acute
rejection but happens within a week rejection but happens within a week
Raised creatinine, anuria or oliguria +/- graft tenderness
and fever
Allograft biopsy shows massive infiltration of
lymphocytes, plasma cells and macrophages lymphocytes, plasma cells and macrophages
Immediate treatment with anti-T cell antibody with IV
pulse steroids may help save upto 50% of the allografts
Copyright Harvard Medical School, 2010. All Rights Reserved.
2046
Acute cellular rejection
T cell mediated injury against renal tubules
Most common type of rejection yp j
Incidence is upto 15% within 1 year and usually happens
within first 3 months
>95% are reversible
Treatment
Mild: pulse IV steroids
M d t & S A ti T ll th t id Moderate & Severe: Anti T cell therapy+ steroids
Recurrent rejections or late ACR adversely affects graft
outcome
Antibody mediated rejection
Usually occurs in the first few weeks post transplant. Pre
sensitized pts are at high risk for AMR.
Morphological features may be minimal. Tubular injury
may be the only manifestation
Diffuse C4d staining is essential to diagnose AMR
Donor recipient crossmatch or single antigent testing
should be performed to look for donor specific should be performed to look for donor specific
antibodies
Plasmapheresis + IVIg is the treatment of choice
Copyright Harvard Medical School, 2010. All Rights Reserved.
2047
Expression of C4d in renal allograft
Normal biopsy
Acute humoral rejection with C4d in PTC
Seemayer et al. NDT 2006
ACR AHR
Mauiyyeddi et al JASN 2002
Copyright Harvard Medical School, 2010. All Rights Reserved.
2048
Case 3 contd
He was treated with thymoglobulin, Pulse steroids
and FK & MMF maintanence.
His creatinine rose to 8 in the following 10 days
inspite of aggressive anti-rejection therapy
He had uremic symptoms and needed HD initiation
What is your differential diagnosis and how would What is your differential diagnosis and how would
you approach?
Case 3 Contd
2 days post biopsy, US revealed new moderate sized
fluid collection in the medial portion of tx kidney fluid collection in the medial portion of tx kidney
US guided aspiration was unsuccessful
Repeat US on 6
th
post biopsy day revealed a 5.5 cm
aneurysm in the renal hilum with moderate amount
of central thrombus
Creatinine worsened to 8 and hence another biopsy
was performed was performed
Biopsy revealed ATN and resolved acute rejection
Copyright Harvard Medical School, 2010. All Rights Reserved.
2049
Color doppler
Case 3 Contd
Percutaneous Angiogram was performed
Pseudoaneurysm arising from a third-order branch
of an upper segmental artery was noted.
Anastomosis was widely patent
Successful embolization of the branch feeding the Successful embolization of the branch feeding the
pseudoaneurysm done.
Creatinine over the next 20 days, improved to 1.4
Copyright Harvard Medical School, 2010. All Rights Reserved.
2050
Recurrent renal disease
A. Primary Renal Disease
FSGS
MPGN I & II
IgA Nephropathy
Anti-GBM disease
Recurrent renal disease
B. Systemic Diseases
Primary Oxalosis
SLE
HUS
DM
Cystinosis
HSP
Amyloidosis
Mixed
cryoglobulinemia
Alports
Cystinosis
ANCA, Anti-GBM
Sickle cell
Fabrys disease
Copyright Harvard Medical School, 2010. All Rights Reserved.
2051
Disclosures
None
Copyright Harvard Medical School, 2010. All Rights Reserved.
2052
LATE POST TRANSPLANT MEDICAL
COMPLICATIONS
John P. Vella MD, FRCP, FACP, FASN
Director of Transplantation
Associate Professor of Medicine
Maine Medical Center, Portland, ME
Tufts University School of Medicine
Harvard Board Review Course
Aug 2010
Conflict of Interest
Disclosure
None
Copyright Harvard Medical School, 2010. All Rights Reserved.
2053
Brennan D et al. N Engl J Med 2006;355:1967-1977
Frequency of Adverse Events at 12 Months
Classification of Complications
Cardiovascular
Cancer
Musculoskeletal
Hematologic
Cosmetic
Neuro-psychi atric
Gastrointesti nal
Copyright Harvard Medical School, 2010. All Rights Reserved.
2054
Cardiovascular Complications
Complication Relationship
Hypertension Pretransplant Hypertension
Calcineurin Inhibitor Therapy
Cyclosporine>Tacrolimus
Corticosteroids
Dyslipidemia Pre-transplant dyslipidemia
Sirolimus>Cyclosporine>Tacrolimus
Corticosteroids
Diabetes Pretransplant diabetes mellitus
New Onset Diabetes After Transplantation
Tacrolimus> Cyclosporine
Underlying disease
Hepatitis C
ADPKD
Obesity
Non-Traditional Risk Factors Pretransplant cardiovascular disease
Allograft dysfunction
Oxidative Stress
Hyperhomocysteinemia
Vella. Complications after renal transplantation
In Therapy of renal diseases and related disorders. Saul and Massry 6
th
Edition 2007
CAC
Copyright Harvard Medical School, 2010. All Rights Reserved.
2055
Hypertension
Dyslipidemia
*P val ue < .05 versus comparator.
4
5
6
7
8
9
Baseline 1 2 3 4 6 9 12
Months
C
h
o
l
e
s
t
e
r
o
l
,

m
m
o
l
/
L
Placebo Azathioprine
Sirolimus 2 mg Sirolimus 5 mg
200
250
300
C
h
o
l
e
s
t
e
r
o
l
,

m
g
/
d
L
*
*
*
*
*
*
*
Copyright Harvard Medical School, 2010. All Rights Reserved.
2056
2007 ADR
Cumulative incidence of diabetes
Figure 7.55
First-time, kidney-only
transplant recipients, 1998
2002 combined; incidence
estimated from Cox
proportional hazards
models & adjusted for
multiple covariates as
described in Appendix A.
What is it about
transplantation that
provides CVD benefit in
spite of augmentation of
traditional risk factors by
immunotherapy?
Copyright Harvard Medical School, 2010. All Rights Reserved.
2057
Kidney Function
GFR is best (inverse) predictor of
hypertension after transplantation
AJT 2007
Transplantation and Inflammation
0
10
20
30
40
50
60
Pre Tx Post Tx, 1wk Post Tx, 1mo
pg/
mL
TNF-a
Si mmons Transpl antati on 2005;79
300
350
400
450
Pre Tx Post Tx, 1wk Post Tx, 1mo
mM
Protein Thiols
0.000
0.020
0.040
0.060
0.080
Pre Tx Post Tx, 1wk Post Tx, 1mo
nmol/mg
protei n
Protein Carbonyl s
Copyright Harvard Medical School, 2010. All Rights Reserved.
2058
Cancer after Transplantation
Complication Relationship
Skin Cancer
(SCC>BCC)
Immunotherapy
HPV
Sun exposure
Viral Mediated Post Transplant
Lymphoproliferative
Disease:EBV
Kaposi
Sarcoma:Human
Herpes Virus 8
Renal Cell Carcinoma Atrophic kidneys
Cumulative incidence of
malignancy
Figure 7.25
Monthspost-transplant
0 6 12 18 24 30 36
0
2
4
6
8
10
12
14
16
Cumulative incidence of malignancy
Any (non-skin)
Skin
First-time, kidney-only transplant recipients, 1995-2001 combined. Cumulative incidence of post-transplant
malignancy estimated from Coxproportional hazards models; adjusted for multiple covariates.
Copyright Harvard Medical School, 2010. All Rights Reserved.
2059
Skin Cancer
Incidence
20% at 10 yrs
60% at 20 years
Geography
UK
Australia
Mean time to presentation: 8yrs
Mean no. of cancers 3.3 per person after 20 yrs
immunotherapy
SCC>>BCC>>Melanoma
Squamous cell carcinoma
Second commonest skin cancer in general
Caucasian populations
Chronic sun exposure
Sunburns
Commonest on sun exposed areas
Preceded by solar keratosis
Metastatic potenti al - to regional lymph
nodes, then liver, lungs etc
Copyright Harvard Medical School, 2010. All Rights Reserved.
2060
PTLD
EBV+ 90%
CD20+ 70%
Polyclonal v. Monoclonal
Patient Age
Type of Transplant
Rx
Reduce immunosuppression
Gancyclovir
Rituxan
Chop
Irradiation (Intracranial)
Effect of Sirolimus on Kaposi's Sarcoma Skin Lesions
Stallone, G. et al. N Engl J Med 2005;352:1317-1323
Before 1-Mo after Conversion
Copyright Harvard Medical School, 2010. All Rights Reserved.
2061
Acquired Renal Cystic
Disease
Denton et al . Preval ence of renal cel l carci noma i n pati ents wi th
ESRD pre-transpl antation: a pathol ogi c anal ysi s. Ki dney Int. 2002
Jun;61(6):2201-9.
Retrospective review of native 260 nephrectomies
6 year period
Acquired renal cystic disease 33%
Renal Adenoma 14%
Renal Cell Carcinoma 4.2%
Oncocytoma 0.6%
On multivariable analysis, ARCD/AD associated
Male sex
Longer dialysis duration
Negatively associated with peritoneal dialysis.
Copyright Harvard Medical School, 2010. All Rights Reserved.
2062
Sirolimus and Malignancy
0
0.1
0.2
0.3
0.4
0.5
0.6
0.7
0.8
0.9
1
SRL SRL/CI CI
RR Cancer
P=0.01
USRDS 04
Hematologic complications of Transplantation
Complication Rel ati onshi p
Anemi a Anti body i nducti on therapy
Anti metabol ite
i mmunosuppressi on
ACE/ARB
Post Transpl ant Erythrocytosi s Endogenous erythropoi eti n excess
Leucopeni a Pol ycl onal anti body i nducti on
Anti-metaboli te
i mmunosuppressi on
Anti vi ral anti bi oti cs
CMV
Bactri m
Thrombotic Mi croangi opathy Cal ci neuri n Inhi bi tors (note for ed.
Presumabl y thi s wi l l be addressed
i n chapter 56?)
Copyright Harvard Medical School, 2010. All Rights Reserved.
2063
Anemia after Transplantation
Graft dysfunction
Immunotherapy
Antiviral therapy
ACE/ARB
Infection
Inflammation
Marrow fibrosis (PTH)
Fe Deficiency
Acidosis
Down regulation of Erythropoiesis Gene Cluster
Hb transcription/synthesis
Iron/Folatebinding and transport
Incidence
Mean HCT
33% at 1 month
40% at 12 months
Proportion of patients with Hct<36
76% at transplantation
21% at 1 yr
36% at 4 yrs
More prevalent in women
Investigated: 36% of pts with Hct<30
46% received Iron
40% received ESA
Mix TC, Kazmi W, Khan S, Ruthazer R, Rohrer R, Pereira BJ , et al. Anemia: a continuing problemfollowing kidney
transplantation. AmJ Transplant. 2003 Nov;3(11):1426-33.
Copyright Harvard Medical School, 2010. All Rights Reserved.
2064
TRansplant European Survey on
Anemia Management (TRESAM)
4263 pts @ 72 transplant centers in 16 countries
39% of patients were found to be anemic
8.5% of patients who were considered severely
anemic
Only 18% were treated with erythropoietin
There was a strong association between hemoglobin
and graft function
Serum creatinine > 2 mg/dL, 60.1% were anemic
29.0% of those with serum creatinine <or= 2 mg/dL (p < 0.01).
Post Transplant Erythrocytosis
(PTE)
Definition: HCT> 51% persisting after renal
transplantation
Incidence: 10% to 15% of graft recipients
Timing: 8 to 24 months after engraftment
Cause: incompletely understood
inappropriately high levels of erythropoietin.
increase in AT1R in erythroid precursors which correlates
with
hematocrit
ACE inhibitors and ARBs can effectively managing
PTE .
Copyright Harvard Medical School, 2010. All Rights Reserved.
2065
PTE Effects
Symptoms: 60%
Malaise
Plethora
Headache
Lethargy
Dizziness
Complications
Thromboembolic 10% to 30%
Death 1% to 2%
Therapy:
Reduce the hematocrit to less than 50%
ACEI or ARB
Periodic therapeutic phlebotomy
Leucopenia
Associati on:
Immunotherapy
Antiviral Therapy
Increased incidence of infections
Aza>MMF
Valganciclovir>Ganci cl ovir
Copyright Harvard Medical School, 2010. All Rights Reserved.
2066
Hemophagocytic syndrome
Syndrome
Fever
Hepatosplenomegaly
Pancytopenia
Hypofibrinogenemia
Liver dysfunction
Definition:
Bone marrow/organ infiltration
Activated, nonmalignant macrophages that
Phagocytoze red blood cells
Causation:
Infection
Cancer
Associations
17 cases of HPS after renal transplantation
Median time 52 days
64% had received ALGs
Viral infection in 9 patients
CMV, EBV, HHV6, HHV8
Bacterial infection in 3 patients
TB, Bartonella henselae
Other infections in 2 patients
Toxoplasmosis, PCP
PTLD 2 patients
Mortality 47%
Graft nephrectomy was performed in 4 of the 9
survivors
Copyright Harvard Medical School, 2010. All Rights Reserved.
2067
Musculoskeletal complications of
Transplantation
Complication Relationship
Osteopenia/
Osteoporosis
Pre-transplant Osteoporosis
Cyclosporine
Steroids
Osteoarticular pain Marrow Edema
CI
Osteitis Residual hyperparathyroidism
Adynamic bone
disease
Over-treatment of osteodystrophy
Tendonitis Quinolone antibiotics (high dose)
Percent change in lumbar spine density
Adapted from Jullian BA et al. N Engl J Med 1991;325:544-50.
0 6 18
0
-3
-6
-9
-12
C
h
a
n
g
e

i
n

B
o
n
e

D
e
n
s
i
t
y

(
%
)
Months after Transplantation
Female
Male
All
*
*
*
*
*
*
Copyright Harvard Medical School, 2010. All Rights Reserved.
2068
Contributing Factors
Genetic predispositi on
Age
Gonadal insufficiency
Immunosuppressi ve drugs
Prednisone
Cyclosporine
Underlying renal osteodystrophy
Persistent hyperparathyroi di sm
0
5
10
15
20
25
Foot Arm Leg Ribs Hip Spine Fingers Pelvis Wrist
N
u
m
b
e
r

o
f

F
r
a
c
t
u
r
e
s
Type of Fracture
Kidney
Liver
Kidney/Pancreas
Heart
Adapted from Ramsey-Goldman R et al. J Bone Miner Res 1999;14:456-63.
Copyright Harvard Medical School, 2010. All Rights Reserved.
2069
Treatment
BMD
Osteopeni a
T= -1 to -2.5
Osteoporosi s
T= < -2.5
1200 mg Ca
2+
/day
400-800mcg vit D/day
Exercise
Weight training
?hormone replacement
?vitamin D analogs
Bisphosphonates
Yearly BMD
T= -1 to -2.5
Treatment with biphosphonates
Cruz et al. Am J Transplant 2002;2:62-7.
Copyright Harvard Medical School, 2010. All Rights Reserved.
2070
Caution with Biphosphonates
Avoid in states of low bone turnover
GI toxicity
Myalgias
Watch as GFR decreases (avoid with GFR
< 30 ml/min)
Jaw Osteonecrosi s
Dental procedure
Non healing
No treatment
Neuro-psychi atric complications of
Transplantati on
Complication Relationship
Tremor Tacrolimus>Cyclosporine
Depression
Mania
Anxiety
Corticosteroids
-blockers
Delirium Polypharmacy
Infection
Non-adherence Medication adverse effects
Psychiatric disease
Education
Recipient age
Visual changes Cataracts
Diabetes
Steroid induced retinopathy
Infection
Copyright Harvard Medical School, 2010. All Rights Reserved.
2071
Psychiatric Disturbances
Anxiety
Depression
Early
Associated with acute/chronic rejection
Common in Heart Transplant Recips
Pretranspl ant psychi atri c hi story
Poor soci al support
The use of avoi dance copi ng strategi es for managi ng heal th
probl ems
Low sel f-esteem.
Non-adherence with diet or medications
Sexual dysfunction
Suicide
Crude suicide rate in kidney transplant recipients
1995-01
24 per 100,000 patient-years
84% higher than the general population
Significant risk factors by multivariate analysis:
age greater than 75 years
male gender
white or Asian race
geographic region
alcohol or drug dependence
recent hospitalization with mental illness
Kurella et al. Suicide in the United States end-stage renal disease program
J Am Soc Nephrol. 2005 Mar;16(3):774-81
Copyright Harvard Medical School, 2010. All Rights Reserved.
2072
Non-adherence (or non-compliance)
Major risk factor for graft rejection episodes
RR 4.2 in non compliant patients
Responsible for up to 25% of early deaths after the
initial recovery period
Risk factors implicated include:
Adverse effects of medications
Lack of knowledge concerning the need for
medications,
Financial concerns
Non-Adherence
Risk Factors (cont)
Younger and older age
Non-married
Anxiety disorder
Denial
Personality disorders
Mental retardedation
Substance abuse
Transplant center
Dosing frequency
Copyright Harvard Medical School, 2010. All Rights Reserved.
2073
Gastrointestinal complications of
Transplantation
Compl i cati on Rel ati onshi p
Gastri tis/PUD Stress
Corti costeroids
CMV
Hepati ti s Non-i nfecti ous
HBV/HCV
Stati n therapy
Aml odi pine
Azathi opri ne
Di arrhea Mycophenol ate
Si rol i mus
Proton pump i nhi bi tors
Stomatitis Non i nfecti ve Si rol i mus
Candi da
MMF & Diarrhea
40% of patients
Dose related (3 versus 2 grams per day)
Altering the drug dose or frequency often
ameliorates symptoms
Under-dosing or disconti nuing MMF increase
the RR rejection 2.3-2.7
The PDR: MPA -v- MMF
constipation 38 versus 39%
nausea 29 versus 27%
diarrhea 23 versus 24%
vomiting 23 versus 20%)
Copyright Harvard Medical School, 2010. All Rights Reserved.
2074
SRL & GI Effects
Adverse GI effects
Abdominal pain (28-36%)
Nausea (25-36%)
Vomiting (19-25%)
Diarrhea (25-42%). The incidence of diarrhea
seems to be higher when sirolimus is used in
the context of calcineurin inhibitor withdrawal
or as adjunctive therapy with MMF
Stomatitis and mouth ulcers 30%
COSMETIC ADVERSE EFFECTS
Hair growth/loss
Weight gain
Gingival Hyperplasi a
Acne
Copyright Harvard Medical School, 2010. All Rights Reserved.
2075
Cosmetic complications of
Transplantation
Complication Relationship
Edema Drugs
Dihydropyridine CCBs
Sirolimus
Thiazolidinediones
Allograft
Proteinuria
Allograft dysfunction
Lymphocele
Gingival Hyperplasia Dihydropyridine Calcium Channel
Blockers
Cyclosporine
Weight Gain/Cushingoi d appearance Corticosteroids
Acne
Warts
Skin Cancers
Corticosteroids
Sirolimus*
Azathioprine
Differences in patient and transplant professional perceptions of
immunosuppression-induced cosmetic side effects
Peters et al. Transplantation. 2004 Aug 27;78(4):537-43
Reported by 80% of patients
Patient reporting exceeded physician reporting
Most patients are " happy to endure" changes " for
the sake of having a transplant
Patients experience emotional and social effects
due to physical changes
Doctors recommended treatment for less than
half of the affected patient
When recommended therapy changes were
pursued, treatments were effective
Copyright Harvard Medical School, 2010. All Rights Reserved.
2076
Wolfe et al. NEJM 1999:341
DAYS SINCE TRANSPLANTATION
106 183 244 365 546 0
0.25
0.32
1.00
2.84
4.00
RISK
EQUAL
R
E
L
A
T
I
V
E

R
I
S
K

O
F

D
E
A
T
H
Bibliography
NephSAP
1. Bennett W, Vella JP. Transplantation NephSAP. J Am Soc Nephrol Vol 4, No.1: Jan
2005
2. Vella JP, Danovitch GD. Transplantation NephSAP. J Am Soc Nephrol Vol 5, No 4:
July 2006
3. Vella JP, Danovitch GD. Transplantation NephSAP. J Am Soc Nephrol Vol 7, No. 1
Jan 2008
4. Vella JP, Cohen D. Transplantation NephSAP J Am Soc Nephrol Vol 8, No. 6: Nov
2009
Book chapters
1. Vella JP Complications after renal transplantation. In Therapy of renal diseases and
related disorders. Saul and Massry 6
th
Edition 2007
2. Vella JP and Sayegh MH. Diagnosis and Management of Renal Allograft
Dysfunction. In: Brady HR, Wilcox CS, Brenner BM, eds. Current Therapy in
Nephrology and Hypertension. Philadelphia: W.B. Saunders, 2009
3. Grafals M, Chandraker A, Vella JP. Noninfectious Complications in Renal
Transplant Recipients. Himmelfarb J, Sayegh MH Eds. Chronic Kidney Disease,
Dialysis, and Transplantation, Third Edition Elsevier 2009
4. Vella JP Approach to Medical Complications after Kidney Transplantation. Core
Concepts in Renal Transplantation, Singh, Sayegh, and Chandraker, eds. 2010 in
Press
Copyright Harvard Medical School, 2010. All Rights Reserved.
2077
A 17 yr. old male presents 11 months post transplantation with fever
and and left sided submandibular lymphadenopathy. A biopsy reveals
polyclonal B cell proliferation. The CD20 stain is positive and the EBV
PCR is positive. There is no evidence of metastatic disease by FDG-
PET scanning.
Which ONE of the following statements about management of PTLD is
most CORRECT?
1.The immunosuppression needs to be stopped immediately in order to
prevent death from disseminated lymphoma
2.No therapy is indicated as the disease is very limited
3.Antiviral therapy alone is likely to be effective
4.Antiviral therapy combined with reduced immunosuppressionis
reasonable initial therapy
5.An aggressive regimen with rituximab/CHOP should be initiated
without delay
A 64yr. old female presents 4 months post with a total cholesterol of
340 mg/dl and LDL cholesterol of 189mg/dl. The triglycerides are
480mg/dl
Which ONE of the following statements about the treatment of
dyslipidemia after transplantation is most CORRECT?
1.The choice of immunosuppression is unlikely to impact post
transplant dyslipidemia
2.No therapy is indicated as statins have been shown not to impact
mortality in patients with ESRD
3.Pravastatin is the treatment of choice as it will also lower the risk of
rejection
4.Therapeutic life style modification is likely to be sufficient to lower the
lipids to goal
5.The use of high potency statin therapy is most likely to be effective in
lowering the lipids to target
Copyright Harvard Medical School, 2010. All Rights Reserved.
2078
Conflict of Interest
Disclosure
None
Copyright Harvard Medical School, 2010. All Rights Reserved.
2079
Infection in Transplant
Recipients
Brigham Renal Board Review Course
August 13, 2010
Sarah P Hammond MD
Instructor in Medicine
Brigham and Womens Hospital
Harvard Medical School
Disclosures
Research funding via NIH-SBIR grant
with Idaho Technology, Inc.
I will discuss the use of medications for
non-FDA approved indications
Copyright Harvard Medical School, 2010. All Rights Reserved.
2080
Overview
Basic Principles
Important pathogens and treatment
Cytomegalovirus (CMV)
BK virus (BKV)
Hepatitis C virus (HCV)
Fungi
Summary
Extra practice questions
Basic Principles
Copyright Harvard Medical School, 2010. All Rights Reserved.
2081
Infection in transplant recipients
Transplant recipients are susceptible to
opportunistic infections and community-
acquired infections
Infection can result from exposure to a low number
of organisms
Canary in a coal mine
The inflammatory response to infection is
suppressed in transplant recipients
Attenuated signs and symptoms of infection
Transplant recipient with infection typically has
high burden of organisms once infection
established
Fishman J A, Rubin, RH. N Engl J Med. 1998;338:1741-51.
Fishman J A. N Engl J Med. 2007;357:2601-14.
Basic Principles
In 1998 Fishman and Rubin described
important principles for transplant recipient
with infection
Net state of immunosuppression
Dose and duration of suppressive medications
Mechanical factors (eg. lymphatic compromise)
Infections contributing to compromise (eg. CMV)
Important epidemiologic exposures
New exposures
Remote exposures with possibility of reactivation
Fishman J A, Rubin, RH. N Engl J Med. 1998;338:1741-51.
Fishman J A. N Engl J Med. 2007;357:2601-14.
Copyright Harvard Medical School, 2010. All Rights Reserved.
2082
Case Presentation #1
54 year old man s/p renal transplant 5 years ago
presents with 2 days of fever and body aches
Past medical history: HTN, hypercholesterolemia,
PKD s/p renal transplant 2005 (baseline Cr 1.2)
Donor CMV-/recipient CMV+
No episodes of rejection
Medications
Tacrolimus
Mycophenolate mofetil
Lisinopril
Atorvastatin
Case Presentation #1 (cont)
Social history: Lives with his wife and 5
year old son in Rhode Island; works for
local sanitation department
Vital signs: T 100.5 P 85 BP 127/74
Exam is notable for faint crackles at the
left base and possible mild tenderness
over the allograft
Copyright Harvard Medical School, 2010. All Rights Reserved.
2083
Case Presentation #1 (cont)
The least likely cause of his illness is:
A. Influenza
B. Viral upper respiratory tract infection
C. CMV
D. Community-acquired pneumonia
E. Pyelonephritis
Timing of Infection after Transplant
Fishman J A, Rubin RH. N Engl J Med. 1998:338;1741-51.
Fishman J A. N Engl J Med. 2007;357:2601-14.
Time after
transplant
0-1 Month 1-6 Months* >6 Months
Type of
infection
Nosocomial
MOSTLY BACTERIA OR
CANDIDA
Cather-related UTI
Line infection
Pneumonia
Procedure-
related
MOSTLY BACTERIA OR
CANDIDA
Wound infection
Urine leak
Donor-derived
RARE
Opportunistic
VIRAL
CMV
BK
VZV
BACTERIAL
Nocardia
Listeria
FUNGAL
PCP
Aspergillus
PARASITIC
Strongyloides
Toxoplasma
Community-
acquired
VIRAL
Influenza
HPV
PTLD (EBV)
BACTERIAL
S. pneumoniae
Listeria
Rhodococcus
FUNGAL
Cryptococcus
Endemic fungi
* Patients treated for rejection after 6 months are also at
risk for these infections
Copyright Harvard Medical School, 2010. All Rights Reserved.
2084
Important Pathogens
Case Presentation #2
57 year old man presents with 10 days of abdominal
pain, nausea, and anorexia
Also endorses low grade fever (100.5)
Denies other symptoms including SOB, diarrhea, dysuria
Medical history
ESRD secondary to HTN
S/p deceased donor renal transplant 8 months ago
CMV donor+/recipient-, received 6 months prophylaxis
Medications
Mycophenolate mofetil
Tacrolimus
Prednisone
Trimethoprim-sulfamethoxazole
Metoprolol
Copyright Harvard Medical School, 2010. All Rights Reserved.
2085
Case Presentation #2 (cont)
Exam
Tm 100.5, Tc 100.2, P 72, BP 110/52, O2
sat 98% RA
Epigastric tenderness to palpation without
rebound
Labs
Cr 1.4 (baseline), ALT 55, AST 62, Tacro 7.7
WBC 2.8, HCT 30.9, PLT 198
Case Presentation #2 (cont)
Initial work up
Blood cultures and urine culture are
negative
Abdominal CT scan: atrophic native kidneys
EGD shows mild
esophagitis with
multiple small shallow
ulcerations in gastric
antrum
Copyright Harvard Medical School, 2010. All Rights Reserved.
2086
Case Presentation #2 (cont)
The most likely diagnosis is:
A. EBV-associated post-transplant
lymphoproliferative disorder
B. Peptic ulcer disease
C. CMV esophagitis and gastritis
D. Helicobacter pylori infection
E. Disseminated BK virus infection
Cytomegalovirus (CMV)
Copyright Harvard Medical School, 2010. All Rights Reserved.
2087
CMV and Renal Transplantation
Post-transplant CMV reactivation associated with direct
and indirect effects leading to increased morbidity
Direct effects
Asymptomatic CMV infection
Viremia can lead to symptomatic disease
Symptomatic CMV disease
CMV syndrome (common)
Viremia, leukopenia, malaise, fever, +/- transaminitis
Gastrointestinal disease (common)
Colitis, gastritis, esophagitis, +/- viremia
Pneumonitis (rare)
Retinitis (very rare)
Indirect effects
Increased susceptibility to other infections
Possible increase in risk of rejection
Risk of EBV-associated post-transplant lymphoproliferative disorder
Fishman J A. N Engl J Med. 2007;357:2601-14.
Diagnosis of CMV Reactivation
CMV Serologies
Should be checked in recipient and donor before transplant to
determine risk for CMV after transplant
Not helpful for determining if CMV is present post-transplant
Quantitative nucleic acid (virus load) testing
PCR-based assay that detects CMV virions in blood or serum
Results given as copies per mL
pp65 antigenemia testing
Quantitative measure of WBCs that stain for CMV in whole
blood
Results given as number of positive cells per high power field
Tissue diagnosis
Method used for definitive diagnosis of tissue-invasive disease
Visualization of inclusion bodies
Identification of antigens by immunohistochemical staining
Kotton CN, et al. Transplantation. 2010;89:779-95.
Copyright Harvard Medical School, 2010. All Rights Reserved.
2088
Risk of CMV Reactivation
Transplant recipients who are seropositive for CMV or
receive transplant from a seropositive donor are at risk
Highest risk: Donor CMV IgG+/Recipient CMV IgG-
40-58% of D+/R- renal transplant recipients develop reactivation
Negligible risk: Donor CMV IgG-/Recipient CMV IgG-
Note D-/R- recipients remain at risk for HSV/VZV
Timing of risk
In absence of CMV prophylaxis, risk is highest in first 3-6
months post transplant
In patients who receive prophylaxis, CMV risk is highest in
first 3-6 months after prophylaxis ends
Delayed-onset CMV infection
CMV risk also high in first few months after treatment of
rejection unless prophylaxis is reinstituted
Arthurs SK, et al. Clin Infect Dis. 2008;46:840-6.
Prevention of CMV Reactivation
Universal prophylaxis
Treatment of all patients who are CMV IgG+ or received
transplant from CMV IgG+ donor with antivirals for 3-6
months
Prevents early reactivation and the indirect effects of CMV
Associated with delayed-onset CMV
Particularly high risk among D+/R- recipients
In a study, of 176 D+/R- recipients, 51 (29%) developed CMV after
stopping prophylaxis
Preemptive therapy
Routine monitoring with either CMV virus load or pp65 CMV
antigen weekly to biweekly for first 6 months after transplant
Ideally patients who are CMV virus load or antigen positive
are treated before CMV disease develops
Associated with decreased antiviral toxicities and cost, but
may not prevent the indirect effects of CMV
Arthurs SK, et al. Clin Infect Dis. 2008;46:840-6.
KottonCN, et al. Transplantation. 2010;89:779-95.
Copyright Harvard Medical School, 2010. All Rights Reserved.
2089
Case Presentation #2 (cont)
The patient is treated with valganciclovir and
discharged home
He presents to clinic 2 weeks later for routine
follow up
He reports improvement in his abdominal pain
and no fevers; generally feeling well
Physical exam: Unremarkable
Labs
WBC 1.2 (54N, 40L, 5M), HCT 31.4, PLT 215
Cr 1.35, LFTs normal
Case Presentation #2 (cont)
Worsening leukopenia is likely due to:
A. Worsening CMV infection
B. Drug-drug interaction between prednisone and
trimethoprim-sulfamethoxazole
C. Valganciclovir-resistant CMV infection
D. On going mycophenolate therapy
E. The recent addition of valganciclovir to
the medication regimen
Copyright Harvard Medical School, 2010. All Rights Reserved.
2090
CMV treatment
Ganciclovir and valganciclovir are the
mainstay of treatment and prophylaxis for
CMV infection
Ganciclovir is a guanosine analogue that inhibits
CMV replication
Available intravenously and orally
Bioavailability of oral formulation is poor ~5%
Renal clearancedose adjustment necessary
Valganciclovir is an oral pro-drug of ganciclovir
Significantly better oral bioavailability (~60%)
Renal clearancedose adjustment necessary
Both FDA approved for CMV prophylaxis in
transplant recipients
Treatment of CMV is an off-label use
BironKK. Antiviral Research. 2006;71:154-63.
CMV treatment (cont)
Ganciclovir and valganciclovir have the same side-
effect profile
Dose-dependent leukopenia is common
Neutropenia, anemia, and thrombocytopenia can also occur
When leukopenia due to valganciclovir develops
Treatment dose should not be reduced or stopped
Dose should only be adjusted for changes in renal function
CMV treatment/prevention with oral ganciclovir or
low-dose valganciclovir or can lead to resistance
Treatment options for ganciclovir-resistant CMV
include off-label foscarnet or cidofovir
Both are nephrotoxic and only available by IV
In case series of 4 renal transplant recipients with proven
resistant CMV, 2 (50%) had graft loss due to treatment toxicity
KottonCN, et al. Transplantation. 2010;89:779-95.
EidAJ , et al. Clin Transplant 2008;22:162-70
Copyright Harvard Medical School, 2010. All Rights Reserved.
2091
BK Virus Infection
In a patient with a history of BK virus
nephropathy now considering retransplantation
A. The patient needs a transplant nephrectomy
B. There is no risk of recurrent BK infection after
retransplantation
C. The patient needs to be treated with
leflunomide after retransplanatation
D. The patient should have screening for
BK viremia after retransplantation
E. The risk of BK virus-associated hemorrhagic
cystitis is high after retransplantation
Copyright Harvard Medical School, 2010. All Rights Reserved.
2092
BK Virus
BK virus is a polyoma virus
60-80% of adults are seropositive worldwide
Primary infection likely occurs during childhood
J C virus is other clinically important polyoma virus
Causes majority of polyoma virus-associated
nephropathy (PVAN) in renal transplant
recipients
Small number of cases thought to be due to J C
virus
Also associated with hemorrhagic cystitis in
stem cell transplant recipients in early post
transplant period
Ramos E, et al. Transplantation. 2009;87:621-30.
Blanckaert K, DeVriese AS. Nephrol Dial Transplant. 2006;21:3364-7.
Polyoma virus-associated nephropathy
Estimated prevalence of PVAN after renal transplant is
1-10%
In absence of intervention, may result in up to 50% graft loss
Some studies have focused on risk with specific
immunosuppressive regimens
Risk is likely highest with greater immunosuppression,
regardless of agent
Diagnosed histologically on renal biopsy and with
immunohistochemical stains
Early PVAN has patchy involvement and may be missed on
biopsy
Development of PVAN typically preceded by first
development of persistent high BK viuria followed by
persistent viremia
Ramos E, et al. Transplantation. 2009;87:621-30.
Blanckaert K, DeVriese AS. Nephrol Dial Transplant. 2006;21:3364-7.
Copyright Harvard Medical School, 2010. All Rights Reserved.
2093
Natural History of PVAN
Vr viuria; Vm viremia; Cr serum creatinine; Bx+ positive biopsy
Ramos E, et al. Transplantation. 2009;87:621-30.
Treatment and prevention of PVAN
Best studied treatment for PVAN is reduction in
immunosuppression
Other treatments including cidofovir, leflunomide,
fluoroquinolones, and IVIg have been examined in
case series and cohort studies
No definitive evidence that any of these are efficacious
Randomized controlled trials needed
Early recognition and treatment thought to portend
better outcomes
Prospective regular screening of renal transplant recipients for
BK viremia has become standard at most centers
Those with significant viremia can be preemptively treated
J ohnston O, et al. Transplantation. 2010;89:1057-70.
Ramos E, et al. Transplantation. 2009;87:621-30.
Blanckaert K, DeVrieseAS. Nephrol Dial Transplant. 2006;21:3364-7.
Copyright Harvard Medical School, 2010. All Rights Reserved.
2094
Hepatitis C Virus
Case Presentation #3
36 year old woman with a history of ESRD
secondary to HCV-associated
glomerulonephritis s/p renal transplant 24
months ago admitted with worsening renal
function
Creatinine increased from 1.1 at baseline to 3.2
Recent 4 day bout of diarrhea
No dysuria or hematuria
PMH: renal transplant as above, active HCV
(failed treatment pre-transplant)
Copyright Harvard Medical School, 2010. All Rights Reserved.
2095
Which of the following is true of a patient
with HCV undergoing renal transplant?
A. The risk of chronic allograft
nephropathy is increased in transplant
recipients with HCV
B. The higher the HCV virus load, the higher the
risk for cirrhosis and HCV-associated renal
disease in the graft
C. Risk of death is higher after renal transplant
than if the patient remained on dialysis
D. There is no risk for recurrence of HCV-
associated glomerulonephritis after transplant
E. The risk of CMV is increased in transplant
recipients with HCV
Case Presentation #3
Renal biopsy showed acute tubular
necrosis (likely related to dehydration
after recent diarrheal illness) and
mesangioproliferative glomerulonephritis
consistent with recurrence of HCV-
associated disease
Copyright Harvard Medical School, 2010. All Rights Reserved.
2096
HCV and Renal Transplantation
Multiple studies have assessed outcomes in HCV
infected renal transplant recipients
Mixed results in terms of graft survival and mortality
Among 133 renal transplant recipients mortality and
graft survival were similar in 87 HCV RNA+ vs. 46
HCV RNA-
HCV RNA+ group split by normal/abnormal ALT; abnormal
ALT HCV+ group had significantly poorer graft survival and
higher mortality
HCV RNA+ group had significantly more chronic allograft
nephropathy
Survival of HCV infected patients with ESRD is
better after transplantation than on continued
dialysis
Avery, R, et al. Am J Transplant 2004;4(Suppl.10):72-82.
Mahmoud IM, et al. Am J Kidney Dis 2004;43:131-9.
HCV and Renal Transplantation
HCV virus load does not correlate with the
degree of liver damage due to HCV
Liver biopsy is the only way to reliably assess
degree of liver damage
Current standard treatment for HCV includes
interferon-alpha and ribavirin
Theoretically post-transplant use of interferon-
alpha may promote acute rejection
Treatment response can be predicted by HCV
genotype
Copyright Harvard Medical School, 2010. All Rights Reserved.
2097
Fungal infection
Case Presentation #4
65 year old man presents with fevers to 102 F,
malaise, nausea, and mild abdominal pain
Past Medical History
ESRD due to lithium toxicity, s/p CRT 4 years ago
Bipolar affective disorder
Anxiety
AF
Medications
Tacrolimus, mycophenolate, prednisone, olanzapine,
lorazepam, verapamil, warfarin
Copyright Harvard Medical School, 2010. All Rights Reserved.
2098
Case Presentation #4 (cont)
Vital signs: Tc 99.7 F, P 87, BP 112/75, O2 sat
98% RA
Physical exam: Overweight older man in no
acute distress; cardiac exam irregularly irregular;
abdomen non-tender, non-distended; skin
without rash
Labs: BUN 13, Cr 0.8, ALT 50, AST 46, WBC 8.0,
HCT 41.3, PLT 249, INR 2.1
Radiology: Abdominal CT unremarkable
Blood cultures: 2/4 bottles grow yeast
Case Presentation #4 (cont)
Based on the information given
possible pathogen(s) is/are:
A. Candida sp.
B. Aspergillus sp.
C. Rhizopus sp.
D. Cryptococcus sp.
E. A, B or C
F. A or D
Copyright Harvard Medical School, 2010. All Rights Reserved.
2099
Fungal Infection and Transplant
TRANSNET survey tracked invasive fungal infection in
transplant recipients at 15 US centers 2001-06
Cumulative incidence of fungal infection in renal transplant
recipients in first 12 months after transplant is 1.3%
Most common fungal infections in renal transplant recipients:
Candida (49%) (YEAST)
Cryptococcus (15%) (YEAST)
Aspergillus (14%) (MOLD)
Endemic mycoses (10%) (DIMORPHIC yeast in laboratory)
Candida infection is often an early post-transplant complication
Related to the procedure and/or hospitalization after the procedure
Epidemiology of fungal infection is geographically
dependent
Zygomycosismost common fungal infection post renal transplant in Iran
Coccidioidomycosisis an important cause of fungal infection after renal
transplant in Arizona-- estimated incidence of 3.8-6.9%
Pappas PG, et al. ClinInfect Dis. 2010;50:1101-11.
Einollahi B, et al. Ann Transplant. 2008;13:55-8.
Blair J E, Logan J L. ClinInfect Dis. 2001;33:1536-44.
Case Presentation #4 (cont)
Yeast identified as Cryptococcus neoformans
Serum cryptococcal antigen 1:1024
The patient is initially treated with liposomal
amphotericin
Fevers dissipate
Lumbar puncture is normal with no fungal
forms on stain and negative CSF cryptococcal
antigen
The patient is switched to fluconazole 400 mg
po daily at discharge
Copyright Harvard Medical School, 2010. All Rights Reserved.
2100
Cryptococcus and Transplant
Cryptococcosis is often seen late after transplant
Median time to cryptococcosis83-85 months after transplant
Spectrum of cryptococcal disease includes
Fungemia(+/- local foci of infection)
Meningitis
Isolated pulmonary infection
Skin/soft tissue/osteoarticular infection (uncommon)
Pulmonary infection has become more common since
2000 and CNS/disseminated infection less common
Asymptomatic pulmonary cryptococcal infection is common
Treatment of cryptococcosis
Lipid formulations amphotericin or fluconazoleare indicated
depending upon site of infection
5-flucytosine used with lipid amphotericin in CNS disease
Sun HY, et al. ClinInfect Dis. 2009;48:1566-76.
Pappas PG, et al. presented at 47th ICAAC, 2007; M-1195.
Mueller NJ , Fishman J A. Transpl Infect Dis. 2003;5:140-3.
Case Presentation #4 (cont)
The patient presents to clinic for follow
up one week later
No major complaints except that his hands
are tremulous
Can no longer write legibly
No further fevers
Labs: BUN 31, Cr 1.97, ALT 28, AST 30,
WBC 5.3, HCT 41.8, PLT 240, INR 4.1
Copyright Harvard Medical School, 2010. All Rights Reserved.
2101
Which of the following is correct?
A. The tremor and worsening renal function are
probably due to undiagnosed cryptococcal
meningitis
B. The worsening renal function is probably due
to rejection stimulated by the recent infection
C. The worsening renal function and
tremor are probably due to the addition
of fluconazole to his regular
medications
D. The worsening renal function is probably due
to new BK nephropathy because one
opportunistic infection leads to another
Azole-drug interactions
Azoles inhibit hepatic enzymes (CYP3A4, CYP2C9, CYP2C19)
This leads to increased concentrations of many
medications metabolized by these enzymes
Tacrolimus, Cyclosporin
Sirolimus
Statins
Some calcium channel blockers (diltiazem, verapamil)
Some benzodiazepines
Some opiates
Warfarin
Initiation or cessation of any azole drug in a patient on
immunosuppressants requires dose adjustment and
close therapeutic drug monitoring
Copyright Harvard Medical School, 2010. All Rights Reserved.
2102
Approximate dose reductions*
Immuno-
supressant
----------------
Azole
Cyclosporin
dose
reduction
Tacrolimus
dose
reduction
Sirolimus
dose
reduction
Fluconazole
~50% ~50-65% **
Voriconazole
~50% ~65-75% ~90%
Posaconazole
~30% ~33-50% ~50%
Cervelli MJ . Transplantation. 2002;74;1477-8.
Pai MP, Allen S. Clin Infect Dis 2003;36:1089-92.
* DISCLAIMER: Based in part on personal observation;
all patients require CLOSE monitoring
Marty FM, et al. Blood 2006;12:552-9
Morris MI. Am J Health-Syst Pharm2009;66:225-36.
Koo S, et al. ICAAC 2009; A1-592.
Summary
Copyright Harvard Medical School, 2010. All Rights Reserved.
2103
Summary Basic concepts
When evaluating transplant recipient with
infection, timing of infection in relation to
transplantation is essential
Infection first month after transplant
Mechanical (related to transplant procedure)
Nosocomial
Donor-derived (rare)
Infection 1-6 months after transplant or after
treatment of rejection
Opportunistic
Community-acquired
Infection more than 6 months after transplant
Community-acquired
Summary Important pathogens
CMV is significant cause of morbidity after
transplant
At centers where prophylaxis is routine, late CMV
infection/disease occurs
CMV also common after treatment of rejection
BK virus PVAN is a significant cause of graft failure
Transplant recipients should be routinely monitored for BK
viremia
Those with significant viremia or biopsy-proven PVAN should
have reduction in immunosuppression
Utility of adjunctive medications (quinolones, leflunomide,
cidofovir) has not been proven in randomized trials
Fungal infection is relatively uncommon after renal
transplant
Candida sp. & Cryptococcus sp. are most common pathogens
Copyright Harvard Medical School, 2010. All Rights Reserved.
2104
Summary Treatment issues
Treatment and/or prevention of infection in
transplant recipients can be complicated by
significant drug-drug interactions and drug
toxicities
Azole antifungals have significant drug-drug
interactions with immunosuppressants and other
common medications
Initiation or cessation of an azole necessitates calcineurin
and mTOR inhibitor dose adjustment and monitoring
Some antiviral medications may have significant
toxicities
Some drug toxicities may be synergistic with
immunosuppresant toxicities
Leukopenia with concomitant valganciclovir and
mycophenolate
Selected References
Fishman J A, Rubin, RH. Infection in organ-transplant
recipients. N Engl J Med. 1998;338:1741-51.
Fishman J A. Infection in solid-organ transplant
recipients. N Engl J Med. 2007;357:2601-14.
Kotton CN, et al. International consensus guidelines on
the management of cytomegalovirus in solid organ
transplantation. Transplantation. 2010;89:779-95.
Pappas PG, et al. Invasive fungal infections among
organ transplant recipients: results of the transplant-
associated infection surveillance network (TRANSNET).
Clin Infect Dis. 2010;50:1101-11.
Ramos E, et al. The decade of polyomavirus BK-
associated nephropathy: state of affairs.
Transplantation. 2009;87:621-30.
Copyright Harvard Medical School, 2010. All Rights Reserved.
2105
Disclosures
Research funding via NIH-SBIR grant
with Idaho Technology, Inc.
I discussed the use of medications for
non-FDA approved indications
Extra Practice Questions
Copyright Harvard Medical School, 2010. All Rights Reserved.
2106
Case Presentation #5
50 year old woman presents with shaking
chills, fever to 102 F, dyspnea on exertion,
and left-sided pleuritic chest pain
Past Medical History
ESRD due to HTN, s/p CRT 6 and 1/2 months ago,
CMV D-/R-; course complicated by delayed graft
function
HTN
Medications
Tacrolimus, prednisone, mycophenolate,
metoprolol, lisinopril
Case Presentation #5 (cont)
Vital signs: Tc 99.0 F, P 87, BP 112/75,
O2 sat 98% RA
Physical exam: Well-appearing middle
aged woman in no acute distress;
crackles at the left base
Labs: BUN 25, Cr 1.5, LFTs normal, WBC
7.1 (94%N, 4%L, 5%M)
Radiology: Chest X-ray with nodular
infiltrate LLL
Copyright Harvard Medical School, 2010. All Rights Reserved.
2107
Case Presentation #5 Chest CT
Which of the following is the most
appropriate?
A. Blood cultures and sputum culture
B. Empiric levofloxacin
C. Empiric valganciclovir
D. Serum galactomannan, beta-d-glucan, and
cryptococcal antigen
E. A and C
F. A, B and D
G. All of the above
Copyright Harvard Medical School, 2010. All Rights Reserved.
2108
Timing of Infection after Transplant
Fishman J A, Rubin RH. N Engl J Med. 1998:338;1741-51.
Fishman J A. N Engl J Med. 2007;357:2601-14.
Time after
transplant
0-1 Month 1-6 Months* >6 Months
Type of
infection
Nosocomial
MOSTLY BACTERIA
OR CANDIDA
Cather-related UTI
Line infection
Pneumonia
Procedure-
related
MOSTLY BACTERIA
OR CANDIDA
Wound infection
Urine leak
Donor-derived
RARE
Opportunistic
VIRAL
CMV
BK
VZV
BACTERIAL
Nocardia
Listeria
FUNGAL
PCP
Aspergillus
PARASITIC
Strongyloides
Toxoplasma
Community-
acquired
VIRAL
Influenza
HPV
PTLD (EBV)
BACTERIAL
S. pneumoniae
Listeria
Rhodococcus
FUNGAL
Cryptococcus
Endemic fungi
* Patients treated for rejection or graft-versus-host disease
after 6 months are also at risk for these infections
Case Presentation #5 (cont)
Fevers to 102F and pleuritic chest pain
persist over the next 3 days despite the
addition of levofloxacin
Blood cultures and sputum culture
remain negative
Serum cryptococcal antigen,
galactomannan, and beta-d-glucan are
negative
Copyright Harvard Medical School, 2010. All Rights Reserved.
2109
The most appropriate next step is:
A. Add empiric foscarnet
B. Change antibiotics to linezolid
C. Repeat sputum culture
D. CT guided fine needle aspirate of
left lower lobe mass/ infiltrate
E. Consider empiric trial of rituximab
Case Presentation #5 (cont)
CT-guided needle aspirate of LLL revealed
beaded branching Gram-positive rods on Gram
stain
Culture of aspirate grew Nocardia sp.
MRI of the brain showed no foci of infection
Antibiotics switched to high-dose
trimethoprim-sulfamethoxazole with marked
clinical improvement
Copyright Harvard Medical School, 2010. All Rights Reserved.
2110
Case Presentation #6
A 35 year old CMV negative woman with a
history of lupus nephritis receives a renal
transplant from a CMV positive donor
Prophylaxis with valganciclovir and
trimethoprim-sulfamethoxazole are started
after transplant
2 weeks post-transplant she presents with a
fever to 102F without other localizing
symptoms
Case Presentation #6 (cont)
Which is the least likely source of her
fever?
A. CMV
B. Wound infection
C. UTI
D. Urine leak
E. Pneumonia
Copyright Harvard Medical School, 2010. All Rights Reserved.
2111
Disclosures
Research funding via NIH-SBIR grant
ith Id h T h l I with Idaho Technology, Inc.
I will discuss the use of medications for
non-FDA approved indications
Copyright Harvard Medical School, 2010. All Rights Reserved.
2112
Take Home Messages -
Transplantation
Anil Chandraker MD FRCP FASN
Disclosures
None
Copyright Harvard Medical School, 2010. All Rights Reserved.
2113
Donor Cell
Host APC
Host T
cell
Shed
antigen
Transplantation Immunobiology
Indirect/Direct Allorecognition
MHC (HLA) Antigens
Cell Surface
Two Types
Class I
Class II
Several Loci
A,B,C
DP,DQ,DR
Many Alleles
Polymorphism
Co-dominant Expression
Copyright Harvard Medical School, 2010. All Rights Reserved.
2114
T Cell Selection in the Thymus
The repertoire of mature T cells is selected by two processes:
Positive Selection: only T cells with receptors that recognize
something that looks like a self MHC molecule
Negative Selection: no T cells with receptors that recognize self
MHC molecules too well (protects against autoimmunity)
T cel l
receptor
T cel l
receptor YES
NO
S S
MHC APC
T Cell
TCR CD4
T Cells Require 2 Signals
S1
Copyright Harvard Medical School, 2010. All Rights Reserved.
2115
MHC APC
T Cell
TCR CD4
T Cells Require 2 Signals
S1
Costimulatory Signal
S2
T cell
Antigen Presenting Cell
CD28
superfamil y
TNF-TNF-R
superfamil y
PD-L1 L2
PD-1
CD28 CTLA4
ICO
S
TCR
-
-
+
CD80 CD86
B7RP1
MHC-
peptide
4-1BBL
CD27(CD70)
CD134
L
CD30L
CD70
(CD27
)
4-1BB
CD30
CD13
4
+
+
+
+
+
+
?
?
CD40
CD154
+
*
*
*
*
*
*
*
*
Copyright Harvard Medical School, 2010. All Rights Reserved.
2116
ALLOGRAFT REJECTION
Indirect
Direct
Cytokine Production
Clonal Expansion
CD4
+
T cell
Allo APC
MHC
Class II
TCR
CD4
+
T cell
Self APC
MHC
Class II
TCR
Activated
Macrophage
DTH PEPTIDE DERIVED
FROM ALLOANTIGENS
Activated
B cell
Alloantibodies
Perforin
Granzyme
CD8
+
T cell
Graft Cell
FAS
FAS L
LMC
ACUTE CHRONIC
High density of
donor-derived
APCs
High precursor
frequency of T
cells with direct
specificity
Low precursor
frequency of
nave donor-
reactive T cells
with indirect
specificity
Direct pathway
hyporesponsiveness
Lack of donor
APCs
Low, persistent
precursor frequency
of indirectly-primed
T cells
Indirect allo-
reactivity with
epitope spreading
Indirect
Direct
ACUTE VESUS CHRONIC REJECTION
Relative Contributions of Direct and Indirect Pathways
Womer et al., Seminars Nephrology 2000
Copyright Harvard Medical School, 2010. All Rights Reserved.
2117
APCs T Cell
B cell
Th2
Th1
IL-2
IL-4
IL-4, IL5, IL-6
IL-10, IL-13
IFN-g
IgG1
IgG2
IgG1
IgG4
IgA
IgE
B7/ CD28
B7/ CD28
MHC/pepti de
TCR
MHC/pepti de
TCR
Antibodies
Humoral Responses
A1 B1
CHROMOSOME 6
5' 3'
B1 A1 B3 B4 A B1
DP DQ DR B C A
MHC CLASS I MHC CLASS II
A1
B1
HLA Complex
Copyright Harvard Medical School, 2010. All Rights Reserved.
2118
Introduction of
immunosuppressants (US)
Adapted from Zand MS. Semin Dial. 2005;18:511-519.
80 85 70 75 80 85 90 95 00 05
Year
0
20
40
60
80
100
P
e
r
c
e
n
t
Radiation
Prednisone
6-MP
Azathioprine
ATGAM
Cyclosporine
OKT3
Cyclosporine Microemulsion
Tacrolimus
Mycophenolate mofetil
Daclizumab
Basiliximab
Thymoglobulin
Sirolimus
Rituxumab
Alemtuzumab
Leflunomide
Rejection <12 months
1-Year Survival
IL-2R
IL-2
Other
T-cells
APC
Ca
++
Activated
calcineurin
Activated
NFAT
CsA /
tacrolimus
x
IL-2R
blockers
x
Alloantige
n
CD3 x
Glucocorticoids
Sirolimus
X
Autocrine
Paracrine
Induction of genes involved in
Cytokine production
T-lymphocyte replication
MMF /
azathioprine
X
DNA replication
Cell division
Purines
X
T-cell cytosol
T-cell nucleus
TCR
SIGNAL 1
SIGNAL 3
SIGNAL 2
Enhanced T cell
activation
OKT3
B7
CD28
IL-2
Thymoglobulin
Multiple surface
receptors
x
x
Copyright Harvard Medical School, 2010. All Rights Reserved.
2119
Induction
OKT3
Thymoglobulin
ATG
Basiliximab
Daclizimab
Campath-1H
Maintenance
Prednisone
Cyclosporine
Tacrolimus
Azathioprine
Mycopheolate
Sirolimus
Copyright Harvard Medical School, 2010. All Rights Reserved.
2120
Special Teams
Rituximab
IVIg
Plasmapheresis
Leflunamide
Antibody induction in kidney
transplantation
0
5
10
15
20
25
30
35
40
45
1996 1997 1998 1999 2000 2001 2002 2003 2004 2005
ATG Thymo OKT3 Zenapax Simulect Campath
Andreoni et al, Am J Transplant 7: 1359, 2007
Copyright Harvard Medical School, 2010. All Rights Reserved.
2121
The Balancing Act..
Prevention of graft rejection
Infection
Malignancy
Cardiovascular Disease
Nephrotoxicity
Vaccinations
Vaccine Pre Tx Post Tx
DT Yes (3) q 10 years
Hep B Yes (3) if titer <10 IU/ml
Pneumococcal Yes (2) ? q5-10years
Hep A At risk Pts At risk Pts
Influenza Yes q yr Yes q yr
Meningococcus At risk Pts At risk Pts
Live Vaccines
MMR Yes (2) NO
Varicella Yes (2) NO
Copyright Harvard Medical School, 2010. All Rights Reserved.
2122
Common Infectious Agents after
Immunosuppression
Bacteria
Gram negative
Gram positive
Anaerobes
Fungal
Candida
Aspergillus
Cryptococcus
Coccidiomycosis
Histoplasmosis
Blastomycosis
Protozoal
PCP
Toxoplasmosis
Viral
Herpes
Hepatitis A, B, C
Polyoma
Papilloma
Respiratory
Influenza
Adenovirus
RSV
West Nile virus
Tick-Borne
RMSF
Lyme disease
Ehrlichia
Parasitic
Strongoloides
BK Virus Nephropathy
Presents as asymptomatic gradual increase in Cr
Complicates 1-10% of kidney transplants
Hirsch et al. NEJM 2002 prevalence rates:
BK viruria 30%
BK viremia 13%
BK nephropathy 8%
Allograft failure occurs in 50-90%
Graft loss occurs in 50% cases over 6-60 months
Improved graft survival reported with screening
programs
Copyright Harvard Medical School, 2010. All Rights Reserved.
2123
BK Algorithm
MON ITOR BK IN SER UM WITH PC R M ONTHL Y FOR 3 MONTHS AND UP ON RENAL
DY SFUN CTION



BLOOD /Uri ne BK
+





Increased se rum creatinine Normal serum creatinine



Biops y Dec rease Immunosuppress ion
Monitor q 2 weeks until clear




Reject ion +BKN BK N al one N o BK N Increas ed creatinine Normal creatin




1) Rx reject ion with IVIG Dec reased immunos uppress ion Re peat biops y Follow-up
2) Decrease immunosuppress ion
3) ?Quino lone or Ci dofovir
4) ? Lefl unomide
Cytomegalovirus
Reactivated or Seroconversion
Multiple presentations
Fever
Leukopenia
Hepatitis
Pneumonitis,
Pancreatitis
Colitis - diarrhea
Meningoencephalitis
Myocarditis
Retinitis
Copyright Harvard Medical School, 2010. All Rights Reserved.
2124
Asymptomatic CMV in the First 100 days Increases
Mortality Later
Sagedal, Kidney Intl 2004, 329-337
Drug Interactions
Decrease CI
levels
Increase CI
levels
Syngergistic
Toxicity
Rifampin
Isoniazid
Nafcillin
Macrolides
Azoles
Tmp-Smx
Gentamicin
Amphoteracin
Vancomycin
Copyright Harvard Medical School, 2010. All Rights Reserved.
2125
Purpose of the Evaluation - Summary
Assess patients medical, surgical and
psychological suitability for transplant
Understand the patients reasons for
wanting a transplant
Discuss risks vs benefits of transplant
Discuss donor options
Plan the immunosuppressive protocol
ahead of time
The Big Questions
Will this pt benefit from transplant
(survival, quality of life)? Wolfe et al.
NEJ M 1999;341: 1725
Is it appropriate to ask a living donor to
donate?
Is it appropriate to take a deceased
donor kidney from the list?
(Responsibilities to individual pt and to
the list)
Copyright Harvard Medical School, 2010. All Rights Reserved.
2126
Defining an Expanded Criteria Donor
Donor condition
Donor age
50-59 yrs
Donor age
>59 yrs
CVA +hypertension +creatinine >1.5mg/dL (133umol/L) X X
CVA +hypertension X X
CVA +creatinine >1.5mg/dL (133umol/L) X X
Hypertension +creatinine >1.5mg/dL (133umol/L) X X
CVA X
Hypertension X
Creatinine >1.5mg/dL (133umol/L) X
None of above X
HLA Typing
Donor and recipient
A
B
C
DR, DQ
Now less important in allocation BUT
still
A marker of immunologic risk
Of particular importance
short and long term survival
Copyright Harvard Medical School, 2010. All Rights Reserved.
2127
Terasaki et al NEJ M 1995; 333:333
PRA: Panel Reactive Ab
A measure of pre-formed Ab in potential
recipients
HLA and non-HLA antigens
Against a PANEL of antigens from MULTIPLE
donor cells from the population
As opposed to a SPECIFICdonor (a crossmatch)
Estimate of likelihood that they will have antibodies to a donor
drawn from that population
Gauges the immune activity of a patient
Copyright Harvard Medical School, 2010. All Rights Reserved.
2128
The UNOS Wait-List 31 Jan 05
Current PRA%
0-19%
20-79%
80-100%
Unknown
#
44,889
7,734
5,754
6,460
Kaplan-Meier
Median Wait-Time
(1998)
3.1 [3.0 3.2]
5.3 [4.6 6.8]
6.4 [4.8 - ****]
Source: OPTN May 2006
New kind of PRA
Solid phase assays utilizing flow cytometry
Microparticles are coated with PURIFIED HLA ANTIGEN
FlowPRA
TM
Luminex
TM
The Beads
Copyright Harvard Medical School, 2010. All Rights Reserved.
2129
What do we do with PRA?
Immunologically high risk >10%, >80%
May need different immunosuppressionor monitoring
Longer times to transplant what is their transplantibility
Worse graft survival
Acute rejection
Identify HLA Identical Donors
Identify Donors with ACCEPTABLE HLA Types
Careful crossmatching
Higher priority in organ allocation
They do not have as many organs available to them
Pre-transplant
The Crossmatch
Tests the SERUM of potential recipients for PRE-FORMED
AB against a SINGLE potenti al donor when an
organ becomes available from that donor
(compare to PRA where we use a PANEL of donors looking for many
different antibodies)
Antibodies to everyone else do not matter at this particular moment
Transplant
Copyright Harvard Medical School, 2010. All Rights Reserved.
2130
CDC Crossmatch
T cell Class I HLA Antigens (and non-HLA)
B cell Class I and Class II HLA Antigens (and non-
HLA)
A positive T cell CDC crossmatch is an
ABSOLUTE contraindication to
transplantation hyperacute rejection
Transplant
CDC Crossmatch
A positive B cell crossmatch (in isolation) may be a
relative contraindication to transplantation (practise
specific)
Repeat transplant (especially with repeat DR antigen)
DR specificity or high titer is known
Represents a pos Class I reaction, but detected on B
cells because of higher density of expression
Negative auto-XM
Copyright Harvard Medical School, 2010. All Rights Reserved.
2131
Flow Cytometry vs. CDC
More sensitive picks up non-complement binding
and low titer Ab
Specific
For IgG
specific for HLA Ab in PRA testing (improves the clinical
significance)
FCXM is not an absolute contraindication to
transplant (unlike CDC)
PRA vs. Crossmatch
PRA Crossmatch
Antigen source Multiple (panel from
population)
A single potential
donor
CDC Yes Yes
Flow cytometry Yes (beads or cells) Yes (cells)
Type of test Screening Confirmatory
Interpretation Transplantability
estimate, immune
activity, a higher
risk patient
If positive for Class I
Ab, DO NOT
TRANSPLANT
(CDC)
Timing Monthly J ust prior to
transplant
Copyright Harvard Medical School, 2010. All Rights Reserved.
2132
Desensitization
removing the contraindication from a
positive XM
Done in living donor transplants when pre-formed Ab are
detected (CDC or FCXM)
IVIG
Plasmapheresis
Immunosuppression eg rituximab
Serial serum monitoring for Ab levels transplant when
a sensitive method shows absence of donor specific Ab
CHRONIC ALLOGRAFT
DYSFUNCTION
Alloantigen-dependent Factors
Acute Rejection
HLA match
Degree of immunosuppression
Alloantibodies
Copyright Harvard Medical School, 2010. All Rights Reserved.
2133
CHRONIC ALLOGRAFT
DYSFUNCTION
Alloantigen-independent Factors
Ischemia/reperfusion injury (DGF)
Brain death, marginal donors
CMV
HTN
Hyperlipidemia
Nephron mass
Senescence
Calcineurin Inhibitor Toxicity
Nanki vel l, B. J. et al . N Engl J Med 2003;349:2326-2333
Point Prevalence of
Histologically Defined
Subclinical Rejection in
Sequential Biopsy
Specimens (Panel A) and
Calcineurin-Induced
Nephrotoxic Effects
(Panel B)
Copyright Harvard Medical School, 2010. All Rights Reserved.
2134
Recurrent diseases
IgA
FSGS
HUS
Membranous
MPGN
Recurrent diseases
Third commonest cause of late graft loss
Risk of graft loss at 1 year is 0.6%
Risk of graft loss at 10 years is 8.4%
Newer immunosuppressive drugs may
reduce rate of recurrence
Commonest cause of new GN post
transplant s membranous GNs
Copyright Harvard Medical School, 2010. All Rights Reserved.
2135
FSGS
Overall rate of recurrence 30-40% Overall rate of recurrence 30 40%
Recurrent disease original FSGS variant
Risk factors for recurrence
Pediatric population
Rapid progression to ESRD
Previous recurrence in a transplant (85-100%)
Mesangial hypercellularity in native biopsy
? Live donors
? Native nephrectomy
Disclosures
None None
Copyright Harvard Medical School, 2010. All Rights Reserved.
2136
TRANSPLANT CASES
Martha Pavlakis, MD
Medical Director, Kidney and Pancreas Transplantation
Disclosures
None None
Copyright Harvard Medical School, 2010. All Rights Reserved.
2137
Which is true about live donation
a) Risk of peri-operative death is 3/10 000 a) Risk of peri-operative death is 3/10,000
b) Risk of ESRD over 40 years is 1.2 X that of
the general population
c) Risk of pneumothorax is higher with
laproscopic nephrectomy versus open
d) Ri k f l i i d d t d) Risk of pre-eclampsia is reduced post
donation
e) Donors can be evaluated by the recipients
nephrologist
Which is true about live donation
a) Risk of peri-operative death is 3/10 000 a) Risk of peri-operative death is 3/10,000
b) Risk of ESRD over 40 years is 1.2 X that of
the general population
c) Risk of pneumothorax is higher with
laproscopic nephrectomy versus open
d) Ri k f l i i d d t d) Risk of pre-eclampsia is reduced post
donation
e) Donors can be evaluated by the recipients
nephrologist
Copyright Harvard Medical School, 2010. All Rights Reserved.
2138
Gout after kidney transplant;
which is true?
a) Is rarely associated with the use of
cyclosporine cyclosporine
b) Is usually treated with NSAIDs
c) If allopurinol is prescribed to a patient on
MMF, the dose of allopurinol should be
reduced
d) Istreatedwithprobenecidif thepatient is d) Is treated with probenecid if the patient is
allergic to allopurinol and GFR is <20ml/min
e) Acute gout is often treated with PO
glucocorticoids
Gout after kidney transplant;
which is true?
a) Is rarely associated with the use of
cyclosporine cyclosporine
b) Is usually treated with NSAIDs
c) If allopurinol is prescribed to a patient on
MMF, the dose of allopurinol should be
reduced
d) Istreatedwithprobenecidif thepatient is d) Is treated with probenecid if the patient is
allergic to allopurinol and GFR is <20ml/min
e) Acute gout is often treated with PO
glucocorticoids
Copyright Harvard Medical School, 2010. All Rights Reserved.
2139
Case-Rise in creat, allograft
pain
A 66 year old male is admitted 3 wks after uncomplicated
d dd kid l i i hi d deceased donor kidney transplantation with increased Cr. He
feels quite well but notices some reduction in urine output
and mild pain over the allograft.
Meds: MMF 1g bid, prednisone 15 mg qd, tacrolimus 3 mg
bid, valgancyclovir 450mg qd, ranitidine 300mg qd, SMX-
TMP400mgqd TMP 400mg qd.
Two days prior to admission, Cr was 1.5 and tacrolimus
trough was 9.4; todays Cr is 2.3.
The pre-transplant T-cell crossmatch was
negative and the allograft functioned
immediately He received thymoglobulin
Case contd
immediately. He received thymoglobulin
induction.
PMH: ESRD secondary to IgA nephritis, 1
previous transplant lost after 6 months from
severe rejection; BPH, HTN.
Exam: P 80, BP 150/90, mild tenderness over
the allograft site; otherwise unremarkable.
Copyright Harvard Medical School, 2010. All Rights Reserved.
2140
Initial management of should include
all of the following EXCEPT:
a) Tacrolimus trough concentration a) Tacrolimus trough concentration
b) Crossmatch against donor T and B cells
c) Ultrasound to r/o allograft hydronephrosis
d) E i i IL2 t bl k b ili i b d) Empiric IL2 receptor blocker eg basiliximab
e) Ultrasound to exclude urine leak
Initial management of should include
all of the following EXCEPT:
a) Tacrolimus trough concentration
b) Crossmatch against donor T and B cells
c) Ultrasound to r/o allograft hydronephrosis
d) Empiric IL2 receptor blocker eg basiliximab
e) Ultrasound to exclude urine leak
Copyright Harvard Medical School, 2010. All Rights Reserved.
2141
Initialstests: tacrolimustrough=65 WBC 45
Case contd
Initials tests: tacrolimus trough 6.5, WBC 4.5,
Hct 30, platelets 160,000
Urgent ultrasound shows no hydronephrosis and no
perinephric collection
Crossmatch studies are pending
Which is the next most appropriate
step in management ?
a) CMV viral load
b) Increase tacrolimus dose
c) Reduce tacrolimus dose
d) Empiric OKT3
e) Allograft biopsy
Copyright Harvard Medical School, 2010. All Rights Reserved.
2142
Which is the next most appropriate
step in management ?
a) CMV viral load
b) Increase tacrolimus dose
c) Reduce tacrolimus dose
d) Empiric OKT3
e) Allograft biopsy
An uncomplicated biopsy is performed.
Light microscopyshows no tubulitis but
Case contd
Light microscopy shows no tubulitis but
does show neutrophil capillaritis.
Staining for C4d is positive in the
perititubular capillaries.
Flow cytometry crossmatch is highly y y g y
positive against donor T and B cells i.e.
antibodies are present against class I
and II HLA antigens of the donor
Copyright Harvard Medical School, 2010. All Rights Reserved.
2143
Which one of the following is true ?
a) The most likely diagnosis is acute antibody
mediated rejection j
b) The most likely diagnosis is acute polyoma
virus infection
c) Plasmapheresis may be useful but is
contraindicated because of bleeding risk
d) Allograft nephrectomy is indicated
e) Cyclosporine should be added to treatment
regimen
Which one of the following is true ?
a) The most likely diagnosis is acute
antibody mediated rejection
b) The most likely diagnosis is acute polyoma
virus infection
c) Plasmapheresis may be useful in removing
anti-donor antibodies but is contraindicated
because of bleeding risk g
d) Allograft nephrectomy is indicated
e) Cyclosporine should be added to treatment
regimen
Copyright Harvard Medical School, 2010. All Rights Reserved.
2144
Antibody mediated rejection
(AMR)
Increasing frequency due to better methods Increasing frequency due to better methods
of diagnosis, more sensitized patients
Treatment: Plasma Exchange, IvIg, Tacro,
MMF
Typical Features
Allograft dysfunction Allograft dysfunction
Anti-HLA antibodies specific to donor (DSA)
Light microscopy: neutrophils in PTC
Immunofloru: C4d staining in PTC
v
Severe antibody-mediated rejection
(AMR): hyperacute or acute
0 Preformed or rapidly
formed anti-donor
antibody*
Binding of Ab to
endothelium, recruitment of
complement and neutrophil
l h
v
polymorphs
Massive inflammation,
thrombosis and graft loss
*Directed against HLA
or ABO or other antigens
Copyright Harvard Medical School, 2010. All Rights Reserved.
2145
Neutrophil capillaritis H&E
(courtesy H Rennke)
0
C4d staining (immunoperoxidase)
(courtesy H Rennke)
Neg control
AcuteAMR
0
Neg control
Acute AMR
Copyright Harvard Medical School, 2010. All Rights Reserved.
2146
Acute Humoral Rejection Polys in peritubular capillaries
Glomerular Neutrophils and Macrophages
Magil
Sund
Copyright Harvard Medical School, 2010. All Rights Reserved.
2147
Fibrinoid Necrosis of Arteries
ATN Day 9 Oliguria
C4d
01E55740
Copyright Harvard Medical School, 2010. All Rights Reserved.
2148
0
Presence of
anti HLA Ab
pre transplant
associated
with higher
immunologic
Opelz G, Lancet 2005 ;365:1570-6
immunologic
risk.
Copyright Harvard Medical School, 2010. All Rights Reserved.
2149
44 yr. old with fever, malaise,
nausea for 5 days
ESRD (IgA), deceased donor kidney txp 8 months
ago, no complications
Donor CMV +, Recipient CMV -
Recent contact with PPD +friend, who is on INH. No
travel, no other symptoms
Meds: MMF 1 gr BID, tacro, bactrim. Valcyte g , , y
stopped 6 months post-transplant
Exam Temp 102, BP 130/82, p90 O2sat 98% RA
Creat 1.2, tacro 11, WBC 6.0, Hg 12, AST 58, ALT 77
Exam unremarkable; Chest Xray with
diffuse subtle increase markings
Copyright Harvard Medical School, 2010. All Rights Reserved.
2150
All are appropriate initial
management EXCEPT ?
a) Induce sputum for PJ P, TB
b) Blood cultures
c) CMV viral load (blood) by PCR
d) Plant PPD
e) Stress dose Steroids
All are appropriate initial
management EXCEPT ?
a) Induce sputum for PJ P, TB
b) Blood cultures
c) CMV viral load (blood) by PCR
d) Plant PPD
e) Stress dose Steroids
Copyright Harvard Medical School, 2010. All Rights Reserved.
2151
CMV disease (lung and liver)
Blood CMV PCR highly positive Blood CMV PCR highly positive
Induced sputum negative
Patient discharged on Valgancyclovir
900 mg BID; MMF reduced to 500 mg
BID
1 week later: afebrile, exam normal.
Creat 1.2, Tacro 7.2, WBC 1.2, Hg 10,
Platelets 188,000
Leukopenia likely due to?
a) Worsening CMV infection; add foscarnet
b) Resistant CMV infection; stop valganciclovir
and start foscarnet
c) Marrow-toxic effect of valganciclovir; reduce
valganciclovir dose
d) Marrow-toxic effect of valganciclovir; stop
valganciclovir and start foscarnet
e) CMV associate splenomegaly-get CT
Copyright Harvard Medical School, 2010. All Rights Reserved.
2152
Leukopenia likely due to?
a) Worsening CMV infection; add foscarnet
b) Resistant CMV infection; stop valganciclovir
and start foscarnet
c) Marrow-toxic effect of valganciclovir; reduce
valganciclovir dose
d) Marrow-toxic effect of valganciclovir; stop
valganciclovir and start foscarnet
e) CMV associate splenomegaly-get CT
CMV disease after transplantation-all
are true EXCEPT?
a) Rare in first month after transplant
b) Most common in CMV positive donor to
CMV negative recipient
c) Valgan is effective in preventing disease
d) Valgan causes frequent thrombocytopenia ) g q y p
e) CMV which is Valgan resistant is rare in
kidney transplantat patients
Copyright Harvard Medical School, 2010. All Rights Reserved.
2153
CMV disease after transplantation-all
are true EXCEPT?
a) Rare in first month after transplant a) Rare in first month after transplant
b) Most common in CMV positive donor to
CMV negative recipient
c) Valgan is effective in preventing disease
d) Valgan causes frequent thrombocytopenia
e) CMV which is Valgan resistant is rare in
kidney transplantat patients
Cytomegalovirus (CMV)
~70% of adult recipients and donors are IgG p g
seropositive at time of transplant or
immunosuppression induction
Major cause of morbidity and mortality prior to era of
routine prophylaxis.
CMV infection- recent seroconversion or presence of CMV infection recent seroconversion or presence of
viremia in absence of overt symptoms.
CMV disease- symptomatic infection.
Copyright Harvard Medical School, 2010. All Rights Reserved.
2154
Cytomegalovirus (CMV)
Presenting symptoms: Fever leucopenia Presenting symptoms: Fever, leucopenia,
malaise, thrombocytopenia, hepatitis,
pneumonitis, pancreatitis, colitis,
meningoencephalitis, and rarely myocarditis
Retinitis rare in renal transplant recipients
Ri k t tifi ti i ti l Risk stratification is essential
Onset usually after cessation of prophylactic
Valcyte, but resistant strains do occur
CMV Disease Treatment
Treatment dose Valgancyclovir or ganciclovir for 12 Treatment dose Valgancyclovir or ganciclovir for 12
weeks (or until viremia has cleared). DOSE
ADJ UST FOR RENAL FUNCTION
GCV IV 5 mg/kg q12 5-7 days, then q24 hours
Valgan p.o 900 mg BID
Consider decrease IS at diagnosis
M i t d V l f 8 12 k Maintenance dose Valgan for 8-12 weeks
Resistance is rare - UL97 mutation
Cytogam reserved for CMV pneumonitis
Copyright Harvard Medical School, 2010. All Rights Reserved.
2155
Evaluation of the recipient: All are
true EXCEPT
a) EBV negative serology INCREASES risk of a) EBV negative serology INCREASES risk of
post-transplant lymphoproliferative disease
b) Breast ductal carcinoma in situe NOT a
contraindication to transplant
c) High anti-HLA antibodies (>50%) associated
with LONGER wait times on deceased donor
li t list
d) ABO-O blood group waits LONGER than AB
e) CMV serology is used to match CMV
negative recipients with negative donors
Evaluation of the recipient: All
are true EXCEPT
a) EBV negative serology INCREASES risk of a) EBV negative serology INCREASES risk of
post-transplant lymphoproliferative disease
b) Breast ductal carcinoma in situe NOT a
contraindication to transplant
c) High anti-HLA antibodies (>50%) associated
with LONGER wait times on deceased donor
li t list
d) ABO-O blood group waits LONGER than AB
e) CMV serology is used to match CMV
negative recipients with negative donors
Copyright Harvard Medical School, 2010. All Rights Reserved.
2156
Sirolimus: all correct EXCEPT?
a) Binds to the same target protein as cyclosporine but
has different downstream effects
b) Metabolism is impaired by diltiazem and
ketoconazole
c) Adverse effects include hyperlipidemia, mouth sores
and impaired wound healing
d) The combination of standard dose cyclosporine and d) The combination of standard-dose cyclosporine and
sirolimus is a/w increased nephrotoxicity
e) Once daily dosing is sufficient in adults
Sirolimus: all correct EXCEPT?
a) Binds to the same target protein as cyclosporine but a) Binds to the same target protein as cyclosporine but
has different downstream effects
b) Metabolism is impaired by diltiazem and
ketoconazole
c) Adverse effects include hyperlipidemia, mouth sores
and impaired wound healing
d) The combination of standard-dose cyclosporine and
sirolimus is a/w increased nephrotoxicity
e) Once daily dosing is sufficient in adults
Copyright Harvard Medical School, 2010. All Rights Reserved.
2157
Sirolimus
Sirolimus (Rapamune) also binds to FKBP but the Sirolimus (Rapamune) also binds to FKBP but the
complex inhibits lymphocyte proliferation to growth
factors and cytokines such as IL-2 (different
effects to those of tacrolimus). Adverse effects
include wound healing, lymphoceles, mouth sores,
acne, hyperlipiemia, thrombocytopenia.
Sirolimus is also metabolized by the Cyt P450 Sirolimus is also metabolized by the Cyt P450
system and affected by drugs such as rifampin
and diltiazem. It is not directly nephrotoxic but
there is evidence that it delays recovery in the
setting of delayed graft function and that it
enhances the toxicity of CNIs.
45 yr old - fever, allograft pain
ESRDDM2 2 weeks after first deceased ESRD DM2, 2 weeks after first deceased
donor transplant. Fever 100 degrees, vague
pain over allograft
Meds: Tacro, MMF 1 gm BID, bactrim, Valgan
450 mg daily
Exam Temp 100, p100, BP 140/80, J P lateral
t d 80 l fl id 80 l/d to wound 80 ml serosang fluid 80 ml/day.
Mildly tender allograft, wound healing well
Creat 1.7 (up from 1.2), WBC 15 (up from 7)
UA 1+blood, 1+protein, 15-20 WBC
Copyright Harvard Medical School, 2010. All Rights Reserved.
2158
All are appropriate steps in
management EXCEPT
a) Order urine culture
b) Order BK-virus by PCR in blood or urine
c) Order blood culture
) O f d) Order creatinine on J P drain fluid
e) Order ultrasound of allograft
All are appropriate steps in
management EXCEPT
a) Order urine culture
b) Order BK-virus by PCR in blood or urine
c) Order blood culture
) O f d) Order creatinine on J P drain fluid
e) Order ultrasound of allograft
Copyright Harvard Medical School, 2010. All Rights Reserved.
2159
Tests are ordered and he is started on Tests are ordered and he is started on
PO ciprofloxacin. The J P fluid creatinine
is 90mg/dL
US shows small collection medial to
allograft and ureteral stent in correct
position. No hydronephrosis
Which one is the BEST next
step in management?
) Additi f PO li lid f ibl a) Addition of PO linezolid for possible
enterococcal infection
b) Pulse IV methylprednisolone 500mg QD for 3
days
c) Flow cytometry crossmatch against donor
cells
d) Removal of the urinary stent
e) Insertion of bladder catheter
Copyright Harvard Medical School, 2010. All Rights Reserved.
2160
Which one is the BEST next
step in management?
) Additi f PO li lid f ibl a) Addition of PO linezolid for possible
enterococcal infection
b) Pulse IV methylprednisolone 500mg QD for 3
days
c) Flow cytometry crossmatch against donor
cells
d) Removal of the urinary stent
e) Insertion of bladder catheter
Urinary Leak
When you suspect a urinary leak always
insert a bladder catheter!!
Copyright Harvard Medical School, 2010. All Rights Reserved.
2161
a) FastingBloodglucose126 150
All are contraindications to living
kidney donation EXCEPT
a) Fasting Blood glucose 126-150
b) History of kidney stones in preceding 2 years
c) Age <18 years
d) CTA i h f bil l fib l d l i d) CTA with of bilateral fibromuscular dysplasia
e) Family history of type 1 diabetes mellitus
a) FastingBloodglucose126-150
All are contraindications to living
kidney donation EXCEPT
a) Fasting Blood glucose 126 150
b) History of kidney stones in preceding 2 years
c) Age <18 years
d) CTA withof bilateral fibromuscular dysplasia d) CTA with of bilateral fibromuscular dysplasia
e) Family history of type 1 diabetes mellitus
Copyright Harvard Medical School, 2010. All Rights Reserved.
2162
Hepatitis C Infection:
a) In the dialysis patient, is typically treated with
interferon-alfa and ribavirin
b) I i kid i i i d i h b) In native kidneys, is sometimes associated with
membranous or membranoproliferative
glomerulonephritis
c) Ribavirin monotherapy in the renal transplant
patient can induce acute rejection
d) Before transplantation, HCV-RNA titers are useful
in assessing the degree of liver damage
e) Immunosuppression typically slows the rate of liver
damage
Hepatitis C Infection:
a) In the dialysis patient, is typically treated with
interferon-alfa and ribavirin
b) I i kid i i i d i h b) In native kidneys, is sometimes associated with
membranous or membranoproliferative
glomerulonephritis
c) Ribavirin monotherapy in the renal transplant
patient can induce acute rejection
d) Before transplantation, HCV-RNA titers are useful
in assessing the degree of liver damage
e) Immunosuppression typically slows the rate of liver
damage
Copyright Harvard Medical School, 2010. All Rights Reserved.
2163
Antibody Therapies: which is correct?
a) Basiliximab are IL-2 receptor blockers and are often
usedtotreat severeacutecellular rejection used to treat severe acute cellular rejection
b) Antibodies are contra-indicated in delayed graft
function
c) Basilizimabblocks the CD3 receptor
d) Include basiliximab, OKT3, rabbit thymoglobulin
e) Thymoglobulin is commonly associated with a
cytokine release syndrome
Antibody Therapies: which is correct?
a) Basiliximab are IL-2 receptor blockers and are
often used to treat severe acute cellular rejection often used to treat severe acute cellular rejection
b) Antibodies are contra-indicated in delayed graft
function
c) Basilizimab blocks the CD3 receptor
) O d) Include basiliximab, OKT3, rabbit thymoglobulin
e) Thymoglobulin is commonly associated with a
cytokine release syndrome
Copyright Harvard Medical School, 2010. All Rights Reserved.
2164
Antibody preparations include non-specific lymphocyte
depleting agents such as thymoglobulin and those targeting
activated T cells only (the anti-CD25 agents such as
basiliximab).
Antibody preparations are commonly used in the setting of
delayed graft function where it is believed that they very
effectively prevent cellular rejection and that they allow the
delayed introduction of calcineurin inhibitors.
IL-2R blockersarenot indicatedfor thetreatment of acute IL 2R blockers are not indicated for the treatment of acute
rejection.
OKT3 (most associated with a serious cytokine release
syndrome) is no longer available.
Chronic allograft nephropathy-
Which is false?
a) May be alloimmune in nature
b) May show findings of peritubular capillary
basement membrane multilayering
c) Is characterized by tubular atrophy and
interstitial fibrosis
d) Is usually treated with increasing calcineurin
inhibitor dose
e) Can be a late consequence of acute rejection
Copyright Harvard Medical School, 2010. All Rights Reserved.
2165
Chronic allograft nephropathy-
Which is false?
a) May be alloimmune in nature
b) May show findings of peritubular capillary
basement membrane multilayering
c) Is characterized by tubular atrophy and
interstitial fibrosis
d) Is usually treated with increasing calcineurin
inhibitor dose
e) Can be a late consequence of acute rejection
Which is NOT a contraindication to
live donation?
a) BP >140/90 with mild hypertensive retinopathy
b) Age <18 years
c) BMI >40 kg/m2
d) Horseshoe kidney
e) Six HLA Ag mismatches with the recipient
Copyright Harvard Medical School, 2010. All Rights Reserved.
2166
Which is NOT a contraindication to
live donation?
a) BP >140/90 with mild hypertensive retinopathy
b) Age <18 years
c) BMI >40 kg/m2
d) Horseshoe kidney
e) Six HLA Ag mismatches with the recipient
Early post transplant fever. Which
is unlikely to be the cause?
a) CMV
b) Wound infection
c) UTI
d) Commoncold d) Common cold
e) Pneumonia
Copyright Harvard Medical School, 2010. All Rights Reserved.
2167
Early post transplant fever. Which is
unlikely to be the cause?
a) CMV
b) Wound infection
c) UTI
d) Commoncold d) Common cold
e) Pneumonia
1998 Classic overview
Fishman & Rubin, NEJ M 1998
Copyright Harvard Medical School, 2010. All Rights Reserved.
2168
2009 update-timeline of infections
Donor
Activation of latent
infections;
Derived
Nosocomial
Technical
Relapse;
Opportunistic
infections
Community
Acquired
TXP
Recipient
Derived
Dynamic assessment of infection risk
<4 weeks
1-6 months
>6 months
Adapted from Fishman AJ T Infections in SOT Supplement 4 2009
Abx-resistant sp
-VRE MRSA
Candida
Assume Prophylaxis
for PJ P, CMV
Comm. Acquired
PNA
Aspiration BK HepC Adeno UTI
<4 weeks 1-6 months >6 months
Aspiration
Pneumonia
BK, Hep C, Adeno,
Flu
UTI
Line infection Cryptococcus M. Tb Aspergillus, Mucor
Wound infection Anastomos. leak Nocardia (TMP-S)
Anastomos leak Without prophylaxis Late Viral:
C. Diff colitis Pneumocystis CMV, Hep B, C
Donor derived
(UNCOMMON)
Rabies, LCMV,
HSV, WNV
Herpes (HSV, CMV
VZV, EBV) HBV,
Listeria, Nocardia,
Toxo, Strongyloides,
Leishmania, T. Cruzi
HSV eceph
SARS, WNV
J C (PML)
EBV (PTLD)
HPV (Skin cancer)
Copyright Harvard Medical School, 2010. All Rights Reserved.
2169
Prophylactic treatment
Valcyte dose adjusted for 900 mg daily Valcyte dose adjusted for 900 mg daily
Prevents CMV, HSV
High risk transplant: 6 months
Medium or low risk: 3 months
Nystatin four times daily swish and swallow OR
Clotrimazole troche TID
Prevents oro-pharyngeal thursh
D rationof therap 1 monthor longer Duration of therapy 1 month or longer
Bactrim DS BIW or SS daily
Prevents UTI, PJ P, Nocardia
Duration of therapy 6 months-life
Prophylactic treatment
Fungal prophylaxis can be with Nystatin OR Azole Fungal prophylaxis can be with Nystatin OR Azole
therapy (Clotrimazole, Fluconazole)
The Azoles will suppress CNI metabolism, raising
levels with initiation and dropping them with cessation
of therapy
Beware CNI toxicity or under immunosuppression
when stopping/starting azole therapy when stopping/starting azole therapy
Copyright Harvard Medical School, 2010. All Rights Reserved.
2170
52 yr old - rise in creatinine
ESRDDM2, 6 weeks after first live donor transplant. ESRD DM2, 6 weeks after first live donor transplant.
Asymptomatic, feels well, exam unremarkable
Meds: Tacro, MMF 1 gm BID, bactrim, Valgan 450
mg daily
Creat 2.7 (up from 1.2), WBC normal, Tacro level 6.7
UA 1+protein, 3-5 WBC, no RBC
US unremarkable RI 0.7-0.8
Biopsy performed
Copyright Harvard Medical School, 2010. All Rights Reserved.
2171
mononuclear cells undermining the endothelium
)
These findings in a small artery
are most consistent with:
a) Acute AMR
b) Mild acute ACR
c) Polyoma (BK-virus) nephritis
d) S ACR d) Severe ACR
e) Atheroembolism
Copyright Harvard Medical School, 2010. All Rights Reserved.
2172
)
These findings in a small artery
are most consistent with:
a) Acute AMR
b) Mild acute ACR
c) Polyoma (BK-virus) nephritis
d) S ACR d) Severe ACR
e) Atheroembolism
Biopsy: ACR (IIA); C4d neg
Case contd
Crossmatch against donor T and B cells:
neg
LDH, haptoglobin, smear for
schistocytes: neg y g
Treated with steroids then thymoglobulin
Baseline tacro dose increased
Copyright Harvard Medical School, 2010. All Rights Reserved.
2173
3 months later, presents with increase in Cr
Case contd
from 1.8 to 2.5
Meds: MMF, tacrolimus, pred
Kidney bx is performed
C4d stain neg
Copyright Harvard Medical School, 2010. All Rights Reserved.
2174
Staining with antibody to SV40
Copyright Harvard Medical School, 2010. All Rights Reserved.
2175
)
The biopsy findings are most
consistent with:
a) Acute AMR
b) ACR + acute AMR
c) BK-virus nephritis
d) CMV infection d) CMV infection
e) Atheroembolism
)
The biopsy findings are most
consistent with:
a) Acute AMR
b) ACR + acute AMR
c) BK-virus nephritis
d) CMV infection d) CMV infection
e) Atheroembolism
Copyright Harvard Medical School, 2010. All Rights Reserved.
2176
)
Which treatment indicated?
a) Valacyclovir
b) Addition of sirolimus
c) Pulse steroids
d) High dose IV cidofovir d) High dose IV cidofovir
e) Stop MMF
Copyright Harvard Medical School, 2010. All Rights Reserved.
2177
)
Which treatment indicated?
a) Valacyclovir
b) Addition of sirolimus
c) Pulse steroids
d) High dose IV cidofovir d) High dose IV cidofovir
e) Stop MMF
Which tests are not strictly necessary
prior to kidney transplant (living or
deceased)?
a) ABO typing of the donor
b) ABO typing of the recipient
c) A flow T cell crossmatch
d) HLA typing of the donor
e) Serumsample to the HLA Lab for anti HLA e) Serum sample to the HLA Lab for anti-HLA
antibody testing
Copyright Harvard Medical School, 2010. All Rights Reserved.
2178
Which tests are not strictly necessary
prior to kidney transplant (living or
deceased)?
a) ABO typing of the donor
b) ABO typing of the recipient
c) A flow T cell crossmatch
d) HLA typing of the donor
e) Serumsample to the HLA Lab for anti-HLA e) Serum sample to the HLA Lab for anti-HLA
antibody testing
Rejection Prevention-testing
Pre-transplant
ABO of donor and recipient
Anti-HLA antibody testing
HLA types of donor and recipient
T cell crossmatch alone if patient low risk
Standard or CDC crossmatch using sera and cells is
sufficient if recipeint has no anti-HLA antibodies and no
reason to be sensitized (prior transplant, transfusions or
pregnancies)
Flow crossmatching if patient at high risk
Copyright Harvard Medical School, 2010. All Rights Reserved.
2179
Pre transplant Hepatitis C Virus
(HCV) -which is true?
a) HCV antibody positive candidate can always be
assumed to be viremic
b) HCV antibody positive candidates are not suitable
for kidney transplantation
c) Candidates with HCV antibody should have a liver
biopsy to assess degree of fibrosis/cirrhosis
d) Candidates with HCV cannot receive kidneys from d) Candidates with HCV cannot receive kidneys from
HCV antibody positive donors
e) Very high viral load is a good marker for cirrhosis
Pre transplant Hepatitis C Virus
(HCV) -which is true?
a) HCV antibody positive candidate can always be a) HCV antibody positive candidate can always be
assumed to be viremic
b) HCV antibody positive candidates are not suitable
for kidney transplantation
c) Candidates with HCV antibody should have a liver
biopsy to assess degree of fibrosis/cirrhosis
d) Candidates with HCV cannot receive kidneys from d) Candidates with HCV cannot receive kidneys from
HCV antibody positive donors
e) Very high viral load is a good marker for cirrhosis
Copyright Harvard Medical School, 2010. All Rights Reserved.
2180
Infection Prevention-testing
Pre-transplant
Hepatitis serology panels Hepatitis serology panels
HIV testing, RPR, Review history of infections
CMV and EBV testing pre-transplant to risk stratify
Complete physical exam and history of travel,
antibiotic use etc
Chest Xray and PPD testing
U d t l t i ( l fl t ) Update relevant vaccines (yearly flu etc)
As indicated by residency: Strongyloides,
Toxoplasmosis, other parasites (flukes, malaria)
Vaccinations
Vaccinations-(responses often sub-optimal)
Inactivated vaccines are safe (AVOID LIVE) Inactivated vaccines are safe (AVOID LIVE)
Hep A, B, inactivated polio, DPT, menigitis and
pneumovax, yearly flu
Ideally household contacts should also be
vaccinated
Pre immunosuppression or pre transplant Pre-immunosuppression or pre-transplant
Treat HIV to suppress viral load, optimize CD4
count
Treat Hepatitis C if possible (variable response)
Begin to treat positive PPD
Copyright Harvard Medical School, 2010. All Rights Reserved.
2181
Which of the following infections are not
associated with post transplant
malignancy?
a) EBV and post transplant lymphoma
b) HHV 8 and Kaposis sarcoma
c) CMV and angiosarcoma
d) HPV and vaginal squamous carcinoma d) HPV and vaginal squamous carcinoma
e) Hep C and Hepatocellular carcinoma
Which of the following infections are not
associated with post transplant
malignancy?
a) EBV and post transplant lymphoma
b) HHV 8 and Kaposis sarcoma
c) CMV and angiosarcoma
d) HPV and vaginal squamous carcinoma d) HPV and vaginal squamous carcinoma
e) Hep C and Hepatocellular carcinoma
Copyright Harvard Medical School, 2010. All Rights Reserved.
2182
Anemia 2 years after transplant is
most likely due to:
a) Valganciclovir bone marrow
suppression
b) Anemia of CKD
c) Mycophenolate related bone marrow c) Mycophenolate related bone marrow
suppression
d) Mycophenolate related hemolysis
e) Tacrolimus related hemolysis
Anemia 2 years after transplant is
most likely due to:
a) Valganciclovir bone marrow
suppression
b) Anemia of CKD
c) Mycophenolate related bone marrow c) Mycophenolate related bone marrow
suppression
d) Mycophenolate related hemolysis
e) Tacrolimus related hemolysis
Copyright Harvard Medical School, 2010. All Rights Reserved.
2183
After one year of kidney allograft,
function, the most likely cause of
graft loss is:
a) Chronic Tacrolimus or CSA toxicity
b) Chronic antibody mediated rejection
c) Death with a functioning graft
d) Lymphoma d) Lymphoma
e) Sepsis
After one year of kidney allograft,
function, the most likely cause of
graft loss is:
a) Chronic Tacrolimus or CSA toxicity
b) Chronic antibody mediated rejection
c) Death with a functioning graft
d) Lymphoma d) Lymphoma
e) Sepsis
Copyright Harvard Medical School, 2010. All Rights Reserved.
2184
Death with a functioning graft
is most often due to
a) Common malignancy (breast, colon etc)
b) Cardiovascular disease
c) Sepsis
d) Rare malignancy (lymphoma, Kaposis etc)
e) Accidental death
Death with a functioning graft
is most often due to
a) Common malignancy (breast, colon etc)
b) Cardiovascular disease
c) Sepsis
d) Rare malignancy (lymphoma, Kaposis etc)
e) Accidental death
Copyright Harvard Medical School, 2010. All Rights Reserved.
2185
Disclosures
None None
Copyright Harvard Medical School, 2010. All Rights Reserved.
2186

S-ar putea să vă placă și